Sei sulla pagina 1di 540

Biology

IV Partnl
Molecular Cell Biology
Sections YI-X
Section VI
Structure and Function
in Cells and Viruses
Section VII

Metabolic Components
Section VIII

Metabolic Pathways
Section IX
Genetic Information
Section X

Expression of
Genetic Information

The

BERKELEY
-%

r.EVI-EWw

Specializing in MCAT Preparation

ERKELEY

I E

F.O. Box 40140, Berkeley, California 94704-0140


Phone:

(510)

843-8378

Internet:

MCATprep@berkeleyreview.com

(510)

THE-TEST

http://www.berkeleyreview.com

The Berkeley Review and The Berkeley Review logo are registered trademarks of The Berkeley Review.
This publication for The Berkeley Review was written, edited, and composed on a desktop publishing system

using Apple Macintosh computers and Microsoft Word. Pages were created on the Apple LaserWrite Pro. Line

art was created using numerous graphics programs designed tor use on Macintosh computers. The majority of the
text type and display type was set in Times Roman and Palatine
Cover Design by MacGraphics.

Copyright 2011, 2010,2009,2007, 2005,2003, 2001, 2000,1995,1994,1993,1992 by The Berkeley Review'. Allrights
reserved.

No part of this publication may be reproduced, stored in a retrieval system, or transmitted, in any form or by any
means, electronic, mechanical, photocopying, recording, or otherwise, without the prior written permission of the
copyright owner.

Biology
Section VI

A. Biological Molecules
1. Amino Acids and Proteins

2. Carbohydrates and Lipids

Structure and
Function in
Cells and Viruses

3. Nucleic Acids

B.

Eukaryotic Cells
1. Cellular Reproduction
a. Eukaryotic Chromosomes

b. The Cell Cycle, Mitosis, and Meiosis

2. Cellular and Molecular Organization


a. Biomembranes and Membrane Transport
b. Nucleus, Nucleolus, and Ribosomes

c. Endoplasmic Reticulum, Golgi Apparatus,


Lysosomes, Peroxisomes, and Mitochondria
d. Microtubules, Microfilaments,
and Intermediate Filaments

C.

Prokaryotic Cells
1. Cellular Reproduction
a. Prokaryotic Chromosomes

b. The Cell Cycle

2. Cellular and Molecular Organization


a. Biomembranes and Membrane Transport
b.

D.

Nucleoid and Ribosomes

Viruses
1. Architecture and Genomes

2. Infection and Genomic Expression


3. Assembly and Release of Progeny Virons

Practice Passages and Answers

T/te

BERKELEY
JJrwi^w
Specializing in MCAT Preparation

Structure and Function in Cells and Viruses


Top 10 Section Goals
Be familiar with the basic biological molecules of cells and viruses.

The four general classes ofbiological molecules are amino acids, nucleic acids, carbohydrates, and
lipids. Be able to recognize their basic differences, and know the functional significance ofeach class.
Know the twenty standard amino acids that are used to synthesize proteins.
Beable to associate amino acid names with their structures, especially the five amino adds (Asp,

Glu, Lys, Arg, andHis) thatcanhave charged polar side chains.

Understand the difference between mitosis and meiosis at the cellular level.

P Know the different stages ofthe cell cycle andhow they relate to one another. Be able to describe
the chromosome number aridploidylevel at any stageof mitosis or meiosis.

Be familiar with the different types of membrane transport processes.

Have a feel for the differences betweensimplediffusion, facilitated diffusion, active transport, and

bulk transport Know something about symports, antiports, uniports, andmembrane charge balance.

Be familiar with the structure of membrane-bound organelles in a eukaryotic cell.

Be awareof how they relate to oneanother within theircellular environment. Know their general
structuralcharacteristics (e.g., single membrane, double membrane, contains DNA, etc.).
Be familiar with the functions of eukaryotic organelles.

For each of the majororganelles in a eukaryotic cell, be ableto define its function in generalterms.
Is it involved in metabolism? Cellular packaging and sorting? Replication? Proteinsynthesis?
Know where the different metabolic processes occur in a cell.

> phosphorylation are common toeukaryotes andprokaryotes. Where do they occur in each cell?

The general metabolic reactions of glycolysis, the Krebs cycle, electrontransport, and oxidative

Know the general differences between Gram-negative and Gram-positive bacteria.


Bacterial cells, for the most part, can be divided into Gram-negative and Gram-positive cells. It is

important to havean understanding ofthestructural differences between the twocell types.

Be familiar with the reproductive cycle of a virus.

Thebiological diversity withinviruses is enormous. Justhave a general idea of the strategies used

byenveloped andnonenveloped viruses when they infect a host cell.

Know the general differences between eukaryotes, prokaryotes, and viruses.


It is important to knowthebasic differences between eukaryotic cells, prokaryotic cells, and viruses.
Havean understanding of the size differences. Howdo theyinteract with eachother?

Biology

Structure &? Function in Cells &Viruses

Biological Molecules

Biological Molecules
Amino Acids & Proteins
Amino Acids

Even though there are more than 400 naturally occurring amino acids, there are a

standard set of 20 amino acids that comprise the proteins of all living species. As
we will see, proteins are simply individual amino acid residues linked together
in a head-to-tail fashion. All of the standard 20 amino acids are referred to as o>

aminoacids (i.e., a 2-amino acid), except for proline which is referred to as an ocimino acid.

An amino acid is composed ofa basic amino group (-NH2), an acidic carboxyl
group (-COOH), a hydrogen (H) atom, and a side chain which is characteristicto

each amino acid. If no one particular amino acid is being discussed, the side

coo

H,N- C- H

chain is usually designated as -R. The amino group of an amino acid is covalently

IOC

attached to the a-carbon atom of the amino acid-hence the name oc-amino acid.

The a-carbon atom (Figure 6-1) is the carbon atom next to the carboxyl carbon
Figure 6-1

atom.

An oc-amino acid.

Each of the 20 standard amino acids has a particular three letter and one letter
abbreviation. For example, proline can be abbreviated either as Pro or as P. Each
amino acid also has a defined molecular weight (Mr) and can be placed,
according to its R group (at pH 7.0), into one of three main families of amino
acids. Thefirst family (Figure6-2) consists of those amino acids with nonpolar R
groups. These amino acids are hydrophobic.
Nonpolar R Groups

II

H3N C- C- O

II

H,N C C- O
I

ll

H3N C C- O

CH,

I ll
0
H2N C- C- O

11

H3N C C- O
i
I

H,NC-C- O

ll

CH,
l

11

H3N C- C- O
3
I
H3C CH
I

H3C CH

CH2

CH3

CH3

Leucine (Leu)[L]

Isoleucine (Ile)[I]

Mr=131

Mr=131

Alanine (Ala)[A]
Mr=89

Valine (Val)[V]
Mr=117

l
I

H3C CH
I

Glycine (Gly)[G]
Mr = 75

H3N C C- O

CH3

H3N C- C- O

CH,
I

CH2
Proline (Pro)[P]
Mr=115

S
I

CH3
Methionine (Met)[M]

Phenylalanine (Phe)[F]

Tryptophan (Trp)[W]

Mr= 149

Mr=165

M. = 204

Figure 6-2
Amino acids with nonpolar side chains.

Copyright by The Berkeley Review

The Berkeley Review


Specializing in MCAT Preparation

Biology

Structure &Function in Cells 6t Viruses

Biological Molecules

Thesecondfamily consists of those aminoacidswith polar uncharged R groups


(Figure 6-3). These amino acids are hydrophilic because they contain side chain
functional groups(e.g., -OH, -SH, or -NH2) that can hydrogenbond with water,
Polar Uncharged R Groups

11

H3N C C- O
3
I
CH2

H3N C C3

11

H3N C C- O

CH,

HO-CH

OH

CH3

Serine (Ser)[S]

Threonine (Thr)[TJ

Cysteine (Cys)[C]

Mr=105

Mr=119

M,= 121

11

H
I

SH

ll

H3N C- C- O

H,N C- C- O

ll

H3N- C C~ O
J

CH2

CH2
I

0= C- NH2

CH2
l

0= C- NH2
Asparagine (Asn)[N]
Mr=132

Glutamine (Gln)[Q]
Mr=146

Tyrosine (Tyr)[Y]
M,= 18I

Figure 6-3
Amino acids with uncharged polar side chains.

The third family (Figure 6-4) consists of those amino acids with charged R
groups. Note that two of the amino acids (Asp and Glu) have side chains which
are carboxylic acids and three of the amino acids (Lys, Arg, and His) have side
chains which are amine bases. The side chains of Asp, Glu, Lys, and Arg are
highly ionized at neutral pH. The side chain of His is weakly ionized at neutral
pH. We will see that the degree of ionization of an amino acid depends on its
acidic and basic properties.
Charged R Groups

II

ll

1 2
0=C0

Aspartate (Asp)[D]
Mr=133

0
ll

H3N C C--O
3
1
CH2

H3N C C- 0

H3N C C- 0

H
l

CH2

1 2
0=C0

Glutamate (Glu)[E]
Mr=147

H3N-

11

CH2
1

CH2

CH2

c c - 0

CH2

CH2

CH2

NH

H
I

0
ll

H3N C C- 0

CH2

'
NH3

H2N- C= NH2

Lysine (Lys)[K]

Mr=146

Arginine (Arg)[R]
M r=174

r^
H-N^ .N-H
N^
Histidine (His)[H]

Mr=155

Figure 6-4
Amino acids with charged side chains.

Copyright by The Berkeley Review

The Berkeley Review


Specializing in MCAT Preparation

BlOlOgy

Structure & Function in Cells &Viruses

Biological Molecules

Although the standard 20 amino acids can exist in either the D or the L

configuration (with the exception of glycine because itis non-chiral), only the Lamino acids are found in proteins. It is not known why evolution chose to incor

porate L-amino acids into proteins instead of D-amino acids. As an example, the
two non-superimposable mirror images of the a-amino acid alanine are shown in

coo

coo

H,N-C-*
H
'.I1

Mi

C-^ NH

Figure 6-5. Note that the a-carbon of alanine is chiral (there are 4 different
substituents attached to that a-carbon). The D isomer and the L isomer of alanine
are called enantiomers. The D and L isomers of the different amino acids are

based on the absolute configuration ofD and L-glyceraldehyde. [All amino acids
mentioned in these readings will be in the L-configuration, unless otherwise
stated.]

CH,

CH,
Mirror

L-alanine

D-alanine

Figure 6-5
Non-supcrimposable mirror images.

Even though D-amino acids are never found in proteins, they do exist in many
organisms. For example, D-alanine and D-glutamate are found within the rigid
cell walls ofsome bacteria. The presence ofthese D-amino acids helps to prevent
the degradation of the bacterial cell wall by specific enzymes called proteases
(more on this later). D-Valine is found in the antibiotic valinomycin, a carrier of
K+ ions across membranes.

Amino acids found within proteins can be modified as shown by the examples
given in Figure 6-6. Two important amino acid modifications can be found in

collagen, the most abundant protein in mammals. Some ofthe proline and lysine
amino acid residues found within nascent molecules of collagen are
hydroxylated to give 4-hydroxyproline and 5-hydroxylysine, respectively. These
modified amino acidshelp to give collagen its high tensile strength. Histamine, a
powerful vasodilator, is formed from the decarboxylation of the amino acid
histidine. S-Adenosylmethionine is an activated form of the amino acid
methionine and can methylate protein or nucleic acid substrates.

H
i

II

I
II

H^N C-C-0
I

H3N C C- O

CH,

NH,

CH,

!i.--:-.-i!

'

CH,

S-

CH,
0
C-0

CH,

H-C OH

H3N CH2

H-Nv

. N
HO

4-Hydroxyproline

5-Hydroxylysinc

Histamine

OH

S-Adenosylmethionine

Figure 6-6
Modified amino acids.

Acid-Base Properties

Body fluids in biological systems generally have a pH range of 6.5 to 8.0. This is
considered to be physiological pH. At these pH ranges in the cell the amino and
carboxyl groups of the standard amino acids are ionized. In other words, the aamino group bears a positive charge while the a-carboxyl group bears a negative
charge. The dipolar or zwitterionic nature of the amino acids is due to the fact

that the pKa value for the a-amino terminal is about 9.4 while the pKa for the acarboxyl terminal is about 2.2. Since amino acids can act as either an acid or a

base they are referred to as ampholytes. Table 6-1 shows the pKa values for the
Copyright by The Berkeley Review

The Berkeley Review


Specializing in MCAT Preparation

Biology

Structure Of Function in Cells &Viruses

Biological Molecules

a-carboxyl and a-amino groups of a selectionof the 20. standard amino acids and
their ionizable side chains. Small pKa values refer to a strong acid while large pKa
values refer to a weak acid.

Amino Acid

pK^a-COOH) pKa(a-NH3+)

Glycine

2.4

9.8

Alanine

2.3

9.9

pl^ (side chain groups)

Aspartic Acid

2.0

9.9

3.9

Glutamic Acid

2.2

9.7

4.2

(Y-COOH)

Histidine

1.8

9.2

6.0

Imidazole)

Cysteine
Tyrosine
Lysine
Arginine
Serine

1.8

10.8

2.2

9.1

2.2

9.2

1.8

9.0

2.2

9.2

8.3
10.1
10.8
12.5
13.0

(P-COOH)

(Sulfhydryl)
(Phenol)
(e-amino)
(Guanidino)

(Hydroxyl)

Table 6-1

The pK values for the ionizing groups of a few of the standard 20 amino acids.

The Henderson-Hasselbalch Equation


The Henderson-Hasselbalch equation (6-1) can be used to determine the fraction
of either the a-amino, a-carboxyl, or side chain groups that are ionized at a

particular pH. In this equation [Ae] is the concentration of conjugate base while
[HAj is the concentration of conjugate acid.

(6-1)

pH = pKa + log [A']

[HA]

Henderson-Hasselbalch Equation

Let's consider the ratio of the protonated to the unprotonated a-amino group of a
general amino acid at a pH of 7.0. The pKa for a typical a-amino group is about
9.4. Using the Henderson-Hasselbalch equation, we find that the ratio of the

protonated a-amino group to the unprotonated a-amino group is about 102-4 or


pH-PKa

[A"]/HA

-3

0.001

-2

0.01

-1

0.1

10

100

1000

251:1.This tells us that the predominant form of the a-amino group at a pH of 7.0
is the protonated form. The same type of analysis for the a-carboxyl group tells
us that the ratio of the a-carboxylate anion to the protonated a-carboxyl group is
about 63,000:1.

(6-2)

7.0 = 9.4 + log

[R-NH2]

[R-NHJ]

2.4 =[R-NH?]=25I (6_3)


[R-NH2]

Similarly, at a neutral pH of 7.0 the ratio of the protonated to non-protonated

Table 6-2

side chain of histidine is about 1:10.

Difference between pH and


pKa and the relationship to the
ratio of conjugate base to
conjugate acid.

If we know the difference between the pH and the pKa (i.e., pH - pKa), then we
can establish a ratio (Table 6-2) between the concentration of conjugate base to

conjugate acid. If a particular amino acid is required at the catalytic site of an


enzyme, knowing the ratio between conjugate base and conjugate acid will help
determine the maximum potential activity of the enzyme.

Copyright by The Berkeley Review

The Berkeley Review


Specializing in WCAT Preparation

Biology

Structure S? Function in Cells &Viruses

Biological Molecules

Isoelectric Point (pi)

The isoelectric point (pi) is that p H at which an amino acid (or a molecule)
carries no net electric charge. By manipulating the Henderson-Hasselbalch

equation we can obtain an expression (6-4) thatstates that the isoelectric point is
simply the arithmetic mean of two pKa values.

, _ [pKat + pKa2]

(6-4)

For example, the isoelectric point for the amino acid glycine is 6.1 while the
isoelectric point for the amino acid lysine is 10.0. If we were to place these two
amino acids in an electric field, we would find that at any pH above their
isoelectric points they would migrate toward the anode (positive electrode) while
at any pH below their isoelectric points they would migrate toward the cathode
(negative electrode). Thischaracteristic allows for separation of amino acids by a
process called electrophoresis. Paper electrophoresis is generally used for
separating mixtures that contain charged molecules which are small while gel
electrophoresis is used for separating proteins and nucleic acids.
Titration Curves

At physiological pH glycine exists in solution as the dipolar ion. The net charge

on this amino acid would be 0. However, ifwe were to add a strong acid to the
dipolar solution, the carboxylate group would become protonated and the
overall charge of the amino acid would be +1.Similarly, addition of a strong base
to the dipolar solution would remove a proton from the protonated a-amino
group, giving the amino acid a net charge of -1. As shown in Figure 6-7, the
ionization of glycine depends on the pH of the solution.
PH=1
~

H
I

O
II

pH 6.1

H
i

ii

H3N C- C- O

H,N- C C- OH =5=

pKa

2.4

[Net charge is+lj

pH14
H

II

H2N C C- O

pKa

9.8

[Net charge is 0]

[Net charge is-1]

Figure 6-7
Ionization of glycine.

The ionization of glycine can be followed by using a titration curve (Figure 6-8).
We can start our titration by adding equivalents of base (such as NaOH) to a

solution of glycine which is fully protonated. The initial pH of this solution might
be in the neighborhood of 1 and the overall net charge on glycine is roughly +1.0.

The first midpoint of the titration curve comes when [NH3CH2COOH] =


[NH3CH2COOe]. The Henderson-Hasselbalchequation tells us that at this first
midpoint the pH = pKa. In other words, the pH at this point is 2.4 (which is the
pKa of the first ionizable proton), and half of the solution is composed of

NH3CH2COOH while the other half of the solution is composed of


NH3CH2COOe. The overall net charge on glycine in solution at this point is
about +0.5.

Copyright by The Berkeley Review

The Berkeley Review


Specializing in MCAT Preparation

Biology

Structure fie Function in Cells &Viruses

Biological Molecules

Addition of more base will eventually convert all of the NH3CH2COOH to


NH3CH2COOe. At this point the first equivalence point of the solution has
been reached. The overall net charge on glycine is 0. The isoelectric point has
been reached and the pH is about 6.1.
In order to reach the second midpoint in the titration we add more base. At the

second midpoint we find that [NH3CH2COOe] = [NH2CH2COO]. Again, the


Henderson-Hasselbalch equation tells us that at the second midpoint the pH =
pKa. The pH at this point is 9.8 and the overall net charge on glycine in solution
is about -0.5.

Second Midpoint

12n

[NH3+CH2COO-] = [NH2CH2COO-]

0.5

[NH3+CH2COOH]

1.0

Equivalence ofNaOH Added

Figure 6-8
Titration of glycine.

By the time the second equivalence point is reached enough base has been added
to completely titrate all the ionizable protons on glycine. The dominant species in

solution is now NH2CH2COOe. The overallnet charge on the amino acid is -1.
Buffers

The ability of a solution to resist a change in its pH when either an acid or a base
is added is the principle behind the buffer capacity of a solution. In Figure 6-8
note that the slope of the titration curve is much less near the first and second
midpoints than near the equivalence point. For each increment of base added in
the vicinity of the pKa's of glycine the pH changes very little. This means that

the numerical value of the log ([Ae]/[HA1) in the Henderson-Hasselbalch


equation doesnot change that much. In other words, [Ae] approximately equals
[HA] in the region of the pKa. If a weak acid is within 1 pH unit of its pKa value,
it resides within a good buffering range.

Copyright by The Berkeley Review

The Berkeley Review


Specializing in MCAT Preparation

Biology

Structure &Function in Cells &Viruses

Biological Molecules

Proteins

The 20 standard amino acids that we have mentioned can be linked together to
form long polypeptide chains. Amino acids can be joined to one another by a
special type of amide bond called a peptide linkage. For example, in Figure 6-9
two amino acids are joined together to form a dipeptide.

As we will seein a laterdiscussion, this reaction requires an inputoffree energy.


This means that the hydrolysis of the dipeptide would be more favorable than its
synthesis. All hydrolysis reactions are favorable reactions.
Peptide Bond

H,0

I
II
0
H3N C C- O

^ HI II 0
H3N C C- O

II

R2

Ri

I
II
0
H,N c-tc-N-r- C - C - 0

J-

H20

R,

Figure 6-9
Formation of a dipeptide from two amino acids.

A polypeptide is a convenient term for any length of a polymer of amino acids. If


we put two amino acids together, we have a dipeptide. If we put three amino
acids together, we have a tripeptide. We might run into the word oligopeptide. An
oligopeptide is simply 10 or so amino acids linked together. A protein is
generally described by 100 or more amino acids linked together.

II

Cysteine

C-N-C C-N-C
I

CH2

SH
SH
I

H
end

HO

HO

1
II
1
II
H,N - C C~ N - - C- C- N l

R,

R2

HO
HO
1
II
1
II
1
II
0
C- C- N - - C- C~ N- C C- 0

ll

Rj

R4

Rj

C - N - C - CN-C

end

CH2
I

II

Cysteine

reduction

oxidation

Figure 6-10
Modified amino acids.

II

C-N-CC-N-C

Amino acid units are often referred to as residues. Consider the pentapeptide
shown in Figure 6-10. This peptide is composed of 5 amino acid residues and is
synthesized (and written) from the amino terminal residue to the carboxyl
terminal residue, left to right, respectively. In other words, if we had the
pentapeptide Tyr-Gly-Gly-Phe-Met, then Tyr would be the amino terminal and
Met would be the carboxyl terminal.
X-ray crystallographic studies performed by Linus Pauling and Robert Corey in
the late 1930s showed that the peptide unit (i.e., the O-C-N-H bonding) is planar
and rigid due to the partial double bond character of the C-N bond (i.e., the
peptide bond). However, there can be rotation about the bonds between the acarbon and the carbonyl carbon and the a-carbon and the nitrogen.

CH,

S
I

S
I

CH,

C-N-C-CN-C
I

II

Cystine

Figure 6-11
Formation of a disulfide

linkage between two cysteine


residues.

Some proteins that contain cysteine residues can form disulfide bonds as shown
in Figure 6-11. For example, within the amino acid sequence of bovine insulin
there are 6 cysteine residues. These residues have the ability to form three
Copyright by The Berkeley Review

The Berkeley Review


Specializing in MCAT Preparation

Biology

Structure &Function in Cells S? Viruses

Biological Molecules

distinct sets of disulfide cross-links. When two cysteine residues are oxidized they
form a disulfide called cystine. Be careful of the difference between cysteine and
cystine.

(a)

In 1951 Pauling and Corey suggested that polypeptide chains have the ability to
fold into a-helices and p-pleated sheets. The a-helix is stabilized by hydrogen
bonding between the CO and NH groups as shown in Figure 6-12a (that are four
residues apart). There are about 3.6 amino acids per turn of the a-helix. They are

separated from oneanother by a translational distance of 1.5 A and a rotation of


100 degrees. The side chains of the amino acid residues extend outward and
away from the helical axis (Figure 6-12b).
Hydrogen bonds between the
C=0 and N-H groups help to
stabilize the a-helix.

In the p-pleated sheet the hydrogen bonding between the CO and NH groups
occurs between different polypeptide chains as shown in Figure 6-13. These
polypeptide chains can either be parallel (running in the same direction) or
antiparallel (running in opposite directions). In the situation of the antiparallel ppleated sheet the polypeptide chain is almost fully extended and the distance
between amino acid residues is about 3.5 A. As a polypeptide chain folds back to
run in the opposite direction, it reverses direction by making a p-Turn. Hydrogen
bonding at the P-Turn occurs between the CO group of one amino acid residue

and the NH group of an amino acid residue which is three residues away.
R
1

0
II

II

<= ^rW-c-fV W "


H

Looking down the a-helix


we see the side chains

II

Hydrogen bond -- =

pointing outward.

0
II

=>^c. N

Figure 6-12
Two views of an a-helix. (a) Side
view, (b) Top view.

l
H

f H. /

(f

ll
0

I
R

H-C- R

II
N
H

'

ll

Figure 6-13

Hydrogen bonding in antiparallel p-sheet.

In 1957John Kendrew worked out the three-dimensional structure of the protein


myoglobin (Figure 6-14). Human myoglobin consists of 153 amino acid residues.
Roughly 75% of this polypeptide chain is in an a-helical conformation. There are
8 major helices called helix A, B, C, D, E, F, G, and H. Within a crevice of the

protein is a heme group which can bind oxygen. The interior of the myoglobin
protein is composed almost entirely of hydrophobic non-polar residues.
Not all proteins have such a high percentage of a-helices within their framework.
For example, ribonuclease S (an enzyme secreted from the pancreas that hydrolyzes RNA) is a protein that is composed of 124 amino acid residues which are
arranged in a number of p-pleated sheets.
Proteins with a single polypeptide chain can be defined by their primary,
secondary, and tertiary structure. The primary structure represents the sequence of
Figure 6-14
Three dimensional structure of

myoglobin.

amino acids in a protein and includes the location of disulfide bonds. The
secondary structure represents the spatial arrangement of amino acids that are
close to one another while the tertiary structure represents the spatial
arrangement of amino acids that are far from one another. A protein with tertiary

Copyright by The Berkeley Review

10

The Berkeley Review


Specializing in MCAT Preparation

Biology

Structure &? Function in Cells &Viruses

Biological Molecules

structure can be referred to as a subunit. If various subunits associate with one

another, then the protein is said to have quaternary structure. For example,
myoglobin is a protein with tertiary structure but hemoglobin, because it
contains four polypeptidesubunits,is a protein with quaternary structure.
The level of structure in a protein can best be understood by considering
hemoglobin. The four polypeptide (two a and two p) subunits of hemoglobin
form a tetramer which is spheroidal in shape. Associations between these
subunits are formed through hydrophobic, ionic, hydrogen bonding, or polar
interactions. It is the interactions between these four polypeptide subunits that
gives hemoglobin its quaternary structure.

However, if we were to examine just one of the polypeptide subunits, we would

find that it would primarily contain a number of a-helical domains (given the
letters A-H). The complete spatial arrangement of all the amino acids to one
another within this polypeptide subunit defines the tertiary structure. The
secondary structure is simply the spatial arrangement of amino acids adjacent to
one another within the polypeptide chain. The primary structure includes the
primary sequence of the polypeptide chain. These four levels of protein structure
are depicted in Figure 6-15. In particular, we have focused in on a set of amino
acids in the E helix of one of the p subunits of hemoglobin.

It turns out that it is the primary structure that determines the way in which a
protein will fold up on itself. Insight into this phenomenon came from the work

Tertiary

that Christian Anfinsen did with bovine ribonuclease. He found that in the

presence of P-mercaptoethanol (which cleaves disulfide bonds) and 8 M urea, the


ribonuclease enzyme was denatured and lost its enzymatic activity.

Enzymatic activity was restored to this protein after the sulfhydryl groups on the
cysteine residues were oxidized and dialysis of the P-mercaptoethanol and urea
was complete (Figure 6-16). This experiment confirmed that it is the primary
structure of a polypeptide that determines the three-dimensional tertiary
structure of a protein.

(fo
CrJ

Chemical treatment
Removal of chemical
treatment

Quaternary

V^SH

Figure 6-15

Xhs^/Vsh

The four levels of protein structure


in the molecule hemoglobin.

hsA-4 ^sh

Ribonuclease

Denatured Ribonuclease

(active)

(inactive)

Figure 6-16
Chemical denaturation of ribonuclease using P-mercaptoethanol.

Out of the many possible ways in which the ribonuclease protein can fold into its
tertiary structure, only one will lead to the correct enzymatic activity. It turns out
that protein folding is not a random search. Why? Because it would take too long
to complete the process. In fact, if a protein had 100 amino acid residues, each

havingthe possibility of assuming3 different positions, then therewould be 3100


possible structures. This would take about 1.6 x 1027 years! The universe,
however, is only about1.5x 1010 yearsold (give or take a few billion years).
Copyright by The Berkeley Review

11

The Berkeley Review


Specializing in MCAT Preparation

Biology

Structure fie Function in Cells fi? Viruses

Biological Molecules

How is it that polypeptides fold into a tertiary structure? One theory says that
sections of the primary structure fluctuate between being in the native linear
(primary) arrangement and either an a-helix or a p-pleated sheet and that when
two such forms of secondary structure interact with one another by diffusion

they tend to stabilize each other. The exact mechanism as to how this happens is
not known.

The idea of two helices being side by side is rather interesting, especially in
proteins that bind to DNA. If two a-helices, each containing at least 4 leucine
residues and located on separate proteins, come together through interdigitation
of those leucine residues, then a leucine zipper will form binding the two proteins
together. Even though the leucine zipper does not directly interact with the DNA
double helix, it does allow for the binding of some DNA regulatory proteins. We
will examinethis in a littlemore detail when we discuss the expression of genetic
information.

Essentiallyall reactionsin a cell are catalyzed by proteins called enzymes. When


a substrate comesinto contact with the active site or catalytic site of an enzyme,
it will be bound at that site in such a way that the enzyme will be able to convert
that substrate into a product molecule. There are other enzymes that, besides
having a catalyticsite, have a regulatory site or allosteric site. The regulatory site
is a site that is different from the catalytic site. Enzymes of this nature are
generally referred to as allosteric enzymes (from the Greek alio for "other" and
stereos for "space" or "site").

We will find that many different proteins have these allosteric regulatory sites
and that these sites can control the binding of various substrate or modulator
molecules. A good example of an allosteric protein is hemoglobin. Hemoglobin
has four polypeptide subunits (it has quaternary structure), each having a heme
group that has the capability of binding an oxygen molecule. When one oxygen
molecule binds to the heme group of one of the polypeptide subunits, that
binding information is somehow transmitted to the other three polypeptide
subunits, thus facilitating their ability to bind oxygen.

Copyright by The Berkeley Review

12

The Berkeley Review


Specializing in MCAT Preparation

BlOlOgy

Structure fi? Function in Cells &Viruses

Biological Molecules

f^k

Carbohydrates
The name carbohydrate is misleading. It was coined at a time when the formulae

of carbohydrates was just becoming known. Almost all carbohydrates had a


similar empirical formula which looked like Cn(H20)n. This formula leads us to

believe that there is a water molecule attached to each carbon atom thatappears
in the structure. However, this is not the case. A better empirical formula to use

H-C-OH
I

is (CH20)n.

CH2-OH

In order for a molecule to be classified as a carbohydrate, it must have an


aldehyde or a ketone functional group and two or more alcohol functional
groups. Carbohydrates are simply polyhydroxylated aldehydes or ketones. If we
follow this rule, we find that the simplest carbohydrate is obtained when n = 3 in
the empiricalformula (CH20)n. Glyceraldehyde and dihydroxyacetone are two

simplest carbohydrates that can be formed (Figure 6-17). Glyceraldehyde has an

Glyceraldehyde
CH,-OH
I

C=0
I

CH2-OH

Dihydroxyacetone

aldehyde functional group while dihydroxyacetone has a ketone functional


group. Each has two alcohol groups. If the carbohydrate contains an aldehyde, it

Figure 6-17

is referred to as an aldose. If it contains a ketone, it is referred to as a ketose.

Molecules with the molecular

formula (CH20)n.

As a group carbohydrates are the most abundant biological molecules on earth.


They can be divided into three major classes: monosaccharides, oligosaccha
rides, and polysaccharides.
Monosaccharides

A monosaccharide, besides being a polyhydroxylated aldehyde or ketone,


contains a backbone of unbranched carbon atoms that are covalently linked to
one another. Glyceraldehyde and dihydroxyacetone are the two simplest
monosaccharides. If the carbohydrate has a 3-carbon backbone, it is called a
triose. Besides the triose sugars, we will also find sugars which are tetroses (4-C),
pentoses (5-C), hexoses (6-C), and heptoses (7-C).
At the turn of the twentieth century Emil Fisher realized that he could devise a
system for naming carbohydrates (and amino acids) based on the structure of
glyceraldehyde. He chose glyceraldehyde because it has just one chiral center (a
carbon atom to which four different substituents are attached). If a molecule has
n chiral centers, it will have 2n stereoisomers. Glyceraldehyde has two stereo

isomers (Figure 6-18). Fisher said that if the hydroxyl group on glyceraldehyde's

Mirror
h

vo

O.

C
I

HO^:^H

H^ ^ OH

CH,-OH

CH,-OH

chiral carbon is to the right, the molecule is in the form of the D-isomer. If the
hydroxyl group is to the left, the molecule is in the form of the L-isomer.

In order to tell if a monosaccharide drawn in the Fisher projection is in the D or L

Figure 6-18

isomeric form, we only need to look at that chiral carbon which is most distant
from the carbonyl carbon. That particular chiral carbon atom is often referred to
as the reference carbon. If the hydroxyl group attached to that reference carbon
is to the right, the molecule is the D isomer. If the hydroxyl group is to the left,
the molecule is the L isomer. Most of the naturally occurring sugars are found in
their D form (while most of the naturally occurring amino acids are found in

Glyceraldehyde

D-Glyceraldehyde and its


enantiomer.

their L form).

Monosaccharides can either be found in the linear form or in the cyclic form.
Many sugars that contain five or more carbon atoms in their backbone prefer to

be in the cyclic form. For example, the aldohexose D-glucose (Figure 6-19) can
Copyright by The Berkeley Review

13

The Berkeley Review


Specializing in MCAT Preparation

Biology
0V

c-i

C
I

C-2

H -

C- OH
l

C-3

HO C-H
I

C-4

H -

C- OH

C-5

H -

C- OH
I

C-6

CH2OH

Structure & Function in Cells & Viruses

Biological Molecules

form a six-membered ring referred to as a pyranose. In the formation of the ring


(Figure 6-20), the oxygen atom of the C-5 hydroxyl group reacts with the
carbonyl carbon atom to form two different diastereomers, a-D-glucopyranose
and P-D-glucopyranose. Because these two molecules differ only in the
configuration about the C-l carbon atom, they are referred to as anomers, and
the C-l carbon atom itself is referred to as the anomeric carbon. In the a anomer

note that the OH group at the anomeric carbon is on the opposite side of the ring
from the CH2OH group that is attached to the reference carbon. In the P anomer
the OH group at the anomeric carbon is on the same side of the ring as that
CH2OH group.

D-Glucose

CH2OH

CH2OH

Figure 6-19
The straight chain form of D-

CH2OH

O
OH

Glucose.

y*o

OH

H y *

OH

OH

CH,OH

C-2

C=0

C-3

HO- C- H

OH

P-D-Glucopyranose

a-D-Glucopyranose
C-l

OH

Figure 6-20
Formation of the two anomers of D-Glucopyranose.

C-4

H -

C- OH

C-5

H -

C- OH

I
I

C-6

CH2OH

Carbohydrates can also exist in a five membered ring. In this case the cyclicsugar
would be called a furanose. The ketohexose D-fructose (Figure 6-21) can cyclize
to form a five membered ring (Figure 6-22). There are two diastereomers, oc-Dfructofuranose and P-D-fructofuranose.

D-Fructose

^H

Figure 6-21
HOH2C

The straight chain form of D-

CH2OH HOH,C

o'\ CH2OH HOH2C

^O^

OH

Fructose.

mH
H V
OH

Hoy ^O =5=^ '\H HO

HO
\# OH

H \/

OH

\# CH2OH
H

p-D-Fructofuranose

oc-D-Fructofuranose

Figure 6-22
Formation of the two anomers of D-Fructofiiranose.

Formation of the pyranose ring in D-glucopyranose results from a general


reaction involving an alcohol and an aldehyde. The resulting derivative is
referred to as a hemiacetal. Similarly, formation of the furanose ring in Dfructofuranose results from the general reaction involving an alcohol and a
ketone. In this case a hemiketal is formed. Formation of these derivatives is

shown in Figure 6-23.

Since D-glucose and D-fructose readily interconvert between the linear and cyclic
forms, they can undergo reactions which are typical to aldehydes and ketones,
respectively. Two oxidizing agents used to identify the functional groups of

carbohydrates is the Tollens' reagent (containing an Ag complex) and the

Benedict's reagent (containing a Cu2 complex). If theTollens' reagent is used to


Copyright by The Berkeley Review

14

The Berkeley Review


Specializing in MCAT Preparation

Biology

Structure &Function in Cells fi? Viruses

Biological Molecules

identify an aldoseor a ketose, the sugar is oxidized and the Ag ion is reduced to
silver metal (which precipitates as a silver mirror on the sides of the reaction
vessel. If the Benedict's reagent is used, the sugar is oxidized and the Cu2 ion is

reduced to give a brick-red precipitate. If an aldose or a ketose is capable of


reducing these ions, those sugars are referred to as reducing sugars.
Carbohydrates that contain a hemiacetal or a hemiketal group give positive tests
with Tollens' and Benedict's reagents.

R2- <\ JO- H

O
ii

R.-C-H + HO-R,

R,

Aldehyde

Alcohol

R2- O^ O- H

O
II

R.-C-R, + HO-R,

R,

Hemiacetal

Ketone

Alcohol

R3

Hemiketal

Figure 6-23
Hemiacetal and hemiketal formation.

Oligosaccharides
Oligosaccharides are relatively short chains of monosaccharides linked to one

another by a glycosidic bond. Disaccharides are the most common oligo


saccharides. The disaccharide sucrose (tablesugar) is formed by the linkage of o>
D-glucopyranose and p-D-fructofuranose. The bond connecting the two mono
saccharides is termed an O-glycosidic bond, and it is the carbohydrate analog of
the peptide bond found in proteins.

The systematic naming of disaccharides (and larger sugars) follows a few simple
rules. Let's consider sucrose as an example (Figure 6-24). First, notice how the
two monosaccharides in sucrose are joined together. They are linked through an
oxygen atom, which means that the bond is an O-glycosidic bond. If we were to
hydrolyze that bond with H2O, we would get the two individual mono
saccharides (glucose and fructose).
(:h2oh

CH2OH
1

VOH

H>

H(

HO :h
1

>-\V
k < HOJ

1
H

OH

)H

0-ha-D-glucopyrai iosyl-(l->:

/CH,OH
H

3-D-fructofuranoside

'/

HOCH2 .0^

3H

HC

H/
)H

ht
i

a- D-(

HO)*
( )H

OH

jlucopyranc>se

H?/
[fCH,OH
H

P-I)-Fructofuranose

(Sucros e)

Figure 6-24
Sucrose is also systematically called 0-a-D-glucopyranosyl-(l-2)-p-D-fructofuranoside.

Locate the anomeric carbon for glucose. Is the OH group attached to that
anomeric carbon in the a or p conformation? It is a because the hydroxyl group
at the anomeric carbon is on the opposite side of the ring from the CH2OH group
that is attached to the reference carbon. Is the OH group attached to the anomeric
carbon of fructose in the a or P conformation? It is P because the hydroxyl group
at the anomeric carbon is on the same sideof the ring from the CH2OH group that
is attached to the reference carbon. This is an unusual linkage in that the Oglycosidic bond is formed between two anomeric carbons. If we view this linkage
Copyright by The Berkeley Review

15

The Berkeley Review


Specializing in MCAT Preparation.

Biology

Structure fir: Function in Cells & Viruses

Biological Molecules

from the point of view of the glucose molecule, we will see that the bond is
formed between the C-l atom of glucose and the C-2 atom of fructose. This is
written as (l-2) in the naming of the molecule. Putting what we have learned so
far together, we could name sucrose as O-oc-D-glucopyranosyl-(l>2)-P-D-

fructofuranoside. The O in front of the name is to remind us that the linkage is


an O-glycosidic linkage. The -ide at the end of the name tells us that sucrose is
not a reducing sugar.

We know that a sugar is a non-reducing sugar if its hemiacetal or hemiketal


group has been converted to an acetal or ketal group, respectively. We can form
an acetal or a ketal as outlined in Figure 6-25. If a carbohydrate contains only an

acetal or a ketal group, it will not react with either the Tollens' reagent or
Benedict's reagent.
R,-0

o-H

,\
R,

R2-0

H0-R3 =

O-R,
C
/

R2"0

O-H

,C

R2-O

+ HO- R4

R, H

Hemiacetal

Alcohol

Acetal

O-R4

R, R3
Hemiketal

Alcohol

Ketal

Figure 6-25
Acetal and Ketal formation.

Lactose (milk sugar) is a disaccharide that is found only in milk. It is composed


of p -D-galactopyranose and p-D-glucopyranose linked together through a
p(l->4) O-glycosidiclinkage (Figure 6-26). Notice that the anomeric carbon of the
galactose residue is tied up in an acetal linkage with glucose. However, the
anomeric carbon of the glucose residue is just part of a hemiacetal group, and
since it can be oxidized, lactose is a reducing sugar. [The structure of lactose has
been given on the MCAT a number of times.]
CH2OH

CH,OH

31

OH

CH,OH

CH2OH

f 2

OH

0-P-D-galactopyranosyl-(1-4)-P-D-glucopyranose

OH

p-D-Galactopyranose

OH

P-D-Glucopyranose

(Lactose)

Figure 6-26
The disaccharide lactose and its two monosaccharide components, D-galactose and D-glucose.

Polysaccharides

Polysaccharides (glycans) are large molecules which contain many mono


saccharide residues linked together through glycosidic linkages. Polysaccharides
can consist of just one type of monosaccharide, in which case they are called
homopolysaccharides, or they can consist of different types of monosaccharides,
in which case they are called heteropolysaccharides. Each type can be either
unbranched or branched. Two important (storage) polysaccharides are starch
and glycogen.

Copyright by The Berkeley Review

16

The Berkeley Review


Specializing in MCAT Preparation

Biology

Structure &Function in Cells &? Viruses

Biological Molecules

Starch is a food reserve in plants. It can be found as a-amylose, a linearpolymer


of unbranched D-glucose residues linked together in a(l-4) linkages, or as
amyldpectin, a branched polymer of D-glucose residues linked together
primarily in a (l-4) linkages but at the branch points in a(l-6) linkages.
Examples of both polymers are shown in Figure 6-27. The branch points in
amylopectin occur roughly every 24 to 30 glucose residues.
CH,OH

O-

OH

OH

o
I

CH2OH

CH2OH

r2
H

OH

37~~f 2
H

CH2OH

l.

6 CH2

VOH H>'
_ o-iy_^L
o

OH

a-amylose

OH

3|

f2

OH

amylopectin

Figure 6-27
The two forms of starch: a-amylose and amylopectin.

Starch can be found in wheat, rice, corn, and potatoes and is the major
carbohydrate source in the human diet. Many of the starches contain about 15%
a-amylose and 85% amylopectin. Digestion of starches begins in the mouth with
the enzyme salivary a-amylase. This enzyme hydrolyzes many of the a(l>4)
linkages in starch and degrades large polysaccharides into smaller oligo
saccharides. Once the oligosaccharides pass through the stomach and into the
duodenum of the small intestine, they are degraded even further by pancreatic ocamylase to disaccharides, trisaccharides, and small branched oligosaccharides
referred to as dextrins. Enzymes within the intestinal system degrade these
oligosaccharides into individual monosaccharides, which are then absorbed by
the intestinal epithelial cells and passed to the blood.
Glycogen is the storage polysaccharide common to all animals and is located
primarily in skeletal muscle and liver tissue. Glycogen has a structure similar to
that of amylopectin, except that the branch points in glycogen come about every
8 to 12 glucose residues.

Lipids
Biological lipids are molecules that can readily dissolve in nonpolar solvents but
are relatively insoluble in water. These compounds are chemically and
functionally quite diverse. Some of the classes of lipids that we will examine are
the fatty acids and triacylglycerols, glycerophospholipids, sphingolipids, and
cholesterol.

Copyright by The Berkeley Review

17

The Berkeley Review

Specializing in MCAT Preparation

Biology

Structure & Function in Cells & Viruses Biological Molecules


Fatty Acids & Triacylglycerols

Fatty acids are carboxylic acids with a hydrocarbon side chain. The majority of

HO.
\

HO.

,0
/

H,C

H,C
CH,

CH,

H,C

H,C
CH,

CH,

H2C

H2C
CH2

CH,

H^

H,C

2\ .H

CH,
H,C

CH,

*
H,C

H^

CH,
/

CH,
(

H2c(
CH2

H2C
CH,
H,C

H3C
Palmitic acid

fatty acids have side chains that are unbranched and contain an even number of

carbon atoms. In plants and animals the more common fatty acids contain either
16 carbons or 18 carbons. If a fatty acid is saturated, all of its carbon atoms
(except the carbonyl carbon) will have a full complement of hydrogen atoms. We
will not find any double bonds between carbon atoms. An unsaturated fatty acid,
however, will contain double bonds between specific carbon atoms. Two
common fatty acids are shown in Figure 6-28.

CH,

and at room temperature tend to become solids referred to as fats. A double

CH3

bond in an unsaturated fatty acid places a kink in the hydrocarbon chain, thereby
decreasing the van der Waals interactions and lowering the melting point.
Triacylglycerols that contain a high degree of unsaturated or polyunsaturated
fatty acids (e.g., oliveoil)tend to be oils at room temperature.

H2C

(C,6)

In nature, fatty acids are rarely free. Rather, they are esterified to a glycerol
backbone to form a triacylglycerol (Figure 6-29). In animals these compounds are
synthesized and stored in adipocytes (fat cells). Triacylglycerols are neutral fats
and serve as storage depots for fuel used in metabolism. Triacylglycerols that
contain a high degree of saturated fatty acids (e.g., butter) pack together quite
well and form crystallinestructures that havefairly high melting points. This is due
to the large van der Waals attractions between the methylene (-CH2-) groups of
the hydrocarbon portion of the fatty acid. As the molecular weight of a saturated
fatty acid increases, so does the melting point. The fluidity of the lipid decreases

Oleic acid

(C,8)

Figure 6-28
Two common fatty acids.
H2c OH

HC- OH

HO

H2CO

HO

HC-0

H2C OH

Glycerol

H2C O

Fatty Acids

Triacylglycerol

Figure 6-29
The formation of a triacylglycerol.

Glycerophospholipids

The glycerophospholipids (or phosphoglycerides) are the predominant lipid


component of all biological membranes. Glycerophospholipids (Figure 6-30)
have two fatty acid residues esterified to the C-l and C-2 carbons of glycerol.
Esterified to the C-3 carbon of glycerol is a phosphate group which bears a
negative charge at pH 7.0. Three of the more common molecules that can be

attached to the phosphoryl head group are ethanolamine (HOCH2CH2NH3),


choline (HOCH2CH2N(CH3)3), and serine. Glycerophospholipids are amphiphilic in that they have nonpolar tails (the hydrocarbon chains of the fatty acids)
and a polar heads(the phosphate group and its associated attachment). Because
glycerophospholipids contain a phosphate group, they are also referred to as
phospholipids.

Copyright by The Berkeley Review

18

The Berkeley Review


Specializing in MCAT Preparation

Biology

Structure fit Function in Cells fit Viruses

Biological Molecules

Sphingolipids

The sphingolipids are not based on a glycerol backbone. Instead, they are
derivatives of amino alcohols. Onecommon derivative is sphingosine. Attached
tothe C-2 carbon is an amino group that can belinked toa fatty acid through an
amide linkage. This molecule is called a ceramide (Figure 6-31a), and is the

H2C O

structural residue that is common to all sphingolipids.

If a phosphoethanolamine or a phosphocholine group is attached to the C-l


carbon of the ceramide, the sphingolipid becomes a sphingomyelin (Figure 631b),and sphingomyelins are abundant in the myelin sheaths that surrounds the
axons of nerve cells in the nervous system.
(a)

C-3

(b)

HO-CH-C=C-(CH2)12-CH3
H

C-2

HC

HC-O

CH3

H2C O- P- O'

N-CH3
CH,

Figure 6-30
Phosphatidylcholine.

HO-CH-C= C-(CH2)12-CH3
H

N
I

C-I

H,C OH
Ceramide

Sphingomyelin

Figure 6-31
Different types of sphingolipids.

When a single monosaccharide like glucose or galactose is attached to the C-l


carbon of the ceramide, the sphingolipid becomes a cerebroside. Cerebrosides
are the simplest of the sphingoglycolipids. The more complex sphingoglycolipids are the gangliosides. In this case several sugar residues (i.e., an
oligosaccharide) are attached to the C-l carbon of the ceramide (Figure 6-32).
The extended oligosaccharide residues of a ganglioside are quite important as
they are thought to be involved in recognition of other molecules at the surface of
the cell. For example, specific gangliosides act as the antigenic determinants in
the human ABO blood group system. Note that cerebrosides and gangliosides do
notcontain a phosphate group.

Sugar Residues

Figure 6-32
A typical ganglioside.

Cholesterol

Cholesterol is synthesized in the cytosol and generally exists as the cholesterol


ester. This lipid is a major constituent of eukaryotic animal plasma membranes
and is an intermediate in the biosynthesis of all human steroids. It has a fairly
rigid set of four fused nonpolar rings and a polar hydroxyl group that gives it a
slightly amphiphilic attribute.

Steroid Hormones

1. Progesterone
2. Glucocorticoids
3. Mineralocorticoids

There are five major classes of steroids (Table 6-3), all of which are synthesized
in the mitochondrion. Synthesis of a steroid hormone begins with the hydrolysis of
cholesterol esters in the cytosol and the subsequent transport of cholesterol into
the mitochondrion. Adrenocorticotropic hormone (ACTH) stimulates the
conversion of cholesterol to pregnenolone, an intermediate in the pathway of
steroid synthesis (Figure 6-33).
Copyright by The Berkeley Review

19

4. Androgens
5. Estrogens
Table 6-3

Steroid classes.

The Berkeley Review


Specializing in MCAT Preparation

Biology

Structure fit Function in Cells fit Viruses

Biological Molecules

Later, when we discuss basic physiological processes, we will return to these


steroids and discuss them in more detail. But for the moment, let's consider their

general function.
Progesterone has a primary function in women where it prepares the uterine
lining for implantation of an ovum. If implantation occurs, this steroid is
necessary to maintain the endometrial lining of the uterus and hence maintain
pregnancy. During pregnancy progesterone stimulates mammary tissue growth
in preparation for parturition. Progesterone is also synthesized in low levels in
the testes of the male and in the adrenal cortex of the adrenal glands (located on
top of the kidneys) of both sexes.
Cholesterol

Pregnenolone

Cortisol is synthesized and secreted from cells in the cortex of the adrenal glands.
In the liver Cortisol acts to increase both glycogen synthesis and gluconeogenesis.
In skeletal muscle this glucocorticoid acts to decrease both glucose uptake and
protein synthesis, and increases protein catabolism. In adipose tissue Cortisol
increases lipid mobilization and decreases glucose uptake. Cortisol is also known
as hydrocortisone.

Progesterone

Aldosterone is also synthesized and released from cells in the adrenal cortex. It

acts to increase the reabsorption of sodium (Na) at the level of the kidney,
Testosterone

Cortisol

intestines, salivary glands, and sweat glands. The net effect is to cause retention

ofNa in theextracellular fluid (ECF) thereby increasing ECF volume. Not only
will this lead to an increase in blood volume, but it will also lead to an increase in

Aldosterone

blood pressure and blood flow.

Testosteroneis synthesized in the male in the Leydig cells of the testes and aids in
sperm maturation. This androgen can also reach the blood and circulates
Estradiol

throughout the body, promoting a variety of biological effects, including


development of the secondary sex characteristics

Figure 6-33
Synthesis of the major steroids
from cholesterol.

Estradiol is the primary estrogen in women and is synthesized in the theca cells

of the ovarian follicles. Estrogens have a variety of biological effects, including


development of secondary sex characteristics, regulation of the ovarian cycle,
and control of certain metabolicprocesses.

Copyright by The Berkeley Review

20

The Berkeley Review


Specializing in MCAT Preparation

Biology

Structure fit Function in Cells fit Viruses

Biological Molecules

Nucleic Acids

Genetic information in organisms is carried in the form of nucleic acids. Nucleic

acids occur as either deoxyribonucleic acid (DNA) or ribonucleic acid (RNA).


Both DNA and RNA are made from molecular units called nucleotides, and each

nucleotide consists of (1) a nitrogenous base, (2) a pentose sugar, and (3) phos
phoric acid. DNA and RNA arepolynucleotides ofhigh molecular weight.
Nitrogenous Bases

DNA contains four different nitrogenous bases. Adenine (A) and guanine (G) are
referred to as purines while thymine (T) and cytosine (C) are referred to as
pyrimidines. These basesare essentially planar and relatively insoluble in water.
RNA alsocontains four differentnitrogenous bases. Theonlydifference between
thebases ofDNAand RNA is that in RNA thymine is replaced by uracil (U). The
structures of these bases are shown in Figure 6-34.

H,N
Adenine

Cytosine

Thymine

Guanine

Uracil

Figure 6-34
The nitrogenous bases of DNA and RNA.

Pentose Sugar

The pentose sugar found in DNA is 2'-deoxy-D-ribose while in RNA it is simply


D-ribose (Figure 6-35). In order to avoid ambiguity between the numbering
system in the nitrogenous bases and the numbering system in the ribose ring, the
carbon atoms of the furanose ring are given a number immediately followed by a
prime (') marking. Note that the C-l' hydroxyl group in both 2'-deoxy-D-ribose
and D-ribose is in the P configuration.
5"

HOCH2

/\H

HV

fy
HO

5'

OH

O.

HOCH2
4' )C

HJ
H

HV

2'\
H

2'-deoxy -D-ribose

,0

OH

H J\
\# H
2'\

HO

OH

D-ribose

Figure 6-35
The P-furanose rings of DNA and RNA.

Phosphorus

The pH of a cell's cytoplasm is approximately 7.2. Phosphoric acid (H3PO4) is an


important molecule in the cell and helps in the buffering of the cytoplasm. There
are three hydrogen atoms which are capable of dissociating, each at a different
pKa value (Table 6-4).

Copyright by The Berkeley Review

21

The Berkeley Review


Specializing in MCAT Preparation

Biology

Structure fit Function in Cells fit Viruses

Biological Molecules

Table 6-4

The three ionizations of phosphoric acid.

What is the relative concentration of free phosphoric acid in the cytoplasm?


In order to determine this we need to use the Henderson-Hasselbalch

equation. We know that the pH of the cytoplasm is 7.2, so we set the pH in


our equation at 7.2. We also know which pKa value to use. It is that pKa
value which is closest to the pH value we are given. We then solve the
Henderson-Hasselbalch equation (6-1) for the relative concentrations of the

conjugate base (HP042e) and weak acid (H2P04e) in solution. We find


that the ratios of the two species are equal (6-5).

7.2 = 7.2 +log

[HPOJ-]

10o_[HPO|l

x_[HPO41-]

[H2P04]

[H2PO4]

[H2PO4]

(6-5)

Why is it important to have an understanding of phosphoric acid? First, the


molecule has an excellent buffering capacity. Since phosphoric acid has
three pKa values, it means there are three regions where the conjugate base
(A") is going to equal the weak acid (HA). If small amounts of acid or base
are added to any one of these three regions, the pH of the solution will
essentially remain constant. This is what defines the buffering capacity of a
solution.

Second, phosphoric acid is a component of the nucleotide unit in both


DNA and RNA. Each nucleotide is linked to the next by a phosphodiester
bond. The backbone of DNA and RNA consists of alternating pentose
sugars and phosphate groups. Since the hydrogen on the hydroxyl group
of each phosphate residue has a pKa close to 3, we find that each
phosphate in DNA and RNA has a negative charge associated with it,
making the backbone quite polar. The nitrogenous bases that are attached
to the sugar residues are hydrophobic and direct them-selves away from the
backbone.
DNA

The short segment of single-stranded DNA shown in Figure 6-36 contains


A polynucleotide sequence of DNA
showing the bases, phosphodiester
bonds, and polarity of the polymer.

Copyright by The Berkeley Review

four nucleotide units and three phosphodiester linkages between the ribose
sugars. The nitrogenous bases are attached to the sugar residues by an Nglycosidic linkage. Both of these linkages, the phosphodiester and the Nglycosidic, were formed by a particular dehydration reaction, and this
means that those bonds can be hydrolyzed. Note that the N-glycosidic
linkage is also an acetal linkage.

22

The Berkeley Review


Specializing in MCAT Preparation

Biology

Structure St Function in Cells fi* Viruses

Biological Molecules

The synthesis of a polynucleotide chain has direction or polarity. Each strand of

nucleic acid will have a 5' end and a 3' end. Single strands ofnucleic acids are (by
definition) written with the 5' end at the left and the 3' end at the right. This is
classically expressed in thenotation 5'-3'. If we are discussing a double-stranded
polymer ofDNA (i.e., a DNA double helix or duplex DNA), then the top strand
ofthe duplex is written with the 5'endat the left andthe 3'end at the right while
the bottom strand takes on the opposite orientation, 3' end on the left and 5' end

onthe right. This canbe represented bysegment ofDNA shown in Figure 6-37.
3.4 A

P\5
o

Hydrogen

P>.

. .V.

Bonding ^

'''

P\

1
p

1I JV

T
II
A

^^p^

OH

A
II
T

III
G

3*1/

HO

p^

Stacking of
bases

5'

Figure 6-37
Duplex DNA showing antiparallel arrangement.

Eachstrand of a DNA double helix is joined to the other through interactions of


the nitrogenous bases. Base pairing rules established by Watson and Crick allow
for adenine and thymine to be joined by two hydrogen bonds and guanine and
cytosine to be joined by three hydrogen bonds (Figure 6-38). As the two nucleic

acid polymers wrap around each other in a helical fashion, the bases begin to
stack on top of one another, adding to the stability of the duplex.
Hydrogen

CH,

Bond

r^Y^v^
-" "%. A N
AN X /
Cytos: ine '" H. m

Ribose-N^

Ribose

N,

Y H
Thymine

U.. X J

Ribose

Ribose

Guanine

Adenine

Figure 6-38
Hydrogen bonding between adenine and thymine and guanine and cytosine.

Copyright by The Berkeley Review

23

The Berkeley Review


Specializing in MCAT Preparation

Biology

Structure &Function in Cells &Viruses

Biological Molecules

The structures of nucleic acids can become quite a pain to write out, especially if
the nucleotide sequence is hundreds or thousands of bases long. A number of
different nomenclatures are used to solve this problem. We can let the bases be
symbolized by their respective single letter abbreviations (e.g., G, C, A, & T). Each
pentose sugar unit can be symbolized by a vertical lineand each phosphate by a P
within a circle. Finally, we can place the 3' and 5' numerals on the vertical lines to
indicate where the phosphate attachment takes place. This type of abbreviated
nomenclature is shown in Figure 6-39.
C

c}
3*

(Bl

5'

A
3'

T
3'

5'

5'

3'

5'
*

Figure 6-39
Abbreviated nomenclature for writing polynucleotide sequences.

It is understood that when we write a nucleotide sequence from the 5' to the 3'
end, there will be an unreacted phosphate group at the 5' end and an unreacted
hydroxyl group at the 3' end. We can simplify our notation even more as shown
below. Both sequences are identical to the nomenclature given in Figure 6-36. In
the sequence on the left it is understood that there is an unreacted hydroxyl at

the 3' end. If we want to just consider the bases in a polynucleotide, the sequence
on the right is the easiest and simplest to write. Based on this nomenclature, a
DNA duplex should not be difficult to imagine.
pGpCpApT

GCAT

RNA

As we have mentioned, RNA contains all of the same components as DNA with
the exception that uracil is found in RNA instead of thymine and the ribose ring
of RNA is hydroxylated at the 2' position whereas in DNA it is not.
There are three types of RNA, and all are synthesized by a process called DNA
transcription. Each RNA transcript that is made carries out a specific function in
the cell. RNA polymers that allow for the synthesis of proteins are referred to as
transcripts of messenger RNA (mRNA). The building blocks of proteins are
amino acids. Amino acids are brought to the site of protein synthesis by an RNA
polymer called transfer RNA (tRNA). The site of protein synthesis is the
ribosome, which itself is composed of protein (i.e., amino acids) and RNA. In
particular, the RNA polymers that help define the ribosome is ribosomal RNA
(rRNA).
We will come back to both DNA and RNA in a later discussion and consider the

regulatory aspects that allow for DNA replication, transcription, and translation.

Copyright by The Berkeley Review

24

The Berkeley Review


Specializing in MCAT Preparation

Biology

Structure & Function in Cells fi* Viruses

Eukaryotic Cells

Eukaryotic Cells
Cellular Reproduction
Eukaryotic Chromosomes
Within the membrane-bound nucleus of eukaryotic cells resides the majority of
the genetic information needed for cellular growth and division. This genetic
information is found in the nondividing (interphase) cell in the form of
chromatin, a complex oflinear, double-stranded DNA, and protein (histones). As
the cell prepares for division (mitosis and cytokinesis) the chromatin becomes
highly condensed into chromosomes.
Haploid & Diploid
In humans the characteristic number of chromosomes in the nuclei of each
gamete (eggs and sperm) is 23. There are 22 autosomes and 1 sex chromosome

(either an X or a Y). Since each gamete has just one set of 23 chromosomes it is
said to be haploid (from the"Greek haplous, meaning single), or n, where n is the
number of chromosome types. During fertilization the fusion of a male and
female gamete produces a fertilized egg, or zygote, which now has a chromo
some complement of 46. Cells bearing this number of chromosomes are diploid
(from the Greek diplous, meaningdouble), or 2n. Human somatic cells (i.e., cells
which arenot gametes) are diploid in thatthey have 23 pairs ofchromosomes.

Each pair of chromosomes is referred to as a homologous pair of chromosomes.


They generally havesimilar shapes and sizes and carry thesame hereditary traits
controlled by segments of DNA called genes. Individual members of each pair

Sister

Chromatids

CentromereC^/

are referred to as homologs.


Metacentric

Chromosomes in the nuclei of non-dividing cells are not visible under the light
microscope. However, when cells do begin to divide, either through mitosis or

meiosis, the chromosomes become highly condensed and can easily take up a
variety of stains, making them quite visible under the light microscope. As we
willsee in a few moments, the best phase of the cellcycle to view chromosomes
is during metaphase.

It is important to rememberthat the chromosomes in metaphase have previously


duplicated during the synthetic phase (S phase) of the cell cycle. These chromo
somes have a characteristic size and shape that define the karyotype of the
organism. In the human karyotype the chromosomes are numbered, the largest

Acrocentric

pair of homologous chromosomes being assigned the number 1 and the smallest
pair being assigned the number 22. These are the autosomes. The number 23 is
assigned to the sex chromosome.
Chromatids fi? Centromeres

Each chromosome arrested at metaphase consists of two sister chromatids, each


held together through a constricted region called the centromere. Individual

chromatids will become individual chromosomes when the centromere joining


the sister chromatids divides and the chromatids are allowed to separate. By
definition chromosomes containing just one chromatid do not exist. This is an
important point as it will help us understand exactly how much DNA there is
and how many chromosomes there are at any given moment in the life of a cell.
Copyright by The Berkeley Review

25

Telocentric

Figure 6-40
General classification of

eukaryotic chromosomes based


on centromere position.

The Berkeley Review

Specializing in MCAT Preparation

Biology

Structure & Function in Cells & Viruses

Eukaryotic Cells

The position of the centromere along the length of the chromosome allows for a
general classification of eukaryotic chromosomes (Figure 6-40) as either being
metacentric, acrocentric, or telocentric.
Histones & Nonhistones

If all the DNA of all the chromosomes within a nucleus of a single human cell

were placed end-to-end, it would stretch to about 6 feet. How is it that all of this

DNA, roughly 6 x 109 base pairs, can be so successfully confined to the diploid
nucleus of a cell and still retain its functionality?

The two major types of proteins associated with the structure of DNA are
histones and nonhistones. The most abundant are the histones, which are basic

proteins consisting of a high percentage of lysine (Lys) and arginine (Arg)

residues. At physiological pH the side chain amino groups (-NH3) of these


amino acids bear a positive charge. An electrostatic relationship can be easily
formed with the negatively charged DNA polymer. Nonhistones are proteins that
associate with DNA, but are not histone proteins. Rather than being basic, these
proteins are acidicand bear a net negative charge. A classic nonhistone protein is
RNA polymerase. This enzyme synthesizes an RNA transcript from a DNA
template.

There are five types of histones, designated as HI, H2A, H2B, H3, and H4. The
association of these histones with a specific length of DNA defines a structure
referred to as a nucleosome. Each nucleosome repeats itself, on average, about

every 200 base pairs (Figure 6-41). The region that separates each nucleosome is
called the linker. There are about 30 million nucleosomes associated with the
DNA in each nucleus.

Astring ofnucleosomes

steps

\\.

nNA

Linker

Nucleosome

Figure 6-41
Nucleosomes containing DNA and histones has the appearance of beads on a string.

The core histones (H2A, H2B, H3, and H4) bind roughly one-and-three-quarters
A replicated and
condensed chromosome

Figure 6-42
There are many different levels
of packing that are presumed to
give rise to the final, highly
condensed chromosome

turns of DNA, or about 146 base pairs. This association allows the length of the
DNA to decrease by about a factor of six.

The HI histone is though to play an important role in pulling the individual


nucleosomes together to create a 30-nm chromatin fiber. This fiber, which is about
1 mm in length, is still much larger than the diameter of the cell's nucleus.
Further packing of the DNA is needed, and this is where current speculation

begins. It could be that there is an ever increasing number of coils until a


particular higher-order structure of chromatin is established (Figure 6-42).

Copyright by The Berkeley Review

%26

The Berkeley Review


Specializing in MCAT Preparation

Biology

Structure & Function in Cells & Viruses

Eukaryotic Cells

The Cell Cycle. Mitosis, # Meiosis


The Cell Cycle

The cells of a eukaryotic organism complete a cell cycle for either reproductive,

growth, or replacement purposes (Figure 6-43). The cycle occurs in both haploid'

and diploid cells and is defined as the sum of all events that occur between the
completion of one cell division and the next. The rate of cell division varies

between different cells, and some, like striated muscle cells and nervecells, never
divide. A typical eukaryotic cell cycle lasts anywhere from 16 hours to 24 hours.
Let's briefly consider the eventsof a cellcycle.
The Phases of the Cell Cycle

The three major stages ofthe cell cycle are interphase, mitosis, and cytokinesis.
Interphase can be divided into the Gi, S, and G2 phases. Mitosis (M) is divided
into prophase, metaphase, anaphase, and telophase. Mitosis is usually followed

by cytokinesis, that portion of the cell cycle that allows for partitioning of the
contents of the cytoplasminto two new daughter cells.

Figure 6-43

A generalized eukaryotic cell cycle.

Interphase

During the first growth phase (Gi), which lasts about 10 hours, RNA and

proteins are actively being synthesized. The centriole pair separates in


preparation for the synthesis of daughter centrioles in the S phase. The cell and
its nucleus begins to increase in volume. If a cell is destined to never divide
again, it will remain arrested in G\.

Each of the 46 strands of chromatin, with the exception of the centromeres, are
replicated by the end of the synthetic phase (S). This event lasts between 6 to 8

hours. If we could visualize the chromatin at this stage, we would find that it
would be extended and loosely coiled. Even though the DNA has doubled to
form 92 sister chromatids, the chromosome number has remained the same at 46.

Daughter centrioles are synthesized at right angles to the parental centrioles. This
process is completed by mitosis.

The second growth phase (G2) lasts about 2 to 6 hours. During this phase the
chromatin is beginning to condense and become more tightly coiled. Protein
synthesis is quite active and the cell prepares for mitosis.
Mitosis

Mitosis is simply nuclear division, and involves the equal partitioning of genetic
material to two new daughter cells. The phases of mitosis are outlined below.
Prophase

At the beginning of prophase the two centriole pairs begin to move apart.
Microtubulesbegin to radiate from each pair in all directions, forming a star-like

Microtubules

structure called an aster. The region from which the microtubules extend

outward is called the centrosome, or microtubule-organizing center (MTOC).


Microtubules leaving the centrosome will eventually form the mitotic spindle, a
structure involved in the separation of the chromosomes during anaphase. By

the end of prophase the condensation of the chromatin is complete and the
chromosomes, each containing a pair of sister chromatids held together at the
centromere, can be visualized using different staining techniques. Many of the
microtubules leaving the centrosome attach at the kinetochore (Figure 6-44), a

Kinetochore

Metaphase chromosome

Figure 6-44
Microtubules attaching to the
kinetochores.

specialized area closely associated with the centromere. Other microtubules


Copyright by The Berkeley Review

27

The Berkeley Review

Specializing in MCAT Preparation

Biology

Eukaryotic Cells

Structure fie Function in Cells S? Viruses

extend from the centrosomes to the equatorial region of the nucleus without

contacting the kinetochores. Thenucleolus disappears and the nuclear membrane


begins to break down.
Metaphase

Thefullycondensed chromosomes alignthemselves along the equator of the cell,


a region also referred to as the metaphase plate. By this time the nuclear
membrane has completely disappeared.
Cytoplasm

Nucleolus

Centrioles \

Chromatin

Centromere

Nuclear

"V

membrane

Nucleus

jLate Interphase (2N)

Prophase (2N)

Metaphase (2N)

Cytokinesis is complete and two new


''
daughter cells are formed
Daughter chromosomes
are pulled to poles

Nuclear envelope
begins to form

Early Interphase (2N)

Telophase (4N)

Anaphase (4N)

Figure 6-45
Mitosis showing a nucleus containing four nonhomologous chromosomes. In this diploid cell, N
refers to the number of chromosomes.

Anaphase

The centromeres of each chromosome aligned on the metaphase plate divide and
the two sister chromatids, which can now officially be called daughter
chromosomes, move towards opposite poles. At this moment in the cell cycle we
should have a total of 92 chromosomes; 46 are moving towards one pole and 46

are moving towards the opposite pole. Movement occurs because of microtubule
depolymerization at the region of the kinetochore. Cytokinesis, the cleavage of
one cell into two daughter cells, usually begins late in anaphase.

Copyright by The Berkeley Review

28

The Berkeley Review


Specializing in NCAT Preparation

Biology

Structure &function in Cells St Viruses

Eukaryotic Cells

Telophase

With the start of telophase all daughter chromosomes have reached their
respective poles. Each chromosome begins to uncoil and extend itself. The

microtubules ofthe spindle apparatus begin to disappear as a nuclear membrane

forms around each of the two daughter nuclei. The nucleolus (or nucleoli)

reappears. The cleavage furrow ofcytokinesis continues to deepen.


Cytokinesis

Cytokinesis simply involves the cytoplasmic division of acell into two daughter
cells. The process generally begins during late anaphase and when itis complete,

signals the end ofmitosis (or meiosis). Mitosis isoutlined inFigure 6-45.

Meiosis

Humans diploid cells (2N) which are destined to become haploid gametes (N)
undergo DNA replication and then two successive nuclear divisions. This

process is called meiosis and the formation of the gametes is called


gametogenesis (discussed in the section on Reproduction and Development). In the
female the gametes are called eggs (ova) and are produced in the ovaries. The
male gametes arecalled sperm (spermatozoa) andare produced in the testes.
The two successive nuclear divisions in meiosis are called meiosis I and meiosis

II. Both meiotic divisions follow the same four phases in mitosis: prophase,
metaphase, anaphase, and telophase. In meiosis we distinguish those phases by
placing the Roman numeral I or II after the phase name (e.g., prophase I or
metaphase II). Meiosis I and meiosis IIare each preceded by an interphase. The
premeiotic interphase before meiosis I proceeds through the familiar Gi, S, and

G2 periods. During the Speriod theDNA replicates and during the G2 period the
commitment to meiosis takes place. After the first nuclear division one cell with
46 chromosomes becomes two cells, each with 23 chromosomes. This is
sometimes called a reductive division. Each of these two cells will enter into the

interphase that precedes meiosis II. During this second interphase theS period
does not existand so the DNAcannotbe replicated again. After these cells leave
the G2 period they enter into meiosis II. At the end of the second nuclear division
four cells remain, each with 23 chromosomes. What we would like to do is

understand the events of meiosis at a basic level and then, during a later
discussion, apply this understanding to the genetic significance of meiosis and
why it is important.
Meiosis I: Prophase I

In the first stage of meiosis (Figure 6-46), prophase I is quite long and
complicated compared to prophase of mitosis. Prophase I can be divided into
five stages: leptotene, zygotene, pachytene, diplotene, and diakinesis. Each of

these events contains terminology which is important to meiosis and important


to understand. Throughout prophase I we will find 46 chromosomes or 92
chromatids in the cell.

Leptotene. The replicated chromosomes have already started to

condense and now become visible. If we wanted to, we could assign 23of
those chromosomes (i.e., 46chromatids) as being maternal and 23 as being
paternal in origin.

Copyright by The Berkeley Review

29

The Berkeley Review


Specializing in MCAT Preparation

Biology
Centrioles

Eukaryotic Cells

Structure & Function in Cells St Viruses

Zygotene. Homologous chromosomes begin to pair up longitudinally


in preparation for crossing-over (genetic recombination). This is referred
to as synapsis. A specialized protein and RNA scaffold called the
synaptonemal complex (Figure 6-47a) appears between the pairing

Metaphase
plate

Nucleus

chromosomes and facilitates their union. The homologous chromosomes

which have undergone synapsis are referred to as a bivalent, and


because there arefour chromatids in a bivalent the paired homologous
chromosomes are also referred to as tetrads. How many tetrads are there

Chromatin

Premeiotic

Synapsis

Interphase
(2N)

Prophase (2N)

at this stage? Only23. How many centromeres are there in each tetrad?
Just two.How manychromosomes would we find? Based on the number
of centromeres, we would find 46. How many pairs of chromosomes are
there? We could count 23 pairs at this stage.

Centromere

Dyads are
pulled
to poles

(b)

(a)

y Dyad is pulled to
/

one pole

Synaptonemal
complex
Chiasma

Centromeres

Chromatids

Microtubules

Figure 6-47
Formation of the synaptonemal complex between a pair of homologous chromosomes.

Pachytene. The chromosomes continue to condense and become much


more distinct. Genetic recombination occurs through a process called
crossing-over.

Diplotene. As the homologous chromosomes begin to separate, the


events of crossing-over become visible at structures called chiasmata
(singular: chiasma). At each chiasma any one of the four chromatids may
be involved in a cross-over event. For each bivalent, there is an average

of about 10 cross-overs, and this leads to a very thorough mixing of the


maternal and paternal chromatids.

In the section on Reproduction and Development we learned that the


developing eggs (oocytes) in a female are arrested in diplotene of
prophase I by about the seventh month of gestation (fetal development).
Thoseeggs will not be able to continue their meiotic adventure until the
onset of puberty. Even then just one egg per month will have the
capacity to develop and be fertilized. The rest will be left in diplotene,

Cytokinesis
is complete.
Two new daughter
cells are formed

some residing in that state for more than 50 years.

Diakinesis. While the nuclear envelope begins to break down and the
nucleoli disappear, the chiasmata move along the lengths of the

Meiotic Interphase

chromatids until they reach the ends. As the homologous chromosomes

(N)

beginto separate theyappear to be joinedto each other at their ends.

Figure 6-46

Metaphase I

Meiosis I showing the different


stages that lead to the formation
of two haploid cells.

Thehomologous chromosomes ahgn on the metaphase plate.Thecentromeres of


eachpair appear to be quitedistantfrom eachother. It seems as if the only parts
of the chromosomes touching one another are the ends. Microtubules from the
spindleapparatusare attached at the centromeres and the nuclearmembrane has
disappeared.

Copyright by The Berkeley Review

30

The Berkeley Review

Specializing in MCAT Preparation

Biology

Structure &? Function in Cells St Viruses

Eukaryotic Cells

Anaphase I

As the microtubules pull the homologous chromosomes apart, the


centromeres do not divide. The pull on each chromosome is from one

direction only. Each chromosome that migrates toward a pole is still


composed of sister chromatids and this pair is referred to as a dyad
(Figure 6-47b). Cytokinesis beings.

Meiotic

Interphase

(N)

J,

Telophase I

The migrating chromosomes (dyads) complete their movement to


opposite poles and the nuclear membrane reforms. Cytokinesis is more
pronounced.
Prophase

Cytokinesis

After cytokinesis is complete we find that a diploid cell with 46


chromosomes has divided into two haploid cells, each with 23
chromosomes. Since there is a reduction in the number ofchromosomes,

(N

Chromosome

this is also called a reductive division.

Interphase II

The second (meiotic) interphase isusually brief. DNA replication does not
take place duringtheS phaseofthisinterphase.

Metaphase

Meiosis II: Prophase II

(N)

J,

^Chromosome,

The stages of meiosis II (Figure 6-48) are quite similar to the stages of
mitosis. As in prophase of mitosis, the chromosomes in prophase II of
meiosis IIbegin to condense. The microtubules of the spindle apparatus
attach to the kinetochores of each chromosome.
Metaphase II

Chromosomes line up along the metaphase plate as the haploid cells


prepare for division.

Anaphase

(2N)

Anaphase II

As the centromeres begin to divide the chromatids of each of the 23

chromosomes are pulled to opposite poles of the dividing cell.


Cytokinesis is in progress. The chromatids which separate from one
another can officially be called chromosomes. There are now 46

chromosomes in the dividing cell. An interesting technicality arises


during anaphase II. The chromatids of each chromosome that are

separating cannot be referred to as sister chromatids. Why? Think about

the difference in DNA between the replication of the chromatin during


thefirst interphase and the events of crossing-over during pachytene of
prophase I of the first meiosis. The DNA is no longer the same as there
Telophase

has been genetic recombination.

J,

(2N)

Telophase II

A nuclearmembrane forms around the decondensing chromosomes. The


cleavage furrow created by cytokinesis deepens.
Cytokinesis

Haploid Gametes (N)

After the completion of cytokinesis four haploid cells, each having 23


chromosomes, will have been derived from one diploid cell. Each of
these four cellsenters into interphase where they are arrested at the G\
stage. These cells willremainquiescent until fertilization takes place.
Copyright by The Berkeley Review

31

Figure 6-48
Meiosis II.

The Berkeley Review

Specializing in MCAT Preparation

Biology

Structure St Function in Cells & Viruses

Diploid

Haploid

Eukaryotic Cells

Asexual & Sexual Reproduction

In asexual reproduction, a new organism develops without a sexual process,


either from a single cell or a group of cells. It can occur in either haploid or

diploid organisms. The offspring produced areidentical to the parents. What is


an example ofa eukaryote that produces asexually (Figure 6-49)? Yeast.
Replication

division

Mitosis / \

, &

cleavage

Animals that reproduce sexually have two types of cells: germ cells and somatic
cells. As we have seen,haploid germ cells are produced by the process of meiosis
while diploid somatic cells areproduced by the process ofmitosis. When haploid
gametes from a male and a female fuse together, a diploid zygote is formed. This
process, termed sexual reproduction, involves the alternation of diploid and
haploid phases of an organism's life cycle. It occurs only in diploid organisms.
The offspring that result are not identical to the parents due to extensive genetic
recombination. What is an example of a eukaryote that produces sexually (Figure
6-50)? Humans.

Figure 6-49
Summary of asexual reproduction.

Replication

Diploid

Homologous
chromosomes

Zygote
(diploid)

Meiosis

Figure 6-50

Summary of sexual reproduction.

Copyright by The Berkeley Review

32

The Berkeley Review

Specializing in MCAT Preparation

Biology

Structure St Function in Cells &Viruses

Eukaryotic Cells

Hdleculw^Qrgg^^

Biomembranes Gt Membrane Transport

The major components of both eukaryotic and prokaryotic membranes are lipids,

proteins, and carbohydrates. Even though there will be differences inthe specific
molecular components of eukaryotic and prokaryotic membranes, we will find
that their structural organization is similar.In both caseswe will see membranes
that are dynamic.
Biomembranes

H2CO

Glycerophospholipids, sphingolipids, and cholesterol are the three major lipid


components of eukaryotic biological membranes. Amphiphilic molecules like the

glycerophospholipids and sphingolipids present a shape similar to that of a


rectangle. Atone endofthe rectangle the molecule ispolar (hydrophilic) while at
the other end it is nonpolar (hydrophobic). In biological membranes the two
most abundant glycerophospholipids are phosphatidylcholine (Figure 6-30) and
phosphatidylethanolamine (Figure 6-51). We can letthese amphiphilic molecules
be represented by the drawing shown in Figure 6-52. The hydrophobic tails of
the phospholipids represent the fatty acyl side chains containing the methylene
groups while the polar head is represented by the phosphate group and its
associated molecular attachments.

HC-O

O
II

H,C- 0~ P- O'
l

CU

Figure 6-51

Phosphatidylethanolamine.

Lipid Structures

The three structures that can be formed by phospholipids are micelles, lipid
bilayers, and liposomes. Micelles are spherical structures that are formed when
enough phospholipids congregate together such that the polar heads interact
with water while the hydrophobic tails exclude water (Figure 6-53a). In our
discussion on gastrointestinal physiology, we will see how bile acids form
micelles and act to solublize fats during digestion.

Polar
Head

Hydrophobic

>

Tail

Figure 6-52
General representation of a
phospholipid.

Figure 6-53
Crosssectionsof a (a) micelle,(b) lipid bilayer,and (c) liposome.

Lipid bilayers are formed when the hydrocarbon tails of two phospholipid
sheets interact with one another to exclude water (Figure 6-53b). These structures
are stabilized through hydrogen bonding and electrostatic interactions of the
head groups with water and by the hydrophobic van der Waals interactions
between the hydrocarbon side chains. The average thickness of a lipid bilayer is

about 60A (or 6 nanometers). If a lipid bilayer folds back on itself, a hollow
aqueous-filled structure will form called a liposome (Figure 6-53c).
Lipid Mobility

The individual phospholipids in the plane of a bilayer have a great deal of


mobility. Neighboring phospholipids can easily exchange places by a process
called lateral diffusion. However, movement of a phospholipid from one lipid
plane to the next, a process called transverse diffusion or flip-flop, is a very rare
event (Figure 6-54).
Copyright by The Berkeley Review

33

The Berkeley Review

Specializing in NCAT Preparation

Biology

Structure &Function in Cells &Viruses

Eukaryotic Cells

Phospholipids that have short-chain fatty acids and fatty acids with increasing

us?

sites of cis unsaturation (i.e., double bonds) tend to increase their mobility (i.e.,

fluidity) in the membrane. The greater the motion of the fatty acid side chains,
the more fluid the membrane. Addition of cholesterol to a membrane acts to

decrease fluidity. The planarsteroid ring insertsbetween neighboring fatty acid


Lateral
Diffusion

(fast)

side chains and interferes with the movement of those chains. Membrane fluidity

is also dependent on temperature. Cooler temperatures act to decrease


membrane fluidity while higher temperatures act to increase it.
Membrane Proteins

A variety of differentproteins can be associated with the lipid bilayer. However,


each of these proteins will have an operational classification. For example,
integral membrane proteins are embedded in the lipid bilayer. These proteins
can either span the bilayer, in which case they are called trans-membrane
proteins, or they can be embedded in the membrane and exposed at just the
exterior surface or interior surface of the bilayer. Integral membrane proteins are

tightlybound to the lipid bilayerby hydrophobicforces and harsh conditions are


needed to remove them. Once removed the membrane is usually disrupted.

Transverse
Diffusion

^y (very slow)

Peripheral membrane proteins are weakly attached to the surface of the lipid
bilayer, either through hydrogen bonding or electrostatic associations. Removal
of these proteins occurs under mild conditions and the membrane is usually left
intact. In Figure 6-55 we see a schematic diagram of these general classes of
proteins.
Glycoprotein
Exterior

Peripheral protein

Integral

j/f proteins N.

Figure 6-54
Diffusion of phospholipids in a
lipid bilayer.

Glycolipid

Sugar
residues

Lipid
Bilayer

Transmembrane
Interior

protein

Figure 6-55
A schematic diagram showing the relationship between integral and peripheral proteins.

Qlycolipids & Glycoproteins

If a membrane lipid is attached to a carbohydrate, it is referred to as a glycolipid.


The majority of glycolipids are found on the exterior surface of a lipid bilayer.
Carbohydrates that are attached to membrane proteins are called glycoproteins.
All sugar residues are found attached to proteins that face the exterior of the
membrane. One important class of glycolipids and glycoproteins that we will be
discussing in a later section are the human blood group antigens that make up
the A, B, and O blood groups.

Membrane Transport
The cell membrane acts as a permeability barrier that selectively allows mole
cules to enter and exit the cell. There are five processes by which this can occur:
(1) simple diffusion, (2) facilitated diffusion, (3) active transport, (4) group
Copyright by The Berkeley Review

34

The Berkeley Review

Specializing in MCAT Preparation

Biology

Structure St Function in Cells &Viruses

Eukaryotic Cells

translocation, and (5) bulk transport. Let's take abrief look at each of these types

of membrane transport.
Simple Diffusion

Another name for simple diffusion is passive diffusion. Simple diffusion


involves the spontaneous movement of solute molecules through a lipid bilayer,
from an area of high concentration to an area of low concentration, down a
concentration gradient. The relative rates of equilibration of a few molecules
across a lipid bilayer is shown in Table 6-5.

Solute/Diffusion Rate

Hydrophobic

02,N2,C02
Diffuse very fast

Facilitated Diffusion

Facilitated diffusion (or carrier-mediated diffusion) issimilar tosimple diffusion


in that it also takes place down a concentration gradient. However, in this case
solute molecules cannot simply pass through a lipid bilayer. Instead, diffusion

depends on the interaction of the solute molecules with integral membrane


proteins that are embedded in the lipid bilayer. If one type of solute molecule
passes through a transmembrane protein in one direction, the transporter is
called a uniport. If two different types of solute molecules pass through a
membrane in the same direction, the transporter is called a symport. If a
transporter allows for the passage of two different solute molecules through a
membrane in opposite directions, it is referred to as an antiport (Figure 6-56).
Protein transporters that allow for the movement of a solute across a membrane

are collectively called permeases. Symports and antiports are also referred to as
cotransporters. Bothfacilitated diffusion and simple diffusion are examples of a

Polar Uncharged (small)


Water, Urea, Ethanol
Diffuse fast

Polar Uncharged (large)


Glucose, Galactose, Fructose

Diffuse slowly
Ions

Na,K, Ca2
Diffuse very slowly
Table 6-5

Lipid solubility of select solutes.

passive transport system.

Figure 6-56
A schematic representation of three different types of transport systems.

Active Transport

If a solute is being transported against its concentration gradient, energy will be


required in one form or another. We will divide active transport into primary
active transport and secondary active transport.

In primary active transport the classic energy source is adenosine triphosphate

(ATP) and the classic example involves the Na-K pump. This pump is also
referred to as the Na-K ATPase and its primary responsibility is to set and
maintain the intracellular concentrations of Na and K. The concentrations of

the major cellular ions are quite different between the intracellular and
extracellular regions of the cell (Table 6-6). The intracellular K concentration is
Copyright by The Berkeley Review

35

The Berkeley Review


Specializing in MCAT Preparation

Biology

Structure St Function in Cells St Viruses

Eukaryotic Cells

about 140 mM while that of Na is about 5 mM to 15 mM. The extracellular K


concentration is about 5 mM while that of Na is about 145 mM.
[Outside]
(mM)

Ion

[Inside]
(mM)

Na

5-15

145

140

Ca2

1-2

3-5

the cell's energy is used to maintain these pumps.

110

Note that as three Na ions move out of the cell and two K ions move into the

cie
Table 6-6

Common ionic concentrations inside


and outside the cell.

The imbalance of these two ions across the plasma membrane of a cell is

maintained by theNa-K ATPase. This pump is an antiportthat transports two


K ions into the cellfor every three Na ions out of the cell (Figure 6-57). The
energy required forthistransport comes from the hydrolysis of ATP to ADP and
Pi (inorganic phosphate). It has been calculated that in nerve cells about 70% of

cell, a net separation of charge (an electrical potential) is established across the
plasma membrane. The inside of the cell becomes negative with respect to the
outside. The membrane potential that is established is generally in the range of
-50 millivolts (mV) to -70 mV. We will come back to this point during a later
discussion.

Another important membrane pump is the Ca2 ATPase. This protein ensures

2K

that the Ca2 concentration within the cell is always at a low level by pumping
two Ca2 ionsout ofthe cytosol for every ATP hydrolyzed.

Outside

Ionic gradients that have been established by primary active transport systems
can provide a driving force that allows for the cotransport of other molecules
against their concentration gradients. This process is called secondary active
i-K

Inside

3Na

ATPase

transport and a classic example involves the cotransport of Na with glucose


into specialized intestinal or kidney cells (Figure 6-58). The symport protein that

facilitates this process has two binding sites; one for Na and one for glucose.
Both need to be occupied in order for the translocation to occur. The transport of

Figure 6-57
The Na-K ATPase.

Na into the cell occursbecause of (1) the concentration difference of Na across


the cell's membrane (high outside and low inside) and (2) the attractive net

negative charge insidethe cell. OnceNa and glucose are brought into the cell,
Na is pumped out through the Na-K ATPase. This helps to reestablish the
electrochemical gradient across the plasma membrane. Other secondary active
Glucose

Na
Outside

transport systems cotransport Na and amino acids into the cell.


Group Translocation

Examples of group translocation can be found in certain bacteria. In this process


a sugar residue like glucose is phosphorylated as it is being transported through
the plasma membrane. This type of transport is coupled to cellular metabolism.
We will return to it when we examine the regulation and metabolism of lactose.
Na^-Glucose

Symporl

Figure 6-58
An example of secondary active
transport:

Bulk Transport

Many animal cells will show an invagination of a portion of their plasma


membrane that will eventually pinch off to form an internalized vesicle. This is
called endocytosis and the vesicles that are formed are called endosomes or
endocytotic vesicles. Within these vesicles is a portion of the extracellular
environment. If the invagination resulted in a vesicle that contains the liquid

portion of the extracellular environment, the process is referred to as pinocytosis


(or cell drinking). However, if the vesicle contains some type of particulate
matter, the process is referred to as phagocytosis (or cell eating). As these
vesicles move toward the interior of the cell some will fuse with others to form

larger structures. Material can also be released from the cell through a process
called exocytosis. Endocytosis and exocytosis are both examples of bulk
transport.

Copyright by The Berkeley Review

36

The Berkeley Review


Specializing in MCAT Preparation

Biology

Structure &Function in Cells St Viruses

Eukaryotic Cells

Nucleus, Nucleolus, Si Ribosomes


Nucleus

The most prominent structure in a eukaryotic cell is a double membrane-bound


organelle called the nucleus (Figure 6-59). The inner membrane of this nuclear

envelope surrounds the majority of the cell's genetic material (DNA). The
remaining genetic material in an animal cell can be found in the mitochondria.

The outer membrane becomes partofan extensive membranous system found in


the cytoplasm called the rough endoplasmic reticulum (RER). Between the inner
and outer membrane is the perinuclear space.
During interphase of the cell cycle the DNA is referred to as chromatin, and the
mixture of chromatin and the aqueous phase of the nucleus is referred to as the

nucleoplasm. During mitosis the extended chromatin begins to condense into


tightly coiled structures called chromosomes. We will see that within the nucleus

DNA is involved in two important events. The first is DNA replication while the
second is the transcription of DNA into messenger RNA (mRNA). Translation of
mRNA into protein (i.e., proteinsynthesis) occurs in the cytoplasm of the cell.

Figure 6-59
The nucleus.

Scattered throughout the nuclear envelope are thousands of nuclear pores with
channel diameters that range from 10 nm to 20 nm. These pores are areas where
the inner andouter nuclear membranes come together to form passage ways that
allow for the two-way flow of selected materials between the cytoplasm of the
cell and the nucleoplasm of the nucleus. Since protein synthesis cannot take place
in the nucleus, all nuclear proteins found within the nucleoplasm must be
imported from the cytosol. The exactmechanism that allows these proteins to be
transported into the nucleus us unknown, but it has been postulated that the
transport process is energy-dependent and involves protein signal sequences.
The major type of protein associated with nuclear DNA are histones. These

proteins are rich in the amino acids lysine and arginine. At physiological pH
these basic amino acids bear a positive charge on their side chain and can associate
with the negatively charged DNA through electrostatic linkages. Histones are
thought to be involved in DNA folding and the highly condensed chromosomal
structures observed during metaphase of mitosis.
Nucleolus

Within the nucleus is a highly organized region called the nucleolus. The
nucleolus, which is not a membrane-bound organelle, is centered around certain
chromosomes that contain nucleolus organizer regions and is involved in the
synthesis of ribosomal RNA (rRNA). The rRNA which is transcribed from

specific genes in the DNA of the nucleolus organizer regions associates with
ribosomal proteins in the nucleoplasm. Together the ribosomal proteins and
rRNA form two different ribosomal subunits, one small and one large, that are
then transported out of the nucleus through the nuclear pores and assembled in
the cytoplasm into complete and functional ribosomes that carry out translation.
If a cell is quite actively involved in protein synthesis, one would expect the
nucleolus to be larger than if a cell were not as actively involved in protein
synthesis.
Ribosomes

Eukaryotic ribosomes are the sites of protein synthesis. They are composed of
two subunits, each differing in size and content of RNA and protein. The size of a
Copyright by The Berkeley Review

37

The Berkeley Review


Specializing in MCAT Preparation

Biology

Structure fie Function in Cells St Viruses

Eukaryotic Cells

ribosome and each of its component parts is based on a sedimentation coefficient,

s, that is expressed in Svedberg units (S), where one S = 10~13 s. The rate at
which a molecule sediments in an ultracentrifuge tells us something about its
mass (i.e., molecular weight). The sedimentation coefficient of a complete
eukaryotic ribosome is expressed as 80S.

If we separate this ribosome into its two component parts (Figure 6-60), we find
that the large subunit sediments at 60S while the small subunit sediments at 40S.
These values are not additive. In other words, 60S + 40S does not equal 100S.
Sedimentation coefficients are not linearly related to molecular weight as they
depend on the size and the shape of the molecule.
The overall dimensions of a complete ribosome is about 20 nm by 30 nm and
contains roughly 60%rRNA and 40% protein. The small subunit is about 9 nm in
diameter and contains roughly half rRNA and half protein. The large subunit is
roughly 25 nm in diameter and contains about 65%rRNA and 35%protein.

60S Subunit

40S Subunit

Figure 6-60
The components of a
eukaryotic ribosome.

The point here is not to memorize values but rather to think about dimensions
and content. We just learned that ribosomal subunits are transported from the
nucleus to the cytoplasm where they are assembled into complete and functional
ribosomes. Based on what we have discussed we would expect the small subunit
to pass through a nuclear pore with relative ease. The large subunit seems to be
at least as large as the diameter of a nuclear pore. What would need to happen to
allow for its passage into the cytoplasm?
What are the two major types of biological molecules contained in a ribosome?
Nucleic acids and amino acids. Which nucleic acids are directly associated with
ribosomes? In the section on Expression of Genetic Information we will learn that
during protein synthesis ribosomes are associated with rRNA, mRNA and
transfer RNA (tRNA).

In any eukaryotic cell that is involved in protein synthesis there are hundreds of
thousands if not millions of ribosomes within the cell. In eukaryotic animal cells
ribosomes are not only found floating free in the cytoplasm, but they are also
found associated with a membranous structure called the endoplasmic
reticulum. What determines if a ribosome remains free in the cytoplasm or is
bound to the endoplasmic reticulum? It depends on what happens during the
initial stages of protein synthesis. All ribosomes involved in the translation of
nuclear genetic information begin polypeptide synthesis in the cytoplasm. If a
particular ribosome is destined to be bound to the endoplasmic reticulum, then
during the initial stages of protein synthesis a short signal peptide is made
which directs the ribosome to the endoplasmic reticulum. This model, called the
signal hypothesis, will be discussed shortly.
Ribosomes are also found in the matrix of the mitochondria. Ribosomes found in

the mitochondrial matrix not only differ in RNA and protein content from those
found in the cytoplasm, but they are also smaller and sediment at about 55S. This
smaller sedimentation coefficient is close to the sedimentation coefficient of

prokaryotic ribosomes, which is about 70S.

Copyright by The Berkeley Review

38

The Berkeley Review

Specializing in MCAT Preparation

Biology

Structure fie Function in Cells fit Viruses

Eukaryotic Cells

Endoplasmic Reticulum & Golgi Apparatus


Endoplasmic Reticulum
The endoplasmic reticulum (ER)is a network of membranes that is found within

the cytoplasm of most eukaryotic cells and is continuous with the outer nuclear
membrane. The space enclosed by this membranous system is referred to as the
lumen. The ER can either be smooth, in which case it is called the smooth endo

plasmic reticulum (SER), or it can be studded with ribosomes, in which case it is

called the rough endoplasmic reticulum (RER). The ER is the largest membrane
system in a eukaryotic cell.
Smooth Endoplasmic Reticulum

The SER has a membrane system that lacks ribosomes and appears more tubular
in shape. The SER is involved in the synthesis of a majority of the cell's
membrane lipids, includingthe neutral fats, phospholipids, prostaglandins, and
steroid hormones. Specific integral membrane proteins embedded in the SERact
as enzymes that help catalyze these reactions.

The SER, especially of hepatocytes (liver cells), is involved in hydroxylation


reactions that aid in the detoxification of drugs. Hydroxylating a compound
makes it more water soluble, thereby making it easier to be eliminated from the
body.

In hepatocytes the SER also plays an important role in the catabolism of liver
glycogen. Recall that glycogen is a storage polysaccharide consisting of many
glucose residues linked together in oc(l-4) and cc(l-6) linkages. If blood glucose
levels fall below normal values, a hormonally mediated set of events occur that
allow for the breakdown of glycogen and the release of glucose. One of those
events involves the enzyme glucose-6-phosphatase. As glycogen is being broken
down, a molecule of glucose-6-phosphate will eventually be produced. The
enzyme glucose-6-phosphatase is embedded in the membrane of the SER and
catalyzes the removal of the phosphate group at the C-6 position of glucosesphosphate. The desired product is glucose, as shown in (6-6). Because glucose is a
neutral compound (i.e., it no longer has any negative charges due to the

phosphate group) it can readily pass from the cytoplasm of the hepatocyte,
through a permease in the plasma membrane, and into the blood.
Glucose-6-phosphate + H20

Glucose-6-phosphatase
Glucose +

P:

(6-6)

In certain cell types the SER regulates Ca2 levels. For example, in muscle cells
the SER is referred to as the sarcoplasmicreticulum (SR). TheSRsequesters Ca2
ions and when stimulated by a nerve impulse releases those ions into the

cytoplasm of the muscle cell. The result is a contraction of actin and myosin
filaments in the muscle cell.

Rough Endoplasmic Reticulum

The RER has a membrane system that is generally flat and sheet-like. Ribosomes
that line the cytoplasmic face of the RER are bound to the membrane by their
large (60S) subunit. The ribosomes synthesize membrane and secretory proteins
that are then passed through the membrane of the RER and into the lumen where
Copyright by The Berkeley Review

39

The Berkeley Review

Specializing in MCAT Preparation

Biology

Structure &Function in Cells St Viruses

Eukaryotic Cells

post-translational modification begins. In other words, after synthesis of the


polypeptide chain by the ribosome, certain amino acids are modified by
hydroxylation and glycosylation events. After a brief period of time in the lumen
of the RER, these proteins are shuttled to the Golgi apparatus (see below) where
post-translational modification continues. These modified proteins are directed
to their final destinations by the Golgi apparatus.
Signal Hypothesis
What is the general mechanism that accounts for the passage of proteins into the
lumen of the RER? Recall that proteins encoded by nuclear genes begin the initial
stages of their synthesis in the cytoplasm of the cell. Some of these proteins have
at their N-terminus a sequence of amino acids that act as a signal sequence
which binds to a signal recognition particle (SRP) in the cytoplasm. Translation
is temporarily halted as the SRP directs the large subunit of the ribosome to a
signal sequence receptor embedded in the membrane of the RER. Once the
ribosome docks at the membrane receptor protein, polypeptide synthesis
continues and the signal sequence is inserted through the membrane of the RER.
The released SRP is free to bind to another developing signal sequence in the
cytoplasm. Meanwhile, the signal sequence that was passed into the lumen of the
RER is cleaved and degraded by the enzyme signal peptidase. Modification of
the growing polypeptide chain begins. After completion of protein synthesis, the
ribosome dissociates from the RER (Figure 6-61). Non-membrane proteins
synthesized by the ribosome are released into the lumen of the RER. These
proteins are usually destined for secretion or incorporation into lysosomes. Other
proteins may span the membrane of the RER, becoming integral membrane
proteins that are destined for the plasma membrane.
Signal
sequence

Cytosol

Figure 6-61

Synthesis of secretory proteins in the lumen of the RER. O Ribosomal subunits associate on
mRNA. @ Signal sequence leaves large subunit. SRP binds signal sequence and ribosome.
Protein synthesisstops. Ribosome-SRP complex binds SRP receptor on RER. SRP dissociates
and protein synthesis starts. Signal sequence guidesgrowing polypeptide into RER lumen. Signal
peptidase cleaves signal sequence. Polypeptide continues to grow. After protein is formed
ribosome dissociates and is recycled. Protein modification continues.

Copyright by The Berkeley Review

40

The Berkeley Review


Specializing in MCAT Preparation

Biology

Structure &Function in Cells &Viruses

Eukaryotic Cells

Golgi Apparatus
Most proteins synthesized in the RER are transported in small vesicles to the
Golgi apparatus, a complex of flattened membranous sacs called cisternae. The

cisternae of the Golgi complex (Figure 6-62) is divided into three distinct regions,
each containing an environment that has a relatively neutral pH. The cis cisterna

Cis Face

face the nucleus and endoplasmic reticulum; the trans cisterna face the plasma
membrane; the medial cisterna are located between the cis and trans cisternae.

As a protein passes from the cis cisterna to the trans cisterna, different chemical

modifications occur along the way. The principal chemical modification involves
the addition of carbohydrates (glycosylation) to the maturing protein in a
sequential fashion. Other modifications involve sulfation (addition of inorganic
sulfate) and proteolysis (reducing the size of the protein).

Once the proteins reach the trans face of the Golgi complex they appear to be
sorted and concentrated into vesicles destined for different regions of the cell.

Trans Face

Lysosomes
Specific glycosylated proteins found in the lumen of the RER are transported to
the cis Golgi where individual (mannose) sugar residues are phosphorylated. In

Figure 6-62
The Golgi complex. The cis
face is often called the forming

the lumen of the trans Golgi these phosphorylated glycoproteins bind to a


(mannose-6-phosphate) membrane-bound receptor that directs them to sorting

often called the maturing face.

face while the trans face is

vesicles. Due to a low pH within the sorting vesicles the phosphorylated


glycoproteins are released from their membrane-bound receptors. Enzymes
called phosphatases remove the phosphate groups from the sugar residues. The
glycoproteins are transported to lysosomes while the membrane-bound receptors
of the sorting vesicles are recycled back to the trans Golgi. Lysosomes number in
the hundreds in most cells.

The glycoproteins that reach the spherically-shaped, single membrane-bound


lysosomes are actually inactive precursors of lysosome enzymes. These inactive
precursors are called proenzymes and they undergo proteolytic cleavage in an
acidic environment (pH = 5) to form the mature and active enzyme.
Lysosomes are unique in their composition in that they contain more that 50

types of hydrolytic enzymes. Since all of these enzymes function at acidic pH


values, they are often referred to as acid hydrolases. The general reaction that
these enzymes catalyze is A-B + H2O > A-H + B-OH. In other words, hydrolytic
enzymes degrade polymers into individual monomeric subunits. Table 6-7 lists
some acid hydrolases common to lysosomes.
Enzyme

Substrate

Bond Hydrolyzcd

Peptides

Peptide

Glycolipids

Glycoside

Phospholipids

Carboxylic ester

DNA

Phosphoric diestcr

Phosphomonoesters

Phosphoric monoester

PROTEASES

Peptidase
GLYCOSIDASES

^hexosaminidase
LIPASES

Phospholipase
NUCLEASES

Acid deoxyribon1id ease


PHOSPHATASES

Acid phosphatase
Table 6-7

Selected lysosomal enzymes.

Copyright by The Berkeley Review

41

The Berkeley Review


Specializing in MCAT Preparation

Biology

Structure &Function in Cells &Viruses


ATP

Cytoplasm
ADF

Figure 6-63
Lysosomal proton pump.

Eukaryotic Cells

Lysosomal enzymes are inactive at neutral pH. Therefore, the environment


within the lysosome is maintained at acid pH values. This is accomplished
through the action of an ATPase-driven proton pump located in the lysosomal
membrane (Figure 6-63). ATP binds to a portion of the protein pump that
extends into the cytoplasm and as it is hydrolyzed allows two protons to enter
into the lysosome. In order to maintain electrical neutrality across the membrane,
a chloride transporter works in conjunction with the proton pump to allow
anions into the organelle.

What happens to the hydrolysis products catalyzed by the different enzymatic


reactions? They simply diffuse out of the organelle and are utilized in a variety of
metabolic processes. If this were not the case, we would expect to see lysis of this
organelle. Why? The products of hydrolysis would increase the solute
concentration in the lysosome and would lead to the osmotic entry of water. The
organelle would swell and eventually lyse. Since this is not observed, it means
that the hydrolysis products are removed from the organelle.
If the activity of a particular hydrolase enzyme is absent, then an inborn error of
metabolism can result. One of the more commonly discussed lysosomal storage
abnormalities is Tay-Sachs disease. This recessive disorder is due to the absence
of the enzyme hexosaminidase A, and results in the excessive accumulation of a
particular glycolipid in the lysosomes of ganglion cells of the cerebral cortex.
Severe mental retardation and motor impairment result. Death usually occurs by
the age of 5.
Peroxisomes

Peroxisomes are single membrane-bound organelles found within the cellular


cytoplasm that contain a variety of enzymes, the most notable being catalase. This
enzyme degrades hydrogen peroxide (H2O2) according to equation (6-7).
(6-7)

2H202

Catalase

2H20 + 02

The peroxisome generates hydrogen peroxide, a potentially damaging oxidant,


as a byproduct of both amino acid oxidation and (very) long chain fatty acid
oxidation. Peroxisomes, which number in the hundreds in most cells, are also

thought to be involved in the dissipation of heat.


Mitochondria

In animals, mitochondria (Figure 6-64) are found in all cells except erythrocytes
(red blood cells). They are one of the largest organelles in the cell, roughly 1500
nm long by 500 nm wide, and can number in the thousands, occupying as much s
25% of a cell's cytoplasm. This organelle is characterized by a double membrane
system and two distinct compartments, the matrix and intermembrane space
(Figure 6-65).
Figure 6-64
The mitochondrion.

The outer membrane is composed of roughly 50% protein and 50% lipid. The
major protein in the outer membrane is porin, a transmembrane protein which
forms channels that allows small molecules with molecular weights of less than
10,000 to freely pass.

The inner membrane is composed of roughly 75% protein and 25% lipid and is
essentially impermeable to all molecules. Many of the proteins associated with
the inner membrane are involved in an electron transport process that is coupled
Copyright by The Berkeley Review

42

The Berkeley Review


Specializing in MCAT Preparation

Biology

Structure St Function in Cells &Viruses

Eukaryotic Cells

to the generation of ATP from ADP and Pj. Other proteins act as carriers or

channels that allow for the transport of certain molecules across this highly
impermeable membrane. The number of reactions that occur along the inner
membrane is greatly increased by the presence ofnumerous convoluted foldings
called cristae. These foldings extend into the matrix of the mitochondrion.
Within the matrix some of the cell's most important biochemical reactions occur.

The majority of the cell's ATP is produced in thematrix. The Krebs (tricarboxylic
acid) cycle, J3-oxidation of fatty acids, ketone bodymetabolism, gluconeogenesis,
and some of the reactions of the urea cycle also occur in the matrix.
Cytosol
Outer
Membrane

GO

Inner

Membrane

Matrix

Figure 6-65
A section of the membrane system and compartments of the mitochondrion

Even though most of a mitochondrion's proteins are encoded by nuclear genes,


we do find that mitochondria have their own circular, double-stranded DNA

(referred to as mtDNA) and their own ribosomes (sedimenting at 55S) and


transfer RNA molecules that allow for the synthesis of certain mitochondrial
proteins. Within the small mtDNA genome are 37 genes, and almost all of the
16,569 base pairs are transcribed and translated, making the presence of introns a
remote possibility. Later, when we consider the endosymbiont hypothesis for
organelle evolution, we will see that the mitochondrial genome and ribosomes
resembles that of a prokaryote rather than that of a eukaryote.
Microtubules, Microfilaments, St Intermediate Filaments
Extending throughout the cytoplasm of a eukaryotic cell is the cytoskeleton, an
intricate protein network of microtubules, microfilaments, and intermediate
filaments that help define the shape of the cell and allow for cell movement and
cell division.

Microtubules

Microtubules are composed of 13 protofilaments arranged side-by side to create a


hollow tube-like structure that is about 25 nm in diameter. Each protofilament is

composed of alternating a-tubulin and P-tubulin proteins, linked together as


dimers and arranged in the same direction. This gives polarity to the molecule.
The growth of microtubules occurs from regions called microtubule organizing
centers (MTOCs). Three common centers are the centrosome (cell center),
Copyright by The Berkeley Review

43

The Berkeley Review

Specializing in MCAT Preparation

Biology

Structure &Function in Cells St Viruses

Eukaryotic Cells

kinetochores (spindle attachment sites on chromosomes), and centrioles. As the

microtubules being to polymerize, tubulin dimers add to the fast growing plus
(+) end and extend toward the periphery of the cell. The opposite end is called
the minus (-) end.
Microfilaments

Microfilaments are about 7 nm in diameter and consist of the protein G-actin


(where "G" refers to globular). As the- individual G-actin monomers begin to
polymerize, they form a long, double-helical, thread-like structure called F-actin
(where "F" refers to filamentous). Each G-actin monomer in the polymer is
arranged in the same direction,giving the microfilamentpolarity. Polymerization
occurs faster at the plus (+) end of the filament while depolymerization occurs
faster at the minus (-) end.
Intermediate Filaments

Intermediate filaments are about 8 to 12 nm in diameter and differ in their

composition, depending upon what tissue they are located in. For example, the
intermediate filaments in epithelial cells are composed of keratins while in
muscle cells they are composed of desmin.

Copyright by The Berkeley Review

44

The Berkeley Review


Specializing in MCAT Preparation

Biology

Structure &Function in Cells &Viruses

Prokaryotic Cells

Prokaryotic Cells
Size and Shape
Bacteria come in all shapes and size. The two most frequently encountered
bacteria are the cocci (singular, coccus) and the rods (sometimes called a

bacillus). Cocci are essentially spherical in shape while rods generally resemble
that of a tube. Bacteria that have a rigid twist to their rod-like structure are called
spirilla. If their twisted structure is more flexible, they are called spirochetes.
Bacteria not only differ in shape, but they also vary in size. They can be as small
as the largest virus or as large as an erythrocyte.
Cell Structure

Prokaryotic cells have a variety of structures important for survival. They all
have a plasma membrane which acts as a selectively permeable barrier to
metabolites entering and leaving the cell. Ribosomes are found within the
cytoplasm and are important for protein synthesis. Inclusion bodies aid in the
storage of a wide variety of substances. The genetic material is found in an
amorphous region of the cell called a nucleoid. Almost all bacteria contain a cell
wall which helps to prevent cell lysis. Some bacteria have layers external to their

cell wall called capsules; others have slime layers. Bacterialmovement is brought
about by the use of flagella (singular, flagellum).
Plasma Membrane

Prokaryotic cells are fairly consistent in their cellular structure. They all have a
plasma membrane which bounds the cytoplasm. Within this plasma membrane
are both proteins and lipids. Even though bacterial membranes do not contain
cholesterol, many do contain a sterol-like molecule that probably functions in
much the same manner as cholesterol. Invaginations of bacterial cell membranes
are called mesosomes. The function of these structures is not known.

Cytoplasm A? Ribosomes

The cytoplasm of bacterial cells lack membrane-bound organelles (e.g., nucleus,


mitochondria, Golgi, etc.) that we often associate with eukaryotic cells. Within
the cytoplasm are structures called inclusion bodies. These structures can
contain organic molecules like glycogen or inorganic molecules like phosphate
granules.

The ribosomes found in the cytoplasm of prokaryotic cells are much small than
the ribosomes of eukaryotic cells. Complete prokaryotic ribosomes are referred to
as being 70S ribosomes. These ribosomes are composed of a large 50S subunit
and a small 30S subunit.
Nucleoid of DNA

Prokaryotic cells do not have a nucleus. Instead, their circular, double-stranded


DNA chromosome is confined to a region in the cell called a nucleoid. These
regions are often associated with either the plasma membrane or mesosomes.
Some bacteria also have other chromosomes which are circular and double-

stranded called plasmids. These additional DNA molecules allow bacteria to be


resistant to some drugs, and give the bacteria a selective advantage.

Copyright by The Berkeley Review

45

The Berkeley Review


Specializing in MCAT Preparation

Biology

Structure 8t Function in Cells &Viruses

Eukaryotic Cells

The Cell Wall

Pore

Outside Cell

In 1884 Christian Gram developed a stain that allowed bacteria to be classified


into two groups. Bacteria that stained purple were called Gram positive (+)
bacteria while those that stained red or pink were called Gram negative (-)
bacteria. The difference between the two types of bacteria has to do with their
cell walls. Structures outside of the plasma membrane are usually referred to
as the cell wall or envelope.

Gram positive bacteria have a rather thick homogeneous peptidoglycan or


murein layer (20 nm to 80 nm) just outside their plasma membrane. They also
contain a large amount of teichoic acids that extend to the surface of the cell

from either the plasma membrane or the peptidoglycan layer.

Inside Cell

Figure 6-66
Gram negative bacterial membrane.

Gram negative bacteria (Figure 6-66) have a much thinner peptidoglycan layer
(about 1 nm to 3 nm) surround their plasma membrane. However, surrounding
the peptidoglycan layer is an outer membrane that contains
lipopolysaccharides and porins. Not only does the polysaccharide help to
stabilize the membrane, but it also acts as an endotoxin and provides a defense
mechanism for the cell. The porins allow for passage of materials smaller than
700 daltons. Larger materials must be transported across the outer membrane.

The peptidoglycan itself layer is composed of two acetylated amino sugars, Nacetylglucosamine (G) and N-acetylmuramic acid (M), linked together in a
P(l,4) linkage, and a small number of amino acids, including D-glutamic acid
and D-alanine. Attached to each N-acetylmuramic acid residue is a
tetrapeptide side chain.
Capsules & Slime Layers

In general, both capsules and slime layers are composed of polysaccharides


that extend from the surface of the cell. This polysaccharide layer is sometimes
called a glycocalyx. The distinguishing features between the a capsule and a
slime layer is that slime layers are easier to wash off. Capsules are much more
resistant. In fact, they help protect the bacterium from foreign invaders. We
will come back to this point when we consider the Griffith experiment
involving the bacterium Streptococcus pneumoniae.
Flagella

Many bacteria use flagella for movement. These protein structures extend from
the plasma membrane and cell wall and provide a propeller-like movement in
a counterclockwise direction that propels the bacterium through its
environment. The major protein component of the flagellar structure is
flagellin.
Why do bacteria need to move in their environment? Nutrients. Bacteria are
attracted by chemical nutrients like sugars and amino acids. Attraction towards
or repulsion away from certain chemicals is referred to as chemotaxis.

Life Cycle
The process of cell division in bacteria is rather simple. The DNA, which is
attached at some point to the inside of the plasma membrane, undergoes
replication at a site called the replication origin. Once the DNA has been
replicated a new plasma membrane and cell wall begin to enclose the newly
synthesized chromosome. The two bacterial cells will eventually separate by a
process called binary fission.
Copyright by The Berkeley Review

46

The Berkeley Review


Specializing in MCAT Preparation

Biology

Structure St Function in Cells St Viruses

Prokaryotic Cells

DNA Transfer

Genetic material can be passed from one bacterial cell to the next by the simple
process of binary fission. However, genetic information can also be transferred to
a bacterial cell by either bacterial conjugation, transformation, or transduction.

In bacterial conjugation the transfer of genetic information occurs by cell-cell


contact. Donor strains of bacteria are referred to as being F+ (male) while the
recipient bacteria are F" (female). The "F" refers to the fertility plasmid. During
conjugation the F factor replicates by a process called the rolling circle
mechanism of replication and the newly synthesized DNA is passed to the
recipient bacterium.

Transformation involves the uptake of genetic material from the surrounding


medium. Usually this new genetic material is incorporated into the host
chromosome.

Transduction is simply the transfer of bacterial genes by viruses. How do viruses


get bacterial genes in the first place? In general, when a virus infects a host cell it
incorporates its own DNA into the DNA genome of the host. When it is time for

the virus to leave, it removes its own DNA from the host genome. It is during
this process that errors can occur. Sometimes the virus removes a few host genes
as well. Once the virus has replicated and the progeny phage have been
assembled and released, they are able to infect other bacteria. Those progeny
phage will bring to those new bacteria a few genes from previously infected host
cells.

We will be returning to these different types of DNA transfer when we begin our
discussion on Genetic Information.

Copyright by The Berkeley Review

47

The Berkeley Review


Specializing in MCAT Preparation

Biology

Structure St Function in Cells &Viruses

Viruses

Viruses
Architecture & Genome
Architecture

Viruses show a wide range of biological diversity and are quite successful at
parasitizing other organisms. Quite simply, at the genetic level, viruses are
obligate intracellular parasites that cause infected host cells to produce viral gene
Lipid membrane

products rather than host gene products.

The architecture of a virus (Latin for poison) is usually based upon one of two
structural motifs (Figure 6-67); those which are isometric (usually in the form of
an icosahedron) or those which are helical. In its simplest form a virus is
composed of a nucleic acid that is surrounded by a protein coat. The protein coat
is formed from capsomers, which are building blocks composed of a specific
number of individual proteins. If there is no nucleic acid within the protein coat

Glycoprotein
(a)
/

/
/

Matrix
protein

Nucleocapsid

Capsomer

or shell, then the empty shell is referred to as a capsid. However, if there is


nucleic acid within the protein shell, the complex is called a nucleocapsid.

If the nucleocapsid of the virus is not surrounded by a lipid membrane, the virus
is referred to as a non-enveloped (naked) virus. However, if the nucleocapsid is
surrounded by a lipid membrane, the virus is referred to as an enveloped virus.
The membrane of an enveloped virus is derived from the host cell that the virus
infected and is attached to the nucleocapsid by matrix proteins. Transmembrane
proteins which have been glycosylated (i.e., glycoproteins) act as antigens and
allow the virus to communicate with its environment. These complexes are

(b)

sometimes called spikes.

Some viruses, like the bacteriophage T4 that infects the bacterium Escherichia coli
(. coli), are quite complex. This phage not only has an elongate iscoshedral head,
but it also has a helical tail section with tail fibers that can attach to the

lipopolysaccharide portion of the host's membrane.


Genome

The genetic information within the genome of a virus may be encoded in either
the language of DNA or RNA. The nucleic acid can either be linear or circular,
single-stranded or double stranded, and it can even be segmented. However, no
matter what type of nucleic acid is found in a viral genome, the translational
process involved in the expression of that genome uses mRNA as a template.
Therefore, by convention, we define that mRNA as being a positive (+) strand
nucleic acid. DNA and RNA polymers that have a base sequence identical to this
positive mRNA strand are also designated as being positive (+) strands.

Figure 6-67
Viruses with (a) enveloped
isometric and (b) non-

enveloped helical geometry.

Remember, in DNA we find that the base thymine is used instead of the base
uracil. DNA and RNA polymers with a base sequence that is complementary to the
positive mRNA strand are referred to as being negative (-) strands. In order to
synthesize a positive mRNA strand, the nucleic acid template must either be a
negative DNA or RNA strand.

The relationship between the positive mRNA strand and the different nucleic
acids allows us to organize viruses into six classes based on a scheme proposed
by David Baltimore. The Baltimore classification for the six viral classes and some
representative viral families are shown in Table 6-8.
Copyright by The Berkeley Review

49

The Berkeley Review


Specializing in MCAT Preparation

Biology

Structure &Function in Cells &Viruses

DNA Viruses
Virus

Viruses

/V::0'^ ;S^:iM^
Host

Genome

Class

Size (kb)

Envelope

Morphology

T4 phage

Bacteria

Linear (+/-) ds DNA

170

No

Xphage

Bacteria

Linear (+/-) ds DNA

46

No

Icosahedral head

Herpes simplex

Animal

Linear (+/-) ds DNA

150

Yes

Icosahedral

M13 phage
<j>X 174 phage

Bacteria

Circular (+) ss DNA

Ila

6.4

No

Helical rod

Bacteria

Circular (+) ss DNA

Ila

5.4

No

Icosahedral

Rotavirus

Mammals

Segmented (+/-) ds RNA

III

1.2-1.4

No

Icosahedral

Poliovirus

Mammals

Linear (+) ss RNA

IV

7.2 - 8.5

No

Icosahedral

Rhinovirus

Mammals

Linear (+) ss RNA

IV

7.2 - 8.5

No

Icosahedral

Rabies

Vertebrates

Linear (-) ss RNA

12

Yes

Helical rod

Influenza

Vertebrates

Segmented (-) ss RNA

13.6

Yes

Helical rod

HIV

Vertebrates

Dimer (+) ss RNA

VI

9.2

Yes

Icosahedral

Elongated icosahedral heaci


Helical tail; Tail fibers
Helical tail

RI\A Viruses

Approximate sizes in kilobases where 1 kb = 1000 nucleotides.


Table 6-8

The six viral classes as proposed by David Baltimore.

The information presented in Table 6-8 might seem a bit overwhelming. It is not
meant to be. And above all, this information is not to be memorized. Rather, it is

presented to give you a feel for the rich biological diversity in the viral world. As
we proceed in our discussions we will be considering a number of these viruses.
We will examine the coliphages T4, X, and <j>X174, as well as the human

imunodeficiency virus (HIV), a retrovirus implicated in the pathogenesis of


human acquired immunodeficiency syndrome (AIDS).

Copyright by The Berkeley Review

50

The Berkeley Review


Specializing in MCAT Preparation

Biology

Structure St Function in Cells &Viruses

Viruses

Adsorption, Penetration & Expression


Adsorption
Viral infections are generally restricted to specific organisms or types of cells,
usually referred to as the host range of the virus. Viral specificityfor a host cell

Bacteriophage

depends on the interaction of a particular virus with the host cell's surface
proteins, glycoproteins, and glycolipids.

Adsorption to
bacterial cell

For example, the bacteriophage T4 has a complex of tail fibers that recognizes a
specific lipopolysaccharide structure as well as a particularprotein porin, both of
whichare located on the cellsurface of the bacteriumE. coli. The bacteriophage X
has a single tail fiber that recognizes an integral membrane protein responsible
for the transport of the sugar maltose into E. coli. The HIV retrovirus has a
glycoprotein (gpl20) associated with its envelope that appears to bind to a
Bacterium

specific protein receptor (CD4) found on the surface of helper T cells and
monocytes, two cellular constituents of blood, and glial cells, a cellular
constituent of the central nervous system.
Penetration

Once a virus adsorbs to the surface of a suitable host, its next step is to introduce
its genome into that host, a process called penetration. The type of virus and the
attachment to the host membrane determines how this will occur.

Bacteriophages
After a bacteriophage like T4 has adsorbed to the membrane of the host cell, the

protein sheath that defines the tail begins to reorganize and penetrates the cell
wall of the bacterium until it contacts the plasma membrane. An opening is
formed in the plasma membrae and phage DNA is passed from the viral capsid,
through the tail section, and into the bacterial cytoplasm (Figure 6-68).

Capsid

The Hershey-Chase Experiment

As shown in Figure 6-68 only the viral DNA enters into the host. The protein
components of the capsid, tail, and tail fibers remains outisde the bacterial cell.
This finding was demonstrated in a classic experiment performed by Alfred

Hershey and Martha Chase in 1952. They infected E. coli bacteria with 32P and

35S labeled T2 bacteriophages. After allowing for phage adsorption the infected
bacterial cells were separated from any unattached phages by centrifugation and
then placed in a Waring blendand subjected to violent shearing forces. The fluid
was again centrifuged. Hershey and Chase found that the bacterial cell pellets

containeda high percentage of*2p while the supernatant, containing the sheared
phages, contained a high percentage of^S. Since protein contains sulfur (in the
amino acids cysteine and methionine) and DNA contains phosphorous (in the
phosphodiester bonds of the backbone), Hershey and Chase concluded that it was
the viral genome, and not the viral protein, that entered the bacterial cells and
caused infection.

Figure 6-68
Sequence of events showing (a)
adsorption of phage to bacterial
cell, (b) penetration, and (c)

Naked (Non-Enveloped) Viruses

cytoplasm.

release of viral DNA into host

Naked viruses gain access to the host's cytoplasm only by recptor-mediated


endocytosis. The receptors on the cell surface of the host are usually located near
specialized depressions called clathrin-coated pits. Clathrin, a non-glycosylated
protein, is believed to act as a scaffold that promotes vesicle formation. After the
virus has been taken into the host by endocytosis, the pH of the vesicle's interior
is lowered as hydrogen ions are transported into the vesicle by proton pumps in
the vesicle membrane. The clathrin coat depolymerizes and the smooth-surfaced
Copyright by The Berkeley Review

51

The Berkeley Review


Specializing in MCAT Preparation

Biology

Structure &Function in Cells &Viruses

Viruses

vesicle is now referred to as an endosome. As the pH of the vesicle falls, the


nucleocapsid disrupts the membrane of the endosome and is released into the
cytoplasm. The naked virus has been brought into the host cell (Figure 6-69).
Naked virus binding to membrane receptor
Plasma membrane

of host

Viralgenome

\
-V^-W-Vr
Cytoplasm
Clathrin

coated pit

Receptor-mediated
endocytosis
Endosome

Figure 6-69
Penetration of a naked virus into a host cell, (a) Binding of the virus to a membrane receptor is
followed by (b) endocytosis and (c) enclosure of the viral genome in an endosome.

Enveloped Viruses
Viruses which are enveloped can either enter their host through receptormediated endocytosis or by direct fusion with the plasma membrane. If an
enveloped virus enters through receptor-mediated endocytosis, a process similar
to the one just outlined is followed. The enveloped virus attaches to the plasma
membrane of the host and is brought into the cell by endocytosis (Figure 6-70).
After a low enough pH is reached in the endocytotic vesicle, the viral envelope
fuses with the vesicle's membrane and the nucleocapsid is released into the
cytoplasm.
Enveloped virus binding to membrane receptor
Plasma membrane

Viral genome

of host

J^J^J^J,
Cytoplasm
Clathrin

coated pit
Receptor-mediated
endocytosis
Endosome

Figure 6-70
Penetration of an enveloped virus into a host cell by receptor-mediated endocytosis. (a) Binding of
the viral envelope to a membrane receptor is followed by (b) endocytosis and (c) enclosure of the
enveloped virus in an endosome.

Copyright by The Berkeley Review

52

The Berkeley Review


Specializing in MCAT Preparation

Biology

Structure &Function in Cells St Viruses

Viruses

An alternate approach is for the envelope of the virus to directly fuse with the
plasma membrane upon initial contact with a specificreceptor (Figure 6-71). This
is how the HIV retrovirus is thought to enter its host cell. The fusion of the viral
membrane with the plasma membrane, whether it is through receptor-mediated
endocytosis or direct membrane fusion, is mediated by fusion proteins in the
viral membrane.

Enveloped virus binding to membrane receptor


Viral genome

Plasma membrane

of host

Cytoplasm

Figure 6-71
Penetration of an enveloped virus into a host cell by membrane fusion, (a) Binding of the viral
envelope to a membrane receptor is followed by (b) fusion of the viral and plasma membranes and
(c) release of the nucleocapsid into the cytoplasm.

Once the virus has entered the host cell, its genome must have access to the
cellular processes that allow for nucleic acid replication and protein synthesis.
This is brought about by a process called uncoating. Here the protein capsid is
removed from the viral genome, thereby allowing the viral nucleic acid to enter
the host cell's cytoplasm.

Expression
We have mentioned that viruses are obligate intracellular parasites that cause
infectedhost cells to produce viral gene products rather than host gene products.
How does this occur? In order to answer that question, we must first determine
if the infected cell is prokaryotic or eukaryotic; if the viral nucleic acid is DNA or
RNA; and if that nucleic acid is double stranded or single stranded.
Viral Replication & Expression in Prokaryotes

There are many examples of viral replication and expression in prokarytotic cells.
One of the most widely studied groups of viruses are those that comprise the Tseries of bacteriophages that infect E. coll Probably that most famous coliphage
from this series is T4.

The genome of the T4 phage is represented by linear, double-stranded DNA with


a rather unique feature. The nitrogenous base cytosine has been modified with a
hydroxymethyl group at the C-5 position of the ring. This modified base, 5hydroxymethylcytosine (HMC), is present only in the viral DNA. It is not found
in the bacterial DNA. The importance of this modified base to the viral genome
is that viral enzymes will easily be able to determine which is viral DNA and
which is bacterial DNA.

The DNA of T4 could contain as many as 150 genes, of which about half code for
regulatory proteins and degradative enzymes. The bacterial enzyme RNA
Copyright by The Berkeley Review

53

The Berkeley Review


Specializing in MCAT Preparation

Biology

Structure St Function in Cells St Viruses

viruses

polymerase, which is continually being modified by viral proteins, is used to


transcribe different genes in the viral genome. Viral gene products not only
disrupt the replication, transcription, and translation of the host genome, but
they also aid in the production and assembly of progeny virions.
For example, within 2 minutes after infection the immediate early viral genes are
transcribed by the bacterial enzyme RNA polymerase. Translation of this plus (+)
mRNA transcript produces viral proteins that have a variety of actions. The
polymerase enzyme is subsequently modified in order to transcribe the delayed
early, quasi-late, and late viral genes. Translation of these plus (+) mRNA
transcriptsproduces viral proteins that eventually act to digest bacterial DNA
We will be returning to viruses in our discussion on Genetic Information.

Copyright by The Berkeley Review

54

The Berkeley Review


Specializing in MCAT Preparation

Structure and Function

in Cells and Viruses


15 Passages
100 Questions

Passage Titles
I.
II.
III.
IV.

V.
VI.
VII.
VIII.
IX.

X.
XI.

Sepsis
Endoplasmic Reticulum Microsomes
Essential Amino Acids
Endotoxin

Spinal Meningitis
Nuclear Envelopes and Pores
Lipids and Membranes
Nonstandard Amino Acids
Amino Acid Characteristics

Fluorescent Recovery after Photobleaching (FRAP)


Gram-Negative and Gram-Positive Bacteria

XII.
XIII.

Raffinose
Mitosis and Meiosis

XIV.
XV.

Lipids: Four Groups


Bacteriophage Lambda

Berkeley
Specializing in MCAT Preparation

Questions
1 -5
6- 11
12- 17
18-23
24-29
30-36
37-43
44-50
51 -58
59-65
66-72
73-79
80-87
88-94
95-100

Suggestions
The passages that follow are designed to get you to think in a conceptual manner about the processes
of molecularbiology at the organismal level. If you already have a solid foundation in molecular biology,
many of the questionsyou read here will seem to be very straight forward and easy to answer. But if you
are new to the subject or if you have not had a pleasant experience with molecular biology in the past,
some of them might appear to come from the void that spreads out beyond the Oort field at the edges of
our solar system.

Pick a few passage topics at random. For these initial few passages, do not worry about the time. Just
focus on what is expected of you. First, read the passage. Second, look at any diagrams, charts, or graphs
in it. Third, read each question and the accompanying answers carefully. Fourth, answer the questions
the best you can. Check the solutions and see how you did. Whether you got the answers right or wrong,
it is important to read the explanations and see if you understand (and agree with) what is being
explained. Keep a record of your results.
After you feel comfortable with the format of those initial few passages, pick another block of
passages and try to do them in one sitting. Be aware that time is going to become important. On average,
you have about 1 minute and 15 seconds to complete a question. Be creative in how you approach this
next group. If you feel comfortable with the outline presented above, fine. If not, then try different
approaches to a passage. For example, you might feel well versed enough to read the questions first and
then try to answer some of them, without ever having read the passage. Maybe you can answer some of
the questions by just looking at the diagrams, charts, or graphs that are presented in a particular passage.
Remember, there are many effective learning styles. You need to begin to develop a format that works
best for you. Keep a record of your results.
The last block of passages might contain at least a few topics that are unfamiliar even to those who
know a good deal about molecular biology. Find a place where the level of distraction is at a minimum.
Get out your watch and time yourself on these passages, either individually or as a group. It is important
to have a feel for time, and an awareness of how much is passing as you try to answer each question.
Never let a question get you flustered. If you cannot figure out what the answer is from information
given to you in the passage, or from your own knowledge base, dump it and move on to the next
question. As you do this, make a note of that pesky question and come back to it when you have more
time. When you are finished, check your answers and make sure you understand the solutions. Be

inquisitive. If you do not know the answer to something, look it up. The solution tends to stay with you
longer that way. (For example, what is the Oort field, anyway?)
The estimated score conversions for 100 questions are shown below. At best, these are rough
approximations and should be used only to give one a feel for which ballpark they are sitting in.
Section VI

Estimated Score Conversions


Scaled Score
>13
11-12

Raw Score

80-100
70-79

9-10

60-69

7-8

50-59

5-6

40-49

<4

0-39

Biology

Sepsis

Passage I

Passage I (Questions 1-5)

The Gram stain used to differentiate between two

main types of bacteria takes advantage of the fact

Sepsis is an extremely complex disease that disrupts

that:

all physiological regulatory mechanisms involved in


maintaining homeostasis. This disorder, which in its

A.
B.

extreme form, may be characterized by organ


malfunction, is brought about by the interaction of

neither type has a membrane-bound nucleus.

Gram-positive bacteria do not have a plasma


membrane.

microbial products with host cells.

C.

bacteria differ from one another in the


structures of their cell walls.

When a host organism first succumbs to the effects of

D.

an acute bacterial infection, the inflammation response


initiated by the body of the host is meant to preserve its
health. Normally, the inflammatory response is turned off
by a variety of mechanisms. Septic shock develops when

different bacteria possess different


characteristic shapes, suchas rod and spiral.

this homeostasis is disrupted.


Septic shock has generally been considered to be

caused by Gram-negative bacteria. The major sepsisinducing factor of a Gram-negative bacterium is its
surface lipopolysaccharide (LPS). This bacterial product
stimulates the production of a number of cytokines and
other inflammatory mediators that have both pro
inflammatory and anti-inflammatory functions. Septic
shock is associated with the excess production of pro
inflammatory mediators. Among these cytokines are
tumor necrosis factor (TNF) and interleukin-1. The target
of the inflammatory activation pathway is the endothelial

2.

According to the passage, cytokine generation and


the inflammatory response are normally repressed
by a number of mechanisms. Which of the following
events would be LEAST likely to repress the
inflammatory response?

cell.

A.

Once LPSs are found in the host in quantities that

Down-regulation of cytokine receptors on host


cells.

cannot be managed by its defense mechanisms, they


recruit monocytes, macrophages, polymorphonuclear
cells, and endothelial cells to initiate an inflammatory
cascade. This response may occur locally in tissue or

B.
C.
D.

Activation of the complement system.


Endogenously generated cytokineantagonists.
Suppression of the immune system by
hormones.

throughout the bloodstream.

In blood, LPS binds to a circulating molecule known


as LPS-binding protein (LBP). Once formed, these
complexes then bind CD 14, a surface molecule on
monocytes. Binding of LPS/LBP to CD 14 leads to
monocyte activation. Endothelial cells are also stimulated

by LPSs, acting with LBP and CD 14 molecules.


However, in this case, endothelial cells lack a membranebound form of the CD 14 molecule.
3.

It is difficult to study sepsis, because most mediators


are capable of promoting the release of other
mediators and even of themselves. This selfstimulation is BEST described as:
A.
B.

Copyright by The Berkeley Review

57

an endocrine function.
a exocrine function.

C.

an autocrine function.

D.

a paracrine function.

The Berkeley Review


Specializing in MCAT Preparation

Biology
4.

Sepsis

Passage I

Septicshock is not a new disease, yet its incidence is


on the rise. Which of the following would be a likely
reason for this increase?

A.

Less invasive procedures are being used in


modem medicine.

B.

A decline in immunosuppresive therapy has

C.

The use of antibiotics has created highly

D.

There has been a decline in the population of

been noted.

resistant strains of bacteria.

persons with treatable serious diseases.

5.

In endothelial cells, LPS is also capable of

stimulation through LBP and CD14. Analysis of


endothelial cells shows a lack of a membrane-bound

form of CD14. This protein is MOST likely to be


found:

A.

bound to the extracellular face of the


membrane on the endothelial cell.

B.

bound to the cytoplasmic face of the


membrane on the endothelial cell.

C.
D.

within the cytosol of the endothelial cell.


within the blood and extracellular fluids of the
tissues.

Copyright by The Berkeley Review

58

The Berkeley Review


Specializing in MCAT Preparation

Biology

Endoplasmic Reticulum Microsomes

Passage II (Questions 6-11)

6.

Passage II

To separate smooth microsomes from rough


microsomes, centrifugation within a sucrose
gradient is used with the following result:

When tissues or cells are disrupted by


homogenization, the endoplasmic reticulum (ER) is
fragmented into many closed vesicles called microsomes.
Microsomes derived from rough endoplasmic reticulum

are called rough microsomes. The ribosomes are always

Smooth
microsomes

found on the outside surface of these microsomes.

Signal peptides were discovered in secreted proteins


that are translocated across the ER membrane prior to

Rough
microsomes

their transport to the Golgi apparatus and eventual


secretion from the cell. The signal hypothesis postulates

that a leader peptide serves as a signal peptide, directing


the eventual secreted protein to the ER membrane. Once

the peptide is inside the ER, the signal peptide is cleaved


off by a special protease.

This result can BEST be explained by the fact that:

The signal peptide is thought to be guided to the ER


membrane by two components. One is the signalrecognition particle (SRP), which binds to the signal
peptide and moves between the cytosol and the ER
membrane. The other component is the SRP receptor, an

A.

integral membrane protein found within the ER


membrane. The signal-recognition particle binds to the

rough microsomes are made of different


proteins than smooth microsomes.

B.

ribosomescontain large amounts of RNA.

C.

smooth microsomes have a much higher

D.

density than rough microsomes.


rough microsomes have much smaller

diameters

when

compared to smooth

microsomes.

signal peptide as soon as the peptide emerges from the


ribosome. A subsequent pause in protein synthesis is
observed until the ribosome carrying SRP binds to the
SRP receptor. The translational arrest is then lifted with
simultaneous translocation of the protein into the
endoplasmic reticulum.
7.

The exterior of a rough microsome is equivalent to


the:

A.

8.

cytosol.

B.

lumen of the nucleus.

C.
D.

lumen of the rough ER.


lumen of the Golgi.

The mRNA of a hormone normally produced by an


endocrine cell is translated by a free ribosome in the
presence and absence of microsomes. In this cell
free system, the hormone is found to be:
A.

the same size in the presence and absence of

B.

longer in the absence of microsomes.

C.

shorter in the absence of microsomes.

D.

longer in the presence of microsomes.

microsomes.

Copyright by The Berkeley Review

59

The Berkeley Review

Specializing in MCAT Preparation

Biology
9.

Endoplasmic Reticulum Microsomes

Passage n

It was discovered that washing microsomes with salt


removes the SRP as part of the salt extract. This
would be MOST likely to result in:

A.
B.

a quicker translational arrest.


elimination of the ability to import protein into
theER.

C.
D.

the loss of the signal peptide.


an increase in the frequency of secreted
proteins.

10.

One reason for the translational arrest observed as

the peptide emerges from the ribosome might be:

A.

to prevent the release of the newly synthesized


protein into the cytosol.

B.

an indication that the ribosome is non


functional.

C.

to ensure that all translation must occurs on the


membrane of the ER.

D.

to allow the ribosome time to proofread the

signal peptide beforeit binds to the SRP.

11.

During the course of a typical secretory pathway, a


molecule leaving the lumen of the ER will next
come in contact with the:
A.
B.
C.
D.

lateral region of the Golgi complex,


cis region of the Golgi complex,
medial region of the Golgi complex,
trans region of the Golgi complex.

Copyright by The Berkeley Review

60

The Berkeley Review

Specializing in MCAT Preparation

Biology

Essential Amino Acids

Passage EOT (Questions 12-17)

12. According to Table 1, which of the following


statements must be TRUE?

Nine amino acids are required in the diet of adult

human beings. They are histidine (His), isoleucine (He),

I.

leucine (Leu), lysine (Lys), methionine (Met), threonine


(Thr), valine (Val), tryptophan (Trp), and proline (Pro).

Allanimal proteins meetthe ideal protein


requirements.

n.
m.

Each of these amino acids has a distinctive side chain that

cannot be formed by enzymes in the human body.


In the

Passage in

Soybeans meetidealprotein requirements.


Lentils are low in sulfur-containing amino
acids.

1950s and 1960s, researchers studied

A.
B.

requirements of each amino acid and produced an ideal

profile of amino acids for human nutrition. Generally, the


flesh of other animals, birds, and fish contains all nine

amino acids in patterns that fulfill the "ideal" protein


requirement. Except for soybeans, the proteins of

I only
II only

C.

n and m only

D.

I, n, and m

vegetable sources are usually lacking one or two of the


essential amino acids. Grains and nuts are usually low in
lysine. Beans andpeas tend to be low in sulfur-containing
amino acids. The combination of plant proteins of
different classes (so that amino acid weaknesses in one

food are complemented by the other food) is called


protein complementation. An amino acid is termed

13. For which amino acid are beans characteristically

"limiting" if it does not meet or exceed the ideal protein


standard. The standard is based on the proportions of
amino acids the body requires for growth and

lower than the ideal protein standard?

maintenance.

Protein
Food
Source

Lys

Met/Cys

Thr

Trp

A.

Methionine

B.

Lysine

C.

Leucine

D.

Phenylalanine

Leu

Ideal

5.5

3.5

4.0

1.0

7.0

Egg

6.4

5.5

5.0

1.6

8.8

Milk

7.8

3.3

4.6

1.4

9.8

Beef

8.7

3.8

4.4

1.2

8.2

Chicken

8.8

4.0

4.3

1.2

7.2

Soybeans

6.9

3.5

4.3

1.5

8.4

Black
beans

6.4

2.6

3.4

1.0

8.7

Lentils

6.1

1.5

3.6

1.0

7.0

14. Whichof the following structures indicates lysine?


A.

Cornmeal

2.9

3.5

4.0

0.6

3.0

Oatmeal

3.7

3.6

3.3

1.3

7.5

Collagen

3.4

0.9

1.8

0.0

3.0

B.

Table 1. Aminoacids as percent protein in foods and compared


to the ideal reference protein.
C.

D.

II

H,N C- C-O

1
CH,
1

CH,
1

CH2
CH,

'
NH3

Copyright by The Berkeley Review

61

H
O
0
1
II
0
H,N- C- C- O

CH2

h^

H-N^N-H

The Berkeley Review

Specializing in MCAT Preparation

Biology
15.

Essential Amino Acids

Passage in

Of the choices listed in Table 1, which has only two

limiting amino acids?

16.

A.
B.

Collagen
Soybeans

C.
D.

Cornmeal
Lentils

An unknown food is examined, and each of its

amino acids is analyzed by gas chromatographywith


flame-ionization detection. The following pattern is
observed:

Lys Met/Cys
3.2

3.7

Thr

Irp

Leu

3.7

1.0

3.2

What is the MOST probable food category of this


unknown sample?
A.
B.
C.
D.

Bean

Grain
Milk
Fish

17. Which of the following amino acids is nonessential?


A.

Phenylalanine

B.

Valine
Isoleucine
Histidine

C.
D.

Copyright by The BerkeleyReview

62

The Berkeley Review

Specializing in MCAT Preparation

Biology

Endotoxin

Passage IV (Questions 18-23)

Passage IV

18. Gram-negative bacteria are distinguished from


Gram-positive bacteria by:

Many potent antibiotics exist thatare effective against


Gram-negative bacteria, but a potential clinical problem
remains with the acute inflammatory response to

A.
B.

endotoxin released from these bacteria. The endotoxin is a


lipopolysaccharide, often called LPS. The acute

an anaerobic culturing process.


an immunoassay.

C. a differential staining technique.


D. high performance liquid chromatography.

inflammatory response to endotoxin is called septic shock.


The death of Gram-negative bacteria leads to the
release of endotoxin, a lipid component of the outer cell

19.

wall of most Gram-negative bacteria. Humans respond to

A person has an infection caused by a Gramnegative bacterium. What effect would an antibiotic

endotoxin with a release of cytokines and other cellular

effective against Gram-negative bacteria have on

mediators. These mediators include tumor necrosis factor

short-term concentration of endotoxin in the blood?

a (TNF-a), interleukin-1 (IL-1), IL-6, leukotrienes, and

thromboxane A2. At high levels, these compounds may


trigger the responses of septic shock, including fever,

A.

shock, hypotension, clotting, and organ failure.

B.

Endotoxin levels would decrease rapidly as the


antibiotic bound and neutralized endotoxin.

Endotoxin levels would not change with


antibiotic treatment.

In the endotoxin molecule, a terminal disaccharide

C.

phospholipid (Lipid A) contains the key structural feature


thatis responsible for toxicity in Gram-negative bacteria.

Endotoxin levels would decrease briefly as

D.

bacteria were killed by the antibiotic.


Endotoxin levels would increase briefly as
bacteria were killed by the antibiotic.

OPCOXOHk
20.

NHR,

The following table shows data from an experiment


on mice, using the experimental agent E5531 given
simultaneously with 3 ug of E. coli LPS.

OR,

^s^^>
>r^ NHR2

0H

(HO),P(0)0D

E5531

Plasma TNF

Plasma TNF

(ug/mouse)

(ng/ml)

inhibition (%)

Mortality
(%)

0 (control)

582 20

100

547 45

80

432 40*

26

20

55

0*

OR,

E. coli Lipid A

Researchers tested a synthetic structural analog of


Lipid A, called E5531. It was designed to work as an
antagonist of endotoxin to avoid the cascade of events
leading to septic shock.

19824t

66

0*

100

71 18t

88

0*

* p < 0.01 versus control,

Which of the following statements is FALSE


regarding these data?

oA^NHR4

A.

0^V^OR7

B.

A dose of 10 ug E5531 allowed all the mice


tested to survive without complete suppression
of plasma TNF changes due to LPS.
The selected dosage of LPS given to the mice

C.

Plasma TNF responded inversely to the dose

OH

(HO),P(0)0

259 28t

t p < 0.001 versus control.

OP(0)(OH),

MeO

10

30

was lethal to about 75% of those tested.

NHR.

ofE5531.

D.
E5531

Copyright by The Berkeley Review

Plasma TNF inhibition led to increases in


survival rates for test animals.

63

The Berkeley Review


Specializing in MCAT Preparation

Biology

Passage IV

Endotoxin

23. E5531 might act as an antagonist of Lipid A in an


organism infected with Gram-negative bacteria by:

21. The following graph shows the results of a series of


experiments involving administration of E. coli
intraperitoneally to mice:

I.

competing for cellular surface receptors in the


host.

II.

directly cleaving the terminal portion of


released Lipid A to promote inactivity.
in. interfering with bacterial release of Lipid A.

10

20

30

40

A.
B.
C.

I only
I and II only
I and HI only

D.

I, n, and m

50

Time after E. coli infection (hrs)

Which conclusion is supported by the data in the


graph?
A.

The antagonist alone provided effective


treatment in promoting long-term survival.

B.

C.

This bacterial strain is resistant to antibiotics.


The combination of antibiotic and endotoxin

antagonist was most effective in promoting


D.

22.

long-term survival.
The antibiotic alone provided effective
treatment in promoting long-term survival.

What is the purpose of administering both an


antibiotic and an endotoxin antagonist during an
Qn
infection involving Gram-negative bacteria?
I.

To increase blood bacterial count while

decreasing plasmaendotoxin concentration.


II.

To

increase concentrations of cellular

mediators to attack the bacteria.

III. To destroy strains of bacteria that may be


resistant to the antibiotic that is used.

A.
B.

I only
I and II only

C.

I, II, and HI

D.

None of the above

Copyright by The Berkeley Review

64

The Berkeley Review

Specializing in MCAT Preparation

Biology

Spinal Meningitis

Passage V (Questions 24-29)

25.

Passage V

According to the passage, the epithelial layer of the


throat:

Meningococcal meningitis, commonly known as


spinal meningitis, is caused by the bacterium Neisseria

meningitidis. The disease is known for its extremely rapid


onset. A infected person first develops fever and malaise.
However, within hours, these symptoms evolve into
severe headache, neck rigidity, and an aversion to bright

A.

traps bacteria and promotes infection.

B.

serves as a barrier against infection by the

C.
D.

secretes mucus, which destroys bacteria.


becomes infected in spinal meningitis, and is

bacteria.

lights. If these symptoms go untreated, a patient canlapse

the ultimate cause of death.

into coma and a fatal form of shock.

Meningococci normally colonize the lining of the


throat. So common is the organism that at any given time,
2%-10% of healthy people carry meningococci. Spinal
meningitis begins when the organism invades the blood

26.

stream and crosses the meninges (the membranes which

cover the spinal cord and brain) into the cerebral spinal
fluid. This fluid acts as a culture medium for rapid growth

In an experiment, tissue samples are taken from both


the spinal cord and brain of a human subject with
spinal meningitis. Which of the following statements
is MOST likely to be true?

of the bacteria.

A.

In 1960, researchers from the Army Institute

investigated the importance of host defense against


meningococci. The group drew blood from thousands of
soldiers as they entered basic training and followed them
throughout this period. As men became ill, their stored
sera were tested for their ability to kill meningococci and
compared with anti-meningococcal activity from the sera
of healthy men. The group found that the disease occurred

B.
C.

In the brain and spinal cord, gray matter is


found in the exterior region, while white
matter is found in the interior region.
In the brain and spinal cord, gray matter is
found in the interior region, while white matter
is found in the exterior region.
In the brain, white matter is found in the

exterior, while in the spinal cord white matter


is found in the interior.

primarily in recruits who had low anti-meningococci


activity in sera before they became ill. Most adults have
protective antibodies against meningococci. The recruit

D.

In the brain, white matter is found in the

interior, while in the spinal cord white matter


is found in the exterior.

study indicated that individuals lacking antibodies who


were exposed to meningococci stood a high chance of
becoming ill. It appears that most people develop
protective antibodies after their first exposure to
meningococci, which keeps them from becoming ill.
27.

People with a genetic deficiency in their


complement system are unusually susceptible to
spinal meningitis. Since the disease is rare in the
United States, a substantial proportion of cases seen
here may result from this deficiency. However, the
defect is unlikely to be a significant cause of an
epidemic. Which of the following statements
supports this claim?

A.

24. Which of the following organelles would be found

B.

within Neisseria meningitidis!


A.
B.
C.

Nucleus
Mitochondria
Ribosomes

D.

Endoplasmic reticulum

Copyright by The Berkeley Review

C.
D.

65

In Nigeria and Gambia (countries with


repeated spinal meningitis epidemics), patients
with complement deficiencies are commonly
encountered during epidemics.
In Nigeria and Gambia, patients with
complement deficiencies are not commonly
encountered during epidemics.
A population's level of antibodies against
meningococci rises after an epidemic.
A population's level of antibodies against
meningococci declines before an epidemic.

The Berkeley Review


Specializing in MCAT Preparation

Biology
28.

Spinal Meningitis

Passage V

Neisseria lactamica, a relative of N. meningitidis, is

part of the normal oral flora in humans. Young


children who acquire N. lactamica throat infections
are frequently protected against meningococci
infections. The MOST likely reason for this
protection is that:
A.

toxins released from N. lactamica destroy N.


meningitidis.

B.

N.

lactamica colonizes in the throat,

eliminating any viable environment for N.


meningitidis.

C.
D.

antibodies produced against N. lactamica are


effectiveagainstN. meningitidis.
antigenic determinants on N. lactamica are
different from N. meningitidis.

29.

Accordingto the passage, neck rigidity is a symptom

of spinal meningitis. Thissymptom is a result of:


A.

endotoxins released from the bacteria, which

affect only neck muscle.

B.

rapid growth of the bacteria in the CSF,


causinginflammation in the meningeal lining.

C.

N. meningitidis over stimulating the nerves


innervating neck muscle.
N. meningitidis under stimulating the nerves
innervating neck muscle.

D.

Copyright by The BerkeleyReview

66

The Berkeley Review

Specializing in MCAT Preparation

Biology

Nuclear Envelope and Pores

Passage VI (Questions 30-36)

30.

Passage VI

According to the passage, the perinuclear space is


continuous with the:

The nuclear envelope of a cell encloses its DNA and

defines the nuclear compartment. The spherical inner

A.

nuclear lumen.

nuclear membrane contacts the chromosomes and nuclear

B.
C.

cytosol.
endoplasmic reticulum lumen.

D.

mitochondria matrix.

RNAs. The inner membrane is surrounded by the outer


nuclear membrane, which is usually studded with
ribosomes engaged in protein synthesis.

The nuclear envelope in all eukaryotes is perforated by


nuclear pores. A typical mammalian cell contains 3000

such pores. The pore is embedded in a large disk-like


structure known as the nuclear pore complex. The
complex is formed by protein granules arranged in an
octagonal arrangement. The pore is an aqueous channel in

31. According to thepassage, a nuclear envelope:

which water-soluble molecules shuttle between the

nucleus and cytosol. The effective size of the pore has


been determined by measuring the rates of diffusion of
labeled, non-nuclear components. The results show the
size of the pore to be 9 nanometers in diameter and 15
nanometers long. To offer a comparison of size, the
diameter of a ribosome is approximately 30 nanometers.

A.
B.

ensures DNA replication.


confinestranslation to the cytosol.

C.

prevents synthesis of ribosomal RNA inside

D.

breaks down during interphase of the cell

the nucleus.

cycle.

Nuclear pores can open to accommodate objects larger


than 9 nanometers in diameter. The pore is like a
diaphragm that opens to just the right size when activated
by a signal on an appropriate protein. The selectivity of
nuclear transport lies in the presence of nuclear import
signals, which are present only in nuclear proteins. The
signal, located anywhere within the protein, generally
consists of four to eight amino acids that bear positively-

32. The MOST likely reason that non-nuclear


components are used to measure the effective size of

the nuclear pore is that nuclear components:

charged side chains.

If a cell is replicating its DNA, then it requires 106


histone molecules every three minutes to package the
information molecule correctly. During mitosis, the
nuclear envelope breaks down, and the contents of the
nucleus mix with the cytosol. Upon reformation of the
nucleus, many soluble proteins are excluded, including
many former residents of the nucleus. In time, these
former residents find their way back inside the nucleus.

A.
B.

cannot be radiolabeled.
are difficult to isolate.

C.
D.

are accommodated by nuclear pores.


are smaller than the pore size.

33. In an experiment, nucleoplasmin (an abundant


nuclear protein) is proteolytically cleaved into a tail
and a head component. Both are attached to a 20-nm
piece of colloidal gold, which is easily visible under
the electron microscope, and placed in the cytosol.
In time, only the gold attached to the tail piece is
found near the nucleolus. The nuclear import signal
is found on the:

A.
B.
C.
D.

Copyright by The Berkeley Review

67

head, and contains Phe and Trp.


head, and contains Lys and Arg.
tail, and contains Phe and Trp.
tail, and contains Lys and Arg.

The Berkeley Review


Specializing in MCAT Preparation

Biology

Nuclear Envelope and Pores

Passage VI

34. In an experiment, a signal sequence is chemically


attached to random amino acids on phosphofructokinase. The result of the experiment is that the
enzyme:

A.

B.
C.

remains in the cytosol, because the location of


the nuclear import signal is unimportant.
remains in the cytosol, because the location of
the nuclear import signal is important.
is transported to the nucleus, because the
location of the nuclear import signal is
unimportant.

D.

is transported to the nucleus, because the


location of the nuclear import signal is
important.

35. According to the passage, nuclear proteins are


MOST likely to:
A.
B.

C.
D.

be resynthesized after mitosis.


be destroyed during mitosis.
lose their nuclear import signal after transport
into the perinuclear space.

keep their nuclear import signal after transport


into the perinuclear space.

36. In an experiment, small RNA molecules injected


into a frog oocyte nucleus are rapidly transported
into the cytosol. When these same RNA molecules
are injected into the cytoplasm, they remain there.
The BEST explanation for this is that receptors on
the:

A.

B.

C.

nucleoplasmic face of the pore recognize the


nuclear import signal.
cytoplasmic face of the pore recognize the
nuclear import signal.
nucleoplasmic face recognize a nuclear export
signal.

D.

cytoplasmic face recognize a nuclear export


signal.

Copyright by The Berkeley Review

68

The Berkeley Review

Specializing in MCAT Preparation

Biology

Lipids and Membranes

Passage VII (Questions 37-43)

Practice Passage VH

Experiment J

Biological membranes are composed of lipids,

In a double-labeling experiment, researchers gave a


suspension of growing bacterial cells a 1-minute pulse of

proteins, and carbohydrates. Membranes control the flow

of molecules into and out of a cell through the use of


exocytosis, endocytosis, ionophores, pumps, gates,
junctions, and other membrane-embedded proteins.

radioactively labeled 32P043". This pulse label was


designed to add radioactively labeled phosphoryl groups
only to phospholipids currently being synthesized by the
cell. After the 1-minute pulse, a reagent specific for PE,
trinitrobenzenesulfonic acid (TNBS), was immediately

Simple lipids include the terpenes and the cholesterolderived steroids. Complex lipids include fatty acids,
triglycerides, waxes, phosphoglycerides and sphingo
lipids. Lipids are soluble in nonpolar organic solvents,
such as chloroform. They are marginally soluble in water.
Because of the relative insolubility of lipids in water,
amphipathic membrane lipids like the phosphoglycerides
and sphingolipids form lipid bilayers. The hydrophilic
portion of the bilayer faces the extracellular space and
cytoplasm of the cell, while the hydrophobic portion

added to the cellular suspension.

h2cocR,
o

HC

0CR,

forms the interior of the membrane.

H2C

The transverse diffusion (flip-flop) of a lipid molecule

0 P O CHjCHjNHj

PE

across the membrane is quite rare, but the lateral diffusion

of a lipid molecule along the entire length of a bacterial


cell like Escherichia coli takes about one second.

II

The composition of lipids in biological membranes


depends primarily on the source of the membrane and, to
a small extent, the diet of the organism producing the
membrane. For example, in Escherichia coli, the weight
percent of total lipid of the phosphoglyceride phospha
tidylethanolamine (PE) is about 65%, while that of
cholesterol is 0%. Even though most membranes are
roughly one-half protein, the protein-to-lipid ratio varies,
depending on the function of the membrane.

H,C

0CR,

HC

0CR,

0,N

H2C

0 P 0 CHjCHj NH
0

Experiment 2

Membrane proteins can be integral or peripheral.


Integral proteins are tightly bound to membranes by
hydrophobic forces, and peripheral proteins are loosely
bound to membranes through electrostatic interactions
and hydrogen bonding. Unlike lipid molecules, membrane
proteins cannot undergo transverse diffusion. They can
diffuse through the membrane only in a lateral fashion.
However, both the lipids and the proteins found in a
membrane are asymmetrically distributed.

In this double-labeling experiment, the procedure is


exactly the same as in Experiment 1, except that a 3minute time period is allowed to pass between the end of

the 1-minute pulse of radioactively labeled ^PO^" and


the addition of TNBS.

37.

Asymmetric membranes are synthesized on the


framework of preexisting membranes. Enzymes in the
membrane synthesize membrane lipids while membranebound ribosomes synthesize protein to be incorporated
into the membrane itself. Selective labeling experiments,

The bacterium Escherichia coli is classified as:

A.
B.

Gram-positive, with a plasma membrane


surrounded by a thick cell wall.
Gram-negative, with a plasma membrane
surrounded by a thin cell wall and an outer

such as the one described below, confirm that membranes

membrane.

can be constructed on the framework of preexisting

C.

membranes.

D.

Gram-negative, with a plasma membrane


surrounded by a thick cell wall.
Gram-positive, with a plasma membrane
surrounded by a thin cell wall and an outer
membrane.

Copyright by The Berkeley Review

69

The Berkeley Review


Specializing in MCAT Preparation

Biology
38.

Lipids and Membranes

Complete hydrolysis of the phosphoglyceride


shown below by a eukaryotic cell yields how many

41.

The inability of an integral protein to diffuse


transversely in a membrane is MOST likely due to:

products?
A.
B.

H2C

O C- (CH2)14CHj

HC

I
I

II

H2C

C.

strong hydrophilic forces between the


membrane and the protein.
the protein's asymmetric distribution.
the protein's numerous hydrophobic amino
acid residues in the membrane's nonpolar

II

interior.

O C- (CH2)7- C= C- (CH2)7CH3

o
II

Practice Passage VH

D.

strong hydrogen-bonding forces between the


protein and the membrane.

O P O O^CHjNHj
O

A phosphatidylethanolamine
A.

B.
C.
D.

3
4
5

42.

The fluidity of a membrane is controlled by the


composition of fatty acids in it and by its
cholesterol content. A decrease in the membrane

fluidity of a bacterial cell is caused by:

39.

I.
the presence of cholesterol.
II. long-chain fatty acids with a cis double bond.
m. an increase in the length of a saturated
hydrocarbon chain on a fatty acid.

In Experiment 1 it is observed that none of the


radioactively labeled PE molecules was labeled
with TNBS. This indicates that:

I.

TNBS can readily cross the bacterial

A.

membrane.

II.

B.

synthesis of PE occurs on the extracellular

C.
D.

face of the membrane.

HI. synthesis of PE occurs on the cytoplasmic

I only
Ilonly
monly
I and in only

face of the membrane.

A.
B.
C.
D.

I only
II only
in only
I and m only
43.

40.

about 4 tons of different waxes (commonly referred


to as whale oil). One such wax is spermaceti, an
ester of a 16-carbon fatty acid and a 16-carbon
alcohol. This wax has a melting point between 42-

In Experiment 2 it is observed that roughly half of


the radioactively labeled PE was labeled with
TNBS. This indicates that:

I.

TNBS can readily cross the bacterial

n.

PE can flip-flop across the bacterial

Sperm whales (Physeter catadon) can dive to


depths of 1100 m. A mature whale's head contains

47 F. Whales use these waxes to:

membrane.
membrane.

CHj(CH2)I4 - C O- (CH2)l5CHj

ID. synthesis of PE occurs on both the


extracellular face and the cytoplasmic face of

Spermaceti

the membrane.

A.
B.
C.
D.

I only
II only
m only
II and IB only

Copyright by The Berkeley Review

70

A.
B.
C.

increase their average density.


control their buoyancy in sea water.
conserve energy at great depths.

D.

all of the above.

The Berkeley Review

Specializing in MCAT Preparation

Biology

Nonstandard Amino Acids

Passage Vm (Questions 44-50)

Passage vm

46. How many codons does it take to specify the


nonstandard amino acids?

The twenty common amino acids dictated by the


standard coding tables are not the only amino acids to

A.
B.
C.
D.

occur in biological systems. Nonstandard amino acids

occur in proteins, in biologically active peptides, such as


neurotransmitters, and in several metabolic pathways.
4-hydroxyproline and 5-hydroxylysine, components of
collagen fibers, are formed by modification of the
standard amino acids after a polypeptide is synthesized.
Collagen is the most abundant protein in vertebrates. It is
present in connective tissue, such as bone, teeth,
ligaments, tendons, and epithelial tissue.

47.

Three
Four
Five
None of the above

The enzyme ornithine transcarbamoylase transfers


the carbamoyl group of carbamoyl phosphate to
ornithine, yielding citrulline. This reaction is
diagrammed below:

Some neurotransmitters are made by modifications of


single amino acids. For example, y-amino butyric acid
(GABA) is made from glutamic acid by the enzyme
glutamic acid decarboxylase. Dopamine is made from the
amino acid tyrosine. Other nonstandard amino acids
function in urea biosynthesis (citrulline and ornithine),
amino acid metabolism (homocysteine), and methylation
reactions (S-adenosylmethionine).

2 ATP + HC03 + NH3

.2
H2N-C- O-POj

r<Carbamoyl phosphate

Kc-NH*]n
C

CH,

m
CH,

CH,

CH,

CH,

CH,

44. The lysine residue modified to make hydroxylysine


in the collagen polypeptide is modified during:

I
NH

Lz

NH,

A.
B.
C.
D.

~V

2ADP + Pj

I 2

H-C-NH,

H-C-NH,

coo

post-translation.
post-transcription.
post-replication.
post-degradation.

Ornithine

IV

coo

Citrulline

The carbamoyl moiety is indicated by:

45. Which of the following amino acids is proline?

A.
B.

I
n

c.
D.

in
IV

B.

A.

H
i ii

^ H
I II
<:
HjN-C-C-0

H,N-C-C-0

48.

CH2

CH,
I

CH,

CH,

I 2
CH2

CH,

hydroxyproline. The rats are killed, and their


collagen is separated and analyzed. In which group
is radioactive collagen synthesized?

CH,

H-C-OH

NH,

NH,

A.

B.

D.

c.
H

C.

ooc

ooc
OH

In a series of experiments, one group of rats is fed

14C-radiolabeled proline as part of their diet. A


second group of rats is fed 14C-radiolabeled

\ /***CH,

D.

c J

Both, because the proline and the


hydroxyproline are both incorporated into the
collagen polypeptide.
Only the rats receiving proline have
radioactive collagen.
Only the rats receiving hydroxyproline have
radioactive collagen.
Neither, because the proline and
hydroxyproline are digested in the small
intestine.

Copyright by The Berkeley Review

71

The Berkeley Review


Specializing in MCAT Preparation

Biology
49.

Nonstandard Amino Acids

Passage VDI

Urea is a vehicle for removing nitrogenous waste

from the body in a relatively nontoxic form. In what


organ is urea synthesized?
A.

The liver

B.
C.
D.

The kidney
The gall bladder
The pancreas

50. Prolyl hydroxylase, the enzyme that hydroxylates


proline, requires vitamin C as a coenzyme. What
would be the effects of vitamin C deficiency, called
scurvy"?

I.
II.

Easy bruising
Poor wound healing

in.

Loose teeth

A.
B.
C.

I only
I and II only
II and m only

D.

I, H, and m

Copyright by The Berkeley Review

72

The Berkeley Review

Specializing in NCAT Preparation

Biology

Amino Acid Characteristics

Passage IX (Questions 51-58)

Passage IX

51. The pH of a solution can be expressedas:

Proteins are initially synthesized from the amino (N)


terminus to the carboxyl (C) terminus during translation
as polypeptide chains from a pool of 20 common amino
acids. Each of these amino acids is coded for by at least
one codon in the genetic code. Once an amino acid is
incorporated into a growing polypeptide chain, it can
undergo post-translational modification, thus allowing for
an even richer diversity in protein structure.

A.

-ln[H+].

B.

logio [H+].
In [H+].
-logI0[H+].

C.
D.

Each of the common 20 amino acids contains an ot-

carbon atom, to which four substituents are bonded: a

52.

hydrogen atom, an amino group, a carboxyl group, and a


side chain. It is the side chain that places each amino acid
in a unique grouping. Nine amino acids have nonpolar
side chains, six have uncharged polar side chains, and five
have charged side chains.

at physiological pH would predominately take on


the characteristic of being:
A.
B.
C.
D.

The a-amino and a-carboxyl group of each amino acid


bear a dissociable hydrogen atom with a characteristic
pKa value. For leucine, an amino acid with a nonpolar
side chain, the pKa for the a-amino group is 9.6, while
the pKa for the a-carboxyl group is 2.4. The addition of
one equivalent of a base (NaOH) titrates the a-carboxyl
group and converts the fully protonated form of leucine
into the zwitterion form whose net charge is zero (Figure
1). The addition of a second equivalent of base titrates the
a-amino group.
e

COOH

h,n-c-h

+ OH

coo

HjN-C-H

_ H2N-C-H

A.
B.

Histidine
Isoleucine

C.
D.

Lysine
Arginine

H3C-C-H

CH,

+H

H3C-C-H
CH,

CH,

Figure 1

54.

If a small number of amino acids are linked together,


the molecule is called a peptide or a polypeptide.
Thyrotropin-releasing hormone (TRH) is an example of a
tripeptide containing three amino acids joined by peptide
bonds (Figure 2). Proteins generally contain 50 or more

The titration of leucine with sodium hydroxide


generates the titration curve shown below. The
isoelectric point for this amino acid is close to:

a2.0

HO

II

O
w

amino acids.

0 I

Equivalents ion

All of the following amino acids are found near the


center of a long polypeptide chain. Which amino
acid in this protein molecule is not ionizable?

CH,

+H

nonpolar.
positively charged and monovalent.
negatively charged and monovalent.
dipolar.

+ OH

CH,

CH,

HjC-C-H

53.

coo

A mixture of the common 20 amino acids in solution

i e

1-5

HO

II

II

H-N-C-C-N-C-C-N-C-C-NH,

o-^>

*^

CH

2.0

4.0

6.0

8.0

10.0 12.0

PH

,.-N^

N-

A.

II

B.
C.
D.

Thyrotropin Releasing Hormone (TRH)

Figure 2

Copyright by The Berkeley Review

73

1.0
3.0
6.0
10.0

The Berkeley Review

Specializing in MCAT Preparation

Biology

Amino Acid Characteristics

58. In Figure 2, the N-terminus nitrogen in TRH is


indicated by which Roman numeral?

55. Phenylalanine, an essential amino acid, is converted


to tyrosinein the following reaction:

J
^-

A.
B.
C.
D.

coo

HjN-C-H
~

Passage IX

I
n
ffl
IV

CH,

Phenylalanine

This reaction can be described as a:

A.
B.
C.
D.

56.

carboxylation.
hydrolysis.
hydration.
hydroxylation.

Proteins are molecules that contain 50 or more

amino acid residues, and they can act as excellent


buffers because of:

A.

their hydrogen-bonding capabilities in forming


secondary and tertiary structures.

B.

the ease with which H+ and OH" ions can be

C.

absorbed once the protein is hydrolyzed.


the ability of the terminal regions of the

D.

protein to acceptor donateH+ ions.


the wide range of pKa values found within the
protein.

57.

Complete hydrolysis of the tripeptide thyrotropinreleasing hormone (TRH) requires how many moles
of water?
A.
B.

2
3

C.

4
5

D.

Copyright by The Berkeley Review

74

The Berkeley Review

Specializing in MCAT Preparation

Biology

Fluorescent Recovery after Photobleaching (FRAP)

Passage X (Questions 59-65)

Passage x

Bleach

Experiment I
In 1970, a hybrid cell was artificially produced by the
fusion of mouse cells with human cells. Two differently
labeled antibodies were used to differentiate between

human and mouse plasma membrane proteins. At t = 0


minutes, the labeled antibodies were confined to their

respective halves. At t = 40 minutes, the two different

Time

antibodies were mixed over the entire surface of the

hybrid cell. This experiment is shown in Figure 1:

Figure 3

59. In Experiment 1, the incubation temperature was


lowered significantly. The time required for the
human and mouse plasma membrane proteins to
reach the state pictured in Figure lb would be:
A.
Figure 1

B.

C.
D.
Experiment 2

increased, due to increased fluidity of the


plasma membrane.
increased, due to decreased fluidity of the
plasma membrane.
decreased, due to increased fluidity of the
plasma membrane.
decreased, due to decreased fluidity of the
plasma membrane.

To study the lateral diffusion rates of membrane


proteins, one can use fluorescence recovery after
photobleaching (FRAP). Most commonly, fluorescent
monovalent antibodies are attached to selective membrane

proteins. The tightly bound fluorescent ligands are


bleached in a small area by a laser beam, and the time
taken for adjacent membrane proteins carrying

60.

unbleached fluorescent antibody molecules to diffuse into


the bleached area is measured. This experiment is shown
in Figure 2 and Figure 3:

The monovalent antibodies used in Experiment 2 are


fragments of antibodies that have only one antigenantibody binding site. The MOST likely reason for
using such antibodies is that:

A.

only

monovalent

antibodies

can

carry

fluorescence.

B.
C.
D.

61.

The results of Experiment 1 provide evidence for:

A.
Figure 2

Copyright by The Berkeley Review

75

only monovalent antibodies can attach to


membrane proteins.
the use of monovalent antibodies prevents
cross-linking between neighboring molecules.
a bivalent antibody would attach to more than
one antigenic determinant.

protein flip-flop.

B.

lateral diffusion.

C.

simple diffusion.

D.

rotational diffusion.

The Berkeley Review

Specializing in MCAT Preparation

Biology

Fluorescent Recovery after Photobleaching (FRAP)


65.

62. The graph shown in Figure 3 shows that the highest


level of recovery is slightlylower than the beginning

Passage x

In eukaryotic cells, the plasma membrane contains


relatively large amounts of cholesterol. At such high

level of fluorescence. This is because:

concentrations, cholesterol has the effect of:

A.

A.

not enough time was allowed during the


recovery process to obtain the full level of
fluorescence.

B.

B.

the level of fluorescence had naturally


declined.

C.

C.

the laser beam only temporarily bleaches a

populationof the fluorescentligand.


D.

the laser beam permanently bleaches a


population of fluorescent ligand.

D.

decreasing the fluidity of the membrane by


inhibiting hydrocarbon chains from
crystallizing.
decreasing the fluidity of the membrane by
promoting hydrocarbon-chain crystallization.
increasing the fluidity of the membrane by
inhibiting hydrocarbon chains from
crystallizing.
increasing the fluidity of the membrane by
promoting hydrocarbon-chain crystallization.

63. The graph below depicts the FRAP results for four
different surface glycoproteins under similar
conditions:

Time

The rate of diffusion is lowest for:

A.
B.
C.
D.

64.

Glycoprotein A.
Glycoprotein B.
Glycoprotein C.
Glycoprotein D.

In an experiment, a single glycoprotein is inserted


into a synthetic lipid bilayer. Using FRAP, it is
determined that the rate of diffusion of the

glycoprotein is significantly higher in vitro than in


vivo. The MOST likely explanation for this increase
in the rate of diffusion is that:

A.
B.

C.

antibodies of a lighter weight are used for


tagging in this synthetic system.
cross-linking occurs between antibodies in this
synthetic system.
the lack of bulky intracellular oligosaccharide
chain interaction
diffusion.

D.

increases

the

rate

of

the lack of bulky extracellular oligosaccharide


chain interaction
diffusion.

increases

Copyright by The Berkeley Review

the

rate

of

76

The Berkeley Review


Specializing in MCAT Preparation

Biology

Gram-Negative and Gram-Positive Bacteria

Passage XI (Questions 66-72)

Passage XI

Pore

Face 4

Outside cell

An important difference between prokaryotic cells and


eukaryotic animal cells is the presence (in prokaryotes) of
a protective cell wall surrounding the plasma membrane.
Bacteria can be divided into two groups: Grampositive and Gram-negative. Division into these two
groups is based on their reaction to the Gram stain, as
outlined in the procedure given below:

Face 3

M^M

G I G

Stepl
A suspension of bacterial cells is stained with
crystal violet. This procedure takes about 1 minute.

M-'-M
G

G
/

G
G
/

M,_x-M
G
G

3*

G '

-<

2 <&

Face 2

Step 2
Iodine (Io) is added to the suspension and
complexes with the crystal violet to fix the cells.
This procedure takes about 3 minutes.
Step 3
Alcohol is added to the suspension and removes
color from the cells. This procedure takes about 30

Face 1

Inside cell
Transmembrane

protein

seconds.

Figure 1

Step 4
Antibiotics like penicillin inhibit transpeptidation
reactions, while enzymes like lysozyme hydrolyze

Safranin, a red-colored counterstain, is added to the

suspension of cells. This procedure takes about 1 to

polymers of N-acetylglucosamine and N-acetylmuramic

2 minutes.

acid. Both actions weaken the cell wall and lead to

Staining techniques can generally be divided into three


types: positive staining, negative staining, and differential
staining. Positive staining employs dyes to stain cells in

eventual osmotic lysis. However, if the cell is in osmotic


equilibrium with its environment, intact spherical
structures called spheroplasts and protoplasts can form.
Spheroplasts retain some fragments of the cell wall, but
protoplasts do not.

order to increase their contrast. Negative staining allows


cells to be seen in outline against a stained background.

Differential staining can involve both positive and


negative staining.

Gram-positive and Gram-negative cells differ from


one another in the structure of their cell walls. The cell

66.

wall of Gram-positive bacteria contains peptidoglycan,


assorted polysaccharides, and teichoic acids. The cell wall
of Gram-negative bacteria contains peptidoglycan,
phospholipids, lipopolysaccharides, and assorted proteins.
In Gram-positive cells, peptidoglycan accounts for about
50 to 90 percent of the weight of the wall; in Gramnegative cells, it accounts for about 10 percent.

A.

Peptidoglycan is composed of two acetylated amino


sugars, N-acetylglucosamine (G) and N-acetylmuramic
acid (M), linked together in a (3(1,4) linkage, and a small

67.

number of amino acids, including D-glutamic acid and Dalanine (Figure 1). Attached to each N-acetylmuramic
acid residue is a tetrapeptide side chain. Transpeptidase

coccus.

B.

rod.

C.
D.

spiral.
spiral helix.

Invariant cell structures in prokaryotes include all of


the following, EXCEPT:

enzymes catalyze cross-linking between some of these


side chains and increase the stability of the peptidoglycan.

Enzymes called autolysins open up the peptidoglycan


during cellular growth.

Copyright by The Berkeley Review

Escherichia coli has the characteristic shape of a


Gram-negative bacterium, namely a:

77

A.

a cell wall.

B.

a nuclear region.

C.

ribosomes.

D.

a plasma membrane.

The Berkeley Review

Specializing in MCAT Preparation

Biology

Gram-Negative and Gram-Positive Bacteria

Passage XI

68. After being subjected to the Gram staining


procedure, Gram-positive and Gram-negative cells
show which of the following colors, respectively?
A.
B.
C.
D.

69.

Violet; colorless
Blue; violet
Colorless; red
Blue; red

The Gram staining procedure is a:

I.
positive staining technique.
II. negative staining technique.
in. differential staining technique.
A.
B.

C.
D.

70.

Based on Figure 1, the enzyme transpeptidase would


be expected to be found near which membrane face
in both Gram-positive and Gram-negative bacteria?
A.
B.

Face 1
Face 2

C.

Face 3
Face 4

D.

71.

I only
II only
m only
I and III only

The antibiotic penicillin weakens the peptidoglycan


layer in virtually all species of prokaryotes by
inhibiting transpeptidation. This can lead to lysis
and eventual death of the cell. Penicillin-induced

lysis can be prevented by:


A.
B.
C.
D.

inhibiting enzymes called autolysins.


increasing the growth rate of the cells.
inhibiting protoplast formation.
decreasing the solute concentration outside the
cell.

72.

One would expect a bacterial cell with the structures


shown in Figure 1 to be:
A.

nonresistant to penicillin, because the


antibiotic can cross the outer membrane.

B.

resistant to lysozyme, because the enzyme


cannot cross the outer membrane.

C.

resistant to both penicillin and lysozyme,


because neither can cross the outer membrane.

D.

nonresistant to lysozyme, because the enzyme


can cross the outer membrane.

Copyright by The Berkeley Review

78

The Berkeley Review

Specializing in MCAT Preparation

Biology

Raffinose

Passage XII (Questions 73-79)

73.

Mammals first arose some 200 million years ago


during the Mesozoic Era. Today, there are at least 4500
species of mammals. They can be divided into three
primary groups on the basis of their mode of
reproduction. The monotremes are the only mammals that
lay eggs. The embryo of the marsupial develops in the
uterus for a very short time after fertilization. As soon as
the immature marsupial is able to survive outside the
mother's uterus, it leaves and crawls into a marsupium
(pouch) where the mother's mammary glands are located
to finish the developmental process. Placental mammals
develop in the uterus to a much later stage of maturity
before parturition.

74.

Raffinose can be described as a:

A.
B.

reducing sugar.
nonreducing sugar.

C.

disaccharide.

D.

glycoprotein.

Fructose can be classified as:

in.
IV.

an aldose,
a ketose.
a furanose.
a pyranose.

A.
B.
C.
D.

I only
II only
II and HI only
I and IV only

I.
EL

The koala (Phascolarctos cinereus) is an arboreal

marsupial native to Australia that feeds exclusively on the


leaves of the eucalyptus tree. One of the naturally
occurring carbohydrates found in eucalyptus leaves is the
oligosaccharide raffinose (see structure below). Placental
mammals, such as humans (Homo sapiens), typically
obtain raffinose from leguminous seeds like peas and
beans.

ch2oh

Practice Passage XII

Bond
I

Raffinose

75.
OH

The bond between glucose and fructose is in the:


I.

oc(l-2) configuration.

n.

P(2-l) configuration.

HI. oc(l-5) configuration.

Fructose

IV. p(51) configuration.


For all mammals, carbohydrates provide a large

portion of the daily caloric requirement. However, before


an oligosaccharide can be utilized by an organism, it must
first be degraded by enzymes (salivary amylase and
pancreatic amylase) into smaller units called

A.
B.
C.
D.

I only
I and II only
m only
m and IV only

monosaccharides. Monosaccharides such as galactose,

glucose, and fructose are rapidly absorbed across the


mucosal cells of the wall of the duodenum and ileum of

the small intestine by a carrier-mediated transport system.


Oligosaccharides that are not hydrolyzed are not
absorbed.

76.

Raffinose is an example of an oligosaccharide that


cannot be hydrolyzed in the small intestine. However,
once this carbohydrate reaches the terminal portion of the
ileum and the beginning of the large intestine, it can be

hydrolyzed into its constituent monosaccharides by


resident bacteria. Many of these monosaccharides can be
anaerobically metabolized by the bacteria to produce
compounds like lactate, methane, carbon dioxide, and
hydrogen gas.
Copyright by The Berkeley Review

79

The bond between galactose and glucose is in the:


A.

oc(61) configuration.

B.
C.
D.

(3(6>1) configuration.
P(l>6) configuration.
a(l-6) configuration.

The Berkeley Review

Specializing in NCAT Preparation

Biology
77.

Raffinose

Practice Passage XD

Galactose and glucose are epimers of one another


about which carbon atom?

B.

C-l
C-4

C.

C-5

D.

C-6

A.

78.

If the galactose residue were to be hydrolyzedfrom


raffinose, the disaccharide left over would be:
A.

79.

maltose.

B.

lactose.

C.
D.

sucrose.
none of the above.

After ingestion of leguminous seeds, many people


experience a period of flatulence. This problem can
be traced to:

A.

C.

the loss of bacterial enzymes that degrade


oligosaccharides.
oligosaccharides that cannot be hydrolyzed by
human intestinal enzymes.
a decrease in the products of anaerobic

D.

a decrease in intestinal motility.

B.

metabolism.

Copyright by The Berkeley Review

80

The Berkeley Review

Specializing in MCAT Preparation

Biology

Mitosis and Meiosis

Passage XIII

Passage XIII (Questions 80-87)

Normal Down's

Mother

Mitosis and meiosis are essential for all eukaryotic


organisms. After fertilization, the zygote depends on
mitosis for growth and development. Mitosis is also
important for replacement of certain tissue cell types
during the life of the organism.

Father

Child

^m

MM

^^
c

6 2,

After cytoplasmic division (cytokinesis) of a cell


during mitosis, the resulting two daughter cells enter
interphase of the cell cycle. Interphase is a period of
intense metabolic activity. During interphase, each
daughter cell proceeds through an initial growth period,
followed by a period of DNA synthesis (S phase), and
then another growth period before mitosis and
cytokinesis. Mitosis occurs in somatic cells and results in
daughter cells, which are diploid.

Autoradiograph of Family 1
Normal Down's
Mother

Father

Child

Child

6
5

In contrast, meiosis occurs in the sex cells and results

in the production of gametes which are haploid. Meiosis

SB

__

consists of two cellular divisions called meiosis I and


meiosis II. Before the onset of meiosis I, chromosomes

have duplicated during the S phase. Homologous


chromosomes pair (synapse) and form a tetrad consisting
of four chromatids. During meiosis I, there is a reductive
division in which each tetrad separates to form dyads,
each containing two sister chromatids. The dyads are
separated during meiosis II into monads. Each monad
represents a single chromosome, the carrier of genetic
information in the form of genes.

Autoradiograph of Family 2
Figure 1
80.

Which of the following sequences represents the


correct order of the phases of the cell cycle?
A.
B.
C.
D.

There are many techniques in genetics that can be used


to analyze genes. In Southern blotting, DNA from a
particular chromosome is cut into fragments using a
restriction endonuclease and then separated according to

size using agarose gel electrophoresis. The DNA in the


gel is next treated with alkali and denatured to yield
single-stranded DNA. A nitrocellulose paper (used

81.

because of its ability to bind single-stranded DNA) is


placed on top of the gel, and a buffer How is used to
transfer the DNA from the gel to the nitrocellulose paper.
The exact DNA fragment pattern in the gel is maintained
on the nitrocellulose paper. Next, a probe containing a
known sequence of radioactively labeled DNA is used for
hybridization to the DNA on the nitrocellulose paper. The
position of the DNA fragments that anneal to the probes
are revealed by autoradiography. In Northern blotting,
RNA fragments can be transferred to nitrocellulose paper.
In Western blotting, proteins can be analyzed using an
SDS gel.

Mctaphasc, telophase, S, G2, prophase


Interphase, S, G\, mctaphasc, prophase
Telophase, anaphase, Gj, G2, metaphase
G2, anaphase, telophase, S, G\

Which of the following mature cell types remains in


the Gj phase of the cell cycle?
I.

Kidney epithelial cells

II.

Skeletal muscle cells

III.

Nerve cells

A.
B.

I only
I and II only
Ill only
II and III only

C.
D.

82. DNA synthesis during the S phase of the cell cycle


can be delected by using radioactively labeled DNA
precursors. The BEST technique for monitoring the
incorporation of these labeled precursors is:

The Southern blotting technique is used to analyze the


DNA of two different families. Each family has a child
with Down syndrome and a child who is unaffected. The

A.
B.
C.
D.

autoradiographs of the DNA from the mother, the father,


and the children of each family arc shown in Figure 1:

Copyright by The Berkeley Review

Child

81

Northern blotting.
Southern blotting.
Western blotting.
autoradiography.

The Berkeley Review

Specializing in MCAT Preparation

Biology
83.

Mitosis and Meiosis

86. In the autoradiographs of Family 1 and Family 2,


which parents show an abnormal division during

Which radioactively labeled precursor would be


BEST for monitoring DNA synthesis during the S
phase of the cell cycle?

A.
B.

3H-thymine
^H-cytosine

C.
D.

3H-uracil
3H-adenine

gametogenesis?

A.
B.
C.
D.

85.

Family 1 (maternal); Family 2 (maternal)


Family 1 (paternal); Family 2 (maternal)
Family 1 (maternal); Family 2 (paternal)
Family 1 (paternal); Family 2 (paternal)

87. The mother of Family 2 has a haploid chromosome


number of 23. How many chromatids are present in
metaphase of the second meiotic division?

84. Down syndrome is due to trisomy:


A.

Passage xm

13

B.

18

A.

23

C.
D.

21
22

B.
C.
D.

46
69
92

During nuclear division, chromosomes or


chromatids move to the opposite poles of the cell.
This is called disjunction. If disjunction were to fail,
two chromosomes or two chromatids would go to
one pole and none would go to the other pole. This
is called nondisjunction. Primary nondisjunction
occurs at the first meiotic division, while secondary

nondisjunction occurs at the second meiotic


division. Which of the following diagrams
represents secondary nondisjunction in
spermatogenesis?
A.

B.

C.

D.

Copyright by The Berkeley Review

82

The Berkeley Review


Specializing in MCAT Preparation

Biology

Lipids: Four Groups

Passage XIV (Questions 88-94)

Passage XIV

The most abundant steroid in animals is cholesterol

(Figure 3). Cholesterol is found in cellular membranes,


and it is also the precursor to many steroid hormones and

Lipids can be divided into four distinct groups: fats


and waxes, complex lipids, steroids, and prostaglandins
and leukotrienes. In animals, an important use for lipids is

bile salts.

the storage of energy, especially in the form of fat. If all


three alcohol groups of glycerol are esterified to the
individual carboxylic acid groups of various fatty acids, a
fat called a triglyceride is formed (Figure 1).
Triglycerides are simple lipids.

H,C

Cholesterol

Figure 3

Prostaglandins and leukotrienes are synthesized from


arachidonic acid, a C20unsaturated fatty acid (Figure 4).
A Triglyceride

Figure 1

Arachidonic acid

Figure 4
Fatty acids attached to the glycerol backbone can be
either saturated or unsaturated, and they can be of varying
lengths. The Ci8 saturated fatty acid attached to the
glycerol backbone in Figure 1 is stearic acid. Saturated
fats are solids at room temperature. Unsaturated fats are
generally liquids at room temperature.

Prostaglandins and leukotrienes mediate hormonal


action, and both cause inflammation and fever.

The membranes that surround cells and cellular

organelles contain complex lipids. These lipids can be


divided into two groups: phospholipids and glycolipids.
Phospholipids contain an alcohol, fatty acids, a phosphate
group, and either choline, ethanolamine, or serine. If the
alcohol is glycerol, the phospholipid is called a
phosphoglyceride. A common phosphoglyceride is
phosphatidyl choline (Figure 2). If the alcohol is

88.

The fat shown in Figure 1 can be described as:

A.

a carboxylic acid.

B.

an alcohol.

C.
D.

an ester.
a ketone.

sphingosine, the phospholipid is called a sphingolipid.


Glycolipids are sphingosine-based complex lipids that
contain carbohydrates like glucose and galactose.
89.

The chemical structures shown in the passage are a

triglyceride (Figure 1), phosphatidylcholine (Figure


2), cholesterol (Figure 3), and arachidonic acid
(Figure 4). The arrangement of these lipids in order
of decreasing polarity is:

Phosphatidyl choline

phosphatidyl choline > arachidonic acid > a

B.

triglyceride > cholesterol.


cholesterol > a triglyceride > arachidonic acid

C.

> phosphatidyl choline.


arachidonic acid > a triglyceride > cholesterol
> phosphatidyl choline.

D.

phosphatidyl choline > a triglyceride >


arachidonic acid > cholesterol.

Figure 2

Copyright by The Berkeley Review

A.

83

The Berkeley Review


Specializing in MCAT Preparation

Biology
90.

Lipids: Four Groups

The following series of diagrams represents the


hydrocarbon portion of individual fatty acids that
make up different triglycerides. What is the
increasing order of the melting points for the
following four triglycerides?

92.

Passage XIV

Arachidonic acid can contribute to the formation of:

I.
simple lipids.
II. complex lipids.
in. prostaglandins.
A.
B.
C.

I and n only
II only
m only

D.

I, H and m

Tryglyceride I
93.

A bacterial culture growing at 38 C is transferred to


an environment where the temperature is 25 C. All
of the following initial responses should be
observed, EXCEPT:

Tryglyceride II

A.

an increase in the synthesis of unsaturated fatty

B.

a decrease in the fluidity of the bacterial

C.

an increase in the synthesis of long chain fatty

acids.
membranes.

acids.

D.

Tryglyceride HI

an increase in the synthesis of short chain fatty


acids.

94. Hydrolysis of membrane phosphoglycerides might


yield all of the following compounds, EXCEPT:

Tryglyceride IV
a.
b.
c
d.

91.

i, iv, m, n
n, i, rv, m
m, rv, i, n
m,i,iv,n

A.
B.

glycerol.
glucose.

C.

choline.
serine.

D.

Elevated levels of cholesterol in the blood serum can

lead to plaque-like deposits on the inner walls of the


arteries, a condition called atherosclerosis. This
disease reduces the diameter of the blood vessels

and leads to adverse clinical conditions, such as a


stroke or heart attack. Atherosclerosis results from
the:

A.

increased synthesis of bile salts from


cholesterol.

B.

increased synthesis of steroid hormones from


cholesterol.

C.

removal of excess cholesterol from cellular

D.

lipid bilayers.
insolubility of cholesterol in water.

Copyright by The Berkeley Review

84

The Berkeley Review

Specializing in MCAT Preparation

Biology

Bacteriophage Lambda

Passage XV (Questions 95-100)

95.

Passage XV

The lambda phage is largely incapable of infecting


bacterial species other than E. coli. This is because:

Bacteriophage lambda is a nonenveloped DNA virus


whose host is the bacterium E. coli. The infection process
begins when the virus is adsorbed to the surface of the
host cell. Once attached, the phage injects its DNA across
the plasma membrane and into the cytoplasm of the E.

A. E. coli has a circular chromosome, while other


bacterial species do not.
B. other bacterial species have circular
chromosomes, while E. coli does not.

coli cell, where it is transformed from a linear strand to a

C. other bacterial species lack a chromosomal


integration site for lambda DNA.
D. other bacterial species lack specific cell
surface proteins that lambda normally binds.

circular one. After this point, the infection may take one
of two different pathways:
Lytic Pathway

After injection, the circularized DNA is transcribed


and translated by the host cell's DNA. The resulting
protein products make up the future viral capsid. The
lambda DNA then undergoes several rounds of
replication, after which it is packaged into the newly
assembled capsids. Multiple new viral particles are
formed, ultimately causing the lysis, or bursting, of the
host E. coli cell. During lysis, the new lambda phages are
released.

96.

Lysogenic Pathway

A strain of lambda phage exists that produces a


defective integrase enzyme, meaning it is unable
to:

After injection, the circularized DNA directs the


production of a viral protein known as integrase. This
enzyme causes the circular lambda DNA sequence to
integrate itself into the bacterial chromosome. Once
inserted, the lambda DNA (now termed a provirus) is
replicated along with the E. coli host cell. The provirus

A. avoid causing the lysis of E. coli upon


B.

infection.
to infect E. coli.

C. to replicate its DNA.


D. to enter the lytic pathway.

can remain hidden and inactive in this latent state for

multiple replications. If the E. coli host is damaged by UV


light or ionizing radiation, the latent lambda provirus can
remove itself from the bacterial chromosome and enter

the lytic pathway.

97.

Which of the following statements would represent


an evolutionary selective advantage for the lambda
virus?

I.

Upon damage to the host E. coli cell, a


lambda provirus can excise itself and enter
the lytic pathway.
II. Lambda does not always kill its host cell
immediately.
ID. Lambda DNA is almost completely resistant
to mutation.

A. I only
B. Ilonly
C. I and II only
D.

Copyright by The Berkeley Review

85

I, II, and HI

The Berkeley Review


Specializing in MCAT Preparation

Biology
98.

Bacteriophage Lambda

Passage XV

A lambda DNA strand is labeled with radioactive

32P and packaged into a viral capsid. The newly


made virus is allowed to infect an E. coli cell. The

cell does not lyse, but instead begins normal


mitosis. At this stage, radioactivity would MOST
likely be detected:
A.

in the nucleus of the E. coli host cell.

B. in the cytoplasm of the E. coli host cell.


C.
D.

99.

in the ribosomes of the E. coli host cell.


outside of the E. coli host cell.

A certain strain of E. coli produces a cytosolic


endonuclease that cleaves lambda DNA at certain
sites. This should inactivate:

A. only the lytic pathway of infection.


B. only the lysogenic pathway of infection.
C. both the lytic and lysogenic pathways of
infection.

D. neither infection pathway.

100.

Lambda phages entering the lytic or lysogenic


infection pathways do NOT differ from each other
in their:

A. mode of viral DNA injection.


B. mode of viral DNA replication.
C. immediate lethality to the host cell.

D. ability to remain latent for long periods.

Copyright by The Berkeley Review

86

The Berkeley Review

Specializing in MCAT Preparation

Biology

Structure and Function in Cells and Viruses

Section vi Answers

Passage 1(1 - 5)
1.

Sepsis

C is correct. Recall that Gram-positive bacteria do not have an outer membrane, while Gram-negative bacteria do.
In addition, the peptidoglycan layer in Gram-positive bacteria is much thicker than the peptidoglycan layer in Gramnegative bacteria. The stain does take advantage of the fact that bacteria have different cell wall structures. The
correct choice is C.

2.

B is correct. Activation of the complement system (a system of proteins found in the blood, which participates in
the immune response) brings about diffusible factors that stimulate the secretion of histamine from mast cells and
basophils. Even if we did not know the details of what complement did, we should be aware of the fact that it takes

part in the immune system response. Therefore, it is unlikely that activation of complement is part of the repression
of the inflammatory response. The rest of the answers all lead to suppression of response. The correct choice is B.
3.

C is correct. This function involves the release of chemical messengers that are secreted into the extracellular fluid.
Those messengers then act upon the cell that secreted them. This is best describing the self-stimulation addressed in
the question. Paracrine function is secretion of chemical messengers that act locally, but not on the cell that secreted
the signal. The correct choice is C.

4.

C is correct. We should realize that sepsis is the result of an acute bacterial infection. Therefore, if an individual is

infected with a bacterium that is unresponsive to antibiotics, the infection may becomes acute and lead to septicshock. Thus, we are looking for an explanation for the rise in sepsis. The creation of highly resistant bacteria
through the overuse of antibiotics certainly would be a valid explanation for the increase in sepsis. The rest of the
choices would all lead to the conclusion that sepsis should be declining: Less invasive procedures should lead to a
smaller possibility of infection. Less frequent use of immunosuppresive therapy would not increase a person's
susceptibility to infection. Finally, a decline in treatable diseases would not create the potential for resistant strains.
The correct choice is C.

5.

D is correct. We are told that this CD 14 molecule lacks a membrane-binding domain. Therefore, it does not bind to

any plasma membrane, regardless of the face. We can automatically eliminate choices A and B. To distinguish
between choices C and D, we need to think about the function of CD 14. It helps transduce a signal from the outside
world into the endothelial cell. Therefore, it is not likely to be found already inside the cytosol of the endothelial
cell. It is found instead in the blood and extracellular fluid of the tissue. The correct choice is D.

Passage II (6 - 11)
6.

Endoplasmic Reticulum Microsomes

B is correct. From the picture in the question, we learn that the rough microsomes fall below in the sucrose gradient
when compared to smooth microsomes. This must mean that they are more dense. Choice C is eliminated. The

rough microsomes have ribosomes attached all over them. They are not made from radically different proteins.
Recall that ribosomes are made up of RNA and protein. This will certainly have the effect to add more mass without
affecting the volume of the ribosome as much. The net effect is that the rough microsomes are more dense and fall
lower than the smooth microsomes in a sucrose gradient. The correct choice is B.

7.

A is correct. In the passage, it is stated that the rough microsome always has on its outside surface the ribosomes.

This is implying that the topology of the ER is conserved. In other words, the outside of the rough ER in an intact
cell is the cytosol. The ribosomes that are attached to the ER are attached on the cytoplasmic face of the ER
membrane. This case is no different for the microsomes. They have not been turned inside out. Therefore, it
becomes clear that the exterior of a rough microsome is equivalent to the cytosol. Consider the other choices. The
lumen of the nucleus is not correct. The lumen of the nucleus is enclosed by the nuclear membrane, and it is not
continuous with any other cellular fluid. The lumen of the ER is of course equivalent to the inside of the microsome.
That is why we take advantage of these mini-ER systems for experiments. Finally, it is evident that the lumen of the
Golgi has little to do with the exterior of the microsome. Choice D can be eliminated. The correct choice is A.

8.

B is correct. This problem requires that we think about the processing that occurs on a protein that is being
transported into the ER. It is stated in the passage that the signal peptide that is responsible for bringing that protein
into the ER is cleaved once in the ER. Therefore, proteins that make their way into the ER will be shorter than they
would have been if they had remained in the cytoplasm. Since microsomes are mini-ERs, the same holds. In the

absence of the microsome, the protein should be longer, and indeed this is the case. Again, the signal peptide is not
cleaved if the protein does not enter the ER. Based on this information, all other answers can be dismissed. The
correct choice is B.

Copyright by The Berkeley Review

87

The Berkeley Review

Specializing in MCAT Preparation

Biology
9.

Structure and Function in Cells and Viruses

Section vi Answers

B is correct. We are told from the passage that the SRP binds to the signal peptide and is ultimately responsible for
bringing the protein to the ER membrane. Let us think about what would happen if the SRP was no longer there.
Would we see an increase in the frequency of the secreted proteins? No, we would see a decrease since the proteins
that are being translated cannot make their way to the ER. This eliminates choice D. Would we see a loss in the

signal peptide? No, nothing would happen to this sequence on the protein except that it would no longer bind to the
SRP because there is not one available. Therefore, we can choice C. Would we see a quicker translational arrest?

Assuming there is no SRP around, we would not see any translational arrest because binding of the SRPto the signal

peptide is responsible for the arrest. We can eliminate choice A. We would see the elimination of the ability to
importproteins into the ER. The correct choice is B.

10.

A is correct. By having a translational arrest, the cell is ensuring that a protein that is needed on the outside of the
cell (orsome organelle inside the cell) will not be released into the cytosol. If the protein is no longer being made on
the ribosome, this is a solid means of insurance. Consider the other choices. There is no evidence that the ribosome

is non-functional. It will translate the protein just fine, but its function is regulated by the SRP. Therefore, we can
eliminate choice B. The notion that all translation must occur on the ER membrane is ridiculous. This is obviously

not the case for all of those proteins that carry out their function within the cytosol. We can eliminate choice C.
There is no evidence in the passage or our body of knowledge that such a translational arrest is a means of

proofreading. Therefore, the most likely reason lies in the idea of ensuring proteins that are destined to be secreted
are not inappropriately released into the cytosol. The correct choice is A.

11.

B is correct. The Golgi stack has two distinct faces: a cis face (or entry face) and a trans face (or exit face). The cis
face is closely associated with the transitional elements of the ER while the trans face is distended into a tubular
reticulum known as the trans Golgi network. Proteins and lipids enter a Golgi stack in small vesicles from the ER on
the cis side and exit for various destinations in vesicles from the trans side of the Golgi. Based on this information,

one can easily eliminate choice D. The medial region of the Golgi is the region in between the cis and the trans face.
Therefore, one can easily eliminate choice C. There is no lateral region of the Golgi complex, so this choice can
easily be eliminated. The correct choice is B.
Essential Amino Acids

Passage HI (12-17)
12.

C is correct. Collagen is an animal connective tissue protein. It is at the bottom of the list in the table. It does not
meet the ideal protein requirements. Statement I is not true. Soybeans do meet the ideal protein requirements, as
seen in the table. Statement II is true. Lentils do not meet the ideal protein requirements for sulfur-containing amino
acids. Statement III is true. The correct choice is C.

13.

A is correct. The passage indicates that beans are usually low in the sulfur-containing amino acids. Methionine is an
amino acid that contains sulfur. Choice A is correct. Neither choices B, C, nor D contain sulfur. The correct choice
is A.

14.

C is correct. Choice A is phenylalanine. Choice B is tryptophan. Choice C is lysine. Choice D is histidine. The
correct choice is C.

15.

D is correct. We can read this answer from the table. Compare the values for each protein food to the ideal standard

given at the top. Collagen is deficient in all five of the listed amino acids. Choice A is incorrect. Soybeans have
adequate levels of the listed amino acids. Choice B is incorrect. Cornmeal has 3 limiting amino acids. Choice C is
incorrect. Lentils have 2 limiting amino acids (count the sulfur-containing amino acids as one choice, since only
methionine is actually needed by the body). The correct choice is D.

16.

B is correct. First, compare the amino acid pattern to the ideal. This is not an ideal protein, so it is probably not fish,
which is a "complete" protein. Choice D is incorrect. Beans, as we learned in the passage, are usually low in sulfurcontaining amino acids. The unknown is not deficient in these, so choice A is incorrect. Milk also meets the ideal

protein standards, not the pattern shown. Choice C is incorrect. Grains are low in lysine, as is the unknown. The
other values are similar to cornmeal and oatmeal, both of which are grains. The correct choice is B.

17.

A is correct. The opposite of essential is nonessential. In the passage, valine, isoleucine, and histidine are indicated
as essential. This means phenylalanine is nonessential. The correct choice is A.

Copyright by The Berkeley Review

88

The Berkeley Review

Specializing in MCAT Preparation

Biology

Structure and Function in Cells and Viruses

Passage IV (18-23)

18.

section vi Answers
Endotoxin

C is correct. The Gram stain is commonly used in microbiology. It involves staining samples with a purple dye,
such as crystal violet, using iodine as a mordant, and then staining with a pink dye, safranin. The differences in cell
wall structure allow characterization of Gram-positive and Gram-negative bacteria. This gives the clinician a broad
approach for treatment. Not all bacteria would grow under anaerobic conditions. Choice A is incorrect. An

immunoassay binds specific molecules, but would not be used to classify Gram-negative or positive. Choice B is

incorrect. HPLC is used to separate compounds based on their molecular characteristics. Bacteria are not separated
this way. Choice D is incorrect. The correct choice is C.

19.

D is correct. The death of Gram-negative bacteria leads to the release of endotoxin from the bacterial cell wall. As

the antibiotic kills bacteria, they would release more endotoxin. At some point, endotoxin levels would rise and then
fall if the antibiotic were able to control the bacterial population. Choice C is incorrect. Choice B is incorrect. The
antibiotic does not bind or neutralize the endotoxin. Choice A is incorrect. Choice D is the correct choice. The
correct choice is D.

20.

B is correct. At the dosage of 10 ug, the mortality was 0%, so all the mice in this group survived, even though TNF
was inhibited only 55%. Choice A is true. As E5531 levels were increased by group, the plasma TNFconcentration
decreased. Choice C is true. Inhibiting TNF increased survival rates. Choice D is true. The dose of LPS given killed
all the control group. The dose was selected to do this, not to kill only 75% of the control group. Choice B is false,
and is the answer we want. The correct choice is B.

21.

C is correct. Neither the antibiotic nor the antagonist alone provided any benefit in terms of long term survival.
Choices A and D are incorrect. In the combined treatment, the antibiotic seemed to work together with E553I. We
cannot make a judgment on the resistance of the bacterial strain based on this data. Choice B is incorrect. The

combination of E5531 and the antibiotic provided the only beneficial effects in terms of survival 50 hours later.
Choice C is correct. The correct choice is C.

22.

D is correct. The purpose of the antibiotic is to decrease the number of bacteria. The purpose of the endotoxin
antagonist is to decrease unfavorable changes caused by the response of cellular mediators to the endotoxin
produced. Blood bacterial count and endotoxin concentration should both decrease with this regime. Choice I is

incorrect. The goal is to decrease cellular mediators such as TNF. Choice II is incorrect. This strategy will not work
if the bacteria are resistant to the antibiotic. The endotoxin antagonist has no direct effects on bacterial number or
reproduction. Choice III is incorrect. Choice D is the correct choice, none of the above. The correct choice is D.

23.

A is correct. The similarity in structure between Lipid A and E5531 should suggest that E5531 can bind to the same
cellular surface receptors in the host. It is probably a competitor that does not elicit the same cellular responses.
Statement I is correct. Usually enzymes (proteins) are responsible for cleavage, not lipopolysaccharides. Statement
II is incorrect. The release of Lipid A as part of the LPS is usually as a response to cell damage or death (see
passage). E5531 combats the effects of release of LPS, not its release directly. Statement III is incorrect. The
correct choice is A.

Passage V (24 - 29)


24.

Spinal Meningitis

C is correct. The question is asking us to think about bacterial structure and organization with reference to
organelles. The cytoplasm of bacteria, unlike eukaryotes, contains no internal compartments and no organelles
except ribosomes. The correct choice is C.

25.

B is correct. The answer can be derived from a careful read of the passage. We are told that the bacteria that causes
this disease normally colonizes on the human throat. However, the disease begins only when the bacteria invades the
blood stream and eventually reaches the cerebral spinal fluid. This statement implies that the bacteria must cross
over the lining of the throat to reach the blood stream. Therefore, the layer of epithelial cells that lines that throat
normally serves as a barrier of penetration to the blood stream. Once that barrier is broken, then the disease takes
hold. The correct choice is B.

26.

D is correct. The concentration of myelin sheaths make tracts of nerve axons white. The rest of the CNS appears
gray. Within the brain, white matter is located in the inner region, whereas in the spinal cord, white matter is located
on the exterior. This answer cannot be derived from the passage (meaning the question tests your personally
acquired knowledge), and in fact is true of all brain and spinal cord tissue, regardless of whether a person has spinal
meningitis. The correct choice is D.

Copyright by The Berkeley Review

89

The Berkeley Review

Specializing in MCAT Preparation

Biology
27.

Structure and Function in Cells and Viruses

Section vi Answers

B is correct. We are told from the question that this lack of complement is unlikely to be a cause of an epidemic.
We are now looking for a statement that will back this claim up. The second answer claims that in countries where

there is a high frequency of epidemics, persons with this lack of complement are not commonly found during the

epidemic. This certainly qualifies as evidence for the claim that the genetics is not a significant cause of an
epidemic. Statement Ais completely the opposite, so it can easily be eliminated. The statements about the rise and
fall of antibody levels are true, and research into the cause of these fluctuations continues. However, these
statements do not offer us support for the claim made in the question. Again, our best evidence comes from the fact
that we do not see a lot of persons afflicted with this lack of complement during spinal meningitis outbreaks in
countries where epidemics are common. The correct choice is B.

28.

C is correct. N. lactamica is a relative of N. meningitidis. Therefore, it is not highly unlikely that they may be
similar in structure in some ways. Children who are infected by N. lactamica produce antibodies against this
oreanism. We are told that children who have had this infection are shown to be protected against spinal meningitis.

Therefore, it is logical to claim that those antibodies against N. lactamica are effective against N. meningitidis. Since

they are relatives ofeach other, they very well may have similar antigenic determinants (eliminate choice D) and

therefore antibodies against one may be effective against the other. There simply is no evidence in the passage or

logic to lead us to believe that choices Aand Bare correct. The correct choice isC.
29.

B is correct. One should realize from that passage that the cerebral spinal fluid is a culture medium for the rapid

growth ofthe bacteria. This increase in the population ofthe bacteria causes inflammation ofthe meningeal lining.

The swelling of the membrane is what causes the stiff neck. It also causes fever, headache, and potentially coma.
While this answer cannot be directly lifted from the passage, one should be able to relate the growth of the bacteria
to swelling of tissue. Furthermore, one may take a process of elimination to this answer.
For choice A, one has to ask themselves why the endotoxin from the bacteria affect only neck muscle. This is not

very likely. For choices C and D, one has to ask themselves how the bacteria would cause stimulation ofnerve cell
innervating the neck muscle. There is no evidence in the passage for such interaction, while there is information
regarding the growth potential ofbacteria in the cerebral spinal fluid. The correct choice is B.
Nuclear Envelope and Pores

Passage VI (30 - 36)


30.

C is correct. The perinuclear space is the space between the inner nuclear membrane and the outer nuclear
membrane. It is stated in the passage that the outer nuclear membrane is studded with ribosomes. This should
indicate that the membrane of the ER is continuous with the outer nuclear membrane. Therefore, the space inside the

.ER, the lumen, should be continuous with the periplasmic space. The periplasmic space cannot be continuous with

the nucleoplasm, because of the division by the inner nuclear membrane. Thus, choice A iseliminated. Furthermore,
the periplasmic space is divided from the cytosol by the outer nuclear membrane. Therefore, this eliminates choice
B. Finally, the mitochondrial matrix is in no way continuous with the periplasmic space. They are separated by
many membranes and should not be construed as continuous. The correctchoice is C.
31.

B is correct. The nuclear envelope ensures that translation of a particular mRNA polymer into a protein occurs in

the cytosol and not the nucleoplasm. One can gather this from the passage by considering the resting size ofthe pore
and size given for the ribosome. The ribosomes is clearly too large to fit into a nuclear pore, and there would not be

a nuclearlmport signal on the ribosome. Therefore, translation is separated and confined from the nucleus. Consider
the other answers. There is no evidence that the nuclearenvelope ensures DNA replication. While nuclear envelope

breakdown is a part of the mitotic cycle, the envelope does not provide a means of insurance. We can eliminate
choice A. Choice C is false. We know that ribosomal RNA is produced in the nucleus and the nuclear envelope has

no known control over that process. Therefore, this answer can easily be eliminated. Finally, the nuclear envelope,
as stated above, breaks down during mitosis, not the interphase portion of the cell cycle. This makes choice D
incorrect. The nuclear envelope ensures that translation remains in the cytosol. The correct choice is B.
32.

C is correct. First, the statement that nuclear components cannot be radiolabeled is simply not true. Think of

labeling nucleotides that will eventually be incorporated into a nucleic acid polymer. Since this statement is

obviously false, it can easily be eliminated. We have no evidence from the passage that nuclear components are
difficult to isolate. We cannot make this assumption. Therefore, this is not the best answer, and choice B can be
eliminated. Let us look at choice D, which states that nuclear components are smaller than the nuclear pore,

implying that they would not be good measures of the size of the pore. This statement is also false. Think about
DNA and RNA polymerase. These molecules are very large and must be imported into the nucleus. Since this

Copyright by The Berkeley Review

90

The Berkeley Review

Specializing in MCAT Preparation

Biology

Structure and Function in Cells and Viruses

Section VI Answers

statement is false, choice Dcan easily be eliminated. This leaves us with choice C. The nuclear pore, according to
the passage, opens up beyond its resting size to accommodate those molecules that are larger than the pore size, yet
are necessary inside the nucleus for proper function. Therefore, it would not be a wise experimental choice (ifone is
trying to measure the non-accommodating resting size of the pore) to use nuclear components. The correct choice
isC.

33.

D is correct. It is clear from the question that only the tail portion entered into the nucleus. Since the tail portion is
attached to a 20 nm piece of gold, the nuclear pore must have opened up to accommodate the piece of gold.
Therefore, we can safely assume that the nuclear import signal is found on the tail piece. Since the gold attached to
the head piece did not make it into the nucleus, we can safely say there is no nuclear import signal on the head

portion ofthe protein. Therefore, we can eliminate choices Aand B. The next question is to decide the makeup of

the amino acids in the nuclear import signal. The passage tells us the signal isrich in positively charged amino acids.
The amino acids Phe and Trp are not positively charged. However, the amino acids Lys and Arg are basic, positively
charged amino acids. Therefore, they are likely candidates for the residues making up the nuclear import signal. The
correct choice is D.

34.

C is correct. We are told from the passage that the nuclear import signal can be found anywhere within the protein
and still be functional. This implies that the location of the signal is unimportant. Based on this information alone,

we can eliminate choices Band D, because they claim the location ofthe signal is important. The question is telling
us that a nuclear import signal is attached to random amino acids onto an enzyme that usually finds its home in the
cytosol. In time, that enzyme should be inside the nucleus, as the protein now has a nuclear import signal. Based on
that information, we can eliminate choice A, which claims the molecule remains within the cytosol. Therefore, the
molecule ends up inside the nucleus, because the location of the nuclear import signal is unimportant. The correct
choice is C.
35.

D is correct. The passage tells us that during mitosis, the nuclear envelope breaks down. It goes on to inform us that
after the nuclear division has taken place, the nuclear envelope begins to reform around the chromosomes. The

passage states that many of the previous molecules of the nucleus are exiled when the envelope reforms. However,
in time, these previous nucleus dwellers find their way back into the nucleus. It is that last sentence which is the
most important in answering this question correctly. If those molecules that were once inside the nucleus find their
way back to the nucleus after the envelope reforms, the nuclear import signal must not be lost on these molecules. In
other words, they must have the signal to get back into the nucleus. If these indeed are the same molecules that were
previously in the nucleus (meaning they were not resynthesized), the signal must have remained. Consider the other

answers. We cannot believe that all molecules that reside inside the nucleus are resynthesized after every nuclear
division. We have no evidence for this claim; and furthermore, it makes little sense, because it seems a terrible waste
of energy. Thus, we can eliminate choice A. Choice B can be eliminated, because it is false. The molecules from the
nucleus are not destroyed. Based on the discussion above, we can eliminate choice C, because the molecules most
likely retain their nuclear import signal. The correct choice is D.
36.

C iscorrect. We are told from the question that a molecule that is placed in the nucleus ofa frog oocyte is exported
out into the cytoplasm. Next, we are told that the same molecule, when placed directly in the cytosol, remains there.
This implies that the molecule has a nuclear export signal and it does not have a nuclear import signal. In other
words, it can only leave the nucleus, and not reenter. Based on this information, one can easily eliminate choices A
and B. The question then becomes where is the receptor that will recognize this nuclear export signal. Will it be on
the cytoplasmic face of the nuclear membrane, or the nucleoplasmic face? The answer seems obvious. In order to

recognize the nuclear export signal, the receptor must be facing the inside of the nucleus. Therefore, the receptor
must be on the nucleoplasmic face of the nuclear membrane. The correct choice is C.

Passage VII (37 - 43)


37.

Lipids and Membranes

B is correct. This question is designed to test your basic knowledge of the bacterium Escherichia coli (abbreviated
as E. coli). This prokaryotic organism is rod-like (bacilli), with a length of about 2 urn and a diameter of about 1

pm. Bacteria are classified as either being Gram-positive or Gram-negative. If a bacterium takes up the Gram stain
(a crystal violet dye and iodine), then it is said to be Gram-positive. If it does not take up the Gram stain, it is Gram-

negative. Surrounding the cytoplasm of a prokaryotic cell is the plasma membrane. Surrounding the plasma
membrane is a peptidoglycan (murein or cell wall) layer which consists of covalently linked polysaccharide and
polypeptide chains.

Copyright by The Berkeley Review

91

The Berkeley Review

Specializing in MCAT Preparation

Biology

Structure and Function in Cells and Viruses

Section VI Answers

Extracellular Face
}

Outer
membrane

Peptidoglycan

\ Periplasmic

(cell wall)

gel
Plasma
membrane

Cytoplasmic Face

Gram-positive bacterial wall


(e.g., Staphylococcus aureus)

Gram-negative bacterial wall


(e.g., Escherichia coli)

In the Gram-positive bacterium the peptidoglycan layer is quite thick (about 250 A) while in the Gram-negative
layer it is rather thin (about 30A). This is where the basic similarities between the two types of bacteria end.

Surrounding the peptidoglycan layer of Gram-negative bacteria is an outer membrane. The space between the
plasma membrane and the outer membrane ofa Gram-negative bacterium is often called the periplasmic space (or,
morecorrectly, the periplasmic gel). The correct choice is B.

D is correct All hydrolysis reactions are favorable. Hydrolysis of this phosphatidylethanolamine will give palmitic
acid, oleic acid, phosphate, ethanolamine, and glycerol. These individual structures are shown below. As we will

38.

learn later, each of these components will become important in metabolism. The correct choice is D.

H2C

HC

I
I

H2C

H2COH

O C- (CH2)l4CHj
H

II
I
I
O C- (CHj), - C= C- (CH2)7CHj

II

4H20
C

:> hcoh

H,C

O P O CH2CH2NH3

OH

Ethanol

Palmitic acid

II

ho c-(CH2)7-c=c-(CH2)7CH3

II

HO P OH
|

Oleicacid

HO CH2CH2NH,

Ethanolamine

Phosphate

A phosphatidylethanolamine
39.

ho c-(CH2)l4CHj

C is correct In bacteria, membrane lipids are synthesized primarily by integral membrane proteins. Thequestion is
whether that synthesis occurs on the extracellular face (outside) or cytoplasmic face (inside) of the membrane. As a
membrane lipid is being synthesized it begins to incorporate molecules like glycerol, fatty acids, ethanolamine, and

phosphate. In this case the phosphate which is incorporated into the membrane lipids isradioactively labeled.
Extracellular Face

^] TNBS complexed with


membrane lipids that
contain PE

Portion of ,
lipid bilayer

^ Radioactively labeled
membrane lipid

Cytoplasmic Face

IfTNBS is added to the cell suspension, we would expect it to bind to PE on the extracellular surface as well asthe
PE on the cytoplasmic face. If membrane lipids like PE were synthesized on the extracellular face, some of them
would be expected to be radioactively labeled and some would bind TNBS. This was not observed. IfTNBS could
cross the cell's plasma membrane, itshould be able to bind to radioactively labeled PE which was being synthesized

on the cytoplasmic face. Since itwas observed that none ofthe PE molecules which were radioactively labeled were

labeled with TNBS, it must mean that TNBS cannot cross thecell's plasma membrane and thatthe membrane lipids
Copyright by The BerkeleyReview

92

The Berkeley Review

Specializing in MCAT Preparation

Biology

Structure and Function in Cells and Viruses

Section VI Answers

are being synthesized on the cytoplasmicface. TNBS cannot cross the membrane because is bears a negative charge.
Remember, membrane lipids have head groups which are polar and part ofthat polarity is due to the negatively
charged phosphategroups. Like charges repel one another. The correct choice is C.
40.

Bis correct First, go back and read the previous answer. TNBS isstill negatively charged when it is added tothe
bacterial suspension. Therefore, it still cannot cross the bacterial membrane. This will eliminate choice A.
Membrane lipids like PE are synthesized on the cytoplasmic face, not the extracellular face. This eliminates choice
C and D. This leaves us (by default) with choice B.
Extracellular Face

\2 TNBS complexed with


membrane lipids that
contain radioactively

Portion of

labeled PE

lipid bilayer

\3 Radioactively labeled
membrane lipid

Cytoplasmic Face

As outlined in the passage, phospholipids in membrane bilayers will take days to undergo a flip-flop (i.e., transverse
diffusion). Just because the phospholipids take days to flip-flop does not mean that membrane lipids like PE cannot
flip-flop in a shorter period of time. A relevant diagram for Experiment 2 is shown below. The correct choice is B.
41.

Biscorrect The passage states that integral proteins are tightly bound tomembranes by hydrophobic forces. Those
hydrophobic forces arise from the interaction of the non-polar interior of the lipid bilayer and the hydrophobic
amino acid residues in that portion of the protein which is in contact with that non-polar interior. The only
hydrophilic forces ofinterest between the integral protein and the membrane will occur at the polar regions ofthe
lipid molecules. These forces, if they exist, are not as great as the interior hydrophobic forces (i.e., the exclusion of

water). Hydrogen bonds are mainly electrostatic interactions, and they occur in the limited polar region of the
membrane. Choice A and D are eliminated as possibilities, and choice C becomes a good candidate for the answer.
Carbohydrate

^3 residue
Outside

Inside

'ooc

Integral Protein

However, let's consider choice B. Asstated in the passage, both the lipids and the proteins found in a membrane are
asymmetrically distributed. This means that an integral protein is different on one side of the membrane than it is on

the other side of the membrane. For example, the C-terminal region ofa protein can befound on the inside ofa cell,
while the N-terminal region can be found on the outside. The N-terminal region on the outside of the membrane
might have more amino acids associated with it while the C-terminal region might have fewer amino acids. Also,
carbohydrate residues can be found attached tothe N-terminal domain (on the outside ofthe cell).
It turns out that it is the asymmetry of the integral protein that makes it so difficult for transverse diffusion to occur.
The hydrophilic domains of the integral protein not associated with the interior of the membrane have a difficult

time rotating and trying to pass through a hydrophobic interior. If transverse diffusion were to begin, the
hydrophobic portion of the protein and the hydrophobic portion of the membrane would still (for the most part)

remain in contact with one another. The correct choice is B.

Copyright by The Berkeley Review

93

The Berkeley Review

Specializing in MCAT Preparation

Biology
42.

Structure and Function in Cells and Viruses

Section VI Answers

C is correct. The question asks about the fluidity in the membrane of a bacterial cell, not a eukaryotic cell.

According to the passage, the bacterium Escherichia coli contains 0% cholesterol in its membrane. Since cholesterol
is not present, eliminate choices A and D as answers. A decrease in the membrane's fluidity means that the
membrane is becoming stiffer. If there were long-chain fatty acids with cis double bonds in the membrane, this
would introduce a kink in the fatty acid and not allow for close packing of the hydrocarbon tails. The membrane
tends to be a bit more fluid. Eliminate choice B. If we increase the length of the hydrocarbon tail of a saturated fatty

acid, this would allow for more hydrophobic interactions between the individual methylene (-CH2-) groups. The

more hydrophobic interactions, the tighter the packing of the hydrocarbon tails, and the less fluid the membrane.
The correct choice is C.

43.

D is correct. Spermaceti has two long hydrocarbon tails, which can pack together quite well. As mentioned in the
question, this wax melts between 42-47 F. It tends to be more of a fluid than a solid above this temperature range.
As the sperm whale dives to great depths, the temperature of the water begins to drop to the low 30 F range. Cold
sea water is circulated through special chambers in the whale's head. Since the cold water has a temperature below

the melting temperature ofthe wax, the wax begins to solidify. As the wax goes from a fluid state to a solid state, its
volume decreases. A decrease in volume is related to an increase in density. Recall that the density is the ratio of an

object's mass to its volume. As the whale's density increases, it is able to stay at these great depths for longer periods
of time without expending considerable energy.

In other words, by controlling the shape of the wax the whale controls its own buoyancy. This is analogous to the
weights that a scuba diver uses on her weight belt as she descends into the water. When the whale ascends, the

temperature ofthe cold water in the chambers increases because ofthe increased circulation of warm blood to the
head. The correct choice is D.

Nonstandard Ammo Acids

Passage VIII (44 - 50)


44.

45.

A is correct. The passage states the amino acid residue is modified after the protein is synthesized. Only posttranslation indicates synthesis is complete. Transcription is the production of a template m-RNA for protein
synthesis. Choice B is incorrect. Replication is the duplication of DNA in the nucleus. Choice C is incorrect.
Degradation means the protein is destroyed. Choice Dis incorrect. The correct choice is A.
D is correct. Choice A is hydroxylysine. Choice B is lysine. Choice C is hydroxyproline. Choice D is proline. The
correct choice is D.

46.

D is correct. The nonstandard amino acids are ones not specified by the standard tRNA codon table. They are
modified versions of the standard amino acids (modified after translation). Therefore, there are no tRNA codons

corresponding to the nonstandard amino acids. The correct choice is D.


47.

B is correct. The box marked II indicates the carbamoyl moiety. Look at the molecule carbamoyl phosphate: [NH2-

C=0]phosphate. The carbamoyl moiety is in brackets. That's your answer. The correct choice is B.
48.

B iscorrect. From the passage, we learned that hydroxyproline is formed by modification of a synthesized peptide.
Thus, proline is incorporated and then modified to hydroxyproline after the peptide is made. This means
hydroxyproline itself would not be incorporated. Choices Aand Care incorrect. Proline is not further broken down
in the digestive tract, so choice D is incorrect. The correct choiceis B.

49.

A is correct. The liver contains the enzymes of the urea cycle and produces urea from excess amino groups.

Although urea passes through the kidney on its way to excretion in the urine, the kidney does not make urea. Choice
B is incorrect. Neither the gall bladder or the pancreas has a role in urea biosynthesis. Choices C and D are incorrect.
The correct choice is A.

50.

D is correct. Collagen would not be synthesized properly if hydroxyproline were not available due to impaired

activity ofprolyl hydroxylase. Collagen is a major component ofconnective tissue and plays an important role in
blood vessel strength, healing of wounds, and healthy gum tissue. All the symptoms listed are symptoms ofscurvy.
The correct choice is D.

Copyright by The Berkeley Review

94

The Berkeley Review

Specializing in MCAT Preparation

Biology

Structure and Function in Cells and Viruses

Passage IX (51 - 58)

51.

Section VI Answers

Amino Acid Characteristics

D is correct. This relationship is quite important and stems from the fact that it is often necessary to express the
concentrations of hydrogen and hydroxide ions in an aqueous solution. These concentrations can be quite awkward

to work with (e.g., 10 Mto 10"14 M) and so acommon logarithmic notation (based on logio) is used for simplicity.

One of the rules of logarithms states that for a given quantity X,

pX = log 1 = - log X
X

This allows the [He] to be expressed as,

pH =log-L-=-log[H+]
or even the [OHe] as,
1

pOH = log

_
=
- log [OH]

|OH'

Natural logarithms have as their base e = 2.718 and are exponents to which e must be raised to give a number. These
are quite different from common logarithms. The correct choice is D.
52.

Discorrect. Physiological pH is considered to be 7.4. All of the common 20 amino acids at physiological pH will
have their a-amino group in the protonated state and their a-carboxyl group in the ionized (deprotonated) state. This

is because the average pKa ofthe a-amino group is about 9.5 and the average pKa of the a-carboxyl group is about
2.2. At physiological pH many of these amino acids will be in their zwitterionic form (where the net charge on the
amino acid adds to zero). Since each amino acid will bear at least two charges at physiological pH, they cannot be
nonpolar and they cannot be monovalent. The correct choice is D.
53.

B is correct. Since all of these amino acids will be found near the center of a long polypeptide chain, their a-amino
group and a-carboxyl group will be tied up in a peptide linkage. The only protons that are able to ionize are those on
amino acids with ionizable side chains. There are only 7 amino acids with ionizable side chains, and three of them
are listed as answers. The one amino acid which does not have an ionizable side chain is isoleucine. The correct
choice is B.

54.

C is correct. The two pKa values for leucine were given in the passage. Leucine's a-amino group has a pKa of 9.6
while its a-carboxyl group has a pKa of 2.4. The pi can be calculated as follows:

pi _ pKa (NH?) + pKa (COOH) = 9.6 +2.4-60


2

Note that leucine will carry no net electric charge at its pi. In the case of leucine this is its zwitterionic form (it is a
dipolar ion). The correct choice is C.

55.

Dis correct. The only difference between phenylalanine and tyrosine is the presence ofa hydroxyl (OH) group on
the ring.

-. coo

o1

H2
i

coo

H,N- C- H
I

CH,

Enzyme

Phenylalanine

A carboxylation reaction would have added a CO2 group. A hydrolysis reaction would have used water to break

(lyse) a molecule. The elements of water would have also been added to the products. A hydration reaction involves
Copyright by The Berkeley Review

95

The Berkeley Review

Specializing in MCAT Preparation

Biology

Structure and Function in Cells and Viruses

Section VI Answers

the addition of water to a molecule without lysing that molecule. Since we do not see any of these three types of
reactions in the conversion ofphenylalanine to tyrosine, it must be a hydroxylation reaction. The correct choice is
D.

56.

D is correct. This means that certain amino acids within a protein can have side chains with dissociable hydrogen

atoms. The pKa's of these amino acid side chains can range from about 3.9 (aspartic acid) to about 12.5 (arginine).
This is quite a wide range with which a protein can donate or accept hydrogen ions (i.e., act as a buffer).
Proteins can form secondary and tertiary structure through internal hydrogen bonding interactions. This occurs
through amino acid interaction within the polypeptide chain itself. Once a protein is in its tertiary form it will stay in

that form until it is denatured. Protons in the medium surrounding the protein are (for the most part) not involved in

the hydrogen bonding that dictates the secondary and tertiary structure of aprotein. If a protein were denatured (by

hydrolysis^), the element of water would add across the peptide bond being broken. Once this happened the ability ot
protons to bind to a free terminus would be diminished. However, protons can add to the N-terminus and the Cterminus of a protein. This action only absorbs (at most) two protons. Many more protons could be absorbed by

amino acids that have ionizable side chains. The correct choice is D.

57.

C is correct. The key to answering this question is to locate all ofthe amide bonds (see the dots in the structure
below). Each amide bond will require 1 molecule of water. After hydrolysis we will get 4 compounds. They are
glutamic acid (Glu). histidine (His), proline (Pro), and ammonia (NH3).
pyro-Glu
H

Pro

His

HO

HO

11 #

i<>

11 9

H- N-C-C-N-C-C-N-C-C-NH,

'

;c

'

'

CH,

Note that glutamic acid in the structure is referred to as pyro-Glu. This is because its side chain has condensed with

its own a-amino group to form a ring structure involving an internal amide bond. Hydrolysis of this bond simply

exposes the glutamic acid side chain. The correct choice is C.

58.

Ais correct. Hydrolysis ofthe internal amide bond in pyro-Glu and the terminal amide bond ofPro will allow us to
see the tripeptide in its "true" linear form.
C-terminus

N-terninus
H

HO

HO

II

II

II

H- N-C- C-N-C-C-N- C- C-OH


I

I H

CH,

NH3
IV

CH,

I "
CH,

- N>^
HO

N-

II

The N-terminus is the a-amino group of Glu. The correct choice is A.

Passage X (59 - 65)


59.

Fluorescent Recovery after Photobleaching (FRAP)

Bis correct. To answer this question, one must be aware ofthe following idea: Adrop in temperature will cause the

bilayer to become more viscous. The increased viscosity will cause adecrease in the fluidity of the membrane. Since
the proteins are embedded inside the lipid bilayer, the time it takes to react the state in Figure 2should increase due
to the decreased fluidity ofthe membrane. In fact, organisms such as bacteria and yeast will change the composition

of the lipids making up their biological membrane in responses to drops in temperature. Based on this information,
all otheranswers are easily eliminated. The correct choice is B.
Copyright by The Berkeley Review

96

The Berkeley Review

Specializing in MCAT Preparation

BlOlOgy
60.

Structure and Function in Cells and Viruses

Section VI Answers

Cis correct. From the passage, we known that monovalent antibodies are used in this process. Normally antibodies

are bivalent, that is, they have two arms, each of which is capable of binding to an antigenic determinant In our

immune system response, such anatomy is beneficial because antibodies can cross-link antigen molecules into a

large lattice as long as each antigen molecule has three or more antigenic determinants (which they usually do)
However, in this experiment, one can imagine that cross-linking between fluorescent antibodies can greatly affect

the movement of proteins and thus the results ofthe lateral diffusion rates. The correct choice is C.

61.

Bis correct. At t=0minutes, the labeled antibodies of the mouse and human are on their own halves of the hybrid

cell. At t=40 minutes, it is clear that the antibodies are mixed. The only way the mixing could occur would be ifthe
lipid molecules readily exchange places with their neighbors within a monolayer. This movement is known as lateral

diffusion, and this experiment provides evidence for such a process. The correct choice is B.
62.

Dis correct. This answer requires one to think about the experiment and interpret the graph. The graph shows that
the asymptotic level of fluorescence reached is lower than the original level of fluorescence. Why could this be?
Look for the obvious. In the experiment, we use a laser beam to bleach some of the fluorescent ligand. We will not
be able to recover our original amount of fluorescent because we have permanently bleached the ligand. We can
assume that the bleaching is permanent and not temporary on two accounts. There is no evidence from the passage
that demonstrates that the bleaching is only temporary. Furthermore, ifthe bleaching is only temporary, over time
we should see the level recover to its original level. We do not see this in the graph, as the level reached is

asymptotic. Therefore, we can eliminate choice C. There is no evidence for choices Aand Bin either the passage or
our body of knowledge. The correct choice is D.
63.

Dis correct. This question requires graphical interpretation. The longer it takes for the recovery process to occur,
the smaller the diffusion rate. Since all ofthe experiments were carried out in similar conditions, we are really
looking a diffusion coefficients. The greater the rate of recovery, the greater the diffusion coefficient of the
membrane glycoprotein. Therefore, it becomes clear that it takes glycoprotein D the longest amount of time to reach
the full rate of recovery. Therefore, glycoprotein D must have the lowest diffusion rate and thus smallest diffusion
coefficient. The correct choice is D.

64.

D is correct. The question requires us to think about several things. First, there is no evidence that the antibodies

used are lighter in the in vitro system. Furthermore, we have no evidence for a correlation between weight and
dittusion rates. Since we do not have this information, we cannot make the assumption. Therefore, we can eliminate
choice A. Next, we are not told of any switching to a bivalent antibody. Therefore, we have no reason to believe that

cross-linking would be occurring. We can eliminate choice B. So now we are left to think about oligosaccharide
interaction on the surface of cell membranes. First, the oligosaccharides are found on the extracellular face, and not
the intracellular face, of the plasma membrane. This alone eliminates choice C. Let us finish the conclusion. With

only one glycoprotein, there is no interaction between these bulky oligosaccharide groups as the proteins are
diffusing laterally. The less interaction leads to higher diffusion rates. The correct choice is D.
65.

C is correct. The question requires us to know a little about cholesterol and its role in our biological membranes. At
the high concentrations that cholesterol is found in eukaryotic cells, it acts to increase the fluidity of the membrane.
While this may seem counter-intuitive, it is indeed the case. Therefore, one can easily eliminate choices A and B.
Now the question becomes how does cholesterol increase the fluidity of the membrane. Well, it prevents the
hydrocarbon chains making up the lipids from crystallizing by preventing their interaction. One can think of the

bulky cholesterol as getting in the way with its large, hydrophobic, classic steroid rings. Therefore, cholesterol acts
to increase the fluidity of the membrane by inhibiting hydrocarbon chain crystallization. The correct choice is C.
Passage XI (66 - 72)
66.

Gram-Negative and Gram-Positive Bacteria

Bis correct. This question is not based on any information in the passage, but rather tests you on information you

might have learned in some of your science courses. Many bacteria have rather distinctive shapes and sizes. Even

though it would be rather difficult to associate all the species ofbacteria with a given characteristic, it might be

worth your while to remember at least one or two of the classic prokaryotic cells and their characteristic shapes'!

Bacteria shaped like spheres are referred to as cocci. The best known examples comes from the genus Streptococcus.
The two species of streptococci that cause most of the streptococcal diseases associated with humans are

Streptococcus pneumoniae (causes pneumonia) and Streptococcus pyogenes (causes impetigo, a superficial skin
Copyright by The Berkeley Review

97

The Berkeley Review

Specializing in MCAT Preparation

Biology

Structure and Function in Cells and Viruses

Section vi Answers

infection commonly found in children). Bacteria shaped like cylinders are referred to as rods (also called bacilli).
The best known example comes from the genus Escherichia and has the species name Escherichia coli (which can
cause bladder and kidney infections). Bacteria shaped like spirals come in different sizes as well. There are those
bacteriawhich are short and twisted into a rigid spiral,and there are those bacteria which are elongated and twisted
into a flexible spiral helix. Representatives of these two forms of bacteria are not as commonly mentioned as the
ones above, but we will give two examples for completeness. A typical spiral bacterium comes from the genus
Spirillum. The species Spirillum volutans is one of the largest bacteria known. One of the better known bacteria that
represents a spiral helix comes from the spirochete genus, Treponema. The species Treponema pallidum causes
syphilis in humans. The correct choice is B.

67.

A is correct Invariant simply means that something (in this case a characteristic) is unchanging or constant.
Another way to look at this question is to ask yourself which are the most important features of a prokaryotic cell.
Certainly the nuclear region which contains the DNA is important. If new proteins need to be made to help in
cellular function, then the DNA must be transcribed into mRNA. This message must then be translated at the level

of the ribosome into a particular protein. Variation in either the nuclear region or the ribosomes could be deadly to
the cell. The same can be said about the plasma membrane. This is the membrane that prevents the cytoplasm from
leaving the interiorof the cell. Lysis of the plasma membrane leadsto immediate destruction of the cell.
The cell wall is the only structure mentioned which can be invariant. If the cell wall were to be removed from a
given prokaryotic cell, that cell would form a protoplast (see the passage). These cellular structures are still capable
of carrying out metabolic processes given the right environment. In fact, one genus of bacteria (Mycoplasma) lacks a
cell wall entirely, simply because it cannot synthesize the precursors needed for the formation of the peptidoglycan
layer. The correct choice is A.

68.

D is correct Recall that Gram-positive bacteria do not have an outer membrane, while Gram-negative bacteria do.
Also, the peptidoglycan layer in Gram-positive bacteria is much thicker than the peptidoglycan layer in Gramnegative bacteria. In Step 1 both types of cells are treated with crystal violet. This procedure will stain both cell
types purple. In Step 2 iodine is added to fix the crystal violet in the cells. A complex of crystal violet and iodine is
formed. Again, both cell types will remain purple. In Step 3 alcohol is added and acts to decolorize the cells.
However, only the Gram-negative cells are decolorized by the alcohol. The Gram-positive cells remain purple. This
is due to the thickness of the peptidoglycan layer of the Gram-positive cells. Alcohol tends to dehydrate this layer,
thus making any pores within the layer itself rather small. These small pores hinder the passage of the crystal violetiodine complex during the extraction process. Since the crystal violet-iodine complex remains trapped in the
peptidoglycan layer, Gram-positive cells still display a violet color at this stage. In Gram-negative bacteria the thin
peptidoglycan layer does not significantly hinder the extraction process, and these cells therefore display no color
(i.e., they are colorless) at this stage. In Step 4 a counterstain (safranin) is added to the suspension. This red-colored
stain is added so the Gram-negative bacterial cells can be visualized. They now display a red color. The purple
colored Gram-positive bacteria also pick up the red stain and now appear blue. The correct choice is D.

69.

D is correct In Gram-positive bacteria the peptidoglycan layer is rather thick, while in Gram-negative bacteria the
peptidoglycan layer is rather thin. The Gram-staining procedure takes advantage of these different characteristics. It
is important to understand that the designations "Gram-positive" and Gram-negative" refer to the fact that a
particular bacterial cell can resist decolorization or be decolorized, respectively.

The technique employed by the Gram procedure is an entirely different matter. In order for a cell to be clearly seen
under a light microscope, organic dyes are used to stain cells. This technique is referred to as positive staining. The
dyes used can either be cations (positively charged) or anions (negatively charged). Crystal violet and safranin are
both examples of dyes which are cations. One of the reasons that these two dyes work rather well in staining cells is
because the membranes of cells show a high degree of negative charge.

Negative staining allows cells to be visualized in an outline form. This technique calls for the staining of the cell's
background while leaving the cell itself stain-free.
Differentialstaining is a procedure that does not stain bacterial cells equally. Either positive or negative stains are
used in this procedure. Therefore, we find that the Gram staining procedure is both a positive staining procedure and
a differential staining procedure. The correct choice is D.

Copyright by The Berkeley Review

98

The Berkeley Review


Specializing in MCAT Preparation

Biology
70.

Structure and Function in Cells and Viruses

Section VI Answers

Bis correct Recall that the enzyme transpeptidase links amino acids together between adjacent glycan chains. This

linkage helps to stabilize the forming peptidoglycan layer. If the enzyme were found near Face 1, it.would not be

close to the peptidoglycan layer and would therefore be unable to catalyze the cross-linkage between adjacent amino

acids of the glycan chains. We can rule out choice Ain both types of bacteria. We can immediately rule out two
other choices if we recall that Gram-positive bacteria do not have an outer membrane, and hence do not have

membrane Face 3 or membrane Face 4. Remember, the question asks for location of the enzyme in both Grampositive and Gram-negative bacteria. Choices C and Dare eliminated. By default, this leaves choice B. Notice that if

the enzyme were near membrane Face B, it could easily catalyze the joining of amino acids in the peptidoglycan
layer. The correct choice is B.

71.

Ais correct In order for bacterial cell growth to occur, the peptidoglycan layer must first be opened up by the

action ofautolysins. Specific sugar derivatives and amino acids are then added to the peptidoglycan layer as the cell
begins to grow. The strength in the peptidoglycan layer comes from the cross-linking of peptide bonds between

individual glycan chains by enzymes called transpeptidases.

If these enzymes are inhibited (by penicillin), then cross-linking can not be completed and the peptidoglycan layer

remains weak. The cell becomes susceptible to lysis. Ifthe autolysins were inhibited, the peptidoglycan layer would
not be able to grow and therefore the cell would stop growing. Since the peptidoglycan layer would not be opened
up, the action ofthe transpeptidases would not be needed. Penicillin is not able to induce lysis in non-growing cells,
because inhibition of the transpeptidase enzyme would have no effect.

Increasing the growth rate of the cells would mean that penicillin could interact with the transpeptidases and inhibit
the action of this enzyme. Even ifthe growth rate were to increase, the growing cells would still lyse.
Under normal conditions, the solute concentration within a bacterial cell is much higher than the solute
concentration outside a bacterial cell. Water wants to flow down its concentration gradient and into the cell, causing
the cell to swell and eventually lyse. Lysis by penicillin can also be prevented by adding a solute such as sucrose to
the medium in which the cell resides. If the solute concentration inside the cell balances the solute concentration
outside the cell, then structures called protoplasts form when the cell wall of the bacterium becomes too weak.

Protoplasts are (spherical) cellular structures that do not have a cell wall. They have lost the ability to retain it.
However, theses structures still have their plasma membrane and all of their intracellular components. Inhibition of
protoplast formation would mean that the solute concentrations across the bacterial cell membrane(s) are not the
same. If penicillin is administered, and the bacterial cells cannot form protoplasts, they will lyse.

Decreasing the solute concentration outside the cell sets up an even larger solute concentration gradient between the
inside and the outside of the cell. More water will want to flow into the cell. If this happens, the cell swells and
eventually lyses. The correct choice is A.
72.

C is correct. First, look at Figure 1. Note that there is an outer membrane (with a pore), a peptidoglycan layer, and
then a plasma membrane. The action of both penicillin and lysozyme is described in the last paragraph of the
passage. Both work on the peptidoglycan layer. In order to work on the peptidoglycan layer, they both must have
access to it. The only types of cells that will allow for such access are Gram-positive cells. Gram-negative cells have
that extra outer membrane and tend to be (more) resistant to penicillin and lysozyme.
Choice A states that the cell is nonresistant to penicillin, because the antibiotic can cross the outer membrane.

Penicillin is a large molecule and would have a difficult time crossing the lipid bilayer of the outer membrane. What
about passage through the pore? Again, penicillin would be too large to pass through the pore. The same is true for

lysozyme. Enzymes are generally quite large (much larger than penicillin). If penicillin cannot pass through the
pore, either can lysozyme. This allows us to eliminate choice A and choice D. Even though choice B is a true
statement, there is a better answer. Do notjust sequentially read down the answers until you find one that is correct.
You need to find the best answer. The correct choice is C.

Passage XII (73 - 79)


73.

Raffinose

B is correct. As shown by the molecular structure, raffinose is clearly not a disaccharide. It is considered to be an

oligosaccharide (or even a polysaccharide). Since there are no amino acids attached to and of the three sugar
residues, it cannot be a glycoprotein. This means that raffinose is either a reducing sugar or a non-reducing sugar.

Copyright by The Berkeley Review

99

The Berkeley Review


Specializing in MCAT Preparation

Biology

Structure and Function in Cells and Viruses

Section vi Answers

Sugars having anomeric carbon atoms that have not formed glycosides (containing an acetal linkage) are called
reducing sugars. Recall that the cyclic and linearforms of both aldoses and ketoses are readily interconverted. The
aldehyde function in galactose and glucose can easily be oxidized by an oxidizing agent like Benedict's reagent (a
solutionof copper(II)sulfate and sodiumcitrate in aqueous base) to the corresponding carboxylic acid (see below).
cho

coo"

H-C-OH

HO-C-H

"-{-OH
H-c-oh
1

ch2-oh

H-C-OH

+ 2Cu2+ +5OH

HO-C-H

Benedict.s

y H-C-OH

reagent
(blue)

h-c-oh
'

'

+ Cu20(s) +3H20

red

precipitate

ch2-oh

D-Glucose

D-Gluconate

(open chain)

Fructose is an a-hydroxy ketone and cc-hydroxy ketones are easily oxidized to the diketone by Benedict's reagent.
The Benedict's reagent, which is a blue solution, is reduced to a red precipitate. If the aldehyde and ketone
functional groups remain tied up in a glycosidic bond (as shown in the structure of raffinose), then they cannot react
with the Benedict's reagent. The correct choice is B.
74.

C is correct Sugars with an aldehyde function are referred to as aldoses, while those with ketone functions are
referred to as ketoses. The open chain form of glucose can undergo intramolecular hemiacetal formation. In this
case, the hydroxyl oxygen of the C-5 carbon attacks the carbonyl carbon of the aldehyde to form the stable
hemiacetal. This hemiacetal, which forms a six-membered ring, is referred to as a pyranose. With this much
information, we can eliminate choices A and D. The open chain form of fructose can undergo intramolecular
hemiketal formation to produce a five-membered ring referred to as a furanose. The hydroxyl oxygen of the C-6
carbon attacks the carbonyl carbon of the ketone to give the hemiketal. The correct choice is C.

75.

B is correct This answer depends on how you view the linkage between glucose and fructose. Locate the anomeric
carbons of each sugar residue. The anomeric carbon for glucose is at the C-1 position while the anomeric carbon for
fructose is at the C-2 position. If we are looking at this linkage from the point of view of glucose, then the linkage

will be 12. Is this linkage in the a or the (3 position? The bond stemming from the anomeric carbon of glucose is
pointing down, below the plane of the Haworth projection. This represents the ot-configuration. Therefore, we can
say, that from the point of view of glucose, the bond is in the oc(l-2) configuration. But what if we look at the bond
from the point of view of fructose? In this case, the linkage is 2->l. Since the bond at the anomeric carbon is

pointing up, above the plane of the Haworth projection, it is in the p-configuration. Therefore, the oc(l-2)
configuration by itself is not good enough. We also need to consider the p(21) configuration as well. The correct
choice is B.

76.

D is correct First you must find the anomeric carbonof each sugar. The anomericcarbons of galactose and glucose
are both at the C-l position. The bondleading form the C-l position of galactose is connecting with the C-6 position
of glucose. Note that the C-l bond of glucose is connecting with fructose. This allows us to eliminate choices A and
B. Now, is the 1>6 linkage in the a or the (3 position? In order to determine this we need to consider the anomeric
carbon that links the two molecules. That anomeric carbon belongs to galactose. Note that the bond stemming from
the anomeric carbon of galactose is pointing down, below the plane of the Haworth projection. This represent the aconfiguration. If the bond were pointing up, it would represent the ^-configuration. The correct choice is D.

77.

B is correct Recall that when sugars differ from one another about one carbon atom they are referred to as epimers.
The only place where the carbon atoms of galactose and glucose differ from one another is at the C-4 position. In
galactose the hydroxyl at the C-4 position is above the plane of the ring, while in glucose it is below the plane of the
ring. The correct choice is B.

78.

C is correct. Hydrolysis of just the galactose residue from raffinose leaves a single disaccharide composed of
glucose and fructose. Sucrose is composed of both glucose and fructose. However, in order to call this disaccharide
sucrose, the two monosaccharides, glucose and fructose, must be linked together in the correct fashion. Note that the
linkage between these two sugars is glucose-oc(l2)-fructose. This is the correct linkage between these two sugar
residues. Lactose is a disaccharide composed of galactose and glucose while maltose is a disaccharide composed of
two glucose residues. The correct choice is C.

Copyright by The Berkeley Review

100

The Berkeley Review


Specializing in MCAT Preparation

Biology
79.

Structure and Function in Cells and Viruses

Section VI Answers

B iscorrect Ifthere had been a loss of bacterial enzymes that degrade oligosaccharides, then the bacteria would not

be able to produce the monosaccharides which (when metabolized) lead to the production of lactate

(CHiCHOHCOO9), methane (CH4), carbon dioxide (CO2), and hydrogen gas (H2). It is the gases that are produced
that lead to flatulence. These compounds can also cause an increase in the motility ofthe intestinal system, fluid

secretions, and cramps. The correct choice is B.

Passage XIII (80 - 87)


80.

Mitosis and Meiosis

Ais correct If we start the cell cycle just after cytokinesis (division of the cytoplasm), we will find the following
order:

G] Phase

S Phase

G2 Phase

(Growth)

(DNA Synthesis)

(Growth)

Prophase -*- Metaphase - Anaphase -> Telophase -* Cytokinesis

Interphase

Mitosis

Note that in choice Bthe S phase is shown to follow interphase. The S phase does not follow interphase, rather it is a
component of interphase. In choice C it says that anaphase comes after telophase. This is not the case as outlined
above. Finally, in choice C the Gi phase comes before the S phase, not after it. The G2 phase comes after the S
phase. The correct choice is A.
81.

D is correct. Two mature cell types in the body which do not replicate at all are skeletal muscle cells and nerve

cells. These cell types are said to be in their "resting" state in which they are maintaining the metabolic functions of

the cell. The resting state for these cell is the Gi phase ofthe cell cycle. It is in the Gi phase that transcription and
translation for the various cell functions take place. The Gi phase is part of interphase. Interphase also includes the

G2 phase (where the chromatin condenses) and the S phase (where DNA is replicated). After interphase is
completed at the end of the G2 phase, a cell enters into the M phase. This is where mitosis and eventual division of
the cytoplasm (cytokinesis) occurs. The correct choice is D.

82.

Dis correct. We are interested in following labeled precursors which are incorporated into DNA. Northern blotting
involves RNA analysis, while Western blotting involves analysis of proteins. This allows us to eliminate choices A

and C. As outlined in the passage, Southern blotting is used to locate certain genes in a segment of double-stranded
DNA. The question is asking us for a way to monitor the incorporation of labeled precursors into DNA, not to locate
a particular gene in the DNA. Autoradiography is employed to locate radioactively labeled molecules. Radioactive
decay from an isotope that has been incorporated into a molecule would reduce silver grains in the emulsion of a

sheet of film. Development of the film would show a deposition of silver grains, indicating the location of the
labeled precursors. The correct choice is D.

83.

A is correct. This question is designed to see if you understand the differences between the bases in DNA and RNA.
Recall that DNA contains adenine (A), thymine (T), guanine (G), and cytosine (C). RNA contains A, G, C, but uses

uracil (U) in place ofT. If we were to use 3H-Uracil, the label would show up in RNA. If we were to use either 3Hcytosine or 3H-adenine, the label would show up both in DNA and in RNA. This allows us to eliminate choices B,
C, and D. If we just use 3H-thymine, the label shows up in the DNA and not in the RNA. The correct choice is A.
84.

C iscorrect. Down syndrome (trisomy 21) isthe most common type of human aneuploidy (the change in number of
one or more chromosomes during gametogenesis). It results from the non-disjunction of chromosome 21 from either
the father or the mother. Individuals with Down syndrome show a number of physical and mental abnormalities.

Life expectancy isrelatively short, with a mean atabout 17 years. Individuals with trisomy 13 (Patau syndrome) and
trisomy 18 (Edwards syndrome) show severe physical and mental abnormalities and usually die soon after birth.
Trisomy 22 is common in abortuses. Since there was no mention of the cause of Down syndrome in the passage, it
was assumed that you would have learned about this genetic condition in your general science classes. The correct
choice is C.

85.

C is correct The best way to see the difference between primary and secondary nondisjunction is to consider
diagrams of each case. Primary nondisjunction (as stated in the question) occurs at the first meiotic division while

secondary nondisjunction occurs at the second meiotic division. These types of disjunctions are drawn below. By
comparing the answers with these diagrams, we see that C is the correct choice.

Copyright by The Berkeley Review

101

The Berkeley Review

Specializing in MCAT Preparation

Biology

Structure and Function in Cells and Viruses

Section VI Answers

Meiosis I

Meiosis I

Meiosis II

Meiosis II

Secondary Nondisjunction

Primary Nondisjunction

If a normal gamete were to unite with one of the gametes of primary nondisjunction during fertilization, half the
zygotes that would arise which would show trisomy (the gain of one chromosome) and half would show monosomy
(the loss of one chromosome). However, if that normal gamete were to unite with one of the gametes of secondary
nondisjunction, then half of the zygotes would be normal, a quarter would show trisomy, and a quarter would show
monosomy. The correct choice is C.
86.

C is correct Maternal simply refers to the mother, and paternal refers to the father. The procedure for establishing
the autoradiograms is well outlined in the passage. All we need to do is read the results.
In both families we are looking for that parent which contributes to DNA fragments to the Down's child. In Family 1
note that the mother contributes the two fragments (one at about 5 kilobases and the other at about 1 kilobase).
Normal Down's
Mother

Father

Child

Normal Down's

Child

Mother

Father

Child

Child

CU

*-

to

c
a
o> s i

ca

||

B 3

8P32

Autoradiograph of Family 1

Autoradiograph of Family 2

The father only contributes one fragment (at about 2 kilobases). In Family 2 it is the father who contributes the two
fragments to the Down child (one at about 3 kilobases and the other at about 2 kilobases). The mother only
contributes one fragment (at about 4 kilobases). The correct choice is C.
87.

B is correct In order to understand where this answer came from, it is important to understand meiosis. Before the
actual onset of meiosis, DNA has already been synthesized during the S phase of the cell cycle. In prophase I of the
first meiotic division the chromosomes become visible and centromeres begin to develop. Homologous
chromosomes begin to pair or synapse at the centromere. This structure of two sister chromatids is referred to as a
bivalent. When two bivalents come together the complex is referred to as a tetrad. A tetrad consists of four
chromatids. The number of tetrads is equal to the number of haploid chromosomes. This means that if we have 23
haploid chromosomes, we must have 4 times that many chromatids. In other words, we have 23 x 4 = 96 chromatids.

After telophase I of the first meiotic division has occurred, two haploid nuclei are formed. These nuclei are haploid
by definition, even though they contain pairs of sister chromatids. Since the sister chromatids are attached by the
same centromere they are considered to be part of one chromosome. The total number of chromosomes in each
nuclei has been reduced by one-half. This is why the first meiotic division is referred to as a reduction division. The
second meiotic division is similar to mitosis and is therefore referred to as a equational division.

Copyright by The Berkeley Review

102

The Berkeley Review

Specializing in MCAT Preparation

Biology

Structure and Function in Cells and Viruses

Section VI Answers

Since the total number of chromosomes in each nuclei has been reduced by one-half, we now have 46chromatids in
each nuclei as we enter into the second meiotic division. During metaphase II of the second meiotic division the

chromosomes align along the equatorial plane. Ifwe stop at metaphase II, we find that the daughter chromatids have
not yet separated. Therefore, we will still have 46 chromatids present in each ofthe two nuclei at metaphase II. It is
not until anaphase II that the centromeres begin to divide and the sister chromatids move to the opposite poles. After

telophase II there will be 23 chromatids in each of the four meiotic nuclei. The correct choice is B.

Passage XIV (88 - 94)


88.

Lipids: Four Groups

C is correct Fats are esters. Recall that esters result from the union of an alcohol and a carboxylic acid. The
backbone of all fats is glycerol, a three-carbon molecule containing a hydroxyl group on each of the carbon atoms.

Fatty acids contain a carboxylic acid group.


o
II

H,C- OH

HO- C

HC- OH

HO- C

H,C- OH

HO- C

.CH,

o
II

CH,

o
II

Glycerol

CH,

3 Fatty Acids (Stearic acid)

A Triglyceride

Since glycerol contains 3 hydroxyl groups, there is the possibility of joining 3 fatty acids to one molecule of
glycerol. This will form a fat called a triglyceride. Each linkage will be an ester linkage. The correct choice is C.

89.

A is correct. This question involves looking at the four structures in the passage and determining the order of
polarity. We want to arrange the polarity from the most polar molecule to the least. The most polar molecule bears a
full charge or charges. The only structure in our four diagrams that bears full charges is phosphatidyl choline. The
remaining three structures do not bear a full charge. However, they all contain oxygen atoms, which are highly
electronegative. We first look for that structure with the most oxygen atoms. It turns out that both the triglyceride
and arachidonic acid have two oxygen atoms. Which is more polar? At physiological pH, arachidonic acid has its
carboxyl hydrogen ionized (not shown in the diagram). The two oxygen atoms on the carboxylic acid group will
share the negative charge between them (through resonance). This leads to stabilization of that functional group and
an increased polarity. The triglyceride contains ester linkages. Because one of the oxygen atoms is tied up with two
other carbon atoms, the polarity of the ester linkage is reduced. However, it is not as reduced as the polarity of the
single oxygen atom of the hydroxyl groupof cholesterol. With this arrangement of polarity, we can order the lipids
in terms of decreasing polarity. The correct choice is A.

90.

C is correct. The more saturated the fatty acid, the more likely it is a solid at room temperature. The more
unsaturated a fatty acid, the more likely it will be a liquid at room temperature. Triglyceride II contains three
hydrocarbon chains which are completely saturated. This allows these three chains to pack close together in a tight
parallel arrangement, thereby increasing the amount of attractive forces between the individual chains. If there are

more attractive forces between the aliphatic chains, then it will require more energy (heat) to pull them apart. We
Copyright by The Berkeley Review

103

The Berkeley Review


Specializing in MCAT Preparation

Biology

Structure and Function in Cells and Viruses

Section vi Answers

would expect the melting temperature to be the highest for triglyceride II. Remember, we are being asked for the
order of the increasing melting point for the four triglycerides?We can immediately eliminate choice B.
We can distinguish between choices A, C, and D by looking for that triglyceride with the most sites of unsaturation
(i.e., double bonds). This turns out to be triglyceride III with 3 sites of unsaturation. The more double bonds there

are in a fatty acid, the more disorder there will be in the packing of the hydrocarbon chains. The more disorder in the
packing, the less interaction there will be between individual regions of each chain. Therefore, less energy will be
required to pull them apart. We would expect the melting temperature to be the lowest for triglyceride III. We can
now eliminate choice A.

All that remains is to locate that triglycerides with the next two highest degree of unsaturation. They are triglyceride
IV (2 sites of unsaturation) and triglyceride I (1 site of unsaturation). Therefore, the order for increasing melting
point of the four triglycerides is III, IV, I, and II. The correct choice is C.
91.

D is correct The structure of cholesterol is shown in the passage in Figure 3. Note that there is just one hydroxyl
group on the molecule. The rest of the molecule is composed of a non-polar cyclohexane and cyclopentane ring
system. Attached to the cyclopentane ring is a long non-polar hydrocarbon chain. Diets high in cholesterol increase
the blood serum concentrations of cholesterol. Because this compound is quite insoluble in water they begin to
adhere to the walls of arteries and help in the formation of plaques.
If there were an increase in the synthesis of bile salts from cholesterol, then less cholesterol would be in the blood

serum and more would be excreted in the feces. As we will learn a bit later, bile salts act to emulsify dietary lipids in
preparation for digestion. Similarly, an increase in the synthesis of steroid hormones means a reduction of
cholesterol in the blood serum. Both conditions would act to reduce the formation of plaques. Once cholesterol is
integrated into a lipid bilayer it usually stays there until the membrane is degraded. There is no (known) mechanism
to remove excess cholesterol from cellular lipid bilayers. Once the membrane is degraded, the excess cholesterol in
the membrane can contribute to plaque formation, but it is the insolubility of cholesterol in water that allows for this.
The correct choice is D.

92.

D is correct The structure of a simple lipid is shown in Figure 1 of the passage. Arachidonic acid has a carboxylic
acid group which can react with an alcohol group of glycerol to form an ester linkage. Therefore, arachidonic acid

can contribute to the formation of simple lipids. As outlined in the passage, the only difference between a simple
lipid and a complex lipid is that there is a phosphate group involved in the formation of complex lipids. There is
nothing to say that we cannot attach arachidonic acid to the alcohol group of a complex lipid. Thus, arachidonic acid
can contribute to the formation of complex lipidsas well. Finally, it was stated in the passage that arachidonic acid
is a precursor to the synthesis of prostaglandins and leukotrienes. The correct choice is D.

93.

C is correct The question states the temperature is being changed from 38 C to 25 "C. The bacterial cultures are
going from a warm environment to a cold environment. At high temperatures membranes are rather fluid; at low

temperatures they tendto be less fluid. However, it is the desire of every cell to maintain some degree of fluidity in
their membranes. This will allow for transport across that membrane. Which of the three answers in the question
lead to an increase in fluidity? If the synthesis of unsaturated fatty acids is increased, there will be less packing
between the fatty acidsidechains and hence more freedom of movement. Choice A will occur. Initially there will be
a decrease in the bacterial membranes. This is simply because the cells will not have had enough time to begin
synthesis of the components needed to maintain membrane fluidity. Choice B will occur. The synthesis of short
chain fatty acids will increase. Why? The shorter thechain, the less packing between the chains. The less packing
between the chains, the less of an attractive force holding the chains together. The membrane becomes more fluid.
Choice D will occur. Choice C will not occur because the bacterial cell does not want to decrease the fluidity of its
membrane. If the fatty acid chains are longer, there will be more attractive forces holding the chains together. This
will lead to a more solid membrane. The correct choice is C.

94.

B is correct A typical membrane phosphoglyceride is shown in the passage in Figure 2. We can hydrolyze this
compound as shown below. The hydrolysis products are glycerol, stearic acid, phosphate, and choline. What about
glucose and serine? The amino acid serine can be part of a phosphoglyceride (attached to the phosphate group like
choline). However, as outlined in the passage, glucose is not part of a phosphoglyceride simply because
phosphoglycerides are based on a glycerol backbone. Sphingolipids are based on a sphingosine backbone. In this
case, glucose would be associated with sphingolipids and not with phosphoglycerides. The correct choice is B.

Copyright by The Berkeley Review

104

The Berkeley Review


Specializing in MCAT Preparation

BlOlogy

Structure and Function in Cells and Viruses

Section vi Answers

Phosphatidyl choline

4H20 J 1Hydrolysis
Fatty Acid (Stearic acid)
H2C-0H

HO-C
o

I
HC-OH

ii
HO-C"
O
ll

H,C-OH

HO-P-OH

HO

Glycerol

0
Phosphate

Choline

Passage XV (95 - 100)


95.

Bacteriophage Lambda

D is correct The question asks why the lambda virus is incapable of infecting other bacterial species. If these
species lack appropriate cell surface proteins, lambda phage may never be able to become adsorbed, or attached, to
the surface of these cells. According to the passage, this attachment is the first step in the process of viral infection.
If it can't occur, the virus doesn't infect the cell. We can approach this problem by a process of elimination as well.
Answer choice A is incorrect because most bacterial species have circular chromosomes, not just E. coli.
Additionally, having a circular bacterial chromosome has little to do with whether the lambda phage can infect a
cell. Don't be confused by the passage, which states that the viral DNA is circular. Likewise, answer choice B is
incorrect, as we know that E. coli has a circular chromosome (as opposed to the linear ones seen in eukaryotes).
Choice C is tempting, because we know from the passage that lack of a chromosomal integration site would prevent
the lysogenic infection pathway from functioning. We cannot say, however, that this would interfere with the lytic
pathway; the virus might still be able to infect the cell and lead to lysis. The correct choice is D.

96.

A is correct If the integrase protein were defective in a certain strain of lambda phage, that strain would be unable
to enter the lysogenic infection pathway. This is because the viral DNA would never be able to integrate into the
bacterial chromosome; functional integrase is normally required for this insertion process. Therefore, this viral strain
would be able to enter only the lytic pathway, always leading to the lysis of the host cell. Choice B is incorrect,
because the integrase-defective viral strain would still be able to infect E. coli; it just wouldn't be able to adopt the
lysogenic pathway. Choice C is likewise invalid, as the lytic pathway does lead to viral DNA replication. We can
also eliminate choice D, because a lack of integrase would not prevent the virus from entering the lytic infection
pathway. The correct choice is A.

97.

C is correct The question essentially asks which statements represent evolutionarily advantageous situations for the
lambda virus. An evolutionary selective advantage is basically anything that better enables an organism to survive,
reproduce, or adapt to change. Statement I represents just such an advantage. Recall, a provirus is lambda DNA that
has integrated into the bacterial chromosome. If the host cell were damaged and later died, the lambda provirus
would also perish. Therefore, it is a selective advantage for the lambda provirus to excise itself and enter the lytic
cycle when the host cell is initially damaged. In this fashion, it can quickly replicate new viral particles and leave the
dying host cell, potentially to infect new neighboring E. coli cells. This is an evolutionary advantage. Statement II is
likewise beneficial evolutionarily. If lambda always killed its host cell immediately (i.e., via the lytic pathway), it
would soon run out of viable host cells. It would also have less chance of being dispersed to new areas. If it didn't
always kill its host cell immediately (i.e., the lysogenic pathway), it could replicate along with the bacterium and

Copyright by The Berkeley Review

105

The Berkeley Review


Specializing in MCAT Preparation

Biology

Structure and Function in Cells and Viruses

Section VI Answers

therefore be more successful at spreading to new areas. This is an evolutionary selective advantage. Statement III,
on the contrary, does not represent an advantage. If lambda DNA were almost completely resistant to mutation, it
could never evolve to meet new environmental challenges. In essence, it would be incapable of adapting to change.
This is not an evolutionary advantage. The correct choice is C.

98.

B is correct The experiment described in the question involves radioactively labeling the lambda viral DNA,
packaging this labeled DNA into a phage protein capsid, and allowing this new virus to infect an E. coli cell. The
question effectively asks where the viral DNA would be after lysogenic infection, not lytic infection (the cell does
not lyse). Recall from the passage, the lysogenic infection pathway leads to the integration of viral DNA into the
host's chromosome. In prokaryotes such as bacteria, chromosomes are located in the cytoplasm of the bacterial cell.
This is were we would detect radiation from the integrated, labeled viral DNA. Choice A is incorrect because
bacteria do not have nuclei. Choice C is invalid, because we would only see labeled viral DNA bound to ribosomes

if it were being translated. After integration via the lysogenic pathway, the viral DNA does not direct protein
synthesis and is therefore latent. Choice D is wrong, because we would only see labeled viral DNA outside the cell

if lysis, or cell bursting, had occurred, releasing new virus particles. Since the virus in question does not enter the
lytic pathway, this would not occur. The correct choice is B.

99.

C is correct A cytosolic endonuclease is an enzyme which can cleave DNA at specific sequences within the strand.
Endonucleases (also called restriction enzymes) are therefore capable of cleaving circular viral DNA. Suchcleavage
would prevent viral DNA from being either translated directly or integrated into the bacterial chromosome. Injection

of lambda DNA is a step common to both infection pathways; the DNA injected at this step is degraded by the
bacterial endonuclease. Therefore, both lytic and lysogenic pathways of infection would be inactivated. The lytic
pathway is prevented because lambda DNA is cleaved before it can direct synthesis of the protein capsid. The
lysogenic pathway is inhibited because lambda DNA is cleaved before it can either direct integrase production or
insert into the bacterial chromosome. This information leads us to eliminates answer choices A, B, and D. The
correct choice is C.

100. A is correct The question essentially requires us to know the differences and similarities between a lambda phage
which adopts the lytic pathway of infection and a lambda phage which adopts the lysogenic pathway of infection.
The question asks about the similarities between the two pathways. From the passage, we learn that the common
step to both pathways is the adsorption ofthe phage tothe bacterial cell surface and the subsequent injection of viral
DNA. This makes choice A correct; the two pathways do NOT differ in this regard. We can also proceed via a
process of elimination. Choice B can be ruled out, because the lysogenic and lytic pathways both adopt different
methods of replicating viral DNA. The lysogenic approach involves replication along with the host chromosome,
while the lytic approach involves replication independent of the host chromosome. Therefore, this answer choice

represents a way in which the two pathways differ. Choice C is likewise incorrect, because it also represents a
difference between the two pathways. While the lytic pathway is immediately lethal to the infected cell (due to
lysis), the lysogenic pathway may allow the cell to undergo multiple replications unharmed. In the same sense,
choice Dcan be eliminated. Only viruses entering the lysogenic pathway can remain latent for long periods. This is
another difference between viruses entering each of the two respective pathways. The correctchoice is A.

Copyright by The Berkeley Review

106

The Berkeley Review


Specializing in MCAT Preparation

Biology
Section VII

A.

Metabolic

Enzyme Kinetics
1.

Transition State

2.

Michaelis-Menten Equation

3.

Lineweaver-Burk Plot

4.

Enzyme Inhibition

Components
B.
His57
O

r=\G>

Ser.gc 0

H^Nl^ - H

II

-O-C-Asp102

r.-C-N-R,

Enzyme Mechanisms
1. Chymotrypsin Mechanism
2.
3.

RNA as an Active Enzyme


Inactive Enzymatic Precursors

4.

Molecular Evolution

5.

Transition-State Analog

&
Unstable tetrahedral

His57

C.

transition state

Ser,c O

:N<^N-H

O - C - Aspt

Jj /*-. Acyl enzyme

R,-C

. Nj-1 intermediate

r^ Amino product

Metabolic Molecules

1.
2.

Adenosine Triphosphate
Cofactors and Coenzymes

3.

Nicotinamide Adenine Dinucleotide

H- N- R2 */^ leaves the scene 4.

5.

Flavin Adenine Dinucleotide

Coenzyme A

Practice Passages and Answers

Berkeley

Ur-e.v.i^ew
Specializing in MCAT Preparation

Metabolic Components
Top 10 Section Goals

Be familiar with the concept of transition states.

It is important to understand the difference between the activationenergy of an uncatalyzed reaction


and that of a catalyzed reaction.

>

Understand how to read a Michaelis-Menten graph.


Don't worry about the mathematical derivation of the Michaelis-Menten equation. Instead, be sure
that you understand the components of that equation in relation to its graphic form.

Understand how to read a Lineweaver-Burk graph.


Beable to interconvertbetween the Michaelis-Menten graph and the Lineweaver-Burk graph. Be
sure that you understand the meaning of the different variables used in each.

Be familiar with the concept of enzyme inhibition.

Enzyme inhibition isa favorite topic inbiology. Know the difference between a competitive anda

noncompetitiveinhibitor and how they function in the presenceor absenceof a substrate.

Understand the basics of the chymotrypsin mechanism.


The details of this mechanismhave appeared on the MCAT before. Those who took the exam then
neededonlya basic understanding ofenzymatic mechanisms to do well. Keep trackofyourelectrons!
Be aware that not all enzymes are composed of amino acids.

Atone time, it was thought that all enzymes were proteins. This isnolonger the case, because RNA
hasbeen shown tohave enzymatic activity. Just keep the significance ofthis discovery in mind.
Be familiar with the concept of molecular evolution.
On occasion, theMCAT hashad a few questions concerning evolution. Be prepared for questions
dealing with the topicsof convergent evolution and divergent evolution.

Know the basic components and functions of the ATP molecule.


ATP is the energy currency of the cell. We will encounter thismetabolic mainstay throughout our
entire review session. Know its components and understand its uses in metabolism.

()#* Know the basic components and functions of the coenzymes WAD and FAD.

'*

NAD and FAD areboth coenzymes. They arenot enzymes. A coenzyme helpsan enzyme function
properly. NAD and FAD are probably the two best known coenzymes.

Be familiar with coenzyme A and its uses in metabolism.

Coenzyme Aisanother important coenzyme, especially inthe Krebs cycle and infatty acid metabolism.

Have some understanding of where this molecule is used in the celland why.

Biology

Metabolic Components

Enzyme Kinetics

Enzyme Kinetics
Transition State
Activation Energy St Catalysts
Chemical reactions occur at a particular set of rates. In the reaction shown in
equation (7-1) there will be a rate under specified conditions at which molecule A

can become molecule B or molecule B can become molecule A. Usually kj is


referred to as the forward reaction rate while k2 is referred to as the reverse

reaction rate. The point of equilibrium is where [Al and [B] no longer change
with respect to one another. If kj > k2, then at equilibrium [B] > [A].

ki
A

(7-1)

k2

If this is true, then why don't all reactions come to equilibrium immediately? In
order to answer this question we need to consider the energy diagram shown in
Figure 7-1. If we have two energies for A and B, we will find that at equilibrium
B will be in higher concentration. It is a bit more stable than A. It has less energy
and is therefore more stable. In order to get from A to B we have to go through a
higher energy state. This is often referred to as the transition state (TS). Do not

think of this reaction as one molecule of A and something else, but rather think of
it as a population of molecules.
TS*

"*
ofthc

Activation Energy

41

CO
o

z~\\

Uncatalyzed / /~\
Reaction

_yL^0-^\_t^

Activation Energy

Catalyzed Reaction

\. \

(Substrate)

\ \

ofthc

A
B

(Product)

Progress of Reaction
Figure 7-1
Transition States and the Effect of a Catalyst.

Within this population of molecules some will have more energy than others
simply because some have been "heated" more recently than others. They have
been in contact with something that has given them energy. The population as a
whole does not have a particular energetic state but rather a range of energetic
states. In order to make the trek up to the transition state, some proportion of the
population of molecules of species A must have enough energy to be at that
transition state. If it were a very small proportion of the population of species A,

then we would expect to see the forward reaction occurring very slowly. If we
were to look at the population of molecules represented by species B, we would
see that it would be a harder trek to go back to species A because of the greater
difference in energy between B and A. Some population of molecules from
species B, though, would be expected to have an energy equal to the difference
Copyright by The Berkeley Review

109

The Berkeley Review

Specializing in MCAT Preparation

Biology

Metabolic Components

Enzyme Kinetics

between the transition state and the average thermodynamic state of species B.
This will be different at different temperatures. At low temperatures fewer
members of the population of A and B will have enough energy to make the trek
across the transition state. This is why chemists, in order to achieve equilibrium,
often raise the temperature. If we raise the temperature, then more of the
population of species A, per unit time, will cross the barrier.

However, in biological materials raising the temperature is usually not a


possibility. For any one particular organism there is a range of temperatures
which is fairly restricted (i.e., around body temperature). A different method
must be employed and that is to use a catalyst. A catalyst is a general name for
any substance (or substances) which can provide an alternative route or an
alternative transition state. A catalyst can change the nature of the intermediate
state to some lower energy state. If the transition state (TS) is lower in terms of its
own energy, then the nature of the relative proportion of the population of A and
B that can obtain a transition state at a particular temperature is increased
(without an increase in the temperature). This is indicated in Figure 7-1.

We can think of the enzyme as being like a tunnel through a mountain (the
energy barrier). Instead of expending a lot of energy to go over the mountain, we
can go through the mountain to the other side (products) by way of the tunnel.
As our molecule A goes through the tunnel (i.e., is worked on by the enzyme) it
is converted to B. Our tunnel allows A to be converted into B. Within the tunnel,

though, molecule A goes through a high energy transition state (less stable)
before it becomesmolecule B. It turns out that one can synthesize molecules that
look like the transition states of molecules that are being examined. These
synthetic structures are called transition state analogs, and they can easily
interact with our enzyme (fit into the tunnel). Since this transition state analog
has nothing to do with A going to B, it actually impedes the process. In other
words, transition stateanalogs are excellent inhibitors of the catalytic process.Thus, enzymes are catalysts which are often employed to lower the transition
state's activationenergy. By definition, a catalyst will not alter the equilibrium of
a reaction. However, a catalyst will alter the rate of a reaction. In other words, an

enzyme accelerates the forward reaction as well as the reverse reaction by


precisely the same factor.

Michaelis-Menten Equation
Derivation St Meaning
How do enzymes carry out their catalysis of a reaction? Enzymes contain specific
regions called active sites to which a specific substrate molecule will bind.
Enzymes also stabilize the transition state and they carry out acid-base catalysis
by precisely positioning the catalytic groups of certain amino acids (e.g., Asp,
Glu, Lys, Arg, His, Ser, Cys, etc.) found within the active site pocket.
For a number of enzymes, the catalytic rate (V) changes with the substrate
concentration ([S]). In equation (7-2) the enzyme (E) combines with the substrate
(S) to form an enzyme-substrate complex (ES) with some rate ki. This ES
complex can continue on to form the product with some rate k3 or it can
dissociate to the substrate and enzyme at some rate k2-

Copyright by The Berkeley Review

110

The Berkeley Review


Specializing in MCAT Preparation

Biology

Metabolic Components

Enzyme Kinetics

k3
E + S
(7-2)

^=

=^= ES

E + P

k2

When [S] is small and the [E] is constant, then Vis essentially proportional to [S],
giving afirst order reaction. However, when [S] is large, then V is essentially
independent of [S], giving a zero order reaction. This can be seen in Figure 7-2.
The type of curve obtained is hyperbolic.

The velocity of the reaction given by equation (7-2) is shown in equation (7-3).
The only way that we can obtain the product is through the EScomplex.
V = k3[ES]

(7-3)

The total concentration of enzyme in the system in question is equal to the


concentration of the free enzyme plus the concentration of the enzyme bound
with substrate in the ES complex. This is shown in equation (7-4).

Substrate Concentration [S]

Figure 7-2
Reaction rates as a function of
substrate concentration.

(7-4)

Etotal = [E]free + Abound

In the following discussion we will consider the derivation of the MichaelisMenten equation (7-14). It is important to know how to use the Michaelis-Menten
equation but not how toderive it. The only reason the derivation is presented here
is for completeness. If you wish, you can skip this derivation and proceed
directly to equation (7-14).
We would like to describe the rate of an enzyme reaction in terms of some

parameter that we can measure. It would be nice to measure ES. Unfortunately


this is rather difficult. Consider the rate of change of IES] over any given time.
This can be seen in equation (7-5). This is simply the rate of formation of ES
minus the rate at which ES is broken down.

(7-5)

d[ES]/dt = k1LE][S] - (k2 + k3)[ES]

Leonor Michaelis and Maud Menten reasoned (circa 1913) that the concen

tration of the intermediates in a steady state process remain the same while the
concentration of the reactants and products change. In other words, d[ES]/dt 0.
This will occur when the rate of formation of the ES complex and breakdown of
the ES complex are equal. This can be seen in equations (7-6) and (7-7).
(7-6)

d[ES]/dt0 = k1[E][S] - (k2 + k3)[ES]

(7-7)

k^S] = (k2 + k3)[ES]

Rearranging equation (7-7) gives equation (7-8).


(7-8)

[ES] = [E][S]/(k2 + k3)/k1

We can now define (k2 + k3)/ki to be the Michaelis constant, Km- Substituting

this constant into equation (7-8) gives equation (7-9).


(7-9)

[ES] = [E][S]/KM

Copyright by The Berkeley Review

111

The Berkeley Review


Specializing in MCAT Preparation

Biology

Metabolic Components

Enzyme Kinetics

Now, rearranging equation (7-4) gives equation (7-10). And substitution of (7-10)
into (7-9) gives equation (7-11).

[E]free = [Eltdtal " [E%ound

(7-10)

[ES] = ([E]total - [ES])[S]/KM

(7-11)

Solve equation (7-11) for [ES]. This gives equation (7-12).

[ES] = [E]total([S]/([S] + KM))

(7-12)

Substituting (7-12) into equation (7-3) gives equation (7-13).

V = k3[E]total([S]/([S] + KM))

(7-13)

Let's define the maximal rate of a reaction as Vmax. This reaction rate can be
obtained when all the enzyme's active sites are saturated with substrate. In other

words, Vmax = k3[E]total- Substituting this expression into equation (7-13) yields
equation (7-14). This equation is the Michaelis-Menten equation and it is an
important equation in enzyme kinetics.

Michaelis-Menten Equation

(7-14)

If we plot the velocity of a reaction as a function of the substrate concentration,

we willget a hyperbolic curve (Figure 7-3). This is the sametype of curve that we
will see when we examine the binding of oxygen to myoglobin. In the case of an
enzyme, we find that when we saturate that enzyme with substrate, the enzyme
is operating at its maximal velocity (i.e., Vmax). In the case of an enzyme, when
[S] = Km/ then V = Vmax/2. In other words, the Km is equal to the substrate
concentration at which thereaction rate is half of its maximalvalue.

C-

The Km canalso be related to the rateconstants in a given reaction. Suppose we


have a situation in which k2>k3. This means that the enzyme-substrate
max

complex (ES) will have a tendency to dissociate to E and S rather than form E
and P. In this situation, Km = k2/ki. It is only when this situation is true that Km
is a measure of the binding strength of the ES complex.

>>

vmax

>

a
/V

In other words, a high Km value indicates a weakbinding of the


ES complex while a low Km value indicates a strong binding of

Substrate Concentration [S]

Figure 7-3
Michaelis-Menten kinetics showing
variation of Reaction Velocity with
Substrate Concentration.

the ES complex.

In practice it is not very convenient to estimate Vmax in the manner just


suggested. This is because as we increase the amount of substrate the solution

becomes more and more viscous (e.g., like honey).The only values that are really
much use to us are those in the area where things are first order.
In some cases we will not have a perfect hyperbola. For example, if we had a
protein that had a structure like that of hemoglobin, where each of the four
subunits has an activesite, then those active sites might act independently of one

Copyright by The Berkeley Review

112

The Berkeley Review

Specializing in MCAT Preparation

Biology

Metabolic Components

Enzyme Kinetics

another or they might influence one another. If one of those subunits was

interacting with a substrate, it could cause a changein the affinity (the Km) of the
other active sites with respect to their interaction with other substrates. Since this

frequently happensit turns out that thereis cooperativity in multimeric proteins


with more than one active site. Allosteric enzymes have a non-hyperbolic curve
(i.e., more sigmoidal) when plotted as velocity versus substrate concentration.

Lineweaver-Burk Plot
Meaning
It became of interest to try and find some way of dealing with Km that did not
involve plotting the hyperbola and then estimating the Vmax/2. Lineweaver and
Burk developed a plot which is now referred to as a double-reciprocal plot. In
this plot we graph 1/V as a function of 1/[S]. If we were to plot the hyperbola
shown in Figure 7-3 onto this graph, we would end up with a straight line. We
can obtain an equation for this plot if we take the reciprocal of the MichaelisMenten equation. This is shown as equation (7-15). Note that this equation is in
the form of y = mx + b, the equation for a straight line.

(7-15)

1 _
V

Km
.^max.

[11
LCS]J

1
^max

Lineweaver-Burk Equation

The solid dots on the Lineweaver-Burk graph are data points (Figure 7-4). If we
were to take the best fit through those data points and then extrapolate the line,
we will intercept the Y-axis at 1/Vmax. If we continue to the X- axis, we will
intercept at -1/Km- Note the region of high [S] and low [S] values. If 1/[S] is

approaching zero, then it must mean that [S] is approaching infinity. At the
intersection of the X-axis and Y-axis the [S] would be equal to infinity. This is the

point at which V became Vmax in Figure 7-3. If the line in Figure 7-4 is not
straight, it indicates that we do not have a hyperbola. If we do not have a
hyperbola, then we have an enzyme which is undergoing some type of

Figure 7-4
Lineweaver-Burk plot.

alternative reaction. This is very often a sign that we are dealing with a

multimeric protein which binds more than one molecule of substrate.

When an enzyme is completely saturated with substrate, then the number of


substrate molecules which are converted to product per unit time is referred to as
the turnover number.

Enzyme Inhibition
Reversible & Irreversible

There are two major types of enzyme inhibition: reversible and irreversible
inhibition. One example of an irreversible inhibitor is diisopropylfluorophosphate
(DIPF), a potent nerve gas. DIPF inhibits the enzymeacetylcholinesterase, which
is involved in the hydrolysis of the neurotransmitter acetylcholine to acetate and
choline. In other words, DIPF blocks cholinergic nerve impulses within the body.
It turns out that DIPF-inhibited enzymes have an unusually reactive serine residue

at the active site. Other enzymes like elastase, trypsin, and chymotrypsin all have
reactive serine residues at their active sites as well. Thus, these enzymes are
grouped into a class of enzymes called serine proteases.

Copyright by The Berkeley Review

113

The Berkeley Review


Specializing in MCAT Preparation

Biology

Metabolic Components

Enzyme Kinetics

There are two types of reversible inhibition. They are competitive inhibition
and non-competitive inhibition (of which allosteric inhibition is a subset).

E + S^LES-^-E + P
+
I

k2

Competitive Inhibition

A competitive inhibitor (Ki) usually resembles the normal substrate molecule


and can readily bind at the active site of the enzyme to form an enzymeinhibitor (EI) complex as shown in Figure 7-5.

Ki
EI

Figure 7-5
Competitive Inhibition.

Uninhibited

Figure 7-6
Competitive Inhibition "appears" to

If there is a competitive inhibitor around, it will compete with the substrate for
the active site on the enzyme. In Figure 7-5 we see that the enzyme is being
utilized in two pathways. One pathway utilizes the normal substrate and the
other pathway utilizes the competitive inhibitor. The consequence of that
competition is a decrease in the rate of catalysis of the enzyme. The rate of
formation of the product is dP/dt = k3[ES]. If some of the [ES] is removed to
become [EI], we essentially have a lower concentration of ES.
A competitive inhibitor can be overcome at high concentrations of substrate
(Figure 7-6). Initially we have a lower Vmax for a given substrate concen
tration, but as we approach an infinite substrate concentration, the
concentration of the competitive inhibitor becomes negligible and the enzyme
catalyzed reaction will once again approach the same Vmax.

When a competitive inhibitor is present it changes the apparent Km of the


enzyme. The word "apparent" is used because each enzyme has a characteristic
Km value for a given substrate. In other words, the Km of a competitively
inhibited enzyme appears to shift to a higher value. However, if we add
enough substrate, we will eventually reach the same Vmax. This can be seen in
both Figure 7-6 and in Figure 7-7.

increase KM.

Shown in Figure 7-7 is the Lineweaver-Burk representation of Figure 7-6. In


each plot the concentrationof inhibitor and enzyme is constant. Again, we find
that the Vmax remains constant while the apparent Km changes. Note that a
competitive inhibitor will increase the slope of the lines found in the
Lineweaver-Burk plot.
[I] = 2Kj

[I] = Ki

l/v
/

^-"^Uninhibited

/^^Slope = Km

l^^

vmax

Vmax
1

oo

r[S]

Km

Apparent --*
Km

Figure 7-7
Lineweaver-Burk Plot for Competitive Inhibition.

Copyright by The Berkeley Review

114

The Berkeley Review


Specializing in MCAT Preparation

Biology

Metabolic Components

Enzyme Kinetics

An example of a competitive inhibitor would be ethanol. Ethylene glycol, a


constituent of antifreeze, can be converted via a number of reactions in the body
to oxalic acid (which causes kidney damage and often results in death). One of
thosereactions involvesthe enzyme alcohol dehydrogenase.
It turns out that alcohol dehydrogenase willalsoconvert ethanol to acetaldehyde
(which can then be converted to pyruvate-more on this in a later discussion). If
ethylene glycol is ingested, all one needs to do is drink intoxicating levels of
ethanol. The end result is that ethanol will compete with ethylene glycolfor the
active site on the alcohol dehydrogenase enzyme, and since there is more ethanol
than ethylene glycol around, the enzyme will convert much more ethanol to
acetaldehyde than it will ethylene glycol to oxalic acid.

Another example of a competitive inhibitor is methotrexate, an anticancer drug.


As we will see later, rapidly dividing cancer cells require a continuous stream of
precursor molecules (called deoxyribonucleotides) for the synthesis of DNA.
Methotrexate resembles a molecule called dihydrofolate, an intermediate in the
reductive synthesis of tetrahydrofolate (Figure 7-8).
The enzyme that carries out this reaction is dihydrofolate reductase.
Methotrexate binds to dihydrofolate reductase and prevents the conversion of
dihydrofolate to tetrahydrofolate, which is not only an essential cofactor for the
synthesis of one of the bases in DNA (i.e., deoxythymidylate (abbreviated as
dTMP)) but it is also a cofactor in the synthesis of all other reactions that depend
on tetrahydrofolate (e.g., the amino acids methionine, histidine, and the purine
biosynthetic pathways).
o

ii

II

C-NH C-COO

c-nh c-coo

CH,

CH,

CH2

CH2

0
COO

coo

CH, N-H

H,N

H,N

Dihydrofolate
Reductase

Dihydrofolate (DHF)

Tetrahydrofolate (THF)
O

II

C-NHC-COO
CH
I

CH

COO

H,N

Methotrexate

Figure 7-8
Methotrexate is a Competitive Inhibitor.

Copyright by The Berkeley Review

115

The Berkeley Review


Specializing in MCAT Preparation

Biology

Metabolic Components

Enzyme Kinetics

Non-competitive Inhibition
*3
E + S -^

ES

k2

E + P

+
I

lh

Ki
EI + S

One characteristic of non-competitive inhibition is that it cannot be overcome


by increasing the substrate concentration. This means that we will have a lower

ESI

Figure 7-9
Non-competitive Inhibition.

max

w V
> vmax

Non-competitive

>

Non-competitive inhibitors do not compete at the active site of an enzyme, but


rather bind to some other site on the enzyme. Binding at this "allosteric site"
will alter the conformation of the enzyme thus giving reversible inactivation of
the catalytic site. These inhibitors can bind to both the free enzyme and the
enzyme-substrate complex (Figure 7-9).

Inhibition

0 KM
[S]

Figure 7-10
Non-Competitive Inhibition
Decreases Vmax.

apparent Vmax for the reaction. However, the Km will not change. Why?
Consider one specific type of enzyme. The non-competitive inhibitor that we add
to the reactionmixture containingthis specific enzyme will bind to the enzyme
at some allosteric site and cause a conformationalchange to alter the enzyme's
active site. This enzyme is no longer functional as a catalytic unit. However,
there are still other enzymes in the reaction solution that may not have had an
encounter with this non-competitive inhibitor. Those enzymes have active sites
that are just as active as they ever were, and they still have the same Km value

as their brothers and sisters (because the Km is a relationship between


substrate and active enzyme). Butsince their numbers have been reduced (by
non-competitive inhibition), they have a lower overall Vmax. This can be seen
in Figure 7-10.

TheLineweaver-Burk representation of the plot shown in Figure 7-10 is shown


in Figure 7-11. Note that the apparent Km remains constant while the Vmax
decreases.

[I] = 2Ki

P] = Ki

Figure 7-11
Lineweaver-Burk Plot for Non-Competitive Inhibition.

Copyright by The Berkeley Review

116

The Berkeley Review

Specializing in MCAT Preparation

Biology

Metabolic Components

Enzyme Mechanisms

Enzyme Mechanisms
In biochemistry one usually learns the mechanism for at least one of four classic
enzymes. Lysozyme is an enzyme that hydrolyzes a specific glycosidic bond in
the polysaccharide component of certain bacterial cell walls. Ribonuclease is an

enzyme that catalyzes thehydrolysis of phosphodiester bonds in RNA polymers.


Carboxypeptidase, is a zinc-containing digestive enzyme secreted by the
exocrine cells of the pancreas. This enzyme hydrolyzes the carboxyl terminal
peptide bond of protein polypeptide chains. Chymotrypsin catalyzes the
hydrolysis of either ester or peptide bonds. All of these enzymes are referred to
as hydrolases. Wateris introduced between a particular bond that had originally
been formed from the loss of water. In this section we will consider the reaction

mechanisms for chymotrypsin.

Chymotrypsin Mechanism
Chymotrypsin catalyzes the hydrolysis of either ester or peptj^e bonds. If a
peptide is hydrolyzed, the products will be an amine and an acid. If an ester is
hydrolyzed, the resulting products will be an alcohol and an acid.
How was this catalysis at the active site revealed? One of the amino acid residues
at the active site was identified using diisopropylphosphofluoridate (DIPF), an
irreversible inhibitor. It turns out that chymotrypsin has 28 serine residues, yet
only one of those residues, Ser-195, reacts with DIPF. Another of the amino acid
residues at the active site, His-57, was identified using tosyl-L-phenylalanine
chloromethyl ketone (TPCK) and a process called affinity labeling. Knowing the
polypeptide sequence of chymotrypsin, making use of the different types of
labeling techniques, and using x-ray studies, it was discovered that His-57 is not
only adjacent to Ser-195 but also to another amino acid residue, Asp-102. These
three residues, shown in Figure 7-12, form what is called a catalytic triad at the
active site of chymotrypsin.
Under physiological conditions, where the pH is about 7.4, Ser-195 will have a
pKa around 13. What this means is that for all practical purposes, Ser-195 will
still retain its hydrogen atom on the hydroxyl group of its side chain. (Remember,

when the pH of the solution is greater than the pKa of the ionizing side chain, the
predominant species is the conjugate base of the side chain, and when the pH of the
solution is less than the pKa of the ionizing side chain, the predominant species is the
acid. Anotherway to think about this is asfollows: pH > pKa, then [HA] < [A~] and if
pH < pKa, then [HA] > [A'].) Therefore, at a physiological pH of about 7.4, only

about 1 x 10"6 molecules of Ser-195 will be in the form of the conjugate base.
If Ser-195 is reacting with DIPF, there must be something that is allowing the
pKa of that serine residue to be lowered. If the pKa of serine is lowered, there will
be more of a chance of finding it in the form of the conjugate base. In this case the
serine side chain would be in the oxyanion form. When alcohols lose their
hydrogens they become alkoxide ionsand an alkoxide ion is a stronger base and

therefore a better nucleophile~and nucleophiles love to give away electrons.


It turns out that the His-57 residue is unprotonated when there is no substrate
present in the active site. Remember, histidine has a pKa of about 6.0, and since
Copyright by The Berkeley Review

117

The Berkeley Review


Specializing in MCAT Preparation

Biology

Metabolic Components

Enzyme Mechanisms

we are at a pH of about 7.4,roughly 96% of it will be unprotonated. The Asp-102

residue that is part of thistriad has a pKa of about 4.4. At physiological pH it will
essentially be in the anionic form. It therefore can act as a base and interact with
His-57, thus making His-57 an even strongerbase than it would have been if the
Asp-102 residue were not present. What this means is that His-57 can abstract a
proton from Ser-195 as the substrate comes into the active site. Once Ser-195
loses the proton from its hydroxyl function, it becomes a rather reactive alkoxide
ion. This portion of the mechanism is shown in Figure 7-12.
His57

/=*

n^:n-h

Ser195O-H

II

- OCAsp102

His57

H N

Ser195 O

/N H

OCAspio2

Figure 7-12

We have mentioned that chymotrypsin catalyzes the hydrolysis of either ester or


peptide bonds. Let's consider the hydrolysis of a peptide bond in a typical
protein substrate (Figure 7-13).
His57

0\

Ser195 O \

h n:

nh

ll

OCAsp102

R, C N R2

IK H
His57

0/
Substrate

H N

Ser195O

^NH

II

- O C Asp102

R,CNR2

O0
Unstable Tetrahderal Transition State

Figure 7-13

Copyright by The Berkeley Review

118

The Berkeley Review


Specializing in MCAT Preparation

Biology

Metabolic Components

Enzyme Mechanisms

Alkoxide ions are strong bases and quite reactive. When the Ser-195 residue is in

the alkoxide ion form it is a strong nucleophile, and nucleophiles love to seek out
carbonyl carbons and pass electrons to them. (Carbonyl carbons are partially
positively charged while the oxygen is partially negatively charged.) However,
this will place too many electrons on the carbonyl carbon. The result is that the
electrons in the double bond of the carbonylwill move to the oxygen.
Once the electrons move to the oxygen we form an unstable tetrahedral
transition state. The function of the enzyme is to stabilize this transition state.

This is accomplished by hydrogen bond interaction of the negatively charged


carbonyl oxygen with the N-H groups of the main chain Ser-195 residue and a
Gly-193 residue located in the vicinity.
At this point the electrons essentially have two paths they can follow. The first is
that they can return to where they came from (reversing Figure 7-13) or they can
proceed in such a way that there is collapse of the tetrahedral intermediate to
form the acylenzyme. This occurs with expulsion of the leaving group, which in
this case is the amino product (Figure 7-14).
His57

Ser195 O

His57

Unstable Tetrahedral

Transition State
O

Seri9 O

:n

NH

II

- OCAspiQ2

I
R,-C

[ A,

O- ^

Acyl Enzyme
Intermediate

|
r~\ Amino product
H N R2
leaves the scene

Figure 7-14

The acyl enzyme intermediate that we have just formed provides a lower energy
pathway to get from the substrate to the product. Once the amino product is
formed it is free to leave the scene. We could consider the steps that we have just
mentioned the First Act in the opera Chymotrypsin (a bit like the First Act in the
German opera Die Zauberflote by Mozart).
With the beginning of the Second Act we need to introduce water in order to
deacylate our acyl enzyme intermediate. As the His-57 function removes a
hydrogen atom from water, there will be a nucleophilic attack on the carbonyl
carbon of the acyl enzyme intermediate by the oxygen atom on the water
molecule. Again, a transient tetrahedral intermediate is formed as shown in
Figure 7-15.
Copyright by The Berkeley Review

119

The Berkeley Review


Specializing in MCAT Preparation

Biology

Metabolic Components

Enzyme Mechanisms

His57
0

2nI/:n-H'

0
II
-0C-Asp102

His57

Ser195-0

H-

N^N_h

II

--O-C-Asp102

R!-CI

<^

Unstable Tetrahedral

Transition State

Figure 7-15

The His-57 residue is now in a position to donate a hydrogen atom to the Ser-195
residue. Again, we have an electron shift. The transient tetrahedral intermediate
collapses and the acid component of the substrate that we started with is free to

leave the scene. Note that we now have the enzyme in its original starting
condition-ready to accept another substrate molecule for catalysis (Figure 7-16).
His57

H\N>N~H

Ser195 OR,- C

II

--0CAspio2

1^

I.

H
His57

Seri95OH

R.-CO

II

O C Aspio2

^>

Acid Product
can leave the scene

Figure 7-16

In the analysis of this mechanism we looked at the hydrolysis of a generic


peptide bond. However, chymotrypsin is rather selective in the peptide bonds
that it will cleave. For example, chymotrypsin will hydrolyze the peptide bond
on the carboxyl side of Phe, Tyr, and Trp, three aromatic amino acid residues.
Copyright by The Berkeley Review

120

The Berkeley Review


Specializing in MCAT Preparation

Biology

Metabolic Components

Enzyme Mechanisms

Chymotrypsin will also hydrolyze on the carboxyl side of large hydrophobic


residues like Leu, He, and Met. It turns out that the side chains of these amino

residues can fit into a special nonpolar pocket near the Ser-195 residue of the

active site. This pocket allows forchymotrypsin's specificity.

RISA as an Active Enzyme


During the early 1980s Thomas Cech and his colleagues discovered that RNA

can act asa catalytic enzyme. In the ciliated protozoan Tetrahymena thermophila, a
414 nucleotide intron is excised from a 6.4 kb ribosomal RNA precursor in the
presence of a cofactor which proved to be a guanosine (G)residue. The releaseof

this 414 nucleotide intron and subsequent splicing of the juxtaposed exons
showed that an RNA molecule can have catalytic activity. The 414 nucleotide
intron undergoes two more rounds of self-splicing, first losing a 15 nucleotide
fragment and then losing a 4 nucleotide fragment. The linear fragment ofrRNA
that is left is called L19 RNA. (The "L" stands for the fact that it is linear and the

"19" means that 19 intervening nucleotide sequences were removed since the

formation ofthe 414 nucleotide intron. Itisnow 395 nucleotides long.) L19 RNA
israther stable and can acton other substrates. As we will see later, this enzyme
is both a nuclease and a polymerase.

Inactive Enzymatic Precurors


Many enzymes areinitially synthesized in theform ofa proenzyme or zymogen.
Zymogens are inactive precursors of the active enzyme. Inorder for these enzymes
to become active, they must first undergo some type of proteolytic activation
(which is an irreversible step). As an example, consider chymotrypsin.
Chymotrypsin is the active form ofthe enzyme. The inactive form ofthis enzyme
(the zymogen) is synthesized in the pancreas and is called chymotrypsinogen.
Once chymotrypsinogen is synthesized in the pancreas it is secreted into the
intestinal tract where it is activated by a proteolytic enzyme called trypsin. The
activated enzyme, chymotrypsin, can then hydrolyze proteins found in the
intestine. If the pancreas were to become damaged, the pancreatic secretions

would rapidly become activated and digest the pancreas, giving rise to the
condition called acute pancreatitis.

There is a naturally occurring substance called ai-antitrypsin which is an


extremely effective inhibitor of trypsin and an even better inhibitor of elastase

(also called antielastase). This inhibitor prevents these digestive enzymes from
digestingthe rest of your body. For example, antitrypsin is present in the tissues
of the lung and preventselastase from digesting connective tissue proteins in the
alveolar walls of the lungs. Genetic disorders leading to a deficiency in this
inhibitor can result in a clinical condition known as emphysema. A person with
emphysema breathes much harder than the normal individual. Individuals who

smoke cigarettes also have the possibility of developingemphysema. It turns out


that cigarette smoke oxidizes a specific Met residue on antitrypsin and converts
it to methionine sulfoxide. The Met residue is essential for binding elastase. Once
Met is converted into methionine sulfoxide, elastase no longer binds to the
inhibitor.

Molecular Evolution
Two other digestive enzymes that are quite similar to chymotrypsinare trypsin
and elastase. These three enzymes are secreted by the pancreas and all have the
Copyright by The Berkeley Review

121

The Berkeley Review

Specializing in MCAT Preparation

BlOlOgy

Metabolic Components

Enzyme Mechanisms

Ser-His-Asp catalytic triad that we have been discussing. They are serine

proteases and theirmechanisms of action are also similar. It turns out that about
40% of the overall amino acid sequences in these proteins are identical. Why?
Because these three enzymes evolved (after mutation and duplication) from a
similar ancestral enzyme. The evolutionary process that allowed for these three
distinctive enzymes is called divergent evolution.

The bacterial enzyme subtilisin (isolated from Bacillus subtilus) is also a serine
protease. If you compare the sequences of amino acids in subtilisin with those in
chymotrypsin (or trypsin or elastase), you will find a remarkably different
composition. These two enzymes (probably) did not have a common ancestral
enzyme and therefore were evolutionarily independent of each other. Even
though subtilisin is a serine protease (as are chymotrypsin, trypsin, and elastase),
the amino acid residues located at the active site of the

enzyme are in

functionally different positions than the aminoacid residues at the active sites of,
say, chymotrypsin or elastase. What this means is that both the bacterialenzyme
and the mammalian enzyme have found a similar way to catalyze a particular
reaction. These two independent processes are probably due to an evolutionary
process called convergent evolution.

Transition state analogs are synthesized with the hopes that they will occupy the
active site of an enzyme. The role of an enzyme is to bind the transition state and
stabilize it. A transition state analog is thus a competitive inhibitor of the actual
reaction.

The vertebrate immune system's single most important function is to recognize


self from non-self. If a foreign substance (an antigen) enters into the body, the
immune system will recognize that substance as being foreign and begin to
synthesizeantibodies or immunoglobulins against it. Suppose the foreign antigen
that entered the body was a transition state analog. The antibody synthesized in

response to the transition state analog will bind to that analog in a very specific
way. This is exactly analogous to the way in which an enzyme would bind a
substrate. What we have done is to make the immune system produce an

"enzyme," which is the antibody. If we remove the transition state analog from
the antibody, what will be able to fit into the space left behind? How about the
substrate or the product from the reaction in which the transition state analog
was competitively inhibiting? This would suggest that such antibodies would
have catalytic activity. It turns out that they do! These synthetic antibody
"enzymes" are not necessarily as good as the naturally occurring enzymes, but
they do have enzymatic activity. This might be a way to design and synthesize
your own protein(s).

Copyright by The Berkeley Review

122

The Berkeley Review

Specializing in MCAT Preparation

Biology

Metabolic Components

Metabolic Molecules

Metabolic Molecules
Metabolism is the generalized word for all of the processes which occur inside
living organisms. Metabolism can be divided into catabolism and anabolism.

Catabolism means the breakdown of complex molecules into smaller and


simpler products accompanied by the release of energy. In most instances the

body attempts to harness this energy in the form of adenosine triphosphate


(ATP). Anabolism means to build up or become more complex. It is the
biosynthesis of small precursor molecules into larger and more complex
molecules. Entropy is reduced in this energy-requiring organizational process.

Complex
Complex

"food"

biomolecules

molecules

We can diagram these twometabolic processes as shown in Figure 7-17.

Adenosine Triphosphate (ATP)


We will first concern ourselves with catabolic processes and then after that we
will look at selective anabolic processes. In order to begin our discussion of
catabolism, we need to look at the chemical properties of ATP. As we will see,
ATP is the currency exchange in the cell. A good analogy for ATPare dollar bills',

Simple

you make them and you spend them in the course of a lifetime.

nutrient

materials

ATP is composed of adenine (a base), D-ribose (a sugar), and three phosphate


groups. Adenine and D-ribose taken together make up the molecule adenosine.
Ifweadd onephosphate to adenosine, wegetadenosine monophosphate (AMP).
Ifwe add two phosphates to adenosine, we getadenosine diphosphate (ADP).
And if we add three phosphates to adenosine, we get adenosine triphosphate
(ATP). Note the numbering of the two rings shown in Figure 7-18. The adenine
ring system (called a purine ring) has the numbers 1 though 9 as shown. The
ribose ringhas the numbers 1' through 5' (the "prime" markings areimportant).

Acid anhydride
linkages

N-Glycosidic

5'

O-P-O-P-O-P-O-CH2 -
'O

'O

ei?K

or

Adenosine is just

N-Acetal linkage

Adenine & D-Ribose

<=> D-Ribose

Phosphomonoester
linkage

AMP

V.

V.

the components of metabolism.

ii 34JU >^Adenine

O
Ij-~ O
|>' O
11
11
11

Figure 7-17
Catabolism and anabolism are

ADP

ATP

Figure 7-18
Adenosine Triphosphate (ATP).

Copyright by The Berkeley Review

123

The Berkeley Review

Specializing in MCAT Preparation

Biology

II

II

Metabolic Components

Enzyme Kinetics

The linkage between the CI1 of the ribose ring and the N9 of the adenine ring is
an N-acetal linkage. It is also referred to as an N-glycosidic linkage. The linkage
between the C5' of the ribose ring and the first phosphate group is a phosphomonoester linkage. The linkages between the first and the second phosphates
and the second and third phosphates are phosphoric anhydride linkages.

H3C- C- 0- C- CH3
Acetic Anhydride
-H20

+H20
O
ll

2 H3C-C-OH
Acetic Acid

Figure 7-19
The hydrolysis of acetic
anhydride to two molecules of
acetic acid.

A similar situation stems from organic chemistry where you may have
encountered acetic anhydride. Acetic anhydride is the anhydride between two
molecules of acetic acid as shown in Figure 7-19.

Both esters and anhydrides can be hydrolyzed with water. However, anhydride
bonds hydrolyze in a manner that releases more energy than hydrolysis of ester
bonds. If you were to hydrolyze acetic anhydride, the equilibrium constant
would favor the acetic acid products more than you would have expected had
you made it analogous to an ester where you would get back an acetic acid and
an alcohol product. One of the reasons for this is that in the acid anhydride
situation the acyl groups do not have the option of becoming stabilized by
resonance in their acid anion form. When acetic anhydride is hydrolyzed you get
two molecules of acetic acid which can be stabilized by resonance. This
stabilization of both molecules of (product) acid probably leads to this large
release of energy that we have mentioned.

Enzymes often require substances other than amino acids to carry out their
functions. These substances are called cofactors. Consider an enzyme with an
active site. This active site is isolated from the rest of the enzyme and within its

cavityare hydrophobic side chains. If you have a substrate (S)which is in solution,


it tells you that it must have some degree of polarityotherwise it would not be
soluble in that solution. There are a number of reactions that must be carried out

in an anhydrous condition (without water). Suppose you wanted to do something


to the substrate that required a non-polar environment. One way to achieve that
situation in a cell that is full of water is to get the substrate to settle down within

the pocket of the active site (which is surrounded by hydrophobic residues). If


this happens, the substrate can be converted into product.
Once the substrate settles down at the active site, what will happen? Recall that
we have a "tool kit" of 20 amino acids. Some of these amino acids are acidic (they

can catalyze a reaction by donating a proton) while others are basic (they can
accept a proton). These types of amino acids are often involved in acid/base
catalysis. There are also amino acids which can be oxidized or reduced like
cysteine. There are also amino acids with hydrophobic side chains which are
good for excludingpolar compounds such as water.
There may be other things that you would like to do to this substrate-things that
cannot be done with our 20 amino acid tool kit. For this you need some

additional apparatus at the active site. In hemoglobin that additional apparatus


was the heme prostheticgroup. Other prosthetic groups are necessary for similar
reasons. You can do things with these prosthetic groups that you cannot do with
any of the 20 amino acid side chains. Therefore, a prosthetic group at the active
site may be necessary to achieve some particular type of catalytic process. There
are a whole series of prosthetic groups that need not be bond in a covalent
manner at the active site. These are the cofactors that we just mentioned. If these

cofactors are required by an enzyme,they are called coenzymes. A coenzymeis a


Copyright by The Berkeley Review

124

The Berkeley Review


Specializing in MCAT Preparation

Biology

Metabolic Components

Metabolic Molecules

non-amino acid, non-polypeptide, non-protein substance which is required for

the activity of an enzyme. In many cases the coenzyme is bound to the enzyme in
a non-covalent manner but there are also many cases in which the coenzyme is
covalently attached. There also may be more than one coenzyme involved with a
single enzyme.

Nicotinamide Adenine Dinucleotide (NAD)o@.

Business end

NAD

One coenzyme that will become very important to us is nicotinamide adenine


dinucleotide (NAD) whose structure is shown in Figure 7-20. Note the reactive
site or "business end" of the molecule.

Where does "dinucleotide" in NAD come from? The phosphorylated adenosine


moiety of NAD is one nucleotide while the other phosphorylated moiety (the
ribose and the pyridine ring) of NAD is the other nucleotide. The "business end"
of NAD contains a pyridine ring linked to an amide. This is called nicotinamide

(or niacinamide). If we add a carboxylic acid function to the pyridine ring, we


will have nicotinic acid. This isalso referred toasniacin. Why call it niacin?
There is a carboxylic acid function in the structure of nicotinic acid, but what
aboutthe "nicotin" prefix? Tobacco was broughtback from the New World to the

Old World by a Frenchman by the name ofJean Nicot. The tobacco plant was
named Nicotiana tabacum after Nicot in his honor. Nicotiana contains a substance

called nicotine. If you were to oxidize nicotine with HNO3, you would get
nicotinic acid.

HO

OH

Figure 7-20
Nicotinamide Adenine
O

isr

Pyridine

11

11

C-OH

C-NH2

yj

Dinucleotide (NAD)

sO

Nicotinic acid

Nicotinamide

(or niacin)

(or niacinamide)

Nicotine

Figure 7-21
Nitrogen-basedring structures.

Humans have the ability to synthesize the nicotinamide (or nicotinic acid)
portion of the NAD molecule provided they have enough of the amino acid
tryptophan in their diet. Tryptophan is an essential amino acid. Once we have

produced nicotinamide the rest of the NAD molecule can be synthesized. If we


had a diet low in tryptophan, we would not be able to synthesize as much
nicotinamide and therefore would have a deficiency in theNAD coenzyme. Since
nicotinamide is an amine and it is "vital" to the diet, we would call it a "vital

amine." Today these are known as "vitamins" (from "vital amine"). If we do not
have enough of the NAD coenzyme to carry out our metabolic reactions, then we
willnot function as well. This is the basisof a vitamin deficiency.
It turns out that nicotinamide was one of the first vitamins ever discovered.

Around the same time that this vitamin was being discovered, it was also
announced that the nicotine in cigarette smoking was bad for you. The general
public had a rather hard time distinguishing between the words nicotine and
nicotinamide. Nutritionists decided that the name "nicotinamide" had to be
Copyright by The Berkeley Review

125

The Berkeley Review

Specializing in MCAT Preparation

Biology

Metabolic Components

Enzyme Kinetics

changed in order to make consumption of nicotinamide more psychologically


acceptable. So, instead of calling it nicotinamide, the same substance was
renamed niacinamide. Nicotinic acid was renamed niacin (Figure 7-21)

The coenzyme NAD is held at a specific site on the surface of the active site of an
enzyme. NAD is held there by its R group (the adenine dinucleotide portion).
When you use a screwdriver the part that engages the screw is just the head of
the screwdriver itself. Without the handle the screwdriver would be useless. The

enzyme is holding the NADcoenzyme by its handle (the R group portion which
has negative and positive charges) and positions the nicotinamide portion
toward the incoming substrate. This can be seen in Figure 7-22.

Flavin Adenine Dinucleotide (FAD)


Substrate

Figure 7-22
Enzyme/coenzyme interaction
at the business end of NAD.

In Figure 7-23a we see the structure of riboflavin (vitamin B2), a coenzyme


component. We cannot synthesize riboflavin. Instead, we must obtain it
elsewhere (e.g., our diet). Riboflavin is a component in the structure of a
molecule calledflavin mononucleotide (FMN), which is shown in Figure 7-23b.
Riboflavin has been phosphorylated to give FMN, the active coenzyme. The
bond involving the phosphate and oxygen atoms in FMN is a phosphate
monoester bond. Phosphorylation of riboflavin has increased the size of the
"handle" that we mentioned concerning NAD. FMN is another type of enzyme
that is often involved in redox reactions.

H-C-H
I

H-C-OH

(b)

H-C-OH

I
H-

H-C-OH

C- OH
I

2-

H2C-0-P03"

CH2-OH

Flavin Mononucleotide

Riboflavin

H-C-OH

CH,O-P-OP2

Flavin Adenine Dinucleotide

Figure 7-25
The Flavin Derivatives

Copyright by The Berkeley Review

126

The Berkeley Review


Specializing in MCAT Preparation

Biology

Metabolic Components

Metabolic Molecules

If we were to add a phosphate, ribose ring, and an adenine base to FMN, we


would get the coenzyme flavin adenine dinucleotide (FAD). This is shown in

Figure 7-23c. The flavin portion ofFMN and FAD is the collection ofthree rings
at the top ofthemolecule. Recall thatwe have mentioned that when you connect
a base (in this case, adenine), a ribose ring, and a phosphate together, it is
referred to as a nucleotide. The FMN molecule itself looks a bit like a nucleotide.
Thus, FMN is called flavin mononucleotide while FAD is called flavin adenine
dinucleotide.

Coenzyme A (CoA)
Another coenzyme that willbe importantto us is Coenzyme A (often abbreviated
CoA). The terminal sulfhydryl group is the active site of CoA. The rest of the
molecule can be thought of as being like a handle, similar to the handle on a
screwdriver. This is shown in Figure 7-24.

Acyl groups can be linked to the sulfhydryl function to form acyl coenzyme A
(or acyl CoA). Acommon acyl group that is linked to CoA isan acetyl moiety,
thus giving acetyl CoA. Because the AG' for acetyl CoA is-7.5 Kcals/mol, this
molecule has a high acetyl group transfer potential. In other words, acetyl groups
can be easily transferred from acetyl CoA to other molecules.

Consider CoA asshown inFigure 7-24 for a moment. Where did the components
of CoA come from? If we hydrolyze the amide bond towards the sulfhydryl
function, and then carboxylate the CH2 moiety next tothe nitrogen, we will have
cysteine. Ifwehydrolyze thesecond amide bond in from the sulfhydryl function,
and then carboxylate the CH2 moiety next to the nitrogen, we will have aspartic
acid. If we hydrolyze the first phosphoester bond closest to the sulfhydryl
function, we find a segment that is synthesized from three acetate residues. The
remainder of the molecule is just resembles ADP.

Coenzyme A
o

11

OHCH,
1 J

HS CH2-CH2-N-C CH2-CH2-N-C-C-C-CH2-0-P-0-P-0
1

CH

tf

CH3

."Business end"
y

Derived from

Derived from

cysteine by
decarboxylation

aspartic acid by
decarboxylation

O.

Derived from
3 acetates

(many steps)

'

o-p=o
1

V.

J
Y
Resembles ADP

Figure 7-24
Coenzyme A.

Copyright by The Berkeley Review

127

The Berkeley Review

Specializing in MCAT Preparation

Metabolic

Components
15 Passages
100 Questions

Passage Titles
I.
II.
III.
IV.
V.
VI.
VII.
VIII.
IX.
X.
XI.
XII.
XIII.
XIV.
XV.

Enzyme Inhibitors
Enzymes, Coenzymes, and Vitamins
Plasma Glucose Measurement

Brown Adipose Tissue


Heme Metabolism

Enzyme Kinetics I
Vitamin B12

Enzyme Nomenclature
Lysozyme Mechanism
Enzyme Kinetics II

Adenosine Triphosphate (ATP)


Protein Degradation (Ubiquitin)
Nutrient and Proximate Analysis
Niacin Experiment
Diffusion-Limited, Enzyme-Catalyzed Reactions

Berkeley
Specializing in MCAT Preparation

Questio
1 -5
6- 11
12- 17
18-23
24-29
30-36
37-43
44 - 50
51 -58
59-65
66-72
73 - 79
80-87
88-94
95- 100

Suggestions
The passages that follow are designed to get you to think in aconceptual manner about the processes
of molecular biology at the organismal level. If you already have asolid foundation in molecular biology,
many of the questions you read here will seem to be very straight forward and easy to answer. But if you
are new to the subject orifyou have not had a pleasant experience with molecular biology in the past,
some of them might appear to come from the void that spreads out beyond the Oort field atthe edges of
our solar system.

Pick a few passage topics at random. For these initial few passages, do not worry about the time. Just
focus on what isexpected of you. First, read the passage. Second, look atany diagrams, charts, orgraphs
in it. Third, read each question and the accompanying answers carefully. Fourth, answer the questions
the best you can. Check the solutions and see how you did. Whether you got the answers right orwrong,
it is important to read the explanations and see if you understand (and agree with) what is being
explained. Keep a record of your results.

After you feel comfortable with the format of those initial few passages, pick another block of
passages and try to do them in one sitting. Be aware that time is going to become important. On average,
you have about 1minute and 15 seconds to complete a question. Be creative in how you approach this
next group. If you feel comfortable with the outline presented above, fine. If not, then try different
approaches to a passage. For example, you might feel well versed enough to read the questions first and
then try to answer some of them, without ever having read the passage. Maybe you can answer some of
the questions by just looking at the diagrams, charts, orgraphs that are presented ina particular passage.
Remember, there are many effective learning styles. You need to begin to develop a format that works
best for you. Keepa record of your results.

The last block of passages might contain at least a few topics that are unfamiliar even to those who
know a good dealabout molecular biology. Find a place where the level of distraction is at a minimum.
Get out yourwatch and time yourself on these passages, either individually or as a group. It is important
to have a feel for time, and an awareness of how much is passing as you try to answer each question.

Never let a question get you flustered. If you cannot figure out what the answer is from information
given to you in the passage, or from your own knowledge base, dump it and move on to the next
question. As you do this, make a note of that pesky question and come back to it when you have more
time. When you are finished, check your answers and make sure you understand the solutions. Be
inquisitive. Ifyou do notknow the answer tosomething, look it up. The solution tends tostay with you
longer that way.(For example, what is theOortfield, anyway?)
The estimated score conversions for 100 questions are shown below. At best, these are rough

approximations andshould beused only to give one a feel for which ballpark they aresitting in.
Section VII

Estimated Score Conversions


Scaled Score
>13

Raw Score

80-100

11-12

70-79

9-10

60-69

7-8

50-59

5-6

40-49

<4

0-39

Biology

Enzyme Inhibitors

Passage I (Questions 1-5)

Competitive inhibitors resemble the substrate and bind

reversibly at the active site. They increase the Km and can


be overcome at high [S]. Uncompetitive inhibitors bind
only to the enzyme-substrate complex. They decrease the
Km and Vmax and cannot be overcome at high [S].
Noncompetitive inhibitors bind to either the enzyme or

In the case of a simple enzyme-catalyzed reaction, an


enzyme E binds a substrate S with a high degree of
affinity, proceeds through a transition state involving the
enzyme-substrate complex ES, and converts that substrate

into a product P (or products). This type of reaction is

enzyme-substrate complex. They decrease the Vmax and


cannot be overcome at high [S].

described by Equation 1:
(1)

-l

E + S

Passage I

k3

ES

-*

Many cellular enzymes are composed of more than a

E + P

single polypeptide chain. Instead of having just tertiary


structure, these enzymes have quaternary structure and
multiple binding sites that can cooperatively interact with
one another. Instead of having hyperbolic kinetics, these
enzymes show sigmoidal or S-shaped saturation kinetics.

The rate (velocity v) of this reaction is dependent not


only on the enzyme concentration [E], but also on the
substrate concentration [S] in the cell. Initially, the
velocity of the reaction is directly proportional to [E].
Metabolic reactions in the cell occur at higher rates when
more enzyme is available. However, the rate of a reaction
is generally determined by [S]. When [S] is low, the rate
of a reaction increases quite rapidly in a linear fashion.
When [S] is high, the rate of the reaction begins to
increase less rapidly until saturation of the enzyme with
the substrate is reached. At this point, the rate of the
reaction is maximal and is given the term Vmax. The

1.

The diagram shown below represents an interaction


between enzyme (E), substrate (S), and inhibitor (I):

Michaelis constant, Km, is the [S] at which the initial


E

velocity of the reaction is one-half the maximal velocity.


The curve developed from this type of saturation kinetics
is that of a hyperbola (Figure 1):

A
v max

5 "max

/"

1/v
E

2
0

[S]

follows

Which of the following reversible enzyme inhibition


graphs BEST represents the diagram shown above?
[Note: Curve C represents the control.]

1/[S]

Figure 1
and

Figure 2

classical

Michaelis-Menten

kinetics

expressed in Equation (2):


(2)

Vmax [S]
Km + [S]
0

1/[S]

l/[S]

1/[S]

1/[S]

Since it is difficult to measure Vmax accurately from a


direct plot of v as a function of [S], the reciprocal of
Equation (2) is used to generate a straight line of the form
y = mx + b. A double reciprocal plot (Figure 2) allows for
a more accurate determination of both Vmax and Km-

The inhibition of an enzyme-catalyzed reaction can be


analyzed in terms of Michaelis-Menten kinetics. The three
primary types of reversible inhibition are competitive,
uncompetitive, and noncompetitive.

Copyright by The Berkeley Review

131

The Berkeley Review


Specializing in MCAT Preparation

Biology
2.

Enzyme Inhibitors

Passage I

the Michaelis constant (Km)?

The Krebs cycle requires eight enzymatic steps, and


in eukaryotic cells it occurs in the matrix of the
mitochondrion. One rate-controlling step involves

A.

It is the substrate concentration at Vmax/2.

the conversion of isocitrate to oc-ketoglutarate:

B.
C.

It characterizes substrate-enzyme interaction.


It is equal to Vmax/2.

D.

It is not altered by noncompetitive inhibitors.

Which of the following is NOT a characteristic of

5.

Enzyme

Isocitrate + NAD+(excess)
oc-Ketoglutarate + NADH + H+ + C02
Three reaction mixtures are examined:

3.

What is the initial velocity of a reaction catalyzed by

the enzyme urease, if the concentration of urea in


solution is 2.5 x 10"l Km?
A.
B.
C.
D.

Mixture 1: Substrate and NAD+.

Mixture 2: Substrate, NAD+, and 2 mM ADP.


Mixture 3: Substrate, NAD+ and 5 mM ADP.

0.20 Vmax
0.25 Vmax
0.75 Vmax
0.80 Vmax

The initial reaction velocity (v) as a function of


substrate concentration [S] of each mixture is

plotted. Based on this analysis one might conclude


that:

4.

Three separate enzyme-catalyzed reactions are run

*>/ v >/

with the same substrate and enzyme at a constant

enzyme concentration. The only difference between


the three reactions is the type of inhibitor used

'8

Reaction 1:

No inhibitor added.

Reaction 2:

Competitive inhibitor added in


increasing concentrations.
Noncompetitive inhibitor added in
increasing concentrations.

Reaction 3:

if/

during catalysis.

Substrate concentration

B.

ADP increases the Km of the enzyme.


ADP is an allosteric inhibitor of the enzyme.

C.

a high [NADH]/[NAD+] ratio stimulates the

A.

The data from each reaction was plotted on the


graph shown below:

enzyme.
D.

a high [ATP]/[ADP] ratio inhibits the enzyme.

The point that is common to both Reaction 1 and


Reaction 2 is:
A.
B.
C.
D.

Point A.
Point B.
PointC.
Point D.

Copyright by The Berkeley Review

132

The Berkeley Review

Specializing in MCAT Preparation

Biology

Enzymes, Coenzymes, and Vitamins

Passage II (Questions 6-11)

Passage II

Active
site

Nicotinic acid is also referred to as niacin. The amide

of nicotinic acid is called nicotinamide, a component of


the coenzymes nicotinamide adenine dinucleotide (NAD)
and nicotinamide adenine dinucleotide phosphate
(NADP). NAD and NADP are loosely bound to a class of
enzymes called dehydrogenases and can easily move
from one enzyme to another. They are involved in the
transfer of hydrogens and electrons in oxidation-reduction
reactions.

The only oxidation step in glycolysis is catalyzed by a


dehydrogenase enzyme. The proposed mechanism for this
reaction is outlined in Figures 1-4:

HCOH

Product ch.opo?"
Figure 4

A niacin deficiency leads to the disease pellagra, a


condition characterized by dementia, dermatitis, and
diarrhea. Meat, fish, beans, and nuts are good sources of
niacin. Very little niacin is found in either milk or eggs.

Riboflavin (vitamin B2) has a complex isoalloxazine


ring system showing conjugation of multiple bonds
(Figure 5). In general, the more conjugated bonds a
molecule has, the longer the wavelength at which the

CH,OPO,

molecule absorbs light.

Figure 1
Oxidized
r isoalloxazine

H,C

\>r^

Active

nh

ring

site

H,C

l2H+

Reduced

H,C

2e-

11

HCOH

ES Complex 1

NH

isoalloxazine

ring

2-

CH2OPO,

H,C

N
k

Figure 2

Figure 5

Riboflavin is a component of two closely related


coenzymes, flavin adenine dinucleotide (FAD) and flavin
mononucleotide (FMN). These coenzymes are covalently
bound to a class of enzymes called dehydrogenases, and
they participate in the transfer of hydrogens and electrons

NAD /""JK,

in oxidation-reduction reactions.

A vitamin B2 deficiency is rare, but it can occur during


pregnancy, in growing children, and during times of
stress. If a deficiency does occur, it usually occurs with
pellagra. Milk, eggs, liver, and meat are good sources of

HC OH

1
2ES Complex 2 CH,OPO,

Figure 3

Copyright by The Berkeley Review

vitamin B2.

133

The Berkeley Review


Specializing in MCAT Preparation

Biology
6.

Enzymes, Coenzymes, and Vitamins

Passage n

11. In solution, succinate is oxidized to fumarate by the


enzyme succinate dehydrogenase.

Electrons on the nitrogen atom of the imidazolium

ring of histidine (His) in Figure 2 ultimately end up


on the:

A.
B.
C.
D.

FAD

carbonyl oxygen of nicotinamide.


nitrogen of the nicotinamide ring.
sulfur atom of cysteine (Cys).
carbonyl oxygen of the substrate.

coo
i

FADHr
FA1

coo
i

CH,

CH

CH,

Succinate

coo

dehydrogenase

coo

Succinate

7.

Phosphodiester
Peptide
Mixed acid anhydride

D.

Amide

Fumarate

The dehydrogenase enzyme contains a covalently

What is the name of the linkage at Q in Figure 4?


A.
B.
C.

bound prosthetic group, flavin adenine dinucleotide


(FAD). Oxidized FAD absorbs light in the visible
region of the electromagnetic spectrum at 460 nm.
Based on the reaction shown above and the graph
shown below, the color change during the course of
the reaction is from:
Green Red

Blue

aLbsiogrpthnom(paxeirfcun)t
oOwl

cone

cone

100 r

8.

The sulfhydryl group of Cys loses its hydrogen atom


between Figure 1 and Figure 2. This is best
explained by the need for the sulfhydryl group to
become:

A.

a nucleophile, attack the aldehyde carbon of

B.

the substrate, and form an acyl thioester bond.


an electrophile, attack the aldehyde carbon of

C.

a nucleophile, attack the aldehyde carbon of

ii

CH

S\

1 V
1 s

cone

/
*

/ *

s
C8
U

the substrate, and form a hemiacetal.


400

the substrate, and form a hemiacetal.

D.

600

700

an electrophile, attack the aldehyde carbon of


the substrate, and form an acyl thioester bond.

9.

500

Wavelength (nm)
A.

blue to red.

B.
C.

yel ow to colorless.
yel ow to blue.

D.

colorless to red

The BEST dietary source for precursors to the


dehydrogenase enzyme would be:
A.

eggs.

B.
C.
D.

meat.
beans.

fish.

10. All of the following oxidation-reduction pairs could


be found at the active site of a dehydrogenase
enzyme, EXCEPT:
A.
B.
C.
D.

reduced NAD; reduced cysteine.


reduced NAD; reduced product.
oxidized NAD; reduced cysteine.
oxidized NAD; oxidized cysteine.

Copyright by The Berkeley Review

134

The Berkeley Review

Specializing in MCAT Preparation

Biology

Plasma Glucose Measurement

Passage HI (Questions 12-17)

Passage m

12. Which of these is the structure of glucono-8-lactone,


a product of Reaction 1?

Researchers often need to measure plasma glucose


levels for reports on clinical experiments. A plasma
glucose analyzer is a machine containing reagents and
enzymes that allows rapid analysis of plasma glucose
levels. A blood sample is introduced into the analyzer,

A.

B.
.H

scx
I

and a probe determines the glucose concentration. The


structure of the probe is shown in Figure 1:

H-C-OH

H - c--OH

HO-C- H

HO- C--H
I

I
H-C-OH

H- C--OH

H-C-OH

H - c --OH

Platinum

CH2OH

Reaction 2

o-c-o0

C.

Reaction 1

D.
CH20H

HOH2C

Silver

))O.
o

Ch2OH

Cellulose Acetate
Membrane
OH

Immobilized
Oxidase

Polycarbonate
Membrane

13.

O-Ring

Figure 1

The first reaction occurs on the membrane containing


immobilized glucose oxidase. Glucose oxidase catalyzes
the following reaction:

P-D-Glucose + Q2

The glucose analyzer could be used to follow


changes in plasma glucose over time. Which of the
following graphs indicates how a person's blood
glucose would change over the three hours
following a meal?
B.

A.

Glucono-8-lactone +H2 Q

8
o

Reaction 1

The second reaction occurs at the platinum electrode

and is shown below:

H202

*-

12

12

Time Following A Meal

Time Following A Meal

(hours)

(hours)

2H + 02 + 2e

D.

C.
Reaction 2

A thin cellulose filter blocks chemical compounds


with molecular weights above 200 between Reaction 1
and Reaction 2. The electrons produced at the electrode
are linearly proportional to the H2O2 concentration, and
therefore to the concentration of the substrate. The current

required on the silver electrode to keep the electrode pair

neutral is recorded as electron flow. Electron flow is

detected and reported by the probe to a computer, which


amplifies and converts the electrical signal into a plasma
glucose concentration.
Copyright by The Berkeley Review

135

12

12

Time Following A Meal

Time Following A Meal

(hours)

(hours)

The Berkeley Review

Specializing in MCAT Preparation

Biology

Plasma Glucose Measurement

17. The function of the platinum electrode in this probe

14. The user of the glucose analyzer first generates a

is to act as:

standard curve like the one shown below. The

computer then has an internal reference for


interpolating the actual glucose samples from the
subject. If the electron reading were 1250, what
glucose concentration would be displayed on the
computer screen of the glucose analyzer ?

100

120

140

160

Passage ID

A.
B.

an anode.

a cathode.

C.

an oxidizing agent.

D.

an oxidase.

200

Blood Glucose (mg/dL)


A.
B.

C.
D.

15.

110 mg/dL
220 mg/dL
330 mg/dL
440 mg/dL

How would the measurement of the plasma glucose


concentration change, if a technician accidentally
added hydrogen peroxide to the plasma sample
instead of saline buffer?

A.

B.

The glucose reading would be erroneously


high.
The glucose reading would not be changed by

C.

The glucose reading would be erroneously

D.

The glucose reading would be zero, because


the hydrogen peroxide would destroy the
glucose.

this addition.
low.

16.

What is the purpose of the cellulose filter between


the immobilized oxidase and the platinum electrode?
A.

B.

The filter blocks hydrogen peroxide.


The cellulose allows passage of many
oxidizable substances, so mat the readings are
accurate.

C.
D.

The cellulose blocks many other oxidizable


substances, so that the readings are accurate.
The filter protects the immobilized enzyme.

Copyright by The Berkeley Review

136

The Berkeley Review

Specializing in MCAT Preparation

Biology

Brown Adipose Tissue

Passage IV (Questions 18-23)

19.

White adipose tissue (WAT) is the most common fat


storage medium in adults and children. The small
amounts of brown adipose tissue (BAT) between the
shoulder blades and along the nape of the neck of
newborn infants is metabolically important for their

Passage IV

The most abundant adipose tissue is called "white,"


although in humans and many other animals, it is
actually yellow, due to stored carotenoids. What
provides the brown color in brown adipose tissue
(BAT)?

survival.

The primary difference between the two tissues is that

A.

Ribosomes in the cytosol

B.

DNA in the nucleus

C.
D.

Cytochromes in the mitochondria


Bacteria in the lysosomes

BAT has several times the mitochondria of WAT. Also,


the inner mitochondrial membranes in BAT are more

permeable to protons than those of WAT. The reactions of


oxidative phosphorylation, particularly the generation of
ATP by ATP synthetase, depend on an inner
mitochondrial membrane that is impermeable to protons.

20.

In the case of BAT, this means that oxidative

phosphorylation is uncoupled, so that fewer ATP


molecules are made when protons are pumped out of the
mitochondria. This uncoupling is caused by a protein
called thermogenin that is present in the inner membrane.
Thermogenin allows the passage of protons back into the
mitochondrial matrix, bypassing the ATP synthetase. This
uncoupling produces beneficial effects for the infant.

Norepinephrine first binds to the beta-adrenergic


receptors and then acts through a second messenger
system to cause changes inside the adipocyte. This
sequence of events is represented in the following
diagram:

Activated

Hormone

Extracellular

receptor

The adipocytes in BAT are controlled by extensive


sympathetic enervation. When stimulated, these nerves
release norepinephrine directly at the adipocytes.
Norepinephrine stimulates lipolysis and oxidation of the
liberated fatty acids. Through oxidation, both NADH +
H and FADH2 are produced. These give rise to the
protons that are pumped across the mitochondrial
membrane. The intracellular metabolic pathways and their
enzymes are not significantly different between BAT and

G protein

GDP
Intracellular

WAT.

GTP

In the diagram shown above, which of the following


intracellular events follows the binding of

norepinephrine to the p-adrenergic receptor?

18. Which of the following would result from the

A.

The hormone is transported into the cytosol.

B.

The production of the Py subunit is increased.

C.

Adenylate cyclase is inhibited.

D.

Concentrations of cAMP increase.

uncoupling of fat oxidation and ATP production?


A.
B.
C.
D.

The production of heat


A local cooling effect
Conservation of body fuel stores
A lowered resting metabolic rate

21. What is the principal storage component of an


adipose cell in white adipose tissue?
A.
B.
C.
D.

Copyright by The Berkeley Review

137

Fatty acids
Triglycerides
Phospholipids
Glycerol

The Berkeley Review

Specializing in NCAT Preparation

Biology
22.

Brown Adipose Tissue

Passage IV

What role can BAT play in glucose homeostasis for


the whole body?

23.

A.
B.

BAT providesglycerol for gluconeogenesis.


BAT provides glucosefor gluconeogenesis.

C.
D.

BAT contains glycogen for glycogenolysis.


BAT provides fatty acids for gluconeogenesis.

Thermogenin, the uncoupling protein, is probably


MOST similar in structure to:

A.
B.
C.

hemoglobin.
chymotrypsinogen.

D.

Na-K pump.

ATP-ADP synthetase.

Copyright by The Berkeley Review

138

The Berkeley Review


Specializing in MCAT Preparation

Biology

Heme Metabolism

Passage V (Questions 24-29)

25.

Passage V

The conversion of biliverdin to bilirubin can be


described as a:

The ability to transport oxygen throughout the body


results from the oxygen-binding capability of hemoglobin
(Hb). In particular, it is the heme prosthetic group of Hb
that carries out this essential function. The prosthetic

A.
B.
C.
D.

group consists of a porphyrin ring bound to a central iron


atom, which in turn directly binds oxygen.

In a disease of porphyrin anabolism, the enzyme

one-electron oxidation.
one-electron reduction.
two-electron oxidation.
two-electron reduction.

26. In the absence of uroporphyrinogen synthase,


uroporphyrinogen I (uro I) is synthesized in place of
the biologically active uroporphyrinogen III (uro

uroporphyrinogen synthase is deficient and a non


functional porphyrin skeleton is formed. As a result,
irregular erythrocytes are made and eliminated. This
disease, congenital erythropoietic porphyria (CEP), is

HI).
coo

believed to be transmitted in an autosomal recessive


fashion.

The breakdown of the heme prosthetic group produces


the end product bilirubin, and the first step involves the
cleavage of an alpha-methane bridge to form biliverdin.

ooc

This reaction involves the release of carbon monoxide.

Biliverdin is then converted into bilirubin, a reaction

0
coo

ooc

outlined in Figure 1. In the liver, bilirubin is attached to


two glucuronate residues, and the complex is secreted into
and is released along with bile. Studies show that bilirubin
is an anti-oxidant and has the ability to eliminate
hydroperoxy radicals. For this reason, it is postulated that
evolution has selected bilirubin as the end product of
heme degradation.

Urol
ooc

coo

NADPH

+ H+

Biliverdin

NADP"1

LA

Bilirubin
ooc

Figure 1

coo

ooc

Uro in
ooc

Conversion of uro I into uro HI requires:


24.

an isomerase, converting a symmetric

The breakdown of heme to bilirubin takes place in


the:

B.

A.

liver.

B.
C.
D.

spleen.
pancreas.
gall bladder.

Copyright by The Berkeley Review

C.
D.

molecule into an asymmetric molecule.


an isomerase, converting an asymmetric
molecule into an asymmetric molecule.

a kinase, converting a symmetric molecule into


an asymmetric molecule.
a kinase, converting an asymmetric molecule
into a symmetric molecule.

139

The Berkeley Review


Specializing in MCAT Preparation

Biology

Heme Metabolism

Passage V

27. Which of the following statements is TRUE

regarding the heme prosthetic group?


A.

Carbon monoxide has a greater affinity for Hb


than for an isolated heme group.

B.

Oxygen has a greater affinity for Hb than does


carbon monoxide.

C.

A small percentage of Hb can be expected to

D.

Hb is known to exhibit a noncooperative


binding mechanism.

be bound to carbon monoxide.

28.

Based on information in the passage, which of the


following statements is FALSE?
A.
B.

Persons suffering from CEP should exhibit a


decreased blood viscosity.
Persons suffering from CEP exhibit a retarded
oxygen transport system.

C.

D.

Persons suffering from CEP exhibit a lowered


arterial p02Persons suffering from CEP should exhibit a
decreased resistance to blood flow.

29.

A man heterozygous for CEP has a child with a


woman who is homozygous dominant for CEP.
What is the probability that their first son will show
the disease phenotype?
A.
B.
C.
D.

0%
25%
50%
100%

Copyright by The Berkeley Review

140

The Berkeley Review

Specializing in MCAT Preparation

Biology

Enzyme Kinetics I

Passage VI (Questions 30-36)


In

Enzymes can also undergo one of three types of


reversible inhibition. Competitive inhibitors resemble the
substrate and bind to the active site of the enzyme, thus
preventing the substrate from binding to that active site.
The initial tendency of lowering the velocity at which the
enzyme converts substrate to product can be overcome at
high substrate concentrations. Noncompetitive inhibitors
can bind to either the free enzyme or the enzymesubstrate complex. This action also lowers the rate at
which substrate is converted to product. Uncompetitive
inhibitors bind only to the enzyme-substrate complex,
thus lowering the rate at which substrate is converted to
product. Unlike competitive inhibition, uncompetitive
inhibition and noncompetitive inhibition cannot be
overcome at high substrate concentrations.

1913 Leonor Michaelis and Maude Menten

proposed that reactions catalyzed by enzymes proceed in


two steps. First, an enzyme (E) rapidly binds a substrate
(S) to form an enzyme-substrate complex (ES). Second,
the enzyme-substrate complex is converted to product (P)
in a rate-determining step. The rates of enzymatically
catalyzed reactions, in comparison to uncatalyzed
reactions, can easily exceed a factor of a million. Since an
enzyme is a catalyst, it does not alter the equilibrium of a
chemical reaction.

kl

E + S -^^- ES

k3

Practice Passage VI

^ E + P

k2

Almost all known enzymes are proteins, and all of the


amino acid residues found in proteins are L-amino acids
or (S)-amino acids. The exception is that L-cysteine is
designated as (R)-cysteine. The actions of enzymes can be

30.

regulated, and they offer a high degree of specificity and


catalysis towards a substrate or reactant molecule. For
example, enzymes readily degrade polypeptides
synthesized from L-amino acids, but not from D-amino
acids.

Trypsin is a digestive enzyme generally referred to


as a peptidase, because it hydrolyzes polypeptides.
The scissile peptide bond to be cleaved in the
polypeptide is on the carboxyl side of the amino
acid residues arginine (Arg) and lysine (Lys).
Which of the following peptide bonds is cleaved by
trypsin?

When a substrate binds to the active site of an enzyme,

it usually does so by non-covalent interactions. Typical


binding forces involve hydrogen bonding, hydrophobic

ii

H,N - C- C- N -

interactions, electrostatic interactions, and van der Waals


interactions. The active sites of enzymes generally form

C- COO

II

CH,

OH-

II

OOC- C - N - C - C- NH,

"I

II

CH3

H,C

CH2

clefts or crevices in the molecule. Those crevices are lined

CH2

with specific amino acid residues that help to stabilize the

I
CH,

transition state of the substrate and allow catalysis to

Alanine (Ala)

CH2
CH2

II

CH2

occur. Even though different enzymes can have

CH,
I

specificities for different side chains next to the peptide

NH3

NH3

bond to be cleaved, they can have similar active sites.

Lysine (Lys)

Enzymes like chymotrypsin, trypsin, elastase, and

subtilisin are referred to as serine proteases, because they


have a highly reactive serine residue at their active site.

H,N -

The presence of active site serine residues can be


determined by using an irreversible inhibitor like
diisopropylphosphofluoridate (DIPF) as a chemical label.

CH,

C- C- N -

C COO

I
CH2

II
H

CH,

II

OOC- C - N - C - C- NH,
II

CH2
CH2

CH2
I

CH2

in

CH2

rv

NH
I

NH

F
i

(H3C)2HC- 0 - P- O - CH(CH3)2
H,N

An

NH,

H,N

NH,

Arginine (Arg)

DIPF

A.
B.

DIPF reacts only with serine residues at the active site


by forming a diisopropylphosphoryl complex with the
enzyme. This irreversible inhibitor has a similar action on
acetylcholinesterase, a serine esterase involved in the
hydrolysis of the neurotransmitter acetylcholine to acetate

I only
II only

C.

I and HI

D.

IV only

and choline.

Copyright by The Berkeley Review

141

The Berkeley Review

Specializing in NCAT Preparation

Biology
31.

32.

Enzyme Kinetics I
34.

The action of the nerve gas diisopropylphosphofluoridate (DIPF) on acetylcholinesterase and


serine proteases depends upon:
A.

allosteric inhibition.

B.
C.
D.

competitive inhibition.
phosphorylation of ADP.
phosphorylation of the active site.

Practice Passage VI

The lysozyme enzyme has two amino acid


residues, aspartate 52 and glutamate 35, at the
catalytic site with pKa values of approximately 4.0
and 6.0, respectively. If these are the only ionizable
groups involved in catalysis, then the velocity of
the reaction varies with pH according to which
graph?

In the following graph, if Curve I represents the


reaction velocity (V) as a function of substrate
concentration (S) plot for an enzyme, then Curve II
represents the reaction of the same substrate in the
presence of a fixed concentration of:
max

33.

A.

an irreversible inhibitor.

B.
C.
D.

a competitive inhibitor.
an uncompetitive inhibitor.
a noncompetitive inhibitor.

Of the four compounds tested at the same


concentration, the MOST effective competitive
inhibitor of the common enzyme is:

A.
B.
C.
D.

pH

A.
B.
C.
D.

Compound I.
Compound n.
Compound HI.
Compound IV.

Copyright by The Berkeley Review

142

Graph I.
Graph II.
Graph m.
Graph IV.

The Berkeley Review


Specializing in MCAT Preparation

Biology
35.

Enzyme Kinetics I

Practice Passage VI

At a pH of about 5, which of the amino acid

residues in the active site of the lysozyme enzyme


provides the general base catalysis and which
provides the general acid catalysis?
A.

Aspartate 52 provides the general acid


catalysis, because its side chain P-carboxyl
group is about 91% deprotonated at a pH of 5.
Glutamate 35 provides the general base
catalysis, because its side chain y-carboxyl
group is about 91% protonated at a pH of 5.

B.

C.

Glutamate 35 provides the general acid


catalysis, because its side chain y-carboxyl
group is about 9% deprotonated at a pH of 5.
Aspartate 52 provides the general base
catalysis, because its side chain P-carboxyl
group is about 9% protonated at a pH of 5.

Aspartate 52 provides the general base

catalysis, because its side chain p-carboxyl


group is about 91% protonated at a pH of 5.
Glutamate 35 provides the general acid
catalysis, because its side chain y-carboxyl
group is about 9% deprotonated at a pH of 5.

D.

36.

Glutamate 35 provides the general base


catalysis, because its side chain y-carboxyl
group is about 9% deprotonated at a pH of 5.
Aspartate 52 provides the general acid
catalysis, because its side chain p-carboxyl
group is about 91% deprotonated at a pH of 5.

According to the following graph, the Vmax of the


enzyme reaction is:

1/[S]
A.

3/2

B.

C.

!/2
2/3

D.

Copyright by The Berkeley Review

143

The Berkeley Review

Specializing in NCAT Preparation

Biology

Passage VII

Vitamin Bi2

Passage VII (Questions 37-43)

37.

Vitamin B12 is a large cobalt-containing molecule that


has a ring structure similar to heme, with Co as the central

Based on the reaction shown in Figure 1, what type


of enzyme is methylmalonyl-CoA mutase?
A.

ion instead of Fe. Vitamin B12 participates in only two


known reactions in the human body, catalyzed by the

enzymes methylmalonyl-CoA mutase and homocysteine

Oxidoreductase

B.

Hydrolase

C.
D.

Transferase
Isomerase

methyltransferase.

38.

Methylmalonyl-CoA

e
coo

mutase

COO

HCCH,

I
CH2

0=CSCoA

CH,

0=1 Cr~ S CoA

Methylmalonyl CoA

In the enzyme methionine synthetase, both B12 and


tetrahydrofolate (THF) are cofactors. If B12 is low,
then THF is trapped as methyl-THF at this enzyme
and is not recycled. This is called the methyl-THF
trap. How can B12 status affect folate status?
A.

Succinyl CoA

B.

Figure 1

C.
D.

coo

I
-C-NH,

Homocysteine
methyltransferase

COO

I
H C NH3

CH,

CH2

CH,

CH2

I 2

A primary B12 deficiency can lead to


secondary folate deficiency.
A primary folate deficiency can lead to
secondary B12 deficiency.
Folate and B12are independent of each other.
B12 is required to synthesize folate.

39.

The production of IF often decreases with age. To


prevent B12 deficiency in older adults, which of the
following would be the BEST treatment?

SH

I.
Intramuscular injection of B12
II. Increase dietary B12
HI. Increase dietary Co

CH,

Figure 2

A.
B.
C.
D.

B12 is synthesized by anaerobic microorganisms. In


the diet, it is found in animal products, particularly in the

I only
II only
I and H only
I and III only

meat and milk of ruminants, because their first stomachs

contain microorganisms that synthesize B12 for the


animal. Complete vegetarians need to include either a
microbiological or a synthetic source of B12 in their diets.

40.

B12 is absorbed in the ileum of the small intestine.

Intrinsic factor (IF), a glycoprotein, is produced in the


stomach. In the intestinal lumen, IF binds cobalamin, and

the complex is taken up by receptors in the ileum of the


small intestine.

A deficiency of IF leads to B12 deficiency, even if the


diet is rich in B12. Signs of a B12 deficiency include:
increased urinary methyl malonic acid, neurological
damage due to abnormal membrane lipids, and

Bacteria in the human colon produce B12. How does


this production affect B12 status in the individual?

A.

Colonic B12 is readily absorbed in total


vegetarians.

B.

Colonic B12 is not absorbed, because is it

C.
D.

distal to the ileal receptor.


Colonic B12 decreases production of IF.
Colonic B12 decreases activity of methyl
malonyl CoA mutase.

macrocytic anemia.

Copyright by The Berkeley Review

144

The Berkeley Review

Specializing in MCAT Preparation

Biology

Vitamin B12

Passage VD

41. Why is increased methyl malonic acid (MMA) in the


urine a sign of B12 deficiency?

coo
1

H-C-CHj
0=C-0H

A.

B.

C.
D.

42.

The blocked metabolic pathway to succinyl


CoA pushes the reactants into the MMA
pathway.
B12 is a competitive inhibitor of the MMA
pathway.
MMA is a by-product of the methionine
synthetase reaction.
MMA results from breakdown of surplus IF.

Which of the following statements regarding


methionine (Met) is FALSE?
A.

Met is an essential amino acid.

B.

Met has a sulfur-containing side chain.

C.

The carbon skeleton of Met can be used in

D.

gluconeogenesis.
Met is the precursor of Tyr, a nonessential
amino acid.

43. Why is IF resistant to digestion in the stomach?


A.

Proteins are not digested by gastric enzymes.

B.

IF is protected by nondigestible surface

C.
D.

carbohydrates.
IF inhibits the activation of pepsin.

IF is not resistant to digestion in the stomach.

Copyright by The Berkeley Review

145

The Berkeley Review

Specializing in MCAT Preparation

Biology

Enzyme Nomenclature

Passage Vm

The substrate-binding site of a protein enzyme is


called the active site. Lining the active site are specific

Passage VDI (Questions 44-50)

Enzymes are catalysts that are quite specific in their

amino acid side chains, which attract the substrate and

choice of reactants (substrates) and in the reactions they

reaction in either the forward or the reverse direction.

enhance catalysis. If the pH of the catalytic environment


is optimal, the rate of reaction tends to be maximized. The
optimal operating pH of an enzyme therefore depends on
the pKa values of the amino acid side chains (Table 2) at

Catalysts have no effect on the position of equilibriumof

the active site.

catalyze. They are able to lower the energy barrier to a


reaction, thereby increasing the rate (velocity) of the
the reaction.

Table 2. pKa Values of SelectIonizableGroups

There are thousands of different types of enzymes that

catalyze a wide variety of reactions. Many of these


enzymes have common names that are descriptive of their
functions and they are derived from the principal reactant
involved in the catalytic process. The suffix -ase is

0>COOH

r>NH3+

Side Chain

pKal

v^a

P1^

Arginine

1.8

9.0

12.5

Aspartic acid

2.0

10.0

3.9

Cysteine

1.8

10.8

8.3

Glutamic acid

2.2

9.5

4.1

Histidine

1.8

9.2

6.0

Lysine

2.2

9.2

10.8

Tyrosine

2.2

9.1

10.1

a-Amino Acid

attached to the common name to indicate an enzyme.


Urease, for example, increases the rate of urea hydrolysis.
o
ii

H2NC-NH2

+ H20

Urease

NH3 + C02

Urea

Some enzymes have names that do not end in -ase.


Pepsin and trypsin, for example, are proteolytic enzymes.
In order to assign a name unambiguously to an
enzyme, the Enzyme Commission (EC) of IUPAC places
each enzyme into one of six enzyme classes (Table 1),
based on the type of reaction they catalyze. Within each

44. Using the expression AG" = -2.3 RT log Keq,


determine the change in the free energy for the
reaction shown below, if the concentration values of

of these six classes are subclasses and sub-subclasses.

the reactants and products at equilibrium is 10

Each enzyme is assigned a specific, four-integer EC


number, along with a common name and a systematic

moles.

name.

Table 1. Enzyme Classification


Enzyme Class

Type of Reaction Catalyzed

Oxidoreductase

Oxidation-reduction

Transferase

Transfer of functional groups

Hydrolase

Hydrolytic cleavage

Lyase

Addition of groups to double

A.
B.
C.
D.

- 2.3 RT

+2.3RT
-4.6RT
+4.6RT

45. In the Krebs cycle, fumarate is converted to malate


by the enzyme fumarase.

bonds or the reverse

0
coo

coo

Isomerase

Transfer of groups within


molecules to give isomeric

CH
II
CH

forms

Ligase

H20

COO

Bond formation coupled with


ATP hydrolysis

Fumarate

HO-C-H

CH,

coo

Malate

This enzyme is a:

Enzymes can exist in alternate forms called


isoenzymes. Lactate dehydrogenase (LDH), for example,
has two different types of subunits designated as M and
H. LDH with M subunits is found primarily in muscle
tissue, while LDH with H subunits is found primarily in

A.
B.
C.
D.

heart tissue.

Copyright by The Berkeley Review

Fumarase

146

lyase involved in a hydration reaction.


hydrolase involved in a hydrolysis reaction.
lyase involved in a hydrolysis reaction.
hydrolase involved in a hydration reaction.

The Berkeley Review


Specializing in MCAT Preparation

Biology

Enzyme Nomenclature

46. The first step in glycolysis is the conversion of


glucose to glucose-6-phosphate by the enzyme

49. The active sites of different enzymes can contain


unique catalytic groups. The properties of these

hexokinase.

groups are pH-sensitive, and the initial rates

generally show bell-shaped curves.

oo-

' /*"

'

O-p-o-P-O-AMP

Q
H^C

II
o

I
0-P = 0

"8

H-jC

Hexokinase

HO

ADP

OH

Glucose

ho

13

0\ |0H

r
5

OH

10

11

12 13

PH

Glucose-6

phosphate

47.

Passage Vm

Hexokinase is a member of a class of enzymes

Which of the following amino acid pairs BEST


represents the catalytic group for the curve shown

called:

above?

A.
B.

ligases.
hydrolases.

C.
D.

transferases.
oxidoreductases.

A.
B.
C.
D.

Hexokinase also has the ability to transfer the yphosphoryl group of ATP to water, but at a rate
50.

which is about 40,000 times slower than its transfer

to the C-6 hydroxyl of glucose, even though the


nucleophilic properties of the C-6 hydroxyl group
and water are similar. The large difference in rate
can be explained by all of the following EXCEPT:
A.
B.

Histidine and aspartic acid


Cysteine and tyrosine
Arginine and lysine
Lysine and cysteine

The isoenzymes of LDH can be resolved by using


electrophoresis. In the polyacrylamide gel shown
below, Lane 1 represents one type of LDH
isoenzyme, while Lane 3 represents another type:
Lane 1

Lane 2

Lane 3

the introduction of a large conformational


change in the enzyme by glucose.
the reduction of active site polarity by the
exclusion of water.

C.
D.

the presence of water in the active site at the


positionoccupied by the C-6 hydroxyl group.
the increased nucleophilicity of the C-6
hydroxyl of glucose and the increased
electrophilicity of the y-phosphoryl group of

Origin

ATP.

Equal amounts of the two isoenzymes are mixed


together. The subunits are next dissociated from one
another and then allowed to reassociate randomly as
indicated in Lane 2. This indicates that LDH is a:

48. The second step of the glycolytic pathway involves


the conversion of an aldose sugar into a ketose
sugar. Enzymes that catalyze this type of reaction
are called:

A.
B.
C.

dimer.
trimer.

A.

transferases.

D.

pentamer.

B.

isomerases.

C.
D.

hydrolases.
ligases.

Copyright by The Berkeley Review

147

tetramer.

The Berkeley Review

Specializing in MCAT Preparation

Biology

Lysozyme Mechanism

Passage IX (Questions 51-58)

Passage IX

Main Chain of Lysozyme Enzyme

Glu 35 I

The cell wall polysaccharide of bacteria is composed


of two types of sugars, N-acetylmuramate (NAM) and Nacetylglucosamine (NAG). These sugars are joined to

O"

CH2OH

CH2OH

each other by a glycosidic linkage.

Lysozyme is composed of 129 amino acids, linked


together through peptide bonds to form a protein that
displays a-helical and (3-sheet regions in its tertiary
structure (Figure 1). This enzyme, which is found in

OH

Rings A-B-C,
NAM

17 N ^ngF

NAG

n-h
_

0=C

human saliva and lacrimal fluid, and in hen egg whites, is


an antimicrobial substance that aids in the hydrolysis of a

N-H
i

o0

0=C

Asp 52

specific substrate bearing a NAM-NAG sugarlinkage.

Main Chain of Lysozyme Enzyme

Figure 3

a-helix

Main Chain of Lysozyme Enzyme


Glu 35
0'

CH2OH

P-sheet

Lysozyme
Rings A-B-C,

Figure 1

NAM

The proposed mechanism for the catalytic hydrolysis


of bacterial cell walls by lysozyme is outlined in Figures
2-5. In this mechanism, a portion of the substrate
containing six glycosidic sugar residues fits into the active
site of the enzyme at positions labeled A, B, C, D, E, and
F. Two catalytic groups, glutamic acid 35 (Glu 35) and
aspartic acid 52 (Asp 52), reside close to and on opposite
sides of the glycosidic linkage to be cleaved. The
neighboring environments of Glu 35 and Asp 52 are quite

n-h
i

0 = C
i

CH,

Asp 52
Main Chain of Lysozyme Enzyme

Figure 4

different from each other.


Main Chain of Lysozyme Enzyme

Main Chain of Lysozyme Enzyme

Glu 35

Glu 35
O"

O"

O- H

CH2OH

CH2OH

CH,OH

O-H

OH

RingF

Rings A-B-C,

Rings A-B-Q
NAM

N-H

0=C

NAG

NAM

N-H

'

N-H

0 = C

0=c

CH,

O-R

CH,

CH,

Asp 52

Asp 52

Main Chain of Lysozyme Enzyme

Main Chain of Lysozyme Enzyme

Figure 5

Figure 2

Copyright by The Berkeley Review

148

The Berkeley Review

Specializing in MCAT Preparation

Biology
51.

Lysozyme Mechanism

Lysozyme works BEST on bacteria that are:

55. The pKa values for the side chain carboxyl groups
on aspartic acid and glutamic acid are usually cited
as 3.9 and 4.1, respectively. Analysis of lysozyme's
active site indicates that the pKa of Asp 52 is still

I.
Gram-positive.
H. Gram-negative.
m. Missing their cell walls.
A.
B.
C.
D.

Passage IX

about 3.9, but the pKa of Glu 35 is now about 6.6.


Glu 35 shows a 2.5 fold increase in pKa, because:

I only
II only
m only
II and HI only

A.

the carboxyl group of Asp 52 is located in a


polar environment.

B.

the ionized carboxyl group of Asp 52


destabilizes the protonated carboxyl group of
Glu 35.

C.
D.
52. The configuration of the linkage between rings D
and E in Figure 2 is:
A.

cc(l->4).

B.

P(4-l).

C.

oc(4->l).

D.

P(l->4).

56.

the carboxyl group of Glu 35 is located in a


nonpolar environment.
the carboxyl group of Glu 35 is located in a
polar environment, where it is stabilized by
hydrogen bonding.

In order to establish clearly which bond is being


cleaved by lysozyme, enzymatic hydrolysis using

water labeled with 180 is used. This heavy isotope


of oxygen is found attached to sugar residue D at
the:
A.

reference carbon.

B.

anomeric carbon.
C-5 carbon.
C-6 carbon.

C.
D.

53. Which of the following general classes of enzymes


BEST represents lysozyme?

57. According to the proposed mechanism outlined in


the passage, lysozyme is involved in acid catalysis.

A.

Hydrolase

B.
C.

Transferase
Oxidoreductase

This mechanism involves:

D.

Ligase

A.

noncovalent catalysis coupled with heterolytic

B.

covalent catalysis coupled with heterolytic

C.

bond cleavage.
noncovalent catalysis coupled with homolytic

D.

covalent catalysis coupled with homolytic

bond cleavage.

bond cleavage.

54. Amino acid charge distribution at the active site of

bond cleavage.

an enzyme is arranged to:


A.

destabilize the transition state in order to

58. Based on the representation of lysozyme shown in


Figure 1, glutamic acid would be expected to be

increase the rate of catalysis.


B.

stabilize the transition state in order to

found in the:

decrease the rate of catalysis.


C.

destabilize the transition state in order to

I.

decrease the rate of catalysis.


D.

stabilize the transition state in order to increase

the rate of catalysis.

A.
B.
C.
D.

Copyright by The Berkeley Review

active site.

II. a-helical regions.


Ml. interior of the protein.

149

I only
II only
I and II only
I and m only

The Berkeley Review

Specializing in MCAT Preparation

Biology

Enzyme Kinetics D

Passage X

The turnover number of an enzyme is the number of


substrate molecules processed per enzyme per second
when the enzyme is completely saturated with substrate.
This value is equal to k3 and is sometimes called the
catalytic constant, kCat- A measure of an enzyme's
catalytic efficiency can be obtained from the ratio of

Passage X (Questions 59-65)

Enzyme activity depends not only on temperature and


pH, but also on the available concentrations of substrates
[S] and inhibitors [I]. As [S] in an enzyme-catalyzed
reaction increases, so does the rate of catalysis and the
formation of products P. At a constant enzyme

kcat/KM- Table 1 lists some values for kcat and Km.

concentration [E], the relationship between the initial


reaction rate (velocity v) and [S] describes a hyperbola

Table 1. Enzyme and Substrate Parameters

(Figure 1).

max

Enzyme (andsubstrate)

KM(M)

W5-1)

Acetylcholine esterase

9.5 x 10 "5

1.4xl04

2.6 x 10 "2

4.0xl05

6.6x10 A

1.9xl02

5.0x10 "6

8.0 x 10 2

3.0x10 A

5.0X10"1

Ribonuclease
(Cytidine 2',3' cyclic phosphate

7.9 x 10 "3

7.9 x 10 2

Urease

2.5 x 10 "2

l.OxlO4

(Acetylcholine)

Carbonic anhydrase

>

(Bicarbonate)

8 ^

Chymotrypsin
max

(N-Acetyltyrosine ethyl ester)


Fumarase
(Fumarate)

Pepsin
(Phe-Gly)

Km

[S]

(Urea)

Figure 1

When an enzyme reacts with a substrate, it forms the


enzyme-substrate complex ES, which then breaks down to
reform the enzyme and release the product. This reaction
is governed by individual rate constants ki, k2, and k3.

Rate constant ki has limits between 108 to 109 M'V1.

ES

=^=

59.

When the substrate concentration is very much


lower than the Michaelis constant, the Michaelis-

k2

Menten equation reduces to:

The relationship between the initial reaction velocity


and [S] was examined during the early 1900's by Leonor
Michaelis and Maude Menten. They concluded that the
initial reaction velocity v can be given by:
(1)

A.

v =

Km
B.

[S]
Km + [S]

V =

KM + [S]

max

C.

v =

Vmax [S]

KM

where [S] is the initial substrate concentration, Vmax is the

maximal velocity of the reaction under saturating [S], and


Km is that [S] where the reaction operates at one-half its
maximal velocity. The lower the value of Km, the higher
the affinity the enzyme has for its substrate. Km is called
the Michaelis constant and Equation (1) is called the
Michaelis-Menten equation.

Copyright by The Berkeley Review

T max

D.

v =

'

Vmax L^J

[S]

150

The Berkeley Review


Specializing in MCAT Preparation

Biology
60.

Enzyme Kinetics II

When the substrate concentration is very much


higher than the Michaelis constant, the MichaelisMenten equation reduces to:

63.

V =

B.

v = Vmax.

C.

v =

D.

V =

By taking the reciprocal of the Michaelis-Menten

equation, a straight line can be graphed on a double


reciprocal plot. This graph (shown below) more
accurately represents the parameters of the curve in

[S]
KM + [S]

A.

Passage X

Figure 1.

i/v

[Si
[sr
VmaxtS]
[S]
-3

The Km of the enzyme with these kinetic data is:

reduces to:
Vmax

i\.

2Km
B.

-1

l/[S]

61. When the substrate concentration is equal to the


Michaelis constant, the Michaelis-Menten equation

-2

y 2VmaX

A.
B.
C.

+0.5.
- 2.0.
+2.0.

D.

+5.0.

Km

62.

C.

v = 2Vmax.

D.

v _ Vmax
2

64.

'

A.
B.
C.
D.

The relationship between [E], [S], [ES], and [P] can


be expressed graphically in terms of steady-state

enzyme kinetics. Which of the following graphs


BEST represents this relationship?

\[S]

[ES],

\jE]

isy^

65.

zs

ca

a
>
U

Carbonic anhydrase and chymotrypsin


Acetylcholine esterase and carbonic anhydrase
Fumarase and acetylcholine esterase
Chymotrypsin and pepsin

B.

A.

Which of the following pairs of enzymes have


achieved a state of virtual catalytic perfection?

<u
u
c

1PJ j

[E]

[ES],
[E]

2.4 x 108 M^s*1, which suggests that it:

x:

Time

Time

The enzyme triose phosphate isomerase catalyzes


the conversion of glyceraldehyde-3-phosphate into
dihydroxyacetone phosphate in both yeasts and
humans. This enzyme has a reported efficiency of
A.

evolved to near maximum efficiency early in


its evolutionary history and has changed very

B.

evolved to near maximum efficiency late in its

litde since then.


C.
c
o
a

evolutionary history and has changed very

py^

\[S1

little since then.

C.

during its evolutionary history, because it is

c
a>
o
c

JES]y

found in distinctly different species.

did not evolve to near maximum efficiency

[E]

D.

Time

Copyright by The Berkeley Review

did not evolve to near maximum efficiency

during its evolutionary history, because of its


inability to change.

Time

151

The Berkeley Review

Specializing in MCAT Preparation

Biology

Adenosine Triphosphate (ATP)


67.

Passage XI (Questions 66-72)

Adenosine triphosphate (ATP), shown in Figure 1,


serves as the primary energy intermediary between the

higher-energy phosphate compounds and the lowerenergy phosphate compounds. Other energy-rich
nucleoside triphosphates that function in this capacity are
guanosine triphosphate (GTP), cytidine triphosphate
Table 1 indicates ATP's energetically central position.

The hydrolysis of ATP to ADP (adenosine diphosphate)


and Pi (inorganic phosphate), or to AMP (adenosine
monophosphate) and PPi (pyrophosphate), provides
energyfor many endergonic reactions in the body,such as

O If

D.

It is unaffected by the cessation of ATP


production.
It slows all metabolic activity and eventually

69. Which compound would release the MOST heat


energy after the phosphorylation of ADP?
A.
B.
C.
D.
AG' (kj/mol)

Phosphoenolpyruvate
1,3-Bisphosphoglycerate
Acetyl phosphate
Phosphocreatine
PPi - 2 Pj
ATP - AMP + PPi
ATP - ADP + Pj

Glucose-1-phosphate
Fructose-6-phosphate
Glucose-6-phosphate
Glycerol-3-phosphate

70.

-61.9

-43.1
-43.1
-33.5
-32.2

71.

-30.5

-20.9
-13.8
-9.2

72.

PPi is very stable due to electrostatic


interactions between its phosphate residues.

compounds can be phosphorylated exergonically by

II.

ATP?

More energy is released than in the hydrolysis


of ATP to ADP.

ID. PPi is insoluble in the cytosol of the cell.

Phosphoenolpyruvate
Glucose-1-phosphate
Acetyl phosphate
Pyrophosphate

Copyright by The Berkeley Review

Phosphoanhydride bond
Phosphoester bond
O-Glycosidic bond
Peptide bond

When ATP is hydrolyzed to AMP and PPi, which of


the following statements must be TRUE?
I.

Based on the data in Table 1, which of the following

Glycolysis
Citric acid cycle
Protein synthesis
Oxidative phosphorylation

What type of chemical bond is indicated by the


arrow in Figure 1?
A.
B.
C.
D.

-13.8

Phosphoenolpyruvate
1,3-bisphosphoglycerate
Acetyl phosphate
Phosphocreatine

Which of the following metabolic processes does


NOT generate a net yield of ATP or GTP?

A.
B.
C.
D.

-49.4

Table 1. Standard free energies of phosphate


hydrolysis of some compounds.

A.
B.
C.
D.

It continues regular metabolic processes


without change by switching to a higherenergy phosphate compound.
It continues regular metabolic processes using

dies.

Figure 1

66.

Active transport
Passive transport

GTP.

O P OP O PO CH2
II
II
II A

Compound

C.
D.

C.

?H

Osmosis
Diffusion

B.

NH2

A.
B.

A.

muscle contraction and transport of ions against


concentration gradients.

Which of the following cellular transport processes


requires ATP?

68. When a compound is used to inhibit ATP production


in the cells of an organism, what happens to that
organism?

(CTP), and uridine triphosphate (UTP).

Passage XI

152

A.
B.
C.

I only
II only
H and D3 only

D.

I, H, and m

The Berkeley Review


Specializing in MCAT Preparation

Biology

Protein Degradation (Ubiquitin)

Passage XII (Questions 73-79)

Passage XII

73. Why do lysosomal enzymes have pH optima close to


5?

Proteins are degraded inside cells by selective and


nonselective means. Lysosomes are used for nonselective
proteolysis. These membrane-bound organelles contain

A.

So that they will be active when released to the

about fifty hydrolytic enzymes, including several types of


proteases. The lysosomal enzymes have pH optima of

B.

about 5. Lysosomes fuse with vacuoles inside the cell and

C. To degrade basic proteins more efficiently


D. To degrade acidic proteins more efficiently

cytoplasm

So that they will be inactive when released to


the cytoplasm

hydrolyze the contents, thereby recycling intracellular


components and digesting foreign particles.

Conversely, the ATP-dependent proteolytic system is


very selective. The molecule ubiquitin, a 76-amino acid
protein, binds to lysine residues on condemned proteins.
The DNA sequence coding for ubiquitin is highly
conserved throughout eukaryotes.

74. What is meant by saying that a DNA sequence is


highly conserved in eukaryotes?

An enzyme system called the ubiquitin-conjugate


degrading enzyme (UCDEN) recognizes the ubiquitin

flags and destroys the labeled protein. The ubiquitin


system is used to destroy abnormal proteins and short
lived enzymes that are at control points in metabolic
pathways. As shown in Table 1, the N-terminal residue of
the protein determines how quickly it is ubiquitinaled and
destroyed.

Long-lived enzymes

Half-Life

Cytochrome c

150 hours

A.

The sequence varies widely among all

B.

eukaryotes studied.
The sequence differs only among phyla of
eukaryotes studied.

C.

The sequence is virtually identical in all

D.

eukaryotes studied.
The sequence is completely identical in all
eukaryotes studied.

A protein in the electron-transport chain

GIyceraldehyde-3-phosphate
dehydrogenase (GAPDH)

130 hours

A protein required for glycolysis


Aldolase

75.

118 hours

A protein required for glycolysis

N-Terminal residue (stabilizing)


Methionine (Met), Serine (Ser),

> 20 hours

Alanine (Ala), Threonine (Thr),

A.
B.
C.
D.

Valine (Val), Glycine (Gly)

Short-lived enzymes
Tyrosine aminotransferase

Based on information given in the passage and in


Table 1 about its role at metabolic control points,
which of the following proteins is MOST likely to
be modified quickly by ubiquitin?

Hall-Life
120 minutes

Cytochrome c
Tyrosine aminotransferase
Ornithine decarboxylase
Glyceraldehyde-3-phosphate dehydrogenase

A protein required for amino acid catoblism

RNA Polymerase I

78 minutes

A eukaryotic protein that synthesizes rRNA

Ornithine decarboxylase
A protein required for polyaminc synthesis

12 minutes

N-Terminal residue (destabilizing)


Isoleucine (He), Glutamic acid (Glu)
Tyrosine (Tyr), Glutamine (Gin)
Phenylalanine (Phe), Leucine (Leu)
Aspartic acid (Asp), Lysine (Lys)
Arginine (Arg)

= 30 minutes

76.

= 3 minutes

= 2 minutes

Table 1. Half-lives of long-lived and short-lived cellular


enzymes as a function of their (potential) N-tcrminal residues.

Copyright by The Berkeley Review

Of the following choices, which N-terminal amino


acid provides the MOST stability for an enzyme?

~ 10 minutes

153

A.
B.

Lysine
Glycine

C.

Glutamate

D.

Leucine

The Berkeley Review


Specializing in MCAT Preparation

Biology
77.

Protein Degradation (Ubiquitin)

Passage XII

79. The antimalarial drug chloroquine, pictured below,

Under what physiological condition would


lysosomal protein degradation bethe highest?

penetrates the lysosome and accumulates as a weak


base inside. What effect does this have on the

A.
B.
C.
D.

lysosomal degradation of proteins?

During a fast
Following a meal
During exercise
During pregnancy

N - CH - (CH,), N(C,H5)2
H

CH3

Chloroquine
78.

The following diagram indicates the steps involved


in the attachment of ubiquitin to a protein:

A.

B.
C.
o

ll

D.

UbiquitinCO + El-SH

The rate of protein degradation is decreased.


The rate of protein degradation is increased.
Chloroquine inhibits the proteases by
modifying their active sites.
Chloroquine activates the proteases by
modifying their active sites.

LATP
AMP + PPj
o

ll

Ubiquitin C S E,

E2-SH

El-SH
o
II

UbiquitinCSEj

Condemned protein (CP)


3
E2-SH
o

ll

Ubiquitin C N Lys- CP

What is the purpose of the reaction involving ATP


in Step 1?

A.

B.
C.

D.

The hydrolysis of ATP to AMP and the


subsequent hydrolysis of PPj provides the
energy of three high-energy phosphate bonds.
The hydrolysis of ATP to AMP provides the
energy of two high-energy phosphate bonds.
The hydrolysis of ATP to AMP and the
subsequent hydrolysis of PPj provides the
energy of two high-energy phosphatebonds.
The hydrolysis of ATP to AMP provides the
energy of three high-energy phosphatebonds.

Copyright by The Berkeley Review

154

The Berkeley Review

Specializing in MCAT Preparation

Biology

Nutrients and Proximate Analysis

Passage XIII (Questions 80-87)

80.

Nutrients are grouped into six classes: protein,


carbohydrate (starches and sugars), and lipid, all of which

Passage xm

The dehydration step of proximate analysis is often


by lyophilization (freeze-drying). The sample is
frozen, and the water is evaporated from the frozen
sample in a vacuum. What is this process called?

provide energy; and water, vitamins, and minerals, which

are noncaloric. Protein and carbohydrate each provide 4


kcal/gram, and triglyceride (a type of lipid) provides 9

A.
B.

kcal/gram.

Melting
Vaporization

C.

Condensation

D.

Sublimation

Average daily water intake for an adult is about 1.5 to

2 liters from liquids, foods, and metabolic water.


Vitamins are divided into water-soluble and fat-soluble

vitamins. The water-soluble vitamins are thiamin (Bi),


riboflavin (B2), niacin (B3), pyridoxine (B6), folate,
cobalamine (B12), folate, biotin, pantothenic acid and

81.

When vitamins and minerals are combusted, which


compounds produce the gas phase and which
produce the ash?

ascorbic acid (vitamin C). The fat-soluble vitamins are

vitaminA (retinol), vitamin E (alpha-tocopherol), vitamin

A.

Both are in the gas phase.

D (dihydroxy-cholecalciferol), and vitamin K. Minerals

B.

Both are in the ash.

includethe major or macrominerals: calcium, phosphorus,


chlorine, potassium, sulfur, and magnesium; and the trace
minerals: iron, zinc, iodine, copper, manganese, fluoride,

C.

Vitamins are in the ash, while minerals are in


the gas phase.

D.

Vitamins are in the gas phase, while minerals


are in the ash.

chromium, selenium, molybdenum, cobalt, vanadium, tin,


silicon, and nickel.

Vitamins and minerals function in specific ways in


the body. Some are important as cofactors or coenzymes,
while others maintain acid-base balance, promote nerve
and muscle activity, or maintain fluid and electrolyte

82. Which of the following is NOT a trace mineral?


A.

balance. Vitamins are organic molecules, while minerals


are inorganic ions and atoms. Many of the B vitamins
function as cofactors in the reactions of energy

Magnesium

B.

Zinc

C.

Manganese

D.

Iodine

metabolism, such as niacin in NAD. Vitamin C works as

an antioxidant to keep metals in a reduced form in some


enzymes. The fat-soluble vitamins are also important.
Vitamin A is crucial for vision. Vitamin D helps regulate
calcium homeostasis. Vitamin K is required for blood
clotting.

83. A portion of chicken is analyzed by the Kjeldhal


technique and found to contain 5 grams of nitrogen.
How many grams of protein are in it?
A.
B.
C.
D.

The nutrient content of food can be determined by


proximate analysis, which involves a series of chemical
analyses. First, the sample is dehydrated, and the mass of
water is calculated by difference. Next, a lipid extraction
with organic solvent is performed to calculate the mass of
lipid. A Kjeldhal analysis determines nitrogen content.
The mass of protein is determined based on the
assumption that a protein contains 16% nitrogen. Fiber is
the residue unaffected by acid and base hydrolysis.
Vitamins and minerals are separated by combustion,
resulting in a gas phase and an ash. Finally, carbohydrate
is calculated as the total weight minus all other nutrients

31 grams protein
0.8 grams protein
80 grams protein
3.1 grams protein

84. If you were to analyze a new food, which


components would you test to determine the calorie
content?

and fiber.

I.

Carbohydrates

II.

Vitamins

m. Lipids

Copyright by The Berkeley Review

155

A.
B.
C.

I and III only


II only
II and DH only

D.

I, n, and m

The Berkeley Review


Specializing in MCAT Preparation

Biology

Nutrients and Proximate Analysis

Passage xm

85. Whichof the following statements is FALSE?


A.

Excess water-soluble vitamins are excreted in

B.

Fat-soluble vitamins require binding proteins


for transport in the blood.

C.

Ascorbic acid is a fat-soluble vitamin.

the urine.

D. An antagonist for Vitamin K could lead to


longer blood-clotting times.

86.

If a person consumes 1800 kcals, and if 30% of the


energy in that food is from triglyceride, how many
grams of triglyceride areconsumed?
A.
B.
C.
D.

87.

540 grams
60 grams
67 grams
54 grams

About 250 mL of water is produced through


metabolism and is called metabolic water. Which of

the following reactions would NOT produce


metabolic water?

A.
B.

The last stepof the electron-transport chain


Synthesis of a protein from amino acids

C.

Glycogen production

D.

The breakdown of triglycerides

Copyright by TheBerkeley Review

156

The Berkeley Review

Specializing in MCAT Preparation

Biology

Niacin Experiment

Passage XIV (Questions 88-94)

Passage XIV

88. If none of the fatty acids were reesterified into

triglycerides in the adipocyte, what ratio of glycerol

Niacin is a B vitamin that is required in milligram


quantities by the body as a cofactor in many of the
enzymes of energy metabolism. It is also used in gram

to fatty acids would be seen in the blood?

quantities, at pharmacological levels, to lower levels of


cholesterol and triglycerides in the blood. This beneficial

effect is believed to result from reduced fatty acids

A.

2:3

B.
C.
D.

1:2
1:3
1:4

leaving the adipose tissue and entering the liver. Once in


the liver, free fatty acids are believed to influence the
hepatic assembly of lipoproteins.

89. For the turnover measurements, the isotopic


enrichment of fatty acids and glycerol extracted

O
ll

from the blood must be measured. If the isotopically


enriched molecules used were [1,2,3,4-I3C]palmitate and [l,l,2,3,3-2H]-glycerol, then what

C - OH

Niacin

instrument would be used for measurement of

isotopic enrichment?

A research group is testing the following hypothesis


about niacin's mechanism of action:

A.

Scintillation counter

Hypothesis

B.
C.
D.

Mass spectrometer
Geiger counter
UV spectroscope

Inside adipocytes, niacin inhibits reesterification of


fatty acids that were liberated by hormone-sensitive
lipase.

90. To extract glycerol from the blood, a student uses a


series of cation and anion exchange resins. A portion
of deproteinized blood is adjusted to pH 7.0. The
blood is applied to a cation resin then to an anion
resin. Following a water rinse through the exchange
resins, where is the glycerol?

Experiment I

Six hyperlipidemic and hypercholesterolemic subjects


are first studied with isotopically labeled palmitate and
glycerol to quantify turnover of free fatty acids and
glycerol. The isotopically labeled compounds are used as
tracers to study the entry of fatty acids and glycerol into
the blood from adipose stores. The subjects then receive
niacin for 2 months, building up to 3 grams per day. After
this period, the subjects repeat the turnover study with
labeled palmitate and glycerol. The following table
indicates the data obtained from this study:

Blood sample

Cation exchange
resin

Phase

Fatty acid turnover

Glycerol turnover

(pmol/kg/min)

(jjmol/kg/min)

3.4

1.2

2.1

1.3

Anion exchange j
Pre-

niacin

resin

Postniacin

Collection

Table 1: Turnover of fatty acids and glycerol (umol/kg/min) in


subjects before and after niacin intervention.

tube

A.

The glycerol is attached to both the cation and

B.

The glycerol is attached to the cation resin.


The glycerol is attached to the anion resin.
The glycerol is in the collection tube.

the anion resins.


C.

I).

Copyright by The Berkeley Review

157

The Berkeley Review


Specializing in MCAT Preparation

Biology
91.

Niacin Experiment

Passage XIV

If fatty acids are released from the adipose tissue,


repackaged in the liver, and sent back for storage in
the adipose tissue, what metabolic effect does this
cause?

A.
B.
C.
D.

92.

It uses energy througha substratecycle.


It creates energy through a substratecycle.
Fatty acid cycling damages the liver.
Fatty acid cycling damages the adipose tissue.

Which of the following statements is supported by


the data in Table 1?

A.
B.

The pre-niacin period involved no lipolysis.


Niacin treatment promoted both lipolysis and

C.

The pre-niacin period did not involve

reesterification.
reesterification.

D.

Niacin treatment increased reesterification of

free fatty acids in the adipose tissue.

93.

If a new experimental drug were studied that


increased fatty acid reesterification to 100%, what
effect would this have on the body?

A.

The body would switch to ketone bodies for


peripheral tissueenergy metabolism.

B.

The brain and nervous tissue would have no

C.

ready supply of fuel.


The muscle and heart would have no ready
supply of fuel.

D.

94.

The body would switch to cholesterol for


peripheral tissue energy metabolism.

In a separate experiment to learn more about


lipolysis, the blood of a group of subjects is studied
before and after being given caffeine. Caffeine is a
stimulator of hormone-sensitive lipase, an enzyme in
the adipose tissue that hydrolyzes triglycerides into
free fatty acids and glycerol. What effect would
caffeine have on glycerol and fatty acid
concentrations in the blood?

A.

Glycerol and fatty acid concentrations would


both increase.

B.

Glycerol concentrations would increase, and


fatty acid concentrations would decrease.

C.

Glycerol concentrations would decrease, and


fatty acid concentrations would increase.

D.

Glycerol and fatty acid concentrations would


both decease.

Copyright by The Berkeley Review

158

The Berkeley Review

Specializing in MCAT Preparation

BlOlOgy

Diffusion-Limited, Enzyme-Catalyzed Reactions

Passage XV (Questions 95-100)

97.

Passage XV

Based on information in the passage, which of the


following statements is TRUE?

In metabolic pathways, the product of one reaction


often serves as a reactant for a subsequent reaction. For
this reason, many enzyme-catalyzed reactions need to be
driven in one direction. To obtain a negative Gibbs free
energy and thereby drive a reaction in one direction, many
processes are coupled to the hydrolysis of ATP into ADP
and Pj. The ratio of ATP to ADP is high, which ensures
an available cellular source of energy.

A.

Many enzymes have inefficient catalytic


properties.

B.

Diffusion-limited reactions are faster in the

cytosol than in the nucleus.

C.

Multienzyme complexes should increase the

D.

The hydrolysis of ATP into ADP and P, has a

rate of diffusion-limited reactions.

positive Gibbs free energy.

A cell's livelihood is dependent upon its ability to


carry out its metabolism efficiently. Efficient metabolism
requires efficient enzymes, working at rates greater than
the rates of unavoidable, competing side reactions. The
step which is rate-determining in many enzyme-catalyzed
reactions is due to the collision frequency of enzyme and

98.

It is discovered that in a particular cell, a membranebound compartment is 20% of the total cellular
volume. The concentration of reactants inside the

compartment can be:

substrate, rather than the actual enzymatic function of the


enzyme. These reactions are termed diffusion-limited, and

A.
B.
C.
D.

in theory, they can be accelerated by increasing the


concentration of the reactants. However, the cell's

cytoplasm has limits on the volume of reactants it can

5 times greater than in the cytosol.


10 times greater than in the cytosol.
15 times greater than in the cytosol.
20 times greater than in the cytosol.

hold, and most diffusion-limited reactions are not


accelerated in this fashion.

99.

The presence of intracellular membranes increases the


rate of reactions limited by diffusion. The formation of
membrane compartments concentrates substrates and
enzymes, and it minimizes the distance molecules must

The following table shows experimental data for


diffusion-limited reactions taking place outside and
within a cell membrane:

Trial

Membrane

Time required for


substrate to hit target

within the membrane itself, using membrane-anchored

None

15 minutes

enzymes, are also accelerated. The reason is that the

Large

1 minute

Small

9 seconds

Internal

travel. Diffusion-limited reactions that are carried out

substrates and enzymes are limited to movement in only


two dimensions. By eliminating a dimension of
movement, there is an enhanced opportunity for substrate
and enzyme collision.
95.

Which of the following statements describes an


organism approaching equilibrium?
A.

It has a high intracellular ratio of ATP/ADP.

B.

It attains the
metabolism.

C.

There is a constant input of energy into the


organism.
It experiences intracellular death and decay.

D.
96.

Based on information presented in the table, it can


be concluded that the frequency of collisions in:

most

efficient

state

A.
B.
C.
D.

of

100. According to the passage, an efficient enzymecatalyzed reaction requires that DNA-binding
proteins should:
A.

In the process of diffusion, the distance a molecule

travels from its origin is directly proportional to the


square root of the time traveled. Based on this

A.

efficient over long and short distances.

B.

efficient over long distances, but inefficient

contain many regions of acidic amino acids.

B.

be translated in the cell's nucleus.

C.

randomly jump onto and off of a DNA


molecule until they hit an appropriate binding

information, it can be concluded that diffusion is:

site.
D.

bind to any region of the chromosome and then

slide along the DNA until they reach an


appropriate binding site.

over short distances

C.
D.

Trial A is 15 times greater than in Trial B.


Trial C is 10 times greater than in Trial B.
Trial C is 100 times greater than in Trial A.
Trial B is 10 times greater than in Trial C.

inefficient over long and short distances.


inefficient over long distances, but efficient
over short distances.

Copyright by The Berkeley Review

159

The Berkeley Review


Specializing in MCAT Preparation

Biology

Metabolic Components

Section VII Answers

Enzyme Inhibitors

Passage 1(1 - 5)
1.

B is correct. We first need to consider the diagram with the enzyme interacting with the substrate and inhibitor

(Figure 1below). Notice that there are two sites at which binding can take place. The substrate (square) and the
inhibitor (diamond) bind at different locations. This is an indication that the inhibition is not competitive. Instead, it
should look like noncompetitive inhibition. Anoncompetitive inhibitor can bind to the free enzyme, or it can bind to
the enzyme-substrate complex.

G>

( E [S

CI*-

1/1S1

Figure 2

Figure 1

Now that we know the interaction is noncompetitive, we must choose the correct graph. A noncompetitive inhibitor

decreases the maximal velocity ofa reaction (Vmax). The more inhibitor added to the reaction, the more the Vmax is
decreased. The slope of the line becomes steeper. However, one important characteristic of a noncompetitive
inhibitor is that the Km remains the same. The constant Km is important, because it allows us to eliminate every
choice except B (see Figure 2 above). The correct choice is B.
2.

C is correct.The Michaelis constant, Km, tells us something about a given enzyme and its relationship to a given

substrate. Km has a simple definition. It is the substrate concentration, [S], that gives a half-maximal reaction

velocity. In other words, when [S] = Km, the enzyme is said to be half-saturated with substrate (i.e., v0 =Vmax/2).
This is what is given in choice A. The Km also characterizes the substrate-enzyme interaction. It is a measure ofthe
enzyme's affinity for asubstrate only when the step leading from the enzyme-substrate (ES) complex to the product
has a rate (ks) that is much smaller than the rate (k>) ofthe ES complex dissociating back to the free substrate and
enzyme. Asmall Km indicates a strong binding, while a high Km indicates a weak binding between enzyme and
substrate. This is indicated by choice B.
ki
E + S

k3
ES

*- E + P

The Km value between enzyme and substrate is not changed ifa noncompetitive inhibitor is added. Only the Vmax is
changed (it decreases). This is what we see in choice D. The Km is not numerically equal to Vmax/2, because Km =
[S] and not the value at Vmax/2. The value ofVmax/2 might be given in units of uM/min, while the value of Km
might be given in units of mM. The correctchoice is C.

Ais correct. This question is asking you to rearrange the Michaelis-Menten equation so the initial velocity can be
solved for in terms of Vmax.
v

Vmax [SI when rearranged gives: v =


Km + [S]

max

V,

Km+{S]_

Km + i

[S]

[SI

[S]

In the question we are told that [S] = 2.5 x 10"1 KM. This is [S] =0.25 KM. Substitution of this value into the
equation shown above gives the following:
v

V,

Km + j
[S]

v,

V,

Km
0.25 Km

+ 1

+ 1

Vmax

4+1

= Ymai = 0.20Vr
5

0.25

The correct choice is A.

Copyright by The Berkeley Review

160

The Berkeley Review

Specializing in MCAT Preparation

Biology

Metabolic Components

Section VII Answers

B is correct. It is important to understand the difference between competitive and noncompetitive inhibitors, how
they act, and what factors they affect in comparison to a control (i.e., with no inhibitor). Acompetitive inhibitor
binds reversibly to the active site of an enzyme. It competes with the substrate for the active site and increases the

Km of the enzyme. At a substrate concentration that is high enough, the substrate outcompetes the competitive
inhibitor. Therefore, at a high substrate concentration, in the presence of competitive inhibitor, the Vmax is the same
as that of the control. If we increase the concentration of inhibitor, the Vmax remains the same (at a substrate
concentration that is high), but the Km continues to increase. This is indicated in Figure 1below.
2x inhibitor

\~3 Inhibitor

\~2 Inhibitor

A noncompetitive inhibitor binds at a site other than the active site and does not compete at the active site for
substrate binding. Binding of a noncompetitive inhibitor inactivates the enzyme and therefore decreases the Vmax. It
does not alter the Km. It we continue to add noncompetitive inhibitor, the Vmax continues to decrease, but the Km

remains the same. This is indicated in Figure 2 above. Combining both of these diagrams gives the graph shown in
the question. Reaction 1 is the control, and Reaction 2 involves the competitive inhibitor. The graph of the
competitive inhibitor is shown above in Figure 1. The point that is common to both the control and the competitive
inhibitor is Point B. The correct choice is B.
5.

D is correct. Note that the graphs for each mixture are not hyperbolic. They are sigmoidal. They follow nonMichaelis-Menten kinetics. However, we can use the ideas put forth by Michaelis and Menten to answer the
question. Let's consider the Krebs cycle reaction and the graphs as they are given in the question.
Enzyme

Isocitrate + NAD+(exces>

a-Kctoglutarate + NADH + H+ + C02

CSS)

Based on the reaction equation and the three mixtures we see that excess NAD is used in each case. We also see
that as we move from Mixture 1 to Mixture 2 to Mixture 3, the concentration of ADP in solution increases. What

can we conclude from this? Does ADP increase the Km of the enzyme? No, because as we increase the
concentration of ADP in solution, the curves move to the left. This is characteristic of a decrease in Km. Eliminate

choice A. Is ADP an allosteric inhibitor of the enzyme? No, because as we add more ADP, the curve moves to the
left, indicating that less substrate is needed to reach half-maximal velocity. Eliminate choice B.

Does a high [NADH]/[NADe] ratio stimulate the enzyme? This is telling us that we have more NADH (the reduced
form) than NAD (the oxidized form). If we have less NAD (i.e., it is not in excess), then the reaction rate will
slow down. There will not be enough of the NAD coenzyme to facilitate catalysis. Eliminate choice C. A high
IATP]/[ADP] ratio means that there is less ADP than ATP. This would mean that the graph would resemble that of
Mixture 1. In other words, a high ATP concentration inhibits this reaction. This makes sense, because if we have

plenty of ATP in the cell, why waste energy making more? The cell makes more ATP only if its concentrations of
this nucleotide have been depleted. The correct choice is D.

Passage II (6 - 11)
6.

Enzymes, Coenzymes, and Vitamins

B is correct. In order to go from Figure 2 to Figure 3 in the passage, we must have a movement of electrons from
the imidazolium nitrogen of histidine to the nitrogen atom of the nicotinamide ring. We can see this if we consider
the flow of electrons in Figure A below. [This diagram corresponds to Figure 2 in the passage.] After electron
movement and the formation of new bonds, we get Figure B. [This corresponds to Figure 3 in the passage.]

Copyright by The Berkeley Review

161

The Berkeley Review


Specializing in MCAT Preparation

Biology

Metabolic Components

Section VII Answers

In Figure A, the electrons on the nitrogen atom of the histidine ring move to an area of electron deficiency. Areas of
electron deficiency are indicated by atoms bearing apositive charge (or partial positive charges). Areas of electron

deficiencies are sometimes referred to as electron sinks. In Figure B, we see that there isnow a pair ofelectrons on

the nitrogen atom of the nicotinamide ring and that the nitrogen atom of the imidazolium ring of histidine bears a

positive charge. Even though the electrons could end up on the atoms indicated in any of the other choices (and in
theory they might, for afleeting moment), they do not produce the stablest end product. The correct choice is B.
Active
site

/TV

/.O-H*

H C OH

ES Complex 1

ES Complex 2 CH2OP03

CH2OPOj"

Figure B

Figure A

7.

C is correct Consider the bond in the question as they are shown in Figure 1 below. If we were to hydrolyze this
bond with water, we would get a carboxylic acid functional group and a phosphoric acid functional group (Figure 2
below). These are two different acids. If we mix them together and lose the element of water between them (an

anhydrous reaction), we get a mixed acid anhydride linkage. Note that this linkage does not resemble the linkage of

eithera phosphodiester bond, a peptide bond, or an amide bond.


Q

fi e

1"
C

OH

o0

H-C-OH

HCOH

CH,OPO?

HO-P-O

CH,OPO,

Figure 2

Figure 1

Examples of a phosphodiester linkage, peptide linkage, and an amide are given below. Notice that a peptide linkage
is just a special class of an amide linkage.
R

ooo
n

ii

ii

it

o-p-o- po-p-o-r
i

CH2

-c-N-R"

Peptide

Phosphodiester

H,N

Amide

The correct choice is C.

C is correct If the sulfhydryl sulfur were to become an electrophile, it would beara positive charge and would not
be able to attack a (partially) positively charged carbon atom of the aldehyde functional group. Therefore, we can
eliminate choices B and D. When the sulfhydryl sulfurlosesits hydrogen atom, it becomes a nucleophile, a species
that seeks out electron-deficient centers (like the carbonyl carbon atom) and pass electrons to them. The question
now becomes one of what is formed afterthis happens. Look at Figure 2 in the passage. Do we see an acylthioester
or a hemiacetal? Consider the word "acylthioester" for a moment. Let's break this into its components. They are

acyl, thio, and ester. We know what an ester looks like-it is an R-CO-O-R' linkage (see below). The prefix thio- is
from thiol, which isjust R-SH. Thiols are the sulfur analogs of hydroxyl groups. Athioester would then look like RCO-S-R' (see below). An acyl group is derived from an acetyl group (CH3-CO-R) and looks like R-CO. It is just a

carbonyl group attached to something. Notice that an acylthioester needs a carbonyl group in the intermediate
structure in Figure 2 of the passage. Wedo not seethat. Therefore, we can eliminate choice A.
Copyright by The Berkeley Review

162

The Berkeley Review


Specializing in NCAT Preparation

BlOlOgy

Metabolic Components
o

Section vii Answers

II

II

R-C-0-R1

R-C-S-R1

Ester

Thioester

What is a hemiacetal? Ahemiacetal can be formed when an aldehyde undergoes a nucleophilic attack by a hydroxyl
group (or in this case, a sulfhydrylgroup). The generalreaction is shown below:

OH

R-C-S-R"

-^

II

i-

R-C-S-R'
II

Hemiacetal

The hemiacetal that is formed is usually too unstable to isolate. However, as shown in Figure 2 of thepassage, this is
the structure that is linked to the enzyme. The correct choice is C.

9.

C is correct. As you read in the passage, dehydrogenase enzymes can contain either niacin or vitamin B2. If the
enzyme contains niacin, it is referred to as an NAD-linked dehydrogenase. If the enzyme contains vitamin B2, it is
referred to as an FAD-linked dehydrogenase. Very little niacin is found in either milk or eggs. This allows us to
eliminate choice A. Even though fish is a good source of niacin, it is not mentioned as being a good source of

vitamin B2. Eliminate choice B. Similarly, beans are a good source of niacin but are not listed as being a good
source of vitamin B2. Eliminate choice D. This leaves meat as a common source of both niacin and vitamin B2. The
correct choice is C.

10.

B is correct This question is asking whether you can recognize the difference between something that is oxidized
and something that is reduced. Only one of the oxidation-reduction pairs is not found at the active site. Consider the
Figures 1-4 in the passage. In choice A, we find the reduced form of NAD (i.e., NADH) and the reduced form of
cysteine (i.e., R-SH) at the active site in Figure 4. Eliminate choice A. In choice C, we find the oxidized NAD (i.e.,

NADe) and the reduced cysteine at the active site of Figure 1. Eliminate choice C. In choice D, we find the oxidized
NAD and the oxidized cysteine (i.e., R-S-R') at the active site of Figure 2. Eliminate choice D.

In choice B, we also find the reduced form of NAD at the active site of both Figure 3 and Figure 4. However, the
product, which is in the active site of Figure 4, has been oxidized (and not reduced). How can we tell whether the
product has been oxidized? The oxidation state of the carbonyl carbon in the substrate in Figure 1 is +1. The
oxidation state of the carbonyl carbon in the product in Figure 4 is +3. As we move from the substrate to the

product, there is a 2-electron change at that carbonylcarbon. If there has been a loss of electrons, then it signifies an
oxidation. A gain of electrons would signify a reduction.
Another way to consider this is to think of just the substrate and the coenzyme. Since they are both involved in an
oxidation-reduction reaction, then one must start off in the oxidized form, while the other is in the reduced form.

After the reaction is over, the one that was originally in the oxidized form becomes reduced, and the one originally
in the reduced form becomes oxidized. In Figure 1, the coenzyme NAD is in the oxidized form (NAD). Therefore,
the substrate must be in the reduced form. At the end of the reaction, NAD is in the reduced form (NADH), and the
product is in the oxidized form. The correct choice is B.

11.

B is correct. The reaction in the question tells us we need to consider the coenzyme FAD. The structure of this
coenzyme is given in the passage in Figure 5. Note that the oxidized form is highly conjugated. We are told in the
passage that "...the more conjugated bonds a molecule has, the longer the wavelength at which the molecule absorbs
light." In the question we are told that "oxidized FAD absorbs light in the visible region of the electromagnetic
spectrum at 460 nm." We are also given a graph of light absorption at wavelengths in the visible region of the
electromagnetic spectrum.

At the beginning of the reaction, we have a solution of succinate (reduced) and FAD (oxidized and highly
conjugated). The oxidized form of FAD is absorbing light at 460 nm. By looking at the graph, we see that the blue
cone is being absorbed. If a wavelength of light is being absorbed, we perceive its complementary color. In this case,
the complementary color to blue is yellow (a mixture of the green and red cones). Therefore, our starting solution is
yellow. We can eliminate choice A and choice D.

Copyright by The Berkeley Review

163

The Berkeley Review


Specializing in MCAT Preparation

Biology

Metabolic Components

Section VII Answers

At the end of the reaction, we have an oxidized product (fumarate) and a reduced coenzyme (FADH2). Note that the

structure of FADH2 in Figure 5 of the passage is not as highly conjugated as the structure in the oxidized state. The
fewer conjugated bonds a molecule has, the shorter the wavelength at which that molecule absorbs light. Because

FADH2 has^so few sites ofconjugation, we would expect absorption to take place outside the visible range of the
electromagnetic spectrum. The final solution will be colorless. The correct choice is B.
Plasma Glucose Measurement

Passage III (12- 17)


12.

D is correct. Choice A is (3-D-glucose. Choice B is saccharic acid. Choice C is fructose. Choice D is glucono-5lactone. A lactone is recognized by the fact that it is a cyclic ester. The correct choice is D.

13.

C is correct. Blood glucose would rise in response to a meal, then fall when insulin was released by the pancreas in

response to elevated blood glucose. A slight dip is common when the glucose level is lower than at the baseline
state, and then it corrects itself. Blood glucose neither rises linearly nor falls linearly. Choices A and D are incorrect.

Blood glucose does change in response to a meal, so choice B is incorrect. The correct choice is C.
14.

A is correct. If we read the standard curve, this is an easy question. Find 1250 on the y-axis. Move horizontally until

you intersect the line of the standard curve. Move down to touch the x-axis, and read off the correct answer, 110.
The correct choice is A.

15.

A is correct. H2O2 from any source would reach the electrode and be oxidized, although the person reading the test
results assumes that only H2O2 from the glucose oxidase reaction is reacting. However, if extra H2O2 somehow got
into the machine, then the reading would be misleadingly high. Choices B and C are incorrect. H2O2 does not
oxidize glucose. The correct choice is A.

16.

C is correct. If the filter blocked hydrogen peroxide, then nothing would reach the electrode and be oxidized to

make a signal. Choice A is incorrect. If many oxidizable substances passed through the filter, then the readings at
the electrode would be amplified by other oxidation reactions. This would make an incorrect reading of plasma
glucose. Choice B is incorrect. Choice C is the correct answer: The filter keeps large oxidizable substances away
from the electrode. Choice D is incorrect. The filter in question is downstream from the immobilized enzyme, so it
does not protect the enzyme. The correct choice is C.
17.

B is correct. Oxidation occurs at the anode. The hydrogen peroxide is oxidized. The platinum acts as a reducing

agent. Platinum acts as an anode. Choices A and C are incorrect. An oxidase is an enzyme. It plays a role in the first
reaction. Choice D is incorrect. The correct choice is B.

Brown Adipose Tissue

Passage IV (18-23)
18.

A is correct. In the uncoupling situation, fewer ATPs are produced per unit of fuel. The dissipation of the

electrochemical H gradient produces heat, which is probably important in regulation of body temperature in
neonates. Choice B is incorrect. More fuel must be burned to make ATP than in the regular, coupled state. Body fuel
stores would be recruited, not conserved. Choice C is incorrect. All this oxidation would lead to an increase in the

resting metabolic rate. Choice D is incorrect. The correct choice is A.

19.

C is correct. The passages tells you that the number of mitochondria is greatly increased in BAT versus WAT.
"Chrome" usually refers to a colored substance. The cytochromes contain iron and are brown in color. The WAT has
the same cytosolic components as the BAT, so they are not causing differences in color between the two types of
tissue. Choices A, B, and D are incorrect. The correct choice is C.

20.

D is correct. The diagram is a second messenger system. The hormone does not enter the cell, but relies instead on
its communication with the cell interior via the second messenger, which is a G protein, in this case. Choice A is

incorrect. The a-subunit is involved in activating adenylate cyclase. The (5- andy-subunits are neutral in this case.
Choices B and C are both incorrect. The correct choice is D.

21.

B is correct. The fat cell is a storage depot for triglyceride. A triglyceride contains glycerol esterified to three fatty
acids. It is neutral and hydrophobic. This is the form that is used for storage in fat cells. Fatty acids are present in the
blood and are oxidized for fuel in the tissues. Phospholipids are located in the membranes, predominantly. Glycerol
is a component of triglycerides, and it also exists free (unbound to anything) in the blood. The correct choice is B.

Copyright by The Berkeley Review

164

The Berkeley Review


Specializing in MCAT Preparation

Biology
22.

Metabolic Components

Section VII Answers

A is correct. Although the adipose tissue does not perform gluconeogenesis, the liver tissue does and can use the
released glycerol for the production of glucose. This plays a role in glucose homeostasis. BAT does not contain

glycogen or release glucose. Choices B and C are incorrect. Fatty acids cannot be made into glucose, by
gluconeogenesis or any other pathway in the human body. Choice D is incorrect. The correct choice is A.

23.

C is correct. Thermogenin is probably most like a protein with a similar role in the body. Chymotrypsinogen is a
hormone that, when cleaved properly, digests certain peptide bonds in food. Choice B is incorrect. Hemoglobin is a
transporter of oxygen in the blood. Choice A is incorrect. Choices C and D exist in the membrane of cells and allow

things to transport through the membrane. However, ATP-ADP synthetase is located in the mitochondrial inner
membrane and allows protons to cross, driving ATP synthesis. Thermogenin allows protons to cross the inner
mitochondrial membrane for free (i.e., without an input of energy). The sodium-potassium pump requires energy.
Choice D is incorrect. The correct choice is C.

Passage V (24 - 29)


24.

25.

Heme Metabolism

B is correct. This question can be answered using previously acquired knowledge. The human red blood cell
contains hemoglobin molecules. Therefore, if we know where the human red blood cell is degraded, we know where
the breakdown of heme occurs. A typical human red blood cell has a life span of 120 days. Old cells are removed
from the circulatory system and degraded in the spleen. The correct choice is B.
D is correct. Looking at Figure 1, we see that the way to determine whether the conversion is a reduction or an
oxidation is to make note of the cofactor involved (in this case, NADPH). The cofactor itself is oxidized as a result

of the reaction, and the oxidation involved two electrons. The two electrons given off had to go somewhere, and
they went to the reduction of biliverdin into bilirubin. Remember, we cannot have an oxidation without a reduction.
Therefore, we can conclude that the conversion is a two-electron reduction. The correct choice is D.

26.

A is correct. We have to look at the two structures given in the question. First, there is no difference in the chemical
constituents of the two molecules. However, the atoms are arranged differently. For this reason, an isomerase is the
enzyme that would be involved. Next, in looking at the picture one should arrive at the conclusion that uro I is a
symmetric molecule, symmetric around the center point of the molecule (where the iron would be bound). By
switching one set of constituents, uro III is produced. Switching these constituents produces an asymmetric
molecule. The correct choice is A.

27.

C is correct. The passage informs us that carbon monoxide is released when the heme group is converted to
biliverdin. From one's previous knowledge of Hb, one should remember that carbon monoxide has a very strong
affinity for the iron atoms of the hemoglobin molecule. That is why the gas is poisonous. Judging from this reaction,
we can assume that the body naturally produces a small amount of this gas. However, the amount is very small
relative to the amount of oxygen in our body. Nonetheless, we cannot ignore the fact that CO is indeed produced,
and a small percentage will indeed bind to Hb. The correct choice is C.

28.

C is correct. The passage informs us that persons suffering from this disease have abnormal red blood cells that are
eliminated by the body. Therefore, we can assume individuals with this disease have a lower than normal red blood
cell count. Does the red blood cell level contribute to the partial pressure of oxygen? The answer is no. The partial
pressure of oxygen is determined only by the amount of oxygen dissolved in the blood, not by the amount of oxygen
bound to hemoglobin. For this reason, persons with CEP would not have a lowered partial pressure of oxygen. The
correct choice is C.

29.

A is correct. This is a very straightforward genetics question. We know from the passage that the disease in
question, CEP, is transmitted in a autosomal recessive fashion. The father is heterozygous for this trait, making his
genotype Cc (letters chosen arbitrarily). The mother is a homozygous dominant, making her genotype CC. The
question asks for the probability that a son will suffer from the disease. Doing the cross between parents, it becomes
apparent that no child will suffer from the disease, as no recessive allele is contributed by the mother. Therefore,
there is a 0% chance. The correct choice is A.

Passage VI (30 - 36)


30.

Enzyme Kinetics I

B is correct. Trypsin is an enzyme that cleaves peptide bonds on the carboxyl side of the amino acids arginine (Arg)
and lysine (Lys). The important clue to answering this question comes from the last sentence of the second
paragraph. It says, "...enzymes readily degrade polypeptides synthesized from L-amino acids, but not D-amino
acids." This tells us that we must find that structure with the two amino acids in the L configuration. How do we do
this? We need to recall our stereochemistry from organic chemistry.

Copyright by The Berkeley Review

165

The Berkeley Review


Specializing in MCAT Preparation

BiolOfly

Metabolic Components

Section vii Answers

Recall that a Fisher projection is a way of representing a tetrahedral carbon (and its substituents) in three
dimensions. The structures of the dipeptides inthe question are drawn in Fisher projections. Each dipeptide has two
chiral carbon atoms, each with four different substituents. Focus on the four bonds attached to those chiral carbons.

Vertical lines represent bonds pointing away from your point of view, while horizontal lines represent bonds
pointing towards yourpointof view.

Rank the four substituents on each chiral carbon in order of decreasing priority. Arrange the order so that the

substituent with the lowest priority ison top. In each case the substituent with the lowest priority ishydrogen (H). If
two substituents need to be exchanged so that hydrogen is on top, then do not forget to exchange another pair of
substituents so that the absolute configuration is retained. Next, trace the order of priority of the three remaining

groups. Ifthe tracing is in the clockwise direction, the stereocenter is designated as R(rectus, Latin for right). Ifthe
tracing is in the counterclockwise direction, the stereocenter is designated as S (sinister, Latin for left). However,

since amino acids are designated by the older Dand L prefixes, we must make a conversion. It turns out that the old
Dprefix isanalogous to the old Rprefix, and the Lprefix isanalogous to the S prefix. We are now ready tofind the
answer.

Consider Structure I (see below). Both chiral carbon atoms have their hydrogens pointing up. All we need to do is

arrange the substituents in order of priority. Let's do lysine first. Nitrogen (N) has an atomic number of 7, while
carbon (C) has an atomic number of 6. Nitrogen is assigned the highest priority. Next, we need to distinguish
between the carbons attached to the chiral carbon. The carbon atom with the oxygen atom attached has the next

highest priority. This is followed by the methylene (-CH2-) carbon atom. The tracing is from nitrogen to carbonyl
carbon to methylene carbon, giving a clockwise direction. This stereocenter is R, which in the old nomenclature is
D. Right away, we know that Structure I cannot be the answer, because we need both chiral centers to be in the L
configuration. Wejust found thatonechiral center is in the D configuration.

R(D) S^~?\ Nc""*X R(D>

Let's continue this example by finding the configuration about alanine's chiral carbon. Again, the hydrogen atom is

pointing up and therefore has the lowest priority. Nitrogen still has the highest priority, followed by the carboxyl
carbon, and finallythe methyl carbon. The tracing is againclockwise, givingthe D configuration.
L

II

D
_

OOC-C-N-C-C-NH,

III3
H,C H
CH,
3

l
CH2

I
CH,

I
CH,

CH2

CH2

NH,

I 3

I I
H H

CH2
I

CH,

I II

H,N-C-C-N-C-COO

I 2

II

jjj

CH,

I 3

II

II
H H

,CsN

H2N

NH2

CH2

I
CH2

jy

CH2

NH
I

OOC- C- N- C- C- NH,

NH

^
H2N

NH2

Arginine (Arg)

Following this same procedure for Structure II, we get a configuration of L-Lys and L-Ala. Structure III has a
configuration of D-Arg andL-Ala. Structure IV has a configuration of D-Ala andL-Arg. The correct choice is B.

Copyright by The Berkeley Review

166

The Berkeley Review


Specializing in MCAT Preparation

Biology
31.

Metabolic Components

Section vh Answers

D is correct As stated in the fourth paragraph of the passage, DIPF is an irreversible inhibitor. Since this inhibitor
reacts with active site serine residues, it must enter into the active site, bind to a specific serine residue (see below),
and shut the enzyme down. Allosteric inhibition refers to the fact that binding is taking place at a site other than the
active site. Since this is not the case here, we can eliminate choice A.
H2C~ Serine

(H,C)2HC - O - P~ O - CH(CH3)2

Serine - CH2OH

*-

(H3C)2HC - O - P- O - CH(CH3)2

I'
O

II
O

DIPF

Phosphorylated activesite

+ HF

If DIPF were a competitive inhibitor, it would mean that it is of the reversible type, which is not what is stated in the
passage. We can eliminate choice B. What about phosphorylation of ADP? If DIPF could phosphorylate ADP (and
in the process form HF), then it would be unable to react with the active site serine residue. This allows us to
eliminate choice C. In order for DIPF to bind irreversibly to the active site serine residue, it must phosphorylate that
residue as indicated above. The correct choice is D.

32.

B is correct. Notice that initially (at low [S]) the reaction velocity for Curve II is lower than the reaction velocity for
Curve I. However, as the [S] is gradually increased, the reaction velocity of Curve II approaches that of Curve I.
Eventually, the two reaction velocities (in theory) will reach the same velocity (i.e., Vmax). This is exactly what is
mentioned in the last paragraph of the passage. The correct choice is B.

33.

A is correct. As outlined in the last paragraph of the passage, competitive inhibitors show the same maximal
velocity. This means that the maximal velocity of the competitive inhibitor crosses the y-axis on the graph at the
same point as the maximal velocity of the control. Only Lines I and III intersect at that point. This allows us to
eliminate choice B and choice D.

The most effective competitive inhibitor is that inhibitor that requires more substrate to reach half-maximal
saturation. In other words, the most effective competitive inhibitor is that inhibitor with the largest apparent Km.
This would give the smallest value for -1/Km. Another way to look at this is that the most effective competitive
inhibitor is indicated by the line with the steepest slope. The correct choice is A.
34.

C is correct. Almost all enzymes are proteins. Since proteins are composed of amino acids, they are quite sensitive

to pH changes. We are told in the question that aspartate 52 (Asp-52) has a pKa of about 4.0, while glutamate 35
(Glu-35) has a pKa of about 6.0. Based on our discussion of pKas we know that at the pKa of a particular amino acid
side chain the dissociable hydrogen atom is half on and half off. The dissociable hydrogen on the side chain spends
half of its time in the protonated state and half of its time in the deprotonated state.

At low pH values (close to 1), we would expect both the Asp-52 and the Glu-35 side chain carboxyl groups to be
protonated. As the pH begins to increase and approach the pKa of Asp-52, we find that the side chain carboxyl of
Asp-52 begins to lose its hydrogen atom. That carboxyl group begins to take on a negative charge. By the time we
are one full pH unit away from the pKa for Asp-52 (at a pH of 5), we find that'the side chaincarboxyl group spends
most of its time negatively charged. This negative charge stabilizes the substrate at the active site. At a pH of 5, we
are one full pH unit below the pKa for Glu-35. This means that the side chain carboxyl of Glu-35 still has its
dissociable hydrogen atom. It is this hydrogen atom thateventually gets transferred to the substrate at the active site.
As we increase the pH even more, say, to one full pH unit above the pl^ for Glu-35, we find that the side chain
carboxyl of Glu-35 now spends most of its time negatively charged. Since we do not have a proton to donate to the
substrate anymore, the reaction rate begins to decrease. Therefore, we would expect an increase in reaction rate as
we approach a pH of 5, and a decrease in the reaction rate as we move to pH values higher than 5. This is exactly
what we see in the bell-shaped curve in Graph III. With this information, you should be able to reason why Graphs I,
II and IV are invalid solutions. The correct choice is C.

35.

B is correct. Aspartate-52 provides the general base catalysis, because its side chain B-carboxyl group is about 9%

protonated ata pH of5. At a pH ofapproximately 5, we find that Asp-52 would acts as the general base (-COOe),
while Glu-35 would act as the general acid (-COOH). Without even dealing with percentages, we can immediately
eliminate choice A and choice D.

Copyright byTheBerkeley Review

167

The Berkeley Review


Specializing in MCAT Preparation

Biology

Metabolic Components

Section VII Answers

Let's consider choice B and choice C. In choice B, we are told that Glu-35 acts as the general acid, while Asp-52

acts as the general base. We know this iscorrect. Are the percentages correct? Will the side chain carboxyl group of
Glu-35 be about 9% deprotonated at a pH of 5? Let's give a rough estimation first. At a pH of 6, we know that the
side chain carboxyl of Glu-35 is 50% protonated and 50% deprotonated. This is because the pKa of the side chain
carboxyl of Glu-35 is given as 6. We can see this quantitatively by using the Henderson-Hasselbalch equation, as
shown in the first column below:

Using pH = 5

Using pH = 6
pH = pKa + log

pH = pK;i + log [AT


[HA]

[A-]
[HA]

6 = 6 + log

[A']

0 = log

A"!

5 = 6 + log

[HA]

[HA]

-l =.ogi*l

[A-

[HA]

[HA]

10o = jAl

,0' = I^lor l&Jm


[HA]

[HA]

, _ [A"] or [HA] = [A]


[HA]

[A"]

10=yiMor 10[HA]
[A-

1[A]

Let's try this same approach with a pH of 5. This is shown in the second column above. At a pH of 5, the side chain
carboxyl of Glu-35 is more on the protonated side and less on the deprotonated side. After applying the Henderson-

Hasselbalch equation, we end up with 10 = [HA]/[Ae]. This states that there is ten times more of the protonated
(HA) form of the carboxyl side chain than the deprotonated (Ae) form. In other words, Glu-35 is about 91%
protonated or about 9% deprotonated. A similar calculation can be made for Asp-52.
The best way to get a feel as to whether or not an amino acid is protonated or deprotonated, and by how much, is to
use the Henderson-Hasselbalch equation in a series of simple calculations. Start like we did in the first column,
where the pH is equal to the pKa of the dissociable hydrogen of interest. Next, move one full pH unit away from the
pKa (in either direction). You will find results similar to the ones in the second column. Move two full pH units
away from the pKa of interest. Try three full pH units away. You should begin to see a pattern develop. In fact, once
you understand this pattern, you will not need to use the Henderson-Hasselbalch equation for the calculation. You
will just be able to do a very quick and fairly accurate estimation. The correct choice is B.
36.

C is correct. The maximal velocity of the reaction is taken from the intersection of the line with the y-axis. The
value of the point at that intersection is 2. All we need to do is remember that we have a reciprocal. Therefore, 1/v =
1/2. The correct choice is C.

Passage VII (37 - 43)

Vitamin B12

37.

D is correct. An oxidoreductase catalyzes a reaction in which one substrate is oxidized and another is reduced.
Choice A is incorrect. A hydrolase catalyzes the hydrolytic cleavage of singe bonds. Choice B is incorrect. A
transferase moves groups from one substrate to another. Choice C is incorrect. An isomerase moves a group from
one position to another within the same molecule. The correct choice is D.

38.

A is correct. This is a question of which came first, B12 or folate deficiency. If B12 is low (i.e., a B12 deficiency),
the enzyme homocysteine methyltransferase can't work properly. Any THF that is around is trapped as methyl-THF
at this enzyme. Folate functions as THF in many aspects of one-carbon metabolism; and in this case, it is delivering
a methyl group to homocysteine methyl transferase. Even if folate status were normal, this trap due to B12
deficiency would cause a secondary folate deficiency. The correct choice is A.

39.

A is correct. An intramuscular injection of B12 bypasses the problem with IF and makes the vitamin directly
available to the blood. Statement I is correct. An increase in dietary B12 would not help if the IF were not produced,
because the vitamin is absorbed as a complex with IF. Statement II is incorrect. Co itself is not the vitamin. The
vitamin contains Co in the B12 molecule, but Co by itself would be of no benefit. Statement III is incorrect. The
correct answer is A.

Copyright by The Berkeley Review

168

The Berkeley Review


Specializing in MCAT Preparation

Biology
40.

Metabolic Components

Section VII Answers

B is correct. If you remember the anatomy of the GI tract, the ileum is the last section of the small intestine, and the

colon or large intestine follows it. The receptor of B12 is in the ileum, so any B12 produced past this receptor (in the
colon) is not absorbed. The makes choice A incorrect. Choice B is correct. Choices C and D imply that the colonic
B12 is absorbed to have reactions on enzymes in the body, and are therefore incorrect. The correct choice is B.

41.

A is correct. If methyl malonyl-CoA cannot be metabolized into succinyl-CoA, then the CoA is hydrolyzed and the
excretion product is MMA. Choice A is correct. B12 is not a competitive inhibitor of the MMA pathway. Choice B
is incorrect. MMA is not a by-product of homocysteine methyl transferase; this is another enzyme altogether.
Choice C is incorrect. MMA is not made from IF. Choice D is incorrect. The correct choice is A.

42.

D is correct. Met is an essential amino acid. This may seem like a trick because of the reaction with homocysteine
methyl transferase. However, this homocysteine had to be made using a Met in the first place. Choice A is true. We
can see from the diagram that choice B is true. Met is used in gluconeogenesis, so choice C is true. The precursor of
Tyr is Phe, so choice D is false. Since you want the false answer, choice D is it. The correct choice is D.

43.

B is correct. IF is a glycoprotein, which is a protein-carbohydrate combination. Just like the protective mucus of the
GI tract, glycoproteins have nondigestible carbohydrate bonds. IF is protected from digestion, because our bodies do
not have the digestive enzymes to break it down. Choice A is incorrect, because pepsin (a protein-digesting enzyme)
is present in the stomach. Choice C is incorrect, because IF does not influence the activation of pepsin. Choice D is
incorrect, because the passage tells us IF makes it to the ileum from the stomach. It must be resistant to gastric
enzymes. The correct choice is B.

Passage VIII (44 - 50)


44.

Enzyme Nomenclature

B is correct. The values of the reactants and products at equilibrium is 10 moles. Using the equation AG"' = -2.3 RT

log Keq, where Keq = [C]/[A][B], we find that AG01 = + 2.3 RT.
AG"' = - 2.3 RT log Kcc

AG' =- 2.3 RT log |C1 =-2.3 RT log [10]

>

[A][B|

AG" = - 2.3 RT log 0.10

[10] [10]

AG'= - 2.3 RT (- 1.0) = +2.3 RT

The standard free-energy change is an important concept to understand, because it will be able to tell us whether a
reaction is spontaneous (exergonic) or nonspontaneous (endergonic). The correct choice is B.
45.

A is correct. Lyases are involved in the cleavage of bonds like C-C, C-N, and C-O. Lyases can cleave double bonds
to make single bonds, or single bonds to make double bonds. Functional groups can be either added across or taken
away from these bonds, respectively.

COO

coo

Fumarase

CH

II

CH2
'
0

CH
I

HO-C-H
I

H-,0

COO

coo

Malate

Fumarate

In the example of fumarate going to malate, we see that water is being added across a double bond. This type of
reaction occurs quite frequently in biochemistry. If we add the element of water to a bond, it is referred to as a
hydration reaction. A hydrolysis reaction involves the use of water (hydro-) to break (-lysis) a bond. The correct
choice is A.

46.

C is correct. Hexokinase aligns the y-phosphoryl group of ATP with the C-6 hydroxyl of glucose and catalyzes a
transfer of that y-phosphate group to the C-6 position of the sugar. Since this reaction involves a chemical group
transfer, the enzyme is a transferase.

Ligases involves the hydrolysis of a high-energy bond, such as those found in ATP; but the energy in that highenergy bond is used to drive the condensation of two molecules via a ligation reaction. Hydrolases involve the use of
water to break a specific bond. Oxidoreductases are enzymes involved in electron transfer through oxidationreduction reactions. The correct choice is C.

Copyright by The Berkeley Review

169

The Berkeley Review

Specializing in MCAT Preparation

BiolOflV

Metabolic Components
0

Section vh Answers

'/*-'

O-P-0- P-0- AMP


vj r

II

. _

n
9

0-P=0
i

O'

H2C

0 0H
\|

H^~r

H2C

Hexokinase

adp

/n

O 0H
\J

"o^i

OH

OH

Glucose

Glucose-6

phosphate

47.

C is correct We can think of the nucleophilic properties of the C-6 hydroxyl of glucose and water as being similar,
if we consider both molecules as having the R-OH format. The R for water is a hydrogen atom, while the R for

glucose is the rest of the molecule other than the C-6 hydroxyl. Even though the hydroxyl functional group in both
cases acts as a nucleophile, the R group in each case is quite different.

The presence of glucose at the active site (along with ATP) induces a large conformational change in the enzyme
and allows the active site to close around the substrates with the exclusion of water. If water is excluded from the

active site, the polarity of the active site'senvironment is reduced. The active site becomes hydrophobic. This means
that the C-6 hydroxyl of glucose and the y-phosphoryl group of ATP are not solvated. Therefore, the C-6 hydroxyl
group's oxygen atom becomes more nucleophilic, while the y-phosphoryl group's phosphate atom becomes more
electrophilic. These events allow for an accelerated transfer of the phosphoryl group to glucose. If water were at the
position occupied by the C-6 hydroxyl group, it would be able to attack the y-phosphoryl groupof ATP. The result
would be the hydrolysis of ATP to ADP and (inorganic) phosphate. In other words, the rate of ATP hydrolysis
would increase. As stated in the question, this is not what is observed. The correct choice is C.
48.

B is correct It is important to know the differences between an aldose sugar and a ketose sugar. Sugars are
saccharides, polyhydroxyl carbonyl compounds that contain an aldehyde or a ketone functional group and at least
two hydroxyl groups. Sugars with aldehyde functional groups are referred to as aldoses, while sugars with ketone
functional groups are referred to as ketoses. Even though you do not need to know the complete structures of the
aldose and the ketose in the second step of glycolysis, you do need to know (recognize) the aldehyde and ketone
functional groups of a sugar. We can draw them as shown below:

Enzyme

H-C-OH
I

<{H*0H

HO-C-H

C = 0
I

HO C-H

Aldose

Ketose

What type of enzyme assists in the conversion of an aldose into a ketose? Notice that in going from an aldose to a
ketose, there is no loss or gain of atoms. These two molecules differ in the sequences in which their atoms are held
together. The empirical formula is the same in both cases. These molecules are isomers of one another. Therefore,
the type of enzyme that catalyzes this reaction is an isomerase. The characteristics of the other three choices are
outlined in the passage in Table 1. The correct choice is B.
49.

B is correct The pKaS for these amino acid pairs are given in Table 2 in the passage. The optimal activity of the
enzyme is where the initial velocity is the greatest, which is at Vmax (i.e., the maximal velocity). The pH at Vmax is
about 9.0. We want that amino acid pair whose pKa values border as close as possible to a pH of 9.0.

We can get a rough estimate of this by adding the two pKa values of the amino acid pair together and then dividing
by 2. For choice A, we get a pH value of 4.95, which is way to the left of the graph and nowhere near the optimal
activity of the enzyme. Choice B gives a pH value of 9.2, which is extremely close. Choice C gives a pH value of
11.65, which is a little further away. Choice D gives a pH value of 9.6, which is close to the optimal activity, but not
as close as choice B. The correct choice is B.

Copyright by The Berkeley Review

170

The Berkeley Review


Specializing in MCAT Preparation

Biology
50.

Metabolic Components

Section VII Answers

C is correct. The passage mentions that lactate dehydrogenase (LDH) has two different types of subunits,
designated as M (for muscle) and H (for heart). Let's assume that the band shown in Lane 1 is composed of all M
subunits (i.e., Mx, where x is the number of subunits), while the band shown in Lane 3 is composed of all H subunits
(i.e., Hx). Therefore, the bottom band in Lane 2 is all M and the top band is all H. The three bands in the middle are
mixtures. This is indicated in Figure 1 below.
Lane 1

Lane 2

Lane 3

Lane 1

Lane 2

Lane 3

(+)
1

II

{ \

1\ /
Hx

1 1,

(+)

11

1 LS
, S,
-i

ii

ii

i \y

DD
DB

M[H3

an

M2H2
M3H,

Origin

M4
\

H4

DD

Hx
1

DO

(-)

\y
M,

\y
M,

Figure 2

Figure

Consider Band 3 in Lane 2. This band falls halfway between the isoenzyme that is all M and the isoenzyme that is
all H. We would expect the isoenzyme indicated by Band 3 to contain an equal mixture of M and H subunits. If
Band 3 were to contain one M and one H subunit, then Band 1 would contain 2 M subunits, and Band 5 would
contain 2 H subunits. This means that Band 2 would have to contain 1.5 M subunits and 0.5 H subunits. Similarly,
Band 4 would have to contain 0.5 M subunits and 1.5 H subunits. Since we cannot have anything but a whole
number for a subunit (otherwise, the subunit would be nonfunctional), we need to make an adjustment in our

analysis.
Instead of a 1:1 mixture of M to H in Band 3, let's assume a 2:2 mixture of M to H. This would allow Band 1 to
contain 4 M subunits and Band 5 to contain 4 H subunits. Band 2 now contains a ratio of 3M:1H subunits, while

Band 4 will contain a ratio of 1M:3H subunits. This is shown in Figure 2 above. This analysis fits the gel pattern and
indicates that there are 5 isoenzymes of LDH, each being a tetramer. The correct choice is C.

Lysozyme Mechanism

Passage IX (51 - 58)


51.

A is correct. Bacterial cells that are Gram-positive have a plasma membrane surrounded by a thick peptidoglycan

layer, the cell wall. Within this layer are two types of sugar residues, N-acetylmuramate (NAM) and Nacetylglucosamine (NAG), which can act as a substrate for lysozyme. This enzyme has a clear path to these sugars.
In a Gram-negative bacterium, there is an additional outer membrane that contributes to the bacterial cell wall. This
outer membrane acts as a barrier that helps to prevent lysozyme from degrading the peptidoglycan layer. If bacteria
are missing their cell walls, chances are that they have already lysed due to osmotic differences between the inside
of the cell and the outside of the cell. If they have not lysed but have formed protoplasts instead, then the lysozyme

enzyme is facing a plasma membrane that does not have the NAM and NAG sugar residues linked as they were in
the peptidoglycan layer. The correct choice is A.

52.

D is correct. You need to recall your carbohydrate chemistry to answer this question. Rings D and E have been
labeled with the C-l and the C-4 positions already (see Figure 2 in the passage). In order to determine the
configuration of the linkage, we first need to locate the anomeric carbon atom of the ring.
The anomeric carbon in the open-chained form is the most oxidized carbon atom. That turns out to be the C-l carbon
or the carbonyl carbon of the aldehyde functional group. When the ring begins to close, the oxygen of the C-5

hydroxyl group attacks the C-l carbon. The carbonyl oxygen at the C-l position picks up a hydrogen atom to form
the C-l hydroxyl. That hydroxyl can reside either above or below the plane of the ring, depending on the direction
of attack. If the C-1 hydroxyl is above the plane of the ring, it is said to be in the ^-configuration; if it is below the
plane of the ring, it is in the a-configuration.

By examining the linkage between rings D and E, we see that the hydroxyl at the C-l position of the D ring is in the
p-configuration. The linkage extends from the C-l carbon of ring D to the C-4 carbon of ring E. Hence, the
configuration of the linkage is said to be p( l->4). The correct choice is D.
Copyright by The Berkeley Review

171

The Berkeley Review

Specializing in MCAT Preparation

Biology
53.

Metabolic Components

Section Vll Answers

A is correct. All enzymes are based on the type of reaction they catalyze. Many enzymes are known by a common
name, usually derived from the principal reactant involved in the catalysis. For example, arginase reacts with the
amino acid arginine.

Arginase falls within a specific class of enzymes, just as lysozyme does. There are six classes of enzymes
established by the Enzyme Commission of the IUPAC. In the answers to this question, we are presented with
enzymes from four of those classes. A hydrolase enzyme is involved in hydrolysis reactions. Water is used tocleave
bonds like C-C, C-N, or C-O. This is exactly what we see in the case of lysozyme (and what we will see in the case

of arginase in a later discussion). A transferase enzyme transfers a functional group. An oxidoreductase is involved
in an oxidation-reduction reaction. A ligase is involved in bond formation (think of ligation) that is coupled to the

hydrolysis of ATP. Wedo not seethe last three examples involving lysozyme catalysis. The correct choice is A.
54.

D is correct. Almost all enzymes are sequences of more than 100amino acids. However, there are only a few amino
acids that can be used to make up the catalyticgroup of amino acids at the active site. These few amino acids form
an enzyme-substrate complex through a variety of bonding interactions. As the substrate forms product, it goes
through an intermediate species (the transition state). The transition state has a free energy that is higherthan either
the substrate or the product. The charge distribution at the active site of an enzyme stabilizes this transition state,

thereby lowering the barrier to activation (AG*) between substrate and transition state. If the barrier to activation is
lowered, the rate of a reaction can be accelerated significantly. The correct choice is D.
55.

C is correct. The last sentence of the third paragraph of the passage reads, "The neighboring environments of Glu

35 and Asp 52 are quite differentfrom each other." In the question it states that the pKa values for Asp 52 and Glu
35 are usually cited as 3.9 and 4.1, respectively. The difference between the two side chains is the additional
methylene (-CH2-) group in glutamic acid. Under normal conditions, the influence of this additional methylene
group has negligible consequences. Thus, under normal conditions we would expect both aspartic acid and glutamic
acid to have similar pKa values for their side chains. Since both side chains are on polar (charged) amino acids, we
would expect their environments also to be polar.
However, we know that the two environments are different from each other. We can assume that one environment is

polar and the other nonpolar. And we know which is which, based on information contained in the question:
"Analysis of lysozyme's active site indicates that the pKa of Asp 52 is still about 3.9, but the pKa of Glu 35 is now
about 6.6." The pKa value of the aspartic acid side chain has not changed. The pKa of the glutamic acid side chain
has increased by 2.5 times. This tells us that Asp 52 is in a polar environment, and that Glu 35 is in a nonpolar
environment. In order to remove the dissociable hydrogen from the side chain of Glu 35, the active site must be at a
pH close to a pKa of 6.6. In other words, it is harder to remove the dissociable hydrogen from the carboxyl side
chain of Glu 35 than it is from the carboxyl side chain of Asp 52. Recall from organic chemistry that dissociation of
a hydrogen from a carboxyl group is more favored in a polar environment than it is in a nonpolar environment.
Even though choice A is a correct answer, it does not address the question. What about choice B? The ionized
carboxyl group of Asp 52 is in a different environment than the protonated carboxyl group of Glu 35 (see Figure 2 in
the passage). If anything, the negative charge of the ionized carboxyl group would help stabilize the (partial)
positive charge of the hydrogen atom of the protonated carboxyl group. However, in this situation the two carboxyl
groups are distant from one another, and a stabilizing or destabilizing influence is negligible. In choice D, we see
that if the carboxyl group of Glu 35 were in a polar environment, then its pKa value would be around 4.1. Hydrogen
bonding would help in stabilization. But we already know that Glu 35 is in a nonpolar environment. With this
reasoning, we can eliminate choices A, B, and D. The correct choice is C.
56.

B is correct This question involves careful inspection of Figure 2, Figure 4, and Figure 5 in the passage. In Figure
2, the C-l and C-4 carbons are identified. Based on this, we can establish where the C-5 and C-6 carbons are

located. We know where to find the anomeric carbon (at the C-l position). The reference carbon is that carbon that
establishes whether a sugar is in the D or L configuration. By definition, the reference carbon is the last chiral
carbon farthest from the most oxidized carbon. In the case of sugar ring D, the most oxidized carbon is at C-l, while

the last chiral carbon is at C-5. Therefore, C-5 is the reference carbon. This is great, because it says that choice A
and choice C are one in the same. Since there should not be two correct answers to the same question, we can
eliminate both of them.

We are left with choice B and choice D. After examination of the hydroxyl groups on the C-6 carbons of Figure 4
and Figure 5, we can conclude that they are one in the same. We can eliminate choice D. By default, we are left with
choice B.

Copyright by The Berkeley Review

172

The Berkeley Review


Specializing in MCAT Preparation

Biology

Metabolic Components

Section VII Answers

Main Chain of Lysozyme Enzyme

Main Chain of Lysozyme Enzyme

Glu 35 |

Glu 35 I
O "

O-H

"""> H
CH,OH

CH,OH

^
."l

OH

Rings A-B-C.

Rings A-B-C.
NAM

N-H
i

= C

NAM
0

CH,

N-H

O = c

ox

x0
C

CH,

Asp 52

Asp 52

Main Chain of Lysozyme Enzyme

Main Chain of Lysozyme Enzyme

However, by examining Figure 4 and Figure 5, you should be able to see how the hydroxyl group of the water
molecule forms a bond with the C-l anomeric carbon, and how the remaining hydrogen of the water molecule forms
a bond with the carboxylate oxygen of Glu 35. This is indicated by the dashed lines in the two diagrams shown
above. The correct choice is B.

57.

A is correct. General acid-base catalysis involves the transfer of a proton in the transition state (see the protonated
form of Glu 35 in Figure 2 in the passage). This transfer does not involve covalent bond formation.
A covalent bond between two atoms like C and H involves the sharing of a pair of electrons. When a C-H bond
breaks, the electrons must do something. If one electron leaves with the hydrogen and the other stays with the
carbon, the reaction is referred to as homolytic cleavage, and two radicals are produced (e.g., H and C). If both

electrons leave with one atom, the reaction is referred to as heterolytic cleavage (e.g., #C,e and H or C and
H,e). In Figure 3 of the passage, we see a carbocation at the anomeric position. The correct choice is A.
58.

C is correct. Glutamic acid is an acidic amino acid with a negatively charged side chain at pH 7.4. The side chain is

polar and tends to interact with other polar molecules (like water). We would expect this amino acid to be found on
the outer surface of the enzyme, where it can be in contact with the aqueous medium. Note that in the diagram, the
a-helical region is towards the outer surface of the enzyme. We would not expect glutamic acid to be buried in the
interior of the protein, where the amino acid residues tend to be quite hydrophobic and nonpolar. Finally, as outlined
in the passage, Glu 35 (glutamic acid) is part of the catalytic group at the active site of the enzyme. The correct
choice is C.

Enzyme Kinetics II

Passage X (59 - 65)


59.

C is correct. When the substrate concentration is quite low, [S] is insignificant compared to Km- In the
denominator, Km + [S] becomes Km + [0], which is just Km. What this is telling us is that the initial reaction

velocity is almost proportional to [S] at very low values of [S]. In other words, at low [S] we observe a straight line
in the graph in Figure 1of the passage. This is characteristic of a first-order reaction. The correct choice is C.
60.

B is correct. When [S] is quite high, it means that Km is quite small. In the denominator. Km + [S] becomes [OJ +
[S], which is just [S]. This reduces the Michaelis-Menten equation to the following:
V =

Vn,ax[S]_
'=vr
[SI

In other words, at very high [S] the reaction approaches some maximal velocity (Vmax)- Note that the velocity of this
reaction is now independent of [S]. This is characteristic of a zero-order reaction. The correct choice is B.
61.

D is correct. If [S] = Km, then substitution into the Michaelis-Menten equation reduces it to:

v= Vmax[SJ_V,
[S] + [S]

[S] _ V,

2[S]

This expression is telling us that Km is that [S] where the reaction operates at one-half its maximal velocity. The
correct choice is D.

Copyright by The Berkeley Review

173

The Berkeley Review

Specializing in MCAT Preparation

Biology
62.

Metabolic Components

Section VII Answers

C is correct This question can best be answered by considering the reaction of E and S. As E and S combine, [S]
and [E] both decrease in order to make the ES complex. Therefore, we would expect the [ES] to increase. As the

[ES] begins to increase, some of it begins to getconverted to P. As a result, the [P] begins to increase. The correct
choice is C.

ki
E

k3
-

ES

"E

+ P

k2

Time

63.

A is correct The reciprocal of the Michaelis-Menten equation is given below. This formula is called the
Lineweaver-Burke equation. Note that it is in the form of the general y-intercept equation for a straight line: (y = mx
+ b).

JKM | J_

\VmJ [S]
The y-axis is 1/v. The slope of the line is KM/Vmax. The x-axis is 1/[S].The y-intercept is 1/Vmax- How do we get
the value of Km? In Figure 1 of the passage, Km is a particular [S] at which the enzyme is half-maximally saturated.
In this case, Km is also a particular [S]. However, the Km here is a reciprocal value (i.e., 1/Km). And since we are in
a quadrant that has a negative sign notation for the x-axis, we place a negative sign in front of 1/Km.It now reads 1/Km. All we need to do now is extrapolate the line from the y-axis to the x-axis as shown below.

i/v

'/v
Extrapolation

4/
-3

-2-10

-4-3-2-1.0

lf[S]

l/K M

l/[S]

The rest is just algebra. We find that:

-J-=-2
Km

Km=^- = + 0.5
-2

The Km value in Figure 1 of the passage is a positive value. Likewise, the Km of the double reciprocal plot should
also be a positive value. The correct choice is A.
64.

C is correct In order to answer this question, we must consider the values for Km and kcat in Table 1 in the passage.
If an enzyme is (almost) catalytically perfect, it must mean that it is extremely efficient at what it does. A measure of
an enzyme's efficiency is given by kCat/KM (from the passage). We also know that rate constant ki has limits

between 108 to 109 M^sec"1. This is telling us that if ki has these limits, then k3 must have them as well. The
amount of product that can be made is determined by how much of the ES complex there is available; and the ES
availability is determined by how fast E and S can come together. You do not need a calculator to do this problem.
All you need to work with are the exponents.

Copyright by The Berkeley Review

174

The Berkeley Review


Specializing in MCAT Preparation

Biology

Metabolic Components

Section VII Answers

Consider choice A: Carbonic anhydrase has a Km = 2.6 x 10"2 M and a kCat = 4.0 x 105 sec"1. If we consider the
exponents alone, we find that kcat/KM = 105/10'2 = 107 M"'sec"'. The actual value is 1.5 x 107 M"'sec"'.

Chymotrypsin has a Km = 6.6 x 10"4 M and a kcat = 1.9 x 102 sec"1. If we consider the exponents alone, we find that
kcat /Km = 102/10"4 = 106 M-'sec"1. The actual value is 2.9 x 105 M-'sec"1.

Therefore, for carbonic anhydrase, the efficiency is about 107 M"'sec"'. For chymotrypsin, the efficiency is about
106 M^sec"1. These exponents (107 and 106) are not too faraway from the limits of 108 to 109 M"'sec"'. Therefore,
this is a pretty efficient enzyme pair. But we do not know if it is the most efficient pair.

Consider choice B: Acetylcholine esterase has a Km = 9.5 x 10"5 M and a kCat = 1.4 x 104 sec"1. If we consider the
exponents alone, kcat /Km = 104/10"5 = 109 M"'sec"'. From choice A, we know that the efficiency for carbonic
anhydrase is 107 M"'sec"1. These two sets of exponents (109 and 107) are even closer to catalytic perfection.
Consider choice C: Fumarase has a Km = 5.0 x IO"6 M and a kCat = 8.0 x 102 sec"1. Therefore, kCat/KM = 102/10"6 =

108 M"'sec"'. From choice B, we know that the efficiency of acetylcholine esterase is 109 M"'sec"'. So far, these
two sets of exponents (108 and 109) appear to be the most efficient.
Consider choice D: Pepsin has a Km = 3.0 x 10"4 M and a kcat= 5.0 x 10"' sec1. The kcat/KM = 10"'/10"4, which is
10-* M"'sec"'. From choice A, we know that the efficiency of chymotrypsin is 106 M^sec"1. These two sets of
exponents (106 and 10-*) make this enzyme pair the least efficient. The correct choice is C.
65.

A is correct. According to the passage, the rate constant ki (for the reaction shown below) has limits between 108 to
109M"1s"1.

k]
E

+ S

k3

ES

k2

This value is predicted from diffusion theory and represents a very extreme case, where (k3 > ki) and every
substrate that collides with an enzyme is converted into an ES complex. This means that the efficiency of the

process is determined by how fast a substrate can be placed into the active site of the enzyme (forming the ES

complex). Having limits between 108 to 109 M'^'means that the enzyme is near catalytic perfection. Since triose
phosphate isomerase has an efficiency of 2.4 x 108 M'V1, it means that this enzyme is almost perfect. There is little
reason for it to change. This leads us to choices A and B.

Now, did this change occur early or late in evolution? The passage does not help you with this part of the answer.
Common sense will. Organisms are believed to have gained the ability to respire (use O2) some 2 billion years ago
(the numbers are not important here). Multicellular organisms are believed to have appeared some 700 million years

ago. Humans are multicellular organisms; yeasts are not. Long ago (early in evolution), there was a divergence in
our ancestral relationship with yeasts. At that divergence point, the isomerase enzyme was passed to each organism.
The enzyme must have been quite catalytically efficient at that time; otherwise, both lineages might have gone the
way of extinction. Since each lineage did not become extinct, the same enzyme (essentially) is present today. If the
maximal efficiency of the enzyme came about late in evolution, we would expect that one lineage might have a

nearly perfect enzyme, while the other lineage would not. This is because of different environmental forces acting
on the two species.

Even though both choice C and choice D are incorrect (primarily because of the first part of the statement), let's
briefly look at the last part of choice D. The isomerase enzyme does have the ability to change through amino acid
substitution (most likely through point mutations or single base changes). If a mutation is to be passed on to future
generations, it must allow the organism to survive and reproduce. Organisms do have the ability to change. The
correct choice is A.

Adenosine Triphosphate (ATP)

Passage XI (66 - 72)


66.

B is correct. ATP is the major carrier of chemical energy in all living cells. Many of the reactions in which ATP is
an intermediate involve a phosphoryl-group transfer from ATP to another molecule, or from an energy-rich
molecule to adenosine diphosphate (ADP) to form ATP. The simplest example of this type of group transfer reaction
can be taken from Table 1 in the passage. This table summarizes the phosphoryl-group transfer potentials of
hydrolysis of some important compounds in metabolism. If ATP is hydrolyzed to ADP and Pi (inorganic phosphate),

wateracts as the phosphoryl-group acceptor. The standard free energy of hydrolysis (AG"') for this reaction is -30.5
kJ/mol. This value lies between that of the hydrolysis of phosphoenolpyruvate (-61.9 kJ/mol) and the hydrolysis of
glyceroI-3-phosphate (-9.2 kJ/mol).
Copyright by The Berkeley Review

175

The Berkeley Review

Specializing in MCAT Preparation

Biology

Metabolic Components

Section VII Answers

Molecules that have a phosphoryl-group transfer potential greater than that of ATP (e.g., phosphoenolpyruvate)
transfer a phosphoryl group to ADP to form ATP. Molecules that have a phosphoryl-group transfer potential less
than that of ATP (e.g., glycerol-3-phosphate) acquire a phosphoryl group from ATP to form ADP and the molecule
being phosphorylated. Which of the compounds listed in the answerchoices can be phosphorylated exergonically by
ATP? We know from general chemistry that reactions that are exergonic are spontaneous and proceed with a release
of free energy. In this case, the change in free energy (AG) is negative, because the products of these types of
reactions have less free energy than the reactants. Reactions that are endergonic are not spontaneous, and they
require an input of free energy. The AG for these reactions is positive, because the products have more free energy
than the reactants. We are therefore looking for compounds that have a lower AG' value than ATP.
Phosphoenolpyruvate, acetyl phosphate and pyrophosphate all have AG' values that are higher than ATP, allowing
us to eliminate choices A, C, and D. Glucose-1-phosphate has a AG' value of-20.9 kJ/mol, which is clearly lower
than the AG8' value for the hydrolysis of ATP. The correct choice is B.
67.

C is correct. Molecules can move across cell membranes by several processes. Diffusion is the net movement of
molecules from a high concentration to a low concentration. Osmosis is the net diffusion of water from a region of
low solute concentration (i.e., high water concentration) to a region of high solute concentration (i.e., low water
concentration). Passive transport (i.e., simple diffusion) does not require carrier molecules or an expenditure of
energy, but rather is the net movement of molecules down their concentration gradient across a membrane. Active
transport is a carrier-mediated process that requires the input of cellular energy and makes possible the transport of
molecules across a membrane against their electrochemical gradient. Only active transport works against the
concentration gradient and requires an input of energy in the form of ATP. Choices A, B, and D are incorrect. The
correct choice is C.

68.

D is correct Any compound that inhibits ATP production, such as cyanide in the electron-transport chain, is called
a poison. ATP is the cell's energy currency, and without it the cell's metabolic processes slow down and the cell
eventually dies. A human being has an energy reserve of about four minutes of ATP and other phosphate
compounds in every cell. This is one of the reasons brain damage is likely to occur after someone has been without a
heartbeat for about four minutes and then is revived. The cell is set up to use ATP, so it cannot suddenly switch to
other phosphate compounds and survive indefinitely. Choices A, B, and C are incorrect. The correct choice is D.

69.

A is correct. If the phosphoanhydride bond in any one of the answer choices is hydrolyzed to yield the product
compound plus Pi, the energy release is in the form of heat. For example, if the phosphoanhydride bond in
phosphoenolpyruvate is hydrolyzed to give pyruvate and Pi, the standard free-energy change of hydrolysis is -61.9
kJ/mol. If the hydrolysis of an energy-rich compound releases more free energy than the hydrolysis of ATP, then
through phosphoryl-group transfer these energy-rich compounds can transfer a phosphoryl group to ADP to form
ATP. Note that the reaction for the phosphorylation of ADP is the reverse of the reaction for the hydrolysisof ATP.
This means that the hydrolysis of an energy-rich compoundcan be coupled to the synthesis of ATP. We can see this
in the coupled example shown below:
o

II
ooc

c=o

Phosphoenolpyruvate

Pyruvate

0
61.9kJ/mol

I
O

II

II
+

00
0

Phosphate
000

AdenosineOPOPO

II

HOPO

CH3

II

+ H2<>

00

II

AG'1

0
COO

A
H

opo

II

II

AdenosineOPOPOPO

HOP 0

Adenosine diphosphate (ADP)

HoO

+ 32.2 kJ/mol

Adenosine triphosphate (ATP)

Phosphoenolpyruvate + ADP

Pyruvate + ATP

- 29.7 kJ/mol

The compound among those in the answer choices that releases the most energy upon hydrolysis would also release
the most heat after the phosphorylation of ADP. This compound is phosphoenolpyruvate. Choices B, C, and D are
all lower in energy released upon hydrolysis and are incorrect. The correct choice is A.

Copyright by The Berkeley Review

176

The Berkeley Review


Specializing in MCAT Preparation

Biology
70.

Metabolic Components

Section VII Answers

C is correct Of the metabolic processes presented to us, three generate high-energy phosphate compounds (i.e.,
ATP or GTP), and one does not. We are looking for the one thatdoes not. Threeof the metabolic processes involve
the oxidation of fuel molecules: glycolysis, citric acid cycle, and oxidative phosphorylation. All of these pathways
produce either ATP or GTP. These metabolic processes are catabolic, and they release energy in the form of ATP
and heat. Protein synthesis, being anabolic, requires an enormous input of energy (in the form of GTP)possibly as
much as 90% of the chemical energy used by a cell during the course of its biosynthetic reactions. Choices A, B, and
D are incorrect. The correct choice is C.

71.

B is correct The bond indicated by the arrow in the ATPmolecule, between the 5' carbon of the ribosering and the
ot-phosphate, is a phosphoester bond. There are two phosphoanhydride bonds. One is located between the a- and 0phosphates, and the other is located between the p- and y-phosphates. Eliminate choice A.
Phosphoester

bond

I I II

/ 5*

OP OPO POCH2

II
o

A
T

II
o

A
n

II
o

N-Glycosidic
bond

Phosphoanhydride
bonds

The only type of glycosidic linkage found in ATP is an N-glycosidic linkage, located between the 1' carbon of the
ribose ring and the N-9 nitrogen of the nitrogenous base. Eliminate choice C. Peptide bonds are formed between an
N-terminus nitrogen and a C-terminus carbon of amino acids. Eliminate choice D. The correct choice is B.
72.

B is correct Pyrophosphate (PPi) is a compound that contains two phosphate residues, and depending upon the pH
of the solution, can exist in several ionic forms. For example, there are times when the molecule might bear two
negative charges, and there are times when it might bear three or four negative charges. The electrostatic repulsion
between the negatively charged oxygen atoms of PPi provide a portion of the driving force for its hydrolysis to two
molecules of inorganic phosphate (Pi). Since PPi is rather unstable, statement I is incorrect. Based on the values for
the standard free energies of phosphate hydrolysis in Table 1 in the passage, we see that more energy is released for
the hydrolysis of ATP to AMP and PPi than for the hydrolysis of ATP to ADP and Pi. Statement II is correct. Since
PPi can bear a variety of negative charges in the cell, it is quite soluble. Statement III is therefore incorrect. The
correct choice is B.

Protein Degradation (Ubiquitin)

Passage XII (73 - 79)


73.

B is correct The pH of the cytoplasm is approximately 7. These lysosomal enzymes would be virtually inactive in
the cytoplasm. This is a protective mechanism, so that a rupture in a lysosome does not lead to the destruction of all
intracellular components. Choice A is incorrect. The acidic or basic nature of the proteins does not matter. They are
all degraded with the same degree of efficiency. Choices C and D are incorrect. The correct choice is B.

74.

C is correct A region of DNA is considered to be highly conserved if it is very similar in the genetic material of
two or more organisms. That means choice A is incorrect. The phyla have nothing to do with conservation in this
question. Choice B is incorrect. "Highly conserved" means virtually identical, while "completely conserved" would
mean completely identical, so choice D is incorrect. The correct choice is C.

75.

C is correct Ubiquitin attaches most quickly to those enzymes that are at metabolic control points in a biochemical
reaction. Some pathways operate pretty much constitutively (all the time), such as glycolysis, the Krebs cycle, the
electron-transport chain, and oxidative phosphorylation. Conversely, your body closely regulates pathways
involving gluconeogenesis, amino acid catabolism, nucleic acid synthesis, or the synthesis of spermine and
spermidine, two polyamines used in DNA-packaging.
Cytochrome c is a peripheral membrane protein found on the side of the inner mitochondrial membrane that faces
the intermembrane space. This protein passes electrons from Complex III of the electron-transport chain to Complex
IV. Under normal cellular conditions, this enzyme has a half-life of about 150 hours and, as indicated in Table 1 in
the passage, is a long-lived enzyme that has a stabilizing N-terminal amino acid residue. It turns out that the Nterminal amino acid in human cytochrome c is glycine (Gly). Since the ubiquitin system is used to destroy abnormal
proteins and short-lived enzymes, we can eliminate choice A.

Copyright by The Berkeley Review

177

The Berkeley Review


Specializing in MCAT Preparation

Biology

Metabolic Components

Section VII Answers

Glyceraldehyde-3-phosphate dehydrogenase is a glycolytic enzyme involved in the conversion of glyceraldehyde-3phosphate to 1,3-bisphosphoglycerate. Its long hall-life of 130 hours indicates that the N-terminal amino acid has a
stabilizing effect. Eliminate choice D.

The remaining two proteins are both short-lived enzymes. Tyrosine aminotransferase has a half-life of 120 minutes
and is involved in the catabolism of the amino acids phenylalanine and tyrosine to acetoacetate and fumarate.
Ornithine decarboxylase has a half-life of about 12 minutes and is involved in the synthesis of spermine and
spermidine, two polyamines used in the packaging of nucleic acid. Even though both of these proteins have
relatively short half-lives, and they each have a destabilizing N-terminal amino acid, we want to select the one that is
most likely to be modified quickly by ubiquitin. The protein we want is the one with the shortest half-life, ornithine
decarboxylase. A short half-life usually means that a molecule is being turned over quickly, and in the case of
rapidly dividing cells, ornithine carboxylase is required in large amounts. With such a short half-life, UCDEN is
probably degrading ornithine decarboxylase just as fast as new proteins are being synthesized to replace it. The
correct choice is C.

76.

B is correct. Look at Table 1 in the passage. The stablest enzyme has the longest half-life. Lysine and leucine both
have half-lives of about 3 minutes. Eliminate choices A and D. Glutamic acid has a half-life of about 30 minutes.

Eliminate choice C. Glycine has a half-life of more than 20 hours. The correct choice is B.

77.

A is correct. During a fast, amino acids are still needed for the repair and synthesis of proteins. Since the person is
not eating, these amino acids must come from the body's stored protein, the muscles. Following a meal, dietary
amino acids are readily available, and lysosomal degradation of proteins should be low. Choice B is incorrect.
During exercise, some proteins may be degraded, but not as many as are degraded during a fast. Choice C is
incorrect. During pregnancy, women are usually well-fed and not fasting. Choice D is incorrect. The correct choice
is A.

78.

C is correct. The conversion of ATP to AMP and PPi (pyrophosphate) involves the hydrolysis of the

phosphoanhydride bond between the (3- and y-phosphates of ATP. The pyrophosphate thus formed is rapidly
hydrolyzed to two molecules of Pi (inorganic phosphate). In this reaction sequence, two phosphoanhydride bonds
have been hydrolyzed. Choices A, B, and D are incorrect. The correct choice is C.

79.

A is correct. The action of lysosomal enzymes requires a pH of approximately 5. Chloroquine, in the uncharged
form, diffuses across the single membrane of the lysosome and accumulates inside. Due to the acidic medium inside

the lysosome, chloroquine becomes protonated and begins to accumulate in the charged form. This increases the pH
within the lysosome, leading to inactivation of the enzymes that require a low pH optimum and a subsequent
decrease in protein degradation. Choice B therefore can be eliminated. Since there is no indication in the passage
that chloroquine modifies the active site of a protease, we must assume that it does not, so choices C and D can also
be eliminated. The correct choice is A.

Passage XIII (80-87)


80.

Nutrients and Proximate Analysis

D is correct. Melting is the change of a solid to a liquid, so choice A is incorrect. Vaporization is the change of a
liquid to a vapor phase, and that is not what is involved in freeze-drying. Choice B is therefore incorrect.

Condensation is the change of a vapor to a liquid or a solid. Choice C is incorrect. Sublimation is the change of a
solid to a vapor, which is an accurate description of the freeze-drying process. The correct choice is D.

81.

D is correct. In the third paragraph of the passage, we learn that vitamins are organic compounds. Therefore, they
can be combusted to yield CO2 and H2O, among other gases. Minerals are just inorganic ions and atoms and would
remain in the ash portion. The correct choice is D.

82.

A is correct. In the second paragraph of the passage, we find that magnesium is not a trace mineral; it is a
macromineral. Zinc, manganese, and iodine are trace minerals. The correct choice is A.

83.

84.

A is correct. According to the last paragraph in the passage, a protein is about 16% nitrogen. The correct solution is
obtained by dividing 5 grams nitrogen by 16% nitrogen per sample of protein, which gives 31 grams of protein. The
answer for choice B is obtained by multiplying 5 grams by 16%. This gives 0.8 grams of protein, which is incorrect.
Choices C and D are the same answers multiplied by some power of 10. The correct choice is A.
A is correct. The calorie content of food is another way of saying its energy content. Only proteins, carbohydrates,
and lipids provide energy. Statements I and III are correct, which means that choice A is also correct. Vitamins do
not provide energy themselves, although they are involved in metabolic reactions. This means that statement II is
incorrect, which allows us to eliminate choices B, C, and D. The correct choice is A.

Copyright by The Berkeley Review

178

The Berkeley Review

Specializing in MCAT Preparation

Biology
85.

Metabolic Components

Section VII Answers

C is correct. Choices A and B make you think about solubility. The blood is an aqueous medium. Water-soluble
vitamins could travel freely in it, while fat-soluble vitamins (like fats) would need a transport protein. Excess watersoluble vitamins would be excreted into the urine by the kidney. Vitamin K is important for blood clotting, so an
antagonist would decrease the ability to clot, leading to longer clotting times. Choices A, C, and D are thus all true.
The false (and best) answer is choice C. Ascorbic acid (vitamin C) is water-soluble. The correct choice is C.

86.

B is correct. In the first paragraph of the passage, we learned that a triglyceride contains 9 kcal/gram. 30% of 1800
kcal = 0.30 x 1800 = 540 kcal. 540 kcal/9 kcal per gram = 60 grams triglyceride. The answers in choices A, C, and
D are the results of various incorrect manipulations of the data. The correct choice is B.

87.

D is correct. We are looking for the answer that does not produce water. The last step of the electron-transport chain
uses electrons to reduce oxygen, producing water. Choice A is true. Synthesis of protein or glycogen involves
condensation reactions and the production of water. Choices B and C are true. The breakdown of a triglyceride
requires the use (not the production) of water in a hydrolysis reaction. Choice D is the false answer, because it does
not produce water. The correct choice is D.

Passage XIV (88 - 94)


88.

Niacin Experiment

C is correct. This question cannot be answered based on information in the passage and requires previous
knowledge. The release of fatty acids from glycerol is regulated by the actions of insulin, epinephrine, and glucagon
through a reaction catalyzed by an enzyme called hormone-sensitive triacylglycerol lipase. Adipocytes hydrolyze
triglycerides (i.e., triacylglycerols) to free fatty acids and glycerol, as shown in the reaction below:
o

ii

O
II

H,COCR
"I

RCOCH

H,COH

3 H20

II

-I
HO C H

Lipase

H,COCR

Glycerol

Triglyceride

hoCR

H2COH

Fatty acid

One of the intermediates in the synthesis of triglycerides is the molecule glycerol-3-phosphate. If the concentration
of this molecule is low in the adipocyte, the free fatty acids produced in the reaction shown above are released into
the bloodstream. They are not reesterified to triglycerides. The ratio of glycerol to fatty acids would therefore be 1:3.
The correct choice is C.

89.

B is correct. The isotopically labeled molecules listed in the question are not radioactive. Therefore, scintillation

counting or using a Geiger counter would not work. Both 13C and 2H are stable isotopes of their parent atoms.
Choices A and C are incorrect. The only difference between 12C and ,3C is one neutron in the nucleus. The same is
true of hydrogen and deuterium (2H). The molecules can be separated by mass only, using a mass spectrometer.
Choice B is correct. A UV spectroscope would not help the situation either, since mass is the only critical piece of
data to gather. Choice D is incorrect. The correct choice is B.
90.

D is correct. The pH is adjusted to 7.0 so that glycerol is not charged. Therefore, glycerol would not interact with
either the cation exchange resin (where the beads are negatively charged) or the anion exchange resin (where the
beads are positively charged). Glycerol would pass through the apparatus and end up in the water in the collection
tube. Choice A, B, and C are incorrect. The correct choice is D.

91.

A is correct. Since this cycling is well-regulated and operates almost 24 hours per day, it is doubtful that the
normally cycling quantities of fatty acid damage either the liver or the adipose tissue. Choice C and D are incorrect.
The cycle does, however, require energy to operate. It is a futile cycle, one that operates in a circle but uses energy.
Choice B is incorrect. The correct choice is A.

92.

D is correct. The turnover of fatty acids and glycerol means they are entering the bloodstream and exiting the
bloodstream. Turnover is a measure of how rapidly this entering and exiting occurs. In the pre-niacin state, you can
see that lipolysis of triglycerides in the adipose tissue must be occurring, since there is a non-zero value for both
fatty acid and glycerol turnover. Choice A is incorrect. You can also see that some fatty acids are being reesterified,
even in the pre-niacin state. If there were no reesterification, then the fatty acid turnover value would be three times
the glycerol turnover value. Since this is not the case (1.3 x 3 = 3.9), then there must be some reesterification in the
pre-niacin state. Choice C is incorrect. Niacin treatment did not increase lipolysis, since the glycerol value did not
change in the pre and post states. Only the value for the fatty acid turnover decreased, so the change was in the
reesterification of fatty acids. Choice B is incorrect. The correct choice is D.

Copyright by The Berkeley Review

179

The Berkeley Review


Specializing in MCAT Preparation

Biology
93.

Metabolic Components

Section VII Answers

C is correct. These free fatty acids are crucial for use as energy during rest and during exercise. The heart

preferentially uses free fatty acids as fuel, as does the skeletal muscle at rest. This drug probably would be called a
poison, since a large dose of it would mean death. Anyway, the body could not switch to metabolizing ketone
bodies, since they are formed from fatty acids. Choice A is incorrect. The brain and nervous tissue rely primarily on
glucose as fuel, so choice B is also incorrect. Cholesterol does not contain usable energy for human beings, so
choice D is incorrect. The correct choice is C.

94.

A is correct. Hormone-sensitive lipase hydrolyzes triglycerides into glycerol and fatty acids. These are both released
from the cell. Blood levels of both would increase following the ingestion of caffeine. Choices B, C, and D are
incorrect. The correct choice is A.

Passage XV (95 - 100)


95.

Diffusion-Limited, Enzyme-Catalyzed Reactions

D is correct. Equilibrium is a stable condition, a state where no further net change is occurring. In a living system,
this condition means vital reactions are no longer netting vital products. Without the expenditure of energy and the

net production of vital products, the organism will decay and approach death. The correct choice is D.
96.

D is correct. The question describes the relationship between time and distance traveled when molecules are
diffusing through a medium: If a molecule requires 1 second to travel 1 micron, then it requires 9 seconds to travel 3
microns, and 16 seconds to travel 4 microns. From this information, we can see that as the distance increases, the

efficiency of diffusion decreases. Therefore, diffusion is inefficient over long distances, but efficient over short
distances. The correct choice is D.

97.

C is correct. We are told in the passage that it is the frequency of collisions between substrate and enzyme that is
the rate-determining step in diffusion-limited reactions. It should follow that if we can bring the substrate and the
enzyme closer together, we can increase the rate of reactions. A multienzyme complex is exactly what its name
implies, a group of enzymes involved in a reaction pathway. By bringing the enzymes close together, we decrease
the distance one product has to travel to the next enzyme to continue the reaction chain. By decreasing the distance,
we increase the frequency of collisions and increase the rate of reaction. The correct choice is C.

98.

A is correct. The question informs us of a membrane-bound compartment that occupies 20% of the total cell
volume. If we put all of the substrate into this compartment, its concentration could be as much as 5 times greater
than it is in the cytosol. How do we arrive at this figure? Let us assume that the concentration in the cytosol is 1 unit
of solute tol unit of volume. In the compartment, the volume is reduced to 0.2. Now, the concentration is 1 unit of
solute to 0.2 units of volume, which gives us a concentration that is 5 times greater than in the cytosol. The correct
choice is A.

99.

C is correct. This answer can be arrived at simply by looking at the time it takes for these molecules to hit their

target. In Trial C, it takes 9 seconds for the molecule to find its target. If we convert 9 seconds into minutes, we get
0.15 minutes, so the frequency of collisions in Trial C is 100 times greater than in Trial A. The correct choice is C.
100.

D is correct. This answer can be arrived at by thinking about movement in different dimensions. Recall that
membranes accelerate diffusion-limited reactions by limiting movement to only two dimensions. The same kind of
thinking can be applied to DNA-binding proteins, as they diffuse through the nucleoplasm in search of their binding
site(s). It is not efficient for the binding protein to jump onto and off the DNA molecule at random places to find its
binding site, because it might miss something. It is more efficient for the binding protein to limit its movement to
going up and down the DNA molecule until it finds its appropriate binding site, indicated by a strong binding
affinity. The correct choice is D.

Copyright by The Berkeley Review

The Berkeley Review


Specializing in MCAT Preparation

180

Biology
Section VIII

A. Metabolic Pathways
1.

Glycolysis

2.

Disaccharide Metabolism

Metabolic

3.

The Krebs Cycle

4.

Electron Transport

Pathways

5.
6.

Oxidative Phosphorylation
Pentose Phosphate Pathway

7.
8.

Gluconeogenesis
Fatty Acid Oxidation

9.

The Urea Cycle

Fatty Acids
Pyruvate

Amino Acids

Practice Passages and Answers

NADH + H+

x ) ^~ OAA

NAD+ l^^
Malate

Krebs
Fumarate

cycle

Isocitrate
3 y nad+

fadh2^J\6
FAD A\

ocKetoglutarate

Succinate
CoA + GTP

GDP + P;

4 / NAD+ + CoA

Succinyl-^(VNADH +H+
CoA

co,

Tfiz

BERKELEY
Ur-E'V'^E-W8
Specializing in MCAT Preparation

Metabolic Pathways
Top 10 Section Goals
Understand the general concepts behind the glycolytic pathway.

> Glycolysis is one of the pathways of central importance to metabolism. You should have a basic
understanding of how it operates, what it generates, and how it is tied in with the Krebs cycle.

Be familiar with the common types of sugars that are metabolized.


Sugars other than glucosecan be hydrolyzedin metabolism. Sucrose, lactose, fructose, and galactose
are but a few of the sugars that are commonly metabolized by a wide variety of organisms.
Be able to link the concepts of glycolysis and the Krebs cycle.

Once thecommitment to aerobic respiration is made, pyruvate is converted to acetyl-CoA, which


then enters the Krebs cycle. This link unites glycolysis with the Krebs cycle ana is important.

Understand the importance of the coenzymes NADH and FADtl

NADH is generated in both glycolysis and the Krebscycle. FADH2 is generated in the Krebs cycle.
Understand how these reducing components are important for the generation of ATP.
Know how the electron-transport chain operates.
It is important to have an understanding of how electrons flow from NADH and FADH2down the
electron-transport chain to oxygen, the ultimate electron-acceptor.

*9

Understand the chemiosmotic hypothesis and the concept of a proton gradient.

Understand how a proton gradient is generated, and how it leads to the eventual synthesis of ATP
in oxidative phosphorylation. Be familiar with inhibitors and how they work.
Be familiar with the general concepts of the pentose phosphate pathway.

Do not memorize the structures of the pentose phosphate pathway.


why this
ay. Just understand
unde
pathway is important. Many of its componentsare also found in the Calvin
" cycle.

Understand the relationship between glycolysis and gluconeogenesis.


Be aware that gluconeogenesisis not the complete reverse of glycolysis, because it has irreversible
steps that must be bypassed. Know the importanceof gluconeogenesis in metabolism.
Know the concepts involved in the (3-oxidation pathway.

Fatty-acid oxidation isimportant inmetabolism. There aremany classic biochemical reactions in this
pathway that illustrate the values of acquiringa good foundation in organic chemistry.

Be able to relate the urea cycle to the functions of the liver and the kidney.

Know where cells inthebody generate urea, andhave asolid grasp oftheconcept behind transamination

reactions. Understand how nitrogenous wastes are removed from the body and in what form.

Biology

Metabolic Pathways

Glycolysis

Metabolic Pathways
t^mxi

'0$!&f&^:
Phase I

The ideas thatwehave recently been considering can beput toimmediate use by
examining the reactions in glycolysis. The first stepin glycolysis is shownin Step
1 in Figure 8-1. The most common and wide-spread enzyme that makes use of
glucose whenit enters the cellis hexokinase. A kinase is an enzyme that involves
the transfer of a terminal phosphate group of an ATP unit to some other

compound. In this case it is D-glucose. When ATP transfers its phosphate onto
D-glucose it doesso at the C-6 position. We willend up witha compound called
Glucose-6-phosphate. Note that we have used 1 ATP. This is an investment step
and is an important control point in glycolysis. Why did we bother to attach a
phosphate onto glucose in the firstplace? Glucose doesnot haveany charges and
can pass back and forth across the cell's membrane relatively easily. However,
once it is phosphorylated it picks up some negative charges and can no longer
pass across the cell's membrane. Glucose, in the form of glucose-6-phosphate, is
trapped inside the cell.The AG0f for this reaction in Step 1 is -4.0 kcal/mol.

AG0' * -4.0

H-C- OH

H-C- OH

AG+0.4

Hexokinase
HO-C-H
I

H-C- OH
I

H-C- OH
I

ATP

ADP

Stepl

CH,-OH
I

Phosphogluco

C=0

isomerase
HO-C-H
I
H-C- OH

HO-C-H

Step 2

H-C- OH

H-C- OH
I

CHr0H

CH

ii

H-C- OH

II

CH,O- P-0

o-p-o

Glucose

0
Glucose-6-phosphate

Fructose-6-phosphate

Figure 8-1

Step 2 in glycolysis involves an equilibrium between two isomers, glucose-6phosphate and fructose-6-phosphate. The reaction takes place at the C-l and C-2
carbons of glucose-6-phosphate and proceeds through an enediol intermediate.
The compound that emerges from this reaction is Fructose-6-phosphate as
shown in Figure 8-1. The enzyme involved here is called phosphoglucose
isomerase. Fructose-6-phosphate is the phosphorylated version of D-fructose (a
keto sugar). The AG* for this reaction is +0.4 kcal/mol.
Step 3 involves a second investment of an ATP molecule and is yet another control
point in glycolysis. The general class of enzyme involved here is a transferase
called phosphofructokinase. This enzyme catalyzes the conversion of fructose-6phosphate to Fructose-l,6-diphosphate. The AG' for this reaction is -3.4
kcal/mol as shown in Figure 8-2.

Copyright by The Berkeley Review

183

The Berkeley Review


Specializing in MCAT Preparation

Biology

Metabolic Pathways

Glycolysis

CH2-OH

ii

AG0 -3.4

Phosphofructo

c=o
I

ch2-o-p-o
1

C=0

kinase

HO- C-H

HO- C-H
1

ATP

H-C- OH

ADP

H-C- OH

Step 3

H-C- OH

H-C- OH
I

CH2-0-P-0

CH2-0-P-0

Fructose-6-phosphate

Fructose-1,6-diphosphate

Figure 8-2

In Step 4 fructose-l,6-diphosphate undergoes an interesting reaction in which the


bond between the C-3 and C-4 carbons is broken. This reaction involves a reverse

aldol condensation and the products are dihydroxyacetonephosphate (DHAP)


and glyceraldehyde-3-phosphate. DHAP and glyceraldehyde-3-phosphate are
the two simplest and smallest carbohydrates. Both are trioses. If DHAP were to
react with glyceraldehyde-3-phosphate, forming fructose-l,6-diphosphate, the
reaction would be called an aldol condensation. The enzyme involved here is
aldolase. It is in the class of enzymes referred to as lyases. The AG' for this
reaction is +5.7 kcal/mol as shown in Figure 8-3.

O
11

II

CH2-0-P-0
1

c= o

~o

n.

AG0 - +5.7

HO- C-H
H-C- OH

Aldolase
""

Step 4

H-C- OH

CH2-0-P-0

II

<r

cH2-oH

DHAP

,0

+
H-C- OH

CH2O-P-0

O
II

CH2-0-P-0

o
Fructose-1,6-diphosphate

Glyceraldehyde-3-phosphate

Figure 8-3

In Step 5 DHAP can be converted into glyceraldehyde-3-phosphate. These


compounds are isomers and the reaction is catalyzed by triose phosphate
isomerase. All of the other significant reactions in glycolysis proceed from
glyceraldehyde-3-phosphate. Even though about 96% of the of the triose
phosphate molecule at equilibrium is DHAP, once glyceraldehyde-3-phosphate
is removed, DHAP is isomerized to glyceraldehyde-3-phosphate (recall Le
Chatelier's principle). In other words, from one molecule of fructose-1,6diphosphate you can get two molecules of glyceraldehyde-3-phosphate. The
AG' for this reaction is +1.8 kcal/mol as shown in Figure 8-4.

Copyright by The Berkeley Review

184

The Berkeley Review


Specializing in MCAT Preparation

Biology

Metabolic Pathways

AG0 =+1.8

CH - 0- P- O0 TriSe Phosphate
I 2

c=o

,0

V'
I

isomerase

fe

StepS

CH2-OH

Glycolysis

H-C- OH
1

O
11

CH2-0-P-0

DHAP

Glyceraldehyde-3-phosphate

Figure 8-4

At this point we have completed Phase I of glycolysis. We haveconverted the 6

carbons of the original glucose molecule into 2 three carbon phosphorylated


intermediates. Because these two phosphorylated intermediates are intercon
vertible, we have a "pool" of triose phosphates. You should remember, we

actually have millions and millions ofglucose molecules forming twice as many
millions and millions of triose phosphate molecules. Up to this point the overall
oxidation states of the carbons has remained constant. There have been no
oxidation or reduction reactions.

Phase II

Let's now consider Phase II of glycolysis. In Step 6 we will see a change in the
oxidation state of the C-l carbon of glyceraldehyde-3-phosphate. The oxidation
state of the C-l carbon on glyceraldehyde-3-phosphate is +1. The enzyme
glyceraldehyde-3-dehydrogenase willconvert glyceraldehyde-3-phosphate into
1,3-Diphosphoglycerate (abbreviated as 1,3-DPG). Note that the C-l carbon of

1,3-DPG has an oxidation state of +3. Wehave a change in oxidation states form
+1 to +3. This is a two electron oxidation. When you look at 1,3-DPG, note the
mixed anhydride linkage and the monophosphoester linkage. How did this mixed
anhydride linkage get there? We needed a phosphate in order to make this
change, and that phosphate comes from inorganic phosphate (Pi) and not from
ATP. This inorganic phosphate and glyceraldehyde-3-phosphate have to be put
together. There must also be an oxidizing agent that will oxidize the C-l carbon of
glyceraldehyde-3-phosphate. This oxidizing agent is NAD. The AG value for
this reaction is +1.5 kcal/mol (see Figure 8-5).

Mixed Anhydride

AG0'+1.5
H

Glyceraldehyde-3-phosphate
dehydrogenase

.0

's

+1C
I

H-C- OH
1

O
II

CH,-O-P-0

Glyceraldehyde-3-phosphate

o'

NAD+ NADH
+ Pi
+ H+
Step 6

Linkage ^,

Ov

O-P-0

+3

H-C- OH

II

CH2- O-P-0
I

1,3-Diphosphoglycerate

Figure 8-5

Themixed anhydride linkage in 1,3-DPG is relatively unstable, thus giving that


compound a high phosphoryl transfer potential. If we were to carry out the
Copyright by The Berkeley Review

185

The Berkeley Review

Specializing in MCAT Preparation

Biology

Glycolysis

Metabolic Pathways

hydrolysis of 1,3-DPG, the AG' would be about -11.8 kcal/mol. However,


instead of usingwaterin this reaction we will use ADP. The enzyme involved
here, phosphoglycerate kinase, will not operate unless thereis ADPpresent. The
product of this reaction, as shown in Step 7 (Figure 8-6), is 3-phosphoglycerate
and ATP. We now have a return on our investment in the form of ATP. In fact,

we have a return of 2 ATP's (recall we split our 6-carbon glucose molecule into
two 3-carbon triose phosphate molecules).

Os

O-P-0

Phosphoglycerate

11

kinase

Os o0
V

H-C-OH
I

O
II

T
ADP

H-C- OH

ATP

CH2" O-P-0

CH2-0-P-0
i

Step 7

0
3-Phosphoglycerate

1,3-Diphosphoglycerate

Figure 8-6

In Step 8,3-phosphoglycerateis converted to 2-phosphoglycerate by the enzyme


phosphoglyceromutase (see Figure 8-7). The phosphate group at the C-3
position of 3-phosphoglycerate is not the same phosphate at the C-2 position of
2-phosphoglycerate. This is because the enzyme itself is a phosphorylated enzyme.
During an intermediate state the phosphorous from the enzyme gets transferred to
the C-2 position of 2-phosphoglycerate while the phosphorous from the C-3
position of 3-phosphoglycerate gets transferred to the enzyme. An exchange has
taken place. The AG' for this reaction is +1.1 kcal/mol. [Note: Recall that when
we discussed hemoglobin, we encountered 2,3-bisphosphoglycerate (2,3-BPG).
This molecule binds in the middle of the hemoglobin molecule and stabilizes the
deoxy state of that moleculeso more oxygen is released to the tissue. It is at this
point in the cellthat 2,3-BPG can be made. The enzyme that would catalyze this
reaction would be 2,3-BPG phosphatase (phosphatases catalyze the hydrolysis of
phosphate esters).]
AG0 +1.1
Phosphoglycero

mutase
i

H-C- OH

II

Step 8

Enolase

ii

0Q

HCO-P-0

HO-H2C

CH,O-P-0

AG0' +0.4 *c ,

0.

Step 9

rt

11

c-o-p-o
II
I

CH2

0@

Phosphoenolpyruvate

2-Phosphoglycerate

3-Phosphoglycerate

Figure 8-7

In Step 9 we have the loss ofwater from 2-phosphoglycerate. This involves the
hydroxyl function at the C-3 carbon and the hydrogen at the C-2 position (see
Figure 8-7). Theenzyme enolase catalyzes the reaction of 2-phosphoglycerate to
phosphoenolpyruvate (PEP). PEP has an unstable high energy phosphate bond.
Why? The phosphate moiety attached to the C-2 carbon is in a planar
arrangement with the rest of the molecule because of the enol configuration.
Copyright by The Berkeley Review

186

The Berkeley Review

Specializing in MCAT Preparation

Biology

Metabolic Pathways

Glycolysis

Electrostatic repulsion is quite pronounced in this situation. The AG' for this
reaction is +0.4 kcal/mol.

If we hydrolyze PEP, we would get the enol of pyruvate plus Pi. The enol form
would be in equilibrium with the keto form. These are referred to as tautomers.

However, the equilibrium between the enol and the keto forms is very much in

favor of the keto form. This is shown in Figure 8-8.

o,

Step 10 is the last reaction in glycolysis (see Figure 8-9). The enzyme pyruvate
kinase will catalyze the transfer of a phosphoryl group from PEP to ADP and
give us pyruvate and ATP. Pyruvic acid may also be called 2-keto-propionic acid
or a-keto-propionic acid. Thisphosphorylation is nonoxidative. Recall that when

V
I
COH
II

CH2

V
I
c=o

CH,

ADP is converted to ATP the AG' is +7.3 kcal/mol. The high energy bond in

enoI-Pyruvate

an overall AG0' of -7.5kcal/mol for Step 10. Note that since we started with a 6-

Figure 8-8
Enol-keto equilibrium with

carbon glucose molecule we now have two 3-carbon pyruvate molecules. In this

pyruvate.

PEP has a AG0' of about -14.8 kcal/mol. Coupling these two reactions will give

keto-Pyruvate

step we have actually made 2 ATP's (netgain).

AG0'-7.5

C
O
l
ll

C-O-P-0

CH2

O0

Phosphoenolpyruvate

Pyruvate
Kinase

S
ADP

o.

0~

V
I

c=o

ATP

Step 10

CH3
Pyruvate

Figure 8-9

Now that we have finished glycolysis, we need to consider the overall


consumption and generation ofATP. Recall that in Step 1 and in Step 3 we invested
1 ATP each. Thus,our total investment is 2 ATP's. In Step 6 we got 2 ATP's back
from our investment (remember, we have two 3-carbon compounds at this
point). And it is in Step 10 that we have made a net profit of 2 ATP's from our
initial investment.

Let!s return to Step 6 in glycolysis (Figure 8-5). In order for this reaction to take

place we used two molecules of NAD and we produced two molecules of

NADH + H. Remember, we actually have two molecules ofglyceraldehyde-3phosphate at this point. In order to continue with glycolysis we must have some
way to restore the reduced NAD's back to their oxidized form. How do we

regenerate NAD?
There are several ways to do this and pyruvate is involved in all of them. If we

usepyruvateitselfto regenerate the NAD, then pyruvate willget reduced. Look


at the C-2 carbon of pyruvate. It is in the keto form and the oxidation state is +2.

If we did a 2 electron transfer, we would end up with a carbon that has a zero
oxidation state. The compound that we can produce from this reaction is Llactate as shown in Figure 8-10. The enzyme involved here is lactate

dehydrogenase. This is one of the possible fates of pyruvate. In this way we can
reoxidize NAD and pay off our debt in Step 6.

Copyright by The Berkeley Review

187

The Berkeley Review

Specializing in MCAT Preparation

Biology

Glycolysis

Metabolic Pathways

ov

Lactate

dehydrogenase

c
'0
H-C- OH

+2 C= 0 "
CH3

Pyruvate

NADH

N\D+

+ H+

CH3
Lactate

Figure 8-10

The reaction in Figure 8-10 occurs in the cells of an organism when oxygen
becomes a limiting factor. For example, during vigorous exercise active skeletal
ntuscle produces lactate. Lactate turns out to be a dead end in metabolism,and as
we will later discover, is transported by the blood to the liver where it is
converted back to pyruvate.

Pyruvate will then be converted into glucose by a process called


gluconeogenesis. Lactobacillus casei, a member of a family of bacteria that are used
in the preparation of yogurt, excrete lactic acid into the surrounding medium as a
waste product. This is what gives yogurt its slightly acidic taste. The reaction
shown in Figure 8-10 therefore allows glycolysis to continue under anaerobic
conditions. Note that lactate is still at the same oxidation state as glucose
(Lactate: C3H603 x 2 = C6Hi206. Glucose: C6Hi206).

0.

V
i

Pyruvate
decarboxylase

Alcohol
H
I

dehydrogenase

CH2

C=0

c=o

OH

CH3

H+

C02

CH,

Acetaldehyde

Pyruvate

JADH
NADH

NAD+

CH3

+ H

Ethanol

Figure 8-11

This is not the only way to regenerate NAD. Yeasts use a slight modification.
Instead of doing a direct reduction of pyruvic acid with reduced NADH, yeasts
first decarboxylate the pyruvic acid to acetaldehyde as shown in Figure 8-11. The
enzyme that catalyzes this reaction is pyruvate decarboxylase (which contains
thiamine pyrophosphate (TPP) as a coenzyme). Pyruvate decarboxylase is a lyase.

Acetaldehyde will next react with the reduced NADH to form ethanol. The
enzyme involved in this step is alcohol dehydrogenase. This process, the
conversion of the sugar glucose into ethanol, is called alcoholicfermentation, and
ethanol, a waste product, is excreted into the surrounding medium.
The Big Picture

Let's consider the overall picture of glycolysis. This is shown in Figure 8-12. One
way to become intimatelyfamiliarwith this pathway is to follow a radioactively
labeled carbon through each series of steps (1-10) that we outlined and see where
that label ends up on the end products. For example, if the C-l carbon of glucose
were radioactively labeled (designated as *C-1), where would that label appear
Copyright by The Berkeley Review

188

The Berkeley Review

Specializing in MCAT Preparation

Biology

Metabolic Pathways

Glycolysis

(if it does at all) in pyruvate, lactate, or even ethanol? We will do it for a *C-1
labeled glucose molecule as shown in Figure 8-19.
* CHO

*CHO

* CH,-0H

H-C- OH

Hexokinase

HO-C-H
I

ATP

H-C OH

Phosphogluco

H-C OH
I
HO-C-H

c=o

Isomerase

HO -

C-H

.I

H-C

ADP

OH

H-C- OH
I

H-C- OH
I

H-C- OH

H-C OH

II

CH2-OH

CH2

0 Po

Glucose-6-phosphate

Glucose

CH2OPO

Fructose-6-phosphate

0-

ATP-^
*C = Label

Phosphofructo

DHAP
j

Triose Phosphate
Isomerase /*
O

O.

aldehyde-

OP0

*'

II

\\ h

I
H-C OH
I

ch2opo"

CH,0PO

NADH

1,3-Diphosphoglycerate

NAD+
0 "

+ H+

Glyceraldehyde3-phosphate

Fructose-1,6-diphosphate

ADP

Phospho
glycerate
kinase

Ov

H-C OH

H-C OH

PO

N~*
\
~~

H-C OH

^CH,O-

3-phosphate
dehydrogenase

II
CH2OPO
I
I

C=0
I
HO-C-H

Aldolase

Glycer

kinase

ADP

CH,0-P0

CHj-CHjOH

NAD+

Ethanol

MT.

ATP

Alcohol

dehydrogenase

NADH

H-C OH

*'CH,O

PO
Lactate
Lactate

3-Phosphoglycerate

dehydrogenase

' Phosphoglyceromutase

ov

o'

Enolase

H-C O P O

I*
HO-H,C

Pyruvate

Kinase

I
copo

I
O

2-Phosphoglycerate

* CH2

ADP

Phosphoenolpyruvate

ATP

Pyruvate

Figure 8-12

There is usually one point of confusion in this process and that comes at Step 4
and Step 5. Whenfructose-l,6-diphosphate splits to DHAPand glyceraldehyde3-phosphate, the label is initially on the *C-1 carbon of DHAP (which was the
labeled *C-1 carbon of fructose-l,6-diphosphate). However, since DHAP is in
equilibrium with glyceraldehyde-3-phosphate, that labeled carbon at the *C-1
Copyright by The Berkeley Review

189

The Berkeley Review

Specializing in MCAT Preparation

Biology

Metabolic Pathways

Glycolysis

position of DHAP is now the labeled carbon at the *C-3 position of glyceraldehyde-3-phosphate. When you split fructose-l,6-diphosphate the numbering
systemof the carbon atoms in the subsequentproducts are now based on a three
carbon molecule. In other words, the phosphate attached to the C-l position of
DHAP is the same phosphate attached to the C-3 position after DHAP has
isomerized to glyceraldehyde-3-phosphate. Therefore, the labeled *C-1 carbon of
DHAP is the same labeled *C-3carbon of glyceraldehyde-3-phosphate.
Regulation

If you look at the glycolytic pathway presented in Figure 8-12, you will notice
three places where the reactions are essentially irreversible. Those reactions are

catalyzed by the enzymes hexokinase (AG0' = -4.0 kcal/mole), phosphofructokinase (AG0' = -3.4 kcal/mole), and pyruvate kinase (AG' = -7.5 kcal/mole),
respectively. It turns out that these reactions all involve control points in the
regulation of the glycolytic pathway. In particular, regulation at the level of
phosphofructokinase is the most important. (We will see why in future
discussions.)

When the cell has plenty of glucose it will make a lot of ATP. Times are good for
the cell and so it doesn't want to waste its source of energy (glucose) by utilizing
it to make more ATP than it really needs. One thing the cell would like to do
with its supply of glucose is store it. As we will later see, the storage form of
glucose is glycogen.
However, before the cell can begin to store this glucose it must somehow slow
down the glycolytic pathway. High levels of ATP tend to allosterically inhibit
phosphofructokinase. Not only do high levels of ATP inhibit phosphofructo

kinase, but high levels ofH inhibit it as well. Oneplacewe could get an increase
in hydrogen ions is from the conversion of pyruvate to lactate (see Figure 8-10). If
you were to produce too many hydrogen ions, then your blood pH would begin
to drop and you would experience acidosis. One other regulatory molecule of
phosphofructokinase is citrate, an intermediate in the Krebs cycle. If the levels of
citrate are high, it must mean that glycolysis is functioning at some optimal rate
(because there is plenty of glucose around). Once again, why waste the glucose
that has been made available to you. High levels of citrate also inhibit
phosphofructokinase.

If high concentrations of ATP inhibit phosphofructokinase, then it must mean


that when the concentrations of ATP are low the activity of phosphofructokinase
is enhanced. What indicates when the levels of ATP are low? If ATP is being
utilized, it is most often converted to ADP and even AMP. It turns out that high
levels of AMP stimulate phosphofructokinase.

Copyright byTheBerkeley Review

190

The Berkeley Review


Specializing in MCAT Preparation

Biology

Metabolic Pathways

Disaccharide Metabolism

Disaccharide Metabolism
Disaccharides such as maltose, sucrose and lactose can be hydrolyzed into their
constituent monosaccharide residues which can then enter into the glycolytic
pathway. In humans we find that maltose can be hydrolyzed by the enzyme
maltase into two molecules of P-D-glucopyranose while sucrose can be hydro
lyzed bysucrase into cc-D-glucopyranose and P-D-fructofuranose. Lactose can be

hydrolyzed by lactase to P-D-glucopyranose and P-D-galactopyranose. These

enzymes are located in the epitheUal cells that line the smallintestine. In bacteria

lactose ishydrolyzed by theenzyme p-galactosidase.

Once maltose is hydrolyzed, the monosaccharide residues are immediately ready


to enter into the glycolytic pathway. However, the p-D-fructofuranose residue

from sucrose and the p-D-galactopyranose residue from galactose both need to
be converted to a form which can enter into the glycolytic pathway.
The hydrolysis of sucrose into a-D-glucopyranose and P-D-fructofuranose is

shown in Figure 8-13. The glucopyranose residue will readily enter into
glycolysis.

h/^"\h
V H \^

..." / v\ /.. Sucras


c

H0-H2C

HO

H,c
n2C

oh

KH HO y\

HV V CH2
F l OH

HO

OH

OH

a-D-Glucopyranosyl-p-D-fructofuranose
Sucrose

OH

OH

a-D-Glucopyranose
(Enters Glycolysis)

p-D-Fructofuranose

Figure 8-13

However, the fructofuranose residue must first be converted into a form which

can enter into the glycolytic pathway. Fructofuranose can either be converted

into p-D-fructofuranose-6-phosphate by the enzyme hexokinase or into p-Dfructofuranose-1-phosphate by the enzyme fructokinase. Both reactions are
phosphorylation reactions and both require ATP.

Triose

Phosphate
Isomerase

HO-H2C

OH

ATP A4DP HO-H2C o

U
/(hho/\CHo-OH ^
Fructokinase

HV

OH
\l CH2-0-P032-

H-C- OH
I

OH

p-D-Fructofuranose

OH

CH2-O-PO32-

p-D-Fructofuranose-

.
Kinase

1-phosphate

Glyceraldehyde
* ,
. {
3-phosphate

0
D-Glyceraldehyde

Glycolysis

Figure 8-14

Copyright by The Berkeley Review

191

The Berkeley Review

Specializing in MCAT Preparation

Biology

Disaccharide Metabolism

Metabolic Pathways

The hydrolysis of lactose into p-D-galactopyranose and a-D-glucopyranose is


shown in Figure 8-15. Once again, the glucopyranose residue will readily enter
into the glycolytic pathway.
CH,-OH

CH2-OH
Lactase,

H H20

H0

H v0H+ VH

OH

H ^|
H

OH

P-D-Galactopyranosyl-a-D-glucopyranose
Lactose

HA

]/ H

OH

K OH

HO \

If OH
OH

P-D-Galactopyranose a-D-Glucopyranose
(Enters Glycolysis)

Figure 8-15

Thegalactopyranose residue must now be converted into a formwhichcanenter


into the glycolytic pathway (this sequence is not shown). Note that galactose is
an epimer of glucose at the C-4 position. It turns out that galactose can be
convertedinto glucose-6-phosphate in 4 steps. Recall that glucose-6-phosphate is
an intermediate in glycolysis.

Copyright by The Berkeley Review

192

The Berkeley Review

Specializing in MCAT Preparation

Biology

Metabolic Pathways

Krebs Cycle

Krebs Cycle

;3iPR^^i^

The citric acid cycle is also called the tricarboxylic acid cycle or the Krebs cycle.
In eukaryotic cells the citric acid cycle occurs inside the mitochondria.

Glycolysis, however, occurs in the cell's cytosol. Recall that the end product of
glycolysis was pyruvate. Under anaerobic conditions, in the absence of oxygen,
pyruvate was used as an oxidizing agent to reoxidize NADH to NAD so'

glycolysis could continue. For example, in exercising muscle and in certain

bacteria pyruvate can be converted into lactic acid while in yeast pyruvate can

be converted into carbon dioxide and ethanol. Under aerobic conditions, in the
presence of oxygen, we will find that all of the carbons of pyruvate will
eventually be converted into carbon dioxide.

The first reaction to consider is the oxidative decarboxylation of pyruvate to form


acetyl CoA. This is a complicated reaction involving three catalytic cofactors
(thiamine pyrophosphate (TPP), lipoamide, and flavin adenine dinucleotide
(FAD)) and two stoichiometric cofactors (CoA and NAD). This reaction is

catalyzed by a group of three enzymes (pyruvate dehydrogenase component,


dihydrolipoyl transacetylase, and dihydrolipoyl dehydrogenase) which are
arranged into a structure called the pyruvate dehydrogenase complex.

TPP adds to the carbonyl function of pyruvate. The addition compound


decarboxylates and the remaining portion ofpyruvate (a hydroxy-ethyl moiety)
is transferred to lipoamide where it is oxidized to form acetyl-lipoamide. The
acetyl moiety ofacetyl-lipoamide is next transferred to CoA to form acetyl-CoA
and dihydrolipoamide (the reduced form of lipoamide). NAD is converted to

NADH + H as it reoxidizes dihydrolipoamide to lipoamide. The essentials of


this reaction can be seen in Figure 8-16.
NADH

+ H+
O
ll

O
ll

NAD+

-3

+2

+3

Pyruvate

CH3C-C-O

CoA-SH

Pyruvate
Dehydrogenase
Complex

O
II

CH3C-S-C0A
-3

+3

o=c=o
+4

Acetyl-CoA

AG0' -8.0
Figure 8-16

The AG' for this reaction is about -8.0 Kcal/mol and is very much infavor of the
products. Not only is entropy increasing because we are converting one molecule,
pyruvate, into two molecules, acetyl-CoA and carbon dioxide, but the carbon
dioxide itself is free to leave as a gas. This makes the reverse reaction rather
difficult.

Wementioned that the reaction shown in Figure 8-16 is an oxidation. Pyruvate is


an a-keto acid. The methyl carbon has an overall oxidation state of -3, the
carbonyl carbon +2, and the carboxylic acid carbon +3. The oxidation state of the

carbon in carbon dioxide is +4. What about the acetyl group of acetyl-CoA? The
methyl carbon still has an oxidation state of -3. The carbonyl carbon, however,
now has an oxidation state of +3. Remember, sulfur is more electronegative than
carbon. Whatis the difference in oxidation statesbetween the reactant, pyruvate,
Copyright by The Berkeley Review

193

The Berkeley Review

Specializing in MCAT Preparation

Biology

Krebs Cycle

Metabolic Pathways

and the products, carbon dioxide and acetyl-CoA? The oxidation state of the
methyl carbon does not change, sowe can ignore it. The combined oxidation states
of the carbon of the carbonyl function and the carbon of the carboxylic acid
function of pyruvate is +5. The combined oxidation states of carbon dioxide's
carbon and the carbonyl carbon of acetyl-CoA is +7. This is a two electron
oxidation process because we have increased from +5 to+7 aswe have gone from
reactants to products. If there is an oxidation, there must also be a reduction.
NAD undergoes a two electron reduction to form NADH +H.
The two carbon unit that we have synthesized in the form of acetyl-CoA is

unique because it has a high energy thioester bond. The problem that we are
faced with is turning the two carbonsof acetyl-CoA into carbon dioxide. We want
to use the energy of the thioester bond in acetyl-CoA to help turn both of these
carbonsinto carbondioxide. The only way we are going to be able to accomplish
this is to extend the carbon chain. We can do this by taking this two carbon acetyl
unit and combining it witha four carbon acceptor molecule called oxaloacetic acid
(OAA). Where did OAA come from? We will discuss this at a later time.
Acetyl
Enz

C S-CoA

CoA

Citrate

Enz

ooc- C= 0
I

COO

C- S-CoA

Base: "I'HrCH,

O"

-H

+ '

COO

Synthetase

AG0 - -7.5

CH,

Base

CH

OH

CH

, 2

COO "

Citryl-CoA

OAA

H20

I 2

OOC-C

I-

f*2

:> ooc-c-oh
CoA-SH

CH2

COO

Citrate

Figure 8-17

The alpha hydrogen of acetyl-CoA will form a bond with the carbonyl carbon of
OAA as shown in Figure 8-17. The reaction proceeds via an aldol condensation
to form the intermediate citryl-CoA. Citryl-CoA is then hydrolyzed to form
citrate with the subsequent regeneration of CoA. It is the hydrolysis of the
thioester bond that allows this reaction to be favorably pulled to completion.
The enzyme that catalyzes this reaction is citrate synthetase. It is in the class of
enzymes called a lyase. Note that there is no net gain or loss of hydrogens in this
reaction. Therefore, it cannot be a dehydrogenationand it certainly cannot be a
hydrogenation. There is no net change in oxidation state either. Those molecules
who gained electrons also lost electrons, and those who lost electrons also gained
electrons. Think of it as a reaction in which you are adding to the double bond of
the carbonyl group in OAA. Thus, it is a lyase.

Even though citrate is a symmetrical molecule, the two -CH2-COO groups do


not react identically. Why? It has to do with the asymmetric nature of the enzyme

aconitase in relationship to the symmetrical nature of citrate. In other words, the


two -CH2-COO groups of citrate are not equivalent. Aconitase,which converts
citrate to cis-aconitate, can distinguish between them. Thus, citrate is a prochiral
molecule and is optically inactive.

There is a hydroxyl function at the beta position of citrate. You might think that
we could oxidize the hydroxyl group to make the beta-keto function so we could
decarboxylate that carbon. We cannot do that because the hydroxyl group is
Copyright by The Berkeley Review

194

The Berkeley Review


Specializing in MCAT Preparation

Biology

Metabolic Pathways

Krebs Cycle

tertiary and would require very forceful conditions to oxidize it. What we can do,

though, is alittle rearrangement. Tertiary alcohols are known for their ability to
lose their hydroxyl function and form a fairly stable tertiary carbonium ion as
shown in Figure 8-18. You can think of this carbonium ion as being an
intermediate in the reaction scheme. What can we do with this tertiary

carbonium ion? Note that it has some alpha hydrogens. When alpha hydrogens are
close to a carbonium ion we may have an elimination reaction. This will give us
cis-Aconitate.

coo 1

CH2
i

' =

r\

COO

-OH

OOC- C- OH

COO

I
I From Acetyl CoA
CH, J

OOC C

Aconitase

3 Hydroxyl

Citrate

I
OOC-C
II

AG0 +2.0

CH,

CH,

coo

CH,

H+

C-H

COO

COO

Tertiary carbonium ion

cis-Aconitate

intermediate

Figure 8-18

If we add water across the double bond in cis-Aconitate, we will produce


isocitrate as shown in Figure 8-19. Aconitase, which catalyzes the reaction from
citrate to cis-Aconitate, also catalyzes the reaction of cis-Aconitate to isocitrate.

coo

COO
i

CH,

CH,

H,0

ooc c

I
OOC- C-H

II

Aconitase

C-H

H-C- OH

'

AG0' - -0.5

COO

COO

cis-Aconitate

Isocitrate

Figure 8-19

Isocitrate has a secondary hydroxyl group that is capable of being oxidized to a


carbonyl function. We can oxidize isocitrate with NAD.

coo

MAD+NADH +H+

CH,

I
OOC- C-H

COO

H-C- OH

COO 0

coo

CH,

CH2

I
e OOC- C-H

CH,

Isocitrate

dehydrogenase

C= 0

AG0" -2.0

COO

Isocitrate

p-Keto Acid

CO,

C=0

coo *

ct-Ketoglutarate

Figure 8-20

Theenzyme that carries out that reaction is isocitrate dehydrogenase. Note that
the intermediate that will form is a (3-keto acid, which is quite unstable. This will
result in a spontaneous decarboxylation at the C-3 position of isocitrate to give ocketoglutarate (ct-KG) as shown in Figure 8-20.

Copyright by The Berkeley Review

195

The Berkeley Review

Specializing in MCAT Preparation

Biology

Krebs Cycle

Metabolic Pathways

Ifyou compare the relative stabilities ofanalpha and a beta keto acid, you will
find that the more stable form is the alpha keto acid. The generalmechanism for
H

o"

R-CVV

the spontaneous decarboxylation of isocitrate can be seen in Figure 8-21. Note


that the enol form is in tautomeric equilibrium with the keto form.

CH,C = 0

(5-Keto Acid

pco2
OH
I

R - C= CH2
Enol

We now have an oc-keto acid which is reminiscent of where we started back at

pyruvate. Pyruvate was also an oc-keto acid. The oc-ketoglutarate will lose the
carboxyl group thatisalpha to the carbonyl. After going though a reaction thatis
similar to the reaction with pyruvate, we will get succinyl-CoA and carbon
dioxide (Figure 8-22). This reaction is catalyzed by the oc-ketoglutarate
dehydrogenase complex. Many anaerobic organisms do not have this enzyme
and therefore cannot carry out this reaction.
e
COO

II

NAD+

CH2

R - C- CH3

COO

NADH +H+

CH2
I

Keto

CoA +

CH2.

a-KG Dehydrogenase
Complex

C=0

Figure 8-21

CH2

COO

co2

C=0
I

AG0'-7.2

S-CoA

Succinyl-CoA

ot-Ketoglutarate
Figure 8-22

We can take advantage of the high energy thioester bond in succinyl-CoA to make
some GTP (guanosine triphosphate) as shown in Figure 8-23. GTP can readily be
converted into ATP (and we will find out how later).

COO
i

CH2

GDP +Pj

CH2 =^=
c=o
I

S-CoA

GTP

COO
i

r<2J_

CH2

Succinyl CoA
Synthetase

CH2

G' = -0.8

CoA-SH

coo"
Succinate

Succinyl-CoA
Figure 8-23

If you like, you can think of these high energy compounds (ATP, GTP, etc.) as
nucleoside triphosphates (NTP)a more generic name. The enzyme that
catalyzes the conversion of succinyl-CoA to succinate is succinyl-CoA
synthetase. This enzyme is in the class of enzymes referred to as ligases. Why? A
ligase will put two molecules together by using a high energy phosphate bond.
When you use GDP and Pi to make GTP, a water molecule is produced.
However, the water molecule that hydrolyzes the thioester bond and the water
molecule produced in the formation of GTP cancel each other out. Therefore, this
is actually an anhydrous reaction. Also note that succinate is a symmetrical
moleculeand the two ends, -CH2-COO0, are indistinguishable from one another.
In the reaction shown in Figure 8-23 we have made a GTP. Since we started off
with two pyruvate molecules (remember, the glucose molecule gets split), we
Copyright by The Berkeley Review

196

The Berkeley Review


Specializing in MCAT Preparation

Biology

Metabolic Pathways

Krebs Cycle

have a total of 2GTP's formed in the Krebs cycle. In other words, atthis point we

have a total of4 net NTP's since the startofglycolysis.

In the following set of reactions we will be reconstructing succinate back into


OAA so we can repeat the Krebs cycle. The only difference between succinate
and OAAis the keto group on OAA.
The first step in the regeneration of OAA is an oxidation in which succinate is

converted to fumarate by the enzyme succinate dehydrogenase. Fumarate is in


the cis configuration. If it were in the trans configuration, it would be called
maleate. Since we have a dehydrogenation we need an oxidizing agent. The
oxidizing agent is flavin adenine dinucleotide (FAD). Let's make some
generalizations. Every time you have seen an NAD involved in an oxidation
reaction it has been involved with turning an alcohol into a ketone or an
aldehyde (or vice versa). It does so by a hydride ion transfer. In the reaction
involving FAD, shown in Figure 8-24, there is no alcohol. In circumstances where

there isnoalcohol, and particularly where you form double bonds, the oxidizingreducing agent will turn out to be FAD. (Note the oxidation states of the C-2 and
C-3 carbons as we proceed from succinate to fumarate.)

coo
I

FAD

-2 CH,
I

-2 CH2
1

COO

co

\^ ^/

-1 C-H

Succinate

-1 C-H

Dehydrogenase

COO

AG0' = 0

Succinate

In Figure 8-25 we find that fumarate will react with water to produce malate.
Malic acid was first isolated from apple trees and it is this substance that gives

FADH2

Fumarate

Figure 8-24

apples their tart taste. The enzyme that catalyzes this reaction is fumarase. It is a
hydration reaction and falls in the lyase class of enzymes.

COO
I
H-C- OH

coo
I
C-H

Note the hydroxyl group on malate. We can convertthis hydroxyl group into a
keto group by the use of an oxidizing agent like NAD. This is the reaction that

will return us to OAA and bring us full cycle (Figure 8-26). The enzyme
involved here is malate dehydrogenase. OAA is now ready to react with
another acetyl-CoA moleculeand repeat the whole procedure again.

We have not made any more NTP's. What has happened to all that energy? It
has not been liberated yet. We have not used oxygen.As far as this is concerned,
everything could have been done anaerobically. We have made a lot of reduced

C-H

Fumarase

CH2

COO

AG0' = -0.9

COO

Fumarate

The function of the Krebs cycle is to turn those hard to oxidize carbons of acetic

Malate

ft
coo

NADH

NAD+

H-C- OH
1

CH,

'

+H+

y
Malate

coo

Dehydrogenase

Malate

flock of ATP's as soon as the oxygen is made available to us.

Figure 8-25

compounds such as FADH2 and NADH + H which can beclassed together as


reduced coenzymes. These reduced coenzymes represent a "dry brush" that could
be ignited with oxygen to turn them into their oxidized form, and in doing so
liberate a lot of energy. Part of this energy will get turned into ATP. Even though
we have made only 2 GTP's by going through the Krebs cycle, we have stored a
great deal of potential energy. This potential energy can be used to make a whole

H,0

II

AG=+7.1

COO
1

C= O
I

CH,

'

COO

OAA

Figure 8-26

acid into something that can be oxidized. By the time we went around the Krebs
cycle, two carbons had come off as carbon dioxide. We had oxidized two

carbons and we regenerated the starting material. Eventually the two carbons
that we put into the cycle will come off as carbon dioxide, too.

The Krebs cycle can be thought of as a catabolic process in which the overall

AG0 value for the complete oxidation of an acetylunit from acetyl CoA is about
-9.8kcals/mol. It would be difficult to reverse this cycle.

At this point in our story we have turned glucose (or some carbohydrate) into
carbon dioxide via glycolysis and the Krebs cycle. In eukaryotic cells glycolysis
Copyright by The Berkeley Review

197

The Berkeley Review


Specializing in MCAT Preparation

Biology

Metabolic Pathways

Krebs Cycle

occurs in the cytosol while the Krebs cycle occurs in the matrix of the
mitochondria. Recall that the average oxidation state of the carbon atoms in our
carbohydrate was 0. The oxidation state of the carbon in carbon dioxide is +4.
This process was therefore an oxidation. In order to balance that oxidation we
needed to have a corresponding reduction. Where did the electrons go? Those
electrons ended up in either NADH or FADH2. In order for these processes to
continue we must reoxidize NADH and FADH2 back to NAD and FAD,
respectively. We can summarize the essentials of glycolysis and the Krebs cycle
as shown in Figure 8-27.

Mitochondrial
membranes

I
0

}Pyruvate -s^CQz +NADH

2 Pyruvate
(A

CoA

Matrix

2 ATP

rr

Acetyl
CoA

2 NADH

Oxaloacetate

CoA

NADH

Malate

Citrate

Glucose

Cytosol

Krebs

Cycle

Fumarate

Isocitrate

t^*-FADH2
Succinate

a-ketoglutarate

1C0A

GTP

NADH-<^- C02

L-CoA
Succinyl CoA ^ \
CO2+NADH

Figure 8-27

The external oxidizing agent, otherwise known as the external electron acceptor,
is oxygen. As oxygen is used as the external electron acceptor, it will be
converted into two water molecules by picking up four hydrogens and four

electrons per oxygen molecule (O2). This can be summarized as shown in Figure
8-28.

Glucose

(CH20)n
co2

NAD+,FAD -^ 2H20
NADH + ff

02 + 4H+ + 4e"

FADH2
Figure 8-28

Copyright by The Berkeley Review

198

The Berkeley Review


Specializing in MCAT Preparation

Biology

Metabolic Pathways

Electron Transport

Electron Transport
A very important feature of the mitochondrion is that it is a double membrane

organelle. The nucleus and the plant chloroplast (site ofphotosynthesis) are also
double membrane organelles. As we will learn, much of the ATP that we utilize
in our metabolic processes is synthesized within the mitochondrion. The
mitochondrion is the site of oxidative respiration in the cell.

When we discuss the chemical reactions that take place in the mitochondrion, it
is important to have a firm grasp of the organelle's anatomy. As we mentioned,
the mitochondrion is composed of a double membrane system. There is an outer
membrane and an inner membrane. Between these two membranes is the inter
membrane space. The inner membrane is folded into structures called cristae.

Outer membrane
Cristae
Inner membrane
Intermembrane
space

Matrix

This increases thesurface area of the inner membrane. The central cavity of the
organelle is called the matrix. These structures are shown in Figure 8-29. Let's
take a brief look at these structures.

The outer membrane is quite permeable and it contains protein channels called
porins that allow molecules with molecular weights as high as 10,000 daltons to
pass through. [One dalton is nearly equal to the mass of a hydrogen atom. In
other words, 10,000 daltons is about 10,000 times the mass of a hydrogen atom.]
Molecules such as amino acids, carbohydrates such as glucose, and small

polypeptides can pass through these porins. The outer membrane is composed

ATP Synthetase
Ribosome

Figure 8-29
The anatomy of a generalized

of about half proteins and half lipids.

mitochondrion.

Moving inwards we next come to the intermembrane space. This area of the
mitochondrion contains a variety of enzymes and also has a high proton concen
tration because of protons which are pumped into this space from the matrix. In
other words, the concentration of protons in the matrix is lower than the
concentration of protons in the intermembrane space. This leads to an

electrochemical gradient being established between the intermembrane space


and the matrix. Gradients have the potential to do work. That potential is utilized
for the synthesis of ATP.
The inner membrane is folded into cristae in order to increase its surface area. A

larger surface area means more proteins. In fact, the inner membrane is

composed of about 75 percent proteins and about 25 percent lipids. Some of the
proteins in this membrane are permeases that allow for the passage of ADP into
the matrix in exchange for ATP. ATP is synthesized within the matrix of the

mitochondrion from ADP and Pj (inorganic phosphate) at the expense of a


proton gradient. Because the inner membrane is quite impermeable to hydrogen
ions (protons), there are special permeases that allow for this ion's passage into
the matrix from the intermembrane space (down theirelectrochemical gradient).
For example, there are permeases that allow for the simultaneous passage of
inorganic phosphate and hydrogen ions into the matrix. Other molecules like
oxygen (02) can readily diffuse through the inner membrane and into the matrix
of the mitochondrion.

Within the matrix of the mitochondrion we find hundreds of different types of


enzymes. This is a region rich in enzymatic activity. One of the important
biochemical pathways that occurs in the matrix is the Krebs cycle (also called the
tricarboxylic acid cycle or the citric acid cycle). The Krebs cycle is the final
common pathway for the oxidation of fuel molecules like glucose. It also
provides intermediates for biosynthetic reactions and it generates carbon dioxide
Copyright by The Berkeley Review

199

The Berkeley Review


Specializing in MCAT Preparation

Biology

Metabolic Pathways

Electron Transport

(CO2) which can diffuse out of this organelle. ATP is synthesized by an ATP
synthase protein which is situated on the matrix side of the inner membrane.
Also, within the matrix we find mitochondrial DNA.

So far we have turned glucose (or some carbohydrate) into carbon dioxide via
glycolysis and the Krebs cycle. In eukaryotic cells glycolysis occurs in the cytosol
while the Krebs cycle occurs in the matrix of the mitochondria. We mentioned
that the average oxidation state of the carbon atoms in our carbohydrate was 0.
The oxidation state of the carbon in carbon dioxide is +4. This process was
therefore an oxidation. In order to balance that oxidation we needed to have a

corresponding reduction. Where did the electrons go? Those electrons ended up
in either NADH or FADH2. In order for these processes to continue we must

reoxidize NADH and FADH2 back to NAD and FAD, respectively. The
external oxidizing agent that we will use for this reoxidation is oxygen. We will
see that as oxygen is used as the external electron acceptor, it will be converted
into two water molecules by picking up four hydrogens and four electrons per
oxygen molecule (O2). This process, which is referred to as electron transport
and oxidative phosphorylation, occurs on the inner membrane of the
mitochondria.

Mitochondria are about the same size (1500 nm by 500 nm) as the prokaryotic
bacterium Escherichia coli. Mitochondria have both an outer and an inner

membrane. Between the two membranes is the intermembrane space. The outer
membrane is permeable to many small molecules and ions. The inner membrane,
however, is essentially impermeable to most polar molecules and ions and it is
highly folded into structures called cristae. Because prokaryotic cells do not have
mitochondria, they carry out the reactions of electron transport and oxidative
phosphorylation on their cytoplasmic membrane (i.e., the inner plasma
membrane). (Prokaryotic organisms have an outer membrane, followed by a cell
wall, and then an inner membrane.)

Mitochondria contain their own DNA. They arise from the growth and division
of existing mitochondria within the cell. Human mitochondrial DNA (mtDNA
for short) is circular and contains 16,569 base pairs. This DNA encodes for 22
tRNAs and 2 rRNAs. It also codes for 7 subunits of the NADH-Q reductase

enzyme. This is a complex located on the inner membrane and is responsible for
proton trans-location across the inner membrane. Cytochrome reductase,
cytochrome oxidase, and ATP synthase subunits are also encoded by this
mitochondrial DNA. We will learn the functions of these complexes in just a bit.
How are the reduced NADH and FADH2 coenzymes reoxidized? Recall that the
Krebs cycle occurs in the matrix of the mitochondria in eukaryotic cells. These
reduced coenzymes, therefore, are released into the mitochondrial matrix.
Associated with the inner mitochondrial membrane are sequences of protein
complexes that act as electron carriers. Taken collectively, the NADH-Q
reductase (also called Complex I or Site 1), succinate-Q reductase (also called
Complex II), cytochrome reductase (also called Complex III or Site 2), and
cytochrome oxidase (also called Complex IV or Site 3) protein complexes are
often referred to as the respiratory chain (due to their eventual involvement with
oxygen) or electron transport chain. There are many of these respiratory chains
associated with the inner membrane. In turn these protein complexes are
composedof an array of prostheticgroups (e.g., FMN, Fe-S, FAD, heme, and Cu)
which can act as electron carriers. As we will see, the electrons from NADH enter

Copyright by The Berkeley Review

200

The Berkeley Review


Specializing in MCAT Preparation

Biology

Metabolic Pathways

Electron Transport

into the electron transportchain at the level of the NADH-Q reductasecomplex


while the electrons from FADH2 enter by way of the succinate-Q reductase
complex and a molecule called coenzymeQ (also knownas ubiquinone). Thiscan
be seen in Figure 8-30.

Let's consider each of the enzyme complexes shown in Figure 8-30 in a little more
detail. We'llstart with the NADH-Q reductase complexfirst. Reduced NADH +

He passes two electrons and two hydrogens tothe oxidizedflavin mononucleotide

NADH

(FMN) prosthetic group associated with this enzyme complex. This results in the
production of reduced FMNH2 and the regeneration of oxidized NAD. The
electrons from FMNH2are passed to a series of iron-sulfur clusters (Fe-S) in which

Reductase

the iron atoms cycle between the reduced ferrous (Fe2e) and the oxidized ferric

Complex I

(Fe3) states. There is an uncertainty as to how many Fe-S clusters there are in

NADH-Q

FADH2

the NADH-Q reductase complex. The electrons will eventually be passed from a
reduced Fe-S moiety to the oxidized form of coenzyme Q (Figure 8-31).

)H-^ f

FMN ^

CoQ<=>

- Fe2+-S -^^- Fe3+-S *. f CoQH2

Cytochrome

'

NADH-Q Reductase

Reductase

Complex n

D+-*<>^ FMNH2^Ss- Fe3+-S -*<^ Fe2+-S-^V_CoQ

'

Succinate-Q

Reductase

Complex HI

Figure 8-31

At the end of the Fe-S complex there is a quinone. It is called coenzyme Q (or
CoQ). The oxidized quinone is just called quinone while the reduced quinone is
called dihydroquinone. Coenzyme Q was first discovered back in the 1930's and it
has turned up in every organism which is capable of doing this type of electron
transport. In the late 1940's the international fraternity of biochemists realized
that they were finding CoQ everywhere and so they thought it would be nice to
call it ubiquitous. Coenzyme Q in the oxidized form is now called ubiquinone
while coenzyme Q in the reduced form is called ubiquinol. The structure of
ubiquinone (CoQ) and ubiquinol (C0QH2) and a semiquinone (free-radical)
intermediate is shown in Figure 8-32.

Cytc

Cytochrome
Oxidase

Complex IV

^*- Water

Figure 8-30

OH

H+ + e"
H3C0

HjCO-jj^S

H++ e'

CH,

HjCOKf?- R

H3COUSsxjJ R

H,CO

Ubiquinone

H3COT|V- CH,

OH

OH

Semiquinone

Ubiquinol

Figure 8-32

Recall that as succinate is oxidized to fumarate in the Krebs cycle by the enzyme
succinate dehydrogenase, FADH2 is generated. This FADH2 needs to be

reoxidized. Succinate dehydrogenase turns out to be part of the succinate-Q


reductase complex. FADH2 is immediately passes its electrons to an Fe-S
protein(s) which then funnels them into the oxidized form of CoQ. FADH2 is
reoxidized to FAD as shown in Figure 8-33.
Copyright by The Berkeley Review

201

The Berkeley Review


Specializing in MCAT Preparation

Biology

Metabolic Pathways

Electron Transport

What we have done so far is to collect all of the electrons from all of the various

substrates into one pocket which is C0QH2. No matter what is happening we


will always have a large turnover of CoQ. Our next interest is to reoxidize CoQ.
FAD

FADH2

Fe2+-S Fe3+-S

Succinate-Q
reductase

I*
i^ CoQ

CoQH2

Figure 8-33

The electron carriers between CoQ and oxygen are called cytochromes.
Cytochromes are electron transporting proteins that contain a heme prosthetic
group with an iron atom that alternates between the Fe2 and Fe3 condition.
Within the cytochrome reductase complex are two cytochromes, b and ci, and an
Fe-Sprotein. As C0QH2 transfers one electron at a time to an Fe-S protein in the
complex it is converted to CoQH* (semiquinone). The reduced form of

cytochrome b (Cyt b2) reacts with CoQH* to give the oxidized form of
cytochrome b (Cyt b3) andC0QH2.
Cytochrome b3 can then oxidize another molecule of CoQH* to CoQ. This is
shown in Figure 8-34. Coenzyme Q is a molecule that carries two electrons at a
time. However, the Fe-S protein can carry only one electron at a time.
Cytochrome b is the go-between that allows this interaction to occur. The
electrons are eventually passed to cytochrome c. The iron atom of the heme
group of cytochrome c is bonded to a sulfur atom of a Met residue on one side
and to a nitrogen atom of a His residue on the other side.

CoQ^^Cyt b_ _CoQH* ^. ^Fe2+-S-^. - Cytcfi *, Cytc+

CoQH-^V.Cyt b3-A*CoQH2-^V-Fe3+-S^ >*Cytc^t^V.Cytc3+


Cytochrome Reductase
3+

Cytb

2+

Cytb

H
*

Figure 8-34

Cyta2+Cyta3+ |
Cyta33+ Cyta32+

o
o
ST

Series of steps
with 4 e"

Figure 8-35

The electrons that end up on the reduced form of cytochrome c in Figure 8-34are
next passed to the cytochrome oxidase complex. This complex consists of two
heme groups, heme a and heme &$, and two copper atoms, one associated with
heme a and the other with heme a3.

X ls

2H20 02 +4H+

V
Cytochrome Reductase

These copper atoms can alternate between the +1 and +2 oxidation states. The
electrons are passed from cytochrome c to the heme a and heme a3 moieties of
the cytochrome oxidase complex and then to oxygen. The transfer of four
electrons to molecular oxygen leads to its reduction to two molecules of water.
This is shown in Figure 8-35.
At this point we have completed electron transport and have reoxidized the
reduced coenzymes. This means that there must have been a strong negative

AG' for this process. The question we want to get at now is how ATP is
synthesized. So far we have not seen anyway for a high energy phosphate bond
to be formed (none of these compounds are phosphorylated).

Copyright by The Berkeley Review

202

The Berkeley Review


Specializing in MCAT Preparation

BlOlOgy

Metabolic Pathways

Oxidative Phosphorylation

There are three areas in this oxidation reduction scheme where ATP can be

synthesized due to a conservation of energy. It turns out that the passage of two
electrons from NADH down this chain allows for the synthesis of 3 molecules of
ATP. This is described by the P/O ratio. The P/O ratio is the number of high
energy phosphate bonds made per atom of oxygen used. The P/O ratio for
FADH2 allows for the synthesis of 2 molecules of ATP. The reason that FADH2
generates only 2 ATPs is because the electrons from FADH2 enter into the

electron transport chain at a lower energy level than the electrons from NADH.

All of the ATP molecules that we will be producing via the electron transport
chaindepend on the presence of oxygen. Bytaking ADPand Pj and making ATP
we are doing a reaction which is referred to as a phosphorylation reaction. The
historic term that is associated with these types of reactions is oxidative
phosphorylation (ox-phos). Oxidative phosphorylation will yield a flock of
ATPs. All the other ATPs that we have made (i.e., in glycolysis and the Krebs
cycle) can be distinguished from ox-phos ATPs because they did not directly
depend on oxygen. All six of the high energy bonds that were formed in both
glycolysis and the Krebs cycle are not referred to as oxidative phosphorylations
but rather substrate level phosphorylations.

How are the ATPs synthesized during oxidative phosphorylation? The


mechanism, which was first proposed in 1961 by Peter Mitchell, is called the
chemiosmotic hypothesis. Mitchell said that ATP synthesis and the electron
transport chain are coupled together by a proton gradient that had established

itself across the inner mitochondrial membrane and not by a high energy
phosphorylated intermediate. The coupling factor for this mechanism is an
enzyme called the F0Fi ATPase. This enzyme allows for the synthesis of ATP at

the expense of the free energy that is released as a proton (H) passes through
this F0FiATPase from the intermembrane space to the matrix of the
mitochondrion. How was the proton gradient established?
In the following diagrams the membrane that is drawn will be taken to be the
inner membrane of the mitochondria. Associated with this membrane are the

complex of proteins we collectively call the electron transport system (or the
respiratory complex). This particular array of enzymes allows for a smooth and
continuous flow of electrons from one end to the other. As we have mentioned,

these respiratory complexes are repetitive within the inner membrane.

If there are reduced substrates (e.g., NADH or FADH2) contributing electrons


either from within the mitochondrion or from without (e.g., the electrons from
the NADH synthesized during glycolysis), then the electrons will move down
the electron transport chain and eventually come into contact with oxygen. The
oxygen will be reduced to water. Mitchell's hypothesis is that there is a
rearrangement of these enzymes as they pass back and forth through their
oxidized and reduced states such that hydrogen ions are removed from the interior
(matrix) of the mitochondrion and pumped out into the exterior (intermembrane

space) of the mitochondrion. Hydrogen ions are thought to be translocated across


the inner mitochondrial membrane at each of the three enzyme complexes (i.e.,
the NADH-Q Reductase, Cytochrome Reductase, and Cytochrome Oxidase
complexes). This is schematically shown in Figure 8-36. If this process repeats
itself, then the pH of the interior will increase (the acidity will decrease) with
respect to the exterior. That is what constitutes the proton gradient.
Copyright by The Berkeley Review

203

The Berkeley Review


Specializing in MCAT Preparation

Biology

Metabolic Pathways

Oxidative Phosphorylation

If there is this gradient ofhydrogen ionswhere the [He] is greater on the outside
than on the inside, then it will allow the flow of hydrogen ions back into the
matrix through the F0FiATPase complex. This is a non-equilibrium situation and

represents potential work. If the hydrogen ions are allowed to enter through this
ATPase complex, then this work energy can be converted into useful chemical
energy through the synthesis of ATP as shown in Figure 8-37.

Inner mitochondrial
membrane

Figure 8-36

Figure 8-37

This ATPase complex is not part of the respiratory unit (electron transport chain).
The oxidation takes place in the respiratory unit while phosphorylation takes
place at the ATPase complex. These ATPase units occur all over the inner surface
of the inner membrane of the mitochondrion and essentially alternate with the
units of the respiratory assembly. These two units taken together, the electron
transport chain and the F0Fi ATPase complex, constitute the oxidative generation
of ATP.

The hydrogen ions cannot freely pass across the inner mitochondrial membrane.
If they could flow freely back and forth across the membrane, no work would be
done. This membrane must exclude hydrogen ions except at the ATPase port of
entry. Not only is this membrane impermeable to protons but it is also
impermeable to almost all charged molecules unless there are specific integral
protein ports which allow the molecule in question to come across. For example,
the membrane would be impermeable to ATP, ADP, Pj, NAD, and NADH
unless there was a specific port to allow these molecules passage.
Consider for a moment the integrity of the mitochondrial membrane. If anything
were to happen to the integrity of the mitochondrial membrane, it would have an
affect on the oxidative phosphorylation but not on the substrate level phos
phorylation of ATP. Physical damage to the mitochondrial membrane can in fact
cause a loss of oxidative phosphorylation. This is one of the early pieces of
evidence that led Mitchell to his hypothesis. If you were to remove a piece of the
mitochondrial membrane, and if there had been some kind of direct transfer of

phosphorus from compounds inside the matrix to ATP, then the removal of the
membrane should not have made a difference and phosphorylation should still
Copyright by The Berkeley Review

204

The Berkeley Review

Specializing in MCAT Preparation

BlOlOgy

Metabolic Pathways

Oxidative Phosphorylation

be taking place. Butthat was not the casebecause every time a holewas punched
in the mitochondrial membrane, oxidative phosphorylations no longer
proceeded.Substrate level phosphorylations did continue though.
It turns out that there is more than one way to punch a hole in a membrane. One

wayto do this is to chemically modify the membrane or evenmodify the passage


of the hydrogen ions across that membrane. Suppose hydrogen ions could cross
themembrane withoutthe membrane realizing it.Ifyoucould modify the way in
which the hydrogen ions can enter into the matrix such that they sneakpast the
ATPase complex without doing any work, then that will wipe out oxidative
phosphorylation.

Ifwe add a hydroxyl group to dinitrobenzene, we will make a compound called


2,4-dinitrophenol (2,4-DNP). This is one of the first compounds discovered that
causes a problem with the integrity of mitochondrial membranes. 2,4-DNP is a

sufficiently strong acid. Recall that phenols are acids-they are slightly acidic. If
2,4-DNP were capable of moving across a mitochondrial membrane and then

dissociating inside, it would have transported a hydrogen ion across the


membrane. The method of getting a proton across the membrane is referred to as
a proton transport.

What will be the effect of the presence of 2,4-DNP on a system such as this where

thehydrogen ion gradientwas the coupling aspect of electron transport and ATP
formation? It willuncouple it. If we wereto uncouple this hydrogen ion gradient,
we would be just getting -52 kcals and no work. This means that all the chemical
energy will be dispersed as heat.

A similar heat production mechanism is utilized by hibernating animals and


newborn human babies. There are specialized fat cells in which mitochondrial
respiration is naturally uncoupled from the synthesis of ATP. These fat cells have
a very large number of mitochondria, and since the mitochondria have
cytochromes which contain iron, they give the fat a brownish color. In infants

thisbrownfat is located in the interscapular space. Adulthumanshave very little


brownfat. Embedded in the mitochondria of the brown fat is a special transport
system that allows protons to move back across the inner membrane. Since the

hydrogen gradient is dissipated,there is no means to capture the energyof ATP.


Instead, this energy is lost as heat.

We can calculate the overall generation of energy from glucose. In the process of
glucosegoing to pyruvate we found that 2 net ATPs were generated directly. In
this process we also produced 2 NADHs which were extramitochondrial. These
extra-mitochondrial NADHs are transported into the mitochondrial matrix by
either one of two shuttle systems-the glycerol phosphate shuttle or the malateaspartate shuttle. If the glycerol phosphate shuttle is utilized, each cytoplasmic
NADH yields two ATPs. If the tnalate-aspartate shuttle is used, each cyto
plasmic NADH yields the normal three ATPs.

In the Krebs cycle the 2 pyruvates that we generated from glycolysis produced 6
CO2S. We also produced 2 more high energy phosphate bonds in the form of
GTP from the Krebs cycle. The NTPs that we have produced so far are referred
to as substrate level NTPs because they have not come into contact with oxygen.
We also found that the Krebs cycle produced 2 FADH2S (one for each pyruvate)
and 8 NADHs (four for each pyruvate). These NADHs and FADH2S will be
intra-mitochondrial.

Copyright by The Berkeley Review

205

The Berkeley Review


Specializing in MCAT Preparation

Biology

Metabolic Pathways

Oxidative Phosphorylation

Recall that we mentioned that the P/O ratio for NAD is 3 ATPs while the P/O

ratio for FAD is 2 ATPs. Consequently, if we utilize the malate-aspartate


shuttle, the 2 NADHs from glycolysis will give us 6 ATPs while the 8 NADHs

from the Krebs cycle will giveus 24ATPs. The 2 FADH2S from the Krebscycle
will give us 4 ATPs. We have 2 ATPs made in glycolysis and we have 2 GTPs
(which can be converted into ATP) from the Krebs cycle. Therefore, the total
ATPs produced is 38. If we had used the glycerol phosphate shuttle, we would
have produced 36 ATPs. It turns out that the thermodynamic efficiency for the
complete oxidization of glucose is about 40% under standard conditions.

Bacteriorhodopsin
Photons

ATP ^V
Synthetase
ATP

Halobacterium halobium (from the Greek hals for salt) is an archaebacterium


that requires an optimal sodium chloride concentration of about 4.3 M.
Compare this to the salt concentration in sea water which is about 0.6 M. One
place where these arachaebacteria can be found is at the south end of San
Francisco Bay. Its membrane can be separated into various fractions, one being
a purple-membrane protein called bacteriorhodopsin which contains seven

Reconstituted Vesicle

Figure 8-38

transmembrane helices and a light absorbing prosthetic group called retinal (a


chromophore). The retinal found in bacteriorhodopsin is identical to the retinal

found in the rhodopsin of the rod cells of vertebrates. When incident photon
flux impinges on this purple membrane in the absence of oxygen, it acts like a
proton pump and translocates protons from the cytosol of the cell to the outside
environment. This proton gradient can be used to synthesize ATP. If oxygen is
present, this organism can also carry out oxidative phosphorylation to generate

NADH

ATP.

Rotenone

NADH-Q

(-)

Walter Stoeckenius and Efraim Racker made synthetic vesicles that contained
the mitochondrial F0FiATPase enzyme and the bacteriorhodopsin of the purple

Reductase

Complex I

FADH2

U
^
CoQ<n

Succinate-Q
Reductase

Complex n
Antimycin

Cytochrome I M

A (-)

Reductase

Complex HI

Cytc ^H Ascorbate

(+)
Oxygen

Cytochrome \' CN>N3>


Oxidase

Complex IV

ft

CO (-)

membrane fragments from Halobacterium halobium. In this experiment the


orientation of these two purified molecules within the reconstituted membrane
were reversed (i.e., reversed from how they are normally found within the
membranes in which they usually reside). As shown in Figure 8-38, incident
photon flux caused protons to be pumped into the interior or the reconstituted
vesicle. This established a proton gradient which was utilized by the ATP
synthetase apparatus to synthesize ATP in the exterior medium. This
experiment (circa 1974) added support to Peter Mitchell's chemiosmotic
hypothesis (circa 1961) involving the coupling of a proton gradient with ATP
synthesis.
One experimental means that was employed in elucidating the pathway of
electron flow in the electron transport chain was by the use of specific
inhibitors. Rotenone is a toxic plant poison that inhibits electron transport
within the NADH-Q reductase complex. Antimycin A, a toxic antibiotic
isolated from a strain of Streptomyces, blocks electron flow between
cytochromes b and ci (at the cytochrome reductase complex). Addition of
ascorbate reduces cytochrome c and allows for electron flow from cytochrome
c to oxygen. The cytochrome oxidase complex can be inhibited by such

molecules as cyanide (CNe), azide (N3e), or carbon monoxide (CO). The sites
of these inhibitors can be seen in Figure 8-39.

Water

Figure 8-39

Copyright by The Berkeley Review

206

The Berkeley Review


Specializing in MCAT Preparation

Biology

Metabolic Pathways

Pentose Phosphate Pathway

;i|^^;:E^

t '*

Let's consider the pentose phosphate pathway. The purpose of the pentose
phosphate pathway is to generate reducing power in the form of NADPH and
five carbon sugars such as ribose-5-phosphate. These reactions occur in the
cytosol of the cell and, as we will learn later, some of them will be involved in

the formation of other sugar moleculesfrom CO2 in photosynthesis.


Recall that during glycolysis, glucose can be phosphorylated to form glucosesphosphate. The enzyme that catalyzed this reaction is hexokinase. Glucoses-

phosphate has many fates and one of them involves the pentose phosphate
pathway.If we dehydrogenate (remove hydrogens) glucose-6-phosphate at the C-l
carbon, we will get a molecule called 6-phosphoglucono-d-lactone. This
oxidation reaction yields NADPH and is catalyzed by glucose-6-phosphate
dehydrogenase. See Figure 8-40.
O

11

O-P-O-CH2

NADPH

r~ \ O-H NADP+ +^+


/oh

HO \

e }~\

^ '

?,

O-P-O- CH2

/oh /=

Glucose 6-phosphate

HO \|

dehydrogenase

^"f

OH

OH

Glucose 6-phosphate

6-Phosphoglucono8-lactone

Figure 8-40

The predominant form of glucose-6-phosphate in solution is in the ring form.


Note that the C-l position is bonded to two oxygen atoms. As shown in Figure 840, oxidation at the C-l carbon will give 6-phosphoglucono-d-lactone. If we have
a carbonyl function that is next to an oxygen atom in a ring structure, we have a
cyclicesteror lactone.
0

11

o-p-o

O-CH

NADPH

H-C- OH

+ H20
O

HO-C-H

-H20
Lactonase

6-Phosphoglucono-

H-C-OH
I
H-C- OH

H-C- OH

C=0

6-Phosphogluconate

ch,-o-po,2-

6-Phosphogluconate

8-lactone

+ h+

OH

NADP+

dHase

CH2-OH

CO

H-C- OH ^
I
H-C- OH

CH2-0-POj2-

c= o

H-C-

OH
1
H-C- OH

CH2-0-P032

P-Keto

Ribulose

Intermediate

5-phosphate

Figure 8-41

The next reaction is the hydrolysis of that cyclicester. The enzyme that catalyzes
this reaction is called lactonase. Lactonase hydrolyzes the lactone and lets the
ring open to form 6-phosphogluconate. Recall that the best way to release carbon
dioxide is to have a keto function at the beta position. In the structure of 6Copyright by The Berkeley Review

207

The Berkeley Review


Specializing in MCAT Preparation

Biology

Metabolic Pathways

Pentose Phosphate Pathway

phosphogluconate we have an alcohol function at the C-3 position. If we could


relieve that hydroxyl group of its hydrogen, we would have an intermediate that

is a beta-keto acid. This reaction is catalyzed by NADP and the enzyme 6phosphogluconate dehydrogenase. The transitional state, which is the beta-keto
acid, rapidly decarboxylates to form ribulose-5-phosphate. This set of reactions is
shown in Figure 8-41. The reactions shown in Figure 8-40 and Figure 8-41
constitute an oxidative way to form 5-carbon sugars.
The enzyme phosphopentose isomerase is able to isomerize ribulose-5-

phosphate to ribose-5-phosphate. This reaction is shown in Figure 8-42 and


proceeds through an enediol intermediate. This reaction should look familiar to
you. Recall the interconversion between glucose-6-phosphate and fructcse-6phosphate in glycolysis. The intermediate in that reaction was an enediol, too.
O.

CH2-OH

CH-OH

C=0

C-OH

H-C- OH

H-C- OH

H-C- OH
i

CH2-0-P03:

H-C- OH

H-C- OH

Phospho
pentose
isomerase

H-C- OH

H-C- OH

CH2-O-PO32-

CH2-O-PO32

Ribulose

Enediol

Ribose

5-phosphate

Intermediate

5-phosphate

Figure 8-42
Formation of Ribose 5-phosphate

What is the value of having something like NADP? It turns out that this molecule
serves a special purpose. All of the reactions that we have considered so far are

energy releasing reactions in which ATP was generated. Because they were
energy releasing reactions they were also breakdown reactions. The entropy was
increasing.

All of the reactions that are degradative and energy releasing are referred to as
catabolic reactions (catabolism). NAD is the coenzyme that is involved in those
reactions. In contrast, there are reactions that require energy to decrease the
entropy by putting things together. These are biosynthetic reactions and are
collectively characterized by the term anabolism. NADP is the coenzyme that is
involved in these reactions.

There are a variety of things that can happen to the 5-carbon sugars that we have
just mentioned. For example, enzymes such as a transketolase, a transaldolase,
or even an epimerase can make use of a specific 5-carbon sugar as their substrate.
Let's examine some of these reactions.

The transketolase enzyme can catalyze the synthesis of xylulose-5-phosphate


and erythrose-4-phosphate from glyceraldehyde-3-phosphate and fructoses-

phosphate. The coenzyme involved in this reaction is thiamine pyrophosphate


(TPP). This is shown in Figure 8-43. Note that the "boxed" portions of the
molecules are those portions that are being transferred.

Copyright by The Berkeley Review

208

The Berkeley Review


Specializing in MCAT Preparation

Biology

Metabolic Pathways

Pentose Phosphate Pathway

CH2-OH
1

C=0
0.

CH2-OH

Trans
ketolase

HO- C-H
1

HO- C-H

I
I

Glyceraldehyde
3-phosphate

H-C- OH

H-C- OH

H-C- OH

CH2-0-P032-

H
1

TPP

H-C- OH

H-C- OH

0N

C=0

CH2-O-PO32-

CH2-O-PO32Xylulose
5-phosphate

Fructose

6-phosphate

H-C- OH

CH2-0-P032Erythrose
4-phosphate

Figure 8-43
Intermediates in the Pentose Phosphate Pathway

Xylulose-5-phosphate can also be converted to ribulose-5-phosphate by the


enzyme phosphopentose epimerase. This enzyme is a member of the isomerase
class of enzymes. Note that these two molecules are epimers at the C-3 carbon.
This is shown in Figure 8-44.
CH2-OH

CH2-OH
1

C=0
1

NAD-Iinked

HO- C-H
I

H-C- OH

C=0
I

H-C- OH

Phosphopentose
epimerase

CH2-O-PO32Xylulose
5-phosphate

H-C- OH

CH2-O-PO32Ribulose

5-phosphate

Figure 8-44

How could we make this epimer? The hydroxyl function at the C-3 carbon of
xylulose-5-phosphatecould be oxidized to the corresponding keto function. This

wouldeliminate the chirality. Reduction of that ketofunction could then give the
epimeric hydroxyl function at the C-3 carbon of ribulose-5-phosphate. The
coenzyme NAD is involved in this oxidation-reduction scheme. NAD would be a

catalyst as it is not involved stoichiometrically.


Once we have ribulose-5-phosphate it can undergo an isomerization reaction to
give ribose-5-phosphate as shown in Figure 8-42. Ribose-5-phosphate could
eventually become incorporated into biomolecules such as RNA, DNA, ATP,
and NAD.

Erythrose-4-phosphate can react with DHAP via an aldol condensation and the
aldolase enzyme to give sedoheptulose-l,7-diphosphate (first isolated from
avocados, which belong to a family of plants generally known as sedum plants
hence the prefix). Sedoheptulose-l,7-diphosphate can be converted to sedo-

heptulose-7-phosphate by a hydrolase enzyme known as sedoheptulose-1,7diphosphate phosphatase. This is shown in Figure 8-45.

Copyright by The Berkeley Review

209

The Berkeley Review


Specializing in MCAT Preparation

Biology

Metabolic Pathways

Pentose Phosphate Pathway

CH2-O-PO32-

CH2-OH

C=0

Sed-l,7-dP

0V

HO-C-H

phosphatase

H-C- OH

CH2-0-PCy

H-C- OH
I

H-C- OH

C=0

"

CH2-0-P032Erythrose
4-phosphate

C=0
HO- C-H

+ H20

H-C- OH

H-C- OH

H-C- OH

Aldolase H~f" 0H .

CH2-OH

CH2-0-P032-

DHAP

Sedoheptulose
1,7-diphosphate

H-C- OH
I

CH2-0-P032Sedoheptulose
7-phosphate

Figure 8-45

Sedoheptulose-7-phosphate can react with glyceraldehyde-3-phosphate to form


ribose-5-phosphate and xylulose-5-phosphate as shown in Figure 8-46. This
reaction is catalyzed by the transketolase enzyme.
CH2-OH
1

C=0

Os

HO- C-H
I

H-C- OH

0V

H-C- OH

Trans
ketolase

H-C- OH

C=0

-1

H-C- OH
1

HO- C-H
1

H-C- OH

H-C- OH

CH2-O-PO32-

CH2-O-PO32-

H-C- OH

CH2-O-PO32-

Glyceraldehyde
3-phosphate

Sedoheptulose
7-phosphate

CH2-OH
1

H-C- OH

CH2-O-PO32Xylulose
5-phosphate

Ribose

5-phosphate

Figure 8-46

Sedoheptulose-7-phosphate and glyceraldehyde-3-phosphate can also react to


form fructose-6-phosphate and erythrose-4-phosphate. This reaction is
catalyzed by a transaldolase enzyme as shown in Figure 8-47.
CH2-OH
CH2-OH

C=0
Trans
aldolase

HO- C-H
H-C- OH

H-C- OH

H-C- OH

CH2-O-PO32Sedoheptulose
7-phosphate

"V"
1

H-C- OH
1

CH2-O-PO32Glyceraldehyde
3-phosphate

*-

C=0

O.

'

H-C- OH

1
+

H-C- OH

CH2-O-PO32Fructose

H-C- OH
1

H-C- OH

6-phosphate

HO- C-H

CH2-O-PO32Erythrose
4-phosphate

Figure 8-47

Copyright by The Berkeley Review

210

The Berkeley Review

Specializing in MCAT Preparation

Biology

Metabolic Pathways

Pentose Phosphate Pathway

Note the portion of sedoheptulose-7-phosphate that is being transferred to


glyceraldehyde-3-phosphate. The reactions shown in Figure 8-42 through Figure
8-47 represent a non-oxidative way to generate 5-carbonsugars.

The pentose phosphate pathway, which occursin the cytosol of cells, is one way
to get five carbon sugars and NADPH. There is also a mitochondrial-linked

method of obtaining NADPH (which we will examine during fatty acid


metabolism). Most eukaryotic cells have mitochondria. However, one of the most
common types of (mature) eukaryotic cells that do not have mitochondria are the
erythrocytes (red blood cells). If mature erythrocytes do not have mitochondria,

then theymust obtain their NADPH from the pentosephosphate pathway.


As oxygen becomes reduced it goes through a number of intermediate stages
such as the superoxide radical, hydrogen peroxide, and hydroxyl free radical
before it is converted into water. All of these intermediate species are "toxic" in
thesense that any oneof them (outside the usualenzymatic channel ofbecoming
water) can cause cellular damage (which is a means of aging). There are various
compounds and enzymes which can scavenge these radicals and convert them
into less toxic substances. For example, compounds like vitamin C, vitamin E

(tocopherols) and ergothioneine can act as free radical scavengers (or anti
oxidants). Superoxide dismutase (SOD) can catalyze the conversion of the
superoxideradical into hydrogen peroxide and oxygen. Hydrogen peroxide can
react with an enzymecalled catalase and be converted intowater and oxygen.
NADP+
NADPH + H+

Glutathione
reductase
Reduced
Glutathione

Oxidized
Glutathione

(GSH)

(GSSG)

H202

HoO

Glutathione

peroxidase
Figure 8-48

Hydrogenperoxide can also react with an enzymecalledglutathione peroxidase


and be reduced to water. Reduced glutathione (GSH) reacts with the peroxide to
form water and the oxidized form of glutathione (GSSG). Oxidized glutathione is
reduced back to the sulfhydryl form (GSH) by NADPH. The enzyme that
catalyzes this reaction is glutathione reductase. This is shown in Figure 8-48.
There are cases in which humans lack one or more of these "housekeeping"
enzymes. For example, there are individuals who are catalase negative (they lack
this enzyme) and who still manage to survive. Individuals have not been found

who are completely lacking in either superoxide dismutase or in glutathione


peroxidase. This suggests that these two enzymes are quite essential for normal
biological function.

Copyright by The Berkeley Review

211

The Berkeley Review

Specializing in MCAT Preparation

BiOlOgy

Metabolic Pathways

Pentose Phosphate Pathway

If an individual were deficient in the enzyme glucose-6-phosphate dehydro


genase (see Figure 8-40), then that person would not be able to synthesize
NADPH. This sex-linked trait is the single largest class of all human mutations
and is quite prevalent in malaria-infested regions of the world like the
Mediterranean area, central Africa, the Middle East, India, Southeast Asia, and

China.It turns out that glucose-6-phosphate deficiency is one means of protection


against the malarialparasite Plasmodium falciparum. (Other means of protection
involves the sickle-cell trait.) How is it that a glucose-6-phosphate deficiency
affords protection against this parasite?
Glucose-6-phosphate dehydrogenase deficiency is sex-linked. It is on the Xchromosome. Females are XX while males are XY. If a male has glucose-6phosphate dehydrogenase deficiency, he is said to be hemizygous for that defect.
A female may carry the defect on one of her X-chromosomes but not on the other

one. She is said to be heterozygous for that defect. Heterozygotes for this
deficiency turn out to have red blood cells that are about ten times more resistant

to the malarial parasite than normal wild type individuals who do not have this
deficiency. Why? The malarial parasite requires products of the pentose
phosphate pathway and reduced glutathione for their survival. If there is a
glucose-6-phosphate dehydrogenase deficiency, then these products are limited.
Note that the female of the species is the one being protected because she is the
one who is heterozygous for the trait. She is the one who bears the offspring!

Individuals with glucose-6-phosphate dehydrogenase deficiency are unable to


form NADPH and therefore unable to form reduced glutathione (GSH) as
shown in Figure 8-48. If GSH is not formed, then peroxides are free to lurk
within the cell and cause damage. Erythrocytes have a typical lifetime of about
120 days before they are degraded in the spleen. However, the erythrocytes of
individuals with this deficiency have a relatively short life span (about 14 days).
Hemolytic anemia (lysis of the red blood cells) results and hemoglobin is released
into the blood. As the hemoglobin is destroyed the protein portion (i.e., globin)
and the iron can be recycled. However, the iron-free porphyrin portion of the
heme is degraded in the liver (and spleen and bone marrow) to a compound
called bilirubin. If the concentration of bilirubin were to exceed what the liver

could excrete, a condition of hyperbilirubinemia results in which bilirubin


diffuses into the tissues and turns yellow. This condition is referred to as
jaundice.

Copyright by The Berkeley Review

212

The Berkeley Review


Specializing in MCAT Preparation

Biology

Metabolic Pathways

Gluconeogenesis

Gluconeogenesis

WS

Gluconeogenesis is the synthesis of glucose from non-carbohydrate precursors


such as lactate, amino acids like alanine, and glycerol Recall that during vigorous
exercise lactic acid accumulates in skeletal muscle. We had mentioned that

lactate is a dead end in metabolism, unless it can be converted back to pyruvate.


In order to convert lactate back into pyruvate it must first be carried by the blood
to the liver. At the liver, lactate is converted to pyruvate, which can be converted
into glucose by way of gluconeogenesis. Glucose can leave the liver and return to
the skeletal muscle to once again undergo glycolysis. These series of reactions, as
shown in Figure 8-49, are referred to as the Cori cycle.
Glucose

Gluconeogenesis K

U V-6 NTP

Pyruvate

Liver Cell

tr

Lactate ^m

Glucose
B

O
O

2NTP<^ Glycolysis
Pyruvate

^H Lactate

Muscle Cell

Figure 8-49
The Cori Cycle.

Molecules like lactate, alanine, and glycerol can be converted into glucose by
gluconeogenesis. Note that all three of these compounds contain three carbon
atoms (C3). It turns out that animals cannot get a net conversion of a two carbon
(C2) compound like acetyl CoA into glucose. Recall that when acetyl CoA enters
into the Krebs cycle and combines with OAA, two carbon atoms leave as CO2. In

otherwords, the OAA that is regenerated is not synthesized de novo whenacetyl


CoA is oxidized in the Krebs cycle. Animals lack the necessary enzymes that will
allow the conversion of a two carbon acetyl unit into glucose. However, plants
and many bacteria can make use of these two carbon precursors because they
have the enzymes necessary for the proper conversions. They use the glyoxylate
cycle and bypass the two decarboxylation reactions in the Krebs cycle by
converting the two carbon acetyl units into a four carbon succinate molecule.
With this in mind, let's consider gluconeogenesis.
Within the cytosol of (eukaryotic) liver cells the ratio of NADH/NAD is low,

thus favoring the oxidation of lactate to pyruvate. Once you have pyruvate you
might think that you can convert it to phosphoenolpyruvate (PEP) by a simple
reversal of the reaction at Step 10 in glycolysis. Recall that the AG' for the
conversion of PEP to pyruvate in glycolysis is -7.5 Kcals/mol. If we were to
reverse this reaction, the AG0' would be +7.5 Kcals/mol. This reaction must be
bypassed for one with a more favorable standard free energychange.

This bypass is catalyzed by the enzyme pyruvate carboxylase. Pyruvate


carboxylase is located inside the mitochondrial matrix of the liver cells and

contains a biotin prosthetic group that carries activated C O2. CO2 was
"activated" at the expense of a molecule of ATP. In other words, ATP facilitated

the attachment of CO2 to biotin. Pyruvate then diffuses into the mitochondrial
Copyright by The Berkeley Review

213

The Berkeley Review

Specializing in MCAT Preparation

Biology

Metabolic Pathways

Gluconeogenesis

matrix and is carboxylated to form OAA. Pyruvate carboxylase is an example of a


ligase enzyme because a high energy bond is being used. If energy was not used,
the enzyme would have been a lyase. This is shown in Figure 8-50.

0N

V
Biotin

ATP

ADP

+ C02

+Pi

Pyruvate
Carboxylase

CH3

[Biotin

C=0

Pyruvate
VJ<Carboxylase

COO

Biotin

CH2
l

c=o

Pyruvate

coo

Pyruvate
Carboxylase

OAA

Figure 8-50

It turns out that pyruvate carboxylase is activated by high levels of acetyl CoA. If
there are high levels of acetyl CoA, then acetyl CoA must not be condensing with
OAA. Why? Because the OAA levels are low and there are not enough to meet
the demand. The formation of OAA from pyruvate is called an anaplerotic reaction
(from the Greek, meaning to "fill up"). What happens to this OAA? If the cell is
low in ATP, OAA will enter the Krebs cycle and condense with acetyl CoA. This
will lead to the eventual synthesis of more ATP. However, if the cell has plenty of
ATP, then OAA will be utilized for gluconeogenesis.
Glucose
y X

Lactate
umk

GDP

NAD

GTP

oDH Pyruvate

+ HH

Potential
Futile

Q
OAA

C02

Vr NADH + H+

N^ NAD+
NAD

Cycle

Malate

Cytosol

Pyruvate

Malate

OAA
m

ATP

+ C02

ADP
+ Pi

XTATMJ

NADH
+ H+

NAD+

Mitochondrial Matrix

Figure 8-51

OAA is next reduced to form malate by NADH-linked malate dehydrogenase.

This enzyme is in the oxidoreductase class of enzymes. Malate is transported


across the mitochondrial membrane and is reoxidized to OAA in the cytosol by

an NAD-linked malate dehydrogenase. OAA is decarboxylated and


phosphorylated by GTP to give phosphoenolpyruvate (PEP). This reaction is
catalyzed by PEP carboxykinase and can be seen in Figure 8-51.
Copyright by The Berkeley Review

214

The Berkeley Review


Specializing in MCAT Preparation

Biology

Metabolic Pathways

Gluconeogenesis

In reaction sequences shown in Figure 8-51 we have the potential for a futile
cycle. Why? If we go from PEP to pyruvate, we synthesize one ATP. Pyruvate to
OAA costs us an ATP. OAA to PEP costs us a GTP. We end up with a net loss of
one NTP. If we were to continue around this cycle, we would eventually run out
of NTPs and still not have gotten anywhere. It would be afutile effort. However,
there are controls that regulate this potential futile cycle. If we were to remove
those controls, a lot of heat would be generated.

Once you have PEP the rest of the reactions in glycolysis are reversible and the
equilibrium will favor moving back towards glucose until fructose-1,6diphosphate is reached. This is true as long as there are high levels of ATP (i.e.,
the energy charge is high). High levels of ATP turn out to allosterically inhibit
pyruvate kinase, the enzyme which converts PEP to pyruvate.

Recall that the AG' for the conversion ofFructose-6-phosphate to Fructose-1,6diphosphate was about -3.4 kcals/mol. This reaction was catalyzed by phospho
fructokinase. Instead of trying to reverse this reaction it would be much easier to
hydrolyze the phosphate at the C-l position.

Recall that hydrolysis reactions are always favorable. The enzyme that catalyzes
this reaction is in the hydrolase class of enzymes and is called fructose-1,6diphosphate phosphatase. This is shown in Figure 8-52. Again, there is the
possibility of another potentialfutile cycle.

CH2-OH

HO- C-H
H-C- OH
I

H-C- OH

Fructose-6-phosphate

Fructose-1,6-diphosphate

Figure 8-52

Fructose-6-phosphate is in equilibrium with Glucose-6-phosphate. Glucosesphosphate can be used to makeglycogen (a storage form of glucose) or it can be
converted into glucose. The brain uses about 120 grams of glucose per day as an
energy source. It would be advantageous, then, to have a means for the
conversion of glucose-6-phosphate into glucose.

Recall that the AG0'value for the reaction of glucose to glucose-6-phosphate is


about -4.0 kcals/mol. This reaction is catalyzed by hexokinase. It would be easier
to overcome this energy barrier by simply hydrolyzing the phosphate from the C6 position of glucose-6-phosphate. This reaction is catalyzed by glucoses-

phosphate phosphatase. These reactions represent yet another potential futile


cycle. This is shown in Figure 8-53.

Copyright by The Berkeley Review

215

The Berkeley Review

Specializing in MCAT Preparation

Biology

Metabolic Pathways

Gluconeogenesis

-4.0

V'
I

H-C- OH

HO- C-H
l

H-C- OH

Glucose-6-phosphatey
Phosphatase

HO- C-H
I

H-C- OH
l

H-C- OH

H-C- OH

H,0

CH2-OH

II

CH2-0-P-0
1

Glucose

Glucose-6-phosphate
Figure 8-53

Recall that glycolysis (from glucose to pyruvate) will give us a net yield of 2
ATPs. Gluconeogenesis (from pyruvate to glucose) will cost us 6 ATPs. We can
think of this cycle as one large potentialfutile loop that could give us a net loss of 4
ATPs.

Copyright by The Berkeley Review

216

The Berkeley Review


Specializing in MCAT Preparation

Biology

Metabolic Pathways

Patty Acid Oxidation

Let's look at fatty acid metabolism. We will startwith fats or triglycerides which
have the general structure shown in Figure 8-54. The carboxyl function of the
fatty acid is in an carboxyester linkage with the hydroxyl of the glycerol. The
structure shown in Figure 8-54 has three carboxyester linkages. The most
common fatty acids are 16 or 18 carbon atoms long and they are the energy
storage form that is mostfrequently usedin animals and plants.

H2C- O- C- CHr(CH2)n.CH3

H-C- O- C- CH2-(CH2)n-CH3

There is more energy available per unit weight of triglyceride than of hydrates
like glycogen. You can see that if you think of-CH2- asbeing the form in which
carbon exists. If we were to oxidize that unit, then it would take 1.5 oxygen

H2C- O- C- CH2-(CH2)n-CH3

molecules to bring it to CO2 and H2O. In contrast, if you were to think of a

A Triacylglycerol

carbohydrate structure like -CHOH-, then you would find that it is already
partially oxidized. As a result, only 1 oxygen molecule isrequired to oxidize that

Figure 8-54

carbonand hydrogen completely to CO2 and H2O.


If the amount of energy that had been concentrated in these neutral fats were

instead stored in the form ofglycogen, then you would be grossly overweight.
There are a couple of reasons for this. Fats are at a lower oxidation state. The

average oxidation state ofthe carbons ina fatty acid molecule is-2. The average
oxidation state of the carbons ina carbohydrate is 0. In the process of turning that
carbon into CO2 there is a lot more oxidation taking place. The result is more
energy being produced from the burning ofa gram offat compared toa gram of
carbohydrate. Fats are also fairly hydrophobic and sowhen they are stored there
is not very much water. This is quite different from the storage of glycogen
where we find the molecule to be full of hydroxyl groups. There is a lot of water
stored along with the glycogen. Onedisadvantage that fathas is that it cannot be
metabolized anaerobically. Fats have tobemetabolized aerobically.

Let's consider the way in which fat is metabolized. This pathway involves 8
enzymatic steps. The first step that we want to consider involves the enzyme
lipase. To mobilize triglycerides (or neutral fats), the first step would be the
hydrolysis of the carboxyester bond. This is accomplished by hydrolysis of the
triglyceride into a molecule of glycerol and 3 fatty acid residues. This is shown
in Figure 8-55.

II

11

H2C- O- C- CH2-(CH2)n-CH3

HO- C- R,

Lipase

H-C- O- C- CH2-(CH2)n-CH3
O
ll

H2C- O- C- CH2-(CH2)n-CH3

H2C- OH
l

11

H-C- OH

3H20

HO- C- R2

H2C- OH

Glycerol

HO- C- R3

Fatty Acids
Figure 8-55

Let's briefly consider whathappens to the fatty acids andglycerol after the lipase
reaction shown in Figure 8-55 has taken place. The fatty acids that areproduced
will be undergo a series of reactions and be converted to acetyl CoA. Glycerol
will be also be converted to acetyl CoA, but by a different series of reactions.
Copyright by The Berkeley Review

217

The Berkeley Review

Specializing in MCAT Preparation

Biology

Metabolic Pathways

Patty Acid Oxidation

Acetyl CoA will then be able to enter into the Krebs cycle and energy, in the form
of ATP, will be produced as we have previously discussed. A very general view
of this outline is shown in Figure 8-56.
FATS

Glycogen

Fatty Acids

Occurs in the
mitochondrion

- Acetyl CoA

Pyruvate

Figure 8-56

Glycerol Metabolism

The activation of glycerolis catalyzed by glycerol kinase and ATP. Glycerol-3-

phosphate is produced. The coenzyme NAD reacts with glycerol-3-phosphate


to form DHAP. This is catalyzed by glycerol phosphate dehydrogenase and
occurs in the cell cytosol. This general reaction sequence is shown in Figure 8-57.
NADH
ATP

H2C- OH
I

H-C-OH

H2C- OH

NAD+ + H+

ADP

^2.

H2C- OH

Glycerol

H2C- OP032' glycerol

H-C-OH

H2C- OH
I

:>

C=0

H2<c_ OP032-

phosphate

Kinase

Glycerol

Glycerol

dehydrogenase

DHAP

3-phosphate
Figure 8-57

Activation of Fatty Acids

The second step that we want to consider is the activation of the fatty acids. We will
find that fatty acids are degraded 2 carbons at a time. If we are going to split
these carbons off two at a time, then we will need some type of "handle" at the Im

position of the fatty acid. What we will need is a keto group at the p-position. If
we had a p-keto acid, the carboxylfunction would decarboxylate and we would
lose that carbon atom. As a result this is not handled as a p-keto acid but rather as

the thioester. The thioester of a p-keto acid does not decarboxylate. How do we
form this thioester linkage?

Copyright by The Berkeley Review

218

The Berkeley Review


Specializing in MCAT Preparation

Biology

Metabolic Pathways

Patty Acid Oxidation

The driving force for forming the thioester linkage between the sulfhydryl group
of CoA and the carboxyl group of a fatty acid isATP. If we are going to do this
reaction with ATP, then one phosphate group will not be enough. The hydrolysis
energy of a thioester is about -7.5 kcal/mol. If we allowed the use of only one
high energy phosphate bond, then we would have a AG' ofonly -7.3 kcal/mol.
Clearly this reaction willnot proceed verywell. What wefind here is that ATP is

hydrolyzed to AMP and pyrophosphate (PPi). Pyrophosphate will be


hydrolyzed in thepresence of water and pyrophosphatase to two molecules of

orthophosphate, giving aAG' of another -7.3 Kcal/mol. The activation of the fatty

acid is now complete as shown inFigure 8-58. This activation reaction takes place
on the outer mitochondrial membrane and iscatalyzed by the enzyme acyl CoA
synthetase (which is in the ligase enzyme class).
O
ii
e
R-C-0

O
ll

ATP ^=

=*=

R-C-AMP

+ PPj

Acyl-CoA Acyl Adenylate

Fatty Acid

Synthetase

O
ll

R-C-AMP

HS-CoA ^

R-C-S-CoA

Acyl Adenylate

AMP

Acyl-CoA

Figure 8-58

Once the fatty acid isactivated in the cytosol ofthe cell itneeds tobetransported
into the mitochondrial matrix where it can be oxidized. Activated fatty acyl CoA
molecules are shuttled across the inner mitochondrial membrane by carnitine.
Once the activated fatty acid is released in the mitochondrial matrix, carnitine

will return to the cytosolic medium andthe process will repeat itself.
The p-Oxidation Pathway

Once the acyl-CoA molecule is in the mitochondrial matrix, we can begin Poxidation. The first four steps of p-oxidation are highly reminiscent ofthe steps
that we saw in the Krebs cycle. The third step in our breakdown reaction is the
formation of a trans double bondbetween the alpha and thebeta carbons in the

acyl-CoA structure. This is accomplished by an oxidation of the acyl-CoA


structure with FAD. The enzyme involved is acyl-CoA dehydrogenase. This
oxidation yields an enoyl-CoA. Acyl-CoA slightly resembles succinic acid
whereas enoyl-CoA slightly resembles fumaric acid in the Krebs cycle. This is
shown in Figure 8-59.

r.

II

Ark
FAD
\

FADH2
A
z
t

V7

R-CH2-CH2-CH2-C-S-CoA i ^^

,,

II

HO

> RCH2-C=C-C-S-CoA

Oxidation

Fatty Acyl-CoA

Acyl-CoA
dehydrogenase

Enoyl-CoA

Figure 8-59

In stepfour we have hydration of the doublebond in enoyl-CoA. Thetwo ends of

this molecule are not symmetrical and therefore the hydration ofenoyl-CoA is
stereospecific. The enzyme thatcatalyzes this reaction is enoyl-CoA hydratase. We
will onlyget the L-isomer, whichis L-Hydroxyacyl-CoA as shownin Figure8-60.
Copyright by The Berkeley Review

219

The Berkeley Review

Specializing in MCAT Preparation

Biology

Metabolic Pathways

Fatty Acid Oxidation

This reaction is analogous to the hydration of fumarate to malate in the Krebs


cycle.

R CH2 - C= C- c - s- CoA
H

HO
l

II

4> RCH2-C-C-C-S-CoA
Hydration

Enoyl-CoA
Hydratase

Enoyl-CoA

L-Hydroxyacyl-CoA

Figure 8-60

In stepfivewe have another oxidation reactionwhere we introduce a keto group at


the P-position. This reaction is catalyzed by the enzyme L-3-hydroxyacyl-CoA

dehydrogenase. The coenzyme involved is NAD. The reaction is shown in


Figure 8-61. This reaction is analogous to oxidation of malate to oxaloacetate in
the Krebs cycle.
NADH

NAD+
HO

ll

R CH2- C- C- C- S- CoA
i

+ H+
OHO

LZ

ll

ll

R CH2 - C- C- C- S- CoA
i

Oxidation

L-3-Hydroxyacyl-CoA
L-Hydroxyacyl-CoA
dehydrogenase

Ketoacyl-CoA

Figure 8-61

In step sixwe have the cleavage of ketoacyl-CoA betweenthe a and p carbons. A


second molecule of CoA-SH is used in the process. The products are an acetylCoA molecule (which can be utilized in the Krebs cycle) and a fatty acyl-CoA
molecule that has been shortened by 2 carbon atoms. This is shown in Figure 8-62.

The enzyme that catalyzes this thiolytic cleavage is p-ketothiolase. Thefatty acylCoAmolecule that we just produced is already activated, which means that we do
not have to repeat the activation step shown in Figure 8-58.
R

ll

ii

H2C C- C- C-

CoA-SH

II

ll

H2CC-S-CoA + CH3- C- S-CoA

S- CoA c

Thiolysis
H

Fatty Acyl-CoA

p-Ketothiolase q carbons shorter)

Ketoacyl-CoA

Acetyl-CoA

Figure 8-62

The Oxidation of Palmitic Acid (a Cie even-numbered fatty acid)

If we were to completely oxidize palmitate using the p-oxidation pathway, we


would go through the cycle that we just described 7 times to get 8 two-carbon

acetyl CoA fragments, 7FADH2 molecules and 7 NADH + H molecules. This


can be seen in Figure 8-63. In return for these products we will get a wealth of
ATP's viatheKrebs cycle, electron transport and oxidative phosphorylation. The
point of this process is that the sum of the ATP liberated by the complete
oxidationof a fatty acidmakes this a very elegantbreakdown process.
Copyright by The BerkeleyReview

220

The Berkeley Review

Specializing in MCAT Preparation

Biology

Metabolic Pathways

Patty Acid Oxidation

Figure 8-63

We are now in a position to calculate the net production of ATP from the

oxidation ofone molecule ofpalmitic acid. Inthe activation step we used 1 ATP
and we hydrolyzed the pyrophosphate bond. In other words, we have used 2

high energy phosphate bonds to activate the fatty acid for p-oxidation. Attheend

of the p-oxidation process we will have formed the equivalent of 129 high energy

phosphate bonds. The restofthesteps in ourprocess areoutlined in Table 8-1.


Process
Activation

ATP Equivalents
-02

Oxidation (7 rounds)

7 FADH2 (worth 2 ATP's each)


7 NADH (worth 3 ATP's each)

+ 14
+ 21

Acetyl-CoA (catabolism)

8 Acetyl-CoA's enter Krebs Cycle


8 Acetyl-CoA x 3 NADH

+ 72

8 Acetyl-CoA x 1 FADH2

+ 16

8 Acetyl-CoA xl GTP

+ 08

+ 129
H

I
Table 8-1

The AG' for this process is about -2340 Kcal/mol. The fraction of energy
available (which is -2340 Kcal/mol) that is actually trapped as ATP is about40%
(129 x 7.3 = 941.7, and then 941.7/2340 x 100% * 40%).

IT ^ ^ C
Y P a
m-Enoyl-CoA
Isomerase

Unsaturated Fatty Acids

Not all fatty acids are completely saturated with hydrogen atoms. Some have
double bonds in them. These arereferred to as unsaturatedfatty acids. There are
two type of situations concerning double bondsin unsaturated fatty acids. One
situation is that you arrive at a double bond in the p/y-position after p-oxidation

while the other situation is that you arrive at a double bond at the a,p-position

R^^C'S-C0A
H

trans-Enoyl-CoA

after P-oxidation.

If the double bond is at the p/y-position, we need to convert it to a double bond at


the oc,p-position. The enzyme that catalyzes this reaction is an isomerase. This is

shown in Figure 8-64. Once you have the a,p-position established you areat the
level of enoyl-CoA in p-oxidation. Theenoyl-CoA double bond is trans. If it were
cis, it wouldneed to be converted to trans by an isomerase.

Copyright by The Berkeley Review

221

P-oxidation

Figure 8-64

The Berkeley Review

Specializing in MCAT Preparation

Biology

Metabolic Pathways

Patty Acid Oxidation

The Oxidation of Pelargonic Acid (a C11 odd-numbered fatty acid)

Pelargonic acid is a C-11 odd chained fatty acid. If we were to completely oxidize
pelargonic acid using the p-oxidation pathway, we would go through the cycle
that we have been discussing 4 times to get 4 two-carbon acetyl CoA residues
and 1 three-carbon propionyl-CoA residue. This is shown in Figure 8-65.
CH3-CH2-CH2-l-CH2-CH2-l-CH2-CH2-l-CH2-CH2-l-CH2-COa

#5

#4

#3

#2

#1

Figure 8-65

Propionyl-CoA Metabolism in Animals


In order to utilize propionyl-CoA we must carboxylate it to form DMethylmalonyl CoA. This reaction is catalyzed by propionyl CoAcarboxylase
(a biotin enzyme). This enzyme is a ligase because ATP is simultaneously
converted to AMP and PPj. Pyrophosphate is subsequently hydrolyzed to two
molecules of orthophosphate. D-Methylmalonyl-CoA is converted to L-Methylmalonyl-CoA by a racemase enzyme (which is in the class of enzymesreferred to
as isomerases). L-Methylmalonyl-CoA is next converted to succinyl-CoA by the
enzyme methylmalonyl-CoA mutase. This is a unique enzyme because it
contains vitamin B12 (cobalamin) as its coenzyme. [There is only one other
reaction known in mammals dependent on vitamin Bj2 and that is the formation
of methionine by methylation of a homocysteine residue.] Once succinyl-CoA is
formed it can then enter into the Krebs cycle.
Can Fatty Acids be converted into Carbohydrates?

Would it be possible for animals to convert fatty acids into carbohydrates? We


know that whenfatty acids are degradedby the P-oxidation pathway, a wealthof
2-carbon acetyl-CoA units are synthesized.These acetyl-CoA units will enter into
the Krebs cycle and condense with OAA to form citrate. However, as citrate goes
around the cycle there are two decarboxylation reactions, catalyzed by isocitrate
dehydrogenase and oc-ketoglutarate dehydrogenase. Even though we will still
form OAA as we continue around the cycle, we will have lost 2 carbons in the

process, which means that we would be unable to convert fatty acids into
carbohydrate material. Plants and many bacteria, though, can utilize acetyl-CoA
for energy production and biosyntheses by using a reaction sequence called the
glyoxylate cycle. However, animals can degrade carbohydrates to acetyl-CoA
units and then take those acetyl-CoAunits to synthesize fatty acids.

Not only does the Krebs cycle function in the oxidative catabolism of amino
acids,fatty acids, and carbohydrates, but it also serves as a primary starting point
in manybiosynthetic reactions forwhich it is able to provide precursors. If these
precursors were removed from the Krebs cycle for various other metabolic
pathways, then therate at which theKrebs cycle operated wouldbegin to decline
(andmight even stall). One of themostimportant anaplerotic reactions (from the
Greek, to "fill up") is the synthesis of OAAfrom pyruvate and CO2. This reaction
is catalyzed by the enzyme pyruvate carboxylase. For example, if a cell was
exclusively catabolizing fatty acids to obtain a high level of ATP, and if there was
no carbohydrate catabolism taking place, then the anaplerotic reaction of

pyruvate to OAA would not take place. The result is that the Krebs cycle would
run down. In other words, fatty acid degradation needs the flickering flame of
carbohydrate degradation to keep it going (i.e., a little conversion of pyruvate to
OAA is needed to ensure that fatty acid degradation will continue).
Copyright by The Berkeley Review

222

The Berkeley Review

Specializing in MCAT Preparation

Biology

Metabolic Pathways

Urea Cycle

fBif^^
If we hydrolyze proteins, we will be able to obtain a variety of amino acids. The
amino acids that we obtain can then be used in a variety of reactions, one of

which is protein synthesis. However, for most amino acids the amino group can
be removed to yield the cc-keto acids which can then be used in citric acid cycle
intermediates to eventually give ATP, C02 and H20. This is shown inFigure 8-

66. Let's focus on the Phase I and Phase II (we have already looked at Phase III
in the Krebs cycle).

Proteins i

H20

> Ammo

p a-Keto Acids
> Krebs cycle
AcJds Phase I
Phase II Intermediates
Phase III
v

Protein Synthesis

ATP,C02,H20

Figure 8-66
Phase I

In this phase of amino acid degradation (the major site in mammals being the
liver) we are essentially dealing with two reactions. The first is amino transfer

from an amino acid to oc-ketoglutarate as shown in Figure 8-67. (The portions


that are in boxes will form the a-keto acid.) a-ketoglutarate is the acceptor
molecule ofthe amino group from most amino acids. The enzyme that catalyzes
this reaction is an aminotransferase (also called a transaminase). There will be a
specific aminotransferase for each amino acid.

For example, if you wanted to transfer the amino group from alanine to ccketoglutarate, then you would use alanine aminotransferase. The mechanism of

these reactions involves the coenzyme pyridoxal phosphate (PLP) which is


derived from vitamin B& (pyridoxine).

H O
PLP

NH3- C- C- O

Acid

II

II

NH3- C- H

R,-C-C-0
Amino-

a-Amino

transferase

COO

COO

CH2
I

a-Keto
Acid

a-Ketoglutarate

CH2

COO
Glutamate

Figure 8-67

The glutamate that is produced in Figure 8-67 gets oxidatively deaminated as


shown in Figure 8-68. Glutamate dehydrogenase catalyzes this reaction and

either NAD or NADP can be utilized. Note that nitrogen is released in the
form of NH4.

Copyright by The Berkeley Review

223

The Berkeley Review

Specializing in MCAT Preparation

Biology

Urea Cycle

Metabolic Pathways

COO

coo

NH3- C- H

+ NAD+ +
(or NADP*)
CH2
1

c=o

A.

H20;

CH,

CH2

Glutamate

dehydrogenase

NADH
(or NADPH)

NH4

CH,

"0

COO

COO

a-Ketoglutarate

Glutamate

Figure 8-68

The sum of the reactions shown in Figure 8-67and in Figure 8-68 can be seen in

Figure 8-69. In terrestrial vertebrates theexcess NH4 produced in Figure 8-68 is


converted into urea and then excreted. In most aquatic organisms, NH4 itself is
excreted, while in birds and terrestrial reptiles the NH4+ is converted to uric acid
before it is excreted.

+ H20

a-Amino Acid + NAD+=== a-Keto Acid + NADH + H++ NH4+

(or NADP+)

(r NADPH)

Figure 8-69
Phase II

Let's consider the degradation of the carbon skeleton of the amino acids. Recall
that carbohydrate metabolism will eventually yield pyruvate which can enter the
Krebs cycle through acetyl-CoA. We also mentioned that fatty acid degradation
generates acetyl-CoA as well. When amino acids are degraded they will form
metabolic intermediates which can be tunneled into the Krebs cycle.

Carbohydrate
Metabolism

Fatty
Acids

Pyruvate

Acetoacetyl
CoA

Leu, Lys,
(Phe), (Tyr),
(Trp)

(Asn, Asp) Oxaloacetate

(JPhlpTyjFu"arate

Ketogenic

Ejrefes

Cycle

Citrate

f(Ile),Met!\Succinyl

V Val J

glutarate

CoA ^

-Keto-

rGlu, GlrO
His, Pro,

Arg

Figure 8-70

Some of these amino acids will provide the carbon skeleton framework for the

net synthesis of carbohydrate via gluconeogenesis. These amino acids are termed
Copyright by The Berkeley Review

224

The Berkeley Review


Specializing in MCAT Preparation

Biology

Metabolic Pathways

Urea Cycle

Some of these amino acids will provide the carbon skeleton framework for the
net synthesis of carbohydrate via gluconeogenesis. These amino acids are termed

glucogenic and they will be degraded to pyruvate, a-ketoglutarate, succinyl-

CoA, fumarate, and oxaloacetate. This is shown in Figure 8-70. Other amino
acids will be degraded to acetyl-CoA and acetoacetyl-CoA. These amino acids
are termed ketogenic because they will eventually give rise to ketone bodies.
Remember, mammals do not have a pathway that will allow acetyl-CoA (or
acetoacetyl-CoA) to be converted to carbohydrate (due to the loss of CO2 in the
Krebs cycle).

As we have seen, amino groups will flow from the various amino acids to

glutamate and then to ammonia by the glutamate dehydrogenase reaction. The


free ammonia that we produced in the glutamate dehydrogenase reaction
combines with CO2 to form carbamoyl phosphate. This isshown in Figure 8-71.
This reaction is catalyzed by carbamoyl phosphate synthetase, requires 2 ATPs,
and occurs in the mitochondrial matrix. Since carbamoyl phosphate isnow a high
energy compound, we can think of it as an activated carbamoyl donor.
O
11

O
11

C02 + NH4+ + 2ATP + H20=> H2N-C-0-P-0 + 2ADP + Pj+ 3H+


Carbamoyl Phosphate
Figure 8-71

In the next reaction, carbamoyl phosphate reacts with a molecule of ornithine (an
amino acid that does not appear in proteins). Carbamoyl transcarbamoylase
transfers the carbamoyl group from carbamoyl phosphate to ornithine to
produce citrulline as shown in Figure 8-72. Citrulline is another example of an
amino acid that does not occur in proteins. Citrulline can leave the mitochondrial
matrix and enter into the cytoplasm.
O

ll

NH3
1 J
CH2
1

CH2
I

H-N- C- NH2
O

Ornithine

11
11
transcarbamoylase
H2N-C-O-P-0 1 ^
>

CH2 ^

I z

H-C- NH3
1

Carbamoyl Phosphate

CH2
CH2
I

CH2
I

Pi

"

H-C- NH3

COO0

COO^

Ornithine

Citrulline

Figure 8-72

If we take citrulline and let it be aminated by aspartate via the enzyme arginosuccinate synthetase, we will form arginosuccinate, which can be cleaved by the
enzyme arginosuccinase to give arginine and fumarate. This reaction is a way to
synthesize arginine from ornithine. How do you get ornithine? If you hydrolyze
arginine with water, you will get ornithine and urea. Urea can then be excreted.
The overall pathway for the urea cycle is shown in Figure 8-73. [Once fumarate is
generated it can react with water and be converted to malate. Malate reacts with
Copyright by The Berkeley Review

225

The Berkeley Review

Specializing in MCAT Preparation

Biology

Urea Cycle

Metabolic Pathways

NAD and is converted to oxaloacetate. Oxaloacetate can react with glutamate to

give aspartate and a-ketoglutarate.]


9 ATP

AD" + "j

NH4+ +COf^=^=

II

II

> H2N-C-0-P-0"

Carbamoyl
phosphate
synthetase

o"
Carbamoyl
phosphate
H

H
+

Ornithine

CH2
i
CH2

> H3N-C-COO

H3N- C- COO i
Ornithine

CH2

transcarbamoylase

Citrulline

CH2
I

CH2

CH2

+NH,

H-N- C- NH2
II

II

H2N-C-NH2.

H3N- C- COO

Urea

Arginosuccinate
synthetase

Arginase

CH2
I

COO'

HoO

Aspartate
H

H
+

CH2

CH2
I

CH2

H3N- C- COO

H3N- C- COO

Arginosuccinase

CH2

CH2

CH2

H-N- C- NH2

H-N- C- NH2
II

NH2
+

Arginine

COO"
CH

II

OOC- C- CH2- COO'

II

CH

COO"

Arginosuccinate

Fumarate

Figure 8-73
The Urea Cycle

Copyright by The Berkeley Review

226

The Berkeley Review


Specializing in MCAT Preparation

Metabolic

Pathways
15 Passages
100 Questions

Passage Titles
I.
II.
III.
IV.
V.
VI.

VII.

VIII.
IX.
X.

XI.
XII.
XIII.
XIV.
XV.

Glycolysis and Electron Transport Chain


Electron Transport Chain
Cholesterol Metabolism and Regulation
Very Low-Density Lipoproteins (VLDLs)
Calvin Cycle
Lactose Intolerance

(^Oxidation

Glycogen Metabolism
Glycolysis and 2,3-Bisphosphoglycerate
Leucine Catabolism

Trehalose Experiment
Fuel Oxidation during Exercise
Urea Cycle
Gluconeogenesis and the Cori Cycle
Starch Blockers

Berkeley
Specializing in MCAT Preparation

Questions
1 -5
6- 11
12- 17
18-23
24-29
30-36
37 - 43
44-50
51 -58
59 - 65
66-72
73-79
80-87
88-94
95- 100

Suggestions
The passages that follow are designed to get you to think in aconceptual manner about the processes
of molecular biology at the organismal level. If you already have asolid foundation in molecular biology,
many of the questions you read here will seem to be very straight forward and easy to answer. But if you
are new to the subject or if you have not had a pleasant experience with molecular biology in the past,
some of them might appear to come from the void that spreads out beyond the Oort field at the edges of
our solar system.

Pick afew passage topics at random. For these initial few passages, do not worry about the time. Just
focus on what is expected of you. First, read the passage. Second, look at any diagrams, charts, or graphs

in it. Third, read each question and the accompanying answers carefully. Fourth, answer the questions
the best you can. Check the solutions and see how you did. Whether you got the answers right or wrong,

it is important to read the explanations and see if you understand (and agree with) what is being

explained. Keep a record of your results.

After you feel comfortable with the format of those initial few passages, pick another block of
passages and try to do them in one sitting. Be aware that time is going to become important. On average,
you have about 1minute and 15 seconds to complete a question. Be creative in how you approach this
next group. If you feel comfortable with the outline presented above, fine. If not, then try different
approaches to a passage. For example, you might feel well versed enough to read the questions first and
then try to answer some ofthem/without ever having read the passage. Maybe you can answer some of
the questions byjust looking at the diagrams, charts, orgraphs that are presented in a particular passage.
Remember, there are many effective learning styles. You need to begin to develop a format that works
best for you. Keep a record of your results.

The last block of passages might contain at least a few topics that are unfamiliar even to those who
know a good deal about molecular biology. Find a place where the level of distraction is at a minimum.
Getout your watch and time yourself on these passages, eitherindividually or as a group. It is important
to have a feel for time, and an awareness of how much is passing as you try to answer each question.

Never let a question get you flustered. If you cannot figure out what the answer is from information
given to you in the passage, or from your own knowledge base, dump it and move on to the next
question. As you do this, make a note of that pesky question and come back to it when you have more
time. When you are finished, check your answers and make sure you understand the solutions. Be
inquisitive. Ifyou do notknow the answer tosomething, look it up. The solution tends to stay with you
longerthat way. (For example, what is the Oortheld, anyway?)
The estimated score conversions for 100 questions are shown below. At best, these are rough

approximations andshould be used only to give one a feel for which ballpark they aresitting in.
Section VIII

Estimated Score Conversions


Scaled Score
>13

Raw Score

80-100

11-12

70-79

9-10

60-69

7-8

50-59

5-6

40-49

<4

0-39

Biology

Glycolysis and Electron Transport Chain

Passage I (Questions 1-5)

Passage I

Louis Pasteur discovered that by depriving a culture


of yeast cells access to oxygen, their growth rate
decreased by a factor of 6, while their glucose

Glycolysis involves a series of reactions that converts

glucose into pyruvate. Under aerobic conditions pyruvate


enters the citric acid cycle, where it is completely
oxidized to CO2 and H2O. During glycolysis and the
citric acid cycle, high-energy phosphate bonds are
produced in the form of ATP and GTP (collectively
known as nucleoside triphosphates or NTPs) by a process

utilization increased. Under aerobic conditions,


yeasts can synthesize 36 NTPs per molecule of
glucose oxidized. The rate of glucose consumption
under anaerobic conditions would increase by
approximately what factor?

called substrate-levelphosphorylation.
The NADH and FADH2 generated from glycolysis and
the citric acid cycle transfer their electrons to the electrontransport chain in the inner mitochondrial membrane. As

A.

B.

C.

D.

the electrons flow down a series of electron carriers to

molecular oxygen, protons are pumped into the


intermembrane space from the matrix, establishing an
electrochemical gradient that is used to do work. ATP is
synthesized from ADP and Pi as protons move through an
F0Fi ATPase located in the inner membrane. This process
is called oxidative phosphorylation.

If oligomycin is added to a growing culture of yeast


cells, the culture will:

A.

B.

Cytosol
Outer Membrane

C.

H+

Intermembrane

Space

H+

Inner Membrane

D.

continue with aerobic respiration, synthesize


36 net NTPs, produce CO2 and H2O, and
increase their growth rate.
switch to anaerobic fermentation, synthesize 4
net NTPs, produce CO2 and CH3CH2OH, and
increase their growth rate.
switch to anaerobic fermentation, synthesize 2
net NTPs, and produce CO2 and CH3CH2OH,
and decrease their growth rate.
continue with aerobic respiration, synthesize 2
net NTPs, produce CO2 and H2O, and
decrease their growth rate.

NADH NAD+ FADH2 AD


+ H+
Matrix

ADP h+ ATP

Both oligomycin and 2,4-DNP are simultaneously


added to an aerobic culture of growing yeast cells.

+ P5

In this case:

The F0F1ATPase can be inhibited by oligomycin,


which binds specifically to the FD portion of the complex.

A.

When oligomycin is present in a concentration sufficient


to allow binding to every one of the Fo subunits, ATP
cannot be synthesized by oxidative phosphorylation.
Under these circumstances, electron transport stops, the
electrochemical gradient increases, and the energy
associated with the electron-transport chain becomes
insufficient to pump any more protons from the matrix
into the intermembrane space.

B.

C.

Other chemical agents can interfere with the


production of NTPs as well. For example, arsenic acid
(H3ASO4) has a chemistry similar to that of phosphorus
acid (H3PO4). The compound 2,4-dinitrophenol (2,4DNP) uncouples electron transport from oxidative
phosphorylation by transporting hydrogen ions from the
intermembrane space to the matrix of the mitochondrion,
thus bypassing the F0Fi-ATPase complex.

Copyright by The Berkeley Review

D.

229

oxidative phosphorylation is stimulated, a


proton gradient will not be formed, electron
transport will increase, CH3CH2OH and CO2
are generated, and 36 net NTPs are made.
oxidative phosphorylation is stimulated, a
proton gradient will be formed, electron
transport will increase, H2O and CO2 are
generated, and 2 net NTPs are made.
oxidative phosphorylation is inhibited, a
proton gradient will not be formed, electron
transport will increase, H2O and CO2 are
generated, and 4 net NTPs are made.
oxidative phosphorylation is inhibited, a
proton gradient will not be formed, electron
transport will increase, H2O and CO2 are
generated, and 2 net NTPs are made.

The Berkeley Review


Specializing in MCAT Preparation

Glycolysis and Electron Transport Chain

Biology
4.

Passage I

Step 6 in glycolysis is catalyzed by glyceraldehyde


3-phosphate dehydrogenase. If arsenate is supplied
in place of inorganic phosphate, the enzymatic
reaction proceeds quite well as shown below. This
linkage between the C-l carbon and arsenate is
unstable and decomposes to 3-phosphoglycerate and
inorganic arsenate.
NADH

<VH

NAD+

+ H+

+ ASj

V J

h- c- oh
Enzyme
h2c-o-po32Glyceraldehyde
3-phosphate

o0
1
0
CU 0 As-0
H -

1
C- OH

HjC-O-POj2'
l-Arseno-3-

phosphoglycerate

Addition of arsenate to a culture of cells growing


anaerobically on glucose:

A.
B.

C.

D.

inhibits glycolysis at Step 6 and prevents


further growth of the cells.
does not inhibit glycolysis but prevents further
growth of the cells, because ADP cannot be
phosphorylated in Step 7.
inhibits glycolysis at Step 7 but prevents
further growth of the cells, because ATP
cannot be produced in a net yield.
does not inhibit glycolysis but induces the
culture to switch from aerobic to anaerobic

metabolism, so they can continue to grow.

5.

In terms of its effect on glycolysis, arsenate is BEST


described as:

I.
a competitive inhibitor.
II. a noncompetitive inhibitor.
III. an uncoupling agent.
A.

B.
C.
D.

I only
II only
I and III only
II and III only

Copyright by The Berkeley Review

230

The Berkeley Review

Specializing in MCAT Preparation

Biology

Electron-Transport Chain

Passage II

Cytosol
Outer

Membrane

1/

I
Inner
f" Membrane

NADH

+ H+

NAD+

ADP
+ Pj

ATP

H+

Transport
Protein

ATPase

Figure 1

The FAD-linked dehydrogenase at complex II


interacts with coenzyme Q in very much the same

Passage II (Questions 6-11)

Mitochondria are organelles that have two membranes

fashion.

(Figure 1). The outer membrane contains proteins called


porins that permit the passage of molecules with
molecular weights of less than 10,000. The inner
membrane is selectively permeable to different molecules
and is quite impermeable to polar and ionic substances.

Complex III contains a number of cytochromes, hemecontaining proteins involved in one-electron transfers, and
an iron-sulfur protein. This complex, also called the
cytochrome be 1 complex, passes the electrons to a small
peripheral membrane protein called cytochrome c (Cyt c).

The matrix is enclosed by the inner membrane and


contains the enzymes necessary for the citric acid cycle,
fatty acid oxidation, and even amino acid oxidation.

Cytochrome c diffuses through the membrane and


passes its electrons to complex IV, known as cytochrome

Between the two membranes is the intermembrane space.

oxidase. These electrons are accepted by cytochromes a,

a3, and two copper ions before they are passed to


molecular oxygen. Electron flow through this complex
results in protons being pumped into the intermembrane

Most of the electrons that enter the electron-transport

chain come from the action of dehydrogenase enzymes.

Many of the dehydrogenases are NAD-specific. These


enzymes remove two hydrogen atoms from their
substrate; one in the form of a hydride ion (:He) and the
other leaves as a proton (He). Dehydrogenases that are
FAD-specific can also remove two hydrogen atoms.

space.

Protons from the intermembrane space can return to

the matrix though the F0 transmembrane proton channel.


Attached to the F0 component on the matrix side is an
ATPase designated as Fi. As protons pass into the matrix
through the F0Fi ATPase, ATP is synthesized from ADP

The electron-transport chain operates within the inner


mitochondrial membrane and is composed of a series of
electron carriers, many of which are integral membrane
proteins that bear prosthetic groups capable of carrying

and Pi.

out electron-transfer reactions.

Inhibitors like dicyclohexylcarbodiimide (DCCD) and


oligomycin inhibit proton transport through F0. Amytal
and rotenone inhibit electron transport from complex I to
coenzyme Q. Finally, electron transport and oxidative
phosphorylation can be uncoupled using hydrophobic
agents like 2,4-dinitrophenol (DNP), FCCP, and CCCP.

The NAD-linked dehydrogenase at complex I not


only transfers hydride ions to coenzyme Q, but it can also

pump protons into the intermembrane space. As QHe


diffuses through the membrane to complex III, it picks up
a proton from the matrix and forms QH2. Electrons are
passed to complex HI, and protons are pumped into the
intermembrane space.

Copyright by The Berkeley Review

231

The Berkeley Review

Specializing in MCAT Preparation

6.

Passage II

Electron Transport Chain

Biology

Electrons from pyruvate enter the electron-transport

10.

The flow of electrons from succinate to coenzyme Q

occurs by way of the FADH2 that is produced in the

chain at:

oxidation of succinate to fumarate.

7.

A.
B.
C.

Complex I
Complex II
Complex III or IV

D.

none of the above

FADH,
FAD

Succinate

The chemiosmotic hypothesis states that ATP is

free energy decreases. This decrease can be


quantified if we know the standard reduction
potentials (E0) of redox pairs and by using the

the:

passage of electrons from NADH and FADH2

equation

to molecular oxygen.

B.

electrochemical gradient that forms across the

AG0' = -(n)(F)(AEo')

inner mitochondrial membrane.

C.

increased permeability of ADP into the


mitochondrial matrix at a specific protein

where AG' is the standard free-energy change, n is

antiport.

D.

the number of electrons transferred, F is the Faraday


constant, and AE0 is the change in standard

increased permeability of ADP into the


mitochondrial matrix at a specific protein

reduction potential between the oxidized and

symport.

8.

Fumarate

As electrons flow down the electron-transport chain,

synthesized in the mitochondrial matrix because of


A.

LX

reduced species.

Synthetic phospholipid vesicles were constructed,


and purified F0 protein was incorporated into the
membrane. High levels of K were added to the

The following information is given:


En (volts)

vesicles. A short time later, valinomycin was added

Redox Pair

to the exterior medium. H translocation was not

Fumarate + 2He + 2ee Succinate

observed in vesicles missing the Fo protein. After

FAD + 2H + 2ee FADH2

valinomycin was added, however, H translocation

0.03

- 0.22

occurred, because valinomycin was able to:


We can conclude that in the reaction involving the

A.

transport H from the exterior to the interior

conversion of succinate to fumarate:

of the vesicle.

B.

transport H from the interior to the exterior


of the vesicle.

C.

create a diffusion potential that allowed an


efflux of K and an influx of H.

D.

11.

Addition of DCCD to cells inhibits H transport


through the mitochondrial F0 protein. Addition of

B.
C.

D.

AG' =
AG' =
AG0' =

- (0.50)(F).
+(0.50)(F).
+(0.10)(F).
-(0.10)(F).

decrease electron
synthesis.
increase electron
synthesis.
increase electron
synthesis.
decrease electron
synthesis.

A.

matrix, while electron transport occurs on the

B.

intermembrane space, while electron transport

C.

cytosol, while electron transport occurs on the

D.

cytosol, while electron transport occurs on the


plasma membrane.

inner mitochondrial membrane.

transport and decrease ATP

occurs on the inner mitochondrial membrane.

transport and increase ATP

outer membrane.

transport and decrease ATP

transport and increase ATP

Copyright by The Berkeley Review

In prokaryotic and eukaryotic cells, the electrontransport chain and oxidative phosphorylation are
coupled. Oxidative phosphorylation in prokaryotic
cells occurs in the:

2,4-DNP would be expected to:


A.

AG =

B.
C.
D.

create a diffusion potential that allowed an


influx of K and an efflux of H.

9.

A.

232

The Berkeley Review

Specializing in MCAT Preparation

Biology

Cholesterol Metabolism and Regulation

Passage III (Questions 12-17)

Passage III

Most of the cholesterol synthesized in the liver is


exported in the form of a bile salt or a cholesteryl ester.
Bile salts are stored in the gallbladder until they are
needed in the small intestine. Cholesteryl esters are
synthesized from cholesterol and a fatty acid from
coenzyme A through the action of acyl-CoA-cholesterol
acyl transferase (ACAT). High intracellular levels of
cholesterol stimulate ACAT, promoting esterification of
cholesterol for storage.

In 1972 Brown and Goldstein began to examine a


dominant human hereditary condition called familial
hypercholesterolemia (FH). Their work on the transport
of exogenous cholesterol in the bloodstream and its
receptor-mediated endocytosis and metabolism by target
tissues (Figure 1) won them the Nobel Prize in 1985.
Cholesteryl
esters

Unlike bile salts, cholesterol and cholesteryl esters are


rather insoluble in water. These lipids are sequestered in a
micelle-like particle called a low-density lipoprotein
(LDL) and transported from the liver in the bloodstream
to various target tissues. LDL receptors on the surface of
a target cell cluster into coated pits, and binding of LDL
to these receptors initiates endocytosis. The internalized
coated pit becomes an endocytotic vesicle. Several of

Cholesterol

Extracellular
space

these vesicles fuse to form an endosome, which in turn

fuses with a lysosome. Enzymes within the lysosome


hydrolyze the cholesteryl esters and the protein portion of
the LDL particle, releasing cholesterol, fatty acids, and
amino acids into the cytoplasm. The LDL receptors are
recycled and move back to the plasma membrane to form
another coated pit. High levels of intracellular cholesterol
inhibit LDL receptor synthesis.

12.

Amino
>
rn
acids ^^V^ CCT^

Based on information in the passage, what is meant


by "exogenous cholesterol"?
A.

Cholesterol in the blood that may enter the cell

B.

Cholesterol inside the cell

C.

Cholesterol sequestered inside arterial plaques

D.

Cholesterol in the nucleus of the cell

oS<*
Cholesterol

Lysosome

Chlosteryl /65;

13.

ester

Brown and Goldstein studied people with FH. The

primary defect of FH is ineffective or missing LDL


receptors. How would FH affect the metabolism of

droplet

cholesterol?

Figure 1. LDL receptor-mediated endocytosis of


I.
II.

Increased intracellular cholesterol synthesis


Decreased activity of ACAT

Cholesterol biosynthesis begins with acetyl-CoA and


occurs in four stages, primarily in the cytosol of

III.

Increased LDL cholesterol in the blood

hepatocytes. The rate-limiting step in this pathway

A.
B.
C.
D.

I only
II only
I and III only

chlosterol and its metabolism.

involves HMG-CoA reductase an enzyme that can be

allosterically inhibited by high levels of intracellular


cholesterol, unidentified cholesterol derivatives, and

I, II, and III only

hormonally regulated by glucagon (inactivation) and


insulin (activation).

Copyright by The Berkeley Review

233

The Berkeley Review

Specializing in MCAT Preparation

Biology

Cholesterol Metabolism and Regulation

Passage III

14. In Canada, public health guidelines are given in SI


units. For cholesterol (MW = 386.64), the public

health guideline is plasma total cholesterol levels


below 5 mM. What is this in mg/dL, the units for
this guideline in the U.S.?
A.
B.
C.
D.

77.2 mg/dL
193.3 mg/dL
1933.0 mg/dL
772.0 mg/dL

15. All of the following events happen after the


lysosome fuses with the endosome EXCEPT:

A.

lipases degrade cholesteryl ester to free


cholesterol.

B.

proteases degrade apolipoproteins to amino


acids.

C.
D.

16.

LDL receptor synthesis increases.


cholesteryl ester is stored in droplets in
cytoplasm.

What would be the physical results of changing to a


no-cholesterol diet?

A.
B.
C.
D.

17.

Intracellular cholesterol synthesis increases


Intracellular cholesterol synthesis decreases
Extracellular cholesterol synthesis increases
Extracellular cholesterol synthesis decreases

Mevinolin is the active ingredient in a class of


cholesterol-lowering drugs. It works as a potent
competitive inhibitor of HMG-CoA reductase and
lowers plasma cholesterol levels. In the following
Lineweaver-Burk plot, which line BEST indicates
mevinolin's action on HMG-CoA reductase?
Without
mevinolin

1/[S]
A.
B.
C.

Line A
Line B
Line C

D.

Line D

Copyright by The Berkeley Review

234

The Berkeley Review


Specializing in MCAT Preparation

Biology

Very Low-Density Lipoproteins (VLDLs)

Passage IV (Questions 18-23)

18.

Phospholipids, triacylglycerols, cholesterol, and


cholesteryl esters are hydrophobic molecules that are
essentially insoluble in water. In order to be transported in
the bloodstream, these lipids combine with carrier
molecules called apolipoproteins to form one of four
major classes of plasma lipoprotein particles. Different
combinations of lipid and apolipoproteins produce plasma
lipoproteins of varying densities and sizes (Table 1).

Particle

Density
(g/ml)

Passage IV

From the plasma samples she draws, the researcher


conducting the three experiments described in the
passage must separate the VLDL from the other

lipoproteins. What is the BEST way to accomplish


this?

A.
B.

C.

Extraction with organic solvent


Centrifugation in a density gradient
Thin-layer chromatography

D.

Extraction with 5% NaCl solution

Diameter

(nm)

High-density
lipoprotein (HDL)

1.13

10

Low-density
lipoprotein (LDL)

1.04

20

Very low-density
lipoprotein (VLDL)

0.98

50

Chylomicron

0.95

500

19.

Based on the information in Table 1, which are the

largest and MOST buoyant lipoproteins?

A.
B.
C.
D.

Table 1. Major classes of plasma lipoproteins.

High-density lipoproteins
Chylomicrons
Low-density lipoproteins
Very low-density lipoproteins

De novo lipogenesis (DNL) is the synthesis of fatty


acids from acetyl-CoA. DNL occurs in the cytosol-of the
hepatocyte with the assistance of the enzyme complex
fatty-acid synthase. After DNL, fatty acids are esterified
to glycerol. The product, triglyeride, is packaged into
20. Which lipoprotein contains dietary lipids?

VLDLs and excreted from the liver.

A researcher uses intravenous l-l3C acetate to label

A.
B.
C.
D.

the triglyceride that is synthesized de novo during the


following experiments. She then analyzes the

incorporation of 1-13C acetate using gas chromatography-

High-density lipoproteins
Low-density lipoproteins
Chylomicrons
Very low-density lipoproteins

mass spectroscopy (GC/MS).


Experiment I

Subjects fast from 8:00 pm until noon the following


day. Blood samples are taken every hour from 6:00

21. Why does the researcher use 1-,3C acetate to label


the VLDL triglycerides?

am until noon.

Experiment II

Subjects fast from 8:00 pm until 6:00 am. A glucose


drink containing 25 grams of glucose is given once
per hour from 6:00 am until noon, during which time
hourly blood samples are taken

I.

The acetate is metabolized to acetyl-CoA.

II.

8 acetyl-CoAs are used to make 1 palmitate


molecule.

HI. 1-,3C acetate contains a stable isotope that is


safe for use in humans at high doses.

IV. 1-13C acetate contains a radioactive isotope


that is safe for use in humans at high doses.

Experiment III

A.
B.
C.
D.

Subjects fast from 8:00 pm until 6:00 am. A fructose


drink containing 25 grams of fructose is given once
per hour from 6:00 am until noon, during which time
hourly blood samples are taken.

Copyright by The Berkeley Review

235

II and III only


I and IV only
I, II, and IV only
I, II, and III only

The Berkeley Review

Specializing in MCAT Preparation

Biology

Very Low-Density Lipoproteins (VLDLs)

Passage IV

22. Analysis on the GC/MS requires transesterification


of the triglycerides to form fatty acid methyl esters.
Which is the BEST reagent to use for this purpose?
A.
B.
C.
D.

Methanol + HC1
Methanol
Ethanol + HCI
Ethanol

23. Subjects participating in which experiment would

show theLEAST incorporation of 1-13C into VLDL


triglycerides?
A.
B.
C.
D.

Experiment I
Experiment II
Experiment III
Subjects in all three groups would show the

same degree of 1-13C incorporation.

Copyright by The Berkeley Review

236

The Berkeley Review


Specializing in MCAT Preparation

Biology

Calvin Cycle

Passage V (Questions 24-29)

Passage V

ATP and NADPH are generated as products of the


light reactions of photosynthesis. These two metabolites
are then used to convert CO2 and H2O into carbohydrates
during the light-independent or dark reactions of
photosynthesis. These dark reactions make up the Calvin
cycle. All of the reactions of the Calvin cycle, except
those catalyzed by enzymes Ei, E10, and E13, can be
found in animal tissue. Many of the reactions are common
to both glycolysis and the pentose phosphate pathway.

Within the chloroplasts of green plants, CO2 can be


fixed in the form of simple organic compounds. The
synthesis of these compounds takes place in a cyclic

pathway called the Calvin cycle (Figure 1), in which


specific metabolites (Table 1) are continually regenerated.

Key intermediates in the Calvin cycle, like 3-PG and


Ru-l,5-BP (Figure 2), were identified as the result of
experiments using green algae and radioactively labeled

carbondioxide (,4C02).
r\ un >
H2C-0-PO_

|/-ATP
^ADP

C=0

H-C-OH

H-C-OH

H-C-OH

I
I

+H,0

H,C-0-PO,2-

H2C-0-P032

3-PG

Ru-l,5-BP

Figure 2. Calvin cycle intermediates.

Experiment I
Step A:

A mixture of green algae is exposed to CO2 and light for


an extended period of time.
StepB:

The light source is removed, and ,4CC2 is added to the

NADPH

mixture.
G-3-P-

NADP
+ H

Step C:

Metabolites are analyzed for 14C labeling.

Figure 1. The Calvin cycle. These reactions fix atmospheric


C02 in the form of carbohydrates.

Experiment II
Step A:

A mixture of green algae is exposed to 14C02 and light


Table 1.

for an extended period of time.

Abbreviation

Compound Name

Ru-l,5-BP

Ribulose-1,5-bisphosphate
3-Phosphoglycerate
1,3-Bisphosphoglycerate
Glyceraldehyde-3-phosphate
Dihydroxyacetone phosphate
Fructose-1,6-bisphosphate
Fructose-6-phosphate
Glucose-6-phosphate
Xylulose-5-phosphate
Erythrose-4-phosphate
Sedoheptulose-1,7-bisphosphate
Sedoheptulose-7-phosphate
Ribose-5-phosphate
RibuIose-5-phosphate

3-PG

1,3-BPG
G-3-P
DHAP

F-1,6-BP
F-6-P

G-6-P
Xu-5-P

E-4-P

S-1.7-BP
S-7-P
R-5-P
Ru-5-P

StepB:

The light source is removed, and CO2 is added to the


mixture.

Step C:

Metabolites are analyzed for 14C labeling.


Experiment III
Step A:

A mixture of green algae is exposed to 14C02 in darkness


for an extended period of time.

Step B:
After all forms of CO 2 are removed from contact with the

mixture, a light source is introduced.


Step C:

Metabolites are analyzed for 14C labeling.


Copyright by The Berkeley Review

237

The Berkeley Review


Specializing in MCAT Preparation

Biology

Calvin Cycle

Passage V

reactions correspond to:

Based on the information in Experiment I, which of


the following graphs BEST represents the levels of
14CinRu-l,5-BPand3-PG?

A.

A.

24. The first five steps in the Calvin cycle are catalyzed

28.

by enzymes Ei, E2, E3, E4, and E5 (Figure 1). These


carboxylation, phosphorylation, reductive
dephosphorylation, enolization, and

2;

condensation.

B.

C.

B.

hydrolysis, phosphorylation, reductive


dephosphorylation, isomerization, and

>>

condensation.

>

carboxylation, phosphorylation, oxidative


dephosphorylation, enolization, and

t5

CB

Ru-l,5-BP

v\ /T^^
x
/v.

hydrolysis, phosphorylation, oxidative


dephosphorylation, isomerization, and

/^-
3-PG

yS*

3-PG

>

Based on the metabolic pathway in Figure 1, what is


the overall balanced reaction for the Calvin cycle?

26.

CO2 + 2ATP + 4H2O + NADPH + H+ ->


Glucose + 4Pj + 2ADP + NADP+

CO2 + ATP + H2O + NADPH + H+ ->


Glucose + Pi + ADP + NADP+

\^Ru-l,5-BP

<a

'"3
03

06

Time

6CO2 + 18ATP + 12H2O + 12NADPH + 12H+


- Glucose + 18Pi + 18ADP + 12NADP+

D.

y^yj>G

'>

+->

6CO2 + 12ATP + 6H2O + 12NADPH + 12H+


-> Glucose + 12Pi + 12ADP + 12NADP+

^t

TL>>

'>
0

C.

Ru-l,5-BP

'3

D.

X^

B.

Time

C.
"t

A.

>

Time

condensation.

25.

3-PG

0
CO

condensation.

D.

2;

29.

Ru-l,5-BP
Time

Based on the information in Experiment III, which


of the following graphs BEST represents the levels
of 14C in Ru-l,5-BP and 3-PG?

After 3 turns of the Calvin cycle, 6 molecules of


glyceraldehyde-3-phosphate are formed. These 6
molecules will be converted into:

A.

3 molecules of Ru-l,5-BP and 1 molecule of


G-3-P.

B.

3 molecules of Ru-l,5-BP and 1 molecule of

glucose.
C.

D.

1 molecule of Xu-5-P, 1 molecule of E-4-P,


and 1 molecule of S-1/7-BP.
1 molecule of F-1,6-BP and 2 molecules of

Time

Time

Time

Time

glucose.

27. If radioactively labeled carbon dioxide (,4C02) is


used as a substrate for the Calvin cycle, it will
appear in the carboxyl group of 3-phosphoglycerate.
A short time later, the label will be found in glucose6-phosphate at carbon atoms:
A.
B.

C-l and C-6.


C-l and C-3.

C.
D.

C-2 and C-5.


C-3 and C-4.

Copyright by The Berkeley Review

238

The Berkeley Review


Specializing in MCAT Preparation

Biology

Passage VI

Lactose Intolerance

Passage VI (Questions 30-36)

31.

Human beings are the only animals that are known to


drink the milk of other species and to continue to drink
milk after weaning. The majority of the adult population
of the world cannot digest lactose, the sugar in milk.
Almost all human infants have sufficient lactase activity
to break lactose down into glucose and galactose. But
with age, the activity of lactase generally declines, except

Which biochemical reaction during fermentation


changes milk so that yogurt and cheese made from it
can be consumed without complication by some
lactose-intolerant individuals?

A.

The lactic acid produced during fermentation

B.

The alcohol produced during fermentation

C.

Microorganisms produce disaccharides during

D.

Microorganisms break down disaccharides


during fermentation.

breaks down lactose.


breaks down lactose.

in some people of northern European origin and certain


groups of Africans, such as the Tutsi and the Felani.

fermentation.

The symptoms of lactose intolerance are unpleasant


but not often fatal, unless milk is the only available food.

After the ingestion of 30-50 grams of lactose as a test


dose, the lactose-intolerant person usually experiences
diarrhea, abdominal cramps, and intestinal gas. Even if
fluid milk is not tolerated, some lactose-intolerant people
may still be able to consume fermented milk products,
such as yogurt or cheese, without complications.

32.

30. Which of the following diagrams represents the


chemical structure of lactose (galactose-p(l-4)glucose)?

What is the cause of the intestinal symptoms


associated with the ingestion of fluid milk by a
lactose-intolerant person?
I.

Bacteria in the colon ferment the lactose to

produce gases.
II. Colon contents are hypertonic to surrounding
cells, and water enters the colon by osmosis.
HI. Bacteria in the colon produce irritating acids

A.
CH,OH

from the lactose.

OH

OH

A.
B.
C.

I only
I and II only
II and in only

D.

I, II, and III

B.

CH2OH
H

OH

OH

33.
C.
CH,OH

CH,OH

Lactase isolated from the mold Aspergillus niger is


available commercially for lactose-intolerant people
to add to milk products before ingestion. How does
this commercial lactase work?

A.

It is carried to the small intestine, where it

functions exactly like the missing human


lactase.

D.
CH,OH

B.

It breaks down the lactose in the food and is


inactivated in the stomach.

C.

It induces the production of lactase by the

D.

Mold is cultivated in the milk, and it digests

intestinal mucosa cells.


the lactose.

OH

Copyright by The Berkeley Review

OH

239

The Berkeley Review


Specializing in MCAT Preparation

Biology

Lactose Intolerance

Passage VI

34. After the ingestion by a test subject of a dose of


lactose to check for lactase deficiency, which of

these physiological variables should the clinician


measure?

35.

36.

I.
II.

Breath hydrogen
Blood glucose

III.

Blood lactose

A.
B.
C.

I only
I and II only
II and III only

D.

I, II, and III

Where in the human body is lactose synthesized?


A.

The thymus

B.

The small intestine

C.
D.

The mammary gland


The adrenal gland

Where is the enzyme lactase secreted during


digestion, in lactose-tolerant individuals?
A.

The mouth

B.

The stomach

C.
D.

The small intestine


The colon

Copyright by The Berkeley Review

240

The Berkeley Review


Specializing in MCAT Preparation

Biology

p-Oxidation

During the early 1950s another German chemist,


Feodor Lynen, discovered that one of the end products of
p-oxidation was not acetate, but rather acetyl coenzyme
A. Working on a theory proposed by an American
biochemist, Albert Lehninger, that fatty acids are
somehow activated before they are degraded, Lynen and
his colleagues discovered that coenzyme A (CoA-SH) is
esterified to the carboxyl group of a fatty acid in an ATPdependent enzymatic reaction.

Passage VII (Questions 37 - 43)


o

II

R-CH2-C-OH

Catalyzed by
enzymes

present in

+ ATP + CoA-SH

Cytosolic
Fatty Acid

the outer
mitochondrial
membrane

R-CH2-CSCoA

+ AMP + PR

Activation of long-chain fatty acids occurs in the


cytosol of a cell, and they are transported across the

Fatty acyl-CoA
j v" Carnitine

Carnitine

mitochondrial inner membrane and into the mitochondrial

transports
Inner

the fatty acid


across the

Membrane

matrix by a shuttle system involving L-carnitine. Shortchain fatty acids can cross the inner mitochondrial
membrane as free fatty acids. Once they are in the matrix,
they are activated. These fatty acids can be obtained in

CoA-SH

inner membrane ] V///////X Fatty acyl-Camitine E?%3


and into the
matrix of the
mitochondrion

CoA-SH.

//

Passage VII

MATRIX

their free form from the blood or released from cellular

triacylglycerols by the enzymatic action of lipase.

Carnitine^)y

Oxidation of a fatty acid occurs exclusively in the


mitochondrial matrix. This is in contrast to fatty-acid

ii

R-CH2-CH2-CH2- C S CoA

synthesis, which occurs exclusively in the cytosol of a


cell. In other words, fatty-acid catabolism and fatty-acid

Fatty acyl-CoA

anabolism occur in different compartments of the cell.

Even though fatty-acid synthesis utilizes acetyl-CoA as a


precursor molecule, the reaction mechanisms are more

complex and are not a direct reversal of P-oxidation.


HO

37. Once a fatty acyl-CoA molecule completes the four


sequential reactions in P-oxidation, it produces an

I
I
II
R-CH,-CCC S CoA

!2
O
ca.

acetyl-CoA molecule and a fatty acyl-CoA molecule


which has been shortened by two carbon atoms. The
reactions sequences of P-oxidation is BEST described

NADH

in terms of:

A. oxidation; hydrolysis; oxidation; thiolysis.


B. oxidation; reduction; dehydrogenation; hydrolysis.

R-CH,- C C C S CoA

C. dehydrogenation; hydration; oxidation; esterification.


D. oxidation; hydration; reduction; cleavage.

II

R-CH,-CSCoA

CHj-CSCoA

Fatty acyl-CoA
(2 carbons shorter)

38. Consider caprylic acid, a Cs saturated fatty acid

Acetyl-CoA

found in small amounts in butter:


o

II
CH3-CH2-CH2-CH2-CH2-CH2-CH2 -C OH

At the turn of the 20th century (circa 1904), the

Germanchemist Franz Knoop began to feed animals fatty


acids with a phenyl group (C6H5) attached to the last
carbon atom in the fatty acid chain, the omega (co) carbon.

Caprylic acid

Analysis of the urine of these animals led him to conclude


that fatty acids are degraded in two-carbon fragments by a

Complete oxidation of one molecule of this short-

process that involves oxidation at the P-carbon to give a


p-keto acid. This is followed by cleavage of the p-keto
acid to give two products: acetate and a fatty acid
derivative (which is two carbon atoms shorter than the P-

of:

keto acid). The pathway for fatty-acid degradation

C.

63 ATPs.
62 ATPs.
61 ATPs.

became known as ^-oxidation.

D.

60 ATPs.

Copyright by The Berkeley Review

chained fatty acid to CO2 and H2O gives a net yield

A.
B.

241

The Berkeley Review

Specializing in MCAT Preparation

Biology

(3-Oxidation

Passage VII

42. More energy is available from the oxidation of fatty

39. If we were to label caprylic acid with a phenyl ring at


the C-8 position, then which of the following
chemical compounds would be present in the highest

acids than from the oxidation of carbohydrates,


because:

concentration in the urine of a rabbit fed with this

I.

modified fatty acid?

II.

B.

A.
o

II

III.

c0

fV^t.
D.

c.

fatty acids are stored in an anhydrous form,


whereas carbohydrates are not.
the hydroxyl hydrogens in a carbohydrate are
released as bare protons with no electrons.
fatty acids are more reduced than
carbohydrates.

A.
B.
C.

I only
II and III only
I and III only

D.

I, II, and III

43. One of the three major biological roles for fatty acids
is that they can be stored as triacylglycerols (neutral
fats) and used as food molecules. The general
structure for a triacylglycerol is shown below. The R
group represents the hydrocarbon chain of the same
fatty acid:
o

II

40. Suppose we were to label the C-l, C-3, C-4, C-6 and
C-7 carbon atoms of caprylic acid radioactively with

H,C-0-C-R

14C, as shown below (where an * = 14C):

H-COCR

II

OH^*l^H2t*H2~t*H2"\-H2"wrl2"t*ri2 c-

II

H2COCR

OH

Caprylic acid

Triacylglycerol

If this molecule were completely metabolized, then in


which of the following molecules would the label be
located predominately?

The melting points of four common Cis fatty acids


are:

Stearic acid (+69.6 C)


Oleic acid (+13.4 C)
Linoleic acid (-5 C)
Linolenic acid (-11 C)

B.

A.

II

CH3- C S-CoA

o=c= 0
*

A decrease in the melting temperature (Tm) of these


Cis fatty acids would indicate:

C.

D.
0

II

II

CH3 - C- -

S-CoA

CH3- C S-CoA
*

41. A rabbit completely oxidizes 1 mole of caprylic acid


to CO2 and H2O. The net yield of water produced is:
A.
B.
C.
D.

greater than
greater than
greater than
greater than

an increase in fluidity.

II.
III.

a decrease in unsaturation.
a decrease in the number of van der Waals
interactions.

A.
B.

I only
I and II only
II and III only
I and III only

C.
D.

0, but less than 20 moles.


20, but less than 40 moles.
40, but less than 60 moles.
60, but less than 80 moles.

Copyright by The Berkeley Review

I.

242

The Berkeley Review


Specializing in MCAT Preparation

Biology

Glycogen Metabolism

Passage VIII (Questions 44-50)

45.

Passage VIII

Based on Figure 1, which of the

following

statements is FALSE?

Human beings store excess energy in glycogen or in


triglyceride. Glycogen in muscle and liver accounts for
about 1200 kcal, while energy stores in triglyceride are
virtually limitless. Glycogen is stored in the liver at 44
g/kg liver tissue and in the skeletal muscle at 14g/kg
muscle tissue. The human liver weighs about 1.8 kg. This
organ metabolizes excess dietary carbohydrate into
triglyceride and releases it in the form of very low-density
lipoproteins (VLDL).

A.
B.
C.
D.

Carbohydrate oxidation increases with


carbohydrate overfeeding.
Carbohydrate is used to fill glycogen stores
before de novo lipogenesis begins.
Glycogen stores cannot be saturated.
Net de novo lipogenesis (DNL) is used in part
to dispose of excess carbohydrate consumed
during Days 5-10.

Swiss researchers studied the disposal of excess

dietary glucose following a glycogen-depleting fast.


Three male subjects were fed a 10% carbohydrate, 75%
fat, and 15% protein weight-loss diet for 3 days. Their
glycogen stores were also depleted by exercise.

46.

What changes in VLDL should be seen in subjects


given a massive overfeeding of carbohydrate for one
week?

A.
B.
C.
D.

Sometime late on Day 2 of this dietary experiment, the


subjects entered a respiratory chamber, a room that
permits the researchers to sample all the air exhaled by

Increased VLDL production


Decreased VLDL production
No change in VLDL production
Complete inhibition of VLDL production

the test subjects. The subjects stayed in the chamber for


ten full days. Beginning on Day 4, they were switched to
a diet consisting of 85% carbohydrate, 3% fat, and 11%

47.

protein for one week. This diet increased in calories from


an excess of 1000 kcal to an excess of 2000 kcal. The

following diagram (Figure 1) shows daily carbohydrate


intake and its disposal during the 7 days of progressive
carbohydrate overfeeding:

1000 r

the respiratory quotient be?

Carbohydrate

intake \

The respiratory quotient is defined as the ratio of


CO2 to O2 in expired air. If glucose (C6H12O6) were
the sole fuel for these test subjects, and if it were
oxidized completely to CO2 and H2O, what would

...;-]
48.

A.

0.70

B.

0.82

C.

0.95

D.

1.00

Shown below is the structure of linked glycosyl

residues in glycogen. What are the linkages at I and


II?
CH,OH

200

Figure 1

44.

Which tissue in the human body has the largest net


supply of glycogen?
A.

Liver

B.

Kidney

A.
B.

I is a-1,6, and II is p-1,4.


lis a 1,4, and II is (3-1,6.

C.
D.

Muscle
Heart

C.
D.

I is a-1,4, and II is a-1,4.


I is a-1,6, and II is a-1,4.

Copyright by The Berkeley Review

243

The Berkeley Review

Specializing in MCAT Preparation

Biology

Glycogen Metabolism

Passage VIII

49. In an experiment with rats, a researcher labels the


glycogen stored in the liver. Animals are first fasted
and then refed with a labeled precursor to glycogen.
Which of the following tracers would NOT
effectively label the glycogen?

A.
B.

1-14C palmitate
1-,3C glucose

C.

1-14C alanine

D.

1-,3C pyruvate

50. The experiment described in the passage is an


extreme version of carbohydrate loading. Athletes
also carbo-load by eating low-carbohydrate foods
and exercising followed by a high carbohydrate feast
right before an event. What benefits might carboloading possibly have?

A.

Increases glycogen stores to boost endurance

B.

Increases fat stores to boost endurance

C.

Decreases glycogen stores to boost endurance

D.

Decreases fat stores to decrease endurance

Copyright by The Berkeley Review

244

The Berkeley Review


Specializing in MCAT Preparation

Biology

Glycolysis and 2,3-Bisphosphoglycerate

Passage IX

Glycolysis involves ten enzymatic steps, during which


glucose is converted to pyruvate. In animals, this is the
only metabolic pathway that produces ATP in the absence

Under aerobic conditions in animal cells, pyruvate


diffuses from the cytosol to the mitochondrial matrix,
where it is converted to acetyl-CoA before entering the
citric acid cycle. Under anaerobic conditions, pyruvate is
converted into lactate in animal cells, while in yeasts it is

of oxygen.

converted into ethanol and carbon dioxide.

Passage IX (Questions 51-58)

Nine of the intermediates in glycolysis are


phosphorylated to prevent them from leaving the cytosol.
This ensures that during specific reactions, phosphate
groups are donated to ADP to make ATP.

H-C=0

ADP

H-C-OH

H- C- OH

Step 1

HO-C-H
I
H- C- OH

ATP
IP

HO-C- H

c,

I
H-C-OH
I

C-OH

H- C- OH
I

CHrO-POj2-

CH2-0H

c6

Glucose

Glucose

6-Phosphate
Step 2

CHrO-PO/-

CHrOH

51.

C=0
I

HO-C- H

HO-C-H

H-C-OH

H-C-OH
I

Step 3

In prokaryotic cells, glycolysis occurs in the:

H-C=0

ADP
LATP

H- C - OH

H-C-OH

A.

mitochondrial matrix.

B.
C.
D.

cytoplasm.
lumen of the Golgi complex.
lumen of the smooth endoplasmic reticulum.

CHrO-PO,2-

CHrO-P03 2-

Fructose

Fructose

1,6-Bisphosphate

6-Phosphate
NADH

Step 4,

Pi +
NAD+

CHr0-P0j2-

Ov

s ^=

CH2-OH

O-POj2-

I
H- C- OH

C=0

52.

h-c-oh

Step 5

Step 6

acetone

CH2-0-P032-

CH2-o-P032Glyceraldehyde
3-Phosphate

Dihydroxy

Lactic acid is the end product of glycolysis in:

+ H+

A.
B.

red blood cells.


brain tissue cells.

C.

adipose tissue cells.

D.

heart and muscle tissue cells.

1,3-Bisphosphoglycerate

Phosphate

I^ADP
Step 7

53. Glycolytic enzymes can catalyze all of the following

^*-ATP
V
^

Step 8

CH2-OH

H-C-OH

CH2-0-P032-

phosphorylations.

B.

isomerizations.

C.

enolizations.

D.

ligations.

3-Phosphoglycerate

2-Phosphoglycerate

A.

H-C-O-KV -

Step 9

reactions in glycolysis EXCEPT:

0V o0

0
O

H20

ADP

V
i
2-

C- O-POj
II

CH

ATP

,N
Nc ,0

^s y^

c=o

Step 10

CH3

Phosphoenolpyruvate

Copyright by The Berkeley Review

54.

The net oxidation state of carbon in glucose is:


A.

Pyruvate

245

-1

B.

C.

+1

D.

+2

The Berkeley Review


Specializing in MCAT Preparation

Biology

Glycolysis and 2,3-Bisphosphoglycerate

55. In red blood cells, the glycolytic intermediate 1,3-

57.

Passage IX

Pyruvate-kinase deficiency is inherited as a rare


autosomal recessive trait. If one parent has the

bisphosphoglycerate (1,3-BPG) can be converted to


2,3-bisphosphoglycerate (2,3-BPG) by the action of

disease and the other parent is heterozygous for the


disease, what is the probability that one of their
children will show a deficiency in pyruvate kinase?

a mutase enzyme.
0

ov

o-po,2-

Mutase

+-

H-C-OH

H-C-OPOj'I

CH2-0-P0,2-

CH2-0-P032-

1,3-Bisphosphoglycerate

2,3-Bisphosphoglycerate

This bifunctional enzyme can also convert 2,3-BPG


to 3-phosphoglycerate and Pi, metabolites used in
glycolysis. How many net ATPs are produced
during glycolysis if this 2,3-BPG shunt is operating

58.

B.

C.
D.

4
6

0%

B.
C.
D.

25%
50%
75%

Based on the glycolytic pathway, which of the


following structures is formed after a series of
enzymatic reactions that includes phosphorylation,
dephosphorylation and reduction, and
isomerization?

in red blood cells?


A.

A.

B.

A.

CH,
1

c
I

H-C-O-PO,2-

C= 0

CH2-OH

CH,-OH

56.

Pyruvate kinase is an important regulatory enzyme


in glycolysis, and it catalyzes the conversion of PEP
to pyruvate. Which of the following oxygenhemoglobin dissociation curves (dashed line) BEST
represents an individual with a mild deficiency of

D.

C.
0.

this enzyme?

H-C- -OH
1

CH2-0-POj2

H - C-OH

CH2-0-PO,2

B.

A.
-j*

cm

CM

//>H7.4

o
t

x:

:*

5
c
o

: $

ca

#
#

CO

'

B /

*S
ca

ha

CO

pH7.4>^^T

-C

' IS
/ Q

o
a

J 0

CO

CO

0^r

^<>+*

p02

Po2
D.

c.
*N

'"

.^""

/pH 7.4

-C

pH7.4>^

x:

ha
c
o
a

%
c

2
3

ca
u.

ca

CO

-0-P032

ca

CO

_^r

Po2

Copyright by The Berkeley Review

p02

246

The Berkeley Review


Specializing in NCAT Preparation

Biology

Leucine Catabolism

Practice Passage X

59. Which of the following types of reactions is catalyzed


by Enzyme 1?

Passage X (Questions 59-65)


In animals, amino acids can undergo oxidative

degradation. Leucine, an essential ketogenic amino acid,


can be degraded to acetyl-CoA and acetoacetate as
outlined in Figure 1. Each reaction is catalyzed by a
specific enzyme and involves either an important cofactor
or coenzyme. Many of these reactions are analogous to
reactions found in other metabolic pathways.

coo
i

coo
I

H,N-C-H

o=c

CH2

CH,

C.
D.

Carboxylation
Dehydrogenation

B.

A.

H3C- CH

Hydration
Transamination

60. In human beings, the ultimate fate of the a-amino


group in leucine is BEST represented by which of the
following metabolic structures?

A.
B.

H,C- CH

Enzyme 1

'

CH,

CH,

0
II

H-N-H
I

H2N-C-NH2

a-Ketoisocaproate

Leucine

Urea

Ammonium ion

NADH + H, CoA-SH

Enzyme 2

D.

C.

NAD, C02

coo

S-CoA

S-CoA

FADH2
4 " FAD
fa

o = c:

CH,

II

Enzyme 3

H,C-C

HjN-C- H

0=C

HN* V

c JL -c=0

H3C- CH

CH2

CH2
1
COO

CH

CH,

Uric acid

Isovaleryl-CoA

P-Methylcrotonyl-CoA

Glutamate

^ /-HC03
Enzyme 4 f (Biotin- Dependent)

Cofactor A

f^

61. The carboxylation reaction catalyzed by Enzyme 4 is


a biotin-dependent carboxylase, which also requires

S-CoA

H20

cofactor A. Cofactor A is:

0=C
I

C-H

Enzyme 5

H.C-C

Hc_ c_

OH

CH,

CH,

CH2

A.

H20.

B.

ATP.

C.

NADH.

D.

FADH2

COO

COO

(3-HydroxyP-methylglutaryl-CoA

|$-Methylglutaconyl-CoA

62. In the reaction catalyzed by Enzyme 4, the


Enzyme 6

bicarbonate ion (HCC3e) is converted into


Compound C. Compound C is:

OOC- CH2 - C- CH3

H3C- C- S-CoA

Acetoacetate

D.

co2.

B.

Acetyl-CoA

Figure 1

Copyright by The Berkeley Review

C.

H2CO3
NH3.
H20.

A.

247

The Berkeley Review


Specializing in MCAT Preparation

Biology

Leucine Catabolism

Practice Passage X

63. The enzyme holocarboxylase synthetase appears to

be responsible for attaching biotin to at least four


different apocarboxylase enzymes in mammals.
Biotin is involved in specific carboxylation reactions.
In the absence of the holoenzyme, which substrate
begins to accumulate most rapidly?
A.
B.

Acetyl-CoA
cc-Ketoisocaproate

C.
D.

p-Methylcrotonyl-CoA
p-Methylglutaconyl-CoA

64. Which of the following types of reactions is catalyzed


by Enzyme 6?
A.

Thiolysis

B.
C.

Aldol condensation
Reverse aldol condensation

D.

Hydrolysis

65. If leucine were radioactively labeled with 14C at the


a-carbon atom, we would expect to find that label

entering cellular respiration at which of the following


stages?
A.
B.
C.
D.

Glycolysis
Krebs cycle
Electron transport
Oxidative phosphorylation

Copyright by The Berkeley Review

248

The Berkeley Review


Specializing in NCAT Preparation

Biology

Trehalose Experiment

Passage XI

Passage XI (Questions 66 - 72)

The following graph indicates the results of a pilot


study performed with four test subjects using the

Trehalose is a naturally occurring sugar with the


following structure:

experimental protocol just described:

CH-,011

50 n

Legend

HO

40
HO

20 g trehalose
H 30 g trehalose

Figure 1. Trehalose

0 g trehalose

40 g trehalose
50 g trehalose

| 30

Trehalase is an enzyme present in the microvilli on the


surface of cells in the duodenum, the first twelve inches

of the small intestine. This enzyme hydrolyzes trehalose,


a sugar found in certain beetle larvae and in mushrooms,
into two monosaccharides. Researcher are studying
human tolerance for different amounts of the sugar,
because it is of interest to people in the food processing
industry as a possible food preservative.

20-

1(1

The following experiment was devised to learn more


about individual tolerances for trehalose. Each subject
was given five incremental doses of trehalose, or fewer if
they felt gastrointestinal distress and had to stop the
experiment. Gastrointestinal distress was defined for this
experiment as diarrhea, bloating, or gas pains. Breath
samples were collected in leak-proof syringes and
injected directly into a gas chromatograph for analysis of
hydrogen concentration.

Subject I

Subject 2

Subject 3

Subject 4

Figure 2. Effects of trehalose ingestion on concentration


of hydrogen in the breath.

Part 1: Subject drinks 200 mL of distilled water.


Breath hydrogen is collected and analyzed

66.

What process causes the increase in breath hydrogen


when a trehalose dose is consumed?

after 2 hours.

A.
Part 2:

Subject drinks 200 ml of distilled water


containing 20 g of trehalose. Breath
hydrogen is collected and analyzed after 2

intestinal tract.
B.

Bacteria in the colon ferment the trehalose and

C.

The interaction between bicarbonate from the

D.

pancreas and trehalose leads to the production


of hydrogen.
Hydrogen is produced when trehalose is

produce hydrogen.

hours.

Part 3:

Subject drinks 200 ml of distilled water


containing 30 g of trehalose. Breath
hydrogen is collected and analyzed after 2
hours.

Part 4:

denatured in the stomach's acid.

Subject drinks 200 ml of distilled water


containing 40 g of trehalose. Breath
hydrogen is collected and analyzed after 2

67.

hours.

Part 5:

Subject drinks 200 ml of distilled water


containing 50 g of trehalose. Breath
hydrogen is collected and analyzed after 2

Which subject exhibited a dose-response effect


during all five parts of the experiment?
A.
B.
C.
D.

hours.

Copyright by The Berkeley Review

Digestion of the trehalose in the small intestine


leads to the production of hydrogen gas in the

249

Subject 1
Subject 2
Subject 3
Subject 4

The Berkeley Review


Specializing in MCAT Preparation

Biology

72. Which subject exhibited a threshold effect during the


experiment? (Exhibiting a threshold effect means

68. Which two monosaccharides does the enzyme

trehalase produce when it hydrolyzes trehalose?


A.

Glucose and galactose

B.
C.

Glucose and fructose


Galactose and fructose

D.

Glucose and glucose

Passage XI

Trehalose Experiment

that after a certain dose, the subject's response

changes dramatically.)
A.
B.
C.

Subject 1
Subject 2
Subject 3

D.

None of the subjects experienced a threshold


effect.

69. What results should be expected, if lactose were


substituted for trehalose in this experiment, using
the same subjects?
A.

Since lactose and trehalose are both


disaccharides, the same pattern of intolerance
sould be expected.

B.

Lactose would be tolerated at higher doses


than trehalose, since it is a commoner

componentof the diet.


C.

Lactose would be tolerated at lower doses than


trehalose, since it is a less common component
of the diet.

D.

One cannot determine the answer from the

information presented in this passage.

70. What can you conclude about Subject 1 from the


results shown in Figure 2?

A.

Subject 1 has a high tolerance for trehalose in


the diet.

B.

Subject 1 has a low tolerance for trehalose in


the diet.

C.

Subject 1 regularly consumes trehalosecontaining foods.

D.

Subject 1 almost never consumes trehalosecontaining foods.

71. The basic function of the gas chromatograph, like


the one used in the experiment described in the

passage to analyze the concentration of hydrogen in


the breath samples, is to separate gaseous

compounds in a mixture on the basis of:


A.

their densities.

B.

their molecular interactions within a column.

C.

the masses of each isotope in the compound.

D.

their solubilities in organic, solvents.

Copyright by The Berkeley Review

250

The Berkeley Review

Specializing in MCAT Preparation

Biology

Fuel Oxidation during Exercise

Passage XII (Questions 73-79)

Passage XII

75. Which of the following compounds is an important


fuel source for resting muscle?

Exercise is a time of dynamic change in fuel oxidation

for skeletal muscles. During rest, skeletal muscle respires


aerobically. Skeletal muscles respire anaerobically for the
first 45-90 seconds of moderately intense exercise. As

A.

exercise progresses, the muscles switch to aerobic


respiration.

Oleate

B.

(J-hydroxy butyricacid

C.

Lactic acid

D.

Glycogen

Table 1 outlines more specifically the fuels used

during different phases of exercise. The following


abbreviations are used: FFA (free fatty acids), GMG

(glucosefrom muscle glycogen), BG (blood glucose), Gly

76. What is the carrier molecule for blood glucose?

(glycogenosis), and Gng (gluconeogenesis).

Phaset
Rest

FFA

GMC

++ +

+++

5-10

BG

A.

Albumin

Liver Glucose Output

B.
C.

Glucose-binding protein
Hemoglobin

from Gly from Gng

D.

There is no carrier.

75%

25%

10-40

++

++

40-90

++

+++

by 240

++ +

++

i
55%

77.

45%

During exercise, which of these hormones


involved in blood glucose maintenance?

is

t Time in Minutes

Table 1

73.

Insulin

B.
C.
D.

Glucagon
Glycogen phosphorylase
Oxytocin

Why is the first interval of moderately intense


exercise a period of anaerobic respiration?
A.

B.
C.
D.

Oxygen supply is transiently lower than


oxygen demand.
The Krebs cycle enzymes required for aerobic
respiration must be activated.
Fatty acids must be metabolized anaerobically.
Glycolysis cannot keep pace with the need for

78. Which fuel provides the MOST energy per gram?

C.

Fatty acids
Blood glucose
Glycogen

D.

Amino acids

A.

B.

energy.

74.

A.

How does the blood glucose level change during


79.

moderate exercise?

Which of the following statements is TRUE of


exercise that continues for more than four hours?

A.
B.

C.

A.

Blood glucose rises rapidly during exercise,


and then plateaus at a higher level.
Blood glucose falls rapidly during exercise,
and then plateaus at a lower level.

B.

decrease.

C.

Blood glucose is relatively unchanged by


exercise.

D.

D.

Blood glucose rise and falls in a cyclic fashion


as exercise progresses.

Copyright by The Berkeley Review

Muscle glycogen
is
repleted by
gluconeogenesis.
The rate of gluconeogenesis continues to

The availability of free fatty acids from


adipose tissue begins to decline.
The rate of glycogenolysis continues to
decrease.

251

The Berkeley Review


Specializing in NCAT Preparation

Biology

Passage XIII

Urea Cycle

PassageXIII (Questions 80-87)

Glutamate, once it passes into the mitochondrial


matrix from the cytosol, can lose its amino nitrogen as the

Every amino acid that is catabolized by the human


body must have its a-amino group removed from its

ammonium ion in a reaction involving water and NAD

carbon skeleton. Amino groups that are not reused in the

indicated by the reverse reaction in Figure2.

or NADP, or it can be converted to a-ketoglutarate, as

synthesis of nitrogenous compounds are converted tourea


The urea cycle involves four enzymatic steps, each of
which catalyzes a unique intermediary compound
necessary for the synthesis of an amino acid(Figure 3).

in the urea cycle (Figure 1):

2 ADP

+ Pj

Carbomyl

phosphate

2 ATP

coo
1

HjN C-H

HCO3 + NH4

CH2

CH,

0=C

<>^

glutarate"^>

T2
Citrulline

NH3

NH

Ornithine

NH,

H,

Citrulline

Arginino-

Arginine

succinate

-AMP

Oxalo-

/C**acetate ~"S

Figure 3

succinate

Malate

E4

Citrate

Krebs*

|E6 cycle

Urea h--Fumarate -^-Fumarate

Hutarate

Succinate

SuccinyICoA

cycle

80. Urea is primarily synthesized in the:

Arginine

I.
Urea

~ NH
I
H2N=C

NH
1

OOC C- C- COO

E8

r.ATP

Argino- +PPi
Ornithine

CH,

Glutamate y

Aspartate

E3

a-keto-

CH2

CH,

H,N = C

<>

CH2
I

Ornithine

CH2

NH

NH,

COO

Aspartate

,
HjN -C-H

CH,

I
CH,

CH,

coo

H,N C-H

CH,

Citrulline

H,N - C - H

CH,

COO

H2

kidney.

B.

blood.
liver.
muscle

C.

Mitochondrial
matrix

Cytosol

A.

D.

Figure 1

Amino acids can have their a-amino group removed

81. According to the passage, what is the overall

by transaminases, enzymes that interconvert a-amino and

balanced reaction for the urea cycle?

a-keto acids during transamination reactions. The ketoacid acceptorof manyamino groupsis a-ketoglutarate.

Urea + 2ADP + AMP + PPi + 2Pi

^ co"

coo

-.

c=o

I
CH,

H,NCH

^:

=^

CH,

oc-Keto-

C. NH4 + HC03e + 3ATP + H20-

COO

Amino
acid

Urea + 2ADP + AMP + PPi + 2Pj

c=o

CH2

COO

I
CH2

B. 2NH4 + HCQ36 + 3ATP + Aspartate + H2O->

coo

H,NCH

coo

A. 2NH4 + HC03e + 3ATP + H20->

Glutamate

Urea + Fumarate + 2ADP + AMP + PPi + 2Pi

Keto

acid

D. NH4 + HCQ3e + 3ATP + Aspartate + H2O->

glutarate

Urea + Fumarate + 2ADP + AMP + PPi + 2Pj

Figure 2

Copyright by The Berkeley Review

252

The Berkeley Review

Specializing in MCAT Preparation

Biology
82.

Urea Cycle

The two nitrogen atoms in urea can be traced back to

86.

NH4 and:
A.

citrulline.

B.
C.
D.

arginosuccinate.
arginine.
glutamate.

Passage XIII

Argininosuccinicaciduria (ASA) is a rare autosomal


recessive disease caused by the absence of
argininosuccinase (E4 in Figure 1). Its primary
symptom is the accumulation of argininosuccinate in
the blood. A man whose mother had ASA and

whose father did not carry the ASA gene marries a


woman with the disorder. What is the probability
that their child will NOT express the trait?

83.

The carbon atom in carbamoyl phosphate comes


directly from the:
A.
B.
C.
D.

84.

glycolytic pathway.
citric acid cycle.
electron-transport chain.
oxidation of fatty acids.

A.
B.

0.25
0.50

C.
D.

0.75
1.00

87. Hyperammonemia can be caused by a deficiency in


arginase (E5 in Figure 1). This disorder can be
controlled by the restriction of protein in the diet and
by diversion therapy (use of an alternative metabolic
pathway to reduce the toxic concentrations of a
particular metabolite). Administration of sodium
benzoate leads to the formation of hippurate, a
compound easily excreted in the urine.

The ammonium ion is quite toxic to brain tissue.


Which of the following amino acids is able to react
with the ammonium ion and convert it into a non

toxic compound?

O^ jo0 Na ^ ^
COOo
^C^
I

B.

A.

h2

H,N-C-H

CH2

CH,

SH

COO

Glycine

The rationale for the use of sodium benzoate as a

therapeutic

procedure

for

hyperammonemic

individuals is that it:

COO

COO

Hippurate

Sodium benzoate

D.

C.

H,N-C- H

HjN-C- H

CH2

CH,

j-v
H-N^N-H

A.

reduces the concentration of ammonia in the

B.

reduces the concentration of ammonia in the

blood by increasing the synthesis of glycine.

=\

CH2

blood by increasing the synthesis of aspartate.

COO

85.

II

Nccor

COO

HjN-C-H

^c'

H3NC H

coo
I

0^

Which of the following types of reactions can result


in the formation of ammonia by the removal of the
a-amino group from an amino acid?
A.
B.
C.

Oxidative deamination
Transamidation
Reductive deamination

D.

Dehydrogenation

Copyright by The Berkeley Review

253

C.

increases the concentration of urea in the blood


so it can be eliminated in the urine.

D.

increases the concentration of Krebs cycle


intermediates in order to produce more
aspartate for the urea cycle.

The Berkeley Review


Specializing in MCAT Preparation

Biology

Gluconeogenesis and the Cori Cycle


90.

Passage XIV (Questions 88-94)

Under normal physiological conditions, the brain uses


about 120 grams of glucose each day as its only fuel
source, compared to the roughly 160 grams of glucose
that the body uses each day. Since the body stores less
than a day's supply of glucose, this carbohydrate must be
synthesized from noncarbohydrate precursor molecules
during times of fasting or intense physical exertion.

Which of these statements would NOT describe the


metabolic fate of lactate under anaerobic conditions?

I.

Lactate is converted to pyruvate.

II.
III.

Lactate is converted to alanine.


Lactate is converted to lactose.

A.

C.

I only
II only
I and II only

D.

I, II, and III

B.

The synthesis of new glucose occurs during a process

Passage XIV

called gluconeogenesis. It takes place primarily in the


liver and, to a lesser extent, in the cortex of the kidney.
91.

Glycerol, lactate, pyruvate, and many amino acids (e.g.,


alanine) are all important precursors of glucose.

What metabolic condition could activate the reverse

Cori cycle, in which there is glucose output from


muscle tissue and lactate output from the liver?

Lactate is an end product of anaerobic glycolysis and

is produced primarily in erythrocytes and skeletal muscle.

A.

Pregnancy activates the reverse Cori cycle.

When lactate is released into the bloodstream, it travels to

B.
C.

Exercise activates the reverse Cori cycle.


Fasting activates the reverse Cori cycle.

D.

There is no metabolic condition that could

the liver where it is taken up by the hepatic portal vein


and reoxidized to pyruvate in the cytosol of hepatocytes.

activate the reverse Cori cycle.

The cycle of reactions involving the production of


glucose from lactate in liver tissue and the production of
lactate from glucose in muscle tissue is called the Cori

92. If lactate dehydrogenase were inhibited during


anaerobic conditions, what metabolic effect would

cycle (Figure 1).

this have?

=>Glucose

Glucose

4ATP>|
2 GTP

Pyruvate

A.

of NAD.

^2 ATP

B.

Pyruvate

C.

D.
v

In muscle tissue

93.

Which of these statements correctly describes


alanine and its biochemical derivation?

Figure 1. The Cori cycle.

A.

89.

Glycolysis would continue, using FAD as the


electron transporter.

= Lactate

Lactate <=

88.

Glycolysis would continue, but without ATP


production.
Glycolysis would eventually stop due to a lack
of ATP.

In liver tissue

Glycolysis would eventually stop due to a lack

Alanine is a nonessential amino acid that is

produced by transamination from pyruvate.

Under what conditions would muscle release the

MOST lactate per minute?

B.

Alanine is an essential amino acid that is

A.
B.
C.
D.

C.

Alanine is an essential amino acid that is

produced by transamination from lactate.

During sleep
During studying
During vigorous exercise

produced by reduction from lactate.


D.

produced by reduction from pyruvate.

Following a light meal

94.

Which of the following enzymes is involved in the


first step of gluconeogenesis in the liver?
A.
B.
C.
D.

When glucose produced by gluconeogenesis leaves


the liver, it exits through which blood vessel?
A.
B.
C.
D.

Pyruvate dehydrogenase
Pyruvate carboxylase
Lactate dehydrogenase
Lactate carboxykinase

Copyright by The Berkeley Review

Alanine is a nonessential amino acid that is

254

The
The
The
The

hepatic
hepatic
hepatic
hepatic

artery
vein
portal vein
portal artery

The Berkeley Review


Specializing in MCAT Preparation

Biology

Starch Blockers

Passage XV (Questions 95-100)

Passage XV

95. One volunteer for the study was not allowed to


participate, because she produced no increase in

In the early 1980s, a popular weight reduction fad was

breath hydrogen following the administration of

the "starch blocker" tablet. Starch blockers were

lactulose in the first phase of the study. Why would


this disqualifyher from the participation?

developed and marketed to inhibit the intestinal

carbohydrate-hydrolyzing enzyme, oc-amylase. In theory,


a dieter could consume unlimited amounts of complex
carbohydrates, take a pill containing the starch blocker,
and be protected from weight gain, since the starches
would not be digested. Instead, they would pass through

A.
B.

the small intestine mainly undigested.

C.

Further claims about starch blockers focused on the

D.

absorption of glucose and the lowered insulin response to


ingested glucose after they were used. Several researchers
planned the following experiment to examine the effects
of starch blockers. Breath hydrogen content was measured
to determine fermentation of undigested starch by colonic
bacteria. Eight men and women participated in each of the
three phases of the study:

She had too many colonic bacteria that ferment


nondigestible carbohydrate.
She did not have enough of the colonic bacteria
that ferment nondigestible carbohydrate.
She had too many colonic bacteria that ferment
digestible carbohydrate.
She did not have enough of the colonic bacteria
that ferment digestible carbohydrate.

96. Figure 1 indicates the breath hydrogen response to


all three phases of the study:
Lactulose

Phase I: Lactulose Test

15 grams of the nondigestible carbohydrate lactulose


were administered, and breath hydrogen concentration
was measured over 4 hours.

60

Phase II: Test meal with placebo

Subjects ate a test meal of regular foods, containing 50


grams of starch. Subjects took a placebo capsule
containing calcium carbonate with the meal. Breath

180

240

300

Figure 1. Breath H2 response as a function of time.

Which of these statements could be supported by the


data in Figure 1?

hydrogen concentration was measured over 4 hours.

I.

The starch blocker acted in a manner that was

similar to the placebo.


Lactulose promoted a rise in breath hydrogen
beginning about an hour after it was given.
III. Intestinal oc-amylase was not inhibited by the

Phase III: test meal with starch blocker

II.

Subjects ate a test meal of regular foods, containing 50


grams of starch. Subjects took a test capsule containing
500 milligrams of phaseolamin, an oc-amylase inhibitor,
with the meal. Breath hydrogen concentration was

starch blocker.

A.
B.

measured over 4 hours.

97.

C.

II and ffl only


I, II, and III

Intestinal oc-amylase is synthesized in the:

C.
D.

255

I only
I and III only

D.

A.
B.

Copyright by The Berkeley Review

120

Time (minutes)

small intestine,

gall bladder,
large intestine,
pancreas.

The Berkeley Review


Specializing in MCAT Preparation

Biology
98.

Passage XV

Starch Blockers

The following graphs show the glucose and insulin

100. How does an actual inhibitor of a-amylase affect the

digestion ofthe simple sugars glucose and fructose?

responses oftest subjects to Phases II and III ofthe


study.

A. Thedigestion of simple sugars is unaffected by


the starch blocker.

B. The digestion of simple sugars is affected in

Placebo

the same way as starch digestion.

Starch blocker
150

100

IS

C. The digestion of simple sugars is completely


blocked by an inhibitor of a-amylase.

D. Simple sugars are destroyed by secondary


actions of the a-amylase inhibitor.

50

30

60

90

120 150 180

Time (minutes)

Figure 2. Glucose response.

140 p

Placebo

30

Starch blocker

60

90

120 150

Time (minutes)

Figure 3. Insulin response.

These data indicate that when phaseolamin was

given to thesesubjects, it produced:


A.
B.
C.
D.

99.

an increased glucoseresponse and a decreased


insulin response compared to the placebo test.
a decreased glucose response and an increased
insulin responsecomparedto the placebo test.
no change in glucose or insulin responses
compared to the placebo test.
decreases in both the glucose and insulin
responsescompared to the placebo test.

The test meal contained 50 grams of starch in foods


the subjects ordinarily consumed. Other components
of the test meal were protein, simple sugars, and fat.
Which of the following foods is mostly carbohydrate
in the form of starch?

A.

Turkey

B.
C.

Rice
Butter

D.

Orange juice

Copyright by The Berkeley Review

256

The Berkeley Review

Specializing in MCAT Preparation

Biology

Metabolic Pathways

Passage 1(1 - 5)

1.

Section VIII Answers

Glycolysis and Electron Transport Chain

Bis correct. The faster the growth rate of an organism, the shorter its doubling time. The guiding principle in

problems such as this one is that growth rate is directly proportional to the amount ofATP available to each cell per
unit time. In the presence ofplenty ofO2, prokaryotic organisms (such as E. coli) can make 38 NTPs per molecule
of glucose oxidized, whereas eukaryotic organisms (such as yeasts) can make either 36 or 38 ATPs. For the

purposes ofthis question, we were to assume 36 NTPs per molecule ofglucose oxidized (under aerobic conditions).

When oxygen is removed, the yeast cells switch over to anaerobic metabolism, and the net yield of NTP per
molecule of glucose falls tojust 2. This is called anaerobicfermentation.

Depriving yeast cultures of O2 causes each cell to produce 18 times less NTP per molecule of glucose consumed

(from 36/2 = 18). Pasteur found that yeast cells grew 6 times less rapidly under these conditions, implying that they
were forming NTPs at a rate only 6 times less rapidly than before. This means that they must be consuming glucose
more rapidly than before (as was noted in the question). In particular, they must be consuming glucose at a rate
approximately 3 times faster (from 18/X = 6, or X = 18/6 = 3). The correct choice is B.

2.

C is correct. In the presence of oligomycin, the F0 transmembrane protein is blocked. The proton gradient formed
by electron transport and proton-pumping can no longer be relieved. The result is that the cells revert to anaerobic

metabolism and just do glycolysis, synthesizing only 2 net NTPs per molecule of glucose consumed. The metabolic
products will be ethanol (CH3CH2OH) and carbon dioxide (CO2). The overall effect is the same as if you had
removed the cells from oxygen, or had added an electron-blocking agent such as cyanide. The correct choice is C.
3.

C is correct. In this case, oxidative phosphorylation is still prevented by the binding of oligomycin to the F0
transmembrane protein; but the presence of 2,4-DNP means that no proton gradient is formed, so electron transport
can and will continue. The end products of glucose metabolism will be CO2 and H2O. If anything, the rate of
electron transport will be a little faster, since no NTPs are being formed during electron transport (i.e., there is no
resistance to electron transport). But overall, the growth rate of the culture would be expected to decline. A net of 4
NTPs are synthesized per molecule of glucose metabolized. Why? 2,4-DNP uncouples all of the NTPs that are
normally formed by oxidative phosphorylation, but leaves unaffected any NTPs formed by substrate-level
phosphorylations. Those substrate-level phosphorylations include the 2 net ATPs formed in glycolysis and the 2 net
GTPs formed during the Krebs cycle. Therefore, in the presence of excess 2,4-DNP, the amount of NTP formed per
glucose molecule when metabolized under aerobic conditions can be expected to decline from 36 to 4. The correct
choice is C.

4.

B is correct. During normal glycolysis, one of the high-energy phosphate compounds, which serves as a source of
phosphorylating energy for the synthesis of ATP, is 1,3-bisphosphoglycerate.

oe
1

H -

11

ATP

f. -

C- OH

ADP

v j
^

^1

H,C-0- PO,2

H-C-OH
I

Phosphoglycerate

H2C-0 - PO,2'

kinase

1,3-Bisphosphoglycerate

Step 7

3-Phosphoglyccrate

In the presence of excess arsenate ions, the synthesis of ATP in Step 7 does not occur. In Step 6, the compound 1arseno-3-phosphoglycerate is hydrolyzed to inorganic arsenate and 3-phosphoglycerate, the intermediate we would
see at the end of Step 7.
The overall result is that glycolysis appears to proceed normally, but there is no net formation of ATP. Two ATPs

are invested in Step 1 of glycolysis; but in the presence of excess arsenate, only two ATPs can be synthesized (in
Step 10 of glycolysis when phosphoenolpyruvate is converted to pyruvate). Therefore, there will be nonet gain in
ATP and no conservation of chemical energy. All of the free energy difference between glucose and pyruvate will
be lost as heat. The cells must stop growing, since they are no longer able to make any ATP under these
circumstances. The correct choice is B.

Copyright by The Berkeley Review

257

The Berkeley Review


Specializing in MCAT Preparation

Biology
5.

Metabolic Pathways

Section VIII Answers

C is correct. Arsenate competes with phosphate for the active site of the dehydrogenase enzyme at Step 6 in

glycolysis. Note that arsenate is an analog of phosphate. If we were to flood the system with phosphate, then
phosphate would outcompete the arsenate, and the (majority of) cells would continue to thrive. If arsenate were a
noncompetitive inhibitor, then: (1) it would not resemble phosphate, and (2) no matter how much phosphate we
added to the system, the inhibition would not be overcome.

Arsenate can also be considered an uncoupling agent, because it uncouples the phosphorylation of ADP to ATP in

Step 7but allows glycolysis to proceed. The action ofarsenate is analogous to the action of2,4-DNP in the electrontransport chain, in that they both uncouple a phosphorylation event. The correct choice is C.
Electron-Transport Chain

Passage II (6 - 11)

6.

A is correct. It is important to know the basics of metabolism. Pyruvate is the end product of glycolysis, which

occurs in the cytosol. Pyruvate can pass from the cytosol to the mitochondrial matrix through a pyruvate-H
symport. Once in the matrix, it is oxidized to acetyl-CoA. In this process hydrogens and electrons are passed to

NAD, and NADH + H is formed. In Figure 1of the passage, we see that NADH + H enter the electron-transport
chain at the level of complex I (also called the NADH-Q reductase complex). The correct choice is A.
7.

B is correct. As shown in Figure 1 of the passage, hydrogen ions are vectored across the inner mitochondrial
membrane from the matrix to the intermembrane space. This not only establishes a difference in the [H] between
the matrix and intermembrane space, but it also establishes a charge difference. Note the net positive charge on the
surface of the inner membrane facing the intermembrane space and the net negative charge on the surface of the
inner membrane facing the matrix. This electrochemical gradient allows the hydrogen ions to come back into the
matrix through the F0F[ATPase. In the process, ATPwill be synthesized from ADP and Pi.

Choice A says that electrons are passed from NADH and FADH2 to O2. This is true, but it does not answer the
question. In choices C and D, ADP is indeed transported into the matrix, and it can occur by way of either an
antiport or a symport. Even though ADP enters the matrix, it does not account for the mechanism behind ATP
synthesis. The correct choice is B.

8.

C is correct First, we need to draw a picture, so that we can visualize what is happening. In Figure A below, we see

a phospholipid vesicle with an F0 protein. Inside the vesicle is an elevated concentration of K(


influx

F0 Protein

Valinomycin

Phospholipid
Bilayer

efflux

Vesicle

Vesicle

Figure A

Figure B

Based on information in the passage, we know that the F0 protein facilitates the specific transport of H through the
membrane. If the F0 protein is not in the membrane, H cannot be transported across. If the F0 protein is in the
membrane, H can be transported across if there is some type of gradient. In the case of the inner membrane of the
mitochondrion, H is vectored into the intermembrane space. This allows for a chemical and electrical separation of
H across the inner membrane. In the case of the inner mitochondrial membrane, the F0 protein allows H to return
to the matrix down its chemical and electrical gradient.

In the case of these synthetic vesicles, the situation is pretty much the same. In order for H to enter the vesicle,
there needs to be a gradient. Within the vesicles is a high concentration of K. Once the antibiotic valinomycin (a
cyclic peptide) is added to the solution, it diffuses through the membrane and into the vesicle's interior, binds a K
ion, and then transports that ion to the exterior of the vesicle (Figure B). Since K is leaving the vesicle, it is
Copyright by The Berkeley Review

258

The Berkeley Review


Specializing in MCAT Preparation

BlOlOgy

Metabolic Pathways

Section VDI Answers

referred to as efflux. Valinomycin can transport up to about 104 K ions/second through the membrane. This
decreases the [Ke] inside the vesicle, which in turn means there is less positive charge at the interior. The F0 protein

now allows the H to pass to the interior of the vesicle. Since H is entering the vesicle, it is referred to as influx.
The correct choice is C.

9.

C is correct. DCCD is inhibiting the passage of H through the F0 portion of the F0Fi ATPase protein. This is
telling us two things: First, since H cannot get back into the matrix, these ions are accumulating in the
intermembrane space. Eventually, there will be such a high [H] in the intermembrane space that H will no longer
be able to be vectored from the matrix to the intermembrane space. In other words, NADH and FADH2 will not be

able to drop off their electrons and hydrogens. The electrons will not be able to be passed down the electrontransport chain. Second, since H cannot pass thorough the F0 pore, ATP cannot be synthesized. The net result of
DCCD addition is elimination of electron transport and ATP synthesis.

2,4-DNP is an uncoupling agent and has the ability to pick up a H in the intermembrane space and transport it
across the inner mitochondrial membrane. This transport of H through the membrane bypasses the F0FiATPase

system. This reduces the [H] in the intermembrane space. In turn, this allows more H to be pumped from the
matrix to the intermembrane space. When this happens electrons are dropped off at the electron transport chain and
the rate of electron transport increases. However, since DCCD is blocking passage of H through the F0FiATPase,
ATP cannot be synthesized. Even if DCCD were not present, we would still see a decrease in ATP synthesis because
of the uncoupling action of 2,4-DNP. The correct choice is C.

10.

B is correct In the question we are given the equation that relates the standard free energy (AG0') to the standard
reduction potential (E0'). We are told that F is the Faraday constant and that n is the number of electrons involved in
the transfer. We do not need the value of the Faraday constant. Even if we did, we could just consider the value as a
constant (i.e., unity) to simplify calculations.

AGO^-OiXFXAEo')
We find the value for the number of electrons in the passage (see second paragraph) and in the table in the question.
FADH2 can pass 2 electrons to the electron-transport chain. Our equation now looks like the following:

AG0, = - (n)(F)(AEo1) = - (2)(F)(AE0')


In the table in the question, we also find the values for E0'. Note that the redox pairs are written as reduction halfreactions.

Redox Pair

EQ (volts)

Fumarate + 2He + 2ee * Succinate

FAD + 2He + 2ee FADH2

0.03

- 0.22

The question now becomes one of arriving at the correct value for AE0'. The question it states that AE0' is the change
in standard reduction potential between the oxidized and reduced species. What is the oxidized species and what is
the reduced species? We need to consider the reaction given in the question.

FAD FADH'

LX Fumarate

Succinate ^^

In this reaction, succinate is the electron donor. Succinate gives up its electrons to FAD, the electron acceptor. The
electron donor is called the reductant, while the electron acceptor is called the oxidant. Succinate is oxidized to
fumarate, while FAD is reduced to FADH2. We can expand on our equation as shown below:

AG0' =-(n)(F)(AEo') =-(2)(F)(AE0') =- (2)(F)(E0' (acceptor) - E0' (donor))

Copyright by The Berkeley Review

259

The Berkeley Review


Specializing in MCAT Preparation

Biology

Metabolic Pathways

Section VIII Answers

If we substitute the appropriate values, we get:

AG0' = - (2)(F)(AE01) = - (2)(F)(E0' (acceptor) - E0' (donor)) = - (2)(F)[(- 0.22) - (0.03)] = - (2)(F)(- 0.25)
We can clean this up a bit:

AG0' = + (0.50)(F)

This question tests our knowledge on oxidation-reduction reactions and the manipulation of signs. The correct
choice is B.

11.

D is correct. It is important to know the differences between prokaryotic and eukaryotic cells. Mitochondria are not
found in prokaryotic cells. They are found only in eukaryotic cells. Therefore, we can immediately eliminate choices
A and B.

All prokaryotic cells have a plasma membrane that surrounds their cytosol. If the prokaryotic cell is a Gram-negative
cell, it also has an outer membrane (outside the peptidoglycan layer). A Gram-positive bacterium has only the

plasma membrane surrounded by a peptidoglycan layer. Since a prokaryotic cell requires energy (ATP) to survive, it
will want that ATP delivered to reactions in its cytosol as quickly as possible. Oxidative phosphorylation occurs in

the cytosol. Since oxidative phosphorylation and electron transport are coupled, it would mean that electron
transport occurs on the plasma membrane. And this is what is observed in both Gram-positive and Gram-negative
bacteria. The correct choice is D.

Passage III (12- 17)

Cholesterol Metabolism and Regulation

12.

A is correct Exogenous means "from the outside." Its antonym is endogenous. For the cell, exogenous means
"extracellular." This rules out answers containing cholesterol inside the cell. So choices B and D are wrong. Choice
C is wrong, too. Although cholesterol in arterial plaques is exogenous to the cell, it is not free to move around. The
word sequestered means "trapped" or "stored away." The correct choice is A.

13.

C is correct From the diagram, we see that LDL particles deliver cholesterol to cells. They must interact with the
LDL receptor. If the LDL receptor is damaged or missing, LDL particles and their cholesterol remain in the blood.
This makes statement III correct. If LDL can't enter, then the negative feedback from exogenous cholesterol is not
available. Therefore, the cell alters itself to increase intracellular cholesterol. This means levels of HMG-CoA

reductase are increased. Statement I is correct. Also, ACAT activity increases to hydrolyze stored cholesteryl esters
into free cholesterol. Statement II is wrong. The correct choice is C.
14.

B is correct This is a simple change in units. (5 mmol/liter) x (386.64 mg/mmol) x (1 liter/10 dL) = 193.3 mg/dL.
The other choices are incorrect due to either multiplying or dividing error by a factor of 10 (choice C) or flipping the
units upside down (choices A and D). The correct choice is B.

15.

C is correct. Three of the answers are true, but the answer we want is the false one. We are told in the question that
the lysosome fuses with the LDL receptor-LDL particle complex. Hydrolytic enzymes degrade both cholesteryl
esters to free cholesterol and the apolipoprotein into amino acids. Free cholesterol is released. This is a signal, and
the cell decreases LDL-receptor synthesis. The free cholesterol can be reesterified into cholesteryl ester and stored in
the cytoplasm. Choices A, B, and D are all true. The false answer is choice C. The correct choice is C.

16.

A is correct First of all, there is no extracellular cholesterol synthesis. Choices C and D are therefore incorrect. If a

diet provided no cholesterol, then the cells would need to synthesize more cholesterol to meet their requirements.
Choice B is wrong. The correct choice is A.
17.

B is correct. Mevinolin is a competitive inhibitor of HMG-CoA reductase. On the Lineweaver-Burk plot, a


competitive inhibitor has the same Vmax as the enzyme acting alone, but the Km is higher. Only Line B crosses the
y-axis at the same point as the line for HMG-CoA reductase. This means choice B is correct. Choice C indicates a
noncompetitive inhibitor. Choices A and D are just tricks. The correct choice is B.

Passage IV (18-23)
18.

Very Low-Density Lipoproteins (VLDLs)

B is correct The names of the lipoproteins comes from their activity during centrifugation. Extraction with an
organic solvent would separate fat-soluble components from water-soluble components, but this is not what you
want. Choice A is thus incorrect. TLC requires organic solvents as well, so the lipids would all mix together. That

Copyright by The Berkeley Review

260

The Berkeley Review


Specializing in MCAT Preparation

Biology

Metabolic Pathways

Section VIII Answers

means choice C is also incorrect. A salt solution would not help: Choice D is incorrect. Take advantage of the table
showing you the different densities. This is how they should be separated, which is best accomplished by
centrifugation. The correct choice is B.

19.

B is correct Read this from the table. Chylomicrons have the largest diameters and the lowest densities. Therefore,
they are relatively large and more buoyant than the other lipoproteins. The correct choice is B.

20.

C is correct. This is not in the passage. We can eliminate VLDL, because the passage tells us those lipids are made
in the liver. This eliminates choice D. If we remember how the lipoproteins transform, we know that VLDL

becomes LDL, so choice B is eliminated, too. The rest comes down to memory. HDL does not contain dietary lipid;
chylomicrons do. The correct choice is C.

21.

D is correct. Acetate is linked to CoA by the enzyme thiokinase, forming acetyl CoA. Statement I is correct.
Palmitate contains 16 carbons, which come from 8 acetyl-CoAs, so statement II is also correct. I3C is a stable

isotope of carbon. Statement III is correct, because stable isotopes are safer for use with human beings than
radioactive isotopes at high doses. Only statement IV is incorrect, so choice D is the best answer. The correct
choice is D.

22.

A is correct. Eliminate choices C and D, since we are trying to make methyl esters. Transesterification requires a
nucleophile in an acidic environment. The correct choice is A.

23.

A is correct. The group that is fasting is not doing much DNL, because that is a pathway that is most activated when
excess carbohydrate needs to be converted to fat. The groups receiving carbohydrate do more DNL than the fasting
group. The correct choice is A.

Passage V (24 - 29)


24.

Calvin Cycle

B is correct In all four answer choices, we find that the steps in the Calvin cycle catalyzed by E2 (phosphorylation)
and E5 (condensation) are the same. Therefore, we do not need to concern ourselves with them, only with enzymes
Ei, E5, and E4. Let's consider these one at a time.

Enzyme Ei is ribulose-l,5-bisphosphate carboxylase. It is also called rubisco. In the reaction, we see that CO2 and
H2O are involved. The reactant Ru-l,5-BP is first carboxylated, which means that the reaction is a carboxylation
reaction. However, after the carboxylation event, the transient six-carbon intermediate is immediately hydrolyzed to
two molecules of 3-PG. Thus, this reaction is also a hydrolytic reaction. This tells us that the reaction catalyzed by
Ei corresponds to either a carboxylation or (once the CO2 has been added) a hydrolysis.
Enzyme E5 is involved in a dephosphorylation reaction, but is it a reductive or oxidative reaction? The coenzyme

here is NADPH (+ He). This is the reduced form of the coenzyme. It reacts with 1,3-BPG, which is the oxidized
form of the three-carbon compound. NADPH is converted to NADP. This is the oxidized form of the coenzyme.
1,3-BPG is converted to 3-PG. In the process inorganic phosphate (Pi) has been lost (i.e., a dephosphorylation) to the
medium. 3-PG is the reduced form of the three-carbon compound. This sequence constitutes a redox reaction. As
written, it is therefore a reductive dephosphorylation. We can easily answer this by considering which form of the
coenzyme is reduced and which is oxidized.
Oxidized form of the

coenzyme (no H-atoms)


Reduced form of the

coenzyme (has H-atoms)


NADP^
NADPH

Lost a phosphate
(dephosphorylation)

+ Pi

+ H+
0
Oxidized

00

H_C_0H

Reduced

H2C O PO32
1,3-BPG

3-PG

At this point, we know that enzyme E3 is involved in a reductive dephosphorylation. This allows us to eliminate
choices C and D. Once we can determine what enzyme E4 does, we will have the answer.
Copyright by The Berkeley Review

261

The Berkeley Review


Specializing in MCAT Preparation

Biology

Metabolic Pathways

Section vm Answers

Enzyme E4 should look familiar to you (think of glycolysis). It is an isomerase enzyme that makes possible the
interconversion between G-3-P and DHAP. Isomerases simply rearrange the carbon and hydrogen atoms on a

molecule. This is exactly whatwe see below. Notice that there has beenno loss of atoms.
ch2-oh
c= o
1

H2C- O- POj2"

%c
_

h - c- oh

E4

DHAP

H2C- 0- PO32
G-3-P

An enolase enzyme promotes the reversible removal of water from a molecule. We do not see the loss of water(or
the addition of water) between G-3-P and DHAP. The correct choice is B.

25.

B is correct. The function of the Calvin cycle is to convert CO2 into carbohydrates. As shown in Figure 1, the

carbohydrate that is produced is glucose, and glucose is a six-carbon sugar(i.e., C6H12O6). Glucose is an important
molecule to remember. If we end up with a six-carbon sugar, we must start with six molecules of CO2. We can
eliminate choices A and D.

The difference between choices B and C involves the number of ATP molecules and the number of water molecules
on the reactant's side. Let's consider the ATP molecules to see if we can deduce the correct answer.

In Figure 1, there are only two places where ATP is used. If we react a molecule of CO2 with the five-carbon
compound ribulose-l,5-bisphosphate, a six-carbon transient intermediate is formed, which is rapidly hydrolyzed to
yield two molecules of 3-phosphoglycerate. Since six molecules of CO2 react with six molecules of ribulose-1,5bisphosphate, the result will be twelve moleculesof 3-phosphoglycerate. The next step involves the conversion of 3phosphoglycerate into 1,3-bisphosphoglycerate. Twelve molecules of 1,3-bisphosphoglycerateare required to make
twelve ATPs. That takes care of the first part of the cycle that generates ATPs. The second part of the cycle that
generates ATPs is in the conversion of ribulose-5-phosphate to ribulose-l,5-bisphosphate. Since six molecules of
ribulose-l,5-bisphosphate are needed to react with six molecules of CO2, then six ATPs are also required to make
the conversion from six molecules of ribulose-5-phosphate to six molecules of ribulose-l,5-bisphosphate. The net
requirement for ATP in a balanced reaction for the Calvin cycle is 18. Without going any further, we can eliminate
choice C. The correct choice is B.

26.

A is correct This question involves carbon balancing. Glyceraldehyde-3-phosphate is based on the molecule
glycerol, which is a three-carbon compound. At some point, glycerol was oxidized to the aldehyde form
(glyceraldehyde) and then phosphorylated, giving glyceraldehyde-3-phosphate. The point is that the molecule is a
three-carbon compound. We have 6 of these molecules for a total of 18 carbons. Note that the Calvin cycle accounts
for all of the carbon atomsnot one is lost through decarboxylation.
In choice B, we have 3 molecules of Ru-l,5-BP and 1 molecule of glucose. Glucose is a six-carbon compound, so
this leaves 12 carbons to account for. How many carbons does Ru-l,5-BP contain? Look at the name. We are told
that the molecule contains 2 phosphate groups, one on the C-l carbon and the other on the C-5 carbon. We could
conclude from this that Ru-l,5-BP contains 5 carbon atoms. We could also arrive at this same idea based on the

name" ribulose" (Ru). In the Calvin cycle, we saw that one place ribulose can come from is ribose. Ribose is a fivecarbon sugar that is found in numerous compounds, including nucleic acids (DNA and RNA). If we have 3
molecules of Ru-l,5-BP, it means we have 15 carbon atoms. Since we need only 12 carbon atoms, there are 3

carbon atoms to account for. Eliminate choice B. We can follow the same procedure for choice C. Xu-l,5-BP
contains 5 carbon atoms, E-4-P has 4 carbon atoms, and S-1,7-BP has 7 carbon atoms. This is a total of 16 carbon
atoms. In this case, we have 2 carbon atoms that are not accounted for. Eliminate choice C.

Choice D is interesting. Glucose contains 6 carbon atoms. Since there are 2 glucose molecules, we have 12 carbon
atoms. F-1,6-BP is on the pathway to glucose. It also has 6 carbon atoms. The total between these three molecules is
18 carbon atoms, which is how many we have with 6 molecules of G-3-P. At first glance, this seems like a possible
answer.

How do we distinguish between choices D and A? To remove glucose as our product carbohydrate, we must recycle
the Calvin cycle intermediates in order to regenerate 6 molecules of Ru-l,5-BP. If we do not, there will be no Ru1,5-BP to condense with the CO2 that is hanging out with rubisco (Ei). This is exactly what will happen if we

Copyright by The Berkeley Review

262

The Berkeley Review


Specializing in MCAT Preparation

BlOlOgy

Metabolic Pathways

Section vm Answers

convert the 6 molecules of G-3-P into 2 molecules of glucose and 1 molecule of F-1,6-BP. However, if we convert
the 6 molecules of G-3-P into 3 molecules of Ru-l,5-BP and 1 molecule of G-3-P, then we can put that 1 molecule
of G-3-P in a conceptual holding tank. The 3 molecules of Ru-l,5,-BP can combine with 3 molecules of CO2. After

the transient intermediate splits, there are again 6 molecules of G-3-P. This is what we started within the question. If
these 6 molecules of G-3-P are converted into 3 molecules of Ru-l,5-BP and 1 molecule of G-3-P (which we also
place in a holding tank), then there are enough Ru-l,5-BP intermediates to continue the Calvin cycle. The two G-3-P
intermediates waiting in the holding tank can be combined to form a product carbohydrate (like glucose). The
correct choice is A.

27.

D is correct. The structure of 3-PG is given in the passage. The question states that the labeled 14C is at the
carboxyl carbon (indicated by the dot () in the structures below). This is also the C-l carbon atom. 3-PG is
eventually converted to G-3-P. Since there is no rearrangement of that C-l carbon atom, the label remains in the
same place. Figure 1 indicates that G-3-P and DHAP combine to form F-1.6-BP. Since DHAP and G-3-P are in
equilibrium with one another through an isomerase enzyme (E4), the label also appears at the C-1 carbon in DHAP.
Again, there has been no rearrangement of carbon atoms (because carbon-carbon bonds have been broken).

oNx>0
^CI

os h

e
n
E2
E3

H-C-OH

*~

X1

*-

H-C-OH

P
t4

CHrOH
1
^

H2C- O- PO,2'

H2C- O- P032-

3-PG

G-3-P

C=0
'

H2C- 0 - POj2"
DHAP

G-3-P and DHAP combine in an aldol condensation to form F-1.6-BP. This is achieved by the covalent union of the
two C-l carbons from both metabolites. The six-carbon sugar is formed, and the label appears at the two central
carbon atoms, which are the C-3 and C-4 carbons.

h2c- o- po32-

^ c

C=0

H-C-OH

HO-C-H

E6

'

H-C-OH

^-

E7

^-

HO-C-H
'

H-C-OH

H-C-OH

H-C-OH

H2C- 0 - POj2'

H2C- O- PO,2"

F-1.6-BP

G-6-P

F-1.6-BP is converted to F-6-P and then to G-6-P with no rearrangement of carbon atoms. The labels do not change.
The correct choice is D.

28.

D is correct. From the passage, we know that ATP and NADPH are synthesized during the light reactions and that
these metabolites are used during the (light-independent) dark reactions. When light is shining on the algae, ATP
and NADPHare synthesized; when there is no light, ATP and NADPH are not synthesized.

In StepA of Experiment I, green algae are exposed to light and CO2 for an extended period of time. A lot of ATP
and NADPH are synthesized. In Step B, the light source is removed, and 14C02 is added. At this point, the cells are
in darkness, so ATP and NADPH cannot be synthesized. This means that the supply of these metabolites to the
Calvin cycle begins to decrease. The reactions that rely on these metabolites are:
NADP+

ATP

ADP

CH2

+H20

Ru_5_P V./, RU-1,5-BP -^

ADP
.

ATP

NADPH +Pi
a

+H+

- 3-PG V X, 1,3-BPG V . S + G-3-P


E2

^13

If the supply of ATP decreases, less Ru-5-P is converted to Ru-l,5-BP. Similarly, less 3-PG is converted to 1,3BPG, and less 1,3-BPG is converted to G-3-P. As we start our addition of 14C02, that label is incorporated into the
Ru-l,5-BP that is still present and is converted to 3-PG. But 3-PG is not as readily converted to 1,3-BPG. Therefore,

Copyright byTheBerkeley Review

263

The Berkeley Review


Specializing in MCAT Preparation

Biology

Metabolic Pathways

Section VIII Answers

the 14C label accumulates in 3-PG and the level of radioactivity of that metabolite begins to increase. This is exactly
what we see in the graph shown in the passage.

Some of this labeled 3-PG is still converted to labeled 1,3-BPG, which in turn is still converted to labeled G-3-P.
This is because there is still some ATP and NADPH around, but their concentrations are ever-decreasing. The label

in G-3-P will eventually find its way to Ru-5-P, and some of this metabolite will be converted into labeled Ru-1,5BP. The levels of radioactivity in this metabolite will be quite low, because not enough of it is being synthesized.
The correct choice is D.

29.

Bis correct. In Step AofExperiment III, the algae are exposed the algae to 14COz in darkness. The labeled l4C is
distributed (as we have previously discussed) . At the end of Experiment I, 3-PG had a high level of radioactivity
and Ru-l,5-BP had a low level of radioactivity.

In Step B, all forms of CO2 are removed from the environment immediately surrounding the algae, and then the
lights are turned on. The light reactions begin to synthesize ATP and NADPH. The reactions in the Calvin cycle that
require these metabolites can once again operate at an increased rate. The labeled 3-PG is converted to the labeled
1,3-BPG, and so on. Eventually, the labeled Ru-5-P will be converted to the labeled Ru-l,5-BP. However, since
there is no CO2, Ru-l,5-BP cannot be converted to 3-PG. Therefore, Ru-l,5-BP begins to accumulate and shows a

high level of radioactivity, while 3-PG begins to disappear and shows a low level of radioactivity. The correct
choice is B.

Passage VI (30 - 36)


30.

Lactose Intolerance

C is correct. This question tests our recognition of the chemical structures of common sugars. Choice A is maltose.
Choice B is sucrose. Choice C is correct, lactose. Choice D is cellobiose. The correct choice is C.

31.

D is correct Bacteria are the most common organisms used in the fermentation of milk to produce yogurt.
However, cheese may be cultivated by the activity of both bacteria and molds. Alcohol is not produced in large
amounts by these particular fermentations (yeast is used for alcoholic fermentation). So, choice B is incorrect. Lactic

acid gives yogurt its sour taste, but it does not break down lactose. Choice A is incorrect. Disaccharides are sugar
dimers. Microorganisms break down disaccharides such as lactose during fermentation. This makes the fermented
product more tolerable to those lacking lactase. Choice C is incorrect. The correct choice is D.
32.

D is correct. Even if one has forgotten most of the digestive physiology that one learned, it is still possible to reason
this out. Lactose is not broken down nor is it absorbed in the small intestine, if lactase is missing. Therefore, the
lactose passes intact into the colon, where there are lots of bacteria. They rapidly ferment the lactose, producing gas
and acidic compounds. The gas production is mentioned in the passage as a symptom, so statement I is correct.
Working from there, the bacterial production of excess acids irritates the colonic epithelium, and the bowel contents
are passed through faster. Statement III is also correct. This produces diarrhea, as mentioned in the passage. Finally,
both the unabsorbed lactose and the acids increase the osmolality of the colon contents, so that more water remains
in the feces. This also increases diarrhea. Statement II is correct. The correct choice is D.

33.

B is correct. The enzyme is produced by (isolated from) a mold, but only the enzyme is commercially
availablenot the mold. This means that choice D is incorrect. Exogenous lactase does not induce the production of

anything in the intestinal mucosa, so choice C is also incorrect. Enzymes are broken down in the stomach, so lactase
cannot pass to the small intestine. Choice A is incorrect. The lactase is added to the food before ingestion. It breaks
down lactose in the food on the plate, in the mouth, and a bit in the stomach, before it is inactivated by stomach
acids. The correct choice is B.

34.

B is correct This question addresses your test-taking skills, as well as your ability to extract answers from the
passage. If the lactose is digested to glucose and galactose, the blood glucose would rise. This makes statement II
correct. There is no mechanism in the digestive tract for the uptake of lactose itself, so the clinician would see no
increase in blood lactose. Statement III is incorrect. Thus, eliminate choices C and D as incorrect answers. Finally,

we are left with statement I. Even if statement I is true, we can see at this point that choice B is a better answer,
since statement II is also correct. It does not say this in the passage, but if the test subject is lactose-intolerant, then

the gas produced by the bacteria in their colon would increase and should be detectable in their breath after lactose
ingestion. Hydrogen is a common gas to test for this assay. The correct choice is B.
35.

C is correct. This is a give-away question. Lactose is a sugar found only in milk, which is synthesized only in the
mammary gland. The correct choice is C.

Copyright by The Berkeley Review

264

The Berkeley Review


Specializing in MCAT Preparation

Biology
36.

Metabolic Pathways

C is correct. Lactase is released from the brush border membrane of the duodenum, the first twelve inches of the
small intestine. This is the only location in the body where lactase is secreted. The correct choice is C.

Passage VII (37- 43)


37.

Section VIII Answers

B-Oxidation

C is correct In the first reaction of p-oxidation, we use the oxidized form of FAD to remove two hydrogens from
the fatty acyl-CoA molecule. The more hydrogens a carbon has on it, the more reduced that carbon atom is. The

fewer hydrogen atoms the carbon atom has, the more oxidized it is. Therefore, the second molecule (enoyl-CoA) in
the P-oxidation pathway is more oxidized than the fatty acyl-CoA molecule. This first reaction could be thought of
as an oxidation reaction. Note that if the reaction is an oxidation reaction, then the oxidized form of FAD is reduced

to FADHt. Also, if we remove hydrogen atoms from a molecule, then we can call that type of reaction a
dehydrogenation reaction. What this means is that we can call this first reaction either an oxidation reaction or a
dehydrogenation reaction. So far, this supports all four answers.

The second reaction involves water (H2O). Note that we add the water across the double bond. Instead of cleaving
the molecule with water (a hydrolysis reaction), we are instead hydrating that double bond. This type of reaction is
called a hydration reaction, and from it we get the hydroxyacyl-CoA derivative. Choices A and B are eliminated.

In the third reaction, we use the oxidized form of the NAD coenzyme to remove the hydrogen atoms associated
with the (3-carbon of the hydroxyacyl-CoA derivative. If we lose hydrogen atoms, the molecule becomes oxidized.
This is therefore another oxidation reaction. Since we are losing hydrogens, it is also called a dehydrogenation
reaction. This reasoning eliminates choice D.

What about the last step? We use coenzyme A (i.e., CoA-SH) to cleave the P-ketoacyl-CoA molecule at the Im
position. CoA-SH contains a thiol (SH) group. We cleave the molecule, which is like lysing the molecule (breaking
it apart). The reaction is indeed a cleavage, but more properly it can be called a thiolysis reaction. It is not a
hydrolysis reaction, because we do not use water to cleave that bond. However, in the answer for choice C, thiolysis
or cleavage is not mentioned. These are one and the same thing, and they do occur. The molecule breaks apart. That
much is obvious. But what else is happening at this step? We are adding a molecule of CoA. When CoA reacts with
the carboxyl group of the fatty acid, which has been shortened by two carbon atoms, there is an esterification
reaction (see the second paragraph of the passage). The correct choice is C.
38.

C is correct. Activation of the Cs fatty acid with CoA will cost 1 ATP. We thereby generate the fatty acyl-CoA
molecule, plus an AMP and pyrophosphate (PPi). The driving force behind this activation is the hydrolysis of PPi to

2 Pi, which is equivalent to the energy contained in a high-energy phosphate bond (or another ATP equivalent).
Hence, activation of the fatty acid costs us 2 ATP equivalents.
The activated fatty acyl-CoA proceeds through 4 rounds of P-oxidation and is converted into 4 acetyl-CoA
molecules. Each acetyl-CoA then enters the Krebs cycle. From the Krebs cycle, we get 1 GTP, 1 FADH2, and 3
NADH molecules. The NADHs drop off their electrons and hydrogens at the electron-transport chain and produce 3
ATPs per NADH. The FADH2 molecule does the same but produces only 2 ATPs per FADH2. The GTP can be
converted to ATP in a separate reaction. Therefore, we find that 1 acetyl-CoA molecule is the potentially equivalent
of 12 ATPs. Since we have 4 acetyl-CoA molecules, we can in theory syhthesize 4 x 12 = 48 ATPs.

Complete oxidation of the C8 fatty acid yields 3 FADH2 and 3 NADH molecules from the P-oxidation pathway
itself. Note that cleavage of the last 4-carbon compound gives 2 molecules of acetyl-CoA. Both of these acetyl-CoA
molecules enter the Krebs cycle, but neither one undergoes P-oxidation again. Thus, from P-oxidation we find that
the 3 FADH2 molecules produce 3x2 = 6 ATPs, and the 3 NADH molecules produce 3x3 = 9 ATPs. From these
two reduced coenzymes, we get a total of 15 potential ATPs.
Let's complete our ATP calculation. We add all the ATPs we can generate from acetyl-CoA, FADH2, and NADH,
which is 48 + 15 = 63 ATPs. However, we invested 2 ATP equivalents to activate our fatty acid. Therefore, we
produce a net total of 63 - 2 = 61 ATPs for the complete oxidation of the Cs saturated fatty acid. The correct choice
isC.

39.

B is correct. When Franz Knoop first ran a similar experiment in Germany in about 1904, he attached a phenyl ring
to the omega (co, or the last) position of various fatty acids and then fed those fatty acids to dogs. If the fatty acid fed
to the animals had an even number of carbon atoms, he discovered large quantities of phenylacetic in their urine
(choice B).

Copyright by The Berkeley Review

265

The Berkeley Review


Specializing in MCAT Preparation

Biology

Metabolic Pathways

Section Vni Answers

II

c-o

Note that caprylic acid is aneven-chained, saturated fatty acid. There would besome phenylbutanoic acid (choice D)
in the urine, but this could still be metabolized to phenylacetic acid. If the fatty acid fed to the animals had an odd
number of carbon atoms, their urine would have contained benzoic acid (choice A). The urine would also have

contained some phenylpropanoic acid(choice C), but thiscompound couldstill be metabolized to benzoic acid. The
correct choice is B.

40.

A is correct. If we degrade caprylic acid through p-oxidation, we get four units of acetyl-CoA, all having labels.
Two of those residues have the label at the C-l carbon, as in choice B. One residue has label at both the C-l and C-2

carbons, as in choice D. One residue has label just at the,C-2 carbon, as in choice C. Based on this alone, you might

be tempted to pick choice B, because that species would predominate. But, what about the carbon dioxide? AcetylCoA is completely oxidized to CO2 and H2O in the Krebs cycle, which means that all the carbon atoms of a fatty
acid can potentially be lost as CO2. Since there are five labeledcarbons in caprylic acid, there will be five molecules
of 14C02 being produced by the Krebs cycle. Thismakes choice A the bestanswer. The correct choiceis A.
41.

A is correct The molecular formula of caprylic acid can be written as CgHi602. We are told that this compound is
completely oxidized to CO2 and H2O,a reaction we can write as follows (remembering to balance it):
C8H1602 + 1102 1

> 8C02 + 8H20

O
The 8 moles of water shown in this equation are produced at the end of the electron-transport chain when oxygen
(O2) combines with the electrons and the protons. Then more water is produced when ADP combines with Pj to
yield ATP. If the complete oxidation of one molecule of caprylic acid has a net yield of 61 molecules of ATP, then
maybe the answer we want is 61 molecules of water:

61 ADP + 61 Pj + 61 H+ 1

> 61 ATP + 61 H20

O
But the overall equation for the complete oxidation of one molecule of caprylic acid is given by:

C8Hi602 + 11 02 + 61 ADP + 61 Pj + 61 H+ 1

> 8 C02 + 8 H20 + 61 ATP + 61 H20

&

Looking at this equation you might be inclined to choose 69 moles of H2O (which would lead you to select choice D
as the best answer). But be careful. To regenerate the ADP and Pithat is needed to resynthesize the ATP, water is
needed to hydrolyze that phosphoanhydride bond between the beta and gamma phosphates of ATP. What this means
is that 61 of those 69 moles of water are continuously being recycled, so that new ATP molecules can be made. The
only water that is produced in a net quantity is the 8 moles from the complete oxidation of caprylic acid at the level
of the electron-transport chain. The correct choice is A.
42.

D is correct Fatty acids are rather nonpolar; and because they are nonpolar, they can be stored in an anhydrous state
(without water). The storage form of carbohydrates in animal tissue is glycogen. Recall that glycogen is composed
of glucose residues. Glucose is a very polar molecule. It has many hydroxyl groups associated with it. Glucose and
glycogen are both stored in a hydrated form. It turns out that on a gram-for-gram basis anhydrous fatty acids when
metabolized yield approximately six times more energy than hydrated glycogen.

When molecules like fatty acids or carbohydrates are oxidized, energy is abstracted in the form of protons (He) and
electrons. Recall that these protons and electrons are dropped off at the electron-transport chain from reduced

coenzymes like FADH2 and NADH (+ He). In other words, if a carbon atom has more hydrogensattached to it then
it has more electrons and protons to give to these reduced coenzymes, which in turn will have more to give to the
electron-transport chain. Note that the methylene carbons (-CH2-) of the saturated fatty acids have two hydrogen
atoms per carbon, whereas the majority of carbon atoms in a carbohydrate (like glucose) have just one hydrogen
atom attached to each carbon atom. This tells us that the fatty acid is more reduced or, if you wish, the carbohydrate
is more oxidized.

Copyright by The Berkeley Review

266

The Berkeley Review


Specializing in MCAT Preparation

Biology

Metabolic Pathways

Section VIII Answers

What about the hydroxyl hydrogens of a carbohydrate? If the hydroxyl oxygen were to lose a hydrogen, it would do
so as a bare proton. The electrons stay with the oxygen atom of the functional group and give it a negative charge.
Putting all of this together, it is clear that choice D is the best answer. The correct choice is D.

43.

D is correct. A decrease in the melting temperature of a fatty acid means that less heat is needed to disrupt the
interaction between any two fatty acid constituents of that type when they are next to one another in a triacylglycerol
molecule. In other words, the hydrocarbon chains of the fatty acids are not well ordered or well packed together. If
this is the case, there are a lot fewer van der Waals interactions between the individual hydrocarbon chains. Fewer
van der Waals interactions means that the carbon atoms in the hydrocarbon chain are farther apart. This could be
because of cis double bonds in the hydrocarbon chain itself. Cis double bonds cause kinks to form in those
hydrocarbon chains and tend to keep them apart from one another. The more kinks, the less chance there is for
stronger van der Waals interactions. Adding kinks through cis bonds means that the carbon atoms participating in
those cis bonds are unsaturated (i.e., they have fewer hydrogen atoms). What about unsaturation with trans double
bonds? Fatty acid hydrocarbon chains can have trans double bonds, but they tend to allow the hydrocarbon chains to
come closer to one another than cis double bonds. The van der Waals interactions are greater and the fluidity
decreases (compared to the cis conformation). The correct choice is D.

Passage VIII (44 - 50)

Glycogen Metabolism

44.

C is correct. There is much more skeletal muscle than liver tissue in a anyone's body. A 70-kg man has about 40%
of his weight in the form of skeletal muscle and a liver weighing 1.8 kg. This means the muscle contains 0.40 x 70 x
14 = 400 g glucose, while the liver has 1.8 x 44 = 80 g. The correct choice is C.

45.

C is correct. We can see that carbohydrate (CHO) oxidation increases from Day 3 to Day 4. The diet changes on
Day 4 to high CHO intake. Choice A is true. We can see from the chart that the glycogen stores are beginning to fill
before DNL kicks in. Choice B is correct. From the diagram, we see that net DNL occurs on Days 5-10. Choice D is
correct. Since we are looking for the false choice, choose C. We can see from Figure 1 in the passage that glycogen
stores are eventually saturated. The correct choice is C.

46.

A is correct. Net DNL is occurring during the overfeeding of CHO. Since the liver packages the fat that it makes
into VLDLs, we would see an increase in that class of compounds in the blood of test subjects on this high-CHO
diet. The correct choice is A.

47.

D is correct. In the question we are given the fact that glucose has a molecular formula of C6H12O6, and that it is
completely oxidized to CO2 and H2O. If a biological molecule like glucose is being oxidized, molecular oxygen
(O2) is being used. This allows us to set up a rough equation that needs to be balanced:
C6H1206 + 02

-*- COt + H-,0

Unbalanced equation

C6Hi206 + 6 02

*-

Balanced equation

6 CCb + 6 1-bO

In the balanced equation we see that there are 6 CO2 molecules produced for every 6 O2 molecules consumed.
Therefore, the ratio of CO2 to O2 is 6 to 6, or 1.00. The correct choice is D.

48.

49.

D is correct. If the hydroxyl (OH) group at the anomeric carbon is down (i.e., below the plane of the ring), the
position is alpha (a). Note that in the diagram every anomeric carbon has the OH group below the plane of the ring.
Therefore, the linkages for I and II are both in the a-position. This allows us to eliminate choices A and B. Linkage
II begins at the C-l carbon and ends at the C-4 carbon. This is an a-1,4 linkage. Linkage I begins at the C-l carbon
and ends at the C-6 carbon. This is an a-1,6 linkage. We can eliminate choice C. The correct choice is D.

A is correct. Both the stable and radioactive forms of carbon can be used as tracers, so ignore the l3C and l4C
tricks. Glucose can proceed to glycogen by the direct pathway. Alanine and pyruvate proceed to glycogen by
gluconeogenesis. Palmitate is a fatty acid, and it is not made into glucose. The correct choice is A.

50.

A is correct. From Figure 1, we can see that the first few days of excess CHO feeding mainly increase the glycogen
stores and not the fat stores. This eliminates choices B and D. The focus is on glycogen. Depletion of glycogen by
exercise and a low-CHO diet, followed by a high-CHO diet might increase glycogen stores and thus increase the
endurance of the athlete. The correct choice is A.

Copyright by The Berkeley Review

267

The Berkeley Review


Specializing in MCAT Preparation

Biology

Metabolic Pathways

Passage IX (51 - 58)


51.

Section VIII Answers

Glycolysis and 2,3-Bisphosphoglycerate

B is correct. The important differences between eukaryotic and prokaryotic cells cannot be emphasized enough.
Eukaryotic cells have mitochondria, so they have a mitochondrial matrix. They also have cytoplasm, in which the
mitochondria and Golgi complex reside. And they have an endoplasmic reticulum, extending from the cell's nucleus.
The only item here common to the prokaryotic cell is the cytoplasm. Prokaryotic cells do not have mitochondria,
Golgi, or an endoplasmic reticulum (because they do not have a nucleus). Therefore, in prokaryotes, glycolysis
occurs in the cytoplasm. The correct choice is B.

52.

A is correct. Besides knowing the general differences between prokaryotes and eukaryotes, it is also good to know
some specific fact about each type of cell. Lactic acid is the end product of glycolysis under anaerobic conditions.
Under aerobic conditions, the end product of glycolysis is pyruvate (which can then enter the citric acid cycle). If
our cells were deprived of oxygen, then lactic acid would be the end product in each one of the four choices.
However, there is only one best answer here, not four. Therefore, we must assume that the question asks about cells
that are utilizing oxygen. If oxygen is present, then glucose is completely oxidized to C02 and H20 (by way of
glycolysis, the citric acid cycle, electron transport, and oxidative phosphorylation). But if lactate is being produced
in a cell and is not being oxidized to C02 and H2O in the presence of O2, then either the citric acid cycle, electron
transport, and oxidative phosphorylation are not being utilized or those pathways are missing from the cell. It turns
out that mature red blood cells lack not only a nucleus, but they lack membrane-bound organelles like mitochondria
as well. The correct choice is A.

53.

D is correct. All we need to do is look at each step in the glycolytic pathway and see what general type of reaction is
catalyzed. There are two phosphorylation reactions, one at Step 7 and one at Step 10. Eliminate choice A. There are
isomerization reactions at Step 2 and at Step 5. The only enolization reaction occurs at Step 9. Only ligation
reactions are left. Ligation reactions are catalyzed by enzymes called ligases. The word ligate means "to bind." In a
ligation reaction two molecules are joined together, at the expense of hydrolyzing ATP to ADP and P\. The correct
choice is D.

54.

B is correct. If we assign hydrogen a value of +1 and oxygen a value of -2, the oxidation state of carbon must be
whatever it takes to achieve charge neutrality. Consider the C-l (aldehyde) carbon. There is no change in
electronegativity between the carbon atoms C-l and C-2. However, the C-l carbon bears a hydrogen (+1) and an
oxygen (-2). This adds up to -1. Therefore, in order to be neutral, the C-l carbon must have an oxidation state of+1.

Consider the C-2 carbon. The hydrogen directly attached to the carbon brings a value of +1. The oxygen of the
hydroxyl group has a value of-2, and the hydrogen of the hydroxyl group has a value of +1. This adds up to 0.
Therefore, the C-2 carbon must have an oxidation state of 0. In fact, the oxidation state of the C-3, C-4, and C-5
carbons are each 0.

Consider the C-6 carbon atom. The two hydrogen atoms directly attached to the C-6 carbon bring a value of +2. The
hydroxyl oxygen is again -2 and the hydroxyl hydrogen is again +1. This adds up to +1. In order to be neutral, the C6 carbon must have an oxidation state of-1. Adding up all the oxidation states of the six carbon atoms gives a net
oxidation state of 0. The correct choice is B.

55.

A is correct. We need to consider the diagram outlining glycolysis. In particular, we need to focus our attention on
Step 7. The 2,3-BPG shunt bypasses the enzyme phosphoglycerate kinase, which at Step 7 in glycolysis converts
1,3-bisphosphoglycerate to 3-phosphoglycerate. In this reaction, ADP is converted to ATP; and since this process
occurs twice per glucose molecule oxidized, we should generate 2 ATPs at this step. But we are bypassing this step,
so we do not get those 2 ATPs. In fact, the only ATPs that are synthesized are those at Step 10 in the conversion of

phosphoenolpyruvate to pyruvate. The 2 ATPs made here repay the 2 ATPs that we used as investments at Step 1
and Step 3 in the first part of glycolysis. Therefore, the net production of ATP is zero (0). [You may wonder how the
red blood cell can survive, if glycolysis yields zero net ATP. It turns out (which was not important for this question)
that about 25% of the glucose metabolized in the glycolytic pathways of the red blood cells is converted to 2,3-BPG.
Recall that this molecule is used to stabilize the deoxy state of hemoglobin. The remaining 75% of the glucose
utilized by the glycolytic pathway can generate those much needed 2 net ATPs.] The correct choice is A.

56.

B is correct. If there is a mild deficiency in this enzyme, PEP cannot be converted to pyruvate as readily. PEP
begins to increase in concentration as do other glycolytic intermediates behind it. An increase in 3-phosphoglycerate
leads to an increase in both 1,3-bisphosphoglycerate and 2,3-bisphosphoglycerate. Recall from our discussion of
hemoglobin that 2,3-BPG stabilizes the deoxy form of hemoglobin and results in a decrease in the affinity of
hemoglobin for oxygen. A decrease in its affinity for oxygen means that more oxygen has been released to the

Copyright by The Berkeley Review

268

The Berkeley Review


Specializing in MCAT Preparation

Biology

Metabolic Pathways

Section VIII Answers

tissue. The oxygen-hemoglobin dissociation curve would then shift to the right. Since the deficiency is mild (rather
than extreme), the shift is only slightly to the right. Also, note that a deficiency in this enzyme leads to a decreased
synthesis of ATP in red blood cells. The correct choice is B.
57.

C is correct. This can be worked out by using the Punnett square shown below. We can use the notation large P for

the good gene and small p for the defective gene. If the trait is expressed, the genotype is pp. If it is not expressed
the genotype is either PP, Pp, or pP.

Pp

pp

Pp

pp

There is a 50% chance that a child of this couple will express the trait. The correct choice is C.
58.

B is correct. The glycolytic pathway is not a one-way street. There are reversible reactions, as indicated by the
equilibrium arrows in Figure 1 of the passage. The entire reaction series follows a sequence. The first event is a
phosphorylation, followed by a dephosphorylation. If we study the glycolytic pathway from glucose to pyruvate, we
can count eight 8 phosphorylation events. There is one at Step 1, one at Step 3, two at Step 6, two at Step 7, and two
at Step 9. At Step 7, ADP is phosphorylated to make ATP. At the same time, 1,3-bisphosphoglycerate is
dephosphorylated. But no reduction is involved in this step. This is the clue that something is a bit out of the
ordinary.

What we are looking for is a phosphorylation step that is immediately followed by a dephosphorylation-andreduction step. The only oxidation-reduction in glycolysis occurs where we see the coenzyme NAD. If we examine

glycolysis in the reverse direction starting with 3-phosphoglycerate, we will see that in order to form 1,3bisphosphoglycerate a phosphorylation event must occur. Even though no ATP is indicated to be going to ADP at
the equilibrium arrows of Step 7 for this reverse reaction, it must be happeningotherwise, 3-phosphoglycerate
could not be phosphorylated to 1,3-bisphosphoglycerate. Notice that once 1,3-bisphosphoglycerate is produced, it
can be dephosphorylated it (which means the loss of Pi) and reduced to glyceraldehyde-3-phosphate. This is
indicated by the reverse of Step 6. The reverse of Step 5 is an isomerization reaction thatconverts glyceraldehyde-3phosphate to DHAP (see the structure below).
ch,-0-po,:"

Dihydroxyacetone
phosphate
CH,-0H

These three reversed steps in glycolysis do occur, and they are part of the pathway called gluconeogenesis, the
synthesis of glucose from specific precursor molecules. The correct choice is B.
Leucine Catabolism

Passage X (59 - 65)


59.

B is correct. Many of the L-amino acids can lose their a-amino group in a transamination reaction. This reaction is
carried out by enzymes called transaminases or aminotransferases. In this reaction, the a-amino group of leucine is
transferred to a-ketoglutarate (a component of the Krebs Cycle), an a-keto acid. This generates L-glutamate and
another a-keto acid, which in this case is a-ketoisocaproate.
e
coo
I
o=c

coo

coo

H,N C- H

H,N- C-H

CH,
CH,

1 '0
COO

a-Ketoglutarate

Copyright by The Berkeley Review

CH,
I

Aminotransferase

CH2
CH,

H,C- CH
I

1 "

CH,

COO

Glutamate

Leucine

269

COO
l
o= c
i

CH,
I

H,C- CH
I

CH,

a-Ketoisocaproate

The Berkeley Review


Specializing in MCAT Preparation

Biology

Metabolic Pathways

Section VIII Answers

Note that there is no net deamination in this reaction sequence. Transferring the amino group to glutamate allows

glutamate either to use that amino group in biosynthetic reactions or to eliminate that amino group (via the urea
cycle) as a waste product.

We can eliminate a hydration reaction as a possible answer choice, because we are not adding water to leucine. The
water molecule H2O has not been added (does not appear) on a-ketoisocaporate. Also, we are not adding a CO2
residue by carboxylation, nor are we removing hydrogens by dehydrogenation in this reaction. The correct choice
isB.

60.

B is correct This question asks you to remember how the human body disposes of nitrogen as a waste product.
Ammonia is quite toxic to animals. Since the pKa of ammonia is about 9.5, we would, at physiological pH, expect to
find this molecule present as the ammonium ion. The amino nitrogen is excreted as the ammonium ion by most
aquatic organisms. These animals are referred to as being ammonotelic. Reptiles and birds excrete the amino
nitrogen as uric acid, and are therefore referred to as being uricotelic. Human beings and many other terrestrial
animals excrete the amino nitrogen as urea, and because of this are referred to as being ureotelic. Recall that urea is
produced in the urea cycle. Glutamate is simply an intermediate in the pathway for nitrogen elimination in humans.
The correct choice is B.

61.

B is correct Adding H2O to the reaction would either hydrate the reactant or hydrolyze it. Neither of those reactions
is occurring. NADH and FADH2 are both reduced coenzymes. Adding either to the reactant means adding
hydrogens across a carbon-oxygen double bond or across a carbon-carbon double bond, respectively. Again, we do
not see either case here. This leaves, by the process of elimination, ATP as our choice.
S-CoA

s-Coa

HCq3

ATP

ADP + Pi

H20

0=C
C-H
II

H,C-C
I

(A biotin-dependent carboxylase enzyme)

ch2

LH3

coo

B-Methylcrotonyl-CoA

P-Methylglutaconyl-CoA

CO2 is a gas that tends to diffuse away. Trapping that CO2 and adding it to the reactant (via the biotin-dependent
enzyme), requires energy in the form of ATP. Carboxylation reactions involve energy. The correct choice is B.

62.

C is correct If HC03e reacts with the reactant to add CO2, then an OH moiety is left over. A hydrogen (H) from
one of the methyl groups of the reactant can combine with this OH group to produce H2O. Since there is no amino
group on the reactant, none can be formed in the products. Similarly, since there is no CO2 group on the reactant, a
decarboxylation reaction will not yield CQ2 in the products. Finally, the addition of a carboxyl group to the reactant

(via HC03e), does not form H2CO3 (carbonic acid). The correct choice is C.
63.

C is correct The enzymes mentioned in the question are proteins. The prefix apo- refers to the protein portion of
the enzyme, while the prefix holo- refers to the enzyme having all of its necessary parts (cofactors, subunits,
prosthetic groups, etc.) in order to function. Holocarboxylase synthetase attaches biotin to a specific apocarboxylase
enzyme. This makes the apocarboxylase enzyme complete (i.e., we could now call it a holoenzyme itself) and ready
to carry out a carboxylation reaction. If the holocarboxylase synthetase enzyme is deficient, biotin cannot be
attached to the apoenzyme, and the reaction does not occur. This means that the substrate that accumulates most

rapidly is the reactant for this reaction, which is (5-methylcrotonyl-CoA.


Neither acetyl-CoA nor p-methylglutaconyl-CoA can be formed, since we cannot get past the reaction catalyzed by
Enzyme 4. Therefore, they do not accumulate. a-Ketoisocaproate does begin to accumulate, but not as rapidly as 0methylcrotonyl-CoA. The correct choice is C.

64.

C is correct If this reaction were a thiolysis, we would need to add the element of sulfur (S) across the bond that
was broken. Similarly, if it were a hydrolysis, we would need to add a molecule of water (H2O) across the bond that
was broken. We do not see this in either case, even though we had a lysis in both situations.

Copyright by The Berkeley Review

270

The Berkeley Review


Specializing in NCAT Preparation

Biology

Metabolic Pathways

Section VIII Answers

The product of an aldol condensation is a molecule with both an alcohol and an aldehyde functional group. These
two groups are the source of the term -aldol, and these are exactly what we would have if we removed the CoA-S

portionof (3-hydroxy-p-methylglutaryl-CoA and added a hydrogen atom. The mechanism for this is shown below:
o0

ENZ-Base:

H,C-C-S-CoA

H - H,C- C - S-CoA

OOC-CH,-C- CH3

ENZ-Base H

Enolate ion

Acetyl-CoA

-*- H,C=C- S-CoA

OOC- CH, - C - CH, - C- S-CoA

H,C- C- S-CoA

CH,

Acetoacetate

An alkoxide ion

OH

OOC- CH, - C- CH, - C -

S-CoA

H -

'

"

OOC- CH, - C- CH, - C -

Base-ENZ

CH,

CH,

An alkoxide ion

S-CoA

:Base-ENZ

P-Hydroxy-p-methylglutaryl-CoA

However, it is evident the reaction outlined in Figure 1 in the passage is proceeding in the direction of a reverse
aldol condensation. The correct choice is C.

65.

B is correct. Glucose enters the glycolytic pathway and is converted to pyruvate in a series of ten reactions.

Pyruvate can react with coenzyme A (CoA) in a decarboxylation reaction to give acetyl-CoA, which is one of the
end products of the degradation of leucine outlined in Figure 1. Once acetyl-CoA is formed, it can enter the Krebs
cycle and be completely oxidized to CO2 and H2O. But acetyl-CoA cannot directly enter glycolysis, electron
transport, or oxidative phosphorylation.

We do find that the carbon atoms of acetyl-CoA can end up in the glycolytic intermediates through the process of

gluconeogenesis. Be aware that acetyl-CoA cannot be used as a precursor to yield a net synthesis of glucose in
mammals. Recall that acetyl-CoA is a two-carbon compound. When it enters the Krebs cycle, 2 carbon atoms are

lost as CO2; yet those 2 carbon atoms are not necessarily the same 2 carbon atoms derived from acetyl-CoA. This
means that if we were to label acetyl-CoA, we would find some of it in the glycolytic intermediates. We just would
not be able to get a net conversion of a two-carbon compound to a six-carbon sugar.

The energy tied up in the bonds of acetyl-CoA can be transferred (via the Krebs cycle) to the coenzymes NADH and
FADH2. These coenzymes can then transfer this energy, in the form of electrons and hydrogen ions, to the electrontransport chain and eventually to the process of oxidative phosphorylation, where ATP is generated. The correct
choice is B.

Trehalose Expenment

Passage XI (66 - 72)


66.

B is correct. The normal digestion of trehalose would not lead to the production of hydrogen, so choice A is
incorrect. No hydrogen is given off from interactions with bicarbonate or with gastric contents. Choices C and D are
therefore incorrect. The reason for increased hydrogen is the passage of undigested trehalose from the small intestine
into the colon, where many bacteria begin to metabolize it. They produce the hydrogen by fermenting the trehalose.
The correct choice is B.

67.

C is correct. Exhibiting a dose-response effect means that every time the dose is increased by a certain increment,

the subject's response follows a similar patterns of increases. Subject 3 has an increase of 10 ppm of hydrogen for
every 10g increase in trehalose, so choice C looks like the best answer. Subject 2 shows no dose-response effect, so
choice B is incorrect. Subject 1 shows no particular pattern, either. Choice A is incorrect. Subject 4 did not complete
the study. Choice D is incorrect. The correct choice is C.

68.

D is correct. Figure 1 shows us that trehalose is made of two glucose residuesjoined in a-1-1 linkage. The correct
choice is D.

Copyright by The Berkeley Review

271

The Berkeley Review


Specializing in MCAT Preparation

Biology
69.

Metabolic Pathways

Section VIII Answers

D is correct. This is one of the few times that "I can't tell from this passage" is the correct answer. Although lactose
is a disaccharide, this does not mean it behaves in the body exactly as trehalose does. Choice A is incorrect. Lactose

may or may not be a commoner component of the diet, depending on what culture you are examining. Butyou really
can't predict someone's lactose tolerance based solely on their degree of tolerance for another sugar. Choices C and
B are incorrect. The correct choice is D.

70.

A is correct. The enzyme trehalase is found in the brush border of the human duodenal mucosa. It may be present in
large or small quantities, depending on one's genetics. It is not induced by eating trehalose like the lac operon is
induced by lactose in bacteria. A person's tolerance for trehalose is not the result of their habitual consumption of
trehalose-containing foods. Choices C and D are therefore incorrect. Subject 1 has almost no increase in hydrogen
upon ingesting increasing doses of trehalose. This means the digestion of this sugar by trehalase is more efficient.
The subject's tolerance for doses of trehalose is high. The correct choice is A.

71.

B is correct. A centrifuge separates the component parts of a mixture by density, so choice A is incorrect. A mass
spectrometer characterizes compounds based on masses of isotopes, so choice C is incorrect. Extraction separates
compounds based on their solubility in organic solvents, making choice D incorrect. The correct choice is B.

72.

B is correct. Up to a certain dose, Subject 2 does little bacterial fermentation of trehalose. But after that threshold

point, a threshold, their enzymes can handle no more, and there is a suddenly big increase in the amount of hydrogen
in their breath. The correct choice is B.

Fuel Oxidation during Exercise

Passage XII (73 - 79)


73.

A is correct. In the initial phases of exercise, the oxygen consumption is not high sufficient to provide enough
oxygen to the body's muscle tissues for the oxidation of fuel molecules through the TCA cycle and oxidative
phosphorylation. During this brief period of lowered oxygen relative to demand, only glycolysis is carried out. The
TCA cycle enzymes are constitutively on; they do not need to be activated by exercise. Thus, choice B is incorrect.
Fatty acids are always metabolized aerobically. They cannot enter glycolysis, which is the only anaerobic fuel-

oxidizing pathway. Choice C is also incorrect. Glycolysis can keep pace with the energy demands of moderately
intense exercise, since this is what occurs. Choice D is incorrect. The correct choice is A.

74.

C is correct. The maintenance of glucose homeostasis has many complex and interacting safety checks. Since the
brain and nervous system are both almost entirely dependent on glucose as their fuel source, blood glucose must be
maintained within a fairly narrow range for all times, during a variety of activities. This may be accomplished
through the intake of dietary glucose, through the use of glycogen stores, and through gluconeogenesis. Choices A,
B, and D are incorrect. The correct choice is C.

75.

A is correct. Oleate is a fatty acid, which makes up about 40% of the circulating fatty acids. Resting muscle, as you
see in Table 1, uses FFAs as fuel primarily. Glycogen is not used at rest, so choice D is incorrect. P-hydroxy butyricacid is a ketone, used during fasting and very strenuous, sustained execise. Choice B is also incorrect. Lactic acid is

a waste product of metabolism in the muscle. It is produced during exercise and exported to the liver for use in
gluconeogenesis. This .means choice C is incorrect, too. The correct choice is A.

76.

D is correct. Since glucose is a small, water-soluble molecule, it does not need a carrier molecule. Usually, special
proteins such as albumin serve as carriers for large, insoluble molecules. Choice A is incorrect. There is no glucosebinding protein, so choice B is incorrect. And hemoglobin carries oxygen. Choice C is also incorrect. The correct
choice is D.

77.

B is correct. Insulin synthesis is a response to dietary carbohydrate entering the blood. Its function is to promote
glucose transfer from the blood into cells and also into the glycogen stores. Insulin levels in the blood are low during
exercise, so choice A is incorrect. Glycogen phosphorylase is an enzyme involved in glucose maintenance, not a
hormone. Choice C is incorrect. Oxytocin is a hormone involved in labor and childbirth, so choice D is incorrect,

too. Glucagon promotes glycogenolysis, so that muscle can use the available glucose for fuel and the liver can break
down and release stored glucose (glycogen) to the blood supply. The correct choice is B.

78.

A is correct. Carbohydrates provide about half the energy per gram that lipids do, so choices B and C are both
incorrect. Proteins, or amino acids provide about the same amount of energy value per gram as carbohydrates.
Choice D is incorrect. The correct choice is A.

79.

D is correct. Muscle glycogen contributes none of the energy demanded by the body during the first four hours of
exercise. Choice A is false. Gluconeogenesis increases during exercise, as you can see from Table 1. There is no

Copyright by The Berkeley Review

272

The Berkeley Review


Specializing in MCAT Preparation

Biology

Metabolic Pathways

Section VIII Answers

reason to expect it to decrease after four hours. Choice B is incorrect. Adipose tissue has about 100,000 kcal of
stored energy in the form triglyceride. It is released as free fatty acids and glycerol into the blood. There would be
no significant change in available fat stores for exercise that continues after four hours. Choice C is incorrect. Liver

glycogen's contribution to blood glucose does drop over the four-hour exercise period, however, and that decrease
would probably continue. The correct choice is D.

Passage XIII (80-87)


80.

Urea Cycle

C is correct. Glulamine is the major form of ammonia transport and is carried in the blood to the liver. The anitrogen is then removed as ammonia in the liver mitochondria. At a neutral pH, most of the ammonia that is
released is in the form of the ammonium ion (NH4e). This toxic ion is converted to urea in the urea cycle and later
exported to the blood from the liver. From there, it travels to the kidneys and eventually eliminated in the urine. The
correct choice is C.

81.

A is correct. Urea contains two nitrogen atoms. One comes from the condensation of NH4 with HC03e and the
other comes from aspartate. In order to place urea on the product side of the equation, there must be a balance of 2
nitrogens on the reactant side. This allows us immediately to eliminate choices C and D. We can eliminate choice B
also, because if we add aspartate on the reactant's side of the equation, would have 3 nitrogen atoms on that side of
the equation and only 2 nitrogen atoms on the product's side of the equation. The correct choice is A.

82.

D is correct. We know one of the nitrogen atoms comes from the ammonium ion. As shown in the diagram of the
urea cycle, the ammonium ion nitrogen is found in the structures of carbamoyl phosphate, citrulline,

argininosuccinate, and then arginine before it gets to urea. Where does the other nitrogen atom come from? The only
other reaction entering the urea cycle comes from aspartate, which is formed from the reaction of oxaloacetate and
glutamate. Glutamate passes its nitrogen to oxaloacetate and in the process is converted to a-ketoglutarate and
aspartate. There are no nitrogen atoms in oxaloacetate or in the citric acid cycle. The correct choice is D.
83.

B is correct. Carbamoyl phosphate is formed from the reaction of the ammonium ion with bicarbonate (HC03e).
The bicarbonate ion is carbon dioxide (CO2) in disguise. The vast majority of metabolic CO2 comes from the citric

acid cycle (Krebs cycle). Carbon dioxide is not produced in glycolysis, from the electron-transport chain, or in the poxidation of fatty acids. The correct choice is B.
84.

C is correct. The only two choices we have that allow a stable reaction with the ammonium ion are choice B

(aspartate) and choice C (glutamate). The ammonium ion reacts at the side chain carboxyl group to form the amide.
The more favorable reaction is the one with glutamate, because its side chain carboxyl group is farther away from

the positively charged a-amino group. There is less steric repulsion. Once the ammonium ion reacts with glutamate.
the amino acid glutamine is formed. Glutamine is a neutral, nontoxic compound. It can readily pass through cell
membranes and eventually into the blood, where it is carried to the liver for removal of the nitrogens. The correct
choice is C.

85.

A is correct. We need to be careful of words here. The word amidation refers to the introduction of an amino group

into an organic compound. However, we want to remove an amino group from an amino acid. Therefore, the
reaction is a deamination. This points to choice A and choice C. A transamination reaction involves the transfer of
the amide nitrogen (from glutamine) to another compound, so eliminate choice B. A dehydrogenation reaction
involves the removal of hydrogens (not nitrogens) from an organic compound. Eliminate choice D. Is the removal of
the amide nitrogen an oxidative or reductive deamination? In the passage, we see the conversion of glutamate to aketoglutarate. The removal of the amide there results in a carbonyl group at the a-carbon. This is an oxidation
reaction. The correct choice is A.

86.

B is correct. Draw a pedigree as shown below. This indicates how a gene is transmitted from one generation to the
next. Let's represent the disorder by aa.
I

AA

II

III

Copyright by The Berkeley Review

Aa
A

<>

Probability of expressing

Aa

Aa

aa

aa

the trait is what?

273

The Berkeley Review

Specializing in MCAT Preparation

Biology

Metabolic Pathways

Section VIII Answers

Individuals who do not express the trait are represented by either AA, Aa, or aA. The man's mother had ASA, so her
genotype must be aa. His father did notcarry the gene, so his genotype is AA. This means that the man's genotype is

either Aa or aA. It does not matter which he is, because he still does not express the trait. Now, the man marries a
woman who has the disorder, so her genotype must be aa. What is the probability that their child will be affected?
Since we do not know the sex of the child, we represent it by a diamond in the pedigree. A simple Punnett square
tells us that the probability the child will express the trait is 50%. The correct choice is B.
87.

A is correct. By increasing the synthesis of glycine, we are pulling more of the ammonium ion out of solution,
thereby decreasing its concentration. If we increase the synthesis of aspartate (by some mechanism), the
concentration of ammonia will begin to fall, because we are passing those nitrogens to the urea cycle and eventually
to urea. However, in the reaction drawn in the question, glycine is reacting with sodium benzoate, not aspartate.
Based on this mechanism, we would not have a way to increase the synthesis of aspartate. Eliminate choice B. We
cannot increase the concentration of urea, because there is a deficiency in the arginase enzyme that converts arginine
to ornithine and urea. Eliminate choice C. Again, if we were to increase the Krebs cycle intermediates to produce
more aspartate, and if aspartate were to drop off its nitrogens at the urea cycle, there would still be a lack of arginase
and a reduced production of urea. The intermediates would back up, and the concentration of ammonia would
increase (hyperammonemia). The best way to eliminate nitrogen is by some other route. The correct choice is A.

Passage XIV (88 - 94)

Gluconeogenesis and the Con Cycle

88.

C is correct. Think of gluconeogenesis as having two roles: maintenance of blood glucose by the liver and
regeneration of NAD in the muscle tissue. Lactate is an end product. When lactate is formed from pyruvate, NADH
is oxidized to NAD. This NAD is free to reenter glycolysis. Under aerobic conditions, NADH would be oxidized
in the electron-transport chain. However, this metabolic detour through lactate is necessary during anaerobic
conditions. The purpose of gluconeogenesis is to maintain constant blood glucose levels, especially during vigorous
exercise and during a fast. Following a meal, glucose is provided by the diet, so choice D is incorrect. During sleep
or studying, little anaerobic activity is happening. Choices A and B are incorrect. Exercise involves rapid glycolysis
to produce energy. Vigorous exercise is anaerobic. The correct choice is C.

89.

B is correct. Pyruvate is first carboxylated to make oxaloacetate (OAA). This is the first step in gluconeogenesis.
Pyruvate dehydrogenase is involved in the conversion of pyruvate to acetyl-CoA and CO2. Choice A is incorrect.
Lactate dehydrogenase is the enzyme the uses NAD(H) to interconvert pyruvate and lactate. Choice C is incorrect.
Finally, there is no lactate carboxykinase, so choice D is false. The correct choice is B

90.

D is correct. Pyruvate is converted to lactate, not the reverse, during anaerobic conditions, so statement I is
incorrect. Lactate is not converted to alanine, but pyruvate is, during anaerobic conditions. Statement II is also

incorrect. Lactate is a three-carbon compound, and lactose is a six-carbon sugar. They are unrelated except
alphabetically. Statement III is incorrect. The correct choice is D.

91.

D is correct. So, you've never heard of the reverse Cori cycle? Pay attention to your intuition, because in this case it

was pointing you to the correct answer. There is no reverse Cori cycle. Muscle tissue never releases glucose into the
blood. It does not have the enzymes to do so. Choices A, B, and C are incorrect. The correct choice is D.

92.

A is correct. NAD is required in glycolysis by the enzyme glyceraldehyde-3-phosphate dehydrogenase. Without


this cofactor, the enzyme could not function, and glycolysis would stop. Glycolysis produces a net gain of 2 ATPs,
so it would not stop due to a lackof ATP. Choice C is incorrect. Since glycolysis is a fixed pathway of enzymes, it
cannot change its ATP output or its cofactor requirements. Choices B and D are incorrect. The correct choice is A.

93.

A is correct. Since the body produces alanine, by definition it must be nonessential. Eliminate choices B and C.

Since the amino group is what distinguishes pyruvate from alanine, reduction is not the metabolic process that
interconverts them. It is transamination. Choice D is incorrect. The correct choice is A.

94.

B is correct. This is an anatomy question. Usually an artery flows into an organ, and a vein flows out of the organ.
However, the hepatic portal vein (choice C), is a specialized venous system that carries nutrients absorbed from the
digestive tract to the liver for processing. That is, the direction of flow is into the liver for this particular venous
system. This was mentioned in the passage so choice C could be easily eliminated as a possible correct answer.
There is no hepatic portal artery, so choice D is wrong. Choice A is also incorrect, because the hepatic artery is
bringing blood to the liver. The hepatic vein drains the liver and carries glucose and other nutrients released by the
liver to the rest of the body, making choice B the best answer. The correct choice is B.

Copyright by The Berkeley Review

274

The Berkeley Review

Specializing in MCAT Preparation

Biology

Metabolic Pathways

Section VIII Answers

Passage XV (95 - 100)


95.

Starch Blockers

B is correct. Carbohydrate that can be digested by the body's own enzymes (i.e., digestible carbohydrate) is
digested in the small intestine and does not reach the bacteria in the colon. Choices C and D are incorrect. Hydrogen
in the breath during the lactulose test is generated by bacteria in the colon as they break down otherwise
nondigestible carbohydrates, like lactulose. Since her breath hydrogen did not rise, her bacteria were not of the type
that ferments lactulose. A change in breath hydrogen was the measured variable in the study, so she could not
participate. Choice A is incorrect. The correct choice is B.

96.

B is correct. The points for the breath hydrogen produced following the starch blocker are not statistically different
from the placebo points. Statement I is correct. The rise in breath hydrogen following lactulose administration was
after the 120-minute mark. Statement II is incorrect. Starch in the small intestine is digested by a-amylase. If this

enzyme does not work, then the starch passes to the colon for fermentation by bacteria. This fermentation gives off
hydrogen, which is measured in this experiment. In people who have enough bacteria in their colon to ferment
lactulose, there are only two possible outcomes: either a-amylase worked, and no extra hydrogen was produced, or
a-amylase did not work, and the colon bacteria produced measurable hydrogen. Since there is no difference in the
amount of hydrogen gas subjects produced in the breath between a meal with a placebo and a meal with a starch
blocker, we can assume that the amylase was not inhibited. Statement III is correct. The correct choice is B.

97.

D is correct. The digestive enzymes trypsin, chymotrypsin, elastin, amylase, and lipase (among others) are produced
by the exocrine portion of the pancreas. They are secreted into the pancreatic duct and then empty into the small
intestine. Choices A, B, and C are incorrect. The correct choice is D.

98.

C is correct. The figures show no significant differences between the placebo and starch blocker tests for insulin

and glucose levels. There was essentially no change due to the administration of the starch blocker. It did nothing to
inhibit starch digestion. Choices A, B, and D are incorrect. The correct choice is C.

99.

B is correct. Starch is a plant's storage form of carbohydrate. Turkey is not a plant, and its storage form of
carbohydrate is glycogen, just like other animals. Choice A is incorrect. Butter is mainly fat, not carbohydrate.
Choice C is incorrect. Orange juice contains carbohydrate, but predominantly in the form of simple sugars. Choice
D is incorrect. The correct choice is B.

100. A is correct. Simple sugars are the monosaccharides, and they are absorbed by the body intact, without any
digestion. Amylase is not needed for their digestion. The inhibitor has no secondary effects that destroy simple

sugars. Even ifa real starch blocker existed and blocked amylase in vivo, simple sugars would not be affected by it.
Eliminate choices B, C, and D. The correct choice is A.

Copyright by The Berkeley Review

275

The Berkeley Review

Specializing in MCAT Preparation

Biology
A.

Section IX
Genetic
Information

B.

15N 15N

II

Switch medium from

15Nto 14N

DNA

15N 14N

I\

1st

I\

14N 14N

2nd

2nd
Generation

50%
X)

Patterns of Inheritance

2.

Gregor Mendel

3.

Genetic Loci and Alleles

4.

The Pedigree

Genetic Information

1.

Central Dogma of Molecular Biology

2.

Function of Nucleic Acids

DNA Synthesis
DNA Polymerase
DNA Replication

Practice Passages and Answers

14N 15N

Generation

14N/14N

1.

1.
2.

14N 15N

Generation

14N 14N

15N 14N

C.

Classical Genetics

II

15N/14N
50%

Direction of sedimentation

Berkeley
Specializing in MCAT Preparation

Genetic Information
Top 10 Section Goals
Be familiar with the transmission of genetic traits.

Understand how genetic traits are transmitted from one generation to the next. Think of meiosis in
this context, especially genetic recombination.

Be familiar with the work of Mendel.

Trace thepioneering work ofGregor Mendel using garden peas. Understand howto usea Punnett
square, and be able to calculate and compare the probabilities of genetic outcomes.

.&* Understand the concept of genetic loci and alleles.

>

Alocus(plural, loci) is the position of a geneon a genetic map.An allele is simplyone of two or more
alternate forms of a single gene at a given locus.

awA Know how to read a pedigree and how to calculate simple outcome probabilities.

jar Don't getlost in the details ofdifficult pedigrees. At most, be able to take a pedigree back three
J

generations. Be able to follow the lineages of all the individuals.

<j* Be familiar with the concept of genetic mutants and replica plating.
Know what is meant bya biochemical pathway and how mutations in a particular enzyme might

affect that pathway. How are mutants isolated, if they cannotgrow in a minimalmedium?

0 ^ Understand the importance of nucleic acids.

Befamiliar with the base-pairing rules establishedby Watsonand Crick. Know how to determine
the base compositionof a given pieceof DNA.

A&fk Be familiar with the Meselson-Stahl experiment.


The Meselson-Stahl experiment is a classic in molecular biology. It proved that DNAreplicates
semiconservatively, as had been proposed by Watson and Crick.

Be familiar with the general process of DMA replication.


Don'tworry aboutallthe enzymes involved in DNA replication. Justhavea simple understanding

of the overall process, and feel comfortablewith some of the names and structures.

()< Understand how the polymerase chainreaction works.


The discovery ofthe polymerase chain reaction ranks among the greatest scientific discoveries of
the 20thcentury. Therehave been many articles on this process, ana it is certainto be on the MCAT.

Be able to relate the material in this section to the topic of genetic expression.

P Besides DNA replication, there isalso DNA transcription andmRNA translation. Once a cell divides,
it must synthesizeproteins in order to survive.You must understand this process.

Biology

Genetic Information

Patterns of Inheritance

Classical Genetics
Patterns of Inheritance
Let's begin a discussion on patterns of inheritance. There were manyearlyideas
about heredity. It was widely believed that strange creatures could be breed by
cross breeding different species. For example, the minotaur from Cretan

mythology was a creature with the torso and head of a humanand the body of a
bull. The giraffe of the African plains was thought to be a cross between a
leopard and a camel. This is even reflected in the scientific classification

(established by Carl Linnaeus) of the giraffe; Giraffa camelopardalis. By the time


the Middle Ages arrived it was believed that crosses were viable only within a
particular species.

One of the early models proposed for heredity was that of pangenesis. This
theory stated that each part of the body produced tiny particles called pangenes
or gemmules. Pangenes were though to be miniature replicas of each organ or
tissue of the body. These pangenes were carried to the reproductive organs by
the circulatory system where they were packaged into the sperm or the egg.
During fertilization the male and female pangenes united in the female's womb.
Eventually a new organism would result. If a pangene was healthy or defective,
it would be passed on to the offspring.The combiningof pangenes led to the idea
of "blending inheritance." This simply meant that the individual was a mixture of
the two parents. The male pangenes were thought to be the major contributors to
this notion of blending. This was the prevailing theory up to the end of the last
century and it is interesting to note that it was proposed by Charles Darwin in
1868.

If the male and female contributions to the offspringresulted in blending, then at


some point in the future all of the offspring should resemble one another. We

know that this is not the case. However, it seemed to present a paradox for the
theory proposed by Darwin. In 1760 Josef Koelreuter, a German botanist, was

crossing different species of the tobacco plant. The offspring that were produced
were fertile and they were able to produce a new generation of plant which was
highly variable. A few members of this new generation resembled the original
species of tobacco plant. Even though these results were not in agreement with
the ideas of blending inheritance they did provide clues for the mechanism of
heredity.

Koelreuter's work was taken seriously by many investigators over the next
hundred years and in the 1790s an investigator named T. A. Knight crossed two
true-breeding lines of the garden pea (Pisum sativum). Within the pea plant

flower are the male (antherproduces pollen) and female (stigmaproduces


eggs) sexual organs. A true-breeding plant is one that if left to itself will self-

pollinate and always produce the same kind of plant. One true-breeding plant
would always produce purple flowers while the other true-breeding line would
always produce white flowers. These plants were designated as the parental (P)
generation. When the pollen from a purple flower was sprinkled on the eggs
from a white flower, only purple flowers were produced in the first filial (Fl)
generation (Figure 9-1).

Copyright by The Berkeley Review

279

The Berkeley Review

Specializing in MCAT Preparation

Biology

Genetic Information

Patterns of Inheritance

Purple Flowers X White Flowers


O

F!

Purple Flowers

F2

Purple Flowers + White Flowers

Figure 9-1

The purple flowers of the Fl generation were allowed to self-pollinate. In the


next generation, the second filial (F2) generation, both purple and white flowers
appeared. This says that some characteristic trait was hidden (masked) in the Fl
generation that reappears in the F2 generation. Knight's comments on this

phenomenon were that there were more purple flowers in the F2 generation than
white flowers. He left it at that.

Copyright by The Berkeley Review

280

The Berkeley Review

Specializing in MCAT Preparation

Biology

Genetic Information

Gregor Mendel

Gregor Mendel
Gregor Mendel was born in Austria in 1822. He entered a monastery in Brunn
and received a formal education. He later attended the University ofVienna and
after two years returned to the monastery because he failed the exams that

would have given him a teaching certificate. While at the monastery he carried
out examinations on the common garden pea. His work marked the beginning of
modern genetics.

Why did Mendel choose the garden pea? There were a number of reasons. (1) A

great deal of work had already been carried out with the garden pea. Many
earlier investigators (e.g., Knight) had produced hybrid peas by crossing
different true-breeding lines. (2) Many different true-breeding varieties were
available for use in experiments. (3) Mendel chose 32 ofthe many different truebreeding pea plants to work with. From these 32 varieties he chose lines that

could be distinguished by 7 different paired traits (e.g., alternative characteristics

like purple versus white, smooth versus wrinkled, etc.) (4) Pea plants were rather
easy to grow, they have a short generation time, and they are small and would
not take up much space in the garden.

As we have mentioned, both the male and female sex organs are contained
within the flower of the pea plant. If the pea plant is leftundisturbed, it willself-

Trait

pollinate. However, if the anthers are removed before pollination and pollen is

Flower Color
Seed Color

introduced from another pea plant, then cross-fertilization will result. The

fertilized eggs in the stigma develop into the embryo (the seeds). Each is the
product of a separate fertilization. The pod that contains the seeds has the

characteristics of the parents while the seeds themselves belong to the next
generation.

Dominant vs

Seed Shape
Pod Color

Pod Shape

Recessive

Purple vs White
Yellow vs Green
Round vs Wrinkled
Green vs Yellow
Round vs Constricted

Flower Position

Axial vs Top

Plant Height

Tall vs Dwarf

Table 9-1

Mendel chose 7 traits that were alternatively expressed for 7 characteristics of the
pea plant. Those 7 traits are shown in Table 9-1. Note which is dominant and
which is recessive.

Mendel chose parent plants that had bred true for many generations. He then
performed crosses by cross-fertilization for each of the 7 pairs of traits shown in
Table 9-1. For example, from the purple flower he used the pollen and from the
white flower he used eggs. Mendel also performed the reciprocal crosses (e.g.,
pollen from the white flower and eggs from the purple flower). These plants
represented the parental or P generation. The offspring of each of these 7
crosses, referred to as the first filial or Fl generation, expressed one of the two
parental characteristics.

Mendel allowed the Fl pea plants to self-pollinate for one generation. He then
scored the characteristics of the second filial or F2 generation. Mendel found that

each of the 7characteristics reappeared in the F2 generation. For example, in the


F2 generation round and wrinkled seeds were observed as were purple and
white flowers. This observation did not agree with the notion of blending
inheritance that we previously mentioned. In contrast to theearlier experimental
observations that Knight made on the garden peas, Mendel counted the number
of different types of plants in the F2 generation.
For each of the 7 traits in the F2 generation, Mendel found close to a 3 to 1 ratio

of plants that expressed one parental trait compared to the other parental trait.
The most frequently expressed trait was always the one that was exclusively
Copyright by The Berkeley Review

281

The Berkeley Review

Specializing in MCAT Preparation

Biology

Gregor Mendel

Genetic Information

expressed in the Fl generation. Mendel proposed that the trait that was
expressed in the Fl generation was dominant while the other unexpressed trait
was recessive. In other words, if a purple flower was crossed with a white flower
and the Fl generation gave all purple flowers, then those purple flowers were
dominant over the white flowers. Even though the white flower trait is masked
in the Fl generation it reappears in the F2 generation.
Mendel proposed that the parent plants do not transmit their physiological traits
or forms directly to their offspring (i.e., there are no pangenes) but rather
transmit "hereditary factors" which act later in the offspring to produce the trait.
Today we call these hereditary factors "genes."
Each individual possesses 2 factors with respect to each trait. For example, there
can be one factor for purple color and another factor for purple color, one factor
for purple color and another factor for white color, or one factor for white color
and another factor for white color (see Figure 9-2). One of these two factors is
contributed by each parent to the offspring.
Purple Purple

White

Purple

White

White

Figure 9-2

The alternative forms of the factors determining a given trait are called alleles.

The flower color trait has a purple allele and a white allele. An individual can
have two different alleles or two identical alleles. If the alleles are different, then

the individual offspring is referred to as being heterozygous. If the two alleles


are identical, the individual offspring is referred to as being homozygous. For
example, in Figure 9-2 the Purple/Purple alleles would be considered
homozygous as would the White/White alleles. The Purple/White alleles would
be heterozygous.

The two alleles contributed by the parents to an offspring do not influence each
other. For example, if an individual pea plant has a purple and a white allele,
then those alleles will stay purple and white. They will not form intermediate
alleles as expected from the blending inheritance hypothesis.
Let's consider one of Mendel's experiments on flower color. In this experiment
two true-breeding pea plants are crossed. The plant with the purple flowers
(dominant) is represented by an upper case W while the plant with the white
flowers (recessive) is represented by a lower case w. [The letter "W" is generally
chosen from the recessive allele.] The purple flower can only produce W gametes
while the white flower can only produce w gametes. We can express this cross in
a Punnett square as shown in Figure 9-3. Note that four heterozygotes are
produced. However, since W is dominant over w, we see that the Fi generation
yields all purple flowers.
In the next step Mendel allowed the Fl generation to self-pollinate. In the
Punnett square in Figure 9-3 we find two homozygous individuals (WW and
ww) and two heterozygous individuals (Ww and Ww). However, since the W
allele is dominant over the w allele, 3 of the offspring from this cross will have
purple flowers while just 1 will have white flowers. The color of the flower that is
observed is referred to as the phenotype. The total number of alleles that an
individual contains is referred to as the genotype. Therefore, the phenotypic
ratio of the F2 generation is 3:1. However, the genotypic ratio of the F2
generation is 1:2:1 (because there is one WW, two Ww, and one ww). Similar
experiments can be performed for the other traits that Mendel studied.

Copyright by The Berkeley Review

282

The Berkeley Review


Specializing in MCAT Preparation

Biology

Genetic Information

Gregor Mendel

White

Parental

Generation c^S
(P)

(Ww)

Ww

Ww
(Ww)

Ww

Ww

Purple
First Filial

Generation nzS

(Fi)

(Ww)

(WW)

F2 (3 Purple: 1White)
WW

Ww

w) \.

Ww

%>. I

(.

k \(ww)

WW

Figure 9-3

Based on experiments like these, Mendel proposed that:


1. The traits that were expressed could either be dominant or recessive. For
example, purple flower color in the pea plants was dominant over the
recessive white flower color. The parental generation does not transmit their

traits or their form to their offspring but rather they transmit hereditary
"factors" while include information about the traits to be expressed. Each
individual possessed two of these hereditary factors for each trait (one
coming from each of the two parents).
2.

The alternative form of a hereditary factor, which leads to an alternative form

of a given trait, is called an allele. For example, the hereditary factor for a
purple flower would be one allele while the hereditary factor for the
corresponding white flower would be the other allele.
3.

An individual who possesses two identical alleles is said to be homozygous


(e.g. WW for the purple flower color or ww for the white flower color) while

an individual who possesses two different alleles is said to be heterozygous


(e.g., Ww for the purple flower color).

4. The two alleles contributed by the parents to an offspring do not influence


each other in any way. In other words, the two alleles come into a cross and
Copyright by The Berkeley Review

283

The Berkeley Review

Specializing in MCAT Preparation

Biology

Gregor Mendel

Genetic Information

leave a cross unchanged. They do not form intermediate alleles as would be


expected from the blending hypothesis.

The presence of hereditary characters in an individual does not ensure that it


will be expressed. For example, in the Fi generation the white allele is
present in the purple and white hybrid but it is not expressed (it is recessive).
Recall that in relationship to what was expressed we mentioned the phenotype
and genotype of an individual. We said that the phenotype is the outward
appearance of a given trait. It is what you can actually see (e.g., flower color). The
genotype is the genetic makeup of an individual (e.g., purple flower color would
be WW or Ww while white flower color would be ww).
.

Pollen

1/2 W

1/2 W

1/2 w

1/4 WW

1/4 Ww

(Purple)

(Purple)

DC

1/2 w

1/4 Ww

1/4 ww

(Purple)

(White)

Figure 9-4

When Mendel crossed the two true-breeding lines of pea plants he found in the
F2 generation a 3:1 ratio of purple flowered plants to white flowered plants.
Another way to express this (other than Figure 9-3) is shown in Figure 9-4. If the
two alleles are alternate alleles and equally contributing, then half of them
should be W (upp'er case) and half should be w (lower case). This would be true
for both the pollen and the eggs. [When Mendel did this work the Punnett square
had not been invented. Instead, he used algebraic expressions.]

Mendel realized that not all the purple flowered plants should have the same
genotype. He said that some of the purple flowered plants would have a

different genotype. He predicted that V3 of the purpleflowered plants should be


WW while 2/3 should be Ww. (We are just considering the purple flowered
plants and not the white flowered plants. The genotypes of the purple flowered
plants in the F2 generation are WW, Ww, and Ww. See Figure 9-3.) He tested his
prediction in two ways.
His first test was to allow the F2 plants to self-pollinate. If a purple flowered F2
plant with the genotype WW self-pollinates, then the offspring will only be
purple flowered plants. If both purple flowered F2 plants with the genotypes
Ww self-pollinate, then both purple and white flowered plants are produced.

This confirms the prediction thatV3 of thepurple flowered F2 plants will be WW


while 2/3 of the purple flowered F2 plants will be Ww. [A similar test could be
done for just the white flowered pea plants.]

His second test involved a test cross. In this cross Mendel used a purple flowered
plant from the Fl generation in which the genotype was not known (i.e., it was
either WW or Ww). In order to determine the genotype of this unknown purple
flowered plant, he crossed it with a white flowered plant which was
homozygous recessive (i.e., ww) as shown in Figure 9-5.
If the unknown allele distribution in the purple flowered plant is WW, then all of
the offspring in the test cross with the homozygous ww white flowered plant
will be Ww. The offspring will be heterozygous and they will all have purple
flowers. However, if the unknown allele distribution in the purple flowered plant
is Ww, then the results of the test cross will give half heterozygous Ww purple
flowered plants and half homozygous ww white flowered plants (Figure 9-5).

Copyright by The Berkeley Review

284

The Berkeley Review


Specializing in MCAT Preparation

Biology

Genetic Information

(ww)

White

Ww

Ww

Ww

(ww)

White

&I GD
Ww

Gregor Mendel

&f GO

y^nt')Ww

Ww

^0

Allele distribution
is unknown. It is
either WW or Ww.

Since all purple flowers,


unknown flower was

homozygous (WW) dominant.

WW

Ww

WW

Since half purple flowers and


half white flowers, unknown
flower was heterozygous (Ww).

Figure 9-5

Mendel's experiments therefore confirmed his hypothesis. Alternative alleles

segregatefrom each other in heterozygous individuals and retain their identity. This is
known as Mendel's First Law of Heredity (also called the Law of Segregation).
Even though Mendel showed this to be true for garden peas, it has been shown
tobeapplicable to alleukaryotic organisms (including humans).

RRGG

rr88

(P Generation)

Q RrGg (Fl Generation)


i
1/4 RG

1/4 RG

1/16

1/16

Resulted from parents

RRGG

RRGg

RrGG

RrGg

with two different

1/16

r.

RrGG

zr.

RRGg

o
1/16

RrGg

1/16

1/16

RRgg

RrGg

Rrgg

1/16

1/16

rrGG

rrGg

Ci

characteristics.

1/16

RrGg

1/4 rG

1/4 rg

F2 Generation

1/16

1/16

13
1/4 Rg
a

1/4 rg

1/16

o
41

Pollen (Male)
1/4 Rg
1/4 rG

Q Round, Yellow

1/16

1/16

1/16

Rrgg

rrGg

rrgg

Ci

Ci

1/16

C^J Round, Green

3 Wrinkled, Yellow
\2zJ Wrinkled, Green

Figure 9-6

Copyright by The Berkeley Review

285

The Berkeley Review

Specializing in MCAT Preparation

BlOlOgy

Genetic Information

Gregor Mendel

Recall that Mendel identified 7 different pairs of traits. He next examined two
different traits segregating in the same plant. The two different traits he
considered were round versus wrinkled seeds and yellow versus green seeds.
The round seeds are dominant over the recessive wrinkled seeds. The yellow
seeds are dominant over the recessive green seeds. [Mendel had determined
which seeds were dominant and recessive in a previous crossing experiment
involving only one of the pairs.] After establishing pure breeding lines of pea
plants with these traits, Mendel crossed a true-breeding pea plant with round
seeds which were yellow with a true-breeding pea plant that has wrinkled seeds
which were green. He wanted to know if a particular allele for one trait (such as
seed color) would influence which allele the gamete had for the other trait (such
as the seed shape). The type of cross Mendel constructed is referred to as a
dihybrid cross (Figure 9-6). [Note that in this example we will let the upper case
R represent round seeds, the upper case G represent yellow seeds, the lower case
r represent wrinkled seeds, and the lower case g represent green seeds. The
reason behind this is because we already used the letter W in the previous
example with flower color.]
Mendel crossed true-breeding RRGG pea plants with true breeding rrgg pea
plants. This gave an Fl generation in which all the pea plants had round yellow
seeds (i.e., RrGg). These dihybrid individuals were next allowed to self-fertilize.
"If the segregation of alleles affecting seed shape were independent of the
segregation of those affecting seed color, then the probability that a particular

pair of seed shape alleles would occur together with a particular pair of seed
color alleles would be simply the product of the individual probabilities that
each pair would occur separately. Thus, the probability that an individual with
wrinkled, green seeds would appear in the F2 generation would be equal to the

probability of observing an individual with wrinkled seeds (V4) times the


probability ofobserving an individual with green seeds (V4), or V16."
The composition of the dihybrid cross shown in Figure 9-6can be predicted if the
mechanism behind placing an R allele or an r allele into a gamete is independent
of the mechanism behind placing a G allele and a g allele into another gamete.
[Recall Mendel's First Law: Alternative alleles segregate (separate) from each other in
heterozygous individuals and retain their identity. Another way to put this is that
"two members of a gene pair segregate from each other into the gametes, so that

V2 ofthe gametes carry one member ofthe pair and the other V2 ofthe gametes
carry the other member of the gene pair." A gene pair could be a dominant gene
(allele) and recessive gene (allele).] Notice that in the dihybrid individuals the
genes involved in the shape of the seed and the color of the seed can each be
represented by a pair of alternative alleles. This means that one would expect
RG, Rg, rG, and rg gametes. The probability that a gamete would be Rgis based
on Mendel's First Law. In other words, the probability that a gamete would be

RG is simply V2 x V2 or V4. The same would hold for the Rg, rG, and rg
gametes. This is where the V4 comes from in the dihybrid cross shown in Figure
9-6. This would also explain why the probability of finding an individual with

wrinkled, green seeds in the F2 generation is Vi$. It is simply the probability of


observing the male gamete rg (which is V4) times theprobability ofobserving the
female gamete rg (which is also V4). In other words, the probability of finding an
rrgg zygote would be V4 x V4 or Vig.
The results of Mendel's dihybrid cross are shown in Figure 9-7. These values are
obtained from the F2 generation. Mendel examined 556 seed types and found
that 315 were round and yellow, 108 were round and green, 101 were wrinkled
Copyright by The Berkeley Review

286

The Berkeley Review


Specializing in MCAT Preparation

Biology

Genetic Information

Gregor Mendel

and green, and 32 were wrinkled and green. This gave a ratio of 9:3:3:1,
respectively.

If you look at the Punnet square inFigure 9-6, you will notice that there are 9/i6

round and yellow seeds, 3/ig round and green seeds, 3/16 wrinkled and yellow

F2 Generation

seeds, and 1/16 wrinkled and green seed. This is a ratio of 9:3:3:1. There are 4
phenotypes in theF2 generation. How many genotypes are there for each ofthe4

Round & Yellow


Round & Green
Wrinkled & Yellow
Wrinkled & Green

phenotypes?

What does this dihybrid cross mean? Itsimply means that the hereditary factors
(genes) for color (yellow and green) and shape (round and wrinkled) assort

Seed#

Ratio

315
108
101
32

9
3
3
1

556

16

Figure 9-7

independently of one another. This is referred to as Mendel's Second Law of

Hereditary (or the Law of Independent Assortment). Another way to put this
would be that the segregation of one gene pair is independent of other gene pairs
during the formation of the gametes. One addition to this statement (which
Mendel did notpostulate) is that independent assortment ofthe genes will occur
if they are located on different chromosomes or are far apart on the same
chromosome.

Mendel published his findings in 1866 but did not receive much attention until

about 1900, some 16 years after his death. When Mendel published his papers he
did not know about meiosis, chromosomes, DNA, or even genes. Today we
know his hereditary factor tobe genes andhislaw ofsegregation tobemeiosis.

Flower

Seed

Color

Color

The word "gene" was first coined in 1911 by Wilhelm Johannsen, a Danish
geneticist. Genes, which are the basic units of hereditary (DNA), are located at

specific locations along the chromosomes. In fact, they are arranged linearly

IW
1

Chromosome

along the chromosomes. Recall that we mentioned that alleles are one of two or

more alternative states of a given gene. Forexample, thewhite and purpleflower


color of the pea plants that Mendel examined is simply due to alternate forms of
the same gene.

If the genes are on different chromosomes, they will assort independently


because the chromosomes themselves assort independently. However, if two
genes are on the same chromosome, then they may assort independently if
crossing-over occurs between them. As we will see, the order and spacing of
genes on a chromosome can be determined from the frequency with which the
different gene pairs recombine. Genes that are far apart will recombine more

4 f\

FlowerPosition

Pod Color

frequently than genes that are closer together.

The frequency with which crossing-over occurs allows one to map the relative
positions of the various genes on chromosomes. For example, consider the seven
traits which Mendel studied. These traits can be located on the chromosomes of

the pea plant as shown in Figure 9-8. Seed color and flower color can be found on
chromosome #1 while seed shape can be found on chromosome #7.

Plant

Pod Height
Shape

4
hi*

WR

M
wrj

If you studied the segregation of seed color and seed shape, you would find that
they would assort independently (because they are on different chromosomes).
Flower colorand seed colorshould assort independently because they are rather

far apart on chromosome #1. Note the location of the genes for plant heightand

Seed Shape

Figure 9-8

pod shape. Those two genes are quite close to one another and therefore should
not assort independently.

Copyright by The Berkeley Review

287

The Berkeley Review

Specializing in MCAT Preparation

Biology

Gregor Mendel

Genetic Information

Let's place the traits for seed color and seed shape on the chromosomes as shown
in Figure 9-9. We willlet G stand for the seed color and WRstand for seed shape.
In the gonial cells we will just use chromosome #1 and chromosome #7.

&^ Ci

S Phase

After Replication

Gonial Cells

WRI

WR

WR

wr

wr

wr

Figure 9-9

After replication we get homologous pairing and then crossing-over as shown in


Figure 9-10.

WR

wr

WR

wr

Homologous Pairing

wr

WR
WR

wr

Crossing Over

Figure 9-10

Note the arrangement of the crossover events as they proceed through Telophase
I. This is shown in Figure 9-11. Two possibilities for Telophase I are shown.

Copyright by The Berkeley Review

288

The Berkeley Review


Specializing in MCAT Preparation

Biology

Genetic Information

Gregor Mendel

Or

WR tv\yfc/j WR wr *&& wr

wr^gwr WR^ffipl WR

Telophase I

Telophase I

Figure 9-11
Walter Sutton

In 1902 the American geneticist Walter Sutton made a logical argument that
stated that chromosomes were the places where Mendel's hereditary factors were
located. Sutton postulated:

1. IfMendel is right, then the sperm and the egg must contribute equally. Also,
since the spermhas verylittle cytoplasm and is mainly composed ofnuclear
material, the hereditary factors must be contained within the nucleus.
2.

The meiotic behavior of chromosomes is the same as that of Mendel's


hereditary factors.

3. Gametes from meiosis have one copy of genetic material for each hereditary
factor. Mendel's hereditary factors are distributed in the same way.
4. Chromosomes assort independently in meiosis as do Mendel's hereditary
factors.

However, there was a problem with this because it was mentioned that there

were many more independently segregating pairs of genes (hereditary factors)


than there were chromosomes.

Thomas Hunt Morgan

In 1910 Thomas Hunt Morgan performed a crucial experiment that

demonstrated that Sutton's theory was correct. Morgan showed that a gene
controlling eye color in the fruit fly Drosophila melanogaster was located on the X
chromosome. As we will see, the results from Morgan's experiments led to the
acceptance of Sutton's theory. In 1909 F. A. Janssens suggested that homologous
chromosomes exchanged material during meiosis. At first Janssens theory was
not widely accepted but later experiments would prove him correct. One of those
experimentswas performed in 1931 by the geneticist Curt Stern who proved that
crossing-over in the X chromosomes of Drosophila melanogaster involved the
physical exchange of chromosomal material. With independent segregation of
chromosomes as well as crossing-over along the chromosomes, there can be more
independently segregating units than chromosomes.
Copyright by The Berkeley Review

289

The Berkeley Review

Specializing in MCAT Preparation

Biology

Gregor Mendel

Genetic Information

Let's consider Morgan's experiment for a moment. The fruit fly Drosophila
melanogaster normally has red eyes (referred to as the wild type because it is
normal). During one of his experiments he had noticed a mutant male fruit fly
with white eyes. Morgan crossed this mutant white eyed fly with a wild type red
eyed female fly. All the progeny (offspring) in the Fl generation had wild type
eyes (i.e., all the offspring had red eyes). Next, a male and a female fruit fly with
red eyes from this Fl generation were allowed to mate. In the resulting F2
generation all the female fruit flies were of the wild type while only half of the
male fruit flies were of the wild type. The remaining male fruit flies had the
white eyed mutation. After examining all of the progeny in the F2 generation,

Morgan concluded that eye color segregated among the progeny as predicted by
Mendel. However, the white-eyed characteristic only expressed itself in the F2
male fruit flies. This is a very unusual result.
After looking at the chromosomes of both sexes of Drosophila melanogaster,
Morgan discovered that the female had two copies of the X chromosome while
the male had only one copy of the X chromosome and a copy of a Y chromosome.
In other words, females flies have an XX genotype while males have an XY

genotype. It turned out that the trait for white eyes lies on the Xchromosome and
is missing from the Y chromosome. If a given trait is determined by a gene on the
X chromosome, it is said to be sex linked (or X linked).

Red-eyed

White-eyed

male

Gene for
white eyes

Gene for

female

red eyes

Parental
Generation

F| Generation

F2 Generation

White-eyed
male

Figure 9-12

Copyright by The Berkeley Review

290

The Berkeley Review


Specializing in MCAT Preparation

Biology

Genetic Information

Gregor Mendel

As shown in Figure 9-12, the Fl generation all have redeyes. This means that the

gene for red eyes is dominant. Notice that the female fly in the Fl generation has
a mutant Xchromosome. Her eyes are red because her onegood Xchromosome
is still able to express red eye color. However, when this Fl female mates with an

Fl male who has red eyes, she passesher good Xchromosome and her defective
Xchromosome to her progeny in the F2 generation.

The Fl male passes his Y chromosome and his good Xchromosome to the
progeny in the F2 generation as well. If a female fly in the F2 generation were to
receive the defective X chromosome, her eye color will still be red because she
would have received a wild-type X chromosome from the male of the Fl
generation. However, if a male received the defective X chromosome from the

female of the Fl generation, thenhe would have white eyes because there would

be no other Xchromosome with the gene for the red eye color available. Why?
Because the genotype of the male is XY while that of the female is XX.
Curt Stern

In 1931 Curt Stern did an experiment that proved that crossing overinvolved the
physical exchange of chromosomal material. Stern examined two genes on the X

chromosomes of the female fruit fly Drosophila melanogaster. The two genes that
he was interested (Figure 9-13) in were the recessive gene for carnation eye color
(car) and the dominant gene for thebar-shaped eye (B).
PES Y Segment
+

Fl Female

Gapi

>

Crossing-over

No

between car

crossing-over
car

andB

CP

Fl Female

Gametes

Sperm from Fl Male Gametes

Carnation
Bar

Normal

Parental Combinations

Recombinant Combinations

Figure 9-13

Copyright by The Berkeley Review

291

The Berkeley Review

Specializing in MCAT Preparation

Biology

Gregor Mendel

Genetic Information

One of the X chromosomes could be identified because it had a "gap" in its


structure (due to inability to stain properly) while the other X chromosome could
be identified because it had a piece of a Y chromosome attached to it. These two
chromosomes could be clearly distinguished from one another under the light
microscope.

As shown in Figure 9-13 the experiment starts out with the Fl female's X
chromosomes. Each has the respective genetic locus (location of a gene on a
chromosome) for the carnation eye color and the bar-shaped eye. Stern crossed
these Fl females with the sperm frorn a male fruit fly who had carnation colored
eyes. He then separated out the cross-over events in the F2 progeny.
He found normal shaped eyes that were carnation colored and normal colored
eyes that were bar-shaped in the progeny that had experienced crossing-over. In
the progeny that had not experienced crossing over, he observed carnation
colored eyes that had the bar-shape and normal colored eyes without the barshape. Stern concluded that the genetic exchanges of various traits on a given
chromosome (such as eye color or eye shape) involve the actual exchange of
portions of those chromosomes in a event referred to as crossing-over. In other
words, whenever genes recombine, chromosomes recombine. [Note that in the
diagram in Figure 9-13 the "+" sign indicates the wild type individual or the
normal individual.]
Alfred Sturtevant

Alfred Sturtevant (a student of Morgan's) constructed the first genetic map in


1913. Sturtevant examined three traits in the female fruit fly Drosophila
melanogaster. They were yellow body color (designated as y), white eye color (w),
and miniature wings (min). These traits resided on the X chromosome and were
sex-linked. These lower cases refer to the recessive alleles. The normal body
color is gray (+), the normal eye color is red (+), and the normal wing (+) is about
50% longer than the miniature wing. [Note that here we are using the "+"
terminology which means the dominant allele. We could have used upper case
Y instead of the +, an upper case W instead of the +, and upper case MIN instead
of the +, respectively. The meaning is the same.]
Female

X Chromosome
X Chromosome

- o -EL

Male
min

+
-D-

X Chromosome

Y Chromosome

min

0
w

X Chromosome
X Chromosome

-D-

min

Fl Females

Figure 9-14

Sturtevant crossed a female fruit fly that was homozygous recessive for the three
traits with a male fruit fly that was normal for these traits. All of the female Fl
progeny were heterozygous as shown in Figure 9-14. The important point here
is that the progeny females are heterozygotes. This means that if crossing-over
Copyright by The Berkeley Review

292

The Berkeley Review


Specializing in MCAT Preparation

Biology

Genetic Information

Gregor Mendel

occursbetween any two traits, gametes will result with different combinations of
the alleles.

In order for Sturtevant to see all the possible recombinant types, he crossed the
Fl females shown in Figure 9-14with males that were recessive to all three traits.
This cross is shown in Figure 9-15.
Female
w

X Chromosome

Male

X Chromosome

min

mm

-ID-

-a-

Q D

X Chromosome
Y Chromosome

Progeny Fruit Flies


Figure 9-15

The progeny were next scored for the phenotype expressing these three traits.
Sturtevant's results are shown in Table 9-2. When you lookat this table, keep in
mind the cross shown in Figure 9-15. Let's consider the body, eye, wing
phenotype (1st row) which reads + + +. How is this phenotype obtained? Look
at the female's X chromosomes in Figure 9-15. If the female X chromosome
which reads + + + combines with the male X chromosome which reads y w min,
then the resultant progeny will be a female with the phenotype +++. Similarly, if
the female X chromosome which reads + + + combines with the male Y

chromosome, then the resultant progeny will be male with the phenotype + + +.
This is one parental phenotype (i.e., + + +) that is observed in Table 9-2. We can
do a similar analysis for the y w min phenotype (2nd row) in Table 9-2 as well.
This is also a parental phenotype. The number of progeny observed for the
parental phenotype + + + is 758 while the number of progeny observed for the
parental phenotype y m min is 700. Because these are random events we do not
get the same numbers.
PHENOTYPES

CROSSOVER TYPES
Number of

Body
Parental

Single Crossover

Double crossover

Eye

y
+

Wing
+

min
min
+

min
+

min

TOTAL

Recombination Frequency (%)

Progeny
758
700
401
317
16
12

Body/Eye

Eye/Wing

401
317

Body/Wing
-

401
317

1
0

16
12
1
0

1
0

2205

29

719

746

1.315

32.608

33.832

16
12
-

Table 9-2

Where did the other six classes of phenotypes come from? Consider the 3rd row
of phenotypes in which we have the + + min phenotype. How did we get this
phenotype? There must have been a crossover as shown in Figure 9-16. When
Copyright by The Berkeley Review

293

The Berkeley Review


Specializing in MCAT Preparation

Biology

Gregor Mendel

Genetic Information

the two female X chromosomes separated we would have yw + and + + min.


Suppose the female + + min combined with the male Xchromosome which reads
y w min. The resulting female progeny would have the phenotype + + min.
[Remember, a genotype of (+ + min)/(y w min) would give a phenotype of + +
min because the dominant + + genes would be expressed over the recessive y w
genes. The min/min genotype is still recessive and so the phenotype would be
min.] Similarly, if the female + + min combined with the male Y chromosome,
the resulting male progeny would have the phenotype + + min. A similar
analysis can be done for the 4th row of phenotypes in Table 9-2. In this case we
get the phenotype y w +. The number of progeny observed for the + + min
phenotype is 401 while for the y w + phenotype it is 317.

Female

X Chromosome

-D- o -

Cross-over

X Chromosome

min

Male

mm
n

-a-

X Chromosome

X
Y Chromosome

' * 0
Progeny Fruit Flies
Figure 9-16

Consider the 5th row of phenotypes in Table 9-2 which reads + w min. We could
obtain this type of phenotype by doing a cross as shown in Figure 9-17. After the

female X chromosomes crossover and pull apart we will have a y + +


chromosome and a + w min chromosome. If the female + w min chromosome

combines with the X chromosome of the male which reads y w min, the resulting
progeny will be female with the phenotype + w min. There are 16 progeny
observed with this phenotype. Similarly, if the female + w min chromosome
combines with the male Y chromosome, the resulting male progeny will have the
phenotype + w min. We can do a similar analysis if the female y + +
chromosome combines with either the male X chromosome which reads y m min
or the male Y chromosome. In both cases the progeny will have the phenotype +
w min. The number of progeny observed in this case is 12. Some will be females
and some will be males.

Male

Female

w
X Chromosome

min

-p-

CrossoverY

XChromosome

ai\ p

w
P

min
D

X Chromosome

Y Chromosome

* -a
Progeny Fruit Flies
Figure 9-17

Copyright by The Berkeley Review

294

The Berkeley Review


Specializing in MCAT Preparation

Biology

Genetic Information

Gregor Mendel

Notice that the cross in Figure 9-17 yields 16 progeny with the phenotype +w
min and12 progeny with thephenotype y + +. Notice that the cross in Figure 916 yields 401 progeny with the phenotype + + min and 317 progeny with the
phenotype y w+. Why the difference in the number of progeny? It has to do
with thedistance thegenetic markers (i.e., the genes) areaway from one another.
The further they are away from one another, the more chance there will be for a
cross. The closer they are to one another, the less chance there will be for a cross.

In other words, the y and w genetic markers are closer together than the w and
min genetic markers. Therefore, we can modify our chromosomes to represent
this as shown in Figure 9-18.
Female

Male
min
a

-Q-

min

-a-

x
Y

Progeny Fruit Flies


Figure 9-18

Consider the 7th row in Table 9-2. We can explain the phenotype+ w + by a
double crossover as shown in Figure 9-19. The number of progeny observed for
this crossover is 1. [I have left out the details of explaining how the
chromosomes combine to get the genotype and then the observed phenotype
because it is explained in the previous examples.] In the 8th row we see the
phenotype y + min. We can obtain this phenotype from the double crossover
shown in Figure 9-19 as well. The number of observed progeny for the
phenotype y + min is 0.
Female

Male
min

xp.

y
-o-

mm

-o

EZCZx
Progeny Fruit Flies
Figure9-19

The total number of progeny observed in this experiment is 2205. The total
number of progeny observed to have crossover events between the y and w
genetic markers is 16 + 12 + 1 = 29. Therefore, this amount of crossing over
represents (29/2205) x 100 or 1.315% of the total number of crossover events.
Similarly, the total number of progeny observed to have crossover events

between the w and min genetic markers is 401 + 317 + 1 = 719. This represents
719/2205 or 32.608% of the total number of crossover events. Finally, the total
number of crossover events between the y and min genetic markers is 401 + 317 +
Copyright by The Berkeley Review

295

The Berkeley Review


Specializing in MCAT Preparation

Biology

Genetic Information

Gregor Mendel

16 + 12 = 746. This represents 746/2205 or 33.832% of the total number of


crossover events. What we have just defined in this discussion is the map
distance between a gene pair. This distance is usually expressed in terms of a
map unit. In other words, if the frequency between two genes (e.g., y and min)is
33.832%, then those genes are said to be separated by about 33.8 map units. A
map unit is sometimes referred to as a centimorgan (cM) in honor of T. H.
Morgan.

Knowing the map units between genetic markers allows us to draw a genetic
map. We can do this for the analysis in Table 9-2. This is shown in Figure 9-20.
min

^^
~
^

1.31

>k
^r*

32.61
33.83

^k

>w

Figure 9-20

Copyright by The Berkeley Review

296

The Berkeley Review


Specializing in MCAT Preparation

Biology

Genetic Information

Genetic Loci & Alleles

Genetic Loci & Alleles


So far we have only considered two alleles at each genetic locus. For example, in
Mendel's garden peas we had a flower color locus in which one allele was able to

express a purple flower while the other allele was able to express a white flower.
Recall that the purple flower allele was dominant over the recessive white flower

allele. It turns out that a given genetic locus can have many alleles. One example
is the ABO blood group.

We have considered only one gene controlling a particular phenotype (e.g.,


flower color, seed shape, etc.). However, as we will see, many genes may control
a particular phenotype. For example, in the biosynthesis of many compounds
within the cell there can be many sequential steps. Each step is controlled by a
separate enzyme which in turn is controlled by a particular gene. One such
biochemical pathway of interest involves the synthesis of the tryptophan. This
amino acid is considered an essential amino acid. An essential amino acid is an

amino acid that an organism cannot synthesize itself and therefore must obtain
that amino acid in its diet.

Tryptophan

One class of mutants that we should consider are the auxotrophic mutants. An
auxotroph is a mutant that will grow only when its medium is supplemented
with a particular compound which is not required by the normal wild type
organism. The wild type organism is referred to as a prototroph. An auxotroph
will not grow on a minimal medium. For example, an auxotroph might require
certain vitamins, amino acids, purines, or even pyrimidines for its growth.
prototroph will grow on a minimal medium.

One specific type of auxotroph that we will consider is an auxotroph for the
amino acid tryptophan. Tryptophan auxotrophs will grow on a complete
medium. They will not grow on a minimal medium unless that minimal medium
has been supplemented with tryptophan.
In order to study these tryptophan auxotrophs we need to isolate them. As we

will see, they are not all the same. How could we isolate such an auxotroph? In
the case of a penicillin mutant all we needed to do was plate bacteria on an agar
plate, add penicillin, and find which bacteria survived. In the case of the sugar
utilization mutants all we needed to do was look at the array of petri plates that
we generated and find the white bacterial colonies. However, in the case of a
tryptophan auxotroph we need to find a colony of bacteria that does not grow.
How can we isolate a bacterium that will grow in a complete medium but will
not grow in a minimal medium?
In 1952 Esther and Joshua Lederberg devised an experimental procedure called
replica plating. This procedure was designed to isolate auxotrophs. The
procedure behind replica plating is straight forward. Bacteria are incubated for a
period of time on a master plate which contains a complete medium. Both the
auxotrophs and the prototrophs will grow on this master plate. These bacteria
are distributed in such a way that they will form individual colonies. Next, you
gently touch the colonies of this master plate with a velvet stamp. The velvet
stamp has fine hairs which can pick up some of the bacterial cells of the master
plate. The velvet stamp is then touched to a new plate so that the bacterial
colonies are set down in the same orientation that they were in on the master
plate. This new plate is referred to as a replica plate (see Figure 9-21). If our
Copyright by The Berkeley Review

297

The Berkeley Review


Specializing in MCAT Preparation

Biology

Genetic Information

Genetic Loci & Alleles

replica plate contained just a minimal medium, then only the prototrophs would
grow. The auxotrophs would not grow because they need to have a particular
supplement in their medium. The replica plate can be compared with the master
plate to determine which colonies are auxotrophic. Once you have determined
which colonies on the master plate are auxotrophic colonies, you can then test
those colonies to see exactly what compound it is that they need for their growth.
velvet stamp
Contains both

prototrophsand auxotrophs

Contains prototrophs

Picked up colonies from


the master plate

missing
'colonies

Replica Plate

Master Plate

Figure 9-21
Replica Plating Technique.

Let's consider some tryptophan (abbreviated as Trp) auxotrophs. A bacterium

which is a tryptophan prototroph is designated as Trp+. This bacterium is the


wild type. It is normal. It does not need tryptophan in order to grow because it
can make this amino acid from the compounds given to it in its medium. A
mutant bacterium which requires tryptophan for its growth is designated as Trp:
It turns out that the study of tryptophan auxotrophs was important in the
elucidation of the biochemical pathway for the synthesis of tryptophan (Figure
9-22).

Gene 2
Genel
Anthranilate

Anthranilate

phosphoribosyl
transferase

synthase

Anthranilate i

Chorismate c

trp E
Gene 5

Gene 4

0-

0-

Enzyme 5

Indole <

> N-(5'-Phosphoribosyl)trP D

Enzyme 4

anthranilate.

trp C\

i Enzyme 3 $3 Gene 3

i Indole-3-glycerol <\
i Enol-1-o-carboxytrpA
phosphate
trpC2
phenylamino1-deoxyribulose
phosphate

Tryptophan

Figure 9-22
The Tryptophan Biosynthetic Pathway.

In Figure 9-22 we see part of the pathway that leads to the synthesis of
tryptophan. We start with chorismate because it has a number of different
Copyright by The Berkeley Review

298

The Berkeley Review


Specializing in MCAT Preparation

Biology

Genetic Information

Genetic Loci & Alleles

biochemical routes that it can take. One is towards the synthesis of tryptophan.
Chorismate is converted toanthranilate by the enzyme anthranilate synthase.
The gene that codes for this enzyme is referred to as trpE. Anthranilate is
converted to a compound called PRA by a transferase enzyme. The gene that
codes for this enzyme is designated as trpD. PRA is converted to a molecule

called CDRP by an isomerase enzyme. The gene that codes for this enzyme is
designated as frpCi. CDRP is converted to a molecule called IGP by a synthase
enzyme. This gene that codes for this enzymeis designated as trpQi. IGP can be
converted to indole by the alpha subunit of the tryptophan synthase enzyme.
Thegene that codes for this enzyme is designated as trpA. Finally, indole can be
converted to tryptophan by the beta subunit of the tryptophan synthase enzyme.
The gene that codes for this enzyme is designated as trpB.

[NOTE: Do not get lost in the names which are involved in this pathway. The
point is to understand what happens to the synthesis of tryptophan if a mutation
were to occur in a gene coding for one of the enzymes in this biochemical
pathway.]

The three mutants that we want to consider in this pathway involve the trpE,
trpA, and frpB genes. If there are mutations in these genes, then the designation
is trpE~> trpA", and trpBT. Consider the entries in Table 9-3. What we would like

to know is whether or not certain types of bacteria will grow if their minimal
medium is supplemented with (a) nothing, (b) anthranilate, (c) indole, or (d)
tryptophan. When we consider this table, we will look at the type or condition of

the bacterium (e.g., trp+), and then follow the row that the bacterium is in, across
and towards the right, as we supplement its medium with the items that we just
mentioned.

Tryptophan Auxotrophs of E. coli


Growth on Minimal Medium Plus:

Type

Nothing

Anthranilate Indole

Tryptophan

Trp+

TrpE"
TrpA'
TrpB"

+
+
+

Table 9-3

Tryptophan auxotrophs.

Note that the trp+ prototroph will grow if nothing is added to the minimal
medium. This is simply by definition (as it is the wild type bacterium). This
bacterium can utilize the components of that minimal medium to synthesize

tryptophan. The trp+ prototroph will still grow if you add either anthranilate,
indole, or even tryptophan itself. This is why we see all pluses (+) in that row.

Consider the trpB" mutant. This bacterium will not grow if nothing, anthranilate,
or indole is added to the minimal medium. This auxotroph will only grow if
tryptophan is added. Thus, as we go from left to right in this row we see three
minus signs (-) and one plus sign (+). What is the explanation for this behavior?
It must mean that the mutation in the trpB gene leads to a defective enzyme that
Copyright by The Berkeley Review

299

The Berkeley Review

Specializing in MCAT Preparation

Biology

Genetic Information

Genetic Loci &Alleles

will not allow indole to be converted to tryptophan. In other words, there is a


block at the level of the trpB gene. Note that anything preceding indole (even if
the enzymes are of the wild type) cannot be converted to tryptophan either.
Why? Because once the products get to indole it is indole that ultimately must be
converted to tryptophan.
Consider the trpE' mutant. This mutant will not grow if nothing is added to the
medium. However, this auxotroph will grow if either anthranilate, indole, or
tryptophan is added. Therefore, as we go from left to right in the table, we see a
minus sign (-) and three plus signs (+). We can use the same analysis as above.
The mutation in the trpE gene leads to a defective enzyme which cannot convert
chorismate acid to anthranilate.

Finally, consider the trpA' mutant. This mutant will not grow if nothing or
anthranilate is added. However, this auxotroph will grow if either indole or
tryptophan is added. Thus, as we go from left to right across the row in this table
we find two minus signs (-) and two plus signs (+). Again, we can use the same
analysis as above. The defective enzyme is a result of a mutation in the trpA
gene. This enzyme cannot convert the precursor of indole to indole. Thus, the
pathway must be blockedat the level of the frpA enzyme.
Epistasis and Pleiotropy

If any one of these genes that is involved in the synthesis of tryptophan is


defective (possibly due to a mutation), then that gene will not be able to produce
the enzyme which converts a particular reactant to a particular product. This
means that tryptophan is not synthesized. In order for the cell to survive it must

be supplemented with this amino acid. Even if two or more of the genes in the
biosynthetic pathway for tryptophan are defective, then the cell will still require
the presence of tryptophan to grow. Therefore, the genes involved in the
biosynthetic pathway of tryptophan are said to act in an epistatic fashion.
Epistasis occurs between different pairs of genes. It does not occur between two
members of an allelic pair. [In other words, two different genes which are not
alleles of oneanother may affect thesame outcome, which, in our example, is the
inability to synthesize tryptophan.] In order for tryptophan to be synthesized, the
dominant allele of all the genes involved in this biosynthetic pathway must be
present (in the absence of tryptophan).

There are other sets ofgenes which may act in an additive fashion. For example,
in yeast there are 7 genes which control the synthesis of the enzyme invertase.
This enzyme hydrolyzes the disaccharide sucrose into its two monosaccharide

constituents, glucose and fructose. If the yeast cell possesses the dominant allele
of any one of these 7 genes, it will be able to ferment sucrose. If the cell carries

more than one dominant allele, it ferments sucrose at a quicker rate. It is likely
thatmanyhuman traits (e.g., height) are controlled by suchgenefamilies.
We mentioned that the dominantgene in Mendel's pea plants expressed a purple
color while the recessive gene expressed a white flower color. As far as it is

known, the gene for white flowers in the pea plant only causes the flowers to be
white. However, there are examples in mice in which a dominant gene causes a
yellow coatcolor. If the mice had one copy of this gene, they had yellow coats. If
the mice had two copies of this gene, it would be lethal for the mice. In this case

not only does the gene affect coat color but another copy of it affects viability.
This is an example of pleiotropy. It is where an individual allele has more than

Copyright by The Berkeley Review

300

The Berkeley Review


Specializing in MCAT Preparation

Biology

Genetic Information

Genetic Loci & Alleles

one effect on the phenotype (e.g., a mouse with a yellow coat or a mouse that is
dead).
Chromosomes

We have mentioned that humans have 23 pairs of chromosomes. Of those 23


pairs of chromosomes 22 pairs appear nearly identical and are called autosomes.
These autosomes are numbered from 1 to 22 and are arranged in order of
decreasing size. This is simply a convention. The size of the chromosomes ranges
from about 200 mb (the largest chromosome) to about 50 mb (the smallest
chromosome). The other pair of chromosomes is the sex chromosomes. The sex
chromosomes are either designated as XX(female) or XY (male).
Let's consider an individual human chromosome such as chromosome 19. This

chromosome contains about 60 x 106 base pairs in length. This is a fairly small
chromosome as far as chromosomes are concerned. The largest human
chromosome is chromosome 1, then chromosome 2, and so on. Chromosomes

can be stained to reveal a banding pattern as shown in Figure 9-23. These


banding patterns are quite specific and they are constant from chromosome to
chromosome. Within these different banding patterns we can assign genetic
markers. The long arm of the chromosome is assigned the letter q while the short
arm is assigned the letter p. Each arm is further divided into regions such as 1, 2,
3, and so on. Each region is divided into band numbers.

band
number

13.3

Insulin receptor gene and


LDLR gene are located

13.2

in this area of the arm

1
region

13.1

j^_

centromere

Excision repair gene


is located in this area
of the arm

1
Chromosome 19

Figure 9-23

There are a variety of known genes on this chromosome. For example, the gene
for low density lipoprotein receptors (LDLR) is located in the p arm while the
gene for excision repair (ERCC1) is located in the q arm. The gene for the insulin
receptor is also located in the p arm.

In one haploid set of 23 chromosomes there are about3 x 109 base pairs of DNA.
Since we are diploid there should be about 6 x 109 base pairs of DNA in every
cell. It has been estimated that there are about 100,000 genes per haploid set of
Copyright by The Berkeley Review

301

The Berkeley Review


Specializing in MCAT Preparation

Biology

Genetic Information

Genetic Loci &Alleles

chromosomes. The size of a gene can range from about 1,000 base pairs up to
about 1,000,000base pairs. The function of only about a thousand out of those
hundred thousand genes is known. In the years to come you probably will be
hearing a lot about the human genome project. This project is a vast undertaking
designed to map the entire human genome. One of the goals of the Human
Genome Project is to find out the function of all of those genes.

Copyright by The Berkeley Review

302

The Berkeley Review


Specializing in MCAT Preparation

Biology

Genetic Information

The Pedigree

The Pedigree
When one established a pedigree a number of different symbols are utilized. For
example, a square represents a male while a circle represents a female. If this

male and female marry, the square and circle are joined by a line. If they have
children, an inverted "T" is dropped to their offspring. Their offspring are
attached to this inverted "T" by short vertical lines. The birth order of the

offspring is arranged from oldest to youngest and runs from left to right.
Separate generations will reside on separate horizontal lines. The most ancestral
generation is always at the top. Each generation is numbered with Roman
numerals. The oldest generation is given the Roman numeral I, the next oldest
generation the Roman numeral II, and so on. If a square or a circle is filled in
with a dark color, then that individual is affected with a given defect. If there is a
slash through the square or circle, that individual is heterozygous for a given
defect. An example of this symbolism is shown in Figure 9-24.
Marriage
Affected

Parents

Female

Male

II 0

( J

Siblings

Heterozygote
Figure 9-24

Recall that when Mendel crossed the Ww genotype from the Fl generation with
itself, he obtained in the F2 generation the genotypes WW, Ww, Ww, and ww.
The flowers with the WW and Ww genotypes were purple while the flowers
with the ww genotype were white. We mentioned that this was the characteristic
3:1 ratio. In human heredity we are able to observe an individual similar to the
genotypes in the F2 generation. We do not know what has happened in the
previous generations (i.e., the Fl generation). What took place in the previous
generations of a human lineage is something that must be deduced. This
deduction is based on the rules for pedigree analysis.
Genetic Diseases:

In our body we have about a hundred trillion cells and in each cell we have
about a hundred thousand genes. In most humans all of those genes in each of
those cells works properly from day to day. It is known that one defective gene
in one of these cells may be enough to cause a cancer. If an individual inherits a
defective gene from one or both of his/her parents, then this may cause a genetic
disease. Today, about 4,000 genetic diseases are recognized.
Faulty chromosome assortment during meiosis may lead to genetic
abnormalities. For example, if there is only one copy of any one of the
chromosomes (called monosomy), the individual will not survive development.
Similarly, three copies (trisomy) of all but chromosomes 13, 15, 18, 21, and 22 is
lethal and individuals who are trisomic for these chromosomes are severely
affected. Trisomy for chromosomes 13,15, and 18 causes severe developmental
Copyright by The Berkeley Review

303

The Berkeley Review


Specializing in MCAT Preparation

Biology

Genetic Information

The Pedigree

problems and such individuals die a few months after birth. Trisomy for
chromosomes 21 or 22 survive to adulthood but are severely affected. The most
common form of trisomy is for chromosome 21. This is Down's syndrome. It is
an example of aneuploidy (that condition in which nuclei have an unbalanced

set of chromosomes-that is, they do not contain an exact multiple of the haploid
number of chromosomes). If the nuclei have a normal complement of
chromosomes, it is referred to as being euploid.

The incidence of Down's syndrome increases dramatically with the age of the
female. For example, the incidence of having a child with trisomy 21 for mothers
who are between the ages of 20 and 30 is about 1 in 1,400. For mothers who over

45years of age the incidence is about 1 in 16 births. All of the eggs that a woman
will ever produce have developed to the level of Prophase I by the time she is
born. Some of these eggs then complete meiosis every menstrual cycle. In other
words, the older the woman is, the older are her eggs that complete Meiosis I
and Meiosis II. It is thought that as time passes there is damage to the eggs and
that interferes with the normal disjunction of chromosome 21.

Nondisjunction of the sex chromosomes can also occur during meiosis. For
example, nondisjunction of the two X chromosomes in females can lead to

gametes that are XX or gametes that do not contain an X chromosome (referred

to as simply O). If the female XX gamete combines with a male Ygamete, the
resulting offspring will be XXY male. This condition is referred to as

Klinefelter's syndrome. If the O gamete from the female combines with an X


gamete from the male, the resulting offspring will be an XO female. This is
referred to as Turner's syndrome.

The many thousands ofgenetic diseases resulting from single gene defects can be
classified into one of the four following traits:
(1)
(2)
(3)
(4)

Autosomal recessive
Autosomal dominant
Sex-linked recessive
Sex-linked dominant

Pedigree analysis allows one to determine which of these four classifications a

given genetic disease falls into. Let's consider some examples of the more
common genetic diseases.

Hemophilia, a blood clotting condition due to a lack of a protein called Factor


VIII, is an example of a sex-linked recessive disease involving the X
chromosome. If a male inherits an allele that has the mutation of Factor VIII, he
will develop hemophilia. Perhaps the most well documented case involves
Queen Victoria of England and her offspring. Queen Victoria had 9 children.

Two of her daughters were carriers while one son was a hemophiliac. Of her 40
children, grandchildren, and great grandchildren, 19 were males and 21 were

females. Ofthose 19 males, 10 (i.e., 10/40 * 25%) developed hemophilia. Note that
those 10 individuals who developed hemophilia were all males. None of the 21
female descendants were affected with hemophilia. This is because it is a
recessive condition.

Consider a sex-linked disease versus an autosomal recessive disease. If a given


disease affects both sexes equally, then it is likely to be autosomal. If males are

affected almost exclusively, the disease is said to be sex-linked. Roughly one


Copyright by The Berkeley Review

304

The Berkeley Review


Specializing in MCAT Preparation

Biology

Genetic Information

The Pedigree

quarter of the children should be affected in either case. Also, both parents may
appear normal.
Consider dominant versus recessive diseases. An individual who is affected with

a dominant genetic trait will have one parent who is also affected. Individuals

affected with a recessive trait may have parents whoboth appear to be normal. It
is possible with a recessive trait to have one parent who is also affected if that
parent is homozygous for the defect.

An example of a hereditary disease that is autosomal dominant is Huntington's


disease. This disease leads to the progressive deterioration of brain cells and
eventual death. The onset of the symptoms of this disease do not usually
manifest themselves until the individual is in mid-life (i.e., 30 to 50 years of age).
A test is available to determine if an individual will get the disease. As you can
imagine, this has created quite an issue for those individuals who think they
might be afflicted with the disease. Is it better to know or not to know?
Autosomal Recessive

Suppose you are a genetic counselor and you are presented with the pedigree
shown in Figure 9-25. In this pedigree there are two couples. This is generation I.
We will number these individuals in these two couples 1,2,3, and 4 (from left to
right). Individuals 1 and 2 are a couple and individuals 3 and 4 are a couple. The
couple on the left have 4 children while the couple on the right have 3 children.
This is generation II. Again, the individuals in generation II can be numbered 1
though 7 (from left to right). One member of each family mates (i.e.,II-4 and II-5)
and have 4 children. This is generation HI. Once again, the children are
numbered 1 though 4 (from left to right).

Zr-r-0

II
12

I
4

0
5

6
7

III

Figure 9-25

Note that the female at II-2 has a genetic disease while the female at III-l and the
male at III-3 have a genetic disease. Is this disease sex-linked or autosomal and is
it dominant or is it recessive? Even though our sample is small we can say that
this disease is autosomal. Why? The disease is affecting both males and females.
This disease is also recessive. Why? The parents of child II-2 are both normal as
are the parents of children III-l and III-3. Thus, the disease is autosomal
recessive.

Copyright by The Berkeley Review

305

The Berkeley Review


Specializing in MCAT Preparation

Biology

Genetic Information

The Pedigree

The next step in analyzing this pedigree would be to figure out the
heterozygotes. Since the disease is autosomal and it is recessive, then the only
way that you could get affectedindividuals would be if both parents are carriers
for the disease.Thus, the parents of the siblings in generation III are both carriers
as are the parentsofsiblings II-l through II-4. What about the parents of siblings
II-5 though II-7? In this case either parent 1-3 or parent 1-4would be the carrier.
It is either one or the other. However, with the information given you do not
know which one is the carrier. [They could both be heterozygotes and still not
have any diseased children just by chance!] Those individuals who are carriers
are indicated in Figure 9-25.

Should the couple 1-3 and 1-4 have any more children? If one of them is
heterozygous, then there should be no chance of them having a child with the
disease. However, the couple 1-1 and 1-2 and the couple II-4 and II-5 should be
advisedthat if theyhavemorechildren, they run the risk ofhaving childrenwith
the disease.

Consider the siblings in generation III. Whatis the chance that siblings III-2 and
III-4 are carriers of this disease? They have a 2/3 chance of being carriers.
Suppose that sibling III-2 matures to adulthood and marries. What is the chance
that her children will be carriers? It would be 1/2.

For many diseases there are tests that can determine if an individual is a carrier

of a given disease. For example, genetic analysis for sickle-cell anemia caneasily
determine if a person has the gene or not for that disease. There is a test for

Huntington's disease that is about 95% accurate. A new development that was
just approved this year by the National Institute of Health is gene therapy. This

type oftherapy is accepted for somatic cell lines but not forgermcell lines. Why?
Consider the sickle-cell gene for a moment. That geneis not present just to be an
annoyance for individuals. It has had a tremendous selective advantage overthe
years in malarial infectedenvironments by giving resistance to malaria to those
individuals who are heterozygotes for sickle-cell anemia. If you were to
introduce a normal gene into the germlineof individuals who are heterozygotes
for sickle-cell anemia, and that gene propagated, then resistance to malaria
would begin to decrease.

Copyright by The Berkeley Review

306

The Berkeley Review


Specializing in MCAT Preparation

BlOlOgy

Genetic Information

Central Dogma of Molecular Biology

Genetic Information
Central Dogma of Molecular Biology
Up until 1944 it was assumed that the carrier of genetic information was
chromosomal proteins. During the late 1940s and early 1950s it was proven that
DNA was the carrier of genetic information (and not proteins as had otherwise
been assumed). Once it was realized that DNA was involved in the genetic of an
organism it was suggested that the flow of information went from DNA to

protein (DNA > Protein). Is the flow of information from DNA to proteins a
direct process?Could information flow in the reverse direction-from proteins to
DNA?

Within a eukaryotic cell there is a defined nucleus and within that nucleus

resides the cell's DNA. Synthesis of proteins, however, occursin the cytoplasm of
the cell. Because proteins are being synthesized in a different cellular
compartment, it was a direct argument that there was not a direct template of
DNA to protein. What is the intermediate step in this process?
Within the cytoplasm are structures called ribosomes. This was the site where
protein synthesis was occurring. Analysis of the ribosome revealed that it was

composed of two different types of biomoleculesribosomal proteins (several


types) and ribosomal RNA (rRNA). Maybe there was an RNA intermediate
between the flow of information from DNA to protein. Where was rRNA
synthesized? Inside the cell's nucleus (and then later exported to the cytoplasm).
Thus, it was imagined that DNA could give rise to a particular type of RNA and
then that RNA could be exported from the nucleus where it would eventually
give rise to proteins. This is almost entirely true but nevertheless somewhat
confusing.
If we look at the analysis of certain proteins like silk, we will find that it is
composed almost entirely of three amino acidsSer, Gly, and Ala. Other proteins
like hair have a very high content of Cys. There are a variety of different proteins
which have quite a variety of different amino acids. However, when we analyze
the base content of the rRNA in ribosomes from a variety of different organisms,
we find that they all have an rRNA base content which is practically identical.
This did not seem right. If we had an organism that synthesized proteins of a
particular type, we would expect to find an intermediate instructional process
that would be different in its base content than we would for an organism with a
very different set of proteins.

Francis Crick suggested that there must be at least two or three different types of
RNA of which rRNA would be only one. His suggestion was taken to heart in
1961by Francois Jacob and Jacques Monod. They postulated that DNA can also
form a type of RNA called messenger RNA (mRNA) which had the instructions
that resulted in the actual protein. They suggested that mRNA is the actual
template for synthesizing proteins and that rRNA is simply a portion of the
architectural framework of the ribosome.
A bacterium does not utilize all of its DNA all the time because it has more

information than it could possibly need at any one given moment. If the food
supply for this bacterium was changed from glucose to lactose, and back and

Copyright by The BerkeleyReview

307

The Berkeley Review


Specializing in MCAT Preparation

Biology

Genetic Information

Central Dogma of Molecular Biology

forth a few times, that bacterium would adapt and change the proteins it was
synthesizing. The mRNA made under one set of conditions will be different from
the mRNA made under a different set of conditions.

For example, when a bacterium is infected by a phage, most of the operations of


the host bacterium cease. In 1962 Jacob, Meselson, and Sydney Brenner were
able to isolate mRNA from a T2 viral phage. No new ribosomes are synthesized.
Thus, no new rRNA is being made by the infected host. However, all of a sudden
the phage DNA will give rise to many proteins that were never seen before the
infection. These proteins must have been synthesized from mRNA that was
made specificallyby the T2 phage DNA. In other words, there is T2 viral specific
mRNA and bacterial specific mRNAandboth of those mRNAs can be translated
on the same ribosome.

What physical properties should mRNA have? One property is for the mRNA to
have a rapid turnover. If the conditions change in the cell, you do not want the
old mRNA making the same old protein anymore. However, rRNA has a rather
long half-life. This is why nobody had discovered mRNA up to this point. It had
a very short half-life and was rapidly turning over in the cell. In fact, only about
4% of the total RNA in the cell is ever mRNA. Almost all of the RNA in a cell is
rRNA.

At the time there was not enough evidence to prove this hypothesis of mRNA
being a template for protein synthesis. Crick saw this and based this article of
faith on what he called a dogma. This lineage of informational flow from DNA
> RNA - Protein soon became the central dogma of molecular biology.
Informational flow from DNA to RNA is called transcription while
informational flow from RNA to protein is called translation. Because DNA can
replicate itself we can draw an arrow on DNA as shown in Figure 9-26.

Figure 9-26

Copyright by The Berkeley Review

308

The Berkeley Review


Specializing in MCAT Preparation

Biology

Genetic Information

Functions of Nucleic Acids

Functions of Nucleic Acids

v S<1

What are the functions of these nucleic acids? Fred Griffith (circa 1928) was
looking at infections caused by the pneumococcus bacterium. This bacterium can
cause pneumonia in mice. The pneumococcus comes in two varieties-the normal

house and garden variety that is surrounded by a smooth (S) polysaccharide


capsule and the mutant variety that does not have this capsule and appears
rather rough (R). The smooth pneumococcus kills mice while the rough
pneumococcus do not kill mice. If Griffith heat-killed the S form of the
pneumococci and then injected the mice with it, the mice lived. However, if he
made a solution of the heat-killed S form mixed with the live R form, and then
injected that into the mice, they would die. Somehow the heat-killed S form of
the pneumococci transformed the live R form into the lethal S form that is able to
kill mice.

It was not until the mid-1940's that this phenomenon was understood. Oswald
Avery, Colin MacLeod, and Maclyn McCarty published a landmark paper based
on Griffith's findings that stated that DNA was the transforming factor (genetic
material) and not protein as others had believed.
In the intervening years it was suggested by Roger Herriot that a bacteriophage
like the T2 virus is actually a core of DNA surrounded by a protein coat and that
the virus somehow injects its DNA into a host cell causing that cell to be
transformed to produce more phage progeny. This hypothesis was tested by
Alfred Hershey and Martha Chase in 1952. Their findings turned out to be yet
another confirmation that DNA is indeed the genetic material.

Bacteriophage
Protein

Coat CP

Figure 9-27

The T2 bacteriophage consists of a head which carries its DNA. Surrounding the
head is a protein coat. Stemming off of the head is a tail with fibers that have the
ability to recognize specific markers on a given host, such as the bacterium E. coli.
This is shown in Figure 9-27. Once the phage attaches to its host it then injects its
DNA into that host via its tail mechanism (which acts like a hypodermic needle).

At the time that Hershey and Chase were doing this experiment they were not
sure which portion of the phage was being injected into the host cell. Was it the
protein from the protein coat or was it really the DNA? Hershey and Chase had

access to various radioactive isotopes, including 32P and 35S. Other radioactive
isotopes include 14C and 3H. Another non-radioactive isotope is 15N. They
reasoned that thephage DNA could be easily labeled with 32P is because ofthe
phosphate atom in the DNA backbone. DNA does not contain any measurable
amount of sulfur. They also knew that proteins were composed of amino acids
and two amino acids had sulfur atoms (Cys and Met). Amino acids do not
contain any measurable amount of phosphate in their structures. Therefore, they

could have proteins labeled with35S.


A culture of E. coli bacteria were infected with phage that had been labeled with

32P and 35S. After a brief period of time the infected solution was agitated in a
Waring Blendor in order to separate the phage from the bacterium. The solution
was centrifuged into a supernatant and a pellet. The bacteriawere collected from

the pellet and analyzed for either 32P or 35S. It was discovered that thebacteria
contained a high percentage of 32P while the phage protein in the supernatant
contained a high percentage of 35S. This was a conformation of the Avery
experiment that DNA was the genetic material.
Copyright by The Berkeley Review

309

The Berkeley Review


Specializing in MCAT Preparation

Biology

Genetic Information

Functions of Nucleic Acids

James Watson and Francis Crick deduced the structure of DNA based on an x-

ray diffraction photograph taken of DNA fibers by Rosalind Franklin and


Maurice Wilkins during the early 1950's. Watson and Crick proposed that DNA
was a double helical molecule. (Linus Pauling, who was also working on DNA at
the time, proposed that it was a triple helix.) The base pairing scheme that
Watson and Crick proposed, adenine (a purine) pairs with thymine (a
pyrimidine) while guanine (a purine) pairs with cytosine (a pyrimidine), was
supported by evidence from earlier studies by Erwin Chargaff in which he found
that adenine and mymine ratios and guanine and cytosine ratios were almost 1:1
in all of the organisms that he examined.

The base pairing of adenine with thymine and guanine with cytosine occurs
through hydrogen bonding. Adenine and thymine have two hydrogen bonds
between them while guanine and cytosine have three hydrogen bonds. Note that
adenine hydrogen bonds via its 1 and 6 positions while mymine hydrogen bonds
via its 3 and 4 positions. This is shown in Figure 9-28a. Similarly, guanine
hydrogen bonds via its 2,1, and 6 positions while cytosine hydrogen bonds via

its 2,3, and 4 positions. Thisis shown in Figure 9-28b. Not only do the hydrogen
bonds between the base pairs help to hold the DNA double helix together, but
the stacking of the bases also plays a major role due to the interaction of the nelectron clouds between juxtaposed bases.

\-H

Pi

0.

-H N 3 ,^)
\2 ' /

^,

Backbone

' ..

Backbone

Adenine

fj

CH,

Thymine

H-N<

fjVs.-H
|

Backbone

N-H
I

1 ,,

Backbone

Guanine

(a)

CH,

Cytosine
(b)

Figure 9-28

These bases are said to be complementary to one another. Not only is there
complementaritybetween specific bases but the two DNA strands that comprise
the double helix run antiparallel to one another. In other words, one DNA strand
will run in the 51 - 3' direction while the other strand will run in the 3* -> 51

direction. As the DNA double helix winds around an imaginary axis, two
grooves are formed. These two grooves are called the major groove and the
minor groove. These grooves (as we will later see) make it much easier for
certain proteins to bind specific sequences in the DNA duplex. This can be seen
in the simplified DNA double helix shown in Figure 9-29.
Major Groove

Minor Groove

it

&

3' 5'

5' 3'

Figure 9-29

Copyright by The Berkeley Review

310

The Berkeley Review


Specializing in MCAT Preparation

Biology

Genetic Information

Functions of Nucleic Acids

How can we denature a double stranded (duplex) molecule ofDNA? One way
would be to add a denaturing agent. Recall that when urea was added to
proteins, it denatured those proteins. One way to denature DNA would be to

change the pH of the solution or even heat the solution. These procedures will
first denature the hydrogen bonds in the AT rich regions ofthe duplex DNA and
then in the GC rich regions. Why? Because adenine and thymine are held
together by only two hydrogen bonds whereas guanine and cytosine are held
together by three hydrogen bonds. If we just denatured the AT rich region of a
DNA double helix, it would be a partial denaturation. If we continue the

denaturation process, we canget complete denaturation of theDNA duplex.


Suppose we have a partially denatured piece of DNA and remove the heat or

adjust the pH backto normal. Whatwillhappen? Onefinds that the DNA duplex
joins back together-a process called renaturation or annealing. In fact, it turns
out that DNA is constantly denaturing a few hydrogen bonds and then
reforming those same hydrogen bonds again~a process called breathing. In
other words, DNA is a dynamic structure and not a rigid crystal.
If we were to heat DNA slowly, we would be able to look for a melting
temperature (Tm). The Tm is that temperature at which half of the helical
structure of the DNA is lost. As we begin to slowly heat DNA there will be a
gradual dissociation from double stranded DNA to single stranded DNA. If the

DNA double helix is rich in GC base pairs, it will have a higher Tm value than
DNA with an abundance of AT base pairs. This can be seen in Figure 9-30.

How is this phenomenon detected? One way would be by viscosity separation.

Another way would be by measuring the relative absorbance of the DNA. If you
have a structure such as DNA that is held together by hydrogen bonds between
the base pairs, it should be possible to break those hydrogens by increasing the
temperature and allow the two strands to separate. This is called reversible
melting and can be observed by following the absorbance properties (at 260 nm)
of the solution in which the melt is taking place. We can follow the absorbance of
this melt because of the different absorption properties between double stranded
DNA and single stranded DNA. Double stranded DNA will have a lower
absorbance (by about 40 to 50%) than single stranded DNA.

AT rich

Single
(0

Stranded

DNA

Normal

"

GC rich

o
Vi

>

tj

Double
Stranded

/
/

DNA^y^.
Tm

Temperature
Figure 9-30

Copyright by The Berkeley Review

311

The Berkeley Review


Specializing in MCAT Preparation

Biology

Genetic Information

Functions of Nucleic Acids

Why is there a change in the absorbance between double stranded DNA and
single stranded DNA? The stacking of the bases in the DNA double helix is quite
important. This close association of the bases brings about a phenomenon called
hypochromicity. Hypochromicity simply means that the absorption of the total
molecule is less than the sum of its parts. In other words, each of the base pairs
has its own particular absorption spectra, and when we allow all these base pairs

to form the double helix we do not realize their full absorptive capability.
When light impinges on these stacked base pairs in the DNA double helix there
is a light induced electronic transition. In other words, certain electrons will be
sent to higher energy levels. Whenever there is an electronic transition, there is

polarization. Once one of the base pairs becomes polarized, the ability for the an
adjacent base pair to become polarized is diminished. Thus, when the bases are
tightly stacked, as in the double helix, there is a decreased absorption. This is the
basis for this phenomenon of hypochromicity. If you heat the DNA double helix
and break the hydrogen bonds between the bases, you will get an increase in
absorptionan effect called hyper-chromicity.
The Tm of DNA naturally depends on its base composition. We can talk about
that base composition in terms of the percentage of GC base pairs in the DNA
duplex of interest. Most mammalian DNAs are about 50% GC. The GC content
of the phage T2 is about 35% while the GC content of the bacterium E. coli is
about 50%. If this phage were to infect an E. coli bacterium, one could determine
which DNA in the bacteriumis actually the bacteria's own DNA and which is the
phage's DNA simply by the difference in GC content.

Suppose you were to takea piece of DNA and denature it for awhile by adding
heat. At somepoint you stop this process and begin to reduce the temperature.
Would the DNA rehybridize? The answer is yes. If you cool the DNA quickly,
you will find that some portions of the single stranded DNA will find each other
and reanneal. However, there will still be a great deal of single stranded DNA
that did not renature. If you cool the DNA slowly, then there will be more time
for the single strands ofDNA to find theirpartnersand joinbacktogether again.
This is shownin Figure 9-32. Themorecomplex the DNA, the lessperfectwillbe
the renaturation process.
Single
o
(J

^v

e
o
Vi

#>

~**5N.

f
I

DNA

\V_

Double /
Stranded /
DNA,/

Cool
Quickly

Cool
Slowly

Stop

Increase T

Decrease T

Figure 9-32

Once DNA was proposed to be double helical in nature, the question of


replication immediately surfaced. How is it that DNA can replicate itself? There
Copyright by The Berkeley Review

312

The Berkeley Review


Specializing in MCAT Preparation

Biology

Genetic Information

Functions of Nucleic Acids

are several possible modes. Two of them are the conservative mode and the
semiconservative mode as shown in Figure9-33.

Parental
DNA

fn.

Daughter

Vs

DNA

Conservative

Semiconservative

(a)

(b)

Figure 9-33

In the conservative mode of DNA replication the parental strands remain


together after replication oftwonewdaughter strands. This isshown in Figure 933a. In the semiconservative mode of replication (proposed by Watson and
Crick) the two parental strands separate and two new daughter strands are
synthesized off of the respective parental template. The parentalstrands do not
rejoin but rather remain with the newly synthesized daughter strands. This is
shown in Figure 9-33b.

It turns out that DNA is replicated semiconservatively, as Watson and Crick had
hypothesized. This was proven by Matthew Meselson and Franklin Stahl in

1957. Meselson and Stahl grew the bacterium E. coli in an ordinary medium

containing 14NH4C1. The nitrogen in this compound, 14N, is the regular house
and garden variety nitrogen. They also grew a different batch of E. coli in a
medium with heavy nitrogen, "N. When new DNA is synthesized these

nitrogen atoms (14N or115N) will be incorporated into the structure of that
molecule. The light DNA (containing 14N) could be separated from the heavy
DNA (containing 15N) by the use ofdensity gradient equilibrium sedimentation.
Agradient ranging from 1.66 to1.76 g/cm3 was established ina concentration of
cesium chloride (CsCl). The density of the house and garden variety DNA is

about 1.7 g/cm3. Two bands were clearly visible as shown in Figure 9-35a. This
was the starting point for their experiment.

Meselson and Stahl grew some E. coli a medium with15N for many generations.
The DNA in these bacteria thus contained 15N in their DNA. Once they had a
batch of E. coli will all 15N in their DNA they started the experiment. At time
zero they switched the 15N to14N in the growth medium (see Figure 9-34).
Copyright by The Berkeley Review

313

The Berkeley Review

Specializing in MCAT Preparation

Biology

Functions of Nucleic Acids

Genetic Information

E. coli
15

nO
114N

14

Ny/ ^(^J) (^_J) Generation

/
\
oooo
14XT/15XT 14XT 14XT 14,T/15XT
N/

N/

Figure 9-34

Now, in each subsequent generation the E. coli would be utilizing 14N instead of
15N. Analysis of the DNA after one generation of growth (E. coli replication)

revealed a hybrid DNA band(14N/15N) between that of the 15N DNA band and
the 14N DNA band. This is shown in Figure 9-35b and implies a
semiconservative mode of DNA replication. If the mode of DNA replication has
been conservative you would still expect to see two bands just like that shown in
Figure 9-35a. Why? Because the heavy band would be from the parental strands
while the light band would be from the daughter strands (see Figure 9-33a).

,4N

14 N

l4N/15N

14n/,5n

l5N

(b)

(a)

(c)

Figure 9-35

What would the bands look like after two generations of E. coli replication?

Again, if this were a conservative mechanism, you would still see two bands.

One would be heavy (15N) and the other would be light (14N), except that now
there would be three times as much 14N as 15N. If the mode of replication were

semiconservative, then you would find one all14N band and one hybrid 15N/14N
band as shown in Figure 9-35c. Thus, we find that the mode of DNA replication
is indeed semiconservative.

Copyright by The Berkeley Review

314

The Berkeley Review

Specializing in MCAT Preparation

Biology

Genetic Information

DNA Polymerase

DNA Synthesis
NA Polymerase
DNA replicates semiconservatively. In 1955 Arthur Kornberg and his colleagues

incubated extracts of E. coli with radioactively labeled deoxyribonucleoside

triphosphates at the a-phosphate position. They discovered that a very small


amount of nascent DNA was synthesized. The enzyme that catalyzes this
reaction was called DNA polymerase I (since it was the first such enzyme
discovered).

What is a deoxyribonucleoside triphosphate? Let's consider the structure of

deoxythymidine as shown in Figure 9-36a. There area few important features to


note about this molecule, so we will consider them one at a time. The sugar in
DNA is deoxyribose, and in particular it is (3-D-2'-Deoxyribose. The "deoxy"
term simply means that the ring structure lacks an oxygen atom at the 2'-carbon.

The C-l carbon ofthe ribose ring isbonded to either the N-9 nitrogen ofa purine
base or the N-1 nitrogen of a pyrimidine base by an N-glycosidic linkage. This
linkage was formed via a dehydration reaction which means that it can be

hydrolyzed. In the case of deoxythymidine it is bonded to the N-1 nitrogen of the


nitrogenous base thymine. This structural unit, the ribose ring bonded to either a
purine or a pyrimidine, is called a nucleoside (note the "side" ending). In DNA

there are four nucleosides: deoxyadenosine, deoxyguanosine, deoxythymidine,


and deoxycytidine.
(a)

0
Thymine

(l>)

II

.1

X-CHi

i
o=c

II

Linkage

!!

Y(<?
0
,0
/

C-CHi

H-N

II

.C-H

.C-H

o=c

HO

-,CH2

Acid Anhydride

.0

O P0

5'

N-glycosidic

^3 linkage

O
p-D-2'-Deoxyribose

3' 2/ h

Phosphomonoester
Linkage

HO

Deoxythymidine
(a nucleoside)

3' 2'/ h
HO

Deoxythymidine-5'-triphosphate
(a nucleotide which can be abbreviated as dTTP)

Figure 9-36

If we add a phosphate group to the 5'-hydroxyl function of the ribose ring of any
of our nucleosides, we will form a phosphate monoester bond (Figure 9-36b).
The name of our molecule now changes from a nucleoside to a nucleotide (note
the "tide" ending). Be careful of what you mean when you use the "side" and the
"tide" endings. For example, we could name one of these molecules as a
deoxyribonucleoside-5'-monophosphate (or as a deoxyribonucleotide). There
could also be two or three phosphate groups attached to the 5'-carbon of the

ribose ring. Recall from organic chemistry that two phosphate groups are
attached to each other by a phosphoanhydride linkage. If we added two
phosphates to our molecule, we would call it a deoxyribonucleoside-5'diphosphate. If we added three phosphates to our molecule, we would call it a
Copyright by The Berkeley Review

315

The Berkeley Review

Specializing in MCAT Preparation

Biology

Genetic Information

DNA Polymerase

deoxyribonucleoside-5'-triphosphate. This can beabbreviated as dNTP. The four


deoxyribonucleoside triphosphates (dNTPs) of interest to us are dATP, dGTP,
dTTP, and dCTP.

DNA Polymerase I adds about 20 deoxyribonucleotide residues (e.g., dATP,


dGTP, dTTP, or dCTP to the 3Vhydroxyl function of a pre-existing DNA strand.
What this means is that a primer is needed in order for DNA polymerase I to
function. The dNTPs are called for by a DNA template and are added to the

newly synthesized DNA chain in 5' - 3' direction (which means the DNA
template is read in the 3' - 5' direction). The incoming dNTP will hydrogen
bond with its complementary base and then there will be a nucleophilic attackby
the 3'-hydroxyl of the primer strand on the a-phosphate of the incoming dNTP.
A phosphodiester bridge is formed between the a-phosphate of the incoming
dNTP and the 3'-hydroxyl function of the ribose ring of the primer strand.
Pyrophosphate is released and subsequently hydrolyzed in order to help drive
this reaction to completion. This is shown in Figure 9-37. Note that the
phosphodiester bond will only be formed if the incoming dNTP is
complementary to its respective base on the template strand.

Base

BaseBase

Base'

Base

Base-

X C oo' OP=0
Base'

Base-

Figure 9-37

Copyright by The Berkeley Review

316

The Berkeley Review

Specializing in MCAT Preparation

Biology

Genetic Information

DNA Replication

a^lllffllil)iioi;
DNA is composed of two polydeoxynucleotide strands arranged in an
antiparallel fashion. DNA is usually a right-handed helix with the purine and
pyrimidine bases arranged on the inside of the helix while the deoxyribose and
phosphate moieties are arranged on the outside of the helix. The DNA double

helix can be represented as shown in Figure 9-38a or in Figure 9-38b. Both


representations will give us similar information.

Double

J Straidd
DNA

'A. Miner

Sr^ Groove
Major

-bP-bp-bp-bp-bp-bp-bp-

bp = base pairs
(A=TorG=C)

Groove

Single
Sugar-

<^~I Phosphate [

'Z3 Straided
DNA

Backbone
3'

(a)

(b)

Figure 9-38

Agreat dealofstructural information about theDNA double helix canbe gained


from X-ray analysis. For example, there are two forms of DNA which can be in a

right-handed helix. They are the A-DNA and the B-DNA (which was proposed
by Watson and Crick in 1953). Recall that we have mentioned that the B-DNA

contains a major and a minor groove (see Figure9-38a). Proteins that recognize
specific DNA sequences can gain access to the DNA double helix via the major
groove.

There are a number of bonds in the DNA polymer about which rotation can
occur. For example, the backbone of the DNA polymer can be rotated about 6
bonds in each monomeric unit. Many of the differences between the A-DNA and
the B-DNA arise from the differentconformations of the ribose ring.
Recall that the ribose ring in DNA can also be called p-D-2-Deoxyribofuranose.
These furanose rings are not planar. They can pucker. In the A-DNA the minor

groove essentially vanishes due to s specific puckering of the furanose ring. This
results in the base pairs of the A-DNA helix being tilted away from the
perpendicular axis by about 19 degrees. However, because of a different type of
puckering, the B-DNA has its bases arranged perpendicular to the axis of the
helix, thus allowing for the distinct characteristics of the major and minor
grooves as shown in Figure 9-38a.

The type of helix that is found in A-DNA is also found in regions of double
stranded RNA (e.g., involving hairpins) and in RNA-DNA hybrids.

Copyright by The Berkeley Review

317

The Berkeley Review

Specializing in MCAT Preparation

Biology

DNA Replication

Genetic Information

NH,

NH2

Adenine N

\\

Adenine ^ ll -N

N-Glycosidic
0CHj

Bond

O CH,

:QSyn Conformation

Anti Conformation

Left Handed Helix, Z-DNA

Right Handed Helix

Purine is Syn, Pyrimidine is Anti

Either A-DNA or B-DNA

(b)

(a)

Figure 9-39

A third type of DNA helix involves rotation about the N-glycosidic bond that
connects the furanose ring to the base. This type of DNA is called Z-DNA
(because the phosphates in the backbone zigzag due to a repeating dinucleotide
unit rather than a mononucleotide unit). Z-DNA is a left-handed helix and

when the pyrimidine bases and the ribose units are far apart it is in an anti
conformation while it is in a syn conformation when the purine bases and the
ribose units are close together. This is shown in Figure 9-39a. The N-glycosidic
bonds in both A-DNA and B-DNA are in the anti conformation. This is shown in

Figure 9-39b.

DNA does not have to be linear, rather it can be covalently joined at its ends
forming a circular structure. For example, the DNA found in mitochondria and
in the chloroplasts of plant cells is circular. The topology of both linear and
circular DNA is rather interesting. DNA is typically right handed. If we were to
twist this DNA molecule around its own axis in the right-handed direction, then
we would introduce into that double helical structure a phenomenon known as
positive supercoiling. Conversely, if we were to twist this DNA molecule
around its axis in the left-handed direction, then we would introduce negative

supercoiling. Supercoilinggreatly changes the overall form of the DNA not only
by making it more compact but also by altering accessibility to the major and
minor grooves.
The number of times that one DNA strand can be wound around another DNA

strand is referred to as its linking number (L). Topoisomers are DNA molecules
that differ only in their linking number. The degree of the linking number in
DNA can be altered by enzymes called topoisomerases. Type I topoisomerases
reversibly cleave one strand of DNA and relaxes negatively supercoiled DNA

while Type II topoisomerases (e.g., DNAgyrase) reversibly cleaves both strands


of DNA and adds negative supercoils.

In order to introduce a supercoil into a DNA double helix it costs energy. For

example, DNA gyrase is a topoisomerase (Type II) found in E. coli which


converts the potential energy of ATP into the torsional energy of a negatively
supercoiled structure. It turns out that negatively supercoiled DNAprepares the
Copyright by The Berkeley Review

318

The Berkeley Review

Specializing in MCAT Preparation

Biology

Genetic Information

DNA Replication

DNA duplex for processes such as replication, recombination, and transcription

where the separation of the two helical strands is required.

In 1958 Arthur Kornberg discovered the enzyme DNA polymerase Iwhich plays

a crucial role in the replication and repair ofDNA. This enzyme adds about 20
deoxyribonucleotide residues (e.g., dATP, dGTP, dTTP, or dCTP (which
collectively can be called dNTPs)) to the S'-hydroxyl function of a pre-existing
DNA strand.

Primer

n
Base

Primer

I
0

Base-

2Pj

H2C

Base

Base-

Base

Base.

t
PPi

OP=0

I
o

Base

Base-

H2C
o

HO

Figure 9-40

What this means is that a primer is needed in order for DNA polymerase I to
function. The dNTP's are called for by a DNA template and are added to the
newlysynthesized DNA chain in 5' -> 3' direction (which means that the DNA
template is read in the 31 -> 5' direction).

The incoming dNTP will hydrogen bond with its complementary base and then
there will bea nucleophilic attack by the 3f-hydroxyl ofthe primer strand on the
a-phosphate of the incoming dNTP. A phosphodiester bridge isformed between
the a-phosphate ofthe incoming dNTP andthe3'-hydroxyl function ofthe ribose
ring of the primer strand. Pyrophosphate is released and subsequently
hydrolyzed in order to help drive this reaction to completion. This is shown in
Figure 4-40. Note that the phosphodiester bond will only be formed if the
incoming dNTP iscomplementary toitsrespective base onthe template strand.
Polymerase Chain Reaction

A laboratory procedure called the polymerase chain reaction (PCR) is having a


significant impact on what can be done with DNA. Suppose we have a relatively
longpiece of double stranded DNA as shown in Figure 9-41. Withinthis DNA is

a portion that we would like to examine. However, we do not have very much of
this DNA available to us. The idea is to amplify the segment of DNA we are
interested in so we can study it.

Copyright by The Berkeley Review

319

The Berkeley Review

Specializing in MCAT Preparation

Biology

DNA Replication

Genetic Information

Figure 9-41

The concept of the polymerase chain reaction would only have been possible
after the discovery of one of the organisms that grows in hot springs. These
organisms (e.g., bacteria, fungi, algae) are thermophilicthey love heat. In
particular, there is a bacterium called Thermoaquaticus (Taq) which has a DNA
polymerase referred to as the Taq DNA polymerase. This polymerase is stable at
temperatures up to 90 "C and does not denature at that temperature. With this
polymerase in mind, let's amplify the DNA in Figure 9-41.

Suppose we know some base sequences as outlined in Figure 9-42a. If we were to


somehow denature that area of the DNA, we could manufacture complementary
base sequences with a free 3'-OH function at the end. These complementary
sequences are made such that their 3'-OH functions point inward. In other
words, we can synthesize a primer as shown in Figure 9-42b.

How can we do this? If we heat this DNA to about 90 "C, the ends will separate
first. We can then add the short chain DNA primers that are complementary to

the base sequences of that portion of the DNA that we already know. If we cool
the solution to about 50 C,our primers will hydrogen bond to their complement
areas as shown in Figure 9-42b.

We want to examine this area of the DNA.


-C-T-T-C-

T-A-C-T-

-G-A-A-G-

A-T-G-A-

I Heatand cool in the

(a)

I presence ofexcess
1 artifical primers
T-A-C-T-

3' -C-T-T-C
i

5'

-G-A-A-G-OH3'
Primer

Primer
3'HO--T-A-C-T-

III!,,
A-T-G-A- 3'

5* -G-A-A-G.

(b)

Figure 9-42

Once we have our primers in place we can then add dNTP's and the Taq DNA
polymerase. This enzyme will add the dNTP's as shown in Figure 9-43.
Copyright by The Berkeley Review

320

The Berkeley Review


Specializing in MCAT Preparation

Biology

Genetic Information

DNA Replication

Remember, the Taq DNA polymerase isstable athigh temperatures (around 70


6C). Bythis time we have twice as much DNA as we did before.
3' -C-T-T-C

T-A-C-T- 5'

I
I I I
G-A-A-G-OH3'
Primer

Primer

3'HO--T-A-C-TI I I I
A-T-G-A- 3'

5' -G-A-A-G.

Taq DNA Polymerase


dNTP's

3'

^T

T-A-C-T- 5'

GAAG^v^xxxxxxvxvvvkxx>^vvxxxvxxvvvvxxxxvxxxxxv^^3
Primer

New DNA v

a. New DNA
,^wxvv>x*xvvvv^vvvwx>^vvv^

Primer
T ACT-

5' -G-A-A-G

A-T-G-A- 3'

Figure 9-43

Butwe still want to amplify our DNA in order to get a better yield. How do we
amplify the DNA that we have produced in Figure 9-43? We heat the DNA to 90
"C in the presence of our primers (which we have in excess). The DNA strands
will separate and the primers will hydrogen bond as before. We now have four
single strands of DNA with their respective primers. Thisis shown in Figure 944. These four strands correspond to their respective strandsfrom Figure 9-43.
3' -C-T-T-C
5

Original DNA

T-A-C-T-

5'

G-A-A-G-OH3'
Primer

*, /-. * a r>
DNA just Synthesized
5 -G A AGv\xvvvvvxvvx>xxxxvvvwvxxvxwx>w

Av T Gw A * 3'
3' HO

-T-A-C-T- 5'
Primer

*>%

~.

DNA just Synthesized

5' -G-A-A-G-OH3'
Primer

5' -G-A-A-G-

Original DNA

A-T-G-A- 3'

3'HO--T-A-C-T- 5'
Primer

Figure 9-44

Taq DNA polymerase in the presence of the dNTP's will once again synthesize
new DNA. This is shown in Figure 9-45. At this point we have now made four
Copyright by The Berkeley Review

321

The Berkeley Review

Specializing in MCAT Preparation

Biology

DMA Replication

Genetic Information

times as much DNA. This processcan be continued indefinitely to amplify the


DNA.

3' -C-T-T-C

T-A-C-T- 5'

5' -gaA-G'"""'"''1"'11'1"111"111'1"111111'1111111111111111"

3'

5' -G A AG*^vax\x\\\\x\\\vxvvxxxx\xvx\\x\xvvvxv

3'

3' l||||j|lllllIIIIIIIIHMHIHIIKIIIIIIIIIIIIIII!IIIIIItHHMIIIIIIIIIIIIIIIIIIIIIIIIIIIIIHIIIIIIIIIIIIIIHIIIIIIIIIIIII!IIIIIIlTACT- 5
3' x'CT T^CvvxxxxxxxxxxxxxxxxxxxxxxxxxxxxxxxxxxxxxxxxxxxxxxxxxvvTACT- 5'

5' G^ A AOii"<tiii"i"""t">">>i'<ti"i>t"iHiHii[iiiiiiiiMiiuiitii[iuitiinMiutiiiitiiiitiiiiiiiiititii[iniiiiiiitiniiiiill|[lli> 3'


5' -G-A-A-G

A-T-

3' *I(II[lltiail,t[iattlit>>>tlIIt>>f[1>>t>>ltc,>tI,>tt>,II,t[>>tI>t[>*t>ttIlt>lt>lt>>t>tlltl>t>tc>t>lt>>III>tllt>tc>IltllIICIlt>(>c>>tl'T'-

-r3'

-T- 5'

Figure 9-45

One of the problems with this process is that Taq DNA polymerase does not
have 100% fidelity. Once in awhile an incorrect bases will be added. However,
the number of errors will be quite small compared to the quantity of DNA that
can be obtained. A general temperature cycle for the polymerase chain reaction is
shown in Figure 9-46.
Condition for

denaturation

Taq likes towork at


this temperature

R
H
a

92

92

4,y

7oy

<D

Q.

E
,1>

\507

\ 1

Annealing of primers

Time (minutes)

Figure 9-46

What are some of the applications of the polymerase chain reaction? One

application involves analysis of the DNA from organisms which have been
deceased for a long time. For example, the tissue of ancient Egyptian mummies
can be analyzed for their DNA content as well as ancient bones from various
burial mounds throughout the world. DNA from these organisms can then be
compared with the DNA from modern organisms can to see if there has been a
significant change (evolution).
Unwinding of Supercoiled Parental DNA

The synthesis of new daughter DNA is coupled to the unwinding of parental


DNA at sites called replication forks. DNA replication does not begin randomly
but rather at sites referred to as an origin (or ori for short). For example, in E. coli
DNA replication will begin at a locus called ori C. In order for the DNA double
Copyright by The Berkeley Review

322

The Berkeley Review


Specializing in MCAT Preparation

Biology

Genetic Information

DNA Replication

helix to unwind, DNA gyrase adds negative supercoils ahead of the advancing

replication fork. This isshown in Figure 9-47a. (Why? As the replication fork is
initially unwound, positive supercoils are introduced thus making DNA
separation rather difficult.)
(a)
DNA

DNA Gyrase adds

Gyrase

'negative supercoils

Origin of

Helicase unwinds

gReplication

DNA (ATP-driven)

Fork

Template

3'

3'

5'

DNA single-stranded
binding proteins
Figure 9-47

Once thenegative supercoiling is introduced, a protein complex called a helicase


binds to the ori site and catalyzes the ATP-driven unwinding of the duplex
DNA. Single-stranded binding proteins then stabilize the unwound portion of
the parental DNA. This is shown in Figure 9-47b.

When a replication forkhas been created, both strands of the parentalDNAserve


as templates for the synthesis of new daughter DNA. Because DNA is replicated
semiconservatively (proposed by Watson and Crick in 1953 and proved by
Meselson and Stahl in 1957), and because the DNA strands are antiparallel, the
synthesis of new daughter DNA must be in the 51 -> 3* direction for one strand

but in the 3' -> 5' directionfor the other strand. Synthesis of one daughter strand
in the 3' - 5' direction seems to be a paradox because of the fact that DNA
polymerase will only synthesize new DNA in the 5' -> 31 direction. This is shown
in Figure 9-48.
5'

3'

Parental
DNA

Replication
Fork

3' ' /

X ^ 5'

5'

3'

"Appears" to grow
3'- 5'

Figure 9-48

Copyright by The Berkeley Review

323

The Berkeley Review

Specializing in MCAT Preparation

Biology

DNA Replication

Genetic Information

Continuous and Discontinuous DNA Synthesis

This problem of one daughter strand of DNA paradoxically growing in the 3' ->
5' direction was resolved by Reiji Okazaki (circa 1969).
5'

3'

As the replication fork proceeds in one direction, one daughter strand is


synthesized continuously in the 51 -> 3' direction. This daughter strand is called
the leading strand. The other daughter strand is called the lagging strand
because it is synthesized discontinuously. Even though the lagging strand is
synthesized discontinuously, it is synthesized in the 5* -> 3' direction (as shown
in Figure 9-49).
Primases
3'

Lagging
Strand

Strand

(Continuous)

(Discontinuous)

How is the lagging strand synthesized? When the DNA template has been
exposed at the replication fork, nascent DNA cannot be made until a primer is
synthesized.

Figure 9-49
DNA

Gyrase

Helicase

Growing
Replication
Fork

Primosome

(Primase)

Parental
DNA

Figure 9-50

An enzyme called a primase (complexed to other polypeptides to form an


aggregate called a primosome), is a specific RNA polymerase that synthesizes
about 5 nucleotides of RNA which are complementary to one of the DNA
templates. DNA polymerase cannot start the synthesis of nascent DNA (we will
come back to this point later). This is shown in Figure 9-50.
DNA Polymerase ID Holoenzyme

Once the primer has been created, the enzyme DNA polymerase III holoenzyme
enters the scene and catalyzes the synthesis of about a thousand phosphodiester
bonds (i.e., about one thousand dNTP's have been used) before it dissociates
from the DNA template.

Why is DNA polymerase III used instead of DNA polymerase I? DNA


polymerase III is quicker and it can add more dNTP's to the growing DNA
strand. Recall that we mentioned that DNA polymerase I only adds about 20
dNTP's to a pre-existing DNA chain.
This segment of RNA primer and newly synthesized DNA is called an Okazaki
fragment. As more Okazaki fragments are synthesized in the 5' > 31 direction, it
Copyright by The Berkeley Review

324

The Berkeley Review


Specializing in MCAT Preparation

Biology

Genetic Information

DNA Replication

eventually leads to overall growth of the daughter DNAin the 3' -> 5' direction.
This is shown in Figure 9-51.

5'

3'

/*" "A DNA

Growing
Replication

Gyrase

Fork

Helicase
Primosome

(Primase)

DNA Polymerase III


Holoenzyme ""

Parental
NA

Figure 9-51

DNA Polymerase I and DNA Ligase

DNA polymerase I has a5'^3' exonuclease activity that allows it to remove the
short segments of RNA primer (starting from the free end). When the primer has
been removed, DNA polymerase I then adds deoxyribonucleotides to the free 3'hydroxyl function of the chain undergoing elongation.

|
Growing

Replication

5'
("

3'
)J?NA

^Gyrase
DNA Polymerase I uses its
5' to 3'

exonuclease activity
to remove RNA primer

DNA Ligase joins'


the fragments.

Figure 9-52

Once the gap that was once occupied by the RNA primer is filled with dNTPs,
DNA ligase will join the free 3'-hydroxyl group of new daughter DNA just
synthesized by DNA polymerase I with the 5'-phospltate group of the Okazaki
fragment just ahead of it. Remember, DNA ligase only joins chains of DNA that
Copyright by The Berkeley Review

325

The Berkeley Review


Specializing in MCAT Preparation

Biology

DNA Replication

Genetic Information

are double-stranded and not two molecules of single-stranded DNA. This is


shown in Figure 9-52. In bacterial cells the source of energy for this reaction
comes from NAD+ while in animal cells it comes from ATP.

Replication Forks (revisited)

The main chromosome in the prokaryotic bacterium E. coli is circular (there are
also minor chromosomes called plasmids). Recall that we have mentioned that in
the E. coli replication starts at a unique site called ori C (an origin of replication).
In Figure 9-47a we drew a segment of DNA and located the origin of the
replication fork. In Figure 9-47bwe proceeded to draw a typical replication fork.
However, what we did not show was that replication can be bidirectional. In
other words, as the parental DNA is opened, replication will proceed at two
replication forks simultaneously. This can be seen in Figure 9-53.
Parental DNA

3'

5'

DNA Gyrase
adds negative
supercoils

.4
Origin of
Replication
Fork

Growing
Replication

Origin

Growing
Replication-

Fork

Fork

Lagging
Strand

Growing
Replication
Forks

Helicase

Figure 9-53
Origin

Eukaryotic chromosomal DNA is a linear polymer and undergoes active DNA


replication during the S-phase of the cell's life cycle. Because the DNA in
eukaryotic cells is quite long compared to E. coli's DNA, it is replicated
bidirectionally from many origin sites as shown in Figure 9-54. This ensures that
there will be rapid replication of the genome.

Copyright by The Berkeley Review

326

The Berkeley Review

Specializing in MCAT Preparation

Genetic
Information

15 Passages
100 Questions

Passage Titles
I.
II.
III.
IV.
V.
VI.
VII.

VIII.
IX.

X.

XI.
XII.

HbA/HbS Gel Electrophoresis


Gregor Mendel and Inheritance
Incomplete Dominance and Codominance
DNA Replication
Color Blindness
Alleles and Coat Color
DNA Calculations

Semiconservative DNA Replication


DNA Structure

Mitochondrial DNA (mtDNA)

Viral Complementation
Atrial Natiuretic Peptide Experiment

XIII.

Griffith's Pneumococcus

XIV.
XV.

Drosophilia eyeless Gene


Meiotic Nondisjunction

"The

T3EKKELEY
JJr.E-V'I^E'W8
Specializing in MCAT Preparation

Questions
1 -5
6- 11
12- 17
18-23
24-29
30-36
37 - 43
44-50
51 -58
59-65

66-72
73-79
80-87
88-94

95- 100

Suggestions
The passages that follow are designed to get you to think ina conceptual manner about the processes
ofmolecular biology at the organismal level. Ifyou already have a solid foundation in molecular biology,
many ofthe questions you read here will seem to bevery straight forward and easy toanswer. But ifyou
are new to the subject or if you have not had a pleasant experience with molecular biology in the past,
some of them might appear tocome from the void that spreads outbeyond the Oort field at theedges of
our solar system.

Pick a few passage topics at random. For these initial few passages, do not worry aboutthe time. Just
focus on what is expected of you. First, read the passage. Second, look at any diagrams, charts, or graphs
in it. Third, read each question and the accompanying answers carefully. Fourth, answer the questions
the best you can. Check the solutions and seehow you did. Whether you got the answers right or wrong,
it is important to read the explanations and see if you understand (and agree with) what is being
explained. Keep a record of your results.

After you feel comfortable with the format of those initial few passages, pick another block of
passagesand try to do them in one sitting. Be aware that time is going to becomeimportant. On average,
you have about 1 minute and 15 seconds to complete a question. Be creative in how you approach this
next group. If you feel comfortable with the outline presented above, fine. If not, then try different
approaches to a passage. For example, you might feel well versed enough to read the questions first and
then try to answer some of them, without ever having read the passage. Maybe you can answer some of
the questions by just looking at the diagrams, charts, or graphs that are presented in a particular passage.
Remember, there are many effective learning styles. You need to begin to develop a format that works
best for you. Keep a record of your results.

The last block of passages might contain at least a few topics that are unfamiliar even to those who
know a good deal about molecular biology. Find a place where the level of distraction is at a minimum.
Get out your watch and time yourself on these passages, either individually or as a group. It is important
to have a feel for time, and an awareness of how much is passing as you try to answer each question.
Never let a question get you flustered. If you cannot figure out what the answer is from information
given to you in the passage, or from your own knowledge base, dump it and move on to the next
question. As you do this, make a note of that pesky question and come back to it when you have more
time. When you are finished, check your answers and make sure you understand the solutions. Be
inquisitive. If you do not know the answer to something, look it up. The solution tends to stay with you
longer that way. (For example, what is the Oort field, anyway?)

The estimated score conversions for 100 questions are shown below. At best, these are rough
approximations and should be used only to give one a feel for which ballpark they are sitting in.

Section IX

Estimated Score Conversions


Scaled Score
>13

Raw Score

80-100

11-12

70-79

9-10

60-69

7-8

50-59

5-6

40-49

<4

0-39

Biology

HbA/HbS Gel Electrophoresis

Practice Passage I

Passage I (Questions 1-5)

Human adult hemoglobin (HbA) is a tetrameric


protein, which has a molecular weight of approximately
65,000 amu. This protein consists of two (identical) aglobin polypeptide chains containing 141 amino acids
each, and two (identical) (3-globin polypeptide chains
containing 146 amino acid residues. Each globin
polypeptide contains one iron atom (55.8 amu), and each
iron atom can complex with one molecule of molecular
oxygen (32.0 amu).

Protein

Protein

sample 1

sample 2

Lane 1

Lane 2

Cathode O

Separated
components

Large proteins like hemoglobin can have many acidic


and basic groups, giving the molecule both negative and
positive charge. These types of molecules are called
polyampholytes, and they will always have an isoelectric

Anode O

point (pi) where the net average charge is zero. The pi of


HbA is 6.9 An ampholyte that has a net charge of zero is

Figure 2. Separation of protein samples by PAGE.

called a zwitterion.

Sickle-cell hemoglobin (HbS) and the wild-type HbA


differ from one another by the substitution of a single

amino acid residue at position 6 in the P-globin chain.


Individuals with normal adult hemoglobin have the

hemoglobin composition a2Aj32A. Individuals with sicklecell trait have both HbA and HbS and a hemoglobin

What is the approximate percentage of iron in

composition of CC2Ap2A and 0i2Ap2S. Individuals with

hemoglobin?

sickle-cell disease have just HbS and a hemoglobin

composition of 0C2Ap2s. The relevant portions of the two

A.
B.
C.
D.

beta chains for both HbA and HbS are shown in Figure 1:

0.09 %
0.17%
0.26 %
0.34 %

HbA H3N-Val-His-Leu-Thr-Pro-Glu-Glu-LysHbS H3N-Val-His-Leu-Thr-Pro-Val-Glu-Lyspl

678

HbS contains a valine residue at the |36 position,


while HbA contains a glutamate residue.

Figure 1.Theamino acidsequence of the P-chains forboth


HbA and HbS.

This difference makes it easy for different proteins to

h,n-c-c- o
i
H,C- CH

be separated from one another by a process called


polyacrylamide gel electrophoresis (PAGE), diagrammed
in Figure 2. Thegel is formed from the polymerization of
acrylamide and N,N'-methylenebisacrylamide and then
suspended between an upper and lower buffer solution.

^
H
H3N C-C-0
CH2

CH2
I

CH,
Valine (Val)

0=C-0

Glutamate (Glu)

pKa - 4.4

The buffer solution usually has a pH of about 9 and is the


same in both reservoirs, as well as in the gel itself.

In HbS, the pi is expected to be:

Different protein samples are placed in designated wells

in the gel, and a direct current of 300 volts is applied for

A.
B.
C.
D.

about 60 minutes. After separation, the protein bands in


the gel can be visualized by a variety of techniques (e.g.,
staining or UV irradiation). Electrophoretic separation is
quite important in clinical laboratories, as it can be used
to identify abnormal hemoglobin patterns.
Copyright by The Berkeley Review

329

equal to the pi for HbA.


slightly greater than the pi for HbA.
slightly less than the pi for HbA.
representative of a nonzwitterionic species.

The Berkeley Review

Specializing in MCAT Preparation

Biology
3.

HbA/tlbS Gel Electrophoresis

Practice Passage I

If HbA and HbS undergo gel electrophoresis at a

buffered pH of 9, then the protein band shown in


Lane 2 of Figure 2 represents:
A.

HbA, because of its movement towards the

B.

HbS, because of its movement towards the

C.

HbA, because of its movement towards the

negatively charged anode.


positively charged anode.
positively charged anode.
D.

HbS, because of its movement towards the

negatively charged anode.

4.

Changes in the nucleotide sequence of DNA


represent mutation. If each amino acid in a protein
is coded for by three bases in DNA, then the
difference between HbA and HbS must be the
result of a:
I.
II.
III.

base-substitution mutation.
base-addition mutation.
base-deletion mutation.

A.

I only
II only
III only
II and III only

B.

C.
D.

5.

Sickle-cell trait is a genetic disorder inherited as an


autosomal recessive disease. If a man has sickle-

cell trait and marries a woman who is heterozygous


for a type of hemoglobinopathy refened to as HbC,
what is the probability that their first child will not
inherit any genetic disorder related to hemoglobin?
A.

0.25

B.
C.

0.50
0.75

D.

1.00

Copyright by The Berkeley Review

330

The Berkeley Review


Specializing in MCAT Preparation

Biology

Gregor Mendel and Inheritance

Passage II (Questions 6-11)

6.

Passage II

Mendel first allowed plants of a given variety to


produce progeny by self-fertilization over many

The first quantitative studies of inheritance involved


the garden pea and were carried out by Gregor Mendel.
His choice of the pea was fortuitous, because a large
number of true-breeding varieties (uniform from one
generation to the next) was available. Mendel selected
seven easily distinguishable traits of these plants (e.g.,
purple versus white flowers, and round versus wrinkled

generations in order to:

seeds).

A.

carry out an experimental cross,

B.

C.

assure himself the plants were of the truebreeding variety.


observe segregation of alternative traits among

D.

carry out a reciprocal test-cross.

progeny,

Pea plants contain both male and female sex organs.


This was also advantageous to Mendel, because he could
either let self-fertilization take place within an individual
flower or perform an experimental cross. A cross involves
removal of a flower's male parts before fertilization can
occur and introducing pollen from a strain with alternative
7.

characteristics.

Mendel first allowed plants of a given variety to

produce progeny by self-fertilization for many


generations. He then conducted crosses. Mendel took a
pea plant producing white flowers and introduced pollen
from a purple-flowered plant. He allowed the hybrid
offspring produced by these crosses to self-pollinate for
many generations. Mendel kept a record of the number of
offspring of each type and in each generation.

Mendel discovered that out of the F2 generation for


any trait, 1/2 of the individuals were not true
breeders. This class of plants, if allowed to selfpollinate, should produce Fj individuals that exhibit
dominant and recessive traits in a ratio of:
A.

1:1

B.

2:1

C.

2.5:1.5

D.

3:1

The hybrid offspring from the experimental crosses are


termed the first filial (Fi) generation. These offspring
always resemble one of their parents. For example, a
cross between wrinkled versus round seeds always

produce Fi progeny with round seeds. Mendel referred to


the Fi trait as being "dominant" over the invisible,
8.

"recessive" trait. Mendel allowed individual Fi progeny

plants to self-pollinate, and he observed the offspring in


this second filial (F2) generation. He found that some
individuals in this generationexhibited the recessive trait.

What is the ratio of homozygous to heterozygous


individuals in an F2 generation?
A.

1:1

B.

3:1

Out of 7,324 F2 individuals resulting from the round-

C.

seeded Fi self-pollination, 5,474 had round seeds and

D.

9:3
16:1

1,850 had wrinkled seeds. The ratio was essentially 3:1.


Mendel examined all seven traits with alternative

forms and in every case obtained the same ratio results:


3/4 of the F2 individuals exhibited the dominant form of

the trait, and 1/4 displayed the recessive form. He


accounted for his data by forming a central assumption
that alternative forms of a trait are specified by alternative

9.

sets of chromosomes are found in its nucleus. For a

alleles, which are discrete.

human cell in prophase I, the cell is said to be:

In other words, the factors specified by a pair of


alternating alleles are separate. This is known as the law

A.
B.
C.
D.

ofallele segregation.

Copyright by The Berkeley Review

The ploidy number (n) of a cell refers to how many

331

haploid (n = 1).
diploid (n = 2).
triploid (n = 3).
tetraploid (n = 4).

The Berkeley Review

Specializing in MCAT Preparation

Biology

Gregor Mendel and Inheritance

Passage II

10. In an effort to determine whether a purple-flowered

pea plant is heterozygous or homozygous, a test


cross is performed with a white-flowered pea plant.
Based on information in the passage, the MOST

likely conclusion is that the purple-flowered


individual is:

A.

heterozygous, if all offspring have purple


flowers.

B.

heterozygous, if half of the offspring have

homozygous, if all offspring have white

purple flowers.
flowers.

D.

homozygous, if half of the offspring have


purple flowers and the other half have white
flowers.

11. The following diagram, illustrating a cross of


wrinkled and round seeded peas, is inaccurate
because the:

P generation

Wrinkled
seeds

Round
seeds

Round seeds

Wrinkled seeds

A.

P generation is not made up of true breeders.

B.

ratio of round to wrinkled seeds in F2 is

C.

segregation is incorrectly shown as being


between pods.
number of progeny in F2 is incorrect.

incorrect.

D.

Copyright by The Berkeley Review

332

The Berkeley Review


Specializing in MCAT Preparation

Biology

Incomplete Dominance and Codominance

Passage III

12. Suppose two pink Japanese four-o'clocks are


crossed. What colors would the offspring, the F2
generation, exhibit?

Passage III (Questions 12-17)

Although many genetic traits are inherited in a


classical dominant/recessive pattern, this is not true of
every genetic trait. Certain genes follow the trends of
either codominance or incomplete dominance.

A.
B.
C.
D.

When a heterozygote for a particular trait has a


phenotype that is intermediate between those of its two
parents, the genes exhibit incomplete dominance.
Although many examples are known in both plants and
animals, a well-studied example of incomplete dominance
in color occurs in a flower called the Japanese fouro'clock. When a homozygous white flower (rr) and a
homozygous red flower (RR) are crossed, the Fi

1:2:1, pink:white:red
1:2:1, red:pink:white
All flowers either pink or red
2:2 pink:white

13. Which of the following terms BEST fits this


definition?

generation is entirely pink (Rr). Both the red color and the
white color are expressed together in the pink offspring.

Genes that

occupy corresponding loci on

homologous chromosomes and govern variations of


the same characteristic.

Rr

Rr

Rr

Rr

A.

Homologous pairs

B.

Pheromones

C.

Segregants

D.

Alleles

Figure 1. Cross of red and white


flowers, producing all pink offspring.

14. Which type of genetic cross is represented in


Figure 1?

Another type of genetic expression is codominance, in


which both alleles are expressed independently and
simultaneously in the heterozygote. For example, a horse
with a red coat (RR) is crossed with a white horse (rr).
The foal (Rr) has a mixed red and white coat, in which
red hairs and white hairs are interspersed. Horse breeders

A.

A monohybrid cross.

B.

A test cross.

C.

A dihybrid cross.

D.

An inbreeding cross.

refer to these crossed breeds as being "roan-colored."

A classic example of codominance in humans occurs


in the ABO blood group patterns. A and B refer to two

specific glycoproteins that can be present on the surface


of red blood cells. The A and B glycoproteins are

15. Suppose a gene independent of the one governing

codominant. If either protein is present, it is expressed.


The lack of either protein results in the O blood type,

leaf form in a classical dominant/recessive pattern,

the color of a Japanese four-o'clock determined its

with straight leaves (L) being dominant over curly

which is recessive to A and B.

leaves (1). Given the following cross:


RRL1 (Parent 1)

rr Ll (Parent 2)

What is the probability of producinga plant with


pink flowers and curly leaves?
A.
B.
C.
D.

Copyright by The Berkeley Review

333

0.25
0.50
0.75
0.00

The Berkeley Review

Specializing in MCAT Preparation

Biology

Incomplete Dominance and Codominance

Passage III

16. The paternity of a royal heir is in dispute. The queen


(known to be the baby's mother) has type AB"

blood, while the king has B\ The baby has A+


blood. The positive or negative Rh factor is
transmitted as a classical Mendelian dominant/

recessive trait, with + being dominant. Is the child


the offspringof the king and the queen?
A.
B.

Absolutely
Possibly

C.

No

D.

Unable to tell from this passage

17. Two people with type O" blood have a child


together. What blood types could their child have?
I.

o-

II.

A-

III.

B"

C.

Ionly
I and II only
I and III only

D.

I, II, and III

A.
B.

Copyright by The Berkeley Review

334

The Berkeley Review


Specializing in MCAT Preparation

Biology

DNA Replication

Passage IV (Questions 18-23)

18.

Passage IV

The replication of the entire E. coli genome requires


a relatively short time, approximately:

The entire Escherichia coli (E. coli) genome contains

4.7 x 106 nucleotide base pairs. At special sequences

A.

known as replication origins, two replication forks move


in opposite directions, with polymerization occurring at a
rate of 500 nucleotides per second.

B.

C.
D.

160 minutes.
100 minutes.

80 minutes.
60 minutes.

The average human chromosome is a single DNA


molecule containing about 150 million nucleotide base

pairs of DNA. In an experiment to determine the pattern


of eukaryotic chromosome replication, human cells are

19.

Which of the following patterns BEST represents


DNA replication of a eukaryotic chromosome?

grown in a culture labeled for a short time with 3Hthymidine. The cells are lysed and placed on the surface
of a glass slide. The glass slide is coated with a
photographic emulsion so the pattern of labeled DNA can
be determined by autoradiography. Results of the
experiment determine that polymerization occurs at a rate

A.

of 50 nucleotides per second.


B.

Autoradiography results of two experiments on


identical pieces of DNA from the same organism are
shown in Figure 1. The patterns of replication are
demonstrated by these results:

-Clou

D.

ds DNA

^ Replication origin

Experiment A

<=

=* dsDNA ^
20. The slower rate of DNA replication on a eukaryotic
chromosome could be explained by:

Silver grains

Experiment B

0=>

A.

the presence of multiple chromosomes.

B.

the presence of exons and introns within the


DNA molecule.

C.

dsDNA

the difficulty in replicating DNA packaged as


chromatin.

D.

the difficulty in replicating DNA packaged


inside the nucleus.

Figure 1

21. Replication forks are MOST likely to be activated


during which of these phases of the cell cycle?

Copyright by The Berkeley Review

335

A.

Gi

B.

C.

G2

D.

The Berkeley Review

Specializing in MCAT Preparation

Biology

DNA Replication

Passage IV

22. In a modification of the experiment discussed in the

passage, a eukaryotic cell was pulse-labeled with

^H-uridine. The autoradiograph results would reveal


the presence of a label:
A.
B.
C.
D.

only in the nucleus.


in both the nucleus and the cytoplasm.
only in the cytoplasm.
only in the nuclear membrane.

23. Which of the following statements could be TRUE

regarding the autoradiography results shown in


Figure 1?

I. The organism is most likely eukaryotic.


II. The polymerases used in Experiment A are
slower than those used in Experiment B.
III.

Further incubation after the addition of an

unlabeled medium occurred in Experiment B.


A.
B.
C.
D.

I only
I and II only
I and III only
II and III only

Copyright by The Berkeley Review

336

The Berkeley Review


Specializing in MCAT Preparation

Biology

Color Blindness

Passage V (Questions 24-29)

24.

In human beings, the ability to perceive color depends


on genes that are both autosomal and X-linked. These
genes initiate the production of visual proteins that are
sensitive to red (565 nm), green (535 nm), and blue (420
nm) wavelengths of light. Each visual pigment is found
separately in a unique cell called a cone. These cells are
concentrated in an area called the fovea, located in the
retina of each eye.

Passage V

Which of the following statements is LEAST likely


to result from gene duplication?

A.
B.
C.
D.

Synthesis of similar amino acid sequences


Synthesis of similar tRNA sequences
Independent mutation of the DNA sequences
Synthesis of longer mRNA sequences

The genes that code for the red and green pigments are
close to one another on the X chromosome. The X

chromosome contains one gene for the red pigment and

up to three genes for the green pigment. The gene for the
blue pigmentis found on chromosome 7. Genes whichare
closely related to one another belong to gene families.
The X-linked genes appear to have evolved through gene
duplication during the past 30 million years.

25.

What percentage of females would be expected to


exhibit red-green color blindness, if one genetic
locus were involved?

Red-green color blindness is a sex-linked recessive

A.

0.04%

B.

0.12%

C.
D.

0.36%
0.64%

trait and is found in about 8% of the male population and

in less than 1% of female population, even though about


15% of the female population are carriers. A defect in the

synthesis of the green-sensitive pigment is referred to as


the deutan type of color blindness. Approximately 6% of
the 8% of all males who are color-blind have a deutan

defect.The remaining 2% have a proton defect, a type of


color blindness resulting from a defect in the synthesis of
the red-sensitive pigment. Both defects lead to poor
recognition of various shades of red and green colors.

26.

Since two genetic loci are involved in red-green


color blindness, the incidence of color blindness

among females in the United States is 0.4% The


percentage of females that would be deutan color

The frequency for X-linked genes in males is the same


as the phenotypic frequency, simply because the

blind is:

phenotype of both the recessive and the dominant alleles


is expressed. The frequency of normal and mutant alleles

A.
B.
C.
D.

can be calculated using the concepts of Hardy-Weinberg


equilibrium and the equation:

0.04%
0.12%
0.36%
0.64%

p2 +2pq + q2=l

where p is the frequency of the dominant allele, and q is


the frequency of the recessive allele. Color blindness is

27.

not a deleterious trait and therefore does not affect either

The percentage of women who are heterozygous


carriers of protan color blindness is about:

the viability or the reproductive success of the individual.

Copyright by The Berkeley Review

337

A.

2%

B.

4%

C.
D.

8%
11%

The Berkeley Review

Specializing in MCAT Preparation

Biology
28.

Color Blindness

Passage V

A woman with normal vision marries a man who is


affected with deutan color blindness. Her sons will
have:

29.

A.

deutan color blindness.

B.
C.

homozygosity for the trait.


heterozygosity for the trait.

D.

normal color vision.

A dominant autosomal allele C is required for proper


development of the cones in the retina. The
recessive allele c in the homozygous form results in
complete color blindness. A woman who has a
deutan defect is homozygous for the autosomal C
allele. She marries a man who is completely color
blind but carries the dominant deutan allele. What

types of visual problems will their children have?


A.

Daughters will have normal vision; sons will

B.

Daughters will be red-green color-blind; sons


will be completely color-blind.
Daughters will have normal vision; sons will
be red-green color-blind.
Daughters will be completely color-blind; sons
will be red-green color-blind.

have normal vision.

C.

D.

Copyright by The Berkeley Review

338

The Berkeley Review

Specializing in MCAT Preparation

Biology

Passage VI

Alleles and Coat Color

Neither the black nor the brown coat color is expressed


when the hamster is homozygous for allele a, found at a
different locus on the chromosome. The suppressive
influence of allele a on an entirely different allele is an
example of epistasis.

Passage VI (Questions 30-36)

A gene is a unit of inheritance located at a specific site


(locus) on a chromosome (DNA). Genes exist in pairs in

diploid organisms, and the individual genes of a gene pahare referred to as alleles. If two alleles of a gene pair are
identical, the organism is homozygous for that gene pair.
If the two alleles are different, the organism is

XX
XX

heterozygous for that gene pair. The complete set of genes


in an organism determines its genotype, while the
expression of those genes as a collection of detectable
characteristics determines its phenotype.

A hamster that is homozygous for allele a has a


genotype that produces a white (albino) coat. If a hamster
has the + (wild-type or normal) allele at one of its a loci,

In a heterozygous individual, a dominant trait is due to


a gene that expresses itself in the presence of its allele. A
recessivetrait in a heterozygote is due to a gene that does
not express itself in the presence of its allele. Incomplete
dominance in a heterozygote involves the expression of

its coat color will be normal.

Two experiments were conducted to investigate the


genetics of coat color in hamsters.

both alleles, with the effect of one allele appearing greater


than the other allele. In a heterozygote, a codominant trait

ExperimentI

involves both alleles producing a combined effect. In this


case, two gene products can be detected. Each product is

A group of true-breeding hamsters with the genotype B/B;


+/+ is crossed with a group of true-breeding hamsters with
the genotype b/b; a/a to give the first filial (Fi)

associated with one of the allelic forms.

Alleles on separate chromosomes can segregate from


one another at the time of gamete formation. This is the
basis of Mendel's second law (independent assortment) and

generation.

results in interchromosomal recombination, which always

ExperimentII

produces a recombination frequency of 50% in the second


filial (F2) generation.

One hamster from the Fi generation of Experiment I


was back-crossed to the parent with the double recessive

However, two or more pairs of alleles can be associated

genotype (b/b; a/a). The progeny of the resulting F2

with the same chromosome. If this is the case, the genes


are said to be linked. If two dominant alleles are linked,

generation are shown below:

thosealleles are in coupling. If a dominant allele is linked


with a recessive allele, those alleles are in repulsion. If

there is crossing over between any two non-sister


chromatids,

intrachromosomal

recombination

results,

which always produces a recombination frequency of less


than 50% in the F2 generation. The recombination

frequency (percent crossing over or %CO) can be

Black offspring:

66

Brown offspring:

34

White offspring:

100

Total offspring:

200

determined from the following equation:


Number of Cross over Individuals in F2

x 100%

Total Number of Individuals in F2

The black coat color in hamsters is due to a dominant

gene, B, at a particular locus on a chromosome. The


recessive allele, b, at this same locus results in a brown
coat color when it is in the homozygous condition:

30. Based on the information given in the passage,

which of the following statements is correct?

XX
XX

Copyright by The Berkeley Review

339

A.
B.
C .

The a allele is dominant over the + allele.


The a allele is recessive to the + allele.
The a allele and the + allele are codominant.

D.

The + allele shows incomplete dominance.

The Berkeley Review

Specializing in MCAT Preparation

Biology

3 5. Based on the information in Experiment II and in the


passage, which of the following BEST describes the
two genetic loci?

31. A true-breeding strain of black hamsters would show


which of the following genotypes?
A.
B.
C.
D.

Passage VI

Alleles and Coat Color

B/b;+/+
B/b\ +/a
B/B;+/+
B/B; +/a

A.
B.
C.
D.

They are linked.


They are unlinked.
They are epistatic.
They assort independently.

36. The genetics of coat color in albino hamsters is an


example of:
32. A different true-breeding strain of hamsters has the
genotypeb/b; a/a at both loci. The phenotype of the

A.

dominance.

coat of these hamsters would be:

B.
C.

recessive epistasis.
incomplete dominance.

I.
II.
III.

black.
brown.
white.

D.

codominance.

A.
B.
C.
D.

I only
II only
Ill only
I and HI only

33. Based on the information in Experiment I, what is


the MOST likely phenotype of the progeny from the
first filial (Fi) generation?
A.
B.

All black
All brown

C.

All white

D.

Both black and brown

3 4. Based on the information in Experiment II, estimate


the minimum genetic map distance between the two
genetic loci. (Note: One genetic map unit (m.u.)
gives a recombinant frequency (RF) of 1 percent.)
A.
B.
C.
D.

17
34
52
68

m.u.
m.u.
m.u.
m.u.

Copyright by The Berkeley Review

340

The Berkeley Review


Specializing in MCAT Preparation

Biology

DNA Calculations

Passage VII (Questions 37-43)

Passage VII

However, the separation of cellular constituents and


subsequent cytokinesis requires a constant time of 20
minutes after the completion of the corresponding round
of DNA replication. Therefore, bacterial cells with
doubling times that are shorter than 60 minutes must
begin DNA replication before the end of cytokinesis of
the preceding cell cycle.

In 1953 James Watson and Francis Crick submitted a

landmark one-page paper to the British science journal


Nature proposing a three-dimensional model for the
structure of DNA. Using X-ray diffraction patterns
obtained by Rosalind Franklin and Maurice Wilkins,
Watson and Crick suggested that native DNA consisted of
two antiparallel polynucleotide strands that wind about
each other in a helical fashion along a common axis. The

is due to adenine-thymine (A-T) and guanine-cytosine


base-pairings (G-C).

Replication of the DNA duplex in E. coli is an ordered


process and is promoted by at least 20 enzymes and
proteins. A helicase enzyme uses the energy of adenosine
triphosphate (ATP) to unwind the DNA at the oriC locus
to give two replication forks, from which the parental

Each base pair has an average molecule mass of 660


Daltons (D). In an ideal situation, the DNA helix has 10
base pairs (bp) per turn of the helix, resulting in a helical
twist of 36 per bp. The rise of the helix per turn (pitch) is

strands serve as templates for the synthesis of new


daughter DNA. Since all known DNA polymerases
require a primer before new DNA can be synthesized, a
short RNA primer of about 5 nucleotides is synthesized
by a DNA-directed RNA polymerase called a primase.

helix is right-handed with a diameter of about 20A. This

34A, while the rise per base pair is 3.4A. Shortly after
Once the primer has been established, DNA
polymerase III begins to synthesize new DNA in the 5' to

Watson and Crick proposed their model for the structure


of DNA, they postulated that DNA replicates
semiconserva-tively. The result is two molecules of
double-stranded DNA (a duplex), each containing the

3' direction (at a rate of about 1000 nucleotides per


second) on the leading strand in a continuous fashion and
on the lagging strand in a discontinuous fashion.
Discontinuous DNA synthesis occurs in the form of lkb
to 2kb segments of DNA called Okazakifragments. DNA

original parental strand of DNA and a newly synthesized


daughter strand of DNA. This hypothesis was verified in
1958 by Matthew Meselson and Franklin Stahl.

polymerase I removes the RNA primers using its


exonuclease activity and then slowly adds new
nucleotides at a rate of about 10 per second. The
fragmented stretches of newly synthesized DNA are
finally joined together by DNA ligase.

In 1957 Arthur Kornberg, using the bacterium


Escherichia

coli,

discovered the enzyme DNA

polymerase I, which is involved in the DNA-directed


synthesis of DNA. In 1969 Paula DeLucia and John
Cairns discovered a mutant of E. coli that did not display

DNA polymerase I activity. This mutant was named the


polAX mutant. Even though the polAl mutant had about
1% of the normal polymerizing function of DNA
polymerase I, it reproduced at the same rate as its parental
strain, thus suggesting that there might be polymerase
activity by an enzyme or enzymes other than DNA

37.

If the average molecular weight of an amino acid


residue is HOD, and if each amino acid is specified

by three contiguous bases on a single strand of


DNA, then for a 40-kD protein the contour length
and molecular weight of a given section of DNA

polymerase I.
This observation led to the discovery of DNA

would be about:

polymerase II and DNA polymerase III. DNA


polymerase III is also involved in the DNA-directed
synthesis of DNA. However, the physiological function of

A.
B.
C.
D.

DNA polymerase II is still unknown.

370A, 7.0 x 104 D


370OA, 7.0 x 104 D
370A, 7.0 x 105 D
3700A, 7.0 x 105 D

In E. coli, the bidirectional replication of the roughly


4000 kilobase pairs found in the circular DNA
chromosome is initiated at a single locus referred to as
oriC. [One kilobase (kb) is equal to 1000 nucleotide base
pairs (bp) of duplex (double-stranded) DNA or 1000
nucleotides of single-stranded DNA]. The 245 bp segment
of the oriC locus is highly conserved among Gram-

38.

negative bacteria. Even though the doubling time of this


bacterium can range from less than 20 minutes to about
10 hours (depending upon the growth medium), it takes

A.
B.
C.
D.

about 40 minutes for the chromosome to replicate itself


completely.

Copyright by The Berkeley Review

The rate of nucleotide incorporation at each growing


replication fork would be roughly:

341

225 nucleotides per second.


425 nucleotides per second.
850 nucleotides per second.
1700 nucleotides per second.

The Berkeley Review

Specializing in MCAT Preparation

Biology
39.

Unlike the circular chromosome of a prokaryotic

42.

organism like E. coli, the linear chromosomes of


eukaryotic organisms have many origins of
replication. These sites of replication are roughly
spaced 30 to 100 kilobases apart from one another.
Suppose that a given tissue culture of mammalian

During DNA replication, the base sequence 5'pApTpApGpApC-3' would give rise to which of
these complementary base sequences?
A.
B.
C.
D.

DNA has 1.2 meters of duplex DNA in every single


cell and that the synthesis phase (i.e., that period in
the cell when DNA is synthesized) for these cells is
5 hours. How many replication forks are there, if the
growthrate in these cells is 16|im/minute?
A.
B.
C.
D.

125 replication forks


250 replication forks
375 replication forks
500 replication forks

43.

II.

In a rich growth medium, bacterial cells can divide


every 20 minutes. Which of the following BEST
describes the cell's bidirectional replication in this

III. DNA polymerase III adds new deoxyribonucleotides to a growing polynucleotide chain
roughly 100 times faster than DNA polymrase

I.

At division, each daughter cell receives a


chromosome that is half replicated.
n. At division, each daughter cell receives a
chromosome that is one-quarter replicated.
III. There is a total of four replication forks.

41.

There is a total of six relication forks.

A.
B.
C.
D.

Ill only
I and III only
II and IV only
I and IV only

In addition to its 5' to 3' polymerizing activity,


DNA polymerase I also has a 5' to 3'
exonuclease activity that excises RNA
primers.
DNA polymerase III can add nucleotides at the
3' end of a growing polynucleotide chain,
while DNA polymerase I can add nucleotides
at the 5' end of a growing polynucleotide
chain.

case?

IV.

3'-UpApUpCpUpGp-5'
5'-pTpApTpCpTpG-3'
5'-pGpTpCpTpApT-3'
3'-TpApTpGpUpGp-5'

Which of the following statements about the action


of DNA polymerases must be true?
I.

40.

Passage VII

DNA Calculations

I.

A.
B.
C.
D.

II only
II and III only
I and III only
I and II only

During the replication of an E. coli chromosome,


roughly how many Okazaki fragments are
synthesized?
A.

100 to 200

B.

C.

1000 to 2000
2000 to 4000

D.

4000 to 6000

Copyright by The Berkeley Review

342

The Berkeley Review


Specializing in NCAT Preparation

Biology

Semiconservative DNA Replication

Passage VIII (Questions 44-50)

Passage VDI

44. Watson and Crick's proposal for the structure of


DNA does not allow for which of the following base
pairings?

After Watson and Crick announced their discovery of


the structure of DNA in the early 1950s, they proposed
that DNA could reproduce itself autocatalytically. They

I.
Cytosine-guanine
II. Adenine-guanine
III. Cytosine-thymine

hypothesized that the two parental strands of a DNA


duplex would separate, and each would serve as a
template for the synthesis of a new daughter strand of
DNA. Once DNA replication was complete, each new
duplex would contain one parental strand and one
daughter strand, linked by hydrogen bonds between
complementary purine (adenine and guanine) and
pyrimidine (thymine and cytosine) bases. Watson and
Crick called this type of replication semiconservative.
Conservative replication, in contrast, would conserve the

A.
B.
C.
D.

I only
II only
II and III only
Ill only

integrity of the parental strands in the DNA duplex after


45.

replication.

After Watson and Crick proposed the structure of


DNA, it was postulated that DNA could replicate in
a

In the late 1950s, Meselson and Stahl were able to

conservative fashion.

After two rounds of

conservative DNA replication, the original duplex


parental strands would give rise to:

confirm Watson and Crick's hypothesis that DNA

replicated semiconservatively by performing a densitygradient equilibrium sedimentation experiment. In this


experiment, a 6Mcesium chloride (CsCl) solution with a

A.

density of about 1.7 g/cm3 is placed in a test tube, which


is spun at high speed in an ultracentrifuge. A smoothly
changing concentration gradient is thus established in the

B.

test tube (Figure 1).

C.
Increasing density
of solution

D.

two duplexes of DNA, each containing one


daughter strand and one parental strand.
four duplexes of DNA, each containing one
daughter strand and one parental strand.
two duplexes of DNA, one duplex containing
two daughter strands and one duplex
containing two parental strands.
four duplexes of DNA, two duplexes each
containing two daughter strands and two
duplexes each containing two parental strands.

CsCl spinning in ultracentrifuge

46. The structural component of DNA is called a


Figure 1

nucleotide (see below):

E. coli were grown in a complete medium containing


15NH4C1 for numerous generations, ensuring that the

OR
i

DNA in these cells would contain the heavy, stable

isotope of nitrogen, 15N. An excess of 14NH4C1 was then

A nucleotide
containing

added to the culture of 15N-labeled bacterial cells.

0=p.~9

the base

Immediately after this addition of 14NH4C1, the DNA

guanine

from a sample of the cultured cells was extracted and


sedimented to equilibrium in the CsCl density gradient.
Ultraviolet absorption photographs (at 260 nm) indicated
the presence of just one band of DNA. This band

OR'

corresponded tothe all heavy 15N-labeled DNA.

The stable, heavy isotope of nitrogen, 15N, can be


found in all of the following components of DNA

The cells that remained in culture were allowed to

EXCEPT:

double and complete one generation of growth. Analysis


of the DNA from a sample of these cells indicated the

presence of a new band, that of a ,4N-15N hybrid. On the

A.
B.

basis of this density-gradient experiment, it was


concluded that DNA replicated in a semiconservative

C.
D.

the pyrimidine rings.


the purine rings.
the ribose rings.
the acetal linkages.

fashion.

Copyright by The Berkeley Review

343

The Berkeley Review

Specializing in MCAT Preparation

Biology
47.

Semiconservative DNA Replication

Ultraviolet absorption photographs (at 260 nm) of

50.

Passage VIII

Macromolecules that are not bound to ions have

densities close to 1.3 g/ml. Which of the following


single-stranded nucleic acid polymers would show a
greater density in a CsCl density gradient?

DNA in a CsCl density-gradient solution indicate a

particular DNA banding pattern. The distribution of


14N and 15N in a CsCl density gradient after two
rounds of semiconservative DNA replication is

represented by which of the following DNA banding

5'-pGpCpApApCpCpGpGpCpC-3'

DNA

5'-pCpCpGpGpCpCpApApCpG-3'

RNA

patterns?
B.

A.

(C = cytosine; G = guanine; p = phosphate)


A.

DNA, because the Cs ions can bind to the

negatively charged phosphate groups and the


free hydroxyl groups of DNA.

Control

B.

RNA, because the Cs ions can bind to the


negatively charged phosphate groups and the
free hydroxyl groups of RNA.

C.

DNA, because the Cle ions can bind to the


positively charged nitrogenous bases of DNA.

D.

RNA, because the CI ions can bind to the


positively charged nitrogenous bases of RNA.

Control

D.

c.

V_J
Control

48.

After

two

Control

rounds

of

semiconservative

DNA

replication, the ratioof 14N to l5N will be:


A.

3:1

B.

2:1

C.

1:2

D.

1:3

49. DNA synthesized from the ,5N isotope is denser


than DNA synthesized from the 14N isotope,
because of the additional:
A.

neutron in 15N.

B.

proton in I5N.

C.

electron in 15N.

D.

water molecules surrounding the 15N DNA.

Copyright by The Berkeley Review

344

The Berkeley Review


Specializing in MCAT Preparation

Biology

DNA Structure

Passage IX (Questions 51-58)

Passage IX

Replication of the DNA helix requires local


denaturation (melting) of hydrogen bonds between base

The molecules that store and transmit information in

pairs. Each single-stranded segment of old (parental)


DNA then serves as a template to which new (daughter)
DNA is attached. After one round of replication is

cells are deoxyribonucleic acid (DNA) and ribonucleic


acid (RNA). The fundamental unit of each of these
nucleic acids is the nucleotide. Each nucleotide contains a

complete, two new DNA duplexes are formed, each


containing one strand of parental DNA and one strand of
daughter DNA. This is referred to as semiconservative
replication.

nitrogenous base, a five-carbon sugar called ribose, and a


phosphate. The bases adenine (A), guanine (G), and
cytosine (C) are found in both DNA and RNA. Thymine
(T) is found in DNA and uracil (U) is found in RNA.

Figure 1 shows the structural components of these two

Three other forms of DNA replication have been


examined. If the DNA helix replicates conservatively,

nucleic acids (Note: dR = deoxyribose; R = ribose):

then after one round of replication one of the DNA


duplexes still contains both parental strands, while the
other DNA duplex contains two new daughter strands. If
replication is dispersive, then after one round of
replication the DNA in each strand of each duplex is
composed of alternating segments of parental and
daughter DNA. Finally, if replication is end-to-end
conservative, then after one round of DNA replication
each strand in each duplex contains one end that is all
parental DNA and one end that is all daughter DNA.

Cytosine

H,N^NAN
51.

Guanine

Adenine

5'
ii
O -P-O-CH,
P-O-CH

Ribose phosphate

ho

Which of the following bases is LEAST likely to


exist in the enol form?

Base

oh (orH)

A.

Adenine

B.

Cytosine

C.

Guanine

D.

Uracil

Figure 1
DNA and RNA can exist in a double-stranded (duplex)
or a single-stranded form, and they can be linear or
circular. Both nucleic acids can also hybridize to each
other. In the nucleus of eukaryotic cells, DNA is
associated with positively charged proteins called

52.

DNA can be classified by all of the following


characteristics EXCEPT:

A.
B.
C.
D.

histones.

it exhibits a polynucleotide arrangement.


it is negatively charged at neutral pH.
it is deoxy at the C-3' position.
it contains acid anhydride linkages.

The DNA double helix consists of two polynucleotide


strands that wind about a common axis, making a

complete turn every 3.4 nm. Each strand is joined by


hydrogen bonding between the nitrogenous bases. The
base pairs are partially stacked on each other and are
separated by 0.34 nm.

53.

The following sequence of a short segment of a


single strand of DNA is given:
5'-A-T-G-C-C-G-A-T-3'

Each strand in the DNA double helix has a specific


The complementary single strand of DNA is:

chemical orientation. Attached to the 5' carbon of the

ribose ring is a hydroxyl group that bears a phosphate; the


3' carbon atom of the ribose ring has a free hydroxyl
group. Polynucleotide sequences are conventionally

A.
B.
C.
D.

written in the 5' 3' direction, and in DNA which is

double-stranded the two polynucleotide sequences are


antiparallel to one another.

Copyright by The Berkeley Review

345

3'-pTpApCpGpGpCpTpA-3'
3'-pTpApCpGpGpCpTpAp-5'
5'-ApTpCpGpGpCpApT-3'
5'-pApTpCpGpGpCpApT-3'

The Berkeley Review


Specializing in MCAT Preparation

Biology

58. An unknown organism is incubated in a complete

54. On average, how many bases are there per turn of

growth medium containing I5NH4C1 for many

the DNA double helix?


A.
B.
C.
D.

Passage IX

DNA Structure

generations. The organism is next transferred to a

new growth medium containing 14NH4C1. After two

5
10
15
20

generations in the new growth medium, its DNA is


analyzed using CsCl equilibrium density-gradient
ultracentrifugation. Next, ultraviolet absorption
photographs of the ultracentrifuge test tubes are
taken, and a microdensitometer trace is made of the

55. Which of the following base pairs is NOT usually

results.

associated with DNA?

A.
B.
C.
D.

75%

Uracil hydrogen bondingto adenine


Adenine hydrogen bonding to thymine
Guanine hydrogen bonding to cytosine
Thymine hydrogen bonding to uracil

Is
*;

.o

|S
13

56. DNA is the carrier of genetic information during cell


growth and division. Which of the following
Direction of sedimentation

characteristics of DNA is NOT essential for the


accurate transmission of this information?

A.
B.

A genetic code that is degenerate


A mechanism for self-replication

Two peaks arre observed, and the amount of DNA in


each peak is recorded. Based on the information in
the graph presenting these data, we can say that

C.

A low mutation rate

DNA replication in this organism is:

D.

A conformationally variable molecule


A.
B.

conservative.
semiconservative.

57. A typical DNA melting curve is shown below.

C.

dispersive.

Denatured DNA shows an increase in ultraviolet

D.

end-to-end conservative.

absorbance. The temperature at which one-half the


maximum absorbance is reached is called the

melting temperature, Tm.

60

70

80

Temperature (C)

All of the following factors will lead to an increase


in the Tm of DNA EXCEPT:

A.

a lower mole fraction of AT base pairs.

B. a higher concentration ofMg2e.


C.

a lower concentration of histones.

D.

a higher mole fraction of GCbase pairs.

Copyright by The Berkeley Review

346

The Berkeley Review


Specializing in NCAT Preparation

Biology

Mitochondrial DNA (mtDNA)

Passage X (Questions 59-65)

61.

The biosynthesis of mitochondria involves the

Passage X

The codon AUG is used as a start codon in the


genetic code of the mitochondria. This codon codes

for which of the following amino acids?

contribution of two separate genetic systems. Most of the


proteins are encoded by nuclear DNA and imported into

the organelle. However, some proteins are encoded by


organelle DNA and synthesized on ribosomes within the

organelle. The protein traffic between the cytosol and the


mitochondrion appears to be unidirectional.
Mitochondrial DNA (mtDNA) molecules are small

A.
B.
C.

Leucine
Histidine
Methionine

D.

N-formyl methionine

62. As stated in the passage, every nucleotide in the


mitochondrial genome is part of a coding sequence.
A probable result of this is that compared to the

and circular. All mitochondria contain multiple copies of


their DNA molecules, and the genome is likely to

nuclear genome, there would be:

resemble bacterial chromatin. The human mitochondrial

genome has several surprising characteristics. One is that


every nucleotide appears to be part of a coding sequence
for a protein, rRNA, or tRNA. Second, only 22 tRNAs are
needed for mitochondrial protein synthesis versus the 31
tRNAs required in the cytosol. Third, the genetic code

A.
B.

fewer mRNA molecules transcribed.


more mRNA molecules transcribed.

C.
D.

more room for regulatory DNA sequences.


less room for regulatory DNA sequences.

used in the mitochondrion differs from the universal code

used in other genomes. It has been shown that four out of


the sixty-four codons in mtDNA have different meanings.
Further investigation has shown that the mitochondrial
genetic code is different in different organisms. For
example, the codon UGA codes for tryptophan in the
mitochondria of mammals, fungi, and protozoans. But in
plant mitochondria, UGA is a stop codon.

63.

Which of the following statements offers a


reasonable explanation for the different genetic code
found in mitochondria?

A.
B.

Both strands of human mitochondrial DNA are

transcribed from a single promoter region on each strand.


Two giant RNA molecules are thus created. The two
molecules are known as the heavy and light strands. The
heavy strand is processed to produce many RNAs,
including ten polyadenine-containing RNAs. Roughly
90% of the light strand contains no discernibly useful

C.

D.

The drift responsible for changes in the genetic


code occurs only in mitochondria.
The small number of proteins encoded in the
mitochondrial genome makes changes in the
meaning of the code tolerable.
The mitochondria has unique tRNAs, which
correct for the different genetic code.
In mitochondria, the primary structure of a
protein has no effect on the function of the
molecule.

information.

64.
59.

The ten polyadenine-containing RNAs noted in the


passage are MOST likely to:

In a human cell, protein transport does NOT move


from cytosol to:

A.

make their way to the cytosol to be translated.

A.

mitochondria.

B.
C.

have an attachment site for amino acids.


be involved with the structure of ribosomes.

D.

code for proteins.

B.

peroxisome.

C.

nucleus.

D.

chloroplast.
65.

The DNA that codes for the light strand is also


known as the:

60.

A.

Which of the following is LEAST likely to be


associated with the mitochondrial genome?

B.

A.
B.
C.
D.

Distinct promoter regions


RNA polymerase protein
DNA polymerase protein
Histone protein complex

C.

D.

antisense strand, because the light chain is


complementary.
antisense strand, because the light chain is
nearly identical.
sense strand, because the heavy chain is
complementary.
sense strand, because the heavy chain is nearly
identical.

Copyright by The Berkeley Review

347

The Berkeley Review


Specializing in MCAT Preparation

Passage XI

Viral Complementation

Biology

66.

Passage XI (Questions 66-72)

The DNA of the bacteriophage T4 encodes all of the

components necessary for the replication of its viral


genome. Which of the following might NOT be

The bacteriophage T4 is a DNA virus which infects E.


coli cells. When a wild-type T4 phage is placed on a
lawn, or thin layer, of E. coli cells growing on an agar

encoded?

plate, a local clearing develops where bacterial cells have


been lysed by the original phage and its progeny. This

A.
B.

tRNA
Primase

C.
D.

Ligase
DNA polymerase

clear region on the otherwise opaque lawn is termed a


plaque.

Two mutant forms of phage T4 have been isolated and


named rllA and rllB, respectively. Both mutants cause

larger-than-normal plaques on plates containing a strain


of bacteria known as E. coli B. However, both of these T4

mutants fail to form plaques at all on a second bacterial


strain, E. coli K. Wild-type T4, however, is capable of
lysing cells of E. coli K.

An experiment is conducted in which rllA and rllB


mutants are allowed to infect E.

67.

E. coli K cells are coinfected with rllA and a new T4

mutant called XI. No plaques form. What can be


concluded from this experiment?

coli K cells

simultaneously. Individual bacterial cells are therefore


coinfected by both mutant forms of T4. While infection
by either mutant alone causes no plaque formation, the
simultaneous coinfection by both mutants causes cell lysis
and the formation of small plaques. This phenomenon is

A.

The

rllA and XI

mutations each affect

different genes.
B.

The rllA and XI mutations both affect the

C.

The rllA and XI mutations complement each

D.

The XI mutant phage could lyse. coli K cells

an example of genetic complementation.

same gene.

In this example, complementation occurs when the


protein that is defective in rllA is provided by rllB, and
vice versa. This complementation results in the restoration
of the wild-type, lysis phenotype (Figure 1). In order for
two mutants to complement each other, they must each
affect different genes. If they both were to affect the same
gene, they would be incapable of complementation,
because neither mutant could provide a functional protein

other.

if it infected them alone.

product.

Genes

(phage rllB)
Pathway
Genes

(phage rllA)

(+)

(-)

rllA

rllB

t
(1)C

^>(2)|

1=

*
(-)

rllA

68.

> Lysis

cells?

I.

rll mutant DNA has sequences that are


recognized and cleaved by endonucleases
produced by E. coli K.
II. rll mutants lack functional gene products
necessary for lysis of E. coli K cells.
III. E. coli K lacks specific surface receptors to
which rll mutant phages bind.

J
(+)

rllB

Figure 1. Genetic complementation in phage T4.

Copyright by The Berkeley Review

Based on information given in the passage, which of


the following would explain why rll mutants are
incapable of forming plaques on lawns of E. coli K

348

A.
B.
C.

I only
II only
I and III only

D.

I, II, and III

The Berkeley Review


Specializing in MCAT Preparation

Biology
69.

Viral Complementation

In Drosophila, recessive mutants A and B each cause


a curly-wing phenotype when homozygous.
However, transheterozygous flies (i.e., flies that are
A/B) have curly wings, even though neither mutation
is homozygous. Which of the following conclusions

72.

A.
B.
C.

D.

70.

The T4 bacteriophagecan BEST be described as:


A.
B.
C.
D.

is MOST consistent with these results?

Passage XI

an obligate parasite.
an obligate heterotroph.
auxotrophic.
prototrophic.

complement each other;


therefore they affect different genes.
Mutations A and B fail to complement each
other; therefore they affect different genes.
Mutations A and B complement each other;
therefore they affect the same gene.
Mutations A and B fail to complement each
other; therefore they affect the same gene.
Mutations A and B

A large number of the T4 mutants rllA and rllB are

added separately to two plates of E. coli K cells. The


plate infected with rllA forms one revertant plaque,
while the plate infected with rllB forms no plaques
at all. Based on these results, which of the following
assumptions may be valid?
A.
B.
C.
D.

rllA is caused by a deletion of DNA, while


rllB is caused by a silent mutation.
rllA is caused by an deletion of DNA, while
rllB is caused by a point mutation.
rllA is caused by a point mutation, while rllB
is caused by a deletion of DNA.
Both rllA and rllB are caused by frameshift
mutations.

71.

Which of the following would be incapable of


exhibiting the type of complementation described in
the passage?
A.
B.
C.

Recessive mutations
Sex-linked recessive mutations
Dominant mutations

D.

Deletion mutations

Copyright by The Berkeley Review

349

The Berkeley Review


Specializing in MCAT Preparation

Passage XII

Atrial Natriuretic Peptide Experiment

Biology

74. Atrial tissue from the animals is isolated and

Passage XII (Questions73-79)

homogenized. The amount of ANP (pg/mg wet


tissue) in the atria is then measured. The results are
given in the following table:

About 10% of the US population has sodium-sensitive

hypertension. As dietary sodium increases, blood pressure


increases in these people. Blood pressure is regulated by
blood volume, peripheral resistance, and thecardiac rate.

Right atria

Animal

Left atria

One molecular mechanism of control involves a 28-

amino acid peptide produced primarily in the atria, called

Homozygous

atrial natriuretic peptide (ANP). It is stored in dense

mutant

granules in the cytoplasm of atrial cells as a larger


precursor, pro-ANP. In response to atrial distention or
stretch, pro-ANP is processed, and ANP is secreted into

Heterozygous

Undetected

Undetected

54.3 7.1

53.7 7.1

114.7 5.9

112.2 7.5

mutant

the bloodstream. It promotes sodium excretion into the

Wild-type

urine and the movement of fluid out of the intravascular


space.

What would be the BEST type of measurement to

Researchers use a genetically engineered mouse model


to study the interrelationships between ANP, dietary
sodium, and blood pressure. Mouse embryos are treated
so that the gene coding for ANP is mutated and

detect and quantify ANP, which is a small peptide?


A.

Lowry assay

nonfunctional. The mice are then interbred to produce

B.

Peptide digestion

homozygous mutants and heterozygous mutants. Control,


wild-type animals are also used. All mice are fed one of

C.

Southern blot

D.

Radioimmunoassay

three diets: either standard chow at 0.5% sodium, an

intermediate diet at 2% sodium, or a high-sodium diet at


8% sodium.
75.
Diet

Homozygous

Heterozygous

Wild-type

% sodium

mutants

animals

animals

0.5

124 3.7*

1151.8

116 2.6

Based on the observed effects of ANP on

intravascular compartments, explain the following


hematocrit values taken from the study animals:

(standard
chow)
2

156 9.0*

Animal

Hematocrit

Homozygous

45.8%

132 6.7

134 6.1

mutant

145 7.8*

118 6.2

Heterozygous

* p < 0.05 versus wild-type.

48.6%

mutant

Table 1. Blood pressure (mmHg) in experimental animals

Wild-type

49.9%

on various sodium-containing diets.


A.

hematocrit

73. Which of the following statements is TRUE about


Homozygous mutant animals were normo-

B.

The 2% sodium diet increased blood pressure


in all groups.
Heterozygous animals mimicked the responses

more

fluid

in

the

The wild-type animals have a higher

C.

intravascular space.
ANP directly affects the blood cells, so that

D.

of wild-type animals as dietary sodium


increased.

D.

to

B.

hematocrit

tensive on the standard chow diet.

C.

due

intravascular space.

the data in Table 1?

A.

The homozygous mutants have a lower

Blood pressure increases in the heterozygous


animals were inversely proportional to dietary

due

to

more

fluid

in

the

more of them are produced in the wild-type


animals, raising the hematocrit.
ANP directly affects the blood cells, so that
fewer of them are produced in the
homozygous mutant animals, lowering the
hematocrit.

sodium.

Copyright by The Berkeley Review

350

The Berkeley Review

Specializing in MCAT Preparation

Biology
76.

Atrial Natriuretic Peptide Experiment

Individuals processing a newly discovered human


mutation are found to be lacking the protease that
liberates ANP from the pro-ANP precursor. What
would be the probable fate of pro-ANP in these

79.

Passage XII

Applying these ANP gene data to human beings, if a


child has one parent who is not salt-sensitive (AA)
and one parent who is heterozygous (Aa) and saltsensitive, what is the likelihood that the child is also

people?

salt-sensitive (either Aa or aa), assuming Mendelian


inheritance?

A.

It would probably be degraded inside the cell


by lysosomal mechanisms.

A.

0%

B.

It would probably be released as pro-ANP and

B.
C.
D.

25%
50%
75%

activated by proteases in the bloodstream.

C.

It would probably be activated by lysosomal


mechanisms and released as ANP.

D.

It would probably be released as pro-ANP and


excreted by the kidney.

77.

Which other theoretical experimental approaches


could provide information similar to what was found
in the experiment described in the passage?
I.

Administer to wild-type animals the same antiANP antibodies and compare to untreated
animals.

II.

Administer to wild-type animals a compound


that irreversibly binds the ANP receptor and
compare to untreated animals.
III. Administer to wild-type animals a compound
that binds and inhibits processing of pro-ANP
and compare to untreated animals.

78.

A.
B.
C.

I only
I and II only
II and III only

D.

I, II, and III

Based on information in the passage, what is the


postulated role of ANP?
A.
B.
C.
D.

To lower blood pressure


To raise arterial blood pressure
To raise atrial pressure
To lower sodium output in the urine

Copyright by The Berkeley Review

351

The Berkeley Review


Specializing in MCAT Preparation

Biology

Passage XIII

Griffith's Pneumococcus

81. Based on information in the passage, which of the

PassageXIII (Questions 80-87)

following conclusions is NOT valid?

In 1928, Fred Griffith carried out the following setof


experiments using mice and the bacterium Streptococcus
pneumoniae, a Gram-positive, facultative anaerobe.

A. The DNA of pathogenic strains codes for the


polysaccharide coat seen on transformed

Experiment 1

B.

R mutants lack enzymes needed for the


synthesis of the polysaccharide coat.

C.

Injection of only the polysaccharide coat into

D.

The transformation of the R-strain to the Sstrain is permanent.

strains.

A live, encapsulated strain of virulent pneumococcus

is injected into a mouse. Themouse dies.

mice will result in their death.

Experiment 2

A live, nonencapsulated strain of nonvirulent

pneumococcus isinjected into a mouse. The mouse lives.


Experiment 3

A heat-killed strain of virulent pneumococcus is

injected intoa mouse. The mouse lives.

82.

Pneumococcus strains used

in

the

Griffith

experiments that were treated with heat were killed


Experiment 4

because of damage to:

A mixture of heat-killed virulent and live nonvirulent

pneumococcus is injected into a mouse. The mouse dies


and

is

found

to

contain

the

live pathogenic

pneumococcus.

A.
B.
C.

lipids.
proteins.
carbohydrates.

D.

nucleic acids.

Griffith concluded that the heat-killed virulent cells

contained a transforming factor that converted the live,


nonvirulent cells, to live virulent cells.

83.

The pathogenicity of pneumococcus can be


categorized by the presence or absence of an outer
polysaccharide capsule. Strains with the capsule are
pathogenic and are denoted as the S-form of the bacteria.
Nonpathogenic strains are denotedas the R-form.

All of the following statements support Avery's


conclusion EXCEPT:

In 1944, a series of experiments by Avery and his

A.

there was no loss of transforming activity after


the addition of trypsin and chymotrypsin.

B.

the addition of ribonuclease had no effect on

C.

coworkers characterized this transforming principle as a


nucleic acid of the deoxyribose type.

D.

the transforming principle.


there was no loss of transforming activity after
the extraction of protein or lipid.
there was no loss of transforming activity after
the addition of deoxyribonuclease.

80. The stain used to distinguish between Gram-positive


(+) and Gram-negative (-) bacteria takes advantage
of differences in which of the following structures?
A.

Plasma membrane

B.

Nuclear region

C.
D.

Ribosomes
Cell wall

Copyright by The Berkeley Review

84. Which of the following BEST describes the


metabolic requirements of pneumococcus?

A.
B.

The organism requires the presence of oxygen.


The organism may function in the presence or

C.
D.

The organism requires the absence of oxygen.


The organism requires the absence of nitrogen.

absence of oxygen.

352

The Berkeley Review

Specializing in MCAT Preparation

Biology

Griffith's Pneumococcus

Passage XIII

85. All of the following statements are FALSEexcept:


A.

pneumococcus translation takes place in the


mitochondria.

B.
C.

pneumococcus does not contain organelles.


pneumococcus cells are larger in diameter than

D.

most eukaryotic cells.


the pneumococcus nucleus is a double
membrane-bound structure.

86.

In an experiment that provided further evidence for


Avery's conclusion, radiolabeled elements were used
to distinguish between protein and nucleic acids.
Which of the following elements were MOST likely
used?

A.
B.
C.
D.

87.

Oxygen and nitrogen


Phosphorus and nitrogen
Sulfur and oxygen
Phosphorus and sulfur

Pathogenic pneumococcus coated with antibodies


can be rendered avirulent. Therefore, it can MOST

likely be concluded that the polysaccharide capsule


normally inhibits:

A.
B.

pinocytosis of the bacterium by P-cells.


phagocytosis of the bacterium by P-cells.

C.
D.

pinocytosis of the bacterium by macrophage.


phagocytosis of the bacterium by macrophage.

Copyright by The Berkeley Review

353

The Berkeley Review


Specializing in MCAT Preparation

Biology

Passage XIV

Drosophila eyeless Gene


88.

Passage XIV (Questions 88-94)

The eyeless (ey) mutation in Drosophila melanogaster

Assume that ey is X-linked. If ey/+ females are


crossed to wild-type males, what are the phenotypes
of the progeny?

was first described in 1915 on the basis of its distinct

phenotype, the partial orcomplete absence of the insect's

A.

compound eyes. Known alleles of the ey gene are


recessive hypomorphs, meaning that the mutant gene
product has reduced function relative to wild-type.
Heterozygous (ey/+) flies have smaller eyes, while
homozygotes (ey/ey) have almost no eyes at all. The
eyeless gene has been cloned and appears to be a
transcription factor. It shares sequence homology to a
gene in mice called small eyes. In situ hybridization
shows that ey is normally expressed only in the
embryonic tissues thateventually form the eyes. Mice and
flies are separatedevolutionarily by 500 millionyears.

B.
C.

50% of males have small eyes, 50% of females


have small eyes; all others are normal.
50% of males have no eyes, 50% of females
have small eyes; all others are normal.
50% of males have no eyes, 50% of males are
normal, 100% of females are normal.

D.

100% of males have no eyes, 100% of females


are normal.

In Drosophila, a technique has recently been


developed in which a gene of interest can be ectopically
expressed in specific tissues in which it is normally not
active. GAM, a transcriptional activator protein originally
isolated in yeast, can bind to an upstream activating
sequence (UAS) on DNA, triggering transcription of the
gene directly downstream from the UAS. "Downstream"

89.

means in the 3' direction, while "upstream" means in the


5' direction.

In Experiment 1, some fertilized eggs died as


embryos. In situ hybridization, a technique whereby
fluorescently-labeled antisense ey mRNA is allowed
to hybridize with sense mRNA in vivo, showed ey
mRNA was expressed throughout the embryo. What
would MOST likely have caused this?

protein

A.
Wing-specific
enhancer

GAIA

B.
C.
D.

UAS

Wing specific expression


ofGAL4

A substitution mutation at the UAS binding


site

Transcription of ey in
wing tissue only

A constitutive mutation in the GAL4 promoter


Expression of the normal genomic ey mRNA
An amber mutation in the ey coding sequence

Figure 1

By ligating the cDNA sequence encoding the GAL4


protein to a weak promoter and a Drosophila tissuespecific enhancer (a sequence that boosts transcription of
nearby genes), GAL4 will be transcribed only in the
specific tissue where the enhancer is active, thereby
activating only the UAS-controlled gene in that particular

90.

tissue.

Experiment I

Which of the following theories CANNOT be


supported by evidence given in the passage?

I.

A transgenic line of flies is engineered with a DNA


construct consisting of wild-type ey cDNA directly

II.

downstream from the UAS site. These flies are crossed to

another line that has a wing tissue-specific enhancer


(active only in wings) adjacent to a GAL4 sequence.
Remarkably, most of the resulting progeny have complete

III.

The ey gene is necessary and sufficient for


normal eye development in Drosophila.
The ey protein acts as a developmental
"master switch," activating transcription of
genes needed for eye formation.
Evolutionarily, eyes originated independently
in insects and mammals.

compound eyes growing from their wings.


Experiment 2

The wing-enhancer/GAL4 line from Experiment 1 is


crossed to flies containing a construct that links the mouse
small eyes gene sequence to the UAS site. The results are

A.
B.
C.

I only
II only
I and III only

D.

I, II, and III

identical to those in Experiment 1.

Copyright by The Berkeley Review

354

The Berkeley Review

Specializing in NCAT Preparation

Biology
91.

Passage XIV

Which of the following is MOST likely be the cause


of the mutant phenotype shown in the ey alleles
mentioned in the passage?
A.
B.

C.
D.

92.

Drosophila eyeless Gene

A point mutation in a non-conserved region of


the ey sequence
A two base-pair deletion near the beginning of
the ey sequence
An inversion of a segment of the ey sequence
A point mutation in a highly conserved region
of the ey sequence

Which of the following findings would be


inconsistent with the results of Experiment 2?
A.

B.
C.

Humans have a gene, Aniridia, which is 90%


homologous to ey.
Ectopic expression of ey in the mouse results
in ectopic compound eyes.
ey mRNA forms hybrids with small eyes
antisense mRNA.

D.

93.

A population of Drosophila is in Hardy-Weinberg


equilibrium with respect to the eyeless gene. Which
of the following conditions must apply?
A.

B.
C.
D.

94.

Several other organisms with visual organs


have genes homologous to ey.

There can be no selective pressure on the


population.
Genetic drift must be present in the population.
Mutations must occur in the population.
There can be no random mating between
members of the population.

A group of 1000 flies is in Hardy-Weinberg


equilibrium. If the frequency of an eyeless mutant
allele is 10%, how many flies in the population will
have small eyes?
A.

B.

C.
D.

180
810

Copyright by The Berkeley Review

355

The Berkeley Review


Specializing in NCAT Preparation

Passage XV

Neiotic Nondisjunction

Biology

96.

Passage XV (Questions 95-100)

The commonest form of aneuploidy in liveborn


infants is Down's syndrome (trisomy 21). It follows
an incidence curve that is indicated in the diagram

Maintaining of the normal chromosome number in the

gametes requires the separation of the paired

shown below:

chromosomes at the first meiotic division and the

separation of the sister chromatids at the second meiotic


division. The failure to disjoin properly at either division

40

will produce gametes with abnormal numbers of

20 i

chromosomes and offspring with an abnormal number of


chromosomes in each cell.

10

.c

Aneuploidy (having an abnormal number of

t!

chromosomes) is the most frequently observed


chromosomal abnormality in human infants and
miscarried fetuses (abortuses). The following table

2H

indicates the incidence of this condition in both live and

&
8
3
W

miscarried fetuses:

1
0.8
0.6
0.4

Incidence in

Incidence in

Live Births

Abortuses

21

1/700-1/1000

8/100 - 9/100

18

1/6000-1/9000

5/100

13

Aneuploidy
Trisomy

1/12,000-1/24,000

6/100

XXX

1/975

Rare

XXY

1/930

Rare

XYY

1/975

Rare

1/2500 -1/5000

20/100

Monosomy X

0.2

0.1

10 15

20 25 30 35 40 45 50 55

60

Maternal age

What can be determined from this graph regarding


the incidence of trisomy 21?

Table 1. Incidenceof aneuplody in livebornchildren and

I.

The incidence is lowest when a woman


conceives a child in the latter half of the
second decade of her life.

II.

The incidence is approximately the same in

in recovered abortuses.

13-year-olds and 35-year-olds.


III. The incidence is highest in the age range
closest to menarche.

A.
B.
C.

I only
II only
I and II only

D.

I, II, and III

95. Karyotypes are pictures of chromosomes that are

prepared by arresting cell development with

97.

colchicine. Paired chromosomes are spread out on a


slide and photographed to produce a karyotype.

How does the karyotype of a person with Down


syndrome differ from the karyotype of a person not
affected by this genetic disorder?

What information CANNOT be learned from the

karyotype of fetal chromosomes obtained from an

A.

amniotic fluid sample of a fetus?


B.
A.

The incidence of trisomy

C.

The presence of developmental defects

D.

The incidence of monosomy

The Down individual has three copies of


chromosome 18.

The sex of the individual

B.

The Down individual has three copies of


chromosome 21.

C.

The Down individual has one copy of

D.

The Down individual has one copy of

chromosome 21.
chromosome 18.

Copyright by The Berkeley Review

356

The Berkeley Review

Specializing in NCAT Preparation

Biology
98.

Meiotic Nondisjunction

Passage XV

Does an individual with trisomy XXY (Kleinfelter's


syndrome) have a definite genotypic sex?
A.

Yes, the person is a female, based on


genotype.

B.
C.
D.

Yes, the person is a male, based on genotype.


No, the person has clearly differentiated
genitalia of both sexes.
No, the XX chromosomes cancel out the

effects of the Y chromosome, producing a


person with genitalia not clearly differentiated
for either sex.

99.

What is the commonest form of aneuploidy among


abortuses?

A.
B.
C.
D.

Trisomy 18
Trisomy 21
Trisomy X
Monosomy X

100. It is possible through an examination of restriction


fragment length polymorphisms (RFLPs) to identify
the parent from whom a nondisjoined chromosome
originated. Given the following gel electrophoresis
pattern for a child with trisomy 21, which parent or
parents contributed the extra chromosomal material?
Mother

mm^m

Father

Child

32

si

mm ^m^m ~

\
1V.

Both the mother and the father

B.
C.

The mother only


The father only

D.

Neither the mother nor the father

Copyright by The Berkeley Review

357

The Berkeley Review


Specializing in NCAT Preparation

Biology

Genetic Information

Section IX Answers

HbA/HbS Gel Electrophoresis

Passage 1(1 - 5)

D is correct. In order to find the percentage of iron in hemoglobin, we must first find the mass of iron in

hemoglobin and then divide that value by the mass of hemoglobin itself. Information needed to do this is given in
the passage. The quotient is multiplied by 100% to give the desired answer. Since each globin protein has one iron

atom, and each iron atom has a molecular weight of 55.8 amu, we multiply 4 iron atoms by 55.8 amu to get an
overall iron mass of about 223 amu. We next divide this value by the molecular weight of hemoglobin, which is

about 65,000 amu. This gives a value of 0.0034. Multiplying this value by 100% gives an overall percentage of iron
in hemoglobin of 0.34%. The correct choice is D.
B is correct. A zwitterion is a molecule (e.g., amino acid or protein) that has an overall net charge of zero. Even

though the overall net charge is zero, the molecule can still have a charge. A change from HbA to HbS involves the
substitution of a negatively charged amino acid (Glu) for a neutral amino acid (Val). HbS is now a protein with one
less negative charge. But having one less negative charge on HbS does not mean that the molecule no longer has a
zwitterionic form. It still does, but the zwitterionic form of HbA is slightly different from the zwitterionic form of
HbS. This allows us to eliminate choice D.

Sincewe are changing the zwitterionic form of the protein when we go from HbA to HbS, we must also be changing
the isoelectric point (pi). Remember, the pi is defined as the pH at which the zwitterion is at its maximum
concentration. This allows us to eliminate choice A.

What happens to the pi if we change HbA to HbS? It's the same as what happens to the pi of a protein when one of
its negative charges is removed. The pi increases (becomes a larger value). As you have learned in general
chemistry, we can determine the value of an isoelectric point through titration. Suppose we start off with two
beakers, one beaker having a fully protonated HbA protein, the other beaker having a fully protonated HbS protein.
We begin to add a strong base (e.g., NaOH) to each of these beakers. As the base is added, we begin to titrate off the
hydrogen atoms on those amino acids in each protein that have ionizable side chains. Note that HbA and HbS are
identical, except for the presence of valine at the [56 position in HbS. In other words, HbS has one less ionizable side
chain. Valine does not have a side chain pKa value and therefore has no ionizable hydrogens on its side chain. If
HbS has one less ionizable side chain, then the pi (i.e., the zwitterionic species) for HbS will be reached slightly
later than the pi for HbA.The pi for HbS is a little higherthan the pi for HbA. The correct choice is B.
3.

C is correct. In the second paragraph of the passage it states the pi of HbA is 6.9. A buffered solution of pH 9 is
about 2 pKa units above the pi of 6.9. This tells us that HbA is negatively charged (and behaves as an anion) at pH
values greater than its pi. This can be verified by using he Henderson-Hasselbalch equation. Negatively charged
anions migrate toward a positively charged electrode. Thus, the positively charged electrode is the anode. This much
information eliminates choices A and D.

Because HbS has a valine residue instead of a glutamate residue at the (56 position, it is missing one negative charge.
We know that the pi for HbS is slightly less than the pi for HbA; the exact pi for HbS is irrelevant. Since it is
slightly less than the pi for HbA, we find that HbS, at a buffered pH of 9, is also negatively charged and migrates
toward the positively charged anode. This is exactly what we see in choices B and C. Since we cannot pick both
answers, there must be something we have overlooked.

What we have overlooked is that both HbA and HbS, at a buffered pH of 9, contain a different amount of negative

charge. HbA has one more negative charge than HbS. Since HbS has one less negative charge, it moves more slowly
toward the anode. There is less force pulling on HbS than on HbA. This is why we are able to see a separation
between HbS and HbA in the two lanes in the gel. Since HbA moves toward the anode at a faster rate than HbS, it is

farther down in the gel. In other words, Lane 2 represents HbA, while Lane 1 represents HbS. The correct choice is
C.
4.

A is correct. The passage clearly states that the difference between HbA and HbS is one amino acid. In the question,
we read that each amino acid is coded for by three bases in DNA. Those three bases represent a reading frame. As
we will see later, an enzyme "reads" the bases in a particular strand of the DNA duplex and then makes a copy
called messenger RNA (mRNA). Then ribosomes next "reads" those copied bases three at a time. Each set of three
bases is referred to as a codon, and each codon codes for a particular amino acid. As the ribosome joins the
individual amino acids together in sequence, a protein is formed.
A base-substitution mutation is the substitution of one base in a DNA sequence for another base. Recall that DNA

contains the bases adenine (A), thymine (T), guanine (G), and cytosine (C). For example, suppose we examine the
Copyright by The Berkeley Review

358

The Berkeley Review

Specializing in MCAT Preparation

Biology

Genetic Information

Section IX Answers

base sequence in the DNA coding strand for the sixth position in both HbA and HbS. In HbA, that sequence is XXX-CTC-XXX-, where "X" is a base we are not interested in at the moment. In HbS, that sequence is -XXXCAC-XXX. If we were to look at a table of the genetic code, we would find that CTC codes for glutamate, while
CAC codes for valine. In this case, there has been a base substitution (A for T) that distinguishes HbA from HbS.
Note that this single base substitution does not affect the rest of the DNA sequence (i.e., it does not alter the reading
frame) and therefore does not alter the rest of the amino acids in the HbS protein.
Let's consider what happens in a base-addition mutation. These types of mutations do alter the reading frame. For
example, suppose we add an "A" between the "CT" bases in -XXX-CTC-XXX. The new sequence, read three bases
at a time, becomes -XXX-CAT-CXX-X-. It turns out that CAT codes for valine. However, the rest of the reading
frame beyond that point has changed as well. This type of mutation does not generate the HbS molecule. A basedeletion mutation has similar consequences. If we were to remove one of the bases in the "CTC" portion of -XXXCTC-XXX-, then we would again alter the reading frame. This type of mutation also does not generate the HbS
molecule. The correct choice is A.

5.

A is correct. If the father has sickle-cell trait, then his hemoglobin composition is HbA and HbS. The mother, who
is heterozygous for HbC, has the hemoglobin composition HbA and HbC. We can set up a Punnett square as shown
below. We find that one quarter (0.25) of the children should not have any hemoglobin disorder.
Father
A

AA

AS

AC

CS

Mother

Only 1/4 of the children


are free of any
hemoglobin disorder.

HbC turns out to be the second hemoglobin disorder identified. This disorder is caused by a change in amino acids

glutamate to lysine at the p6 position. Individuals who carry the gene for HbC have a milder form of the disease,
compared to those who carry the gene for HbS. The correct choice is A.

Gregor Mendel and Inheritance

Passage II (6 - 11)

B is correct. This was the first part of Mendel's experiment, by which he was able to assure himself that the forms
of the traits he was studying were indeed constant, transmitted regularly from generation to generation. With this
constancy in mind, he felt safe to carry out the experimental crosses. In hindsight, we must consider the entire
experiment to appreciate the significance of choice B, the correct answer. From this step, Mendel could go on to
carry out crosses to get the Fi generation. If the P generation had not been true breeders (if they were
heterozygotes), he might have seen the recessive trait in the Fi generation and not realize that it is indeed recessive.
Remember, he saw only one trait in the F\ generation of true-breeding varieties of peas, and this is considered the
dominant form of the trait. The correct choice is B.

D is correct. If 1/2 of the individuals in the F2 generation are not true breeders, they are heterozygotes. If

heterozygotes are allowed to self-pollinate, they produce progeny that exhibit a dominant trait in a ratio of 3:1 over a
recessive trait. Let's take white-flowered versus purple-flowered individuals. A heterozygous individual is Ww. If
we cross Ww x Ww, we get WW, 2Ww, and ww. This shows that the expression of the dominant to recessive trait is
in a ratio of 3:1. The correct choice is D.

A is correct. The second filial F2generation comes from the self-pollination of the Fi generation. Recall that all Fi
individuals are heterozygotes, because they are the product of two different, but true-breeding parents. Therefore, if
we cross Ww x Ww, we get WW, 2Ww, and ww. From this cross, it becomes clear that the ratio of homozygotes to
heterozygotes is 1:1. This is different from asking for the ratio of phenotypic expression. Since the question is
asking about the genotypic makeup, 1:1 is correct. The correct choice is A.

B is correct. A human cell that is in prophase of meiosis I has a replicated set of chromosomes, but homologous
pairs have not yet separated. They still have 46 chromosomes. Even though the amount of genetic material in the
cell has been doubled, the number of chromosomes remains the same. One could call the chromatin ploidy number
4, but still only two sets of chromosomes remain in the cell. The choice becomes diploid, with n = 2. The correct
choice is B.

Copyright by The Berkeley Review

359

The Berkeley Review


Specializing in MCAT Preparation

Biology
10.

Genetic Information

Section IX Answers

B is correct. We are performing a test cross to determine the genotype of the flower. We can do this by crossing it
with a plant of known genotype (the white flower must have the genotype ww). If the plant in question is
heterozygous, we have the following cross: Ww x ww. The possibilities are 2Ww, and 2 ww. Therefore, if the plant
in question is heterozygous, we should expect to see that 1/2 of the plants have purple flowers. The correct choice
isB.

11.

C is correct. The pods themselves do not segregate and contain uniform types of peas as shown. Each pea within a
pod is an individual case. Let us consider the other choices. The ratio of wrinkled to round seeds is correct; but
again, not all the wrinkled seeds would be found in one pod. The number of progeny in the F2 generation is correct,
as only two of the pods from Fi are used to create the F2 generation. Finally, we have no reason to believe that the P
generation are not true breeders. In fact, we have evidence to the contrary. That evidence is that the peas in the Fi
generation are all uniform. Again, each pea, not an entire pod, is an individual case where segregation of alleles is
occurring. The correct choice is C.

Passage III (12-17)


12.

Incomplete Dominance and Codominance

B is correct. The following Punnett square shows why. Each pink parent is Rr. This gives 1 homozygous red (RR),
2 homozygous whites (rr), and 2 heterozygous pinks (Rr). The correct choice is B.
R

RR

Rr

Rr

rr

13.

D is correct. Homologous pairs are chromosomes paired with each other; they are not specific gene loci. That
means choice A is incorrect. Pheromones are substances secreted by one member of a species that affect other
members of the same species, so choice B is incorrect. Choice C is a nonsense answer and is incorrect. Alleles fit the
definition given. The correct choice is D.

14.

A is correct. A monohybrid cross involves individuals that differ only with respect to the alleles at a single locus.
The flower example assumes only one trait, color, distinguishes the flowers. Choice A is correct. A test cross
involves a homozygous parent and an unknown parent. By examining the offspring, one can determine the genotype
of the unknown parent. Choice B is incorrect. A dihybrid cross involves individuals that differ at two alleles. This
would yield a 16-box Punnett square, not a 4-box Punnett square, so choice C is incorrect. Inbreeding means
crossing closely related individuals. The parents of the Fi generation are not closely related. Choice D is also
incorrect. The correct choice is A.

15.

A is correct. To get a pink flower with curly leaves, the genotype must be Rr 11. The Rr yields a pink color due to
incomplete dominance. The 11 yields a curly leaf due to classical dominance. Make a 16-box Punnett square to prove
the answer to yourself. Only 4 out of the 16 offspring would be pink flowers with curly leaves. This is a 25%
probability, or 0.25 since probabilities are expressed as a decimal fraction of 1. The correct choice is A.

16.

C is correct. The passage tells us that the A and B proteins are codominant, and that O is recessive to A and B. The
text of the question tells us that the Rh factor is classically dominant/recessive, with the + being dominant. So the
queen contributes A and B alleles, and the king contributes B and X (either B or O) alleles. The offspring of this
king and queen must be either AB, AO, BB, or BO. Since the presumed heir is A, this is compatible with the AO
choice. So far, so good. However, the answer lies in the Rh-factor part. Both the king and the queen are homozygous
recessive, or negative, for the Rh factor. Since the presumed heir is Rh positive, this allele had to come from another
father. Neither the queen nor the king could have provided it. The presumed heir is not the child of the king. Choices
A and B are incorrect. The passage and the text of the question did provide what is needed to answer the question, so
choice D is also incorrect. The correct choice is C.

17.

A is correct. If neither the A nor B proteins are expressed on the RBC of an individual, then that person is type O.
Type O blood is recessive in the ABO blood group system. The person with type O blood is homozygous recessive.
A child born of two people who are homozygous recessive for the same trait will also be homozygous recessive.
Since neither parent expresses A or B, the child cannot express them, either. Statements II and III are incorrect.
Incidentally, the same reasoning holds true for the Rh factor. The correct choice is A.

Copyright by The Berkeley Review

360

The Berkeley Review


Specializing in MCAT Preparation

Biology

Genetic Information

Passage IV (18- 23)


18.

Section IX Answers

DNA Replication

C is correct. This can be calculated from the information given in the passage. The genome of E. coli is said to

contain 4.7 x 106 nucleotide base pairs. It isalso stated in the passage that each replication fork polymerizes ata rate
of 500 nucleotides per second. Since there are two replication forks (the replication is bidirectional), the total rate is
about 1000 nucleotides per second or 60,000 nucleotides per minute. Simple division gives one an answer closest to
choice C, 80 minutes. If one did not take into account the two replication forks, one would come up with 160
minutes, or choice A. Choices B and D can be eliminated with the above information. The correct choice is C.

19.

A is correct. Remember, the eukaryotic chromosome has so much DNA that multiple origins of replication are
needed to have any sort of timely, complete replication. Let us consider the other possibilities: Choice B is incorrect,
because the chromosome shown is circular. Circular chromosomes are found in prokaryotes; the question specifically
asks for a eukaryotic chromosome. Consider choice C. While this chromosomes is linear, there are no replication
bubbles, and the question asks for what a chromosome will most likely look like during DNA replication. Consider
choice D. This answer can be tempting, but one must remember that eukaryotic chromosomes have multiple
transcription bubbles, and this shows only one. Yes, this may happen and does happen; but the question asks for the
best representation, which is the chromosome with multiple replication bubbles. The correct choice is A.

20.

C is correct. Chromatin is DNA wrapped around histone proteins. The appearance is that of beads on a string. One
speculative model claims that the histone is split in half while the DNA is replicating. The two halves stay on one
parent strand as the new DNA is being synthesized. After the DNA is completely replicated, the two halves
recombine to form the functional histone.

Consider the other possibilities: Choice A is incorrect, because we are talking about the replication of a single
chromosome, and the presence of other chromosomes will have no effect on replication. Choice B is incorrect,
because the enzymes involved in polymerizing are not prejudiced towards whether the bases are coding (exons) or
are noncoding (introns). They read the template and insert the correct base pair in the growing strand. Choice D is
incorrect, because the mere location of the DNA does not affect the rate of polymerization. We need to be looking at
the actual structure of eukaryotic DNA compared to prokaryotic DNA. When we do this, we remember that

eukaryotic DNA is packaged with proteins to make chromatin. While choice D is not the correct answer, the
principle behind the claim is extremely important and represents one of the fundamental differences between
eukaryotes and prokaryotes. Eukaryotes have their DNA enclosed in a nuclear envelope, while prokaryotes do not
have a nuclear envelope. The correct choice is C.

21.

B is correct. This question is very straightforward and calls upon your knowledge of the cell cycle. The replication
forks will become activated when DNA is undergoing replication. This occurs in the S division of the cell cycle. Gi,
S, and G2 are known collectively as interphase. Interphase usually takes up about 90% of the total cell cycle time.

Gi and G2 are stages where biosynthetic activities of the cell take place, preparing the cell for DNA replication and
division. S, as already stated, is where the DNA replication takes place. Finally, M is the mitotic stage where we see
both a nuclear and a cellular division. Based on this information, we are most likely to see the replication forks

activated during the S phase of the cell cycle. The correct choice is B.

22.

B is correct. The question tells us we have the addition of 3H-uridine, which will be incorporated not into DNA, but
into RNA. RNA is initially made in the nucleus, but will eventually be removed from the nucleus and enter the

cytoplasm. In the cytoplasm, it will undergo translation to make a functional protein. Therefore, we are most likely
to see the label in both the nucleus and the cytoplasm. Based on this information, we can easily eliminate choices A

and C. They are both true but incomplete, and therefore not the best answers. Choice D is incorrect, because the
membrane will not have nucleotides incorporated into its structure. The nuclear membrane is composed of primarily

phospholipids and large protein granules making up the nuclear pores. The correct choice is B.
23.

C is correct. Statement I is correct, because we see the presence of multiple replication bubbles, which should

immediately suggest a eukaryotic organism. Statement II is incorrect, because there is no evidence for this claim.
Yes, we see more label in B, but half of the label is less dense, indicating some sort of change in the level of
radioactivity. The level of radioactivity is not an indication of the rate of polymerization. If we look carefully, the
dense regions are the same in both A and B, indicating that the polymerases are not different, but exactly the same.
Therefore, statement II is false. Statement III is correct, because in B we see more silver grains from the

radioactivity. That means that more incubation occurred. However, we can definitely recognize the low level of
radioactivity. That must mean that there was a pool of unlabeled nucleotides added to the medium. When this
happens, the labeled and unlabeled are competing for the polymerase, and we should see a smaller level of
radioactivity. This is what we see by the lesser density of silver grains. The correct choice is C.
Copyright by The Berkeley Review

361

The Berkeley Review

Specializing in MCAT Preparation

Biology

Genetic Information

Color Blindness

Passage V (24 - 29)


24.

Section IX Answers

D is correct. Gene duplication involves copying the gene or genes in the DNA that are already there. If gene
duplication occurs, then replication of those genes into new duplexes of DNA will give the same DNA sequences.
Similarly, the transcription of DNA into mRNA will be similar, as will the translation of the mRNA into a
polypeptide chain (composed of amino acids). Ifsimilar mRNA sequences can be synthesized, then similar tRNA
(transfer RNA) andeven similar rRNA (ribosomal RNA) sequences can be synthesized. Therefore, choices A and B
both occur. We can eliminate them. Duplication of genes results in more of the same types of genes. And this means

that more (of the same) genes can begin to diverge. Each of the newly duplicated genes canexperience its own form
of mutation, independent from the other duplicated genes. This is how natural selection begins to take hold. Since
choice C is correct, we can eliminate it. If we duplicate a gene, we will have two genes of equal size and not one

gene which is twice as long. Therefore, when a transcript of mRNA is made, it will still be as long as the gene that
was duplicated and not any longer. The correct choice is D.

25.

D is correct. A genetic locus refers to a given gene location on a chromosome. Males have one X chromosome (and
one Y chromosome), while females have two X chromosomes. The frequency for color blindness in the male is

given as 8% (0.08) in the passage. In order for females to be color-blind, they must be homozygous for the
condition. The third paragraph of the passage says that red-green color blindness is sex-linked and recessive. What is

the chance of a female being color-blind (i.e., homozygous for the allele)? It would just be a1, or (0.08) x (0.08),
which is 0.0064 or 0.64%. The correct choice is D.

26.

C is correct. Paragraph 3 of the passage says that about 6% of all males who are color-blind have a deutan defect,
which results in abnormal synthesis of the green-sensitive pigment. The percentage of females that would be deutan
color-blind is (0.06) x (0.06) = 0.0036 = 0.36%. Note that the percentage of females who are protan (red-sensitive
pigment defect) color-blind is (0.02) x (0.02) = 0.0004 = 0.04%. If we add 0.36% and 0.04%, we get 0.40%, the
incidence of red-green color blindness among females in the United States. The correct choice is C.

27.

B is correct. In the passage we learn that about 6% of all males who are color-blind have the deutan defect while 2%
of all males who are color-blind have the protan defect. In a heterozygote, the dominant allele is /; and the recessive
allele is q. We see that q = 2% = 0.02 and that /; = 98% = 0.98. From the Hardy-Weinberg equation, we find that !pq
= 2(0.98)(0.02) = 0.0392 x 100% = 3.92% or about 4%. Note that the percentage of heterozygote women who are
carriers of deutan color blindness is 2(0.94)(0.06) = 0.1128 x 100% = 11.28% or about 11%. If we add 11% + 4%,

then we get 15%, which is roughly the percentage of women who carry the color blindness defect. The correct
choice is B.

28.

D is correct. If we let the dominant allele for deutan color blindness be represented by D, then the recessive allele
can be represented by d. If the husband is deutan color-blind, he has the recessive allele, d, on his one and only X
chromosome. His wife has normal color vision. She has two X chromosomes, and they both carry a dominant D
allele.
Father

Mother

(colorblind)

(normal)

I
XdY

xDxD

a
xxd

xDy

Daughter

Son

(carrier)

(normal)

If they have a daughter, then she will receive one X chromosome with the D allele from her mother and one X
chromosome with the d allele from her father. The daughter will be heterozygous for the deutan trait. In other words,
she will be a carrier. If they have a son, the mother donates an X chromosome with the D allele but the father now
donates a Y chromosome, which has no defect for color blindness. The son will thus have normal color vision. The
correct choice is D.
29.

C is correct. The easiest way to follow the alleles is to draw a pedigree as shown below. The symbol D represents
the dominant form of the allele for the deutan trait, while the allele d represents the recessive form. In order for the

Copyright by The Berkeley Review

362

The Berkeley Review


Specializing in MCAT Preparation

Biology

Genetic Information

Section IX Answers

trait to be expressed, the genotype must be dd in the female or d- in the male. The symbol C represents the dominant
autosomal allele required for proper development of the cones. If the genotype cc is present, the individual will have
complete color blindness.
Father

Mother

-O

(does not have deutan trait)


(does not have color vision)

XDY

(will express deutan trail)


(will have color vision)

xdxd
C-lrC

IE
Daughter
(carrier for deutan irait)
(will have color vision)

Son

XdY

X&Xd

(will express deutan trail)

(will have color vision)

By examining the pedigree, we see that both children will have color vision. However, only the son will show redgreen color blindness by expressing the deutan trait. The correct choice is C.

Passage VI (30 - 36)


30.

Alleles and Coat Color

B is correct. The last paragraph of the passage says that if a hamster is homozygous for the a allele (i.e., a/a), then
its coat will be white. In other words, the hamster will be albino. The same paragraph also states that the normal

(wild-type) allele is usually designated as +. If a hamster has a + allele at one of the two loci, then the coat color will
be normal. This must mean that the + allele is dominant over the a allele, or that the a allele is recessive to the +

allele. If the a allele and the + allele were codominant, we would see both types of coat color. If the + allele showed

incomplete dominance, we would also expect to see both types of coat color. Also, the a allele, as written in the
answer choices, is written in the lower case. This is the nomenclature that has been agreed upon by geneticists to
indicate a recessive allele. The correct choice is B.

31.

C is correct. A true-breeding strain of black hamsters will always produce black hamsters (unless there is a mutation
of some type). In order for progeny hamsters to be purely black, they must always receive a B and a + allele from

each parent. Therefore, each parent and the progeny hamsters must have the genotype B/B +/+. The correct choice
is C.

32.

C is correct. In the third paragraph of the passage, we see that the recessive allele b results in brown coat color
when it is present in the homozygous (b/b) condition. The fourth paragraph says that if the hamster has the allele a

present in the homozygous (a/a) condition, then the coat color will be white. The genotype a/a means that these
hamsters do not have any pigmentation (that is why they are white). The correct choice is C.
33.

A is correct. We can use the outline shown below to help us determine the coat colors of the first filial (Fi)
generation progeny.
b

/>'
( 1

Black coat

color

cz

White coat
color

f B

Parents (P,)

C amete
fo rmation

a
I

Gamete
formation

o
+
XT

Cross

First filial (F,) C


generation
CI

Copyright by The Berkeley Review

+
a

363

Black coat

color

The Berkeley Review

Specializing in MCAT Preparation

Biology

Genetic Information

Section IX Answers

We see thatin the Fi offspring, the genotype is B/b +/a. Since B is dominant overb, the coat color will be black.
Since the wild-type allele + is dominant over the a allele, the coat color will benormal, which in this case is black. If
therehadbeen twoa alleles (e.g., a/a), thecoatcolor would have beenwhite. The correct choiceis A.
34.

B is correct In orderto estimate the genetic mapdistance between the two genetic loci, we must use the equation
given in the passage, which is:

_ Number of Crossover Indidivuals in F2

CO =

x 100%

Total Number of Individuals in E,


We need to determine the number of crossover individuals in F2, and we need to determine the total number of

individuals in F2 before we can estimate the genetic map distance. Refer to Experiment II.

In Experiment n, we learned that one hamsterfrom the Fi generation of Experiment I was back-crossed to the parent
with the double recessive genotype (b/b a/a). We determined that the genotype of the hamster from the Fi
generation is B/b +/a. The result of this cross will give the F2 progeny, namely 66 black hamsters, 34 brown
hamsters, and 100 white hamsters. The total number of hamsters is 200. At this point we know the total number of
individuals in F2. Our equation now becomes:
CO =

Number of Crossover Indidivuals in F2

x 100%

200

This alone does not tell us the number of crossover individuals we have. Let us next assume that the two loci are

linked, and let us also suppose that no crossover events took place. What would we expect? Let's look at this backcross as outlined by Experiment II. We can set up the chromosomes as shown below and generate the F2 progeny.
First filial (F,)
generation
B

Black coat

<^>ifri"^0"

color

32E

^^^.^.-^riiJWkc^Mx4*>a

Gamete

formation

'

l^art^.j^jrict

l-'j-fxinrfrw*.*

White coat

color

Gamete
formation

o
+

S>

a_

^^j3B3He^^^3Ss^

Cross

car.w^-^o.w.'

" } Gamete

iTF^FTfuSP*

V
s

B
Gamete

rfjSsp^<;**

B
6s&M3 -.* *Wk-1

>^

<

b
Gamete

<3 im*

a
1 I

Black coat
color

b
f~*!*i!WtL*

It-. >".K-

a
|<i/.Si x.B-4*>a

f>

r4iB ^

^Z53TWTa-*Ju-^C l.*rj.i+r,-*

.>

1 ]

. . > . SS^JJ

White coat
color

F2 progeny from test cross

Copyright by The Berkeley Review

364

The Berkeley Review

Specializing in MCAT Preparation

Biology

Genetic Information

Section IX Answers

If no crossover took place, we would expect to find half of the progeny B/b +/a and half the progeny b/b a/a. In
other words, we would expect that 50% of the hamsters would be black and 50% would be white. We would not

expect to find any brown hamsters. But this is not what is observed in the results of Experiment II, which was 34
brown hamsters. This tells us that the brown hamsters are the crossover or recombinant hamsters. If we use this

value in our equation, we get:

% CO = 34- x 100% = 17%


200

Since one genetic map unit (m.u.) gives a recombinant frequency (RF) of 1 percent, we have a recombination
frequency of 17% or 17 m.u., which is choice A. But choice A is not the correct answer.
Let's see what happens if a crossover event does take place. In paragraph 4 of the passage, we read: "If there is
crossing over between any two non-sisterchromatids, intrachromosomal recombination results." Crossing over must
take place between non-sister chromatids. One possibility of how this could happen is shown below in Crossover 1:
B

Wi!U**;*lj>**4-l v^^ssSwESS^KaSPTi

<52^W^*;<3i:*fef l^*sii*t*rerW!5*

^KSJPS*t56J<.fcWi|

&r -J^.a-.-.|

lir.aaJA-,Hi-s5w-^B:3e*3i

b
Crossover 1

^Crossover

b
<ss;";w&&8mDt&H V*it*iaB-JiD

&?^&WK?7x^trx&f)4-1 J^***t "y v-fr5BProC%

),;.w***.*~0

Black coat
color

C*'*J*!-V(.|

ft*h!.*nU*H-ffc.aa

<S >'H <>*'..ttl

|i^MmbiHmA

White coat
color

There is also another possible crossover event, as shown below in Crossover 2. Again, this crossover is taking place
between non-sister chromatids:

a_

czai

Crossover 2

B \/

^Crossover

White coat
color

f?-'Ma^A'ai;Piv

CEE3

wjy*uru

Brown coat
color

In Crossover 1, we produced blackand white hamsters in equal numbers. In Crossover 2, we now produce brown
and white hamsters in equal numbers. Wecan clearly tellfrom the phenotype which are the hamsters are brown, and
we know that these brown hamsters can arise only from crossing over. What about the white hamsters? We can get
white hamsters from non-crossover events (see above). How can we tell which white hamsters came from noncrossover events and which came from crossover events? We can't! Here is where the insight comes in. In order to

get a more accurate number of recombinants, we must assume that there are also 34 white hamster recombinants.
This is because when we produce one brown hamster recombinant, we alsoproduce one white hamster recombinant.
And since we have 34 brown hamster recombinants, we must add an additional 34 white hamster recombinants to

the total numberof crossover individuals in the F2 generation. (Why don't we need to do this for Crossover 1?)

We can now use our equation to calculate the genetic mapdistance between the two loci:
CO

= 2_x_34 x 100% = 34%


200

In other words, there are 34 m.u. between the genetic loci. [Note: Since there can also be double crossover events,

the actual genetic distance between the loci is probably greater than 34 map units. What we have calculated is the
minimum map distance.] The correct choice is B.
35.

A is correct If the genetic loci were not linked, we would expect to see a recombination frequency in the F2
generation of 50% (from the third paragraph of the passage), which means that there was interchromosomal
recombination(i.e., between the chromosomes). If the recombination frequency is less than 50%, it means that there

Copyright by The Berkeley Review

365

The Berkeley Review


Specializing in NCAT Preparation

Biology

Section IX Answers

Genetic Information

was intrachromosomal recombination (i.e., within the chromosome). If there is intrachromosomal recombination,
then genetic loci must be linked.

There is another way to look at this: We would expect half of the hamster offspring to have white coats, which they
do. What about the other half of the hamster population? If the two genetic loci were not linked, we would expect to

see this half of the hamster population having 50% black coats and 50% brown coats. But when we look at the

values given in Experiment II we do not see this. Instead, we see that there are almost twice as many hamsters with
black coats as there are with brown coats. In order for this to happen, the genetic loci must be linked. If they were

not linked, they could be called unlinked (i.e., they assort independently). The genetic loci are not entirely epistatic,
because we do see black and brown hamster coats. This means that the a allele is not homozygous (a/a) but rather

heterozygous (a/+). Thea allele has to be homozygous before it can influence a different genetic locus. Besides, the
B and b genetic lociare notepistatic to the a allele. The correct choiceis A.
36.

B is correct. In the sixth paragraph of the passage, we see that neither the black nor the brown coat color of a
hamster will be expressed, if the hamster is homozygous for the a allele. For example, consider these chromosomes:
B

d7

.....

If the a allele were not present, the coat color of the hamster would be black. This is because the B allele is dominant
over the b allele. However, the presence of the a allele, in the homozygous form, suppresses the activity of the allele
for the black coat color. Because the a allele is recessive (note that it is written in a lower case), we can call this

recessive epistasis. As stated in paragraph 2 of the passage, incomplete dominance and codominance are concerned
with heterozygotes. Both of these characteristics involve expression of either one of the same allele (e.g., B or b). In
the case of epistasis, allele a influences some other allele, such as allele B or b. The correct choice is B.

Passage VII (37 - 43)


37.

DIN A Calculations

D is correct. Since the average molecular mass of an amino acid residue is HOD, the protein has 40,000/110 = 360
residues. This is specified by 3 x 360 nucleotides. Each base pair has an average molecular mass of 660 D. Hence,

the molecular mass of the DNA is 660 x 1080 = 700,000 D (or 7 x 105 D). The length of this molecule, considering
that B-DNA has a rise along its helix of 3.4 A per base pair, is 3.4 A x 1080 = 3700 A = 0.37 pm. The correct
choice is D.

38.

C is correct. There are 4.0 x 106 bp in the E. coli chromosome, and it takes40 minutes to replicate the chromosome.
Thus, (4.0 x 106 bp)/(40 min) x (1 min/60sec) = 1,666 bp/sec replicated. But since there are two replication forks,
there are about (1,666 bp/sec)/2 or about 833 nucleotides per second added. The closest answer is 850 nucleotides.
The correct choice is C.

39.

B is correct. At 16 pm/min, each replication fork travels 4800 pm or 4.8 x 10"3 m in 5 hours (300 minutes). To
replicate the entire content of DNA (1.2 m) in this interval, there must be (1.2m)/(4.8 x 10"3 m/replication fork) =
250 replication forks. The correct choice is B.

40.

D is correct. In a rich medium, a second round of bidirectional replication begins at the origin when the first round
is only half-completed. This second initiation results in four new replication forks, making a total of six.

o
Ori site

2 replication forks

6 replication forks

Thus, one round of replication is competed every twenty minutes, and each daughter cell at division receives a
chromosome that is already half-replicated. The correct choice is D.
Copyright by The Berkeley Review

366

The Berkeley Review


Specializing in MCAT Preparation

Biology
41.

42.

Genetic Information

Section IX Answers

C is correct. Okazaki fragments are about 1000 to 2000 nucleotides in length. The E. coli chromosome consists of

about 4 x 106 bp. Thus, E. coli must produce about 2000 to 4000 Okazaki fragments. The correct choice is C.

C is correct. Since DNA is antiparallel, the complementary strand must be 5'-pGpTpCpTpApT-3*. The correct
choice is C.

43.

C is correct. In the next-to-last paragraph in the passage, it we read that "...all known DNA polymerases require a
primer before new DNAcan be synthesized...." And in the last paragraph of the passage, we are told that "Oncethe
primer has been established, DNA polymerase III begins to synthesize DNA in the 5' to 3' direction." From this
information, we can assume that DNA polymerase I must also add its nucleotides in the 5' to the 3' direction (i.e., at
the 3' end) of a growing polypeptide chain and not in the 3' to the 5' direction (i.e., at the 5' end). This allows us to
eliminate statement II, which in turn allows us to eliminate choices A, B, and D. The correct choice is C.

Passage VIII (44 - 50)


44.

Semiconservative DNA Replication

C is correct. There were two ways to answer this question. First, you could have remembered that the base-pairing
arrangement in a DNA double helix (a duplex) follows the rules that adenine (A) base pairs with thymine (T)
through 2 hydrogen bonds and that guanine (G) base pairs with cytosine (C) through 3 hydrogen bonds. The only
base-pairing arrangement in the answers that shows this is the bonding between cytosine and guanine. The adenineguanine and cytosine-thymine base-pairing is not allowed. This allows us to pick choice C as the answer. If you did
not remember this, then you needed to rely on information in the first paragraph of the passage. It was stated that the
purine bases were adenine and guanine, while the pyrimidine bases were thymine and cytosine. It was also stated
that complementary purine and pyrimidine bases are linked together through hydrogen bonding. In the answer we
need to find that base pair that depicts a purine bonding with a pyrimidine. The only choice is the cytosine-guanine
pair. The adenine-guanine pair represents two purines, while the cytosine-thymine pair represents two pyrimidines.
The correct choice is C.

45.

D is correct. The second paragraph of the passage says that "conservative replication would conserve the integrity
of the parental strands in the DNA duplex after replication." In other words, after one round of DNA replication, we
would see one DNA duplex that contained both parental strands, and another DNA duplex that contained the two
new daughter strands. This is shown schematically below:

When the second round of replication begins,


One round of

both duplexes are now treated as parental

replication

strands each giving rise to duplex daughter

3> I

strands of DNA. Thus, in the end we have

and

four duplexes of DNA. Two of the duplexes


contain all parental DNA, and two of the
duplexes contain all daughter DNA.

Duplex DNA

As the second round of DNA replication begins, we now treat both duplexes as being parental strands. After the
second round of replication, we would find two DNA duplex that contained all parental DNA and two DNA
duplexes that contained all new daughter DNA. The correct choice is D.
46.

C is correct. This question is designed to get you to think about the fundamental components of DNA. The answer
cannot be obtained from the passage. Instead, it must be deduced from your fundamental knowledge of DNA. As
mentioned in the question, DNA is composed of nucleotides. Each nucleotide contains a base (either thymine or
cytosine, which are pyrimidines; or adenine or guanine, which are purines). Each nitrogenous base is attached to a
ribose ring (i.e., a pentose sugar). Each ribose is attached to a phosphate. Each of those ring systems could

incorporate 15N. The ribose ring does not contain any nitrogen, and it seems like a good answer choice. But what
about the acetal linkages? The linkage between the sugar and the nitrogenous base is referred to as a glycosidic
bond. In particular, it is an N-glycosidic bond because of its attachment to a nitrogen atom in the base. This
particular type of linkage is also considered an acetal linkage. And since the acetal linkage contains a nitrogen atom

thatcould be replaced by 15N, wecan eliminate it as a possible answer. The correct choice is C.
47.

C is correct. The passage mentions that the CsCl solution is less dense near the top of the test tube and denser near

the bottom of the test tube. If we were to centrifuge the DNA that was labeled exclusively with l5N, we would find
Copyright by The Berkeley Review

367

The Berkeley Review


Specializing in MCAT Preparation

Biology

Genetic Information

Section IX Answers

that its band would appear at a lower position in the test tube compared to DNA that was exclusively labeled with
14N. This is what is represented by the control test tube (see below).

All ,4N DNA CZj>


All 15N DNA C^>

<^Z1 Hybrid 14N/15N DNA

W
Control

If we analyze the DNA after one generation following the incorporation of 14N into the growth medium, then that
DNA would be neither exclusively all heavy (15N) norexclusively all light(l4N). Instead, the DNA would represent
a hybrid of 14N and l5N DNA. The banding pattern (after analysis with ultraviolet absorption) would be
intermediate between the two bands shown in the control. Note that we do not see this pattern in any of the answer
choices.

After two rounds of DNA replication, we find duplexes that contain the hybrid DNA (14N and l5N) and duplexes
that contain exclusively light DNA (l4N). We would expect to find two bands: One intermediate between the all-l4N
band and the all-l5N band (characteristic of the hybrid DNA), and one that is the same as the all-I4N band on the
control. There is only one answer choice that gives this pattern. The correct choice is C.
48.

A is correct. As described in the passage, one round of semiconservative replication would give two duplexes of

DNA, each containing one strand with ,5N and one strand with l4N. After the second round of replication, each
hybrid duplex would generate two duplexes. One duplex would still be a hybrid (14N-I5N), but the other duplex
would contain DNA that is all I4N. This is outlined below.

Parental

duplex DNA

1st Generation

2nd Generation

In the second generation, note that there are four duplexes of DNA, for a total of eight strands of DNA. Out of these

eight DNA strands we see that two are ,5N and six are 14N. This gives a ratio of l4N to 15N of 3:1. The correct
choice is A.
49.

A is correct. The characteristic that distinguishes one atom from the next is the number of protons contained within
an atom of a particular element. The atomic mass unit (amu) of a proton is about 1.0073. If the number of protons
changes, the atom changes, and therefore its elemental name changes as well. Any mass difference between atoms

of the same element is due to the difference in the number of neutrons. These atoms are referred to as isotopes. The
amu of a neutron is about 1.0087. The natural isotopes found in the nitrogenous bases are hydrogen (atomic mass of

1.01), 12C (12.00), 14N (14.01), and 160 (16.00). The corresponding heavy isotopes would be deuterium (2.01), 13C
(13.01), 15N (15.00), and 180 (18.00). Heavy isotopes have a greater density, because they have one or more
neutrons in their nuclei. Electrons have an amu of about 0.0006 and do not significantly effect the mass of an atom.
The same number of water molecules would surround l5N DNA as would surround 14N DNA. The correct choice
is A.

Copyright by The Berkeley Review

368

The Berkeley Review


Specializing in MCAT Preparation

Biology
50.

Genetic Information

Section IX Answers

B is correct. First, note that the polymers of DNA and RNA in the question have the same base composition.
Therefore, we must look elsewhere to find differences in the density of these two nucleic acids. It is important to
know the basic differences between DNA and RNA. We know that DNA contains adenine (A), guanine (G),
cytosine (C), and thymine (T). RNA contains the same bases, except it replaces thymine with uracil (U). The
question avoided the differences in the structures of thymine and uracil by leaving them out of the question. One
other important difference is the presence of a hydroxyl (OH) group at the C-2' position of the ribose ring of RNA.
DNA does not have this C-2' hydroxyl group (hence the name "deoxyribonucleic acid" for DNA). This tells us that
each nucleotide of the DNA polymer is missing an oxygen atom. In other words, the RNA polymer has ten more
oxygen atoms than the DNA polymer. The RNA polymer is heavier and shows a greater density. This allows us to
eliminate choices A and C. Consider choice D for a moment. Nucleic acids do not have positively (or negatively)
charged nitrogenous bases at physiological pH. The only charge they show is on their negatively phosphate groups.
We can eliminate choice D. The positively charged cesium ion (Cs) can bind to the negatively charged phosphates
of both DNA and RNA. It can also bind to those free hydroxyl groups at the C-2' position of RNA as well. The
correct choice is B.

Passage IX (51 - 58)


51.

DNA Structure

A is correct. Examine the bases shown in Figure 1 of the passage. The purine and pyrimidine bases can exist in
alternate forms called tautomers. Keto groups can be converted to enol groups, and enol groups can be converted to
keto groups. Similarly, amino groups can be converted to imino groups, and imino groups can be converted to amino
groups. Selected examples are shown below:

UN

H2N

Guanine (keto)

H?N

Uracil (keto)

Guanine (enol)

Adenine (amino)

Uracil (enol)

Adenine (imino)

In order to form the enol in one of the bases, we must first have a keto group in the molecule. Not only that, but we

must have a hydrogen atom on a neighboring nitrogen atom that can leave and participate in the formation of the
enol. All of the bases except adenine have keto groups. Even though cytosine has a keto group, there is no hydrogen
atom on a neighboring nitrogen atom that can participate in enol formation. Therefore, the bases adenine and

cytosine cannot form enol derivatives. The only bases that can form enol derivatives are guanine, thymine, and
uracil. What about adenine? This base has an amino group at the C-6 position of the purine ring. Tautomerism leads
to the imino form. The correct choice is A.
52.

C is correct. DNA and RNA are polynucleotides that contain repeating nucleotide units linked together through

phosphodiester linkages. Remember, a nucleotide is composed of a base, a ribose sugar, and a phosphate group. The
phosphodiester linkages are formed between the 5' carbon of one ribose ring and the 3' carbon of the adjacent ribose
ring (see below). In order for the acidic phosphate group to be positioned between the two ribose rings, there was a
loss of water during the reaction. Anhydrous refers to the loss of water. Therefore, this phosphodiester linkage can
also be called an acid-anhydride linkage. At neutral pH, each nucleotide contains a negatively charged phosphate
group.
~~~~-o
Base

Phosphodiester
linkage
H;isc

3' end of the polymer

Copyright by The Berkeley Review

369

The Berkeley Review

Specializing in MCAT Preparation

Biology

Genetic Information

Section IX Answers

If the nucleotides were deoxy at the C-3' position, the DNA polymer would not be able to form, because the oxygen
atom of the hydroxyl group at the C-3' position attacks the next incoming nucleotide's phosphate group in order to
form phosphodiester linkage. The correct choice is C.
53.

D is correct Selection of the correct answer depends on two things. First, we must obtain the correct sequence of
the desired DNA strand in the 3' - 5' direction. Following the Watson and Crick base-pairing rules, we get the
following sequence:
3'-T-A-C-G-G-C-T-A-5'

Notice that in all four answer choices the DNA bases read the same in the 3' > 5' direction. Be aware of the

sequence from both ends of the polymer. Just because we are looking for a sequence that reads from the 3' - 5'
direction does not necessarily mean that it will be written with the 3' end on the left and the 5' end on the right.
Once we have determined the sequence, the second thing we need to consider are the 3' and 5' ends. The 3* end of a
DNA polymer represents the C-3' hydroxyl group on the ribose ring. The 5' end of the DNA polymer represents the
phosphate group attached to the C-5' hydroxyl group of the ribose ring. We are looking for an answer choice that
bears a phosphate (P) at the 5' end and a hydroxyl (OH) at the 3' end. When writing out a DNA sequence, it is
sometimes customary to include the phosphate group at the 5' ends of the bases. The 3' hydroxyl groups are left out

of the picture. They are understood to be at the 3' end. In choices A and B, we find a phosphate at the 3' position,
which is not allowed. Therefore, we can eliminate the first two answers. In choice C, the phosphate at the 5' end is
missing. We can eliminate this answer, too. In choice D, we see that the phosphate group is at the 5' end. The 3' end
shows no phosphate group. It does not show a hydroxyl group either, but the hydroxyl group is understood to be
there. The correct choice is D.

54.

D is correct The distance between base pairs is 0.34 nm. The helix undergoes one complete turn (twist) every 3.4
nm. This tellsus thatthere are 10 base pairs (3.4nm/0.34 nm= 10) per turn of the DNA double helix. We might be
inclined to pickchoice B. This would be wrong. Remember, it is 10base pairs per turn. A base pairis composed of
two bases held together by hydrogen bonds. Thequestion specifically asks for the number of bases (notbase pairs)
per turn of the DNA double helix. The correct choice is D.

55.

D is correct Theclassical hydrogen bonding between base pairs in DNA occurs between adenine andthymine (two
hydrogen bonds) and between guanine and cytosine (three hydrogen bonds) as shown in the following drawing. This
allows us to immediately eliminate choices B and C.
Cytosine h

rrN- V/

Ribose

Thymine

Hydrogen

CH,

Bond

f^r\

""o

-NN^N'",

Ribose

H> AII

>f

H,

H^M-H

'"/,

O
N
Ribose

Ribose

Guanine

Adenine

What about choices A and D? Both answers contain bases (adenine and thymine) found in the DNA double helix.
Both answers also contain uracil, a base found in RNA.

Transcription Bubble
A-G-T-G-A^

a-g-u-cta
T-c-A-G

DNA

mRNA

Hybrid DNA-RNA
Copyright by The Berkeley Review

370

The Berkeley Review

Specializing in NCAT Preparation

Biology

Genetic Information

Section IX Answers

Consider choice A. Can uracil hydrogen bond to adenine? If it can, that base pair associated with DNA? The answer

to both of these questions is, yes. As the DNA double helix unwinds and forms a transcription bubble, two single
strands of DNA are exposed. One of those strands is used as a template to make messenger RNA (mRNA). As the
mRNA is synthesized, it is hydrogen-bonded to the DNA. During transcription, a DNA-RNA hybrid is temporarily
formed (see above).

If these is a cytosine (C) in the DNA template, then in the mRNA a guanine (G) is incorporated. If there is a thymine
(T) in the DNA template, then in the mRNA an adenine (A) is incorporated. If there is an A in the DNA template,
then a uracil (U) is called for in the mRNA.

Remember, in RNA the base U replaces the base T. A hydrogen-bonding arrangement between adenine (in DNA)
and uracil (in mRNA) is shown above. The correct choice is D.

56.

D is correct In order to transmit information accurately from one cell to the next, there must be an alphabet that

makes the communication possible. In DNA, this alphabet is composed of the bases adenine (A), guanine (G),
cytosine (C), and thymine (T). A triplet of these bases represents a codon, and a codon specifies a single amino acid.

There are 64 different codons (from43 = 64), and among these 64 codons there is redundancy. In other words, more
than one codon can code for the same amino acid. Because of this, the genetic code is referred to as being

degenerate. If there were no mechanism that would allow DNA to replicate itself, then the information contained in
these codons would not be passed to the next generation. Thus, a means of self-replication is crucial. As the
information is replicated, the fidelity must be maintained. If a mutation were to be incorporated into the next
generation of DNA, then it could have deleterious consequences. Therefore, a very low mutation rate is essential.
DNA can exist as either a single strand or a double strand. In the doubled-stranded form, it can exist in a numberof
different states (e.g., A-DNA, B-DNA, or Z-DNA). DNA is said to have variable conformations. The information

passed on to the next generation is still contained in these different forms of DNA. It is just a matter of having the
appropriate mechanism to transmit that information. DNA, as a conformationally variable molecule, would notpose
a problemfor the transmission of genetic information. The correct choice is D.
57.

C is correct. If there is an increase in Tm, it means that it takes a higher temperature to melt (denature) the DNA.
This must mean that the DNA is stabler. What would make the DNA stabler? If there were more hydrogen bonds

between the basepairs, then the stability of the double helix would increase slightly. Since there are three hydrogen
bonds between GO base pairs and two hydrogen bonds between AT base pairs, we would want an increase in the
amount of GC base pairs and/or a decrease in the amount of AT base pairs. Both of these factors lead to an
increased stability of the DNA duplex. We can eliminate choices A and D.

What about the divalent Mg2+ ion? DNA is quite negatively charged. If these negative charges were not shielded
from one another, they would tend to blow the duplex apart (i.e., make it less stable). The result is a decrease in the
Tm. However, if there is a higher concentration of Mg2+ ions in the medium, there is also a greater chance of these
magnesium ions associating with the negatively charged phosphate groups and shielding those negative charges
from one another. The DNA double helix becomes slightly stabler. We can eliminate choice B.

Histones are proteins that bind to DNA. These proteins have a large proportion of the amino acid residues arginine
(Arg) and lysine (Lys). Both of these amino acids have side chains that are positively charged at physiological pH.
These positively charged residues ionically bind DNA's negatively charged phosphates and help stabilize the
molecule. Histones can be dissociated from their interaction with DNA by using a sufficiently concentrated salt
solution that interferes with these ionic interactions. The DNA becomes slightly less stable (due to the deshielding of

the negatively charged phosphates) and easier to melt. The correct choice is C.

Copyright byThe Berkeley Review

371

The Berkeley Review


Specializing in NCAT Preparation

Biology
58.

Genetic Information

Section IX Answers

A is correct The question is based on a modest understanding of theMeselson-Stahl experiment performed in 1957.
Recall that Meselson and Stahl were able to prove that DNA replicated semiconservatively. They grew the bacteria
E. coli in a growth medium containing ,5NH4C1 for many generations. Ata particular moment in time, they then
transferred the bacteria toa growth medium containing 14NH4C1. Atthe time of transfer, they analyzed the bacterial

DNA and found it to contain the heavy isotope of nitrogen, ,5N. After one generation, they found that the DNA in

one strand of the double helix contained *5N, while the DNA in the other strand contained 14N. After two

generations, the DNA was diluted even more with 14N.


We would therefore expect to find two peaks in the graph. One peak would correspond to two duplexes of DNA,
both of which were hybrid 15N- 14N DNA. The other peak would correspond to two duplexes of DNA, both of which
were all 14N DNA. In other words, each peak would correspond to 50% of a particular type of DNA. We can
eliminate semiconservative DNA replication as a possible answer choice. Note that in semiconservative replication,
the DNA strands that contain the 15N and 14N strands become increasingly rare as the number of bacterial
generations increase.

15N 15N

14N/14N

II \

Switch medium from

15N to 14N

50%

DNA

14N 15N

15N/14N
50%

0>

1st

Generation

14N 14N

I\

Direction of Sedimentation

14N 15N

2nd

Generation

What would the replication process look like for conservative replication? In conservative replication, the original

parental strands of the DNA double helix (which are labeled with 15N) serve as templates for new daughter DNA.
However, after the first round of replication, we find that the parental strands recombine. Two duplexes result. One

is completely labeled with 15N, andtheother is completely labeled with 14N.


This type of replication continues through each succeeding generation. Note that after two generations, we have one

duplex completely labeled with 15N, and three duplexes completely labeled with 14N. The corresponding graph
shows two peaks. One peak represents that DNA which is completely labeled with 15N (25%). The other peak
represents thatDNA which is completely labeled with ,4N(75%).
14N/14N

15N 15N
Switch medium from

15N to 14N
15N 15N

75%

II

Bi

DNA X

14N 14N

1st

Generation

15N 15N
2nd

Generation

14N 14N

/
14N 14N

II \

15N/15N

ca vo

<U CM

25%
13

14N 14N

Direction of Sedimentation

II

What about dispersive and end-to-end DNA replication? After one generation, we see that both types of replication

follow thesemiconservative mode in thateach duplex is halfheavy (15N) and halflight(14N).


Copyright by The Berkeley Review

372

The Berkeley Review


Specializing in NCAT Preparation

Biology

Genetic Information

Section IX Answers

I5N 15N

15N 15N
Switch medium from

15Nto14N ./
r

II \.

DNA

dna\

SB

\\

Both are

Both are

Both are

Both are

15N/14N

15N/14N

15N/14N

1st Generation

15N/14N

Dispersive

End-to-end
conservative

However, if we were to denature these duplexes after the first generation, we would find that every single strand

would be composed of half heavy (15N) and halflight (14N) DNA. This is quite different from what we would find
if we denatured a DNA duplex after the first round of replication of either semiconservative or conservative DNA.
What do you think the graphs would look like for these two proposed types of replication? The correct choice is A.

Passage X (59 - 65)

Mitochondrial DNA (mtDNA)

59.

D is correct. Human cells do not have chloroplasts as organelles. Therefore, there should be no protein transport
from the cytosol into the chloroplast. There is protein transport from the cytosol to the mitochondria, nucleus, and
peroxisome. A peroxisome is an organelle that carries out oxidative reactions. It has no genome and is surrounded
only by a single membrane. Human beings are not sessile organisms. Therefore, we do not need to photosynthesize
in order to obtain energy. The correct choice is D.

60.

D is correct. From the passage, we learn that the mitochondrial genome is very similar to that of the bacterial
genome. Therefore, one must realize that a bacterial genome does not have its DNA packaged with histone proteins,
as is the case with human DNA. The mitochondrial genome has distinct promoters and must have both DNA

polymerase and RNA polymerase in order to replicate and transcribe its genome. Thus, the mitochondrial genome
lacks histone proteins. The correct choice is D.

61.

D is correct. This problem requires the we know which amino acid is called for by a start codon. It is neither
histidine or leucine. Therefore, choices A and B can easily be eliminated. Now the question becomes, is it
methionine or N-formylmethionine, the modified amino acid used in prokaryotes? There is our big clue. The
mitochondrial genome is very similar to the prokaryotic genome. Thus, mitochondria use N-formylmethionine
instead of methionine. The nuclear genome uses methionine. The correct choice is D.

62.

D is correct. If every nucleotide is used for coding purposes, this leaves little room for any regulatory sequences in
the mitochondrial genome. Let's consider the other possibilities. Choice A is a true statement. There will be fewer
RNA molecules coming from the mitochondrial genome. However, the reason is not because every nucleotide in the
mitochondrial genome is a coding nucleotide. The reason is simply the size of the respective genomes. Therefore,
choice A can be eliminated. Choice B is a false statement and can be eliminated. It now becomes only a matter of
discriminating between choices C and D. The correct choice is D.

63.

B is correct. The question is asking why we see a different genetic code in the mitochondria. The answer lies in the
volume of proteins that are produced by the genome. Relative to the nuclear genome, there is a small number of
proteins produced by the mitochondrial genome. Thus, a change in the genetic code is not very far-reaching. In other
words, the change is tolerable because more likely than not, a small number of proteins would be affected.
Furthermore, the changes that do occur in the protein as a result of the change in genetic code may be harmless (they
may not affect function). Because of the increased number of proteins that are coded for by the nuclear genome,
there is a higher probability that a change in the genetic code would be very far-reaching, affecting a good many
proteins. Therefore, the understanding comes from thinking about the probabilities and how that relates to the size of
the genome.

Consider the other choices: There is no evidence that genetic drift occurs only in mitochondria. Furthermore, it
would be difficult to rationalize how such a force would affect only one particular genome. The reason we do not
Copyright by The Berkeley Review

373

The Berkeley Review

Specializing in MCAT Preparation

Biology

Genetic Information

Section IX Answers

see the drift in other genomes probably has to do with the elaborate proofreading systems that maintain their fidelity.
Therefore, we can eliminate choice A. For choice C, we have no reason to believe that the tRNAs in the
mitochondria have any special correcting function. They are bound by the same physical laws that all other
molecules abide by. In other words, we have codon-to-anticodon base-pairings, and the mitochondrial tRNAs do not

change those base-pairing rules to accommodate a different genetic code. Finally, like proteins anywhere else, the
primary structure definitely affects the function by dictating the final shape of the molecule. Therefore, we can
eliminate choice D. The correct choice is B.

64.

Discorrect.The question is asking us to identify the function ofthe ten polyadenine-containing RNAs noted in the
passage. The polyadenine tail is the clue to this question. Recall that mRNA has a polyadenine tail. We can assume
this RNA molecule to be mRNA. Choice A assumes that the mRNA is translated in the cytosol, but this is

happening in the mitochondria. The mRNA that is created is translated in the mitochondria and not in the cytosol.
Eliminate choice A. Choice B indicates that the RNA is a transfer RNA. This is not the case, because tRNA does not

have a polyadenine tail. Eliminate choice B. Choice C clearly indicates that the RNA is rRNA. Again, the
polyadenine tail indicates that the RNA is mRNA, not rRNA. The mRNA is functionally responsible for coding for
ribosomal proteins. The correct choice is D.

65.

D is correct. We are told from the passage that the heavy strand of RNA is responsible for encoding many proteins,

while the light chain of RNA is over 90% nonsense. One should realize that the heavy chain of RNA is nearly
identical to the strand of DNA that gave rise to the light chain of RNA. The only difference is that the thymines in
the DNA have been replaced by uracils. Since the heavy chain of RNA contains all of these functional codons, the
strand that is nearly identical to it is called the sense strand. The piece of DNA that actually gave rise to the heavy
strand of DNA is called the antisense strand. The correct choice is D.

Viral Complementation

Passage XI (66 -72)


66.

A is correct. The question tells us that the T4 phage codes for all of the proteins necessary for its own DNA
replication. Therefore, we can approach this problem by choosing the answer that has nothing to do with DNA
replication. Transfer RNA, or tRNA, is a ribonucleic acid compound that is important during translation, rather than
DNA replication. Therefore, the phage genome need not contain instructions for making tRNA; the virus exploits
the host cell's tRNA. This problem can also be answered by eliminating choices that actually do play a role in DNA
replication. Recall that primase is an enzyme which lays down RNA primers during DNA replication. Eliminate
choice B. Ligase seals nicks introduced on the lagging strand during DNA replication; rule out choice C. And
finally, DNA polymerase is the enzyme responsible for extending newly replicated DNA. Eliminate choice D. All of
these choices affect replication of DNA. The correct choice is A.

67.

B is correct. The experiment described in the question is basically identical to the complementation experiment
detailed in the passage. In this case, however, the results are different; no plaques form, suggesting that the two
coinfecting mutants could not complement each other. Recall that complementation occurs when one mutant
provides a functional protein product that the other one lacks, and vice versa. Therefore, the net effect would be a
wild-type phenotype, as all of the protein components of the wild-type pathway would be present (see Figure 1 in
the passage). In the case of the experiment described in the passage, however, there are no wild-type plaques formed
as the result of coinfection of the rllA and XI phages. A logical conclusion to be drawn from this failure of

complementation would be that both mutants affect the same gene. In other words, neither mutant could provide the
other with a functional copy of the defective protein. As a result, a wild-type phenotype can't be achieved. With this
knowledge, we can eliminate the other answer choices. Choice A is incorrect; if the two mutations affected different
genes, they would complement each other by each providing a good copy of the protein that the other lacks. This
would result in wild-type plaque formation, which is not observed. By the same reasoning, we can eliminate choice
C; the two mutations do not complement each other, which is why they are incapable of lysing E. coli K cells.
Choice D can be eliminated, because the experiment does not support this statement in any way (even though it may
not directly contradict it). Recall, the question asks, "What can be concluded from this experiment?" Definitely not
answer choice D! The correct choice is B.

68.

B is correct. The question essentially asks us why rll mutants of phage T4 are incapable of lysing E. coli K cells.
Recall that wild-type T4 is capable of lysing these cells (resulting in plaque formation). Additionally, the rll mutants
are capable of lysing E. coli B cells. Why can't they form plaques on lawns of E. coli K? Let's approach this

problem first by eliminating unlikely answers: Statement I postulates that the mutant viral DNA sequences contain
new restriction sites that are recognized and cleaved by bacterial endonucleases (or restriction enzymes). Recall that
such enzymes recognize palindromic sequences and cleave them. Normally, this would be a perfectly plausible
Copyright by The Berkeley Review

374

The Berkeley Review


Specializing in MCAT Preparation

BlOlOgy

Genetic Information

Section IX Answers

explanation for why rll mutants can't lyse E. coli K cells. But we must remember that the question is prefaced,
"Based on information given in the passage...". From the passage we learn that when a single E. coli K cell is
coinfected with both mutants at once, the result is lysis of the bacterial cell and the formation of a plaque. Could this
occur if both mutants' DNA were chopped up by restriction enzymes? Most likely not, so we must eliminate
statement I. Statement III hypothesizes that since E. coli K cells lack specific receptors for mutant phage T4, the
phages could never bind or inject their DNA in the first place (recall the sequence of viral infection: adsorption to
the cell surface, injection of DNA, etc.). Could this be the case, considering that the successful complementation
experiment described in the passage required the injection of mutant DNA into E. coli K cells? Probably not, so
eliminate statement III. This leaves us with statement II, which states that rll mutants lack functional gene products
(i.e., enzymes) necessary for the lysis of E. coli K cells. Referring to Figure 1 in the passage, we see a pathway for
virally-induced cell lysis, which directly supports statement IPs hypothesis. Statement II is our winner. The correct
choice is B.

69.

D is correct. This question basically proposes an experiment similar to the one described in the passage. In this case,
however, we are using a diploid organism, Drosophila melanogaster (the fruit fly). The question states that mutants
A and B are each recessive, meaning that they normally express their curly wing phenotype only when they are
homozygous (i.e., A/A or BIB). However, transheterozygous flies were made by crossing the two mutant lines to
each other, resulting in some progeny which were AIB. Each individual mutant is still heterozygous, but for some
reason AIB flies have the curly wing phenotype. What is going on? It turns out that this experiment is a
complementation test. We are trying to learn whether mutant A and mutant B affect the same gene. Hypothetically, if
they affected different genes (as in Figure 1 from the passage), each would provide a functional protein which the
other lacks, resulting in completion of a wild-type pathway leading to normal wing formation. On the other hand, if
the two mutants both affected the same gene, flies that are AIB would lack a functional protein encoded by the
mutually-affected gene. Therefore, wild-type wing formation would not occur and a curly wing phenotype would be
observed. This is the case in the experiment described in the question. Therefore, mutants A and B fail to
complement each other and must therefore affect the same gene. The correct choice is D.

70.

C is correct. The experiment described in the question is basically testing whether each of the T4 mutants can revert

to the wild typeand form plaques on a lawn of E. coli K cells. Recall that normally, rllA and rllB can't form plaques
(i.e., lyse cells) on this bacterial strain. The question informs us that rllA forms one single revertant plaque,
however. The mutation that prevents rllA from lysing E. coli K cells has spontaneously reverted. For example, if the
mutation were caused by a single base-pairsubstitution, another spontaneous mutation that switches the substitution
back to the wild type would cause a reversion of the mutant phenotype (i.e., plaques could now form). This is most
likely what has happened to rllA. What about rllBI Norevertant plaques form at all. Wecan infer from this that the
type of mutation that causes the rllB defect is difficult or impossible to revert. The bottom line is thatthis question
tests your understanding of thedifferent types of mutations that can occur in DNA. Let's go overtheones mentioned
in the answer choices: A pointmutation occurs when a single base pairis altered (i.e., G is changed to A, etc.). Point
mutations revert relatively easily, because it only takes another mutation in that base pair to switch it back to the

wild type. Since rllA is capable of reverting, we can fairly assume thatr//A is caused bya point mutation. A deletion
mutation occurs when a segment of DNA is removed. Deletions hardly ever revert, mainly because it is nearly
impossible to replace a segment of DNA spontaneously (eliminate choices A and B). Since rllB doesn't revert, we

can fairly assume that it may becaused by a deletion mutation. Therefore, choice C is the correct answer. Frameshift
mutations result from deletion or insertion of base pairs, resulting in the altering of the normal reading frame of a

gene. Frameshift mutations are difficult or impossible to revert (eliminate choice D). The correct choice is C.
71.

C is correct. Answering this question requires a solid understanding of what complementation is all about.
Complementation in diploid organisms occurs when two mutants which affect different genes each provide a
functional protein which the other lacks. A dominant mutation is one which shows a mutant phenotype even when
heterozygous, or present in one copy. In other words, there is one wild-type chromosome present in organisms that
are heterozygous for a dominant mutation. This wild-type chromosome, although providing a full complement of
wild-type proteins, does not complement the dominant mutation; in other words, the mutant phenotype is still
expressed. This problem can be approached by eliminating answers that are incorrect. Forexample, choices A and
B are both recessive mutations. Recessive mutations don't express their phenotype when they are heterozygous,

because the homologous wild-type chromosome provides enough functional protein to make up for that which is
defective in the mutant chromosome. In other words, the wild-type chromosome complements the defect present in
the mutant chromosome. We can eliminate choices A and B. Choice D can likewise be ruled out; a deletion mutation

would most probably be a recessive mutation, because the wild-type chromosome could provide a normal protein
product. The correct choice is C.

Copyright by The Berkeley Review

375

The Berkeley Review


Specializing in NCAT Preparation

Biology
72.

Genetic Information

Section IX Answers

A iscorrect. The key to this question is knowing our terms. An obligate parasite is an organism that must rely on

other organisms solely to survive. Avirus can't reproduce without a host cell; therefore, it is an obligate parasite.

This BEST describes the T4 bacteriophage. Let's eliminate the other answer choices: An obligate heterotroph is an

organism that must feed on others to survive. More specifically, it must consume complex carbon molecules.
Animals are obligate heterotrophs, while plants are autotrophs; they can synthesis their own complex carbon
molecules (i.e., carbohydrates, proteins, etc.). Therefore, we can eliminate choice B, because the virus does not feed
on anything directly. It simply uses its host to reproduce. Prototrophic organisms, such as certain bacteria, can
survive on a minimal media, while auxotrophs need supplementation of certain nutrients to survive. These are terms
most often used to describe bacteria, and therefore they wouldn't apply to bacteriophage T4. Eliminate choices C
and D. The correct choice is A.

Atrial Natriuretic Peptide Experiment

Passage XII (73 - 79)


73.

B is correct. The homozygous mutant animals were hypertensive on the standard chow diet. Eliminate choice A.
The response of the heterozygous animals differed from that of the wild-type animals at the 8% dietary sodium
level. Eliminatechoice C. Blood pressure increases in the heterozygous animals were directly proportional to dietary
sodium. Choice D is incorrect. The correct choice is B.

74.

D is correct. The Lowry assay is for protein concentration, and it cannot discriminate among proteins. Choice A is
incorrect. To digest the peptides in a tissue would create a bunch of amino acids, but no real information on ANP as
an intact molecule. Choice B is incorrect. Southern blotting is for DNA fragments, not proteins. Radioimmunoassay

uses antibodies to the compound of interest to separate it from a mix of compounds. Also, it is sensitive to at very
low levels (picograms). The correct choice is D.
75.

A is correct. They do not have any detectable ANP, so the action of ANP to direct fluid out of the blood is not
available. The intravascular fluid level tends to be higher in these animals, as shown in the hematocrit. The
hematocrit measures the % of the blood that is RBCs. A lower hematocrit indicates more fluid in the blood relative

to the RBCs. Choice B is incorrect. ANP does not directly affect the RBCs, and this is not stated in the passage.
Choices C and D are incorrect. The correct choice is A.

76.

A is correct. The pro-ANP is stored in granules, cleaved by a specific protease, and then released from the cell as
ANP. A person missing the enzyme would not secrete ANP. The question is whether pro-ANP is released intact.
Compare this to what you know about glycogen. Glycogen is made of glucose polymers stored in granules, but only
the subunit, glucose, is released from the cell. It is unlikely that the larger pro-ANP precursor is released directly.
Also, there are no proteases in the bloodstream to activate the pro-ANP, since that would be disastrous to cells.
Choice B is incorrect. Choice D is incorrect. The correct choice is A.

77.

D is correct. Instead of genetically altering animals, other approaches are to somehow remove ANP from regular

wild-type animals. These are all theoretical answers. If all the ANP is bound by antibody in the blood, then the effect
is that no ANP is present. Statement I is correct. If the receptor is bound by something else and cannot interact with
ANP, then the effect is the same. Statement II is correct. If pro-ANP were bound and not processed to ANP, the
effect is the same. Statement III is correct. The correct choice is D.

78.

A is correct. The atria would be stretched in the case of too much pressure. ANP lowers fluid volume in the blood
and promotes sodium excretion (and water follows sodium). The lowering of fluid volume promotes a lower blood
volume and lower pressure. No pressures are raised, therefore choices B and C are incorrect. Sodium output in the
urine is increased by ANP ("natriuretic" means promotes sodium in the urine). Choice D is incorrect. The correct
choice is A.

79.

C is correct. Use a Punnett square to figure this out. We cross AA x Aa. AA means having 2 good copies of the
ANP gene, and Aa is the heterozygote. The offspring will be 50% AA and 50% Aa. The Aa are salt-sensitive, just
like the Aa parent. The correct choice is C.

Passage XIII (80 - 87)


80.

Griffith's Pneumococcus

D is correct. We must come up with this answer based on previous knowledge, and not something stated in the

passage. The gram stain takes advantage of differences in the cell walls of bacteria. The cell wall of a Gram-positive
bacteria contains peptidoglycan, assorted polysaccharides, and teichoic acids. The cell wall of Gram-negative
bacteria contains peptidoglycan, phospholipids, lipopolysaccharides, and assorted proteins. The cell wall of a Grampositive cell does not have an outer membrane, and the peptidoglycan layer is very thick when compared to the layer
in the Gram-negative bacterium. The correct choice is D.

Copyright by The Berkeley Review

376

The Berkeley Review

Specializing in MCAT Preparation

Biology
81.

Genetic Information

Section IX Answers

C is correct. This is not the case, and can be determined from one of Griffith's experiments. Let's look at the
injection ofa heat-killed, pathogenic bacteria. The mice lived. If the polysaccharide itself was the cause ofdeath,

then this would not have been the case. The mice would have died. The death must result from something a live
bacterium can produce; and furthermore, the polysaccharide coat must serve another function besides causing death.
The correct choice is C.
82.

Bis correct. Recall that proteins are very sensitive to temperature. When the temperature becomes too high or too
low, the protein shape is altered. That protein shape is the key to protein function, and helps a protein carry out a
cell's everyday function. If the proteins are non-functional, then nothing can continue. There can be no synthesis or
metabolic processes. If no processes can be carried out (even the replication of nucleic acids), the cell will not be
able to produce energy and it will die. The correct choice is B.

83.

D is correct. This question is very straightforward. Avery's conclusion was that the transforming principle was
DNA. Recall that deoxyribonuclease destroys DNA. If Avery's conclusion held true, the addition of

deoxyribonuclease should eliminate transforming activity. We are looking for a statement that does not support
Avery's conclusion. The correct choice is D.
84.

B is correct. This question is straightforward. We are told from the passage that the bacterium is a facultative
anaerobe. This means that the bacteria can function either in the presence or absence of oxygen. An obligate
anaerobe cannot function in the presence of oxygen, while aerobes can function only in the presence of oxygen. The
correct choice is B.

85.

B is correct. Remember that ribosomes are considered to be organelles. They are just not membrane-bound
organelles. The bacterium certainly requires the presence of ribosomes to carry out the translation of mRNA to make
protein. The bacterium does contain organelles. The correct choice is B.

86.

D is correct. We are looking to use two elements that will enable us to discriminate between proteins and nucleic
acids. This question is based on the experiments of Hershey and Chase. The backbone of nucleic acids contains
phosphate groups. Nucleic acids do not contain sulfur. Proteins contain sulfur, but do not contain phosphorous.
Therefore, phosphorus and sulfur are the two elements we can use that will enable us to discriminate between
nucleic acids and proteins. The correct choice D.

87.

D is correct. This question comes in two parts. The first requires one to think whether a cell taken up by another cell
is pinocytosis or phagocytosis. The answer is phagocytosis. Recall that solid particles endocytosed by a cell is
referred to as phagocytosis, while endocytosis of dissolved particles is termed pinocytosis. The bacterium can be
classified as a solid particle, enabling one to eliminate two of the answers. Next, is phagocytosis carried out by P-

cells or macrophage? P-cells produce antibodies, while macrophage are considered the body's professional
phagocytes. The correct choice is D.

Passage XIV (88 - 94)

Drosophila eyeless Gene

88.

B is correct. Since we are assuming that eyeless is on the X chromosome (X-linked), we can infer that the
genotypes used in the cross are: ey/+ females x +/Y males. Fruit flies have the same sex chromosome setup as
humans (two X chromosomes for females and an X and Y for males). The real trick to this question is remembering
from the passage that flies heterozygous for ey have small eyes, while those homozygous for it have no eyes. After
that, it's just a matter for a Punnett square. The correct choice is B.

89.

B is correct. As you recall, a promoter is an untranscribed sequence of DNA upstream (towards the 5' end) of the
DNA sequence that is actually transcribed (and that codes for the gene's protein product). The relative "strength" of
a promoter (i.e., the degree to which it promotes transcription) is dependent on the promoter's sequence. In
Experiment 1, GAL4 has a very weak promoter. The is the reason that GAL4 is not normally transcribed or
expressed. The wing-specific enhancer can "enhance" transcription of GAL4 only in wing tissue, overcoming the
weakness of the promoter. In this question, this system has failed and GAL4 is transcribed everywhere, thereby
activating the UAS-controlIed eyeless gene everywhere. This is why the in situ hybridization experiment showed

thatey mRNA was present throughout the embryo when we should expect it to be present only in certain cells. A
constitutive (meaning "always active") mutation in the GAL4 weak promoter could have made it "stronger," thereby
increasing transcription. Answer choice A is wrong, because a mutation in the UAS site could only prevent GAL4
from binding, thereby reducing ey expression. Answer choice C is wrong, because the normal genomic copy of ey is
expressed only in cells that will become eyes (from the passage). Answer choice D is wrong, because an amber
mutation (a stop codon) would cause an abnormally short ey protein but would not affect where it is transcribed.
The correct choice is B.

Copyright by The Berkeley Review

377

The Berkeley Review


Specializing in MCAT Preparation

Biology
90

Genetic Information

Section IX Answers

C is correct. Statement I is an unsupported statement, because from the passage we know that while the ey gene

may be necessary, it is not sufficient for normal eye development. We know this because in heterozygotes, there is

one good copy of the gene, but the phenotype is still abnormal (small eyes). Moreover, no evidence in the passage

supports the claim that ey is sufficient for normal eye development (there are in fact hundreds of other genes that are
necessary to make an eye). We can infer only that ey is adevelopmental "master switch," possibly acting to turn on

other genes needed for eye formation. From the passage, we see that ey is a transcription factor, which serves to
back this claim. Transcription factors bind to DNA and promote transcription ofcertain genes. This means statement
II can be supported by the passage. Statement III says that eyes arose independently in insects and mammals. This

means they evolved separately from different ancestors and have nothing in common evolutionarily. The results of
Experiment 2as well as the homology between Drosophila eyeless and mouse small eyes shows this to be false.

This evidence suggests that eyes evolved from acommon ancestor. The correct choice is C.
91.

Dis correct. The mutant eyeless allele is a recessive hypomorph, meaning that its function is reduced, or weaker. A

point mutation in a nonconserved region of the sequence would probably not have much effect on protein function,

because nonconserved sequences vary throughout evolution without altering protein function to any great degree. A

two base-pair deletion near the beginning ofthe sequence would cause a frameshift mutation that would completely
wipe out protein function, and this can't be the case. An inversion would cause the same effect, so that leaves choice

D. A mutation in a sequence ofeyeless that has been highly conserved throughout evolution would cause a reduction
in the function ofthe protein. Tliese sequences are conserved, because they are crucial to the proper functioning of
the protein. The correct choice is D.

92.

B is correct. Ectopic means "not in the normal place." Since Experiment 2 showed that the mouse small eyes gene
made normal fruit fly eyes in Drosophila, a finding that the reciprocal experiment failed to make normal mice eyes
would be inconsistent. Such a finding is shown in choice B. Answer choices A and D both confirm that eyeless is
evolutionarily conserved, which agrees with the results of Experiment 2. Answer choice C confirms that eyeless is
highly homologous tosmall eyes, a case that supports the results of Experiment 2.The correct choice is B.

93.

A is correct. In order for a population to be in Hardy-Weinberg equilibrium, answer choices B, C, and D must not
be true, while choice A must be true. This is a memorization question; but if you remember that at equilibrium there

is no net change in the frequency of alleles in the gene pool, then you could come up with the right answer. The
correct choice is A.

94.

C is correct. This type of question was given on a previous MCAT. You must know how to use the Hardy-

Weinberg equation, which states that p2 + 2pq + q2 = 1. In this equation, p is the frequency of the wild-type allele,
while q is the frequency of the recessive allele in the population. The questions asks for the number of flies that will
have small eyes, and from the passage, we know that this means those heterozygous for eyeless. The 2pq term
represents heterozygotes. Thus, 2pq = 2(0.9)(0.1) = 0.18. This is the frequency of heterozygotes in the population.
When we multiply this by the population of 1000, we come up with 180flies. The correct choice is C.

Meiotic Nondisjunction

Passage XV (95- 100)


95.

C is correct. The karyotype gives a picture of the chromosomes, which means that the sex of the fetus is
immediately discernible. Choice A is thus not the best answer. The incidence of extra or missing chromosomes is
also immediately visible, so choices B and D are poor selections as well. Developmental defects may or may not be

due to genetic errors. Some have environmental causes. Choice C is therefore false, making it the bestanswer. The
correct choice is C.

96.

C is correct. The second decade of one's life spans the years from 10-19. We finish the first decade when we turned
ten. The chance of bearing a Down syndrome child is lowest for women in the years from 15-19, or the later portion
of the second decade, so statement I is correct. Reading from the graph, we see that the incidence in both 13-year-

olds and 35-year-olds is about 0.9 per 1000 births, so statement II is correct, too. Menarche is the beginning of
menstruation, while menopause is the conclusion of menstruation. The incidence of Down births is higher in women
near menopause (age 40-50+) than in women close to menarche (9-15+). Statement III is therefore incorrect. The
correct choice is C.

97.

A is correct. Down syndrome is also called trisomy 21. Tri means "three," somy refers to body (in this case, the
bodies of the chromosomes), and 21 refers to chromosome 21. Choices B and D, which refer to chromosome 18, are
both incorrect. Also, choice C (monosomy 21) is incorrect. The correct choice is A.

Copyright by The Berkeley Review

378

The Berkeley Review


Specializing in MCAT Preparation

BlOlOgy
98.

Genetic Information

Section ix Answers

B is correct The presence of the Ychromosome means the person is a genotypic male, so choice A is incorrect.
This question is about the genotype, not the phenotype. Choices C and D are thus incorrect, since they refer to
phenotype. The correct choice is B.

99.

Dis correct Table 1provides us with the answer: Monosomy X is the highest, at 20/100. There is no trisomy X
listed in the table, so choice Cis incorrect. Both trisomy 21 and trisomy 18 occur less frequently than monosomy X,
so choices A and B are incorrect. The correct choice is D.

100. B is correct RFLPS can be used for this kind of work because restriction enzymes cleave different DNA strands

into fragments ofdifferent lengths, based on the location ofspecific restriction sites. The child has three copies of

chromosome 21. One clearly came from the father and two from the mother. The mother is the source of the
nondisjunction in this instance. Choices A, C, and D are incorrect. The correct choice is B.

Copyright by The Berkeley Review

379

The Berkeley Review


Specializing in NCAT Preparation

Biology
A. RNA Processing

Section X
Expression of
Genetic
Information

B.

RNA Polymerase and Promoters


RNA Synthesis and Modifications
Protein Synthesis

4.
5.

The Lactose Operon


The Tryptophan Operon

Mutations

1.

Mutations and Proofreading

2.

The Ames Test

2nd Position

1st
Position

3rd

Phe
Phe

Ser
Ser

Tyr
Tyr

Cys
Cys

Leu

Ser

STOP

STOP

Leu

Ser

STOP

Trp

u
c
A
G

Leu

Pro

His

f^i

Leu

Pro

His

Leu

Pro

Gin

V^

Leu

Pro

Gin

Arg
Arg
Arg
Arg

U
C
A
G

(5' End)

1.
2.
3.

jE\
fJ~^4
^

It

(3' End)

He

Thr

Asn

Ser

lie

Thr

Asn

Ser

He

Thr
Thr

Lys
Lys

Arg
Arg

U
C
A
G

Val

Ala
Ala

Asp
Asp

Val

Ala

Glu

Val

Ala

Glu

Gly
Gly
Gly
Gly

U
C
A
G

Met

Val

C. Genetic Engineering
1.

Genetic Engineering, Sequencing,


and Cloning

Practice Passages and Answers

Berkeley

JJr-e-v-ke-w
Specializing in MCAT Preparation

Expression of Genetic Information


Top 10 Section Goals

oj)

Understand the differences between BTCA and DNA polymerase.

RNA polymerase isused totranscribe DNA into RNA language. DNA polymerase replicates DNA
beforecellular division. Understand these functions and how they apply to the cell.
Be familiar with the concept of a transcription promoter.

RNA polymerase does not begin transcription just anyplace on the DNA helix. There are specific
addresses that tell the polymerase where tobind. Understand thebasics ofthese functions.
Know the differences between eukaryotic and prokaryotic BTCA processing.

Eukaryotic RNA must undergo extensive processine in the nucleus before it can be used in the
cytosol for protein synthesis. Prokaryotic RNA isused immediately andis notprocessed.
Be able to read the genetic code.

DNA codons onthecoding strand are the same asthemRNA codons, andDNA codons onthetemplate
strandarethesameasthe anticodons ontRNA, except forthereplacement ofTs withUs).

Have a feel for transcription and translation in eukaryotes and prokaryotes.

In prokaryotes, transcription and translation are tightly coupled. In eukaryotes, transcription and
translation are separatedboth in space and in time. Understandthis important distinction.
Be familiar with the actual process of transcription.
You should have a firm grasp of tRNA activation and subsequent binding of the activated tRNAs
to die P site and A site on the ribosome. Understand translocation and protein synthesis.
Understand the lactose operon and how regulation is achieved.

Thelactose operonties in all of theseaspects of transcription and translation discussed so far. It is


an excellent example of how a bacterialcellregulatesits need for a particular metabolite.

>

Understand the basic tools used in genetic engineering.

Probably the most important genetic engineering tools are the restriction endonucleases. Do not
memorize their recognition sequences. Instead, knowthe basics ofhow theyfunction.
Be able to read autoradiographs from polyacrylamide gels.

Understand the process ofgelelectrophoresis andwhyit is thatbandsarereadfrom the bottom of


thegel to thetop ofthegel. Be familiar with theSanger dideoxy sequencing method.
Be familiar with the process of cloning foreign DNA into plasmids.
Thisis the coreof genetic engineering. Understand how to remove a segmentof foreign DNA,open
up a plasmid, insertthat foreign DNA intothe plasmid, and thenexpress the gene orinterest.

Biology

Expression of Genetic Information

RNA Polymerase & Promoters

RNA Processing
RNA Polymerase & Promoters
RNA Polymerase

The enzyme RNA polymerase transcribes DNA into RNA. Let's compare the
differences between replication and DNA-directed DNA polymerase with
transcription and DNA-directed RNA polymerase.
1.

Both polymerases proceed in the 5' -> 3' direction.

2.

DNA polymerase uses dNTP's (releasing PPj) while RNA polymerase


uses NTP's (also releasing PPj).

Both polymerases require a DNA bubble foroperation.


With DNA polymerase we find that DNA replication is semiconservative

3.
4.

while in the case of RNA polymerase the two DNA strands will
eventually rewind, thus conserving the process.

DNA polymerase requires a primer with a free 3'-OH group to initiate


chain elongation. RNA polymerase does not require a primer.
The DNA directed DNA polymerase is almost always involved in
synthesizing DNA bidirectionally. Both DNA strands are being copied.
In the case of DNA directed RNA polymerase only one strand of the

5.

6.

DNA is being copied.

One way to demonstrate thatRNA polymerase does notneed a primer is to use a


labeled substrate such as ATP labeled at the pandyphosphates (Figure 10-1).
.

II*

II*

II

0 P

Adenine

5'

0 P0 P0 CH2

1
_0
Y

.i .<5

P a

y^j

p{H

110

OH

Adenosine triphosphate
(A ribonuclcoside triphosphate)

Figure 10-1

Are any of the P or yphosphates incorporated into the nucleic acid? If we try this
using dATP and DNA polymerase, the answer would be no. Why? Because we
are starting with a primer and when we add dATP to the free 3'-OH group, the p

and y phosphates of dATP are lost in the reaction mechanism (as pyro
phosphate). However, with RNA polymerase we do get some p and y labeled
phosphates in the RNA polymer. This is because the first nucleotide (ATP)
incorporated into the growing RNA polymer has at its 5' end a labeled P and y
phosphate.
The time course of incorporation of the p-y labels and also of a labels in a
growing RNA polymer can be measured. It takes about 1 or 2 minutes to make

an average mRNA molecule in a test-tube using RNA polymerase and the DNA
from a phage such as T2 or T4. If we measure the incorporation of the P-y label,
Copyright by The Berkeley Review

383

The Berkeley Review

Specializing in MCAT Preparation

Biology

Expression ofGenetic Information

RNA Polymerase &Promoters

we will find that within a few seconds that label is in the RNA polymer and then

no more is incorporated. However, if we measure the incorporation of the a


label, we will find that the time course is more even up to about two minutes
(5-y-labeledNTPs

when RNA synthesis stops. This can be seen in Figure 10-2. This graph tells us
that the p-ygoes in first while the a label goes in second.
Promoters

In E. colithere is only one RNA polymerase and this RNA polymerase has to
make transfer RNA (tRNA), ribosomal RNA (rRNA), and many different types

Figure

Time (minutes)
10-2

of messenger RNA (mRNA). It prefers to start transcription at a particular site on


the DNA duplex. What tells an RNA polymerase where to start transcription? It
turns out that there are very specific starting sites for transcription which are
different in both prokaryotic and eukaryotic organisms. Consider the double
stranded DNA in Figure 10-3.
Upstream
Start Site

Downstream

10

-35

5'

Coding Strand

3'

Duplex

TATAAT

TTGACA

DNA

Template Strand

5'

5' P-P-Pl

3'
mRNA

Figure 10-3

Note the 51 and 3' ends of the duplex. Let's suppose that this small segment of
DNA is about a thousand or so base pairs and that we wish to copy the lower
strand. The direction of RNA synthesis will be in the 51 - 31 direction as
previously mentioned. The very first base pair of the transcription process is
referred to as the +1 base pair. Immediately to the left (upstream) of +1 we have
the -1 base pair, -2 base pair, -3 base pair, etc. Immediately to the right
(downstream) of +1 we have the +2 base pair, +3 base pair, etc. There is no base
designated as "0".

The signals that tell the RNA polymerase where to begin transcription are
upstream from the +1 site. In a prokaryotic organism these areas are usually
centered around the -10 region (TATAAT) and the -35 region (TTGACA). The

-10 region is referred to as the Pribnow box while the -35 region is simply the -35
region. As we will see, there is some flexibility in these numbers from gene to
gene or organism to organism. Therefore, we can speak of these regions as being
consensus sequences. As the RNA polymerase moves along the DNA duplex it
is looking for the proper signals that will tell it where to promote transcription.
Hence, these regions are called promoter sites.

What are the consensus sequences for eukaryotic organisms? In the case of
eukaryotic organisms the promoter areas are centered around a -25 region
(TATA box) and a -75 region (CAAT box), upstream from the +1 site. This is
shown in Figure 10-4. The -25 region is referred to as the Hogness box The
CAAT box may or may not sometimes be present. It might even be moved
further upstream.

Copyright by The Berkeley Review

384

The Berkeley Review

Specializing in MCAT Preparation

Biology

Expression ofGenetic Information

Upstream
Start Site

-25

-75

5'-

Downstream

+i
Coding Strand

GGNCAATCT

RNA Polymerase fif Promoters

TATA

3f

Duplex
DNA

Template Strand

3-

5'
5* P-P-P
RNA

Figure 10-4

There are also signals in the DNA that tell the RNA polymerase when to stop
transcription. These occur in specific areas downstream from the +1 site.

Elongation of the RNA chain occurs bya nucleophilic attack ofthe 3'-OH group
at the end of the growing RNA chain with the alpha phosphate of the next
incoming ribonucleotide triphosphate. A phosphodiester bond is formed. This
reaction is similar to that of DNA synthesis.
DNA Template

3'-GGG TCGGGCG GATTACT CGCCCGA AAAAAAACTTGTTT-5,


5'-CCC IAGCCCGC CTAATGA GCGGGCT TTTTTTTGAACAAA-3,

GC-Rich region

|^ AT-Rich region -J
A

S'-CCCAGCCCGCCUAAUGAGCGGGCUUUUUUUU-OH-S'
RNA Transcript

G
C
A
C-G

G-C

Figure 10-5

C-G

TheRNApolymerase will proceed down the DNA templateuntil it runs into the
terminator sequences. These stop signals are composed of a GC-rich region
followed by an AT-rich region on the DNA template (Figure10-5).
In E. coli the terminator sequence for the end of protein synthesis is a basedpaired hairpin sequence on the newly synthesized RNAstrand (Figure 10-6).

C-G
C-G
G-C
S'-CCCA UUUUUUUU-OH-3

Termination

Hairpin

Figure 10-6

Thehairpin structure is formed by the complementary base-pairing of a region of


the RNA sequence that is rich in the bases G and C. Following this hairpin
structure is a sequence of four or more U residues (referred to as the poly-U tail).

Once this hairpin pairing occurs in the RNA molecule the RNA polymerase
pauses (stalls). The polyribonucleotide uracil and polydeoxyribonucleotide
adenine that are still annealed to each other (the RNA-DNA hybrid) are rather
unstable. The result is that the RNA chain will dissociate from RNA polymerase
and the DNA duplex. This process is referred to as rho-independent termination.

The second way in which termination can be accomplished involves the rho (p)
protein and is referred to as rho-dependent termination. The rho complex is a
hexameric protein consisting of 46 kd subunits and has an ATPase activity that
allows it to specifically bind newly synthesized single-stranded RNA and pull
Copyright by The Berkeley Review

385

The Berkeley Review


Specializing in NCAT Preparation

Biology

Expression of Genetic Information

RNA Polymerase fit Promoters

itself towards the replication bubble where it will dislodge RNA polymerase
from the DNA template. The end result is a fully transcribed transcript of RNA, a
free rho protein, and a free RNA polymerase enzyme. In higher organisms it is
not entirely clear what the termination signals are.
The overall process which we have just described happens at a variety of places
on the DNA duplex and the result is that a lot of specific RNA molecules are
synthesized. The three phases of RNA synthesis are called initiation, elongation,
and termination.

Copyright by TheBerkeley Review

386

The Berkeley Review


Specializing in NCAT Preparation

BlOlOgy

Expression ofGenetic Information

rna synthesis fir Nodifications

RNA Synthesis & Modifications \,_v, MM


RNA, like DNA, is synthesized in the 5' -> 3' direction. One immediate difference

between DNA synthesis andRNA synthesis is that the enzyme RNA polymerase
incorporates ribonucleoside triphosphates into the growing RNA chain. In DNA
synthesis deoxyribonucleotide triphosphates were used. Another difference is

that RNA polymerase does not need a primer in order to start transcription of
the RNA polymer.

When the RNA polymer is synthesized during transcription, only one of the
DNA template strands is utilized. This is shown in Figure 10-7. As RNA
polymerase moves alongthe DNA duplexit unwindsabouta 17basepair section
ofthe DNAduplex and reads the templatestrand in order to synthesize the RNA
transcript. Local unwinding of the DNA duplex occurs ahead of the polymerase
while local rewinding takes place to the rear of the polymerase. Note that
towards the 3' end of the RNA transcript we have an RNA-DNA hybrid.

Local

Local

Rewinding

Unwinding

4
Coding Strand
Template Strand

RNA-DNA

RNA Polymerase

Hybrid

Figure 10-7

Types of RNA

Recall that there are three major classes of RNA. They are messenger RNA
(mRNA), transfer RNA (tRNA), and ribosomal RNA (rRNA). In the case of
prokaryotic bacterial cells there is one type of RNA polymerase that is able to
transcribe all three of these types of RNA. However, in the case of eukaryotic
cells there are three separate types of RNA polymerases. RNA polymerase I will
transcribe rRNA, RNA polymerase II will transcribe mRNA, and RNA
polymerase III will transcribe tRNA.
Transcription Promoter Sites

Within the DNA molecule are regions called promoter sites that bind RNA
polymerase. These promoter sites determine where transcription is to begin. In
bacteria such as E. coli there are two sequences on the DNA template which are
important to the RNA polymerase, the Pribnow box and the - 35 region. The
Pribnow box is located upstream (towards the 5' end) by 10 nucleotides (i.e., at
-10) from the region of the DNA template where the first nucleotide is
transcribed (denoted as +1). The -35 region is located upstream by 35 nucleotides
(i.e., at -35). RNA polymerase will move along the DNA template and
transcription of one strand will take place until a signal to stop (a terminator
region) is reached. This can be seen in Figure 10-8.

Copyright by The Berkeley Review

387

The Berkeley Review


Specializing in NCAT Preparation

BiolOQy

Expression ofGenetic Information

RNA Synthesis fit Modifications

Promoter

1
-35

-10

TTGACA

TATAAT

-35 Region

Pribnow

fl

Genu

DNA

Box
DNA

Template

Start of
RNA

End of
RNA

Figure 10-8

Note that the -10 region and the -35 regions of the promoter are not palindromes.
They are not like the CATC box that we will discuss when we look at the
mismatch repair enzyme system or even like the EcoRI sites that we will discuss
when we look at restriction enzymes. If the promoter region were palindromic,
RNA polymerase would not know what to do.
Promoter Strengths Vary

The promoters for eukaryotic rRNA genes are simply not recognized by RNA
polymerase II or RNA polymerase III. Within a class of genes the promoter
strengths can also vary. Consider the gene for a glycolytic enzyme. Since energy
metabolism is a major activity of the cell, these enzymes need to be transcribed at
a high rate. Thus, the enzymes for glycolysis are always quite prominent in cells.
For example, there might be in excess of 100 transcripts per cell for any of the
glycolytic enzymes. If you were to assume that these transcripts were degraded
at the same rate as their synthesis, then there must be a lot of transcription of
glycolytic genes. What this means is that glycolytic genes have strong promoters.
In contrast, consider the coenzymes that we have been discussing. Coenzymes
are present in cells in catalytic amounts. In other words, the enzymes for the
coenzyme biosynthetic pathways are present in very small amounts. You might
only need one copy of an enzyme molecule per cell for each step in the pathway.
Thus, the number of transcripts in the cell might be on the order of one. What
this means is that the genes in the coenzyme biosynthetic pathway have weak
promoters.

This is telling us that you will not find the same promoter sequence in front of all
genes. The -10 region (TATAAT)and the-35 region (TTGACA) of the promoter
sequence are simply two common motifs which are consensus sequences based
on the analyses of literally hundreds of promoter regions.
Viral Promoters

The promoter region in front of the gene that codes for the RNA polymerase of
the T7 viral bacteriophage is very strong. Once this phage's DNA is injected into
the host cell, its strong promoter region attracts the RNA polymerase of the host
cell and forces it to transcribe the viral RNA polymerase. Once the viral RNA

polymerase is synthesized it will recognizeonly its own genes and not the genes
of the bacterial host cell.

Copyright by The Berkeley Review

388

The Berkeley Review


Specializing in NCAT Preparation

BlOlOgy

Expression of Genetic Information

RNA Synthesis fir Modifications

RNA Polymerase Holoenzyme

E. coli RNA polymerase is composed of 4subunits, 0:2, P, (3', and a. Together


these four subunits are called the RNA polymerase holoenzyme. The sigma (a)
subunit is responsible for locating the promoter site that initiates transcription.
Once RNA synthesis begins, the sigma subunit will dissociate from the RNA
polymerase holoenzyme. RNA polymerase, which now has three subunits

(0:2PP'), is called the core enzyme. The core enzyme can still initiate transcription

even though it has lost its sigma subunit, but it does so poorly and anywhere.
Because the core enzyme has lost one of its subunits (sigma), it isan apoenzyme.

The holoenzyme and the apoenzyme are represented in Figure 10-9. Note that
we are giving directionality to this enzyme.

D>

Core^^

a2pp'

a2PPa

Holoenzyme
(A complete enzyme)

Apoenzyme
(An incomplete enzyme)

Figure 10-9

Sigma factors are quite important in bacterial transcription. For example, they
have the ability to (1) lower the affinity of the RNA polymerase for general

regions in theDNA duplex by a factor ofabout 104, and (2) they raise thebinding
constant (affinity) for one class of promoters. Thus, sigma would be an example
of a regulatory subunit. It regulates the specificity of the RNA polymerase
enzyme.

How does RNA polymerase find its promoter region? The RNA polymerase will
bind to the DNA duplex and then slide along it looking for the -35 region and
the -10 region. In this search, transient hydrogen bonds are formed with the
hydrogen acceptor and donor groups of base pairs exposed in the grooves of the
DNA duplex. This one dimensional search for the promoter site is much faster
than a three dimensional search in which the RNA polymerase would have to
repeatedly bind and dissociate from the DNA duplex (see Figure 10-10).
Promoter

1
"N

>
^A

-35

.in

TTGACA

TATAAT

-35 Region

Pribnow

Polymerase

+1

GE

Box

RNA Polymerase slides along the DNA duplex


looking for the promoter site. It does not repeatedly
bind and then dissociate in its quest for this site.

Figure 10-10

Copyright by The Berkeley Review

389

The Berkeley Review


Specializing in NCAT Preparation

Biology

Expression ofGenetic Information

RNA Synthesis fir Modifications

Transcription

RNA polymerase will synthesize RNA from a DNA duplex in three stages,
initiation, elongation, and termination, as indicated in the sequence of diagrams
shown in Figure 10-11.
Initiation

GENE
35

-10

Promoter

Region

^-*Q
Elongation

RNA

Rho

Termination

GENE
-35

-10

*
RNA

Figure 10-11

When the RNA polymerase binds to the promoter region of the DNA duplex an
initiation complex is formed. As soon as the initiation complex is formed, the
sigma subunit dissociates from the polymerase leaving the core enzyme bound
Copyright by The Berkeley Review

390

The Berkeley Review


Specializing in NCAT Preparation

BlOlOgy

Expression ofGenetic Information

rwa Synthesis &Modifications

to the DNA duplex atthe promoter site. With sigma missing, the core enzyme is
able to bind more strongly to the DNA template. As the DNA template is
unwound and read in the 3'- 5* direction by the RNA polymerase core enzyme,

newly synthesized RNA is made in the 5' -^ 3' direction. This region of activity is'

referred to as a transcription bubble. Nascent RNA is synthesized until a


termination signal is reached.

Accuracy of Replication and Transcription

We can consider the accuracy ofreplication andtranscription by looking at Table


10-1. Recall that during DNA replication there were twoprocess thatensured the
accuracy of the final product. These were (1) editing by the DNA polymerase
enzymes and (2) the repair enzymes. As a result, the probability of an error
occurring during DNA replication is quite low. Roughly 1010 bases will be
replicated before an error is made thatis not caught byeither theediting aspect
of DNA polymerase or the repair enzymes. Therefore, the mutation rate is very
low.

In contrast, the RNA polymerase do not have a polymerase editing function and
they do not have an enzyme repair system. As a result, the probabilityof an error

occurring is about one in every104 or 105 transcribed bases.


Coding
Process

Polymerase
Editing

Repair
Enzymes

Replication

Transcription

Probability
of Error

10-io

lO"4, 10"5

Table 10-1

Why, then, does DNA replication begin with an RNA primer? The editing system
of the DNA polymerase system will not work when replication is just starting
from the first base. The DNA polymerase enzymes will only work properly if
there is a complementary double stranded structure to start working on. The
RNA primer that begins the Okazaki fragments has a high probability of error
and they will eventually need to be removed. Since there is now a double
stranded structure to work with, the DNA polymerase enzyme can use its
editing function to remove the RNA primer (and fill in the gap with DNA
nucleotides).
Inhibitors of RNA Polymerase

RNA polymerase can be inhibited. For example, the antibiotic actinomycin D


binds specifically to double stranded DNA and prevents RNA polymerase from
using it as a template for transcription. Actinomycin D has a phenoxazone ring
(similar to the anthracene ring) which can act as an intercalator and slip between
neighboring G-C base pairs in the DNA duplex.
Promoters and Enhancers

We mentioned that in the prokaryote E. coli, the promoter sites can be found at
about the -10 and -35 regions. In eukaryotic DNA the promoter regions are
centered around -25 (called the TATA box or Hogness box) and around -75
(called the CAAT box and ranges between -40 and -110).There are also enhancer
sequences which also help to stimulate transcription. These enhancer sequences
are usually quite far away from the promoter region (e.g., several kilobases
Copyright by The Berkeley Review

391

The Berkeley Review


Specializing in NCAT Preparation

BiolOgy

Expression ofGenetic Information

RNA Synthesis fir Modifications

away) and can be either upstream or downstream from the promoter region.
They can even be within the transcribed gene itself.
The process of whether or not a gene gets transcribed not only depends on the
correct promoter region but also on the interaction with specific enhancers. This
can be seen in Figure 10-12. For example, consider the gene that codes for the
synthesis of hemoglobin. Hemoglobin will only be expressed in one cell type,
even though you have a great variety of different cell types in your body with the
same hemoglobin gene in every one of them. It might be that cell or tissue
specific proteins act as passwords which are capable of binding specific enhancer
regions. This interaction, in turn, will allow RNA polymerase to sit down on a
given promoter region and begin transcription of a specific gene.
Promoter
Several

Region

kilobases
away

-75

-25

Enhancer

GENE

Region
CAAT

TATA

DNA loops so distant


sites are juxtaposed

GENE -

DNA

Multiprotein
Complex
Figure 10-12

The use of enhancers can also be seen in the hormonal action of glucocorticoids.
Glucocorticoids can bind to a soluble receptor protein that is specific for a certain
enhancer region. Once this hormone-receptor complex binds to glucocorticoid
enhancer region, transcription is stimulated.

For example, you will not be able to transcribe the genes that are important for
mammary function unless you have the hormone as well. Males and females
both have mammary glands. However, females have certain hormones which can

bind specific proteins. Interaction of this hormone-protein complex with specific


enhancer regions allows for the transcription of mammary genes specific to
females. Since males do not have enough of this hormone, transcription of their
mammary genes does not take place.
Modification of the Transcripts
A variety of modifications to RNA transcripts can occur. There can be (1)
trimming and splicing (in which the ends of the RNA transcript can be excised or
Copyright by The Berkeley Review

392

The Berkeley Review

Specializing in NCAT Preparation

Biology

Expression of Genetic Information

RNA Synthesis & Modifications

certain intervening sequences removed). There can be (2) addition of bases to the

primary transcript (such as the addition of the sequence CCA to the 3' end of
tRNA molecules).

There can also be (3) modification ofbases, a feature that isparticularly evident
with tRNA molecules. (Processes (2) and (3) are quite common in prokaryotic
cells.) For example, once you have a tRNA thathas already been synthesized, its
bases can be modified (post-transcriptionally). Once such case would a
modification from uridine to pseudouridine as shownin Figure 10-13. Note the
difference in attachment of the glycosidicbond.
HO

Themost extreme modifications are those seen in eukaryotic transcripts. Recall


that within the nucleus of a eukaryotic cell we find DNA replication,

OH

Uridine

transcription, and processing taking place before the transcripts are sent out into
the cytosol. Once the transcript is in the cytosol, translation occurs at the
ribosomes.

Consider the processing of the primary RNA transcript. This primary RNA
transcript can be modified by (1) capping at its 51 end, (2) addition of a poly
adenine (poly-A) tail at its 3' end, and (3) splicing which involves the excision of
introns.

5' Capping the of the Nascent RNA


If the first residue at the 51 end of our nascent RNA were not modified, then you
would expect to find three phosphate groups attached to the ribose ring. These
"bare" 5' ends can easily be degraded by phosphatases and nucleases lurking

HO

OH

Pseudouridine

Figure 10-13

within the cell. However, the 5' end is modified in a reaction with GTP.

First, the gamma phosphate is hydrolyzed at the 5' end of the RNA. Next, the
beta phosphate at the 5' end of the RNA attacks the alpha phosphate of GTP,
releasing pyrophosphate in the process (which is subsequently hydrolyzed to
two molecules of orthophosphate). An unusual triphosphoanhydride linkage is
formed between the 51 end of the RNA and the 51 end of the guanylate residue as
shown in Figure 10-14.

II

11
H2N

CH3

II)
^ N

N 0 H2C~

HO

II

II

II

op 0 P0 p

0 CH,

OH

J
Y
Cap

0
'

Base

OCHj

0-P=0

R^0

Base

OCH3

Figure 10-14

Copyright by The Berkeley Review

393

The Berkeley Review


Specializing in NCAT Preparation

BlOlOgy

Expression of Genetic Information

RNA Synthesis &Modifications

S-adenosylmethionine (SAM) is the methyl donor that next adds a methyl group
to the N-7 position of the purine ring system of guanine. Also, the 2'-hydroxyl
groups of the adjacent ribose rings can be methylated by SAM as well. Capping
protects the 5f end of the polymer from degradation and thereby enhances the
stability of the RNA molecule.
3' Polyadenylation of the Nascent RNA

5'

3'

As eukaryotic RNA polymerase nears the end of its transcription process, a


sequence of bases is recognized in the primary transcript by a specific
endonuclease. The primary transcript is cleaved and about 250 adenine residues
(from ATP) are added to the free 3' end of the RNA polymer to form the
polyadenylated tail as shown in Figure 10-15. The poly-A tail helps to protect
the 3' end of the RNApolymerfrom nucleases and phosphatases.

CAP HHH A-A-A-(A)n-OH


Nascent
RNA

Figure 10-15

Splicing (an overview)

Within many genes in eukaryotic DNA are sequences which are untranslated.
These untranslated sequences, called intervening sequences (introns), lie
between regions of the gene that are translated or are expressed (exons). The
introns are spliced out of the primaryRNA in a process calledsplicing and the
exons are joined together. The exons are then used as the coding domains for
proteins. The 5' non-coding region of the RNA is important for ribosome
recognition. This is shown in Figure 10-16.
Primary
RNA
Exon

Exon
Intron

Intron

Non-coding
region

region

Processing Events:
Splicing; Capping;
Polyadenylation
mRNA

5'-Cap

Exon Exon Exon

Poly-A 3'

Figure 10-16

It is still a mystery as to why the exons are separated by the introns. One
possibility might deal with recombination. A related possibility is that it might
allow enhancers to come and go. This would allow for genetic variation within
and among species with regards to the signals that say just how turned on a gene
should be. Recall that we mentioned that enhancer sequences can often lie within
a genein particular, the enhancers reside within these introns.
snRNPs can form Spliceosomes

Within the cytosol and nucleus of many eukaryotic cells are RNA molecules that
are associated with specific proteins. When this situation exists in the nucleus
these complexes are referred to as small nuclear ribonucleoprotein particles
(snRNPs), or more fondly referred to as "snurps". snRNPs are involved with the
Copyright by The Berkeley Review

394

The Berkeley Review

Specializing in NCAT Preparation

Biology

Expression of Genetic Information

RNA Synthesis fi? Modifications

base pairing reactions at the spliced junctions of introns and exons. snRNPs can

recognize the G-U and A-G. If this processing is not done accurately, then a
frame shift error might result.
Self-Splicing RNA

The details of how snRNPs react are stillnot clear. However, it is quite possible
that the actual chemistry is done by the RNAs and not the associated proteins.
This came into consideration because of the discovery in ciliated protozoans that
the self-splicing reaction involves rRNA (not mRNA) and not the associated

proteins. In this case the reaction can be catalyzed by the attackof a 3'-hydroxyl
of a guanosine cofactor residue on the phosphodiester bridge between exon A
and the intron. This is shown in Figure 10-17.

Exon A ends up with a free 3'-hydroxyl that can next attack the phosphodiester
bridge between exon B and the intron. The result is the splicing of exon A and
exon B and the excision of the intron with the guanosine residue at its 5' end and
a free 3'-hydroxyl. Further reactions (not shown) will ultimately release the
guanosine residue, meaning that guanosine has a catalytic function. This type of
intron self-splicing is referred to as being in Group I.

Exon A -5'

Exon B -3'

Exon B -3'

5'- Exon A Exon B -3'

Exon B 3'

I Intron

Attack

HO- Exon A -5'

Figure 10-17

This was the first clear example in which RNA could be involved in its own
chemistry without the help of any enzymes. Up until a few years ago it was one

of the holy laws of biochemistry mat only proteins and their coenzymes could be
catalysts. The clearest case in which RNA can catalyze reactions by itself are from
the protozoan rRNAs.

Copyright by The Berkeley Review

395

The Berkeley Review


Specializing in NCAT Preparation

Biology

Expression of Genetic Information

RNA Synthesis fi? Modifications

So, under the category of self-splicing there can be Group I introns or Group II

introns. Group I introns are spliced out by guanosine at the 3'-hydroxyl. Group
II introns are spliced out by adenosine at the 3'-hydroxyl.

There can be mutations in DNA which will affect splicing. For example, you
could mutate the 5' splice site so that the G-U sequence is changed to some other
sequence that is not recognized by the snRNP. Similarly, you could mutate the 3'
splice site to something that is not recognized by the snRNP. Also, the region
that contains the adenosine moiety involved in the 2', 5'-phosphodiester bond
could also be mutated as well.

New splice signals couldalsobe created. If a new A-G splice sequence came too
soon in the intron, the exon at the 5' end could attack the wrong site. The
resulting functionalmessage would have a piece of the 3' end of the intron which
would not be a coding sequence. This would lead to a defective protein.
Mutations like these can turn up in the hemoglobin genes, and one of the more
common types ofmutations arethose thatcause thalassemia (defective synthesis
of a hemoglobin chain(s)).

Copyright by The Berkeley Review

396

The Berkeley Review


Specializing in NCAT Preparation

Biology

Expression of Genetic Information

Protein Synthesis

Protein Synthesis
Let's take a look at protein synthesis. Protein synthesis occurs on the ribosomes
in three steps: initiation, elongation, and termination. However, before we can
begin to consider these processes, we first need to consider how it is that amino
acids are brought to the site of protein synthesis.
Amino acids are brought to the ribosome by an adapter molecule called transfer
RNA (tRNA). This molecule contains a template recognition site for the mRNA

Rest of the tRNA


molecule

Adenine

codon called the anticodon and an amino acid attachment site. An amino acid

(like Phe) would be attached to the 3'-OH function at the 3' end of the tRNA

molecule. This can be seen in Figure 10-18.


The base sequence at the 3' end of the tRNAs is 5'-CCA-3'. The 5' end of the
tRNA has a free phosphate group. Hydrogen bonds between U-A and G-C
ribonucleotides help to hold the tRNA together at the stems. About half of the
base pairs in tRNA can be found in these stems. Certain groups of bases within
the tRNA molecule are not base-paired. Some of these groups are found in the
loops of the molecule. For example, there is the dihydrouracil (DHU) loop, the
ribothymine-pseudouracil-cytosine (TyC) loop, and the anticodon loop. These
features define the secondary structure of tRNA.

O'
O 3'

OH

I
0=C

I
HCR
I

NH,

An amino
acid

Figure 10-18

The Genetic Code: How was the genetic code deciphered? In 1961 Marshall
Nirenberg took a solution of E. coli, lysed the cells, and removed the cellmembrane and cell-wall fragments. He was left with an extract that contained
various other constituents of the cell, including DNA, mRNA, tRNA, enzymes,
and ribosomes. When amino acids and an energy source, such as ATP or GTP,
was added to this extract and then incubated at 37 C for a brief period of time,
protein synthesis occurred on the ribosomes. Nirenberg was able to stop this
protein synthesis by degrading the DNA template used for mRNA synthesis. It
turned out that mRNA had a rather short half life (because it is degraded by
specific cellular enzymes). Protein synthesis could be started once again if mRNA
was added back to the extract.

Nirenberg had access to an enzymatic protein called ribonucleotide phosphorylase. This enzyme was an undirected enzyme that allowed for the synthesis of
synthetic chains of ribonucleic acids as long as the right ribonucleotides were
added to the medium. Recall that DNA polymerase and RNA polymerase were
both directed enzymes. Ribonucleotide phosphorylase uses ribonucleoside
diphosphates (NDPs) and catalyzes the synthesis of polyribonucleotides (RNA)
with the subsequent release or orthophosphate (Pi). This can be seen in Figure

nNDP + (RNA)n

(RNA)n+1 + nP|

Figure 10-19

10-19.

Note that in the case of RNA polymerase, pyrophosphate (PPi) is formed which
can then be hydrolyzed by inorganic pyrophosphatase in order to help drive the
reaction (of RNA synthesis) to completion. What this means is that in vivo the
reaction involving ribonucleoside phosphorylase cannot be driven to the right
(completion) by the subsequent hydrolysis of pyrophosphate. Why? Because
there is no pyrophosphate produced. Why? Because in this case we are using
ribonucleoside diphosphates (NDPs) instead of ribonucleoside triphosphates
(NTPs). In other words, the equilibrium for the reaction shown in Figure 10-19 is
to the left (degradation of RNA). In fact, ribonucleotide phosphorylase actually
takes partially degraded mRNA and phosphorylates it to form ribonucleoside
diphosphate substrates. So, how does ribonucleotide phosphorylase catalyze the
Copyright by The Berkeley Review

397

The Berkeley Review


Specializing in NCAT Preparation

Biology

Expression of Genetic Information

Protein Synthesis

synthesis of mRNA? It depends on the concentration of the ribonucleotides


added to the reaction mixture. If high concentrations of a given ribonucleotide
are incubated with ribonucleotide phosphorylase, then the reaction shown in
Figure 10-19 will be driven to the right.

2nd Position

1st

Position

(5' End)

TT

1
\J

g^i
I
V^
A

r\
g~H

il.
VT

Phe

Ser
Ser

Tyr
Tyr

Cys
Cys

3rd

(3' End)

Leu

Ser

STOP

STOP

Leu

Ser

STOP

Trp

u
c
A
G

Leu

Pro

His

Leu

Pro

His

U
c
A
G
U
c
A
G

Phe

Leu

Pro

Gin

Leu

Pro

Gin

Arg
Arg
Arg
Arg

He

Thr

Asn

Ser

He

Thr

Asn

Ser

He

Thr
Thr

Lys
Lys

Arg
Arg

Val

Ala
Ala

Asp
Asp

Val

Ala

Glu

Val

Ala

Glu

Gly
Gly
Gly
Gly

Met

Val

U
c
A

G
Table 10-2

The Genetic Code.

When Nirenberg used high concentrations of the ribonucleotide UDP in the

presence of ribonucleotide phosphorylase, the polyribonucleotide UUUUUU


(i.e., poly-U) was synthesized. It turns out that ribosomes are not designed to
take substrates like poly-U. In order to get protein synthesis, artificial conditions

had to be employed. Forexample, 0.01 M Mg2e ions wereused,which is at least


ten-fold more magnesium than normaUy found in cells. Once poly-U was
translated on the ribosome, the protein that was synthesized contained all Phe
(i.e., poly-Phe). Nirenberg was able to postulate that the nucleotide triplet UUU
coded for the amino acid Phe.

How does the ribosome read the mRNA in order to make proteins? We know
that proteins are composed of amino acids. In 1961 various features of the base
sequence in DNA and the amino acid sequence in proteins led to the
establishment of a genetic code. There are four different bases in DNA. If the
code were a simple single-base code, then only 4 different types of amino acids
could be specified. And since we know that we have at least 20 different amino
acid this does not seem too likely. If we had a two-base code, then 16 different
types of amino acids could be specified. Again, this is still less than the 20
different amino acids that we know we have. However, if we had a three-base

Copyright by The Berkeley Review

398

The Berkeley Review


Specializing in NCAT Preparation

Biology

Expression of Genetic Information

Protein Synthesis

code, then we could specify 64 amino acids. Indeed, the 20 different amino acids
that we have been exposed to are coded by a three-base code on the DNA

molecule. Thesegroups of three bases are referred to as codons (see Table 10-2)
Which way are the proteins synthesized? Is it from the amino terminal to the
carboxyl terminal or is it from the carboxyl terminal to the amino terminal? For
proteins it was postulated that they were synthesized from the amino end to the
carboxyl end.
In order to determine this, hemoglobin synthesis from the red blood cells (RBCs)

of rabbits was analyzed. There are immature RBCs that make almostnothing but
hemoglobin. If you inject a rabbit with hydrazine, it will destroy the rabbit's
RBCs. To compensate for this the rabbit will begin to synthesize new RBCs. If
you had a start site for hemoglobin at one end of the protein or the other, then
you would make a longer and longer chain until you reached the end. It should
be possible to label chains that are almost complete just by adding radioactively
labeled amino acids to your reaction mixture for just a few seconds. The ends of
the polypeptide chains that was synthesized last would be labeled.
The rabbit RBCs were lysed and amino acids with a label were added to the
solution for a few seconds. The mature hemoglobin was isolated in order to find
out where the label was. The label was found to be at the carboxyl end of the
polypeptide. Therefore, the carboxyl end was synthesized last while the amino

end was synthesized first. You must also realize that as the protein is being
synthesized from the N-terminus to the C-terminus, that region near the Nterminus can begin to fold up on itself to give tertiary structure even before the
rest of the protein has been synthesized.

5,-AAAAAAAACAAA-3,

J IRibonuclease
5,-AAAAAAAAC-3,
+ 5'-AAA-3'

Translation

N-Lys-Lys-Asn-C

[1 Carboxypeptidase
N-Lys-Lys-C + N-Asn-C
Figure 10-20

In what direction is the mRNA read? Polynucleotide phosphorylase was


incubated with a lot of ADP and a little bit of CDP in order to make a mixed

polymer. This polymer mostly consisted of A residues. OccasionaUy a C residue


would be incorporated. The polymer was then treated with pancreatic
ribonuclease which cleaves on the 3' side of C residues. What is observed are

polymers with long runs of As ending in a C residue at the 3' end. This polymer
can then be used as a translation system. What you will get is a polymer with a
lot of Lys in it and a very little bit of Asn. When this polymer is treated with
carboxypeptidase, Asn will be released from the poly Lys polymer. This is shown
in Figure 10-20.
Carboxypeptidase is a digestive enzyme that hydrolyzes the carboxyl-terminal
peptide bond in polypeptide chains, particularly if that residue has a bulky
aliphatic side chain or an aromatic ring. Thus, Asn must be at the carboxyl end of
the polymer (the end that is synthesized last). This means that the coding triplet
for Asn is AAC. In other words, the coding triplet for Asn is at the 3' end of the
polymer and also at the carboxyl end of the protein. The direction of translation
is from 5' 3'. When RNA polymerase is transcribing DNA in order to
synthesize RNA, it makes RNA in the same direction in which you get
translation.

About half of the codons were determined by a triplet binding assay. Suppose
that ribosomes were able to bind to trinucleotides. The example that we wiU use
is 5'-AUU-3'. Once this binds to the ribosomes, then tRNA can bind. In this case

the tRNA for He would be the only tRNA that would bind to the ribosomes.
Copyright by The Berkeley Review

399

The Berkeley Review


Specializing in NCAT Preparation

Biology

Expression of Genetic Information

Protein Synthesis

Thus, AUU was said to be the coding unit for lie. In this way various other
codons could be worked out

The remainder of the genetic code was filled out by Gobind Khorana. Khorana
was able to synthesize polyribonucleotides with a repeating sequence that was
well defined from one of the two strands of a double stranded DNA molecule

that had the repeating deoxyribonucleotide residues 5'-TAC-3' and 5'-GTA-3'. In


order to be selective about which of the two DNA strands that were to be

transcribed, only three of the four complementary ribonucleoside triphosphates


were added to the reaction mixture. This technique allowed one strand of the
duplex DNA to be transcribed but not the other strand. When Khorana added

ATP, UTP, and CTP to the mixture containing the double stranded DNA, he got
the repeating RNA sequence 5'-UACUACUACUACUACUAC-3'. When he

added GTP, UTP, and ATP to the mixture containing the double stranded DNA,
he got the repeating RNA sequence 5-CATCATCATCATCATCAT-3'.

Once the defined RNA polymers were synthesized they were used as templates
for proteinsynthesis on the ribosome. Forexample, if the mRNA polymerwere
poly -UAC, therewouldbe threeways to read it. Wecould either read it as polyUAC, poly-ACU, or poly-CUA. This will give us three different polymers. If it is
read as poly-UAC, then we will get poly-Tyr. If it is read as poly-ACU, then we
will get poly-Thr. If it is read as poly-CUA, then we will get poly-Leu. This can
be seen in Figure 10-21a.

5'-U-A-C-U-A-C-U-A-C-U-A-C-3'

Tyr

Tyr

Tyr

Tyr

S'-U-A-C-U-A-C-U-A-C-U-A-C-S'
Thr

Thr

Thr

5'-U-A-C-U-A-C-U-A-C-U-A-C-3'
Leu

Leu

(a)

Leu

5'-G-U-A-G-U-A-G-U-A-G-U-A-3'
Val

Val

Val

Val

5'-G-U-A-G-U-A-G-U-A-G-U-A-3'

Stop

Stop

Stop

S'-G-U-A-G-U-A-G-U-A-G-U-A-^
Ser

Ser

Ser

(b)

Figure 10-21

However, if we read the complement of the poly-UAC strand, which is polyGUA, then we will get only two polymers. Why? We could read poly-GUA as
either poly-GUA, poly-UAG, or poly-AGU. If we read it as poly-GUA, then we
will get poly-Val. If we read it as poly-AGU, we will get poly-Ser. However, if
we read it as poly-UAG, then nothing ever appears. It turns out that 5'-UAG'-3' is
a stop codon. The other termination codons are 5'-UAA-3' and 5'-UGA-3'. This
can be seen in Figure 10-21b.

Not only are there stop signals but there are also start signals. The start signals
could not be determined by using Nirenberg's system because his translation
system would start anywhere (due to the high magnesium levels). In order to
Copyright by The Berkeley Review

400

The Berkeley Review


Specializing in NCAT Preparation

Biology

Expression of Genetic Information

Protein Synthesis

find the start signals, a natural mRNA molecule was needed (not a synthetic
mRNA).

The RNA from RNA bacteriophages can be recognized from regular DNA
because RNA is denser than DNA (due to the presence of uracil and the extra
oxygen atom at the 2' position of the ribose ring). The RNA from these phages,
when first isolated, was thought to have about 3,000 nucleotides (or about 3
genes). A start site on the RNA from one of these RNAphages was determined in
the following manner.
Sticking out of this length of phage RNA is a small protrusion that is the start
signal for the coat protein of the phage. Ribosomes can be denatured into the 30S
and 50S subunits in low magnesium concentrations. The 30S subunits of the
ribosome will bind to the start signal for the coat protein. If ribonuclease is added
to the solution, then the rest of the RNA will be degraded. The only RNA that is
left will be that stretch of RNA "protected" by the 30S subunit of the ribosome. In
other words, the start signal is not denatured. Within this protected piece of RNA
is the sequence 5'-AUG-3' followed by a few more coding residues. This triplet
codes for the amino acid Met. If you now take the phage coat protein and
analyze the amino terminus, you will find that it starts with Met, followed by the
corresponding coding residues.
The start codons for the maturation protein (only one copy at the vertex of the
phage) and the replicase protein (copies RNA to make more RNA) were
discovered in a similar fashion. The RNA was sheared in a Waring Blendor to
obtain smaller fragments. The same experiment was repeated by binding the 30S
subunit of the ribosome to all the fragments and then looking for protected
segments. Both of these genes also start with 5'-AUG-3'.
It turns out that the most common start codon is 5'-AUG-3' which codes for Met.

There are some rare cases in which the start codon is 5'-GUG-3\ This triplet
codes for Val.

Eukaryotic cells also start their proteins by using the codon 5'-AUG-3'. Instead of
fMet as the first amino acid we find that it is simply Metbut with a slight
variation. Eukaryotic messages get processed before they even reach the
ribosome. For example, if the first residue at the 5' end of our nascent RNA were
not modified, then you would expect to find three phosphate groups attached to

the ribose ring. These "bare" 5' ends can easily be degraded by phosphatases and
nucleases lurking within the cell.

CH3
I
GHHHB A-A-A-(A)n-OH
5' Nascent

3*

RNA

Figure 10-22

However, the 5' end is modified in a reaction with GTP. First, the gamma

phosphate is hydrolyzed at the 5' end of the RNA. Next, the beta phosphate at
the 5' end of the RNA attacks the alpha phosphate of GTP, releasing
pyrophosphate in the process (which is subsequently hydrolyzed to two
molecules of orthophosphate). An unusual triphosphoanhydride linkage is
formed between the 5' end of the RNA and the 5' end of the guanylate residue.

A methyl donor (S-adenosylmethionine (SAM)) adds a methyl group to the N-7


position of the purine ring system of guanine. This process is called capping.
Also, the 2'-hydroxyl groups of the adjacent ribose rings can be methylated by
SAM as well. Remember, capping not only protects the 5* end of the polymer
from degradation but it also enhances the stability of the RNA molecule as well.
This can be shown in Figure 10-22.
Copyright by The Berkeley Review

401

The Berkeley Review


Specializing in NCAT Preparation

Biology

Expression of Genetic Information

Protein Synthesis

As eukaryotic RNA polymerase nears the end of its transcription process, a


sequence of bases is recognized in the primary transcript by a specific
endonuclease. The primary transcript is cleaved and about 250 adenine residues

(from ATP) are added to the free 3' end of the RNA polymer to form the
polyadenylatedtail as shown in Figure 10-22. The poly-A tail helps to protect
the 3' end of the RNA polymerfromnucleases and phosphatases.
Within many genes in eukaryotic DNA are sequences which are untranslated.
These untranslated sequences, called intervening sequences (introns), lie
between regions of the gene that are translated or are expressed (exons). The
introns are spliced out of the primary RNA in a process called splicing and the
exons are joined together. The exons are then used as the coding domains for
proteins. The 5' non-coding region of the RNA is important for ribosome
recognition. This is shown in Figure 10-23.

Primary
RNA
Exon

Exon|^^^V^3'

Exon

Intron

Intron

t
Non-coding
region

Processing Events:
Splicing; Capping;
Polyadenylation

mRNA

5'-Cap

Exon Exon Exon

Poly-A 3'

Figure 10-23

It is still a mystery as to why the exons are separated by the introns. One
possibility might deal with recombination. A related possibility is that it might
allow enhancers to come and go. This would allow for genetic variation within
and amongspecies with regards to the signals that say just how turned on a gene
should be.

Amino Acid Activation

The activation of amino acids is similar to the process in which fatty acids are
activated and to the process in which DNA ligase joins to segments of DNA.

Peptide bond formation between two amino acids is thermodynamically


unfavorable. This unfavorablesituation is overcomeby activation of the carboxyl
group of incoming amino acids. In the first step of the reaction ATP reacts with
an amino acid to form an activated mixed anhydride linkage in the form of
aminoacyl-AMP. In the second step the aminoacyl group of aminoacyl-AMP is
transferred to a tRNA molecule to form aminoacyl-tRNA This transfer can take
Copyright by The Berkeley Review

402

The Berkeley Review


Specializing in NCAT Preparation

Biology

Expression of Genetic Information

Protein Synthesis

place at either the 2'-hydroxyl or the 3'-hydroxyl of the ribose moietyat the 3' end
of the tRNA molecule. The amino group, however,can migratebetween either of
these two hydroxyl functions. Aminoacyl-tRNA is the activated intermediate in

protein synthesis and the enzyme that catalyzes this reaction is an activating
enzyme (i.e., a specific aminoacyl-tRNA synthetase). This is shown in Figure 1024.

Stepl

II

HO

H,NC-C-0

I II
||
H3N C C - 0 P O - Ribose

ATP

pp.

I
Adenine

Aminoacyl-AMP
Transfer RNA
I
O

Step 2

0p=o
I

0-CH2

Adenine

Aminoacyl-AMP + tRNA

0 PO- Ribose

WHN

Adenine

II
1 ~
O

OH

AMP

I
0=C

I
H-C-R

NH3

Aminoacyl-tRNA

Figure 10-24

You should be aware that there are many different tRNA molecules within the
cell, and they might average about 80 bases long. This means that there has to be
great specificity as to which amino acid is placed on which tRNA molecule.
There is a minimum of 20 different activating enzymes, one for each amino acid.
Each activating enzyme is quite selective. For example, it will only place valine
on the tRNA specifying valine and not on the tRNA specifying isoleucine. Valine
and isoleucine are rather similar to one another and differ only by the fact that
isoleucine carries an extra methylene (-CH2-) group. How does the activating
enzyme distinguish between the two amino acids?
Aminoacyl-tRNA Synthetase Specificity
We need to consider the accuracy of the aminoacyl-tRNA synthetase enzyme
(i.e., the activating enzyme). Roughly 1 mistake is made for every 3000 catalytic
events. One way to assure this accuracy is to have a proofreading system
(analogous to that in DNA replication). In many tRNA synthetases there are two
catalytic sitesa hydrolytic site and an acylation site. The hydrolytic site is

concerned with the aminoacyl-AMP molecule while the acylation site is


concerned with the aminoacyl-tRNA.
Suppose we want to attach isoleucine to a tRNA molecule but in the initial step
we mistakenly activate a valine molecule (i.e., valine-A MP). Since valine is
smaller than isoleucine it can easily enter the hydrolytic site where it is
Copyright by The Berkeley Review

403

The Berkeley Review


Specializing in NCAT Preparation

Biology

Expression of Genetic Information

Protein Synthesis

subsequently hydrolyzed. Since isoleucine is larger than valine it cannot fit into
the hydrolytic site and therefore is not hydrolyzed. However, isoleucine can fit
into the acylation site where it activated and attached to an isoleucine specific
tRNA.

During replication we mentioned that the error rate is about 1/1010. During
transcription the error rate is about 1/104 or 1/105. Why is transcription more
error prone compared to replication? Most transcripts are rather short (less than

104 or 105 bases long). If you have a probability of error of 1/104 or 1/105, then
most of the molecules that are made thatareless than 104 or 105 units long will
not have any errors. In other words, you do not need to have an error rate of

1/1010 ifyou are making a molecule that isonly 1/104 or 1/105 nucleotides long.
The same holds true for protein synthesis. Error rates of 1/104 are fine because

you are notmaking proteins that are 104 amino acids long.
tRNA Design

All tRNA molecules are capable of forming a cloverleaf structure that is about 80
ribonucleotides long. An amino acid like phenylalanine would be attached to the
terminal adenosine residue at the 3f end of the tRNA. The 51 end of the tRNA has

a free phosphate group. Hydrogen bonds between U-A and G-C ribonucleotides
help to hold the tRNA togetherat the stems. About half of the base pairs in tRNA

can be found in the stems. One will occasionally see G-U base pairing (we will
come back to this point later). Certain groups of bases within the tRNA molecule
,, mRNA

Codon UU U
Anticodon

are not base paired. Someof these groups are found in the loops of the molecule.
There is the dihydrouracil (DHU) loop, the ribothymine-pseudouracil-cytosine
(TyC) loop, and the anticodon loop. The features that we have just mention
define the secondary structure of tRNA.

The tRNA molecule is not flat and planar but rather has tertiary structure in
which the molecule is L-shaped and contains two segments of double helix, each
containing about 10 base pairs. Base pairing in the nonhelical regions of the

tRNA

molecule are rather unusual in that there can be base-base, base-backbone, or


backbone-backbone interactions. The backbone interactions involve the 2'-

hydroxyl function of the ribose ring.

Figure 10-25

Codon-Anticodon Interactions

A triplet of three bases in an mRNA polymer represent a codon that calls for a
particular amino acid. The amino acid that is called for should be attached to a
tRNA molecule that has an anticodon that is complementary to the codon of the
Codon

Codon

UUU^UUC
Specifies
Phe

AAG

AAG

Anticodon

Anticodon

Figure 10-26

mRNA. This can be seen in Figure 10-25. Note that the codon-anticodon
relationship is anti-parallel.

The rules for base pairing of the codon-anticodon junction are like those that
operate in DNA. In this case A will pair with U and G will pair with C. However,
there is an additional possibility in which G can pair with U. Recall that there are
only 20 different amino acids but there are 64 triplets (3 stop codons and 61
informational codons). On the average there are 3 codons per amino acid. There
are not 61 tRNA molecules, which means that a given tRNA can recognize more
than one codon. For example, S'-UUU-S* and 5'-UUC-3* are codons that specify
phenylalanine. Both of these codons are recognized by the same tRNA which has
the anticodon sequence 5'-GAA-3\ Codon-anticodon pairing can be seen in
Figure 10-26. How can one tRNA base pair with two different codons? The basis
for this phenomenon has to do with wobble in the base pairing.

Copyright by The Berkeley Review

404

The Berkeley Review


Specializing in NCAT Preparation

Biology

Expression of Genetic Information

Protein Synthesis

These two codons, S'-UUU-S* and 5'-UUC-3', are both recognized by the same
tRNA as shown in Figure 10-27. The tRNA that is recognized by the activating

enzyme is tRNAPne, and in particular it is recognized by a specific activating


enzyme for Phe called phenylalanyl-tRNA synthetase. When this specific

activating enzyme transfers the aminoacyl group of Phe to tRNAPne, we can

designate that activated aminoacyl-tRNA as Phe-tRNAPne. Note that a charged


tRNA would have the amino acid attached to it while the uncharged tRNAdoes
not have the amino acid attached to it.

3'-Phe

3'-Phe

51
tRNA

u
5'

u-u-u

3'

mRNA

(a)

5'

U-U-C

(b)

Figure 10-27

Note that in Figure 10-27a we have a non-standard base pair between U and G
while in Figure 10-27b we have a standard base pair between C and G. We can
represent these pairings as shown in Figure 10-28. We can line the structures up
such that positions 2, 3, and 4 in the pyrimidines and positions 2,1, and 6 in the
purines are facing each other.
In the case of the non-standard base pairing between U and G, the pyrimidine
base is displaced with respect to the purine base. This is shown in Figure 10-28a.
If this base pairing were to occur in a DNA double helix, it would distort the
backbone. However, in tRNA this does not matter too much because we are not

making that large of a double helical structure. We simply have three bases of
mRNA pairing with three bases of tRNA. In other words, there is a great deal
more flexibility in this type of interaction in tRNA (compared to DNA). Standard
hydrogen bonding between the C and G base pairs will occur as shown in Figure
10-28b.

H N H

X
N

N0

Backbone

"XvO

HN

Cytosine

Backbone

Guanine

Backbone

Guanine

(b)

Figure 10-28

Copyright by The Berkeley Review

405

The Berkeley Review


Specializing in NCAT Preparation

Biology

Expression of Genetic Information

Protein Synthesis

Wobble Rules in Base Pairing


Wobble is simply steric freedom in the pairing of the third base of the codon. The
wobble rules apply at the first base of the anticodon and the third base of the
codon. These rules are:

1. U can pair with A (standard) or G (non-standard).


2. G can pair with C (standard) or U (non-standard).
3. Inosine (I) can pair with either A, U, or C.
We could have a four-fold degenerate codon such as 5'-GGG-3', GGA, GGU,
GGC, where they would all specify glycine. Three of these codons could be read
by one tRNA if the tRNA had hypoxanthine at the first position of its anticodon.
The other codon (GGG) would need to be read by a different tRNA. Recall that
hypoxanthine is the purine base of inosinate (IMP), an intermediate in nucleotide
biosynthesis.
Ribosome Structure

About two-thirds of the weight of a ribosome is rRNA. Ribosomes in E. coli are

large molecules, about 200 A in diameter and having a mass of about 2700 kd.
The ribosome is described in terms of a sedimentation coefficient called the

Svedberg unit (S). A ribosome (70S) can be dissociated into a small subunit (30S)
and a large subunit (50S). This is shown in Figure 10-29. Note that it is a non
linear relationship between size and S-value. The dissociation of the 70S

ribosome is controlled by the concentration of Mg2

Low Mg2 causes

dissociation of the ribosome.

70S
Ribosome

30S
subunit

50S
subunit

23S rRNA

5S rRNA

+ 34 proteins

16SrRNA

+ 21 proteins

Figure 10-29

The 50S and the 30S subunits can be further dissociated in the presence of urea.
Dissociation of the 50S subunit yields about 34 proteins and 23S rRNA and 5S

Copyright by The Berkeley Review

406

The Berkeley Review


Specializing in NCAT Preparation

Biology

Expression of Genetic Information

Protein Synthesis

rRNA. Dissociation of the 30S subunit yields about 21 proteins and 16S rRNA.

rRNA has a well defined folding pattern containing many short duplex regions.
Ingredients for Protein Synthesis

In order to makea protein we would need the following items:

1. 20 aminoacyl-tRNA's, each attached to the correct tRNA (more may be


needed).

2. mRNA that contains the codons that specify the proteinsequence that is
desired.

3.

Ribosomes.

4. Additional proteins called initiation factors that are not part of the
ribosome. There are initiation, elongation, and termination factors.
5. GTP as an energy source.
Initiation of Protein Synthesis
Consider a fairly late stage in initiation when the 50S and the 30S subunits have

joined together to form the 70S ribosome. The mRNA is appropriately placed
withits 5' end in the 30S subunit. ThemRNA sequence which has recently been
synthesized from our DNA duplex has signals which are important. For
example, the mRNA molecule has polarity-it has a 5' end as well as a 3' end. The
initiating codon in mRNA is the base triplet 5'-AUG-3\ About 10 nucleotides

upstream from the initiating codon is a purine-rich sequence of bases (e.g.,


AGGAGGU) called the Shine-Dalgarno sequence. The Shine-Dalgarno
sequence is responsible for binding the mRNA to the 16S subunit~an

architectural constituent of the 30S ribosomal subunit. Binding of the ShineDalgarno sequence to the 3f end of the 16S ribosomal subunit forms the initiation
complex as shown in Figure 10-30.

3'end of 16S

ribosomal RNA

mRNA

I
5'-GAUUCCU AGGAGGU UUGACCU AUG CGAGCU UUU AGU-3'

fMet-Arg-Ala-Phe-Ser-

Shine-Dalgarno
Sequence

Polypeptide
Sequence

Figure 10-30

The codon in mRNA that starts protein synthesis is 5'-AUG-3\ This codon is
calling for a special modified amino acid called formylmethionine (fMet). How is
this modified amino acid brought to the ribosome for protein synthesis? It is
brought to the ribosome by an adapter molecule called tRNA. We can abbreviate
this tRNA adapter molecule with fMet on it as fMet-tRNAf. Remember, a tRNA
molecule contains a template recognition site (for the mRNA codon) called the
anticodon and an amino acid attachment site.

Copyright by The Berkeley Review

407

The Berkeley Review


Specializing in NCAT Preparation

Biology

Expression of Genetic Information

Protein Synthesis

After the 30S subunit has formed a complex with the initiation factors that we
mentioned as part of the ingredients in protein synthesis, GTP will bind to a
particular initiation factor called IF2. This allows the mRNA and the fMettRNAf initiator signal to join the 30S subunit as well. The subsequent release of
these initiation factors allows the 50S complex to attach to the 30S complex,
thus forming the 70S initiation complex. The 70S initiation complex is now
primed for protein synthesis. This can be seen in Figure 10-31.
It turns out that there are two tRNAs for methionine. One form of tRNA, called

mRNA

tRNAf, has the initiation role we just mentioned, while the other tRNA, we'll
call tRNAm, will simply bind to AUG codons elsewhere in the mRNA.

30S Initiation Complex

Let's consider the process by which methionine is attached to its respective


tRNA. We would expect that an activating enzyme will attach methionine to
tRNAf and form Met-tRNAf. We would also expect the same activating
enzyme to attach methionine to tRNAm.

SOS Subunit

50S

Subunit

In the next step a formyl group attached to THF will react with Met-tRNAf to
form formyl-methionine-tRNAf (abbreviated as fMet-tRNAf) and THF
without the formyl group. The formyl group has been attached to the
methionine residue as shown in Figure 10-32. Because the amino group on
methionine is formylated, it is called N-formylmethionine-tRNA.
mRNA

Elongation
70S Initiation Complex

Figure 10-31

Once the 70S initiation complex is primed for synthesis, what happens?
Suppose the next codon in our mRNA is 5'-CGA-3' as shown in Figure 10-30
and Figure 10-31.Looking at our genetic code we find that this codon codes for
the amino acid arginine (Arg).

HO

II

II

HCNCC- O tRNA,

CH,

Arginine is attached to the 3' end a specific tRNA by a specific aminoacyltRNA synthetase and is then carried by this arginine-specific aminoacyl-tRNA
to the empty A-site on the 70S complex. Next to the A-site is the P-site. Note
that the P-site is already occupied by fMet-tRNAf. This is shown in Figure 10-33.

I
CH,

I
S
I

CH3

Formylmethioninyl-tRNA f
(fMet-tRNAf)

Figure 10-32

Figure 10-33

Copyright by The Berkeley Review

408

The Berkeley Review


Specializing in NCAT Preparation

Biology

Expression of Genetic Information

Protein Synthesis

Once we have amino acids in both the P-site and A-site, the enzyme peptidyl
transferase will catalyze the formation ofa peptide bond. The amino nitrogen of
Arg attacks (nucleophilic) the carbonyl carbon of the ester linkage in fMettRNAf. The result is cleavage of that ester bond and the formation of a peptide
bond between fMet and Arg. In other words, the fMet moiety of the P-site will
be transferred to the amino group of the Arg residue of the A-site. The formation
of the dipeptidyl-tRNA at the A-site is a favorable reaction.
The empty tRNAf will leave the P-site and the mRNA will move a distance of

three nucleotides thus bringing the dipeptidyl-tRNA (5'-fMet-Arg-3') to the Psite. Thisis called translocation and can be seen in Figure 10-34. We are now left

with the A-site empty and ready for thenext incoming amino acid (specified by
themRNA codon) during the secondround of elongation.

5' -

AUG

mRNA

70S Initiation Complex


Figure 10-34

Energetics of Protein Synthesis

Consider the ATP equivalence involved in protein synthesis. Recallthat each of


the amino acids has to be activated by binding to tRNA. This processinvolves 2
ATP equivalences (one for the hydrolysis to get pyrophosphate and one for the
hydrolysis of pyrophosphate itself). The binding of an aminoacyl-tRNA to its
respectivecodon costs an ATP equivalence. In the translocationstep we find that
no ATP equivalences are required for the first amino acid because it is already at
the P-site. However, each additional amino acid will bind at the A-site first and

then translocate over to the P-site. This process will require 1 ATP equivalence
each. If we add up the sum in terms of ATP equivalences, we will find that 3 are
required for the first amino acid and then 4 for each subsequent amino acid. This
is summarized in Table 10-3.

Activation

fMet

aa2

aa3

aa4

Binding

Translocation

ATP Equivalence

Table 10-3

Copyright by The Berkeley Review

409

The Berkeley Review

Specializing in NCAT Preparation

Biology

Expression of Genetic Information

Protein Synthesis

Termination

Suppose we are closeto the end of the 3' end of the mRNA. In Figure 10-35 is our

mRNA with some ntn codon. Attached to the n* codon is a polypeptide chain
with some ntn amino acid. This is all at the P-site. What we will find is that a

termination signal (e.g., UAG, UAA, or UGA) will come into view. Let's use 51UAG-3' as our stop signal These stop signals are not recognized by a tRNA but
instead are recognized by a protein release factor that will bind to the signal
when it comes into view. This release factor has an effect on peptidyl transferase,
the enzyme that forms peptide bonds between two amino acids. Peptidyl
transferase will now use water instead of an amino group to attack the ester

linkage of the ntn amino acid at the P-site. This will release the polypeptide
chain.

fMet

Arg
aa-aa-aa-aa-aa-aa.

H3C*. ^ - CH3

5' -

XXX

mRNA

70S Initiation Complex


Figure 10-35

Puromycin
You should be aware of the fact that inhibitors were useful (and continue to be)
O-CH,

Figure 10-36
Puromycin

in working out the pathways of protein synthesis. One inhibitor that we can
mention is puromycin, which is shown in Figure 10-36. Puromycin binds at the
A-site and acts as an analog of an aminoacyl-tRNA, thus preventing other
aminoacyl-tRNAs from entering the A-site. The a-amino group of puromycin
forms a peptide bond with the carboxyl group of the amino acid at the P-site.
Once this happens peptidyl puromycin leaves the 70S ribosome, thus causing
premature termination of protein synthesis.

Copyright by The Berkeley Review

410

The Berkeley Review


Specializing in NCAT Preparation

Biology

Expression of Genetic Information

The Lactose Operon

The Lactose Operon -^^^Sfe.il^^ffl^^^p


Inbacteria the majority of regulation occurs at the level of transcription (i.e., to
make or not to make a particular mRNA). We will now consider the lactose

operon. What exactly is an operon? An operon is simply a transcription unit


which isinvolved in the expression ofmultiple genes. Another way toword this
would be that an operon is an arrangement of genes on the DNA which are
cotranscribed.

In 1961 Francois Jacob and Jacques Monod proposed the operon model for the
regulation ofprotein synthesis from work they were doing on themetabolism of
the disaccharide lactose (glucose and galactose). Even though the bacterium E.

coli prefers glucose as a substrate it can survive onalternate carbohydrates such


as lactose, galactose, and even arabinose. However, in order for E. coli to utilize

these other sugars it mustsynthesize a whole new series ofenzymes. E. coli does
not want to be synthesizing these enzymes unless a particular substrate, like
lactose, is in the medium.

Thus, E. coli can induce the expression of certain genes only when they are
needed. A priori one can imagine an inducible system working in two ways: (1)
Ifyou have a system thatisalways onand then you turn it off or (2) ifyou have a
system that isalways off and then you turn it on. This "turning on" and "turning
off can be mediated by a variety of different mechanisms.

We mentioned that an operon is simply a transcription unit which is involvedin


the expression of multiple genes. For example, the lactose operon includes a
promoter(P), an operator (O), and structural genes (z, y, and a). Upstream from

the lactose promoter (on the 5* side of the lactose promoter) is the regulatory
gene (i) with its own promoter(Pi). These elements of the lactose operon model
are shown in Figure 10-37. We will modify this diagram as the discussion
continues.

Regulatory
gene

Control
sites

|v ^|<
<
z

]>|<

Structural Genes

>.l

Pi

l^

Lactose C

Figure 10- 37

When the enzyme RNA polymerasebinds to the lactose promoter it can begin to
transcribe the structural genes of the lactose operon. Structural gene z codes for
b-galactosidase whose major function is to hydrolyze lactose to glucose and
galactose (and whose minor function is to convert lactose to allolactose).
Structural gene y codes for galactoside permease, a carrier molecule that
transports lactose into the cell. Structural gene a codes for thiogalactoside
transacetylase whose role is still uncertain.

The regulatory gene (i) codes for a repressor protein that can bind to the
operator and prevent the transcription of genes z, y, and a when lactose (the

inducer) is not present in the cell's medium. In other words, the products of
those genes act on lactose in order to metabolize the disaccharide. If lactose is not
Copyright by The Berkeley Review

411

The Berkeley Review

Specializing in NCAT Preparation

Biology

Expression of Genetic Information

The Lactose Operon

present, why waste the cell's energy to synthesize those gene products? [An
operator is simply a DNA sequencethat binds a repressor.]
The conclusion that Jacob and Monod came to was that there is a diffusible

component in the cell that is involved in turning down the expression of a whole
series of genes which are all involved in the metabolism of lactose. The set of
genes that was needed to metabolize lactose would only be turned on and then
synthesizedwhen lactose was present in the medium. Thoseparticular genes are
the structural genes (mentioned above).

Let's consider what happens when lactose is not present in the medium. When
lactose is not present, you do not want RNA polymerase to transcribe the
structural genes z, y, and a. The expression of these three genes are controlled by
the regulatory gene (i). The regulatory gene codes for a repressor protein as
shown in Figure 10-38. This gene is transcribed separately from the structural
genes of the lac operon.

Figure 10-38

Once the repressor protein is synthesized it can do one of two things. If lactoseis
not in the medium, the repressor protein will bind to a region in the DNA
sequence called the operator (O) and prevent RNA polymerasefrom transcribing
the structural genes. The repressor protein binds tightly to the operator region as
shown in Figure 10-39.
X^s^5x
<
z

Pi

RNA polymerase cannot

Repressor protein binds

transcribe the structural genes

to the operatir

Figure 10-39

Even though the RNA polymerase can bind to the promoter site while the
repressor protein is boundto the operator site,it seems that the repressor protein
prevents transcription by RNA polymerase because it interferes with the
formation of the transcription bubble. It turns out that there is an overlap in the

Copyright by The Berkeley Review

412

The Berkeley Review

Specializing in NCAT Preparation

Biology

Expression of Genetic Information

The Lactose Operon

binding of RNA polymerase and the repressor protein to the duplex DNA
molecule.

However, if lactose is present in the medium, then a small amount of it will be

converted to allolactose by a few molecules ofp-galactosidase that are present in


the cell. This is shown in Figure 10-40.
HO-H2C

HO

H2C

H0-H,C

Jo (

\J

P-Galactosidase

HO

\ OH

OH

>

0H

Lactose

Figure 10-40

Allolactose will bind to the repressor and decrease the repressor's affinity for the
operator site. Thus, allolactose is the inducer of the lactose operon. Once the
inducer binds to the repressor, the repressor (and inducer) dissociates from the

operator and RNA polymerase canbegin to transcribe the lactose operon genes.
The polycistronic mRNA that is transcribed (mRNA which codes for two or

more polypeptide chains) will eventually betranslated into the desired enzymes.
This is shown in Figure 10-41.

Inducer-Repressor complex no longer


has affinity for the operatorregion
RNA Polymerase can transcribe

the lactose structural genes CZ

P-Galactosidase
Permease

Figure 10-41

It turns out that transcription of the E. coli lactose operon also depends on the
relative concentrations of glucose within the cell's medium. E. coli will utilize

glucose as the preferred energy source over lactose. Therefore, when glucose is
present in the medium there is reduction in the synthesis of the enzymes needed
to utilize lactose. This is called catabolite repression. However, when glucose is
absent from the medium, the rate of transcription of the enzymes needed to
utilize lactose is increased dramatically.
Copyright by The Berkeley Review

413

The Berkeley Review

Specializing in NCAT Preparation

Biology

Expression of Genetic Information

The Lactose Operon

^53^^
<

Pi

z
Q

CAPcAMP

crp

gene |

fv^^y
A decrease in

tuRNA**""*

(cAMP)

[Glucose]

^~r

means an increase in

[cAMP].

PAD ^^<

Figure 10-42

A considerable distance downstream from the structural genes of the lactose

operon is a gene [youdo not need to know the name but if you are interested it is
the crp gene) that codes for a protein called the catabolite activating protein
(CAP). CAP is sometimes called the cAMP receptor protein (CRP) as well. CAP
is a DNA binding protein that has the ability to mediate catabolite repression of
many inducible operons, such as the lactose operon. CAP can only bind to DNA
when it is complexed with cyclic adenosine monophosphate (cAMP). cAMP is
synthesized from ATP and increases in concentration when glucose levels are
low. When cAMP binds to CAP it forms the cAMP-CAP complex which can then
bind to the DNA at the CAP site, located right next to the lactose promoter on
the 5' side (upstream). This is shown in Figure 10-42.
RNA Polymerase can transcribe

the lactose structural genes c

CAP

^S^

Pi

CAPcAMP

y
^y

crp

gene |

SL Transcription

mRNA

(Polycistronic)

\y Translation

p-Galactosidase

Permease

Trans-

acetylase

Figure 10-43

When the cAMP-CAP complex is formed it acts as an activator and binds to the
CAP-cAMP site. Transcription of the lactose operon is enhanced by somehow
converting a weak lactose promoter into a stronger lactose promoter. This is
shown in Figure 10-43.
Mutants

A normal wild type E. coli bacterium would have a genotype i+z+y+a+. One of
the mutants that Jacob and Monod characterized had the ability to synthesize the

three proteins shown in Figure 10-43 at normal levels in the absence of the
inducer. This type of mutation was called a constitutive mutant (meaning that
the genes for those three proteins are always expressed and unregulated). The
mutant cells that they examined had a regulatory gene (the i gene) that had been
altered. Instead of this regulatory gene having an i+ genotype it now had an i"
Copyright by The Berkeley Review

414

The Berkeley Review


Specializing in NCAT Preparation

Biology

Expression of Genetic information

The Lactose Operon

genotype. In other words, this mutant E. coli bacterium would have thegenotype

,+y+al '"
i_z+**"

'"
*
i+. These i"mutants
can synthesize the three
proteins either in the"presence

orthe absence ofthe inducer whereas the i+ cells can only synthesize the three
proteins in the presence of the inducer. Jacob and Monod were able to show that

the i gene mapped close to thestructural genes.

Anormal i+z+y+a+ cell can make the repressor protein. If lactose is present, then

the repressor protein will bind with the inducer. This repressor-inducer complex

will notbindto theoperator and thus RNA polymerase will beable to transcribe

the structural genes. Acell which is i"z+y+a+ cannot make a functional repressor

protein. Therefore, whether the induceris presentor not doesnot matterbecause

there is no functional repressor to bind to the operator to prevent transcription.


Thus, the normal levels ofthe structural genes are always expressed. Hence, the
constitutive mutant.

Recall that we mentioned that bacterial cells such as E. coli have a large circular
chromosome. Besides this large circular chromosome they can also have a
smaller circular chromosome called the fertility factor (orF factor). The F factor
canexist in two states. It can eitherbe free in the cytoplasm of thebacterial cellor
it can be integrated into the larger circular chromosome. Since this F factor can
exist in eitherof these two states it is referred to as an episome (a word coined
forprecisely thischaracteristic). See Figure10-44.

Host

factor

Integrated F factor

chromosome

E. coli
Figure 10-44

Suppose we have a situation on the host chromosome in which the genotype is


i"Vy+a+. In this situation a normal repressor would be produced which would
bind inducer. The operator would no longer beoccupied and the structural genes
would be transcribed. However, because there is a mutation in the lac z gene,
giving the z" genotype, the enzyme b-galactosidase willnot besynthesized (i.e., it
will be defective). Suppose we have a situation on the F factor in which the

genotype is i"z+y+a+. In this case the repressor protein is non-functional, as we


have mentioned,the structural genes are continuously transcribed.

If we have both of these genotypes in the cell simultaneously (one on the host
chromosome and one on the F factor), then the cell acts as if it were a partial
diploid. Thisis indicated as i+z'/i"z+ as shown in Figure 10-45. When this partial
diploid was analyzedit was found to be inducible. In other words, the protein bgalactosidase was being synthesized. This meant that the normal i+ allele is
dominant to the recessive i" allele. If there is no inducer in the cell, then the

repressor protein from the i+ gene binds to the operator regions of both
chromosomes. Synthesis of (3-galactosidase does not occur. However, if the

inducer is present, then the repressor protein produced by the i+ gene and
Copyright by The Berkeley Review

415

The Berkeley Review

Specializing in NCAT Preparation

Biology

Expression of Genetic Information

The Lactose Operon

inducer bind and dissociate from the operator. In this case p-galactosidase can be
made. It was this type of genetic analysis that led Jacob and Monod to
hypothesize the presence of a repressor protein.

Figure 10-45
Partial diploid

Mutations were also discovered in the operator region of the lac operon. These
mutants were referred to as operator constitutive mutants (or Oc mutants) and

they allow for the continual transcription of the lac structural genes. Why? If
there is no inducer present, then the repressor would be free to bind to the

operator region. However, since there is a mutation in the operator region the
repressor cannotbind. Therefore, RNA polymerase can transcribe the structural
genes.

Considera partial diploidwhichhas the genotype i+0<z+/i+0+z+ (whichwe can


make by using an F factor). What happens in the absence of the inducer? In the
absence of inducer the repressor protein that is synthesized cannot bind to the Oc
site of the first chromosome because it has been mutated. Thus, RNA polymerase
can transcribe the structural genes. However, the repressor protein can bind the
0+ site of the other chromosome and thus prevent transcription. This says that
the Oc mutation is dominant over the wild type 0+ situation. In other words, in
the absence of the inducer there will be a constitutive synthesis of the gene

products in a cellwhich is Oc/0+.


The Oc mutants are said to be cis-acting mutants while the i" mutants are said to
be trans-acting mutants. A trans-acting situation refers to a product of a gene
that is diffusible. For example, in the case of the i" mutants we saw that the gene
product from the i+ gene of the partial diploid i+z"/i"z+ was able to diffuse over
to the mutant chromosome and bind to the operator in the absence of inducer.
The structural genes were not synthesized. A cis-aeting situation is characteristic

of a gene that affects only the genes juxtaposed to it. In other words, the Oc
mutation is felt only on that chromosome and not on the chromosome with the
wild type 0+ gene. This says that for a cis-acting situation there is no diffusible
product. Thismakessensebecause the operator region of the lac operon does not
produce any mRNA or protein that can diffuse anywhere.

Copyright by The Berkeley Review

416

The Berkeley Review

Specializing in NCAT Preparation

Biology

Expression of Genetic Information

i^il^tiia^^Oiewif

The Tryptophan Operon

V.OXJ*"'*!*J*l}ljf^

How can transcription be regulated at the level of termination? What are


terminators? There are two types of termination that occur in E. coli. The first is a
spontaneous termination, referred to as rho (r) independent termination; the

second is a factor dependent termination, called rho dependent termination.


Anti-Sense
or

Coding Strand

O
5'-N-N-N-N-N-N-N-N-N

DNA

.T-T-T-T-T -T-T-T -T-3',

-^ /- U-U-U-U-U-OH-3' mRNA
A
^-A-A-A-A-A-A-A-A^A-S'A U

J'-N-N-N-N'

duplex'

3-.N-N-N-N-N-N-N-N-N

G..CC

Sense

or

G C

Hairpin Loop

Template Strand

C G

/~| followed by a

Nis

any nucleotide
specific to DNA or RNA

C
G

G
C

series of U's.

Figure 10-46
Rho Independent Termination.

Rho independent termination usually occurs when RNA polymerase recognizes


a looped hairpinstructure followed bya run ofU's (Figure 10-46). Note the high
GC content in the stemof the loop. When thishairpin structure forms it appears
to jam the transcriptional process and causes the RNA polymerase to stall for a
few seconds. The result is dissociation between the deoxy-A residues in the
template strand of the DNA and the ribo-U residues in the mRNA. These two
bases form a relatively weak hybrid and are therefore not too stable.

Rho dependent termination occurs in the presence of a hexameric protein


referred to as the rho factor. This enzyme is actually a helicase thancancatalyze
the unwinding of RNA-RNA and RNA-DNA double helices. This enzyme
recognizes an 80 to 100 nucleotide sequence on the nascent mRNA that is

upstream from the termination site. Rhofactor binds to this recognition site and
moves along the mRNA (5'- 3') until it finds an RNA polymerase that has
paused. It then begins to unwind the RNA-DNA duplex and terminates
transcription.

During the early 1950's Charles Yanofsky began to study the regulation of
tryptophan synthesis by looking at the genes of the tryptophan operon
(abbreviated as trp operon). He began by isolating two types of mutants. One
type of mutant involved structural gene mutations. These types of mutants were
auxotrophic for tryptophan. These mutants required tryptophan for growth as

they were unable to synthesize this amino acid. Yanofsky developed a map for
these genes as shown in Figure10-47. The trpE, trpD, trpC, trpB, and trpA genes
coded for a polycistronic message which gave rise to the enzymes needed to
convert the precursor molecule chorismate to tryptophan.
Copyright by The Berkeley Review

417

The Berkeley Review

Specializing in NCAT Preparation

Biology

Expression of Genetic Information

The Tryptophan Operon

Attenuator

S \trpR |

trpP,Q

trpL
Transcription

trpE

trpD | trpC

trpB

\ trpA

Translation

Indole-

Repressor
Anthranilate
Anthranilate phosphoribosyl
synthase
transferase

Chorismate i

^'~

Phosphoribosyl
anthranilate
isomerase

^r

3-glycerol
phosphate Tryptophan
synthase
synthase

S Tryptophan

Figure 10-47

The other type of mutant Yanofsky obtained was a regulatory mutant. These
mutants were able to constitutively synthesize the enzymes necessary for the
biosynthesis of tryptophan. Somehow they were altered in their ability to
regulate the expression of the tryptophan structural genes.
In other words, the trpR gene, which codes for the tryptophan repressor, is not
effective in regulating the synthesis of tryptophan. The gene for frpR mapped in
another quadrant of the E. coli chromosome, at about 100 map minutes, while the
trp operon mapped at about 28 minutes. Yanofsky purified the dimeric trp
repressor protein and discovered that it does not function alone. In order to
regulate the synthesis of tryptophan the repressor must bind the end product of
that metabolic pathway. The end product is tryptophan.

Therefore, tryptophan acts as a corepressor for its own biosynthesis. When the
concentrations of tryptophan are high, the repressor binds to tryptophan and this
repressor-tryptophan complex binds to the operator region of the trp operon.
When this complex is bound to the operator it prevents RNA polymerase from
initiating transcription of the structural genes. This is an example of feedback
repression at the transcriptional level.
In contrast, if the concentration of tryptophan is low in the cell, the repressor-

tryptophan complex will not form and therefore will not bind to the operator.
RNApolymeraseis able to transcribe the structural genes and tryptophan will be
synthesized.

Around 1975 Yanofsky isolated more mutants in the trp operon and came up
with some results that could not immediately be explained. Upstream from the
start of the structural genes and downstream from the operator is a 162

nucleotidesequence of DNAcalled the leader sequence. This sequenceis coded


for by the trpL gene. Yanofsky had isolated trp deletion mutants in the leader
region that resulted in a 8 to 10-fold increase in the expression of the structural
genes of the trp operon. How could this be explained?
Towards the 5' end of the 162 nucleotide leader sequence is a region of 14 amino

acids called the leader peptide. The 10th and 11th amino acids of this leader
Copyright by The Berkeley Review

418

The Berkeley Review

Specializing in NCAT Preparation

Biology

Expression of Genetic Information

The Tryptophan Operon

peptide are tryptophan residues. These two amino acids are also part of a
nucleotide sequence that isGC rich. There are four GC rich regions inthe leader
sequence. They are sometimes called region 1, region 2, region 3, and region 4.
This is shown in Figure 10-48. Because these GC regions are complementary to

each other, we find that two alternative secondary structures resembling


hairpins can be formed.

One secondary structure involves base pairing between region 1 and region 2
and between region 3 and region 4. Note that there are a series of ribo-U's after

the hairpin formed by regions 3 and 4. This represents a rho-independent


termination site. The other secondary structure involves base pairing just
between region 2 and region 3. This base pairing sequence will only occur if
region 1 is not available for base pairing with region 2. Note that GsC base
pairing is rather stable (asopposedto A=T basepairing).

We mentioned that when the concentration of tryptophan is high the repressor


protein binds to tryptophan and forms the repressor-tryptophan complex. This
complex binds to the operator and prevents transcription. If the concentration of
tryptophan is low, then the repressor-tryptophan complex does not form, the
operator is left open, RNA polymerase binds to the promoter, and transcription
of the structural genes occurs.
Attenuator

trpT*,0 | trpL B

trpE

Leader peptide

^>

Trp-Trp

Region 1
(GC rich)

Region 2
(GC rich)

Region 3
(GC rich)

Region 4
(GC rich)

or
U-U-U-U-3"

U-U-U-U-3'

Figure 10-48

Let's examine transcription of the leader region in a little more detail. Suppose
that the cell has a sufficient level of tryptophan. As RNA polymerase transcribes
region 1 and region 2 these two regions begin to form a hairpin structure that
causes the polymerase to pause. By this time the Shine-Dalgarno sequence
(towards the 5' end of the mRNA) has been made and a ribosome binds and

begins translation (which we will discuss in more detail in a future lecture). As

the ribosome translates the leader peptide region it disrupts the hairpin created
between regions 1 and 2.

RNA polymerase then begins to transcribe region 3. Meanwhile, the ribosome


reaches the tryptophan codons and inserts the required tryptophan amino acids
Copyright by The Berkeley Review

419

The Berkeley Review

Specializing in NCAT Preparation

BiolOfly

Expression ofGenetic Information

The Tryptophan Operon

(supplied by a charged tryptophanyl-tRNATrP molecule) into the growing


polypeptide chain. As the ribosome moves downstream a few more codons it
encounters a termination codon and stops translation. The ribosome dissociates

and the hairpin between regions 1 and 2 reform. As RNA polymerase finishes
transcription of region 4 ahairpinstructure forms between region3 and region 4.
This hairpin is a termination signal because of the ribo-Us that immediately
follow region 4.Once this termination signalis formed the transcription complex
dissociates from the duplex DNA and the structural genes are not transcribed.
Tryptophan will not be synthesized.

Suppose that the levels of tryptophan in the cell are quite low. It should be
obvious that the cell needs to synthesize more tryptophan. Initially the RNA
polymerase and the ribosome follow the same sequence of events we just
mentioned but with one important difference. When the ribosome reaches the
tandem tryptophan codons it pauses because the concentrations of tryptophan in

the cell are quite low. There are not enough charged tryptophanyl-tRNATrP
molecules to bring tryptophan to the site of protein synthesis. This results in the
ribosome sitting on region 1 and covering it up. Meanwhile, RNA polymerase has
transcribed region 2 and region 3. Since region 1 is not available for hairpin
formation with region 2 it turns out that region 2 will form a hairpin structure
with region 3. By this time RNA polymerase has transcribed region 4 and the
string of ribo-U's.
Region 3 will not base pair with region 4 because the hairpin structure formed
between region 2 and region 3 is more stable than the hairpin structure that could
be formed between region 3 and region 4. This means that the termination
hairpin is not formed. The transcription complex is not disturbed and RNA
polymerase transcribes the structural genes of the trp operon. This type of
regulation of the synthesis of mRNA is referred to as attenuation and the control
element which is responsible for this phenomenon is called an attenuator.
With this information in mind we can return to the question we posed earlier.
Why would mutants in the leader region have 8 to 10 fold levels increased
tryptophan biosynthesis? The deletion mutants that were made had a deleted
terminator sequence. If this occurs, tryptophan will be synthesized.

Why bother with two regulatory systems (i.e., the repressor-tryptophan complex
and attenuation)? The repressor system controls the level of the tryptophan
enzymes some 70 fold. Attenuation gives an 8 to 10 fold range of expression. The
roughly gives a 600 to 700 fold range of control. In other words, the control is
broad. The tryptophan operon began to explain other data that researchers could
not interpret. For example, a repressor could not be found for the histidine

operon, the phenylalanine operon, the leucine, threonine or valine operons. After
the discovery of the tryptophan operon and its regulation the leader regions of
these other operons were sequenced. It was discovered that all these leader
regions were rich in the amino acids which they controlled.
The histidine operon had a leader region that contained 7 tandem histidine
codons. The phenylalanine. operon had a leader region that contained 7
phenylalanine codons. These leader regions also contained alternative hairpin
structures which could act as termination sequences. In some cases there was
multivalent repression. There were several end products from some of these
pathways and more than one amino acid could feedback to regulate
transcription.

Copyright byThe Berkeley Review

420

The Berkeley Review


Specializing in NCAT Preparation

Biology

Expression of Genetic Information

Nutations fir Proofreading

Mutations
ism*i

The most common type of mutation is where there is substitution of one base

pair for another base pair. A transition mutation occurs when one purine is
replaced byanother purine orone pyrimidine isreplaced by another pyrimidine.
A transversion mutation is simply thereplacement ofa pyrimidine by a purine

A-T

or a purineby a pyrimidine. These mutations aresummarized in Figure 10-49.

Tautomerism

How can transition mutations occur? Watson and Crick noticed that certain

T-A

C-G

hydrogen atoms oneach ofthe four bases can change their positions thus giving

G-C
Transitions

rise to tautomeric structures. Forexample, consider thepyrimidine base thymine


(in the lactam form). Under normal conditions thymine will bond with adenine
as shown in Figure 10-50.
^^.Hin

A-T ^ - ^ T-A
i

CHj

C-G <
/

Backbone

//

>

G-C

Transversions

Backbone

Figure 10-49

Thymine

Adenine

(lactam form)

Figure 10-50

However, if the hydrogen atom on the nitrogen at position3 in the thymine ring
movesto the oxygen atom at position 4, then we have a rare form of thymine (the
lactim form) that is capable of hydrogen bonding with guanine as shown in
Figure10-51. The fraction of thymine in this rare form is about 10"4.
Omni

iiiinH0

Backbone

Guanine

N H

iiiiiimiiO

Bacl
Backbone

Thymine
(lactim form)

Figure 10-51

Suppose we have a duplex of parental DNA, as shown in Figure 10-52a, in which


we focus on a particular A-T base pair. Each strand is labeled 1 and 2 so we can
follow them during replication. The DNA duplex goes through the first round of

replication as shown in Figure 10-52b. Parental strand 1 successfully gives rise to


a daughter strand (labeled 3). However, as parental strand 2 is being replicated
our thymine base tautomerizes into the rare lactim form of thymine (indicated by
T*). In the nascent daughter strand (labeled 4) a guanine will now be inserted
instead of an adenine.

Copyright by The Berkeley Review

421

The Berkeley Review


Specializing in NCAT Preparation

Biology

Expression of Genetic Information

Nutations 6c Proofreading

During the secondround of replication, as shown in Figure 10-52c, we will get


two normal DNA duplex polymers from the duplex with strands 1 and 3. Since
our rare tautomeric form of thymine does not last that long we find that during
the second round of replication it returns to the normal lactam form. After the
DNA duplex with strands 4 and 2 go through the second round of replication,
we will get one normal DNA duplex polymer, indicated by strands 2 and 8,
and one mutant DNA duplex polymer, indicated by strands 4 and 7. What we
have done is gone from a normal situation in which the A-T base pairs in
strands 1 and 2 have spontaneously mutated (transition mutation) to a G-C
base pair as shown in strands 4 and 7.

DNA

Duplex

Proofreading via 3'-> 5' Exonuclease Activity


Since the fidelity of the genetic message needs to be preserved in order to
proliferate the organism, there are methods of detecting mutations like the
ones that we have just mentioned. Both DNA polymerase I and DNA
polymerase III possess a 3' -> 5' exonuclease activity.

IIIIII!I
A=T A=T

G=C

AzT

IIIIIMI

15

I
Normal

Mutation

Normal

Suppose we have a hypothetical template strand composed of adenine bases as


shown in Figure 10-53. During our replication process we are incorporating
thymine bases into the growing nascent daughter strand. A mistake occurs and
a rare tautomeric form of cytosine is brought in and becomes part of the nacent
DNA chain (because it looks like thymine).
DNA polymerase is continually checking the segment of new DNA that it has
just synthesized. If it detects an error, such as a cytosine where there should be
a thymine, then that error will be removed by the 3' > 5' exonuclease activity
of the enzyme. After the correction is made the polymerase will continue with
the replication of the DNA template. This proofreading mechanism reduces the
error rate by about a thousand times. Cells with high mutation rates have a
defective DNA polymerase (HI) exonuclease activity.

Figure 10-52

Cytosine will be removed


with 3' to 51 exonuclease

activity.

5' -T-T-ThT-T-Ct

I I I (l I I I

3'

' )-

3' -A-A-A^A-A-A-A-A-A-

5'

DNA Polymerase HI
Figure 10-53
Deletion

Supposein our hypothetical templatestrand composedof adenine basesone of


those bases slips out of register as shown in Figure 10-54. As the nascent DNA
strand is being synthesized a thymine base will not be incorporated at the point
of slippage (or looping out) of the adenine base. The result is that there will be
one less thymine base in the nascent DNA strand. In other words, there has
been a deletion of a base.

Copyright by The Berkeley Review

422

The Berkeley Review


Specializing in NCAT Preparation

Biology

Expression of Genetic Information

Nutations fir Proofreading

Deletion of base

5' T T T

3'-A-A-A

'

j T T T

A-A-A-A-A5'

SlippageI
of base

DNA Polymerase m

Figure 10-54
Insertion

Suppose, once again, in our hypothetical template strand of adenine bases there

has been aslippage ofa thymine on the nascent DNAstrand asshown in Figure
10-55. This will resultin an additional thymine base being added to the growing
DNA strand. In other words, we have an insertion of a base. Sequences like
these, where you have a run of the same base, are hot spots for length mutations
(e.g., either deletions or insertions).

H,N

Insertion of base

Slippage
ofbase

5' T T T^

II II II
3' - A - A - A Template

\x-T-T 3'

#//

Backbone

Cytosine

)1-

Deamination

A"* A- A- A- A- A5'
DNA Polymerase in

Figure 10-55
O

Backbone

Deamination

Recall that the bases cytosine, adenine and guanine have amino groups on them
that can be hydrolyzed. Roughly 5,000 amino groups are lost from these bases
per cell per day. The example that we will consider is cytosine being deaminated
to form uracil as shown in Figure 10-56.

Uracil

Figure 10-56

Uracil is an analog of thymine. It base pairs just like thymine. If a cytosine were
to be deaminated to form uracil in the template strand of DNA, then the
polymerase would put in an adenine at the corresponding position on the
nascent DNA strand instead of a guanine. This is another way in which
transition mutations can arise.

However, the cell has a repair system that recognizes these uracils and removes
them. Consider the DNA duplex shown in Figure 10-57 and the cytosine that has
spontaneously deaminated to form uracil. The enzyme uracil-DNA glycosidase
hydrolyzes the N-glycosidic bond between the deoxyribose ring and the uracil
base. The uracil base is removed.

Copyright by The Berkeley Review

423

The Berkeley Review


Specializing in NCAT Preparation

Biology

Expression of Genetic Information

II

II

III

II

III

Ucut G

Nutations ec Proofreading

Figure 10-57

This site on the DNA duplex is called an AP site (either apurinic or apyrimidinic)
because it is without either purine base or a pyrimidine base. This base defect is
recognized by an enzyme called AP endonuclease which cleaves the bond on the

3' side of the phosphodiester bond of the nucleotide with the missing base. DNA
polymerase I cleaves the phosphodiester bond at the 31 end on the next

nucleotide unit via its 51 -31 exonuclease activity. Thisis shown in Figure 10-58.

3'

"nJVI^JVJW^

5'

II

AP

III

II

III

Site

5'

<g^ mjm ^^ ^ \,h

Cut by AP
endonuclease

Cut by DNA
Polymerase I

Figure 10-58

Once the defective ribose-phosphate unit is removed, DNA polymerase I will


read the complementary strand and find the base that corresponds to the AP site.
In this case it turns out to be a guanine. This is how DNA polymerase I knows to
insert a cytosine at the AP site on the damaged DNA strand. DNA ligase will
then seal the inserted cytosine into the damaged strand in the usual fashion.
Mismatch Repair

The most common types of mutations are caused by tautomerism and slippage
during replication. Suppose we already have a DNA duplex that has already
been replicated. During this replication we had a tautomerism in which a
thymine momentarily looked like a cytosine. The result was that a guanine was
incorporated into the duplex rather than an adenine. Let's also say that our
proofreading system failed to catch this error. DNA polymerase might leave

about one mistake per 108 replicated base pairs. Because of this imperfect base
Copyright by The Berkeley Review

424

The Berkeley Review


Specializing in NCAT Preparation

Biology

Expression of Genetic Information

Nutations &Proofreading

pairing (G-T) there is a slight bulge in theDNA duplex at that point. This is
shown in Figure 10-59.
Mismatch Repair Enzyme

^synthesized
DNA strand

J*&ZS$&
/
CTAG'
GATC

I
CH,

Template DNA
Strand

Figure 10-59

The mismatch repair enzymeslides along the DNA duplexfeeling for bulgeslike
the one we just mentioned. Once this enzyme finds one of these bulges it does
not know which base was incorporated into the replicating DNA in error. Is it
the guanine or is it the thymine?

The enzyme needs to make a decision. It scans the duplex DNA for help. What
the enzyme is searching for is the palindromic sequence of bases G-A-T-C. This
is referred to as a GATC box. This box will tell the enzyme which is the newly
synthesized DNA strand. How?

The template strand (the parent strand) turns out to have the adenine base
methylated in each of these GATC boxes. The methyl groups are attached to
adenine by SAM (S-Adenosylmethionine). This process of methylation is a signal
that identifies the old DNA strand from the new DNA strand. The newly
synthesized DNA strand is not methylated until a few seconds or minutes after
the replication process. This gives the mismatch repair enzyme plenty of time to
locate these errors.

Once the mismatch repair enzyme finds a bulge in the DNA duplex, how far

does it have to travel before it will find a GATC box? The probability is (1/4)4.
The probability that a particular base at a particular position is, say, a guanine is
1/4. The probability that a guanine will have an adenine right next to it is
(1/4)(1/4). Therefore, the probability that there will be a sequence GATC is
(l/4Ml/4)(l/4)(l/4) or (1/4)4 or 1/256.
This means that the enzyme will need to more along the DNA duplex about 256
residues before it will randomly encounter a GATC box. (Note that this is not

quite correct. What we have done is to calculate how far apart the GATC boxes
are from one another on a duplex of DNA.)
We now have a way of telling which is the correct base and which is the incorrect
base. Once the incorrect base is located, the mismatch repair enzyme binds to the
unmodified GATC box (with the non-methylated adenine) and to the incorrectly
inserted base as shown in Figure 10-60a.

Copyright by The Berkeley Review

425

The Berkeley Review


Specializing in NCAT Preparation

Biology

Expression of Genetic Information

Nutations & Proofreading


(b)

(a)

Template

CTAGGATC-

I
CH,

Figure 10-60

The mismatch repair enzyme then removes the mispaired base and the
intervening sequence of DNA up to the point of the unmethylated GATC box as
shown in Figure 10-60b.

DNA polymerase (I) will then resynthesize the DNA that has been removed by
the mismatch repair enzyme and will replacethe mispaired base with the correct
base, which in this case would be a adenine. DNA ligase will then catalyze the
formation of a phosphodiester bond between the remaining 3'-hydroxyl at one
end of the replaced DNA strand and the 5'-phosphate of the previously
synthesized DNA strand. This is shown in Figure 10-61.
Template DNA

DNA

Strand

Polymerase

f5"
DNA Ligase will form
phosphodiester bond

Figure 10-61

Mutagens

Certain types of external agents can not only cause mutations but can also
increase the possibility of mutations. These agents are called mutagens. Three
types of mutagens that we will briefly consider are base analog, chemical, and
ultraviolet mutagens.
Base Analog

A base analog mutagen can easily substitute for a naturally occurring base in
DNA. For example, the compound 5-bromouracil is a base analog of thymine.
Copyright by The Berkeley Review

426

The Berkeley Review

Specializing in NCAT Preparation

Biology

Expression of Genetic Information

Nutations fir Proofreading

This can be seen in Figure 10-62. Even though the bromine group of 5bromouracil is more electronegative than the methyl group of thymine, the
methyl group and the bromine group have about the same van der Waals radius.
One of the possible base pairs that results after replication from the substitution

of5-bromouracil in DNA is 5-bromouracil-adenine. After a subsequent round of


replication occurs, a transition mutation would result.
O

CH3

,-H

HN 3

/-"
O

Backbone

Thymine

Backbone

5-Bromouracil

Figure 10-62

Chemical (Nitrous Acid (HN02))

Roughly 5,000 amino groups are lost from the bases cytosine, adenine and
guanine per day. Nitrous acid can cause the deamination of cytosine to uracil as
shown in Figure 10-63. Cytosine wants to base pair with guanine. However, if
cytosine is deaminated to uracil, then uracil will have a tendency to base pair
with adenine. The result is that a transition mutation would occur.

H2N

f4

>\

Deamination

V ,>
0

/4

v\

>H-v ,)

Backbone

Cytosine

Backbone

Uracil

Figure 10-63
Ultraviolet (UV) Radiation

If two pyrimidineresiduessuch as thymine are adjacent to one another, then the


energystored in UV radiation (at a wavelength of about 260 nm) can link them
togetheras shown in Figure 10-64. Thymine dimers inhibit DNA replication.

Sugar N ,H

Phosphate

// N'/

SugarN /

Thymine Dimer

lf=z O

Figure 10-64

Copyright by The Berkeley Review

427

The Berkeley Review


Specializing in NCAT Preparation

Biology

Expression of Genetic Information

Nutations & Proofreading

Pyrimidine dimers can be corrected by a repair mechanism. A specific


endonuclease enzyme (called UvrABC) will cut on both sides of the dimer and
remove a segment of about 12 nucleotides as shown in Figure 10-65.
Thymine

Dimer \. Endonuclease Enzyme

DNA

Damaged DNA is removed by enzyme

TBT-

TTTtf I I I I I I I
DNA

ll
DNA Polymerase I fills in the
missing nucleotides and is
sealed by DNA Ligase

I
T-T"

DNA

Figure 10-65

The resulting gap is filled in by DNA polymerase I and (eventually sealed by


DNA ligase.
Intercalators

Thetricyclic benzenoid hydrocarbon called anthracene, whichis shownin Figure


10-66, is the basic planar structure for many ring systems which can intercalate
Anthracene

Figure 10-66

between adjacent basepairsin the DNA molecule. These mutagensare about the
size of a base pair, making them quite capable of slipping in between adjacent
base pairs in DNA and causing frame shift mutations (e.g., either the insertion
or deletion of one or more base pairs) during the replication process. If it were
not for the repair systemsthat we have been discussing, an organism would soon
die due to the various lesions that can be introduced into DNA.

Copyright by The Berkeley Review

428

The Berkeley Review


Specializing in NCAT Preparation

Biology

Expression of Genetic Information

The Ames Test

Ames Test

The biosynthetic pathway for histidine requires ten enzymatic steps. Even
though histidine is an essential amino acid for humans, bacteria like E. coli and

Salmonella typhimurium are quite capable of synthesizing it. In thesebacteria each


ofthese ten enzymes is encoded forby a gene and allofthese genes are clustered

together in an operon, which is essentially a setoffunctionally related genes. We


cansimplifythe histidine pathway as shown in Figure10-67.
Possible point
of mutation

Enzl

A'

Enz 2

> B'

Enz 3

Enz 4

Enz 5

Enz 6

> C = > D => E = > F '

Enz 10

Histidine <

Enz 9

iJ <

>G

Enz 8

11 <

lBm7
iH

Figure 10-67

If there is a mutation in any one of these genes, then these bacteria will no longer
be able to make histidine. If they cannot make histidine, their growth will stop
because of the requirement of histidine in many metabolic pathways. For
example, histidine is a five-carbon amino acid which can be degraded to ocketoglutarate which can then enter into the Krebs cycle. If histidine is required
for growth, then those organisms are referred to as being His". Such an organism
is auxotrophic (it needs something for its growth). If the organism has no
problem in synthesizing histidine, it is referred to as being His+. Such an
organism is prototrophic (it can make what it needs).
Ames started with a gram negative bacterium that was a histidine auxotroph
(His"). These mutations are very often transition point mutations. The transition
mutation not only has the possibility of being created from a normal gene, but it

also has the possibility of being reverted from a mutant form back to a normal or
wild type form (e.g., His+). The reversion of a point mutation back to a normal
base pair can occur by way of a transition mutation. A transition mutation is
simply the replacement of one pyrimidine by another pyrimidine or one purine
by another purine (e.g., the base pair A-T for G-C). A transversion mutation,
however, involves the replacement of a pyrimidine by a purine or a purine by a
pyrimidine (e.g., the base pair A-T for C-G). This is the basis of the Ames Test. It
examines the efficiency of conversion of a His" cell (an auxotroph) to a His+ cell
(a prototroph) that can grow on a minimal plate without histidine.
A lawn of bacteria that are His" are spread out to confluence on a petri plate that
lacks histidine. The His" bacteria settle down and may go through one or two cell
divisions before they stop growing. They need histidine to grow. A test spot
containing a chemical that you suspect is a mutagen is then applied in the middle
of the plate. If this chemical causes mutations, then with some frequency it will
revert the His" auxotrophic mutations back to His+ prototrophs. If this happens,
then colonies of bacteria will emerge in the vicinity of the test spot. This can be
seen in Figure 10-68.The potency of the mutagen is roughly proportional to the
Copyright by The Berkeley Review

429

The Berkeley Review


Specializing in NCAT Preparation

Biology

Expression of Genetic Information

The Ames Test

size of the ring of colonies mat forms around the test spot. Ames has been able to
show that chemical mutagens have a high probability of also being carcinogenic.
Test Spot
(suspected mutagen)

Revertant colonies

in vicinity of
mutagen

Plate with minimal medium

(no Histidine)

Figure 10-68

If we are providing so much mutagen that there might be a chance that new
mutations will be introduced into the same bacterium that has sustained a

reversion event, then we might lose that bacterium if the new mutations also

inactivate one of the 10 enzymes required for, say, histidine biosynthesis. This
capacity is diminished by the fact that (1) mutations that are used to measure

reversion frequencies are particularly easily mutable and revertible. In other


words, they are hot spots (i.e., the context in which the mutation is located

readily recruits mutagens). The effective dose of mutagen detected by this test is
such that (2) the reversion event is one of very few mutations that would be
caused by a mutagen. Wedon't want to placeso much mutagen in your test spot
such that we create lots of mutations in each bacterium. We only use a little
mutagen so that we are detecting relatively small numbers of mutations in each
bacterium.

Dilute region

Area of high mutagenic


concentration

His"1
His

Plate with minimal medium

(no Histidine)

Figure 10-69

If we were to use a high concentration of the mutagen, then that area


immediately around the test spot would be void of any revertant colonies. This is
because we are creating other mutations that prevent the revertants from
growing. As the mutagenic agent moves out and away from the point of
application, its concentration decreases (i.e., it becomes more dilute) as shown in
Figure 10-69. If we compare Figure 10-68 with Figure 10-69, we will find that the
concentration of the mutagenic agent in Figure 10-68 is not as concentrated as the
mutagenic agent in Figure 10-69.

Copyright by The Berkeley Review

430

The Berkeley Review


Specializing in NCAT Preparation

Biology

Expression of Genetic Information

Genetic Engineering &cloning

Genetic Engineering
Genetic Engineering & Cloning
Let's consider some of the technology involved in analyzing and altering genes.
Recombinant DNA technology (or genetic engineering) notonly depends on the
restriction enzymes and the modification systems found in bacteria but also on

the enzymatic processes inherent in specific proteins that are designed to


synthesize and process the DNA and RNA found in both eukaryotes and
prokaryotes.

Recall that we have mentioned that proteins could be cut into smallerfragments
by usingenzymes such as trypsin or chymotrypsin. It wasn't until the early 1970s
that this application could be applied to nucleic acids. Restriction enzymes (or
restriction endonucleases) recognize specific sequences in a DNA polymer and
then cleaves that DNA at those sites. For example, bacterial cells have a variety of
restriction enzymes which can degrade the foreign DNA of an attacking virus.
The bacteria's restriction enzymes do not degrade its own DNA because its DNA
has been methylated at specific locations by modification enzymes. When the
viral DNA is injected into the bacterial cell its DNA has not been methylated yet

Phage particles create


a clear plaque in lawn

and so the bacteria's restriction enzymes recognize that DNA as being foreign.
Petri dish with agar and a

Bacteria can be grown on a solid surface of nutrient agar in a petri dish. This

solid medium is called a plate. If you add about 108 . coli bacteria to your plate,
they will eventually form a "lawn" of bacteria that will cover the surface of the
nutrient agar. If a phage infects a bacteria in this assay, then that infected
bacterium will lyse and release more phage progeny which in turn will infect
other bacterial cells. After several hours of repeated infection by the phage
progeny, a clear, circular region will form in the lawn of bacteria. This clear
region, as shown in Figure 10-70, is called a plaque. The efficiency of plating
(EOP) for most bacterial viruses is about 1. The EOP is that fraction of phage
progeny that can form a plaque.

confluent lawn of E. coli

Figure 10-70

X- Phage
RI Plasmid
EOP = 1 in 500

C ^)
Lawn of E. coli

ffi^
Lawn of E. coli with

RI Plasmid

Figure 10-71

In 1953 it was discovered that if you try to grow a bacterial virus on a new strain
of bacteria, that virus will run into trouble. The bacteria virus called lambda (1)

phage had been grown on a standard E. coli strain. The EOP on these plates was
about 1. If you took phage from a plaque and tried to grow it on an E. coli strain
carrying an RI plasmid that was known to have drug resistance genes (the "R"

stands for drug resistance), then the EOP decreased to 2 x 10"3. In other words,
Copyright by The Berkeley Review

431

The Berkeley Review


Specializing in MCAT Preparation

Biology

Expression ofGenetic Information

Genetic Engineering S? Cloning

about 1 in 500 virus particles would make a plaque. This is shown in Figure 1071. It could be shown that most of the 1-phage DNA had been degraded.
If you take a phage from one of those 1 in 500 plaques that could be formed and
plate it on the same plate, then the EOP would be 1. If you plated a phage from
that plaque back on the original strain of E. coli, then the EOP would also be 1.

However, if you now take one of these plaques and replate it on the plate with
the E. coli stain carrying the RI plasmid, you will once again get an EOP of about

2 x 10"3. This canbeseen in Figure 10-72.


EOP = l

Lawn of coli

Lawn of E. coli with


RI Plasmid

EOP = 1 in 500

Figure 10-72

Initially the phage were restricted in their growth but later became modified in

order to grow on the E. coli strain with the RI plasmid. If these modified phage
grow for a number of generations on the E. coli strain without the RI plasmid,
they will eventually lose their modification and once again become restricted.
Thus, it was thought that the RI plasmid codes for some genes that have the

EcoRI
Cut

ability to restrict and modify phage DNA.

S'-pG-pA-pA-pT-pT-pC-S* Herbert Boyer(at UCSF) purified the RI plasmid restriction endonuclease. The
S'-Cp-Tp-Tp-Ap-Ap-Gp-S' sequence of DNA that is recognized by the Eco RI restriction endonuclease is

ft
Cut

fl
/

Sticky End
*

5'-pG
3'~Cp-Tp-Tp-Ap-Ap
v
'
Sticky End

Figure 10-73

arrows. Once the phosphodiester bonds in the duplex DNA are nicked, it is a

simple task to denature the hydrogen bonds holding that portion of the duplex
DNA together. Note that this particular cleavage site has twofold rotational
symmetry. It is palindromic. A palindrome is a word (e.g., radar and rotator), and
expression (e.g., nurses run), or a sentence (e.g., a man, a plan, a canal, Panama) that

pA-pA-pT-pT-pC3'
Gp-5'

shown in Figure 10-73. This enzyme cuts the DNA at the sites indicated by the

reads the same from right to left as it does from leftto right. Once the hydrogen
bondsbreak wewill beleft withtwoends which are cohesive (i.e., "sticky ends"),
meaning that they canjoinbacktogetheragain given the right conditions.
How often does this enzyme cut DNA? There are 6 nucleotides in the Eco RI

sequence. There are also 4 possible nucleotides at the first position in the Eco RI
sequence,4 possiblenucleotides at the second position, and so on. If we multiply
these possibilities together (4x4x4x4x4x4), it turns out that (on the average) every
4,096 nucleotides (or about every 4 kb) there will be an Eco RI restriction site.
Remember, though, that this is only in theory.

When you look for restriction sites in DNA you want to look for two-fold
rotational symmetry. For example,consider the DNA sequence shown in Figure
10-74. Only one strand has been shown because the complementary strand is
quite easy to write. However, you should still be able to find regions of two-fold
Copyright by The Berkeley Review

432

The Berkeley Review

Specializing in MCAT Preparation

Biology

Expression of Genetic Information

Genetic Engineering & Cloning

rotational symmetry ifpresented with only one strand ofDNA. In this example
there are two possible regions oftwo-fold rotational symmetry.
Regions of twofold

InFigure 10-74 we have one region of two-fold rotational symmetry that is 6


nucleotides in length. We mentioned that (on average) we would expect to find
one cutmadebysucha restriction enzyme every 4,096 base pairs. This particular
restriction enzyme is called Bgl II (from Bacillus globigii). Also shown in Figure
10-74 is a region of two-fold rotational symmetry thatis 4 nucleotides in length.
We would expect this restriction enzyme to make a cutin the DNA (on average)

rotational symmetry.

-A-A-G-A-T-C-T-G-G-C-CFigure 10-74

every 256 base pairs. This particular restriction enzyme is Hae III (from
Haemophilus aegyptius).
Let's take a look at Bgl II and Hae III in a little more detail. The restriction sites
for Bgl II are shown in Figure 10-75a. Note that if we make our cuts at the
arrows, we will have a 4-base overhang that we call the sticky end(s). These

overhangs are also referred to as being5'-overhangs because of the 51 phosphate


group that is still attached to that particular end. This is similar to the restriction
enzyme Eco RI. Hae III does not give sticky ends when it makes its cut but
rather gives blunt ends as shown in Figure 10-75b.

There are also restrictionendonucleases which will give 3'-overhangs when they
make their cut in the DNA. An example of such an enzyme is Hae II. This is
shown in Figure 10-76. In this particular example, "Pu" stands for a purine while

(a)
Cut

5'-pA-pG-pA-pT-pC-pT-3'
3'-Tp-Cp-Tp-Ap-Gp-Ap-5'
Bgl

"Py" stands for a pyrimidine.

Cut

Cut

5'-pG-pA-pT-pC-pT-3'
Ap-5'

5'-Pu-pG-pC-pG-pC-pPy-3'
3'-Pyp-Cp-Gp-Cp-Gp-Pup-5'
Cut

5'-pA
S'-Tp-Cp-Tp-Ap-Gp-S*

I I Hae II

(b)

5'-Pu-pG-pC-pG-pC-3'

3'-Pyp

Cut

pPy-3'
3'-Cp-Gp-Cp-Gp-Pup-5'

5'-pG-pG-pC-pC-3'

S'-Cp-Cr^Gp-Gp-S'

Figure 10-76

Cut

The modification enzyme for EcoRI (a DNA methylase enzyme) recognizes the

same sequence as the restriction enzyme and transfers a methyl group from a
molecule called S-Adenosylmethionine (SAM) to the C-6 position of the purine
adenine to make 6-methyladenine. This process is a signal that identifies this
DNA as being the DNA of the host cell. The A,-phage that survived the transfer to
the E. coli plate with the RI plasmid did so because that phage's DNA was
methylated before the restriction enzymes had a chance to degrade it.

o
5'-pG-pG-3'
3'-Cp-Cp-5'

Hae III

5'-pC-pC-3'
3'-Gp-Gp-5'

Figure 10-75

Let's consider how to make a map of restriction sites on a duplex of DNA. In


order to separate large fragments of DNA an agarose gel is used whereas small
fragments are separated on an acrylamide gel. The smaller fragments (in both
gels) have an easier time of getting through the gel matrix and therefore migrate
faster. The larger fragments migrate slower. If A,-phage is digested with the
Copyright by The Berkeley Review

433

The Berkeley Review

Specializing in MCAT Preparation

BlOlOQy

Expression of Genetic Information

Genetic Engineering &? Cloning

restriction endonuclease Eco RI, then you should (in theory) get about 10
fragments. These fragments can be separated on the basis of size.
DNA is digested with
restriction enzyme(s)

Add fragments
at top
Nitrocellulose
\ paper

Paper towels

i>

solution

Electrophoresis

Restriction fragments
are separated by gel
electrophoresis

Nitrocellulose
paper

Figure 10-77

How would you know the order in-which these fragments went together? One
method to determine the order of the fragments would be by Southern blotting.
The DNA sample is digested with a particular restriction endonuclease and
separated into fragments using gel electrophoresis. The DNA is then denatured
(because later you will want to hybridize to the DNA) with alkali and transferred
from the gel onto a piece of nitrocellulose paper. Paper towels can be used to

absorb the buffer solution. The DNA binds to the nitrocellulose paper and gives
you a permanent record of the DNA fragments that you had on the gel. This
procedure is shown in Figure 10-77.

Suppose we were to cut our DNA sample with Eco RI and Hind III. What you
would like to know are the restriction sites for Eco RI and for Hind III. How

would the Hind III fragments relate to the Eco RI fragments? The chances are

the Hind III fragments and the Eco RI fragments will overlap. A possibility is
shown in Figure 10-78.
Cut

n
JL

EcoRI

Fragment

Eco RI
Fragment

|-.-.y-,:v

ft

Hind IH

<>

Fragment

Figure 10-78

How do we analyze this possibility? The DNA to be analyzed is cleaved with


each restriction enzyme in separate testtubes and each set of restriction fragments
(in each testtube) is separated by gel electrophoresis. We will have a gel for the
Eco RI restriction enzyme and a gel for the Hind III restriction enzyme. We
follow the procedure outlined in Figure 10-76 for making a nitrocellulose record
of the fragments from our two restriction enzymes. We can pick a fragment cut
by the Hind III enzyme and radioactively label it and use that radioactive
sequence as a probe. If we apply this probe to the nitrocellulose paper with the
fragments cut by Eco RI, then it will hybridize to those sequences of DNA in the
Copyright by The Berkeley Review

434

The Berkeley Review

Specializing in MCAT Preparation

Biology

Expression of Genetic Information

Genetic Engineering & Cloning

Eco RI fragments which are complementary. If the labeled Hind III fragment
overlaps two Eco RI fragments (as shown in Figure 10-78), then we should see

two dark bands on our developed x-ray film indicating which Eco RIfragments
contain the complementary sequences to the Hind III fragment. Repeating this
procedure with other fragments will eventually lead to a map of your DNA
sample.

How can the sequence of a piece of DNA be determined? Two techniques which
have been used successfully for sequencing DNA are the Maxam-Gilbert

method, involving specific chemical cleavages, and the Sanger dideoxy method,
involvinga controlledinterruption of DNA replication. Since the Maxam-Gilbert
method is not used that much anymore, we will turn our attention to the Sanger
dideoxy method.
Sanger Dideoxy Sequencing

The Sanger dideoxy technique makes use of the fact that DNA polymerase I can
be inhibited by a 2'/3'-dideoxynucleoside triphosphate (ddNTP) analog of one of
the four bases A, T, G, or C. The example shown in Figure 10-79 is ddATP.

Figure 10-79

1. Select the DNA strand to be sequenced. Attach a short radioactively labeled

section of primer to the DNA strand being analyzed. The label of choice is 35S
instead of 32P because 35S has a longer half life than 32P. Also, 35S has such a
weak emission that it doesn't require shielding. The 35S atom replaces theoxygen
atom that is double bonded to the a-phosphate of dNTP.
To each of four solutions add DNA polymerase I, the four deoxynucleotides
triphosphates (dNTP)-which are dATP, dTTP, dGTP, and dCTP~and pne. of the
ddNTPs (ddATP, ddTTP, ddGTP, or ddCTP) in a controlled ratio.

2. DNA polymerase I will take the dNTPs and add them to the primer. The
2',3'-dideoxy analog can be incorporated into the growing DNA chain by reaction
with its 51 triphosphate function. However, when the next deoxynucleotide
triphosphate is used in the reaction sequence it cannot be added to the 2',3'dideoxy analog because the analog does not contain a 3*-OH function to form a
phosphodiester bond. Termination of the DNA chain occurs specifically where
the nucleotide analog was incorporated. This yields DNA fragments of different
length containing the analog at the 3* end.

Copyright by The Berkeley Review

435

The Berkeley Review


Specializing in MCAT Preparation

Biology

Expression of Genetic Information

5'

Genetic Engineering fir Cloning

DNA backbone

3'

1 ,

_,

1
T

1
T

1
A

1
G

1
A

1
T

I -U
35S
j^l
a

Labeled
Primer

5'

3'

DNA backbone

1 ,

1
T

1
A

1
G

1
A

1
C

1
C

1
A

Hv

>

5'

"

..I

35

5'
+

H,

1 1 |

1 1

1 1 1 1 1 |

1 -kiis
5'

H,

rJ-

>

1 _L_35
5'

Figure 10-80

3. The labeled fragments shown in Figure 10-80 are thenseparated by sizeon a


polyacrylamide gel by electrophoresis. The pattern of fragments from the
autoradiogram shown in Figure 10-81 yields the DNA sequence that was
synthesized using theradioactively labeled complementary DNA primerstrandnot the original DNA strandyouwished to analyze. Since this "primer" strand is
complementary to the DNA sequence of the original strand, there should be no
problemin obtaining the basesequence of that originalstrand.
ddATP

ddTTP

ddCTP

ddGTP

DNA sequence of
complementary strand

Figure 10-81

Copyright by The Berkeley Review

436

The Berkeley Review


Specializing in MCAT Preparation

BlOlogy

Expression of Genetic Information

Genetic Engineering fir Cloning

Cloning

Recall that we mentioned that there were restriction enzymes that cut DNA at

specific sequences. Those sequences have a unique characteristic in that they


have twofold rotational symmetry. Some restriction enzymes leave a 51 overhang
while others leave a 3* overhang. There are also restriction enzymes that leave
blunt ends.

There are severalmethods that will allow you to makea map of restriction sites
and restriction fragments in your DNA. One method that we mentioned was

Southern blotting. If you wanted to determine the sequence of the DNA


fragment, then you could use the Sanger dideoxy technique (or the MaxamGilbert technique).

When the DNA that you are going to analyze is labeled, you want to make sure
that it is labeled on the end of your fragment of DNA. Why?One reason involves
sensitivity. In the case of the Maxam-Gilbert technique, labeled 32P at the end of

the DNA fragment might give you about 109 radioactive counts per minute.
Another reason to use end labeling involves the simplicityof the pattern that you
see on your gel.

With the use of restriction enzymes, one can use recombinant DNA technology to
create new combinations of genes which can then be cloned. For example, a piece
of foreign DNA cut with a specific restriction enzyme, such as Eco RI, can be
inserted into a DNA vector which has also been cut with Eco RI. RepUcation of
this recombinant DNA molecule takes place in the host cell. A suitable vector for
E.coliwould either be Arphage or a plasmid.
It turns out that there are a few problems involved in the cloning procedure. One
problem concerns the copy number. In the case of the first plasmid used, there
were only three per E. coli chromosome. Another problem is the size of the
plasmid. The plasmid that was first used in a cloning experiment was about 100
kb, which is a lot of DNA to have to work with.

There is also a problem with reclosure of the DNA. For example, if you cut your
DNA with Eco RI, then that same piece of DNA can close right back up again
without ever taking in the foreign piece of DNA that you want to clone. One way
to get around this problem is to add an excess of the foreign DNA so that you get
more collisions between the foreign DNA that you want to insert and the
plasmid.
Another way to get around this reclosure problem is to use phosphatase on the
plasmid DNA. If you want to join a nick between two nucleotide residues in a
segment of double stranded DNA, then the enzyme DNA ligase is used. The

energy source for this procedure comes from either ATP or NAD, depending
on the organism in use. (In bacterial cells the energy source is NAD while in
animal cells the energy source is ATP. NAD is an abbreviation for nicotinamide
adenine dinucleotide.)

In Figure 10-82 we see a segment of double stranded DNA with a nick. Note the
free 3' hydroxyl and the free 5' phosphate at the nick site. DNA ligase will
eventually form a phosphodiester bond at that position in order to seal the nick.

The 5' phosphate on the DNA duplex is absolutely required in order for DNA
ligase to work. If we were to treat the DNA segment shown in Figure 10-82 with
Copyright by The Berkeley Review

437

The Berkeley Review


Specializing in MCAT Preparation

Biology

Expression ofGenetic Information

Genetic Engineering fir Cloning

phosphatase and remove that 5' phosphate residue, we would not be able to close
the nick with DNA ligase.

Figure 10-82

Suppose we have 5' phosphates hanging out at the ends of our cleaved DNA
plasmid. If we remove those 51 phosphates with phosphatase, then we will not be
able to join these ends of the plasmid. This is shown in Figure 10-83.

y Phosphatase
OH I
>

A restriction enzyme
cuts die duplex DNA

The ends will

not rejoin

Figure 10-83

If a foreign piece of DNA that has 5' phosphates at two of its ends is now inserted
into our plasmid (from Figure 10-83), then only two strands of the DNA can be
joined. The other two strands of DNA will be left with nicks as shown in Figure
10-84. However, there will be some integrity to this plasmid.

3'HO ,^^^N\ Nick

5'P

yho \

P51

Foreign DNA to be inserted


DNA without 5'

DNA with two nicks

phosphates
Figure 10-84

Suppose we have cut our plasmid with the restriction endonuclease Eco RI and
we would like to insert a piece of foreign DNA that has also been cut with Eco
Copyright by The Berkeley Review

438

The Berkeley Review


Specializing in MCAT Preparation

BlOlOgy

Expression ofGenetic Information

Genetic Engineering fir Cloning

RI. One way this could be done is shown in Figure 10-85. Note the arrow above

the sequence 5'-TAC-3' in the fragment of foreign DNA. Let this represent a
direction for the insertion of the foreign piece of DNA. This direction is "in

phase" with the direction of the arrow for the plasmid. The "B" in our example
simply represents any base.

5' B-G

A-^A-T-T-C-B

DNA from plasmid


cut with EcoRI

V B-C-T-T-A-/

G-B

Foreign DNA
5'A-A-T-T-C-T-A-C-T-G-G-G

cut with EcoRI

V G-A-T-G-A-C-C-C-T-T-A-/

Annealing fragments with

\f

DNA ligase

<=^
5 B-G-^-A-T-T-C-T-A-C-T-G-G-G-^A-A-T-T-C-B

r3' B-C-T-T-A-AG-A-T-G-A-C-C-C-T-T-A-AG-B
P
llj
IB
J'
Figure 10-85

Our foreignpiece of DNA could also be inserted into our plasmid in the opposite
direction as shown in Figure 10-86. The two arrows are "out of phase." This is an
important aspect of cloning because if you are inserting a gene that you want to
clone into a plasmid, then in order for that gene to be properly expressed it must
go into the plasmid in the right orientation. Why? Because if the foreign DNA
goes into the plasmid in the wrong orientation, then when RNA polymerase
transcribes it, the protein that will eventually be produced (if it is produced at all)
most likely will be non-functional.

5'

B -G -A-A-T -T -C-C-C -A-G-T -A-G -A-A-T-T-C-B

3' B-C-T-T-A-A-G-G-G-T-C-A-T-C-T-T-A-A-G-B

Figure 10-86

Let's consider the plasmid pBR322, which has a high copy number of about 30
per chromosome. This plasmid has an ampicillin resistance gene (Amp1) and a
tetracycline resistance gene (Tetr). Both of these genes have promoters and the
direction of transcription in each case is indicated by the arrows. Within the
tetracycline gene is a restriction site for the restriction endonuclease Bam HI. This
is shown in Figure 10-87.

Copyright by The Berkeley Review

439

The Berkeley Review


Specializing in MCAT Preparation

Biology

Expression of Genetic Information

Genetic Engineering fir Cloning

Promoter

Promoter

Ampr

Figure 10-87

If we open the plasmid with the restriction endonuclease Bam HI, then we can

insert a foreign DNA fragment in two possible orientations. If this fragment


contains a gene without a promoter, then it is important as to which orientation

the foreign DNA is inserted. If the gene is inserted in the direction givenby the
arrow for the gene, thenthetetracycline promoter will not work with that gene.
This is shown in Figure 10-88a. However, if the gene is oriented in the same
direction as transcription, then the tetracycline promoter will work with the
gene. This is shown in Figure 10-88b.

(a)

(b)

Figure 10-88

How would you know which way the foreign piece of DNA has inserted itself
into the plasmid? Suppose we have an Eco RI site in our plasmid as shown in
Figure 10-89. Also suppose we have an Ssp site in our piece of foreign DNA. If
our piece of foreign DNA inserts as shown in Figure 10-89a, and we cut at the
Eco RI and Ssp sites, then we will get two fragmentsone large and one of
moderate length.
Copyright by The Berkeley Review

440

The Berkeley Review


Specializing in NCAT Preparation

BlOlOgy

Expression of Genetic Information

(a)

Genetic Engineering fir Cloning

(b)

Figure 10-89

However, if our piece of foreign DNA inserts in the opposite direction as shown

in Figure 10-89b, andwe cutat the Eco RI and the Ssp sites, then we will geta
large fragment and a small fragment. If you take your clones and do a restriction

digest, youwill seetwodifferent patterns. You canjustsave one sample ofeach


until you are sure which one you want.

A plasmid carrying genes for drug resistance is useful because if you were to
insert a piece of foreign DNA into one of those genes that codes for a drug
resistance, you would lose theactivity ofthat particular gene. Ifa piece offoreign
DNA has inserted into, say, the tetracycline resistance gene of pBR322, then you
would geta host cell that has tetracycline sensitivity. If thepiece offoreign DNA
did not properly insert into the tetracycline gene, then the host cell would still be
resistant to tetracycline. This would tell you that your insertion reactionfailed.

To remove the problem of orientation of the insertion fragment, two restriction


endonucleases were used that gave different cleaved ends. For example, in
pBR322 thereis an Eco RI site and a Hind III sitenot toofar away. If you cut the
pBR322 plasmidwith thesetwo enzymes and purifythe large fragment, you will
have two differenttypes of ends. Now, if you have a piece of foreign DNAthat is
cut with Eco RI and Hind III, then this insertion fragment will insert into the
pBR322 plasmid in only one orientation.

The pBR322 plasmid also has a Pvu II (Proteus vulgaris) site which gives blunt
ends. These blunt ends can anneal with the blunt ends produced by any other
restriction enzyme that gives that type of cut.
You could also join together two half restriction sites that have the same cohesive
ends but different recognitions. Two examples are Mia I and Bss HII. The
sequences that these endonucleases recognize are shown in Figure 10-90. This is

S'-A-C-G-C-G-T-S'
3'-T-G-C-G-C-A-5*

ft
Mia I

S'-G-C-G-C-G-C-S'
3'-C-G-C-G-C-G-5'
BssHU
Figure 10-90

good for insertion of the foreign DNA but you cannot remove that DNA once it
has been inserted (because you lose the restriction site).
There are certain types of plasmids that have been engineered at UC Davis called
the pUC plasmids. These plasmids are about 2 kb and have a very high copy
number, about 90 per chromosome. These plasmids have a gene for ampicillin
Copyright by The Berkeley Review

441

The Berkeley Review

Specializing in MCAT Preparation

Biology

Expression of Genetic Information

Genetic Engineering fir Cloning

resistance and in one small region, called a polylinker, there are about 17
restriction sites.

Another common vector is lambda (A) phage. This phage infects the bacterium E.
coli and injects its linear DNA into the host. The viral DNA soon becomes circular
and is quickly presented with two choicesto become part of the host (the
lysogenic pathway) or to produce more progeny and destroy the host in the
process (the lytic pathway). It turns out that the genome of this phage is about 48
kb, and not all of it is essential for the phage's life cycle. What this means is that
portions of the phage's genome can be removed and foreign DNA inserted. Once
the bacterium has been infected, the fragments of foreign DNA can be amplified
many fold, thus giving rise to a genomic library. This library is then analyzed to
find the gene of interest. A particular gene of interest can be located by using a
radioactively labeled probe.
One useful application of cloning is the production of proinsulin, a precursor to
insulin. Within a given plasmid is a region of inserted DNA that is
complementary (called cDNA) to the mRNA for pancreatic proinsulin. In order
to make this cDNA from the mRNA, the enzyme reverse transcriptase was
utilized. In order to ensure maximum transcription of the cDNA, it is inserted
into the plasmid in the correct reading frame and next to a strong promoter.
In the late 1970s the complete sequence was determined for the filamentous
phage M13. This 6.4 kb phage has nine known genes. The area between genes
VII and IX is capable of coding for a small peptide. Mutants were made for this
region. A mismatched oligonucleotide was made in which a C residue was made
to be opposite a T residue. This is shown in Figure 10-91. When this region is
transcribed there will be a 5'-UAG-3' stop codon in the middle of the gene. This
inactivates the gene (which is necessary for growth of the phage), thus causing a
mutation. However, the mutation rate was only about 1% because within the E.
coli bacterium is a mismatch repair enzyme that is constantly checking the DNA
and correcting errors in the base pairing.

3'-T-A-G-T-G-T-A-T-T-C-T-T-T-C-G-C-C-5'

DNA

3'

5'

3,-G-A-U-5'

mRNA

Figure 10-91

Copyright by The Berkeley Review

442

The Berkeley Review


Specializing in MCAT Preparation

Expression of Genetic
Information

15 Passages
100 Questions

Passage Titles
I.
II.

Southern Blotting
Restriction Endonucleases

III.

Ribozymes and HIV

IV.

The Genetic Code

V.

Polymerase Chain Reaction (PCR)

VI.
VII.
VIII.
IX.
X.
XI.
XII.
XIII.

Lactose Operon
Enhancers and Gene Expression
Replication Block and the Cell Cycle
Northern and Southern Blotting
Restriction Enzymes
Arabinose Operon
Gene Therapy Strategies
Tryptophan Operon

XIV.
XV.

Translation
Cancerous Gene

Specializing in MCAT Preparation

Questions
1 -5
6- 11
12- 17
18-23
24-29
30 - 36
37 - 43
44 - 50
51 -58
59-65
66-72
73-79
80-87
88-94
95- 100

Suggestions
Thepassages that follow are designed to get you to thinkin a conceptual mannerabout the processes
of molecular biology at the organismal level. If you alreadyhave a solid foundation in molecular biology,
many of thequestions youreadherewillseem tobe very straight forward and easyto answer. Butifyou
are new to the subject or if you have not had a pleasant experience with molecular biology in the past,
someof them might appear to come from the void that spreadsout beyond the Oort field at the edges of
our solar system.

Picka few passage topicsat random. For theseinitialfew passages, do not worry about the time.Just
focus on what is expected of you. First, read the passage. Second, look at any diagrams, charts, or graphs
in it. Third, read each question and the accompanying answers carefully. Fourth, answer the questions
the best you can. Check the solutions and see how you did. Whether you got the answers right or wrong,
it is important to read the explanations and see if you understand (and agree with) what is being
explained. Keep a record of your results.

After you feel comfortable with the format of those initial few passages, pick another block of
passages and try to do them in one sitting. Be aware that timeis goingto become important. On average,
you have about 1 minute and 15 seconds to complete a question. Be creative in how you approach this
next group. If you feel comfortable with the outline presented above, fine. If not, then try different
approaches to a passage. For example, you might feel well versed enough to read the questions first and
then try to answer some of them,without ever having read the passage. Maybe you can answer some of
the questions by just lookingat the diagrams,charts,or graphs that are presented in a particular passage.
Remember, there are many effective learning styles. You need to begin to develop a format that works
best for you. Keep a record of your results.

The last block of passages might contain at least a few topics that are unfamiliar even to those who
know a good deal about molecular biology. Find a place where the level of distraction is at a minimum.
Get out your watch and time yourself on these passages, either individually or as a group. It is important
to have a feel for time, and an awareness of how much is passing as you try to answer each question.

Never let a question get you flustered. If you cannot figure out what the answer is from information
given to you in the passage, or from your own knowledge base, dump it and move on to the next
question. As you do this, make a note of that pesky question and come back to it when you have more
time. When you are finished, check your answers and make sure you understand the solutions. Be
inquisitive. If you do not know the answer to something, look it up. The solution tends to stay with you
longer that way. (For example, what is the Oort field, anyway?)
The estimated score conversions for 100 questions are shown below. At best, these are rough
approximations and should be used only to give one a feel for which ballpark they are sitting in.
Section X

Estimated Score Conversions


Scaled Score

>13

Raw Score

80-100

11-12

70-79

9-10

60-69

7-8

50-59

5-6

40-49

<4

0-39

Biology

Southern Blotting

Passage I (Questions 1-5)

Passage I

Gel electrophoresis is commonly used to separate


molecules and molecular fragments. What does the

From a mixture of DNA sequences, a particular

separation of DNA into fragments depend on when a

fragment may be isolated with the Southern transfer

Southern blot is done?

technique, also known as Southern blotting. The basics of


this technique are:

1) DNA is fragmented into lengths that are appropriate


for the agarose gel. This may be accomplished by
sonication, mechanical shearing, or enzymatic action.
Conventionally, about 100,000 base pairs (bp) is the
upper limit on size for agarose gel electrophoresis of

A.

Charge

B.

Size

C.
D.

Radioactivity
Linking number

DNA.

2) The DNA on the gel is denatured with NaOH. A

4.

If you were probing for the DNA fragment:

sheet of nitrocellulose paper is placed over the gel,


covered with paper towels, and pressed down. This

5,CGATTACCCG3'

blots liquid and any single-stranded DNA from the


gel.

which of the following probes would you use?


A.
B.
C.
D.

3) The nitrocellulose paper is dried under vacuum with


heat, permanently fixing the adhered DNA.

4) A DNA probe, labeled either radioactively or


enzymatically, that is complementary to the sequence
of interest is applied to the nitrocellulose paper.
5)

Heat is applied over several hours to allow


renaturation. Then the paper is washed to remove
only unbound probe.

5.

Huntington's chorea (HC) is a fatal, degenerative


brain disease that is transmitted in an autosomal
dominant fashion. In the results shown below for a

6) The paper is read either by autoradiography on x-ray


film or by quantifying the enzymatic reaction.

gel electrophoresis done totest for a DNA fragment


containing a region common to people who have
HC, assume that A and B are the parents, and C and
D are the offspring. Which of the following

Southern blotting is particularly useful for identifying


the presence of a specific DNA fragment in a large
mixture of fragments. It can be used to identify carriers of
certain inherited diseases, for example.

1.

S'GCTAATGGGCS*
3'CGGGTAATCG5'
3'GCTAATGGGC5'
S'CGATTACCCGS'

statements is TRUE?

Offspring

Parents

Which types of enzymes are commonly used to


fragment DNA for Southern blotting?
A.

Restriction enzymes

B.

Proteases

C.
D.

Nucleotide synthases
DNA gyrases
A.

If Fragment II is the HC fragment, then both


children are affected.

2.

When DNA is denatured by NaOH in Step 2, what


structural change takes place?

B.
C.

Both parents carry the HC Fragment, V.


If Fragment III is the HC fragment, then both

A.

D.

The children show identical patterns of

children are affected.

The phosphates of DNA become positively


charged.

B.
C.
D.

inheritance.

DNA strands join to form a quad structure.


The base-pairing of DNA is altered.
DNA is separated into single strands.

Copyright by The Berkeley Review

445

The Berkeley Review

Specializing in MCAT Preparation

Biology

Passage n

Restriction Endonucleases

Restriction endonucleases can cut DNA into fragments


of varying length, which can then be separated from one
another by gel electrophoresis and identified using
autoradiography. Larger fragments are located near the
top of the gel, while smaller fragments are located near

Passage II (Questions 6-11)


When viral DNA enters a bacterial cell, a defense

mechanism in the bacterial cell degrades the DNA of the

virus. The activity of the viral DNA is restricted from


operating within the bacterial cell by the action of

the bottom.

bacterial enzymes called restriction endonucleases. These


restriction enzymes recognize a particular sequence of

If a single restriction endonuclease is used to cut a


sample of DNA, the procedure is called a single digest. A
double digest involves the use of two different restriction
endonucleases. DNA fragments are described in terms of
length. One kilobase (kb) is equal to 1000 bases (or base
pairs). A restriction map can be established by comparing
the lengths of the DNA fragments from different digests.

bases in double-stranded DNA and then cleave the DNA

at specific sites within those sequences.

Recognition sequences are written in the 5' - 3'


direction and, by convention, only one DNA strand is
given. A vertical arrow that represents the cleavage site is
placed within the written DNA sequence to indicate
where the restriction endonuclease cuts the DNA. For

example, the DNA sequence AiAGCTT is an


6.

abbreviation for:

Which of the following restriction endonucleases


would produce 5' sticky ends?

Cut

5'-AAGCTT-3'
^-TTCGAA-S'

I.

Pstl

II.

Sail

III.

Tag I

A.

I only
II only
III only
II and HI only

Cut

B.

The restriction endonuclease Hindlll recognizes this

C.

sequence and cleaves the DNA at the cut sites. Once the
two phosphodiester bonds holding the DNA backbone
together are cleaved, the hydrogen bonds between
complementary bases break apart. If the cut sites are
staggered, then complementary sticky ends are produced.

D.

If the cut sites are not staggered, then blunt ends are
produced. Most restriction endonucleases recognize DNA
sequences that are palindromic (the sequence is the same
if read forwards or backwards). Examples of some

The plasmid pBR322 is a circular duplex of DNA


that can replicate autonomously in the bacterium
Escherichia coli. This plasmid contains 4.3 kb of
DNA. In a random sequence of DNA, how many
cuts would be expected, if the restriction enzyme

restriction endonucleases and their recognition sequences

Alu I is used?

7.

are shown in Table 1.

Recognition
Sequence

Enzyme

Recognition
Sequence

Enzyme

AGlCT

Alul

GAGCTlC

Sac I

GlGATCC

Bam HI

GiTCGAC

Sail

AlGATCT

Bgl II

CCCiGGG

Smal

GlAATTC

EcoRI

TlCGA

Taq I

CTGCAiG

Pstl

CCCiGGG

Xmal

8.

A.
B.

8
11

C.
D.

14
17

A polylinker is a segment of DNA that contains


several restriction endonuclease sites. Based on the

information in Table 1, how many unique restriction


endonucleases can cut the following segment of
duplex DNA?
5'-CGGATCCCGGGTCGACG-3'
S'-GCCTAGGGCCCAGCTGC-S'

Table 1. Restriction Endonucleases

Bacterial DNA is protected from the cell's own


restriction enzymes by the action of modification
endonucleases. These enzymes catalyze the methylation
of either an adenine or a cytosine base within a particular
recognition sequence.
Copyright by The Berkeley Review

A.

B.

3
4
5

C.
D.

446

The Berkeley Review


Specializing in MCAT Preparation

Biology
9.

Restriction Endonucleases

A segment of bacterial DNA is cut by the restriction


endonuclease Bam HI. Two complementary sticky
ends are formed. These sticky ends are able to
establish hydrogen bonds at the sticky ends formed
by which of the following restriction endonucleases?
A.

Hind III

B.

Bgl ll

C.
D.

Pstl
Sac I

11.

Passage n

Various combinations of restriction endonucleases

are used to digest a length of bacterial DNA. The

fragments from these single and double digests are


analyzed by agarose gel electrophoresis and
compared to a known standard (S), as shown below:
Eco RI

Pstl

Eco RI

BglII EcoRI Pstl BglII BglII Pstl

nn

n_ji_n

7kb

6kb

5kb

10. Two linear DNA molecules are digested with the


same restriction enzyme (EcoRI) in different

4kb

experiments, and their fragments are separated by


size using agarose gel electrophoresis(see below):

3kb

DNA
Lane 2

2kb

lkb

Based on the DNA fragment patterns in the gel


shown above, which of the following restriction
maps BEST represents the length of bacterial DNA?
A.
Pstl

Bgl II

DNA

Eco RI

1
1 '

Pst I

EcoRI

Bgl II

B.
Pstl

Based on the separation of fragments in the gel, the


two DNA molecules seem to be similar. The only
difference is that Fragment D in Lane 1 does not
show up at the same position in Lane 2. Instead,
two new fragments, X and Y, have appeared. The

Pstl

DNA

1 '

C.

presence of Fragments X and Y is due to the:

Bgl II
Pst I

A.

DNA

addition of DNA that contains an Eco RI site

I 3 '

to Fragment D.

B.
C.

deletion of DNA from Fragment D.


addition of DNA to Fragment D.

D.

deletion of DNA from Fragment D, resulting

Pst I

3 I,

D.
EcoRI

Pstl

in the subsequent formation of a new Eco RI

Bgl II

Pstl

DNA

site.

Copyright by The Berkeley Review

Eco RI

447

The Berkeley Review

Specializing in NCAT Preparation

Biology

Ribozymes and HIV


13.

Passage III (Questions 12-17)

Passage 01

HIV is notorious for mutating in response to


treatment. For example, giving a particular

Although it is well known that enzymes are catalysts


(molecules having the ability to increase reaction rates), it

experimental drug led to a two-week decline in the


number of HIV particles, followed by a dramatic

is less well known that RNA molecules can have catalytic

mutation resulting in a 95% new population of HIV


particles. How could the ribozyme approach BEST
fight the mutation problem?

activity as well. The term ribozyme refers to an RNA


molecule with catalytic activity. Thomas Cech of the
University of Colorado and Sidney Altman of Yale
University won the 1989 Nobel Prize in chemistry for

A.

their work that indicated RNA was more than a passive


carrier of genetic information. RNA can act to catalyze

B.

Increase the concentration of the ribozyme by


a more efficient transfection process
Follow one ribozyme with another in two
weeks

itself or other molecules of RNA.

C.

Ribozymes were probably the first catalytic agents


formed during the evolution of life. Later, RNAs began to
code for protein synthesis, and proteins are superior
enzymes. Proteins became the dominant form of catalysts.

D.

Create a specific ribozyme for the strain of


HIV the patient has
Link multiple ribozymes to attack HIV mRNA
at several locations at once

Cech's work in 1982 indicated that some of the RNA

found in Tetrahymena is autocatalytic. It has the ability to


splice out introns and edit its own mRNA sequence. One
year later, Altman discovered some RNA molecules that
14.

could cleave other molecules as well.

Ribozymes are currently being studied to combat


human immunodeficiency virus (HIV). Researchers

Why do Hampel and Wong-Staal target a highly


conserved region of the HIV mRNA that is common
among many different strains of HIV?
A.

Arnold Hampel and Flossie Wong-Staal are modifying a


tobacco virus ribozyme to recognize and cleave a highly
conserved region common among HIV strains. A
specially designed ribozyme can chop up the HIV mRNA
in test-tube experiments. In culture, this modified
ribozyme reduced HIV transcripts by 10,000-fold.

B.

C.

Cech and Bruce Sullenger are focusing on packaging


the ribozyme gene so it co-locates with the virus into the
virus packaging center of the cell. This means that the
ribozyme against HIV will be present inside the "HIV
factory" and will destroy copies of the viral mRNA. One
clinical approach is to remove T cells from the patient,
infect them with a ribozyme that cleaves HIV mRNA, and
return it to the patient. Although this scheme will not be

D.

The highly conserved regions are known for


their wide range of variability, and this
approach targets more mutants.
Usually, highly conserved regions code for
well-designed functions critical for survival,
and this approach targets more mutants.
The HIV particle resists attack by ribozymes
except at these highly conserved sites.
The highly conserved regions attract
ribozymes for more efficient cleavage of
mRNA.

tested in vivo for several years, these are the big plans
some researchers are exploring right now.
15.

What is the immediate effect (prior to mutational


changes) of decreasing with a ribozyme the number
of available mRNAs for HIV in a T cell?

12. If the right ribozymes attacked an animal cell, which


A.

structures could be damaged?


I.

because each mRNA becomes more efficient


at translation.

Ribosome

B.

II. Transcript copy of DNA


III. Amino acid carrier for protein translation

C.
A.
B.
C.

I only
I and II only
II and III only

D.

I, II, and III

Copyright by The Berkeley Review

The number of virus particles does not change,

D.

448

The number of virus particles decreases, due to


fewer copies of the transcript.
The number of virus particles increases, due to
more copies of the transcript.
The number of virus particles does not change,
due to competing HIV mRNAs.

The Berkeley Review


Specializing in NCAT Preparation

Biology
16.

Ribozymes and HIV

Passage III

In the course of evolutionary history, RNA-based


catalysts were replaced by protein-based catalysts.
Why?

I.
II.

A wider variety of subunits provides greater


flexibility in the structure and function of
protein-based catalysts.
Enzymes outcompeted ribozymes and chopped
them up.

III. Enzymes are more advantageous, because they


change the equilibria of reactions.

17.

A.
B.
C.

I only
I and II only
II and IH only

D.

I, II, and III

Which of the following statements would be TRUE


about catalysts?

I.

Catalysts remain unchanged after completing a


reaction.

II. Catalysts increase the rate of a reaction.


III. The body does not synthesize catalysts.
A.
B.

I only
I and II only

C.

II and III only

D.

I, II, and III

Copyright by The Berkeley Review

449

The Berkeley Review


Specializing in NCAT Preparation

Biology

Passage IV

The Genetic Code

18. The difference in the number of possible kinds of


amino acids between a single-base code and a three-

Passage IV (Questions 18-23)

In a pioneering biological experiment, Marshall


Nirenberg showed how to produce polyphenylalanine
from the addition of polyuridylate to a cell-free protein
synthesizing system. To begin, E. coli bacteria were
broken by agitation and subsequently centrifuged. The
resulting supernatant contained DNA, tRNA, mRNA,

base code is:


A.
B.
C.
D.

4
16
60
64

ribosomes, enzymes, and other cytoplasmic components.

Protein synthesis could be obtained by the addition of


GTP, ATP, and amino acids. The mixture was incubated

at physiological temperature for some time, after which

19. The following base sequence is given:

trichloroacetic acid was added. The protein containing

precipitate was washed, and the number of14C counts per

minute(cpm) was measured.

AT C G G TAT A

An essential component of this system is the addition


of deoxyribonuclease (DNAse). The mRNA present when

If the genetic code is overlapping, a mutation in the


basecytosine (C) may alter how manyamino acids?

DNAse is added is short-lived, and protein synthesis soon

ceases. But synthesis resumes with the introduction of


A.

additional mRNA, making possible a protein-synthesizing


systemthat is entirely cell-free.

The synthetic polyribonucleotide that is added


(polyuridylate) is created using polynucleotide
phosphorylase, an enzyme that catalyzes the following

B.

1
2

C.
D.

3
4

reaction:

20. The following base sequence is given:

(RNA)n + Ribonucleoside diphosphate = (RNA)n+i + Pi

12

This enzyme does not use a template, and the relative


composition of ribonucleotides present determines the
RNA to be synthesized. A polynucleotide consisting of

9 10 1112

ACTGTTACATTG

Insertion of the base thymine (T) between the 7th


and 8th base may affect how many amino acids?

two different bases is created by using two types of

ribonucleoside diphosphates in the presence of

polynucleotide phosphorylase. Table 1 shows the amino


acid incorporation created by a random copolymer of 76%
uracil (radiolabeled) and 24% guanine.

A.

B.
C.
D.

1
2
3

Relative amount

Amino Acid

incorporated

Phenylalanine

100

Valine

37

Leucine

36

Tryptophan
Glycine

14

21.

be used?

A.
B.
C.
D.

12

Tfeble 1. Amino acid incorporationresulting


from a random copolymer of U* (76%) and
G (24%). (Note: *Radiolabeled)

Copyright by The Berkeley Review

To obtain measurable results in this experiment,


which of the following cellular components MUST

450

Radiolabeled mRNA
Radiolabeled amino acids
Radiolabeled ATP
Radiolabled ribosomes

The Berkeley Review

Specializing in NCAT Preparation

Biology
22.

The Genetic Code

Which of the following graphs BEST represents


protein synthesis as measured throughout the
experiment?
A.

B.
.s
<u

/
/

S"H
Q. N
O

<U

x:

"

E
<

S"S
O. N

o'O

*- JS

2 &

<C
Time

C.

Time

D.

Time

23.

Passage IV

Time

Based on the information in Table 1, the triplet that


codes for glycine MOST likely contains:
A.
B.
C.
D.

more guanine than uracil.


more uracil than guanine.
equal amounts of guanine and uracil.
all guanine and no uracil.

Copyright by The Berkeley Review

451

The Berkeley Review


Specializing in NCAT Preparation

Biology

Polymerase Chain Reaction (PCR)

A DNA polymerase called Taq, from the thermophilic

Passage V (Questions 24-29)

bacterium Thermus aquaticus, is added to the mixture.

The polymerase extends the primers and completes the


first cycle of DNA amplification. The result is two new

The polymerase chain reaction is a process that


amplifies DNA within a certain region of a chromosome.
The first step (Figure 1) is to denature the targeted DNA
by heating it. Next, an excess of two DNA primers, about
15 to 20 oligonucleotides in length, is added to the

DNA double helices where there was once just one.

Each cycle, which takes approximately five minutes,


doubles the number of DNA molecules present. The cycle

denatured DNA. For proper hybridization, the two

is usually repeated 20 to 40 more times. The DNA


synthesized in subsequent cycles extends to just the ends

oligonucleotide primers must be complementary to the


opposite strands of DNA.

of the primers.

One application of PCR involves the screening of


particular steps in cloning. For example, one must screen

dsDNA

\/
mu

Passage V

Targetedsequence

\y

transformants after the step of inserting a DNA fragment

MINIM

I I I I I II I I I I I IT

into a plasmid. The reason is to check to see whether the


transformants contain the inserted fragment and to verify

Denature

Cycle 1
V
1 II I M I II II II II I II

that the clone is suitable for further work. In this

Anneal primers

screening process, transformant colonies are picked and


placed into tubes used for PCR. The cells are lysed, and
primers (indicated by / and 2 in Figure 2) are used that

1111111111X11?

flank the site into which the fragment is to be cloned. The

^uM^^rTTT^^n|^U^^^^^^^^l^IH^^

product of the PCR amplification is run on a gel.

Primer extension

Cloning site

\7

Inserted DNA

Denature

Cycle 2

Anneal primers
\7

T^^^^^ffll^^ffl^ffl^^HTfflf^^^W

o
Products of PCR

" II ITTT=TT I I II I I I | | | | | | | I I LU.

Figure 2

I I I Ml M I I I I I M I I I I IJJ.UMMI Ml'

^u^mj^^M^^^^^j^^^^^M^^^j,
24. The MOST likely reason that the DNA polymerase
from Thermus aquaticus is used preferentially over
the polymerase from E. coli is that the:

Primer extension
\7

A.

polymerase from Thermus aquaticus has a


much faster rate of polymerization.

" ii n itti n 11 ii i mi ii i n |i iq;

B.

heat used to denature the DNA would destroy

the polymerase from E. coli.

C.

polymerase from Thermus aquaticus is known


to be much easier to isolate.

D.

polymerase from E. coli is not functional in


cell-free systems.

Figure 1

Copyright by The Berkeley Review

452

The Berkeley Review

Specializing in MCAT Preparation

Biology
25.

Polymerase Chain Reaction (PCR)

A brief heat treatment is used to separate the doublestranded DNA into single-stranded DNA. The result

28. After seven cycles of the PCR (as depicted in Figure


1), one would expect to see how many strands of

of this heat treatment should result in:

A.

B.
C.
D.

increased absorbance of light at


effect known as hyperchromism.
increased absorbance of light at
effect known as hyperannealing.
decreased absorbance of light at
effect known as hypochromism.
decreased absorbance of light at
effect known as hypoannealing.

DNA?

260 nm, an

A.

260 nm, an

C.

128

C.
D.

256
612

260 nm, an

Separation of the PCR products in Figure 2 by gel


filtration chromatography would be MOST likely to
result in the product of the:
A.
B.

64

B.

260 nm, an

29.

26.

Passage V

The primers used in the polymerase chain reaction:


A.
B.

are complementary to each other.


contain the same nucleotide sequence.

C.

should contain uracil.

D.

are designed using portions of the sequence


surrounding the region of DNA to be
amplified.

suitable clone emerging first, based on its size.


suitable clone emerging first, based on its
charge.
unsuitable clone emerging first, based on its
size.

D.

27.

unsuitable clone emerging first, based on its


charge.

Shown in the diagram below are four different DNA


melting curves for four different molecules of
duplex DNA:

Temperature (C)

Which of the four curves would be expected to have


an abundance of A-T base pairs?

A.

B.
C.
D.

II
Ill
IV

Copyright by The Berkeley Review

453

The Berkeley Review


Specializing in NCAT Preparation

Biology

Passage VI

Lactose Operon

Wild-type (normal) structural genes of the lac operon

Passage VI (Questions 30-36)

are written as Z+Y+A+. This designation indicates that


normal levels of the three proteins coded for by these

Lactose is synthesized in the mammary gland by

genes are synthesized. A mutation in any one of these


genes (e.g., Z"Y+A+) could lead to an inactive form of the

lactose synthase, an enzyme composed of two subunits,


one with catalytic activity (galactosyl transferase) and one
without catalytic activity (a-lactalbumin). In human

corresponding protein.

infants, this milk sugar is hydrolyzed by lactase, an

Independent mutations can also occur in the trans


acting (capable of activity on another chromosome)
regulatory gene and in the cis-acting (capable of activity
onlyon the samechromosome) operator gene. Regulatory

enzyme located in the epithelial cells of the small


intestine. In some adult humans, lactase synthesis is

inhibited, leading to a condition known as lactose


intolerance.

gene mutations are designated as I" and are called


especially if glucose, the preferred carbon source, is not

constitutive mutants. This type of mutation leads to an


increase in the synthesis of the lac operon proteins in the

available. Metabolism of lactose requires galactoside

absence of an inducer. In contrast, normal inducible

permease, a cotransporter of protons and lactose across


the bacterial plasma membrane, and p-galactosidase, an
enzyme that catalyzes the hydrolysis of lactose into

bacteria synthesize the proteins of the lac operon only in


the presence of lactose. A constitutive mutation in the

The bacterium E. coli can also utilize lactose,

operatorgene is designatedas Oc.

glucose and galactose.

The enzymes required for the catabolism of lactose in


E. coli are derived from genes organized into an operon.

The lactose (lac) operon contains three structural genes

designated as lac Z (p-galactosidase), lac Y (galactoside


permease), and lac A (thiogalactoside transacetylase). The
control region of the lac operon is adjacent to the lac Z
gene and contains a promoter (P) that binds RNA
polymerase, and an operator (O) that binds a repressor
protein transcribed from the (regulatory) lac I gene. A
genetic mapof thesesites is shown in Figure 1:
DNA

Protein <^>

lacZ

lacY

0-

Lactose intolerance can lead to all of the following


EXCEPT:
A.

diarrhea.

B.

CO2 production.

C.

epithelial cell uptake of water from the

D.

intestinal bacterial fermentation.

intestinal lumen.

lac A

30.

31.

Figure 1
If lactose is absent from the medium, the lac repressor

binds to the operator, and RNA polymerase is unable to


transcribe the structural genes. When lactose is present in

Which of the following transport systems is MOST


likely to be associated with galactoside permease?
A.

Active

B.
C.
D.

Uniport
Symport
Antiport

the medium, it enters the cell and is converted to the


inducer, allolactose. The inducer binds to the repressor

and reduces the repressor's affinity for the operator. RNA


polymerase can now initiate transcription.

32. The process of synthesizing mRNA from DNA


occurs with the use of a:

When glucose is in the medium, cAMPconcentrations


in the cell are low and transcription of the lac operon is
reduced by 50-fold. However, in the absence of glucose

A.
B.
C.
D.

the cAMP levels in the cell are high, allowing cAMP to

bind to the cAMP receptor protein (CRP). The CRPcAMP complex then binds upstream from the lac
promoter and increases the rate of transcription of the lac

RNA-directed DNA polymerase.


DNA-directed DNA polymerase.
RNA-directed RNA polymerase.
DNA-directed RNA polymerase.

structural genes by RNA polymerase.

Copyright by The BerkeleyReview

454

The Berkeley Review

Specializing in MCAT Preparation

Biology
33.

Lactose Operon

Which of the following arrangements is necessary


before transcription of the structural genes can be

36.

Passage VI

Wild-type donor bacteria can be crossed with mutant

recipient bacteria to produce partial diploid cells


called merozygotes. The genotype of one such

initiated?

crossing is shown below:


A.
B.

C.
D.

Lactose must bind to the repressor before RNA


polymerase can bind to the operator.
DNA polymerase must bind to the repressor
before the inducer can bind to the promoter.

n+Ocz+Y-ATO+Z-Y+A+

RNA polymerase must bind to the promoter,


and the inducer must bind to the repressor.
Allolactose must bind to the repressor, and the
inducer-repressor complex must bind the

Based on this genotype, all of the following genes


code for enzymes that are produced constitutively
EXCEPT:

promoter.

34.

A culture of E. coli is transferred from a growth


medium containing lactose to a growth medium
containing glucose. Which of the following will be

The repressor will become inactive, cAMP

B.

and P-galactosidase will be synthesized.


The repressor will become active, cAMP

LacZ
LacY

III.

Lac A

A.
B.

C.
D.

observed after the transfer is made?

A.

I.

II.

I only
II only
III only
II and III only

levels will increase, CAP will become active,

levels will decrease, CAP will become

inactive, and P-galactosidase will not be


C.

synthesized.
The repressor will become active, cAMP

D.

and P-galactosidase will not be synthesized.


The repressor will become inactive, cAMP

levels will increase, CAP will become active,

levels will decrease, CAP will become

inactive, and P-galactosidase will not be


synthesized.

35. The following haploid genome is found in E. coli:

Lac I"0+Z+Y+A+. Which of the following enzyme


levels BEST represents this genome, both in the
presence and in the absence of an inducer?
A.
lacZ

lacY

lac A

low

low

low

lacZ

lacY

lac A

high

low

high

lacZ

lacY

lac A

high

low

low

lacZ

lacY

lac A

high

high

high

B.

C.

D.

Copyright by The Berkeley Review

455

The Berkeley Review


Specializing in NCAT Preparation

Biology

Enhancers and Gene Expression

Passage VII

The effect of each mutation on transcription is tested


(Figure 3):

Passage VII (Questions 37-43)

Experiments have shown DNA sequences near the


RNA start site are required for gene transcription. These

sequences are known as promoters and usually contain


the consensus sequence TATA. Interestingly, sequences
further upstream known as enhancer elements arre

Leaky RBC

required to give the same level of transcription as a

>^

normal gene.

oo
Cloned plasmids

Ljf

containing
mutant enhancer

The activity of each enhancer and promoter can

Q.Q

generally be focused on a segment of DNA 100 to 200


nucleotide pairs long. The function of both regulatory
elements requires binding of specific gene-regulatory
proteins, which are often tissue-specific. In other words,
most regulatory elements function best in the particular
cell types that express the gene with which they are
normally associated. One example is the p-globin gene in

1 hour incubation

Production of CAT

(37 C)

activity is measured

chickens.

Figure 3

To identify gene-regulatory proteins that bind to the pglobin enhancer, one must first determine the exact

Results of the mutant enhancer activity (Figure 3) are


then correlated with data showing the location of protein
binding sites for proteins in the normal enhancer. These

nucleotide sequence necessary for function (Figure 1):


1 | | .:,
2| I

3 1 .,- '.
41- .
-:
SI
c

ca

->--,,

-'-.VV:
1
-: -".::|

.:.-.

-.--.

6 1 .- -,.:,- ---
7 1
,.,

:-.-

v:

>

Regions in enhancer DNA covered by proteins

1
1

Aii\
ii
II
II /
I
I / I iI /rII

*
24 |
25|
26|

binding sites have been determined by gel-mobility shift


assays and DNA footprinting. The results of the
correlation are shown in Figure 4:

- --.v.::.:-.,,^!

-.--,,-.<,!
-.-.;.^-...l,.-.t

:-.

27 1

NF1-

API- AP2-

like

like

like n

CACCC Eryfl Eryfl


factor

5 100

VV-.ll

108 Nucleotide pairs


50

Figure 1

15

A series of mutated versions of the enhancer sequence

can be joined (Figure 2) to a reporter gene whose product


is easily measured. This reporter gene is a bacterial
enzyme called chloramphenicol acetyl transferase (CAT):
Coding sequence

\ for CAT

Tlnnll

Mr

10
15
20
Mutant Number

25

Figure 4
These and other studies have led to the following

P-globin promoter

general conclusions regarding enhancers and promoter


elements:

Plasmid

1. Each regulatory element consists of a specific


nucleotide sequence, which binds a corresponding

^>

series of gene-regulatory proteins. Some of these

proteins are found only in specific tissues, while


some are found in all cell types.

2. Some gene-regulatory proteins activate transcription


when they bind, while others inhibit transcription.

Recombinant

Poly-A addition
signal

\P-globin enhancer
(mutant)

3. Enhancer and promoter elements seem to bind to


many of the same proteins.

Figure 2

Copyright by The Berkeley Review

456

The Berkeley Review


Specializing in NCAT Preparation

Biology
37.

Enhancers and Gene Expression


41.

The chicken P-globin enhancer lies downstream


from the P-globin transcription unit. It forms a
nuclease-hypersensitive site only in erythrocytes and
no other cells. The MOST likely explanation for this

A newly discovered regulatory element from a


vertebrate gene is analyzed, and it is found that
many of the proteins that bind to the region have
been previously found regulating other genes. This

statement is:

finding suggests:

A.

A.

the enhancer sequence is found only in


erythrocytes.

B.
C.

38.

nucleosome complexes.
red blood cells are nuclease-hypersensitive,

B.

the net effect of a regulatory element depends


on the combination of regulatory proteins

C.

a relatively small number of gene-regulatory


proteins control transcription in higher
eukaryotes.
promoters and enhancers often act
synergistically in promoting transcription.

bound.

in other cell types, gene regulatory proteins


bind only to promoters and not to enhancers.

D.

42. Which of the following molecular complexes is


MOST likely to bind to a gene-regulatory element?

Histone octamers serve as a protein core around


which double-stranded DNA can loop twice. The

histone proteins are MOST likely to have a high


proportion of which of the following amino acids?
A.

Valine and serine

B.

Glutamine and cysteine

C.

Lysine and arginine

D.

Histidine and isoleucine

promoters and enhancers affect transcription in


similar manners.

gene-regulatory proteins found only in


erythrocytes bind to the enhancer, displacing

because loss of their nucleus is inevitable.

D.

Passage VH

A.

Steroid hormone

B.

Steroid hormone-receptor complex

C.

Protein hormone

D.

Protein hormone-receptor complex

43. According to Figure 4, which gene-regulatory


proteins make the largest contribution to the
enhancer's stimulation of transcription?
39.

According to the protocol in the passage, how many


nucleotide pairs are changed in each enhancer
mutant?

40.

A.
B.

C.
D.

27
108

A.

AP2-like, NF1 -like, CACCC factor.

B.
C.
D.

Eryfl, CACCC factor, NFl-like.


API-like, AP2-like, Eryfl.
AP2-like, CACCC factor, Eryfl.

Figure 3 illustrates the testing of mutant enhancers


for their effects on transcription. In this experiment,
transcription is measured:

A.
B.

directly by quantifying levels of RNA.


directly by recording rates of nucleotide
incorporation into RNA.

C.

indirectly by monitoring levels of P-globin

D.

protein.
indirectly by monitoring CAT enzyme activity.

Copyright by The Berkeley Review

457

The Berkeley Review


Specializing in MCAT Preparation

Biology

Rereplication Block and the Cell Cycle

Passage VIII (Questions 44-50)

44.

In the normal S phase of a cell cycle, the genome must

be replicated exactly once and no more. However, in the


middle of a eukaryotic S phase, some parts of a
chromosome will not yet have started replication, while
other regions will be completed. This presents an
enormous problem during the middle and late stages of
the S phase. Some replication origins and their respective
DNA sequences already used and duplicated are
presumably identical to other replication origins not yet
used. Since each replication origin may be used only once
in each S phase, there must exist some form of record
keeping and regulation.

Passage VDI

When an S phase cell is fused with a GI phase cell,


DNA synthesis is induced in the GI phase nucleus.
This information suggests that MOST likely the
transition from:

A.
B.

C.
D.

S to GI is mediated by a diffusible activator of


DNA synthesis.
S to GI is mediated by a nondiffusible
activator of DNA synthesis.
GI to S phase is mediated by a diffusible
activator of DNA synthesis.
GI to S phase is mediated by a nondiffusible
activator of DNA synthesis.

Initiator proteins
45.

The S phase of a eukaryotic cell can BEST be


described as:

A.
B.
C.
D.

synchronous, with one origin of replication.


synchronous, with multiple origins of
replication.
asynchronous, with one origin of replication.
asynchronous, with multiple origins of
replication.

Initiator protein
(inactivated)

46.

G2

Fragments of bacterial DNA that replicate when


injected into a fertilized frog egg can be shown to be
affected by the rereplication block. This
information:

G2

A.

C.

indicates the mechanism providing for a


rereplication block requires a highly specific
origin of replication.
indicates the mechanism providing for a
rereplication block does not require a highly
specific origin of replication.
provides evidence for the activator-removal

D.

provides evidence against the activator-

\7

B.

GI

GI

model.

DNA can replicate once again

removal model.

Figure 1

Cell fusion experiments have provided an important


clue to the answer to this problem. When an S phase cell
is fused with a GI phase cell, DNA synthesis is induced
in the GI phase nucleus. In contrast, when the S phase

47.

cell is fused with a G2 phase cell (a cell that has

A.

stimulated to begin DNA replication by some

B.

stimulated to begin DNA replication by some

C.

diffusible molecule found in the nucleoplasm.


prevented from DNA replication by some

D.

prevented from DNA replication by some

nondiffusible molecule bound to its DNA.

completed the S phase), the G2 nucleus is not stimulated


to replicate DNA, while the S phase cell continues DNA
synthesis. One model which could explain this
rereplication block is based on tightly bound initiator
proteins, which are inactivated by the passage of a
replication fork. This model, known as the activator-

nondiffusible molecule bound to its DNA.

diffusible molecule found in the nucleoplasm.

removal model, is illustrated in Figure 1.

Copyright by The Berkeley Review

It can be inferred from information in the passage


that a G2 nucleus fused with a S phase nucleus is:

458

The Berkeley Review


Specializing in NCAT Preparation

Biology

Rereplication Block and the Cell Cycle

Passage VIII

48. According to the activator-removal model, the


addition of new initiator molecules occurs during:
A.
B.

GI.
S.

C.
D.

G2.
mitosis.

49. In several types of secretory cells found in fly


larvae, all the homologous chromosome copies
remain side by side, creating a giant polytene
chromosome. In the salivary glands of Drosophila
larvae, each of the four chromosomes has been

replicated through ten cycles without separation of


the daughter chromosomes. How many identical
strands of chromatin are lined up side by side?
A.
B.
C.
D.

128
256
512
1024

50. The initiator proteins used in the activator removal


model:

A.

are translated in the nucleoplasm and

B.

transported into the cytosol.


are translated in the cytosol and diffuse across

C.

contain a significant hydrophobic region of

D.

contain a nuclear import signal.

the nuclear membrane.


amino acids.

Copyright by The Berkeley Review

459

The Berkeley Review

Specializing in NCAT Preparation

Biology

Northern and Southern Blotting

Passage IX (Questions 51-58)

53.

Passage IX

Based on the information in the passage, detergents


are MOST likely used to inactivate:

To determine the nature of the defect in mutant mice

that produce abnormally low amounts of the protein


albumin, the following protocol can be followed: First,

A.
B.

proteases.
nucleases.

one collects tissue samples from defective and normal


mice. Using strong detergents, the cells are disrupted to
inactivate cellular molecules. After removing proteins by

C.

lipases.

D.

cellulases.

phenol extraction, one can precipitate the nucleic acids


with alcohol. To separatethe DNA from the RNA further,

54.

The RNA harvested from the normal mouse cells is

one can make use of their different solubilities in alcohol.

MOST likely used to:

A technique known as Northern blotting can be used


to analyze the albumin-encoding RNAs. Intact RNA

A.
B.

test the equipment.


establish an experimental control.

molecules from defective and normal liver cells are

C.

establish the size of the mutant RNA

D.

molecules.
establish the

fractionated by gel electrophoresis. A replica of the gel is


blotted onto a sheet of nitrocellulose paper. Then,

amount

of

mutant

RNA

molecules present.

radiolabeled DNA probes are incubated along with the

paperso that the RNA encoding albumin hybridizes with


the correct DNA probe. The sheet is then washed
thoroughly afterward, and the hybridized probe is
detected by autoradiography. Using this technique, one
can gather information about the size of the albumin RNA

55.

Washing the nitrocellulose paper after incubation


with the probes removes all:
A.
B.
C.
D.

in both the defective and normal mice.

By an analogous method, DNA instead of RNA can be


examined. This technique is known as Southern blotting.
DNA fragments are separated according to their size by
gel electrophoresis, and complementary probes are
identified by blotting and hybridization. By repeating this
procedurewith several differentrestriction endonucleases,
a restriction map of a particular portion of the genome can

56.

be constructed.

radiolabel before autoradiography.


radiolabel after autoradiography.
background radiolabel before autoradiography.
background radiolabel after autoradiography.

Shown below is a Northern blot indicating both


defective and normal RNAs encoding for albumin.
Band A corresponds to the wild-type RNA, while
band B corresponds to the defective RNA.

51. The protein albumin is MOST likely to be found in:


A.
B.
C.
D.

the cytosol of white blood cells.


cerebral spinal fluid.
blood plasma.
the cytosol of red blood cells.

These results indicate that the:

A.

defective protein is smaller, and the


differences can be explained by a mutation in

B.

defective protein is larger, and the differences


can be explained by a mutation in the defective

the defective cell's RNA.

52. The cells that produce albumin can be found in the:

cell's DNA.

A.
B.
C.

spleen.
gall bladder.
pancreas.

D.

liver.

C.

wild-type protein is smaller, and the


differences can be explained by a mutation in
the defective cell's RNA.

D.

wild-type protein is larger, and the differences


can be explained by a mutation in the defective
cell's DNA.

Copyright by The Berkeley Review

460

The Berkeley Review


Specializing in NCAT Preparation

Biology

Northern and Southern Blotting

57. In the following experiment, two different restriction

58.

endonucleases, ResEnd A and ResEnd B, are used to

create a restriction map of a lOkb DNA molecule:

10 kfi DNA Molecule

Passage IX

In sickle-cell anemia, the exact nucleotide change in


the mutant gene is known. DNA probes can be
constructed for both the normal and defective genes
and used in prenatal diagnosis. A child's mother is
homozygous dominant, while the child's father is
heterozygous for the sickle-cell defective gene.
What is the chance that both normal and defective

DNA probes will react with the child's genome?

2kbl

3kbl

5kb|

~~

A.
B.

25%
50%

C.
D.

75%
100%

3kb

7kb[
ResEnd B

2kb

8kb

Which of the following conclusions is consistent


with the data? [Note: The numbers indicated the

length of the DNA in kb, while the capital letters


indicate the restriction endonuclease cut sites.]

A.

rr^r

B.
B

C.
A

I
2

1
1

D.
B

K^l^l-

Copyright by The Berkeley Review

461

The Berkeley Review

Specializing in NCAT Preparation

Biology

Restriction Enzymes

60.

Passage X (Questions 59-65)

Passage X

Which of the following statements is TRUE of DNA


conservation?

About 99% of human DNA is identical between any

two individuals. The other 1% contains regions of

A.

hypervariability that differ dramatically from person to


person. By examining these differences through DNA
fragmentation, it is possible to produce a unique DNA

B.

identification for each individual.

C.

One of the most important biochemical tools for DNA


manipulation, Type II restriction enzymes are often used
in producing fragments for analysis of DNA, to generate a
DNA "fingerprint." These enzymes recognize and cleave
double-stranded DNA at specific sequences of base pairs,
usually 4-8 bases long and palindromic (reading the same

D.

DNA is highly conserved among human


beings.
DNA is not highly conserved among human
beings.
DNA is completely conserved among human
beings.
DNA differs dramatically from person to
person.

61.

forwards and backwards).

By treating DNA with a series of restriction enzymes,


a researcher produces precisely defined DNA fragments.
Since genetic differences between individuals create or
eliminate restrictions sites, each person has a unique
pattern of fragments. Differences also exist between
corresponding segments of homologous chromosomes in
the same person. The fragments from chromosomes

Although bacteria are the source of restriction


enzymes, their own DNA is protected against
cleavage by specific methylations. What is the
probable purpose of the bacterial restriction
enzymes?
A.
B.
C.

To activate foreign DNA


To inactivate foreign DNA
To incorporate foreign DNA into the bacterial

D.

To promote cell lysis in response to foreign

genome

treated with restriction enzymes are of different lengths


and exhibit "restriction fragment length polymorphisms"
(RFLPS). The fragments can be separated according to
size by gel electrophoresis.

DNA

62.

To screen for an inherited disease, probes can be used


to check the fragmented DNA for the warning marker,
indicated by a unique, predicted cleavage pattern.
Screening of fetal tissue for sickle-cell anemia and other

This diagram of gel electrophoresis results indicates


DNA fragments corresponding to the gene for the
beta chain of hemoglobin. The DNA fragments were
produced by the restriction enzyme Mstll. The
parents are A and B, and their children are C and D.
Hb A refers to normal adult hemoglobin, and Hb S
refers to mutated hemoglobin that causes the
sickling of red blood cells.

inherited diseases can now be carried out. In the case of

sickle-cell anemia, the substitution of valine for glutamate


at one site in the beta chain leads to the elimination of a

restriction site. This changes the pattern of fragments and

can be used as an identifier of inheritance of mutated


genes.

In some criminal investigations, small samples of


tissue taken from the crime scene can be amplified with

HbS

the polymerase chain reaction (PCR), fragmented with


restriction enzymes, and compared with samples taken

HbA

from suspects.
Figure 1

59. Assuming DNA has an equal proportion of all bases,


how many fragments would a restriction enzyme
that recognized 4 base pairs produce from a DNA
fragment that is 10,000 base pairs long?

Which of the following statements are FALSE,

based on the fragmentation pattern depicted in the


gel shown above?
A.

A.
B.
C.
D.

625
63
390
39

Copyright by The Berkeley Review

Child C does not have sickle-cell anemia.

B.

Both parents are heterozygotes.

C.

Child D does have sickle-cell anemia.

D.

The parents have a 50/50 chance of having a


child with sickle-cell anemia.

462

The Berkeley Review


Specializing in NCAT Preparation

Biology

Restriction Enzymes

Passage X

63. In the gel depicted in Figure 1, why do the children


have only one band, instead of two like their
parents?

I.

Each of the children carries only one copy of


the gene for the beta chain of hemoglobin.

II. The children are homozygotes.


III. The two fragments migrated to the same
location on the cell.

64.

A.
B.
C.

I only
I and II only
II and III only

D.

I, II, and III

How can fetal tissue be collected in utero for

analysis of risk of inherited genetic diseases?


A.

Collect fetal cells in amniotic fluid by

B.

Collect fetal cells from the maternal blood

C.

supply.
Perform a therapeutic abortion.

D.

Perform a Cesarean section.

amniocentesis.

65. For which of the following projects would restriction


enzymes be used advantageously?
I.
Isolating and sequencing DNA
II. Solving a paternity dispute
III. Genetic counseling for potential parents
A.
B.
C.

I only
I and II only
II and in only

D.

I, II, and III

Copyright by The Berkeley Review

463

The Berkeley Review

Specializing in NCAT Preparation

Biology

Arabinose Operon

Passage XI

68. The following reaction is carried out by the product

Passage XI (Questions 66-72)

of the araA gene:

The arabinose operon is found in bacteria and is


responsible for the bacteria's ability to use arabinose as a
source of energy. The operon (Figure 1) consists of a
regulator gene, three control sites, and three structural
genes. Production of the C protein (from the araC gene) is
under control of the araOl operator. Transcription of the
araC gene occurs when the level of C protein and cAMPCAP complex is low. CAP is the catabolite gene-activator
protein found in many inducible operon systems. If the
level of cAMP-CAP complex is low while the level of C
protein is high, a C protein binds to the araOl site and

CH2OH

CHO

H-C-OH

C=0
I

HO-C-H

HO C-H
I

HO-C-H
I

HO C-H
I

CH2OH

CH2OH

The enzyme that catalyzes this reaction is:

inhibits transcription of the araC gene.

The transcription of the structural genes B, A, and D is


dependent on the formation of a DNA loop. If cAMP
levels are low and C protein levels are high, a C protein

A.

an epimerase.

B.

an isomerase.

C.

a dehydrogenase.

D.

a kinase.

binds to both the ara02 and ara I sites by forming a

Which of the following concepts is NOT illustrated


by the arabinose operon?

hairpin turn within the DNA. Aral is adjacent to the


promoter for the structural genes, and a bound C protein

A.

blocks transcription of those genes. In the situation where


both arabinose and cAMP are abundant, the hairpin loop
is not formed. The reason is that the C protein with bound

A protein can alter its own production by


altering the transcription of its gene.

B.

arabinose has an altered conformation and binds to both

C.

Structural genes are regulated only by DNA


sites found in contact with those genes.
Changes in transcription brought about by a
signal molecule are reversible.

69.

araO] and aral, but not to ara02. In addition, the cAMPD.

CAP complex binds between the araOl and aral sites. The
result of these changes is a polymerase that is now able to
bind to the araBAD promoter site and promote
transcription of these genes. The products of these
structural genes promotes the uptake of arabinose and
converts it to a usable form of energy.
ara02

araC

araO]

aral

araB

araA

Regulator

Control

Structural

gene

sites

genes

66.

67.

transcription, depending on the presence of a


signal molecule.
70.

araD

71.

High levels of cellular cAMP are associated with:


A.

high levels of arabinose.

B.

low levels of arabinose.

C.
D.

high levels of glucose.


low levels of glucose.

The polymerase responsible for the transcription of


the structural genes in the arabinose operon is most
likely:

A.
B.
C.
D.

The synthesis of the C protein is BEST described as:

A.
B.
C.
D.

Proteins can act as inhibitors or activators of

unregulated.
deregulated.
regulated by arabinose.
autoregulated.

72.

RNA polymerase.
RNA polymerase I.
RNA polymerase II.
RNA polymerase III.

An organism lacking the araC gene would exhibit:

It has been discovered that the genes responsible for


the uptake of arabinose are not in the operon shown
in Figure 1. This may have selective value, because:

A.

A.

increased use of glucose as a source of cellular

B.
C.

C.
D.

of arabinose.
a mutation in one area would not affect the

B.

a lack of cAMP production.


inefficient transcription of structural genes ara
B, A, and D.

D.

increased use of arabinose as a source of

cellular energy.

Copyright by The Berkeley Review

there is an increase in the amount of arabinose

transported into the cell.


there is a preferential use of glucose over
arabinose as a cellular energy source.
the design leads to a more efficient metabolism

energy.

integrity of the other region.


464

The Berkeley Review


Specializing in NCAT Preparation

Biology

Gene Therapy Strategies


74.

Passage XII (Questions 73-79)


Our modern understanding of the molecular nature of

genetic expression has paved the way for potential


breakthroughs in the field of gene therapy. It is now
possible for us, at least in vitro, to prevent the expression

Passage XII

A plasmid is transfected into a cell line with the aim


of blocking the expression of a wild-type
chromosomal gene called XI. In order for this to be
accomplished, the plasmid would have to contain:
A.

a mutant XI gene sequence with a strong

B.

a mutant XI gene sequence with a weak

C.

a wild-type XI gene sequence that is inverted


with respect to its promoter.
a wild-type XI gene sequence that is separated
from its promoter.

of disease-causing genes. Additionally, the same

promoter.

techniques can prevent the expression of viral genes,


thereby preventing cellular production of new viruses.
Two major approaches have been at the forefront of gene
therapy research in recent years.

promoter.

D.

The first takes advantage of the mRNA molecule's

ability to form double-stranded RNA-DNA hybrids.


Artificial DNA can be made that is complementary to the
"sense" mRNA of a disease-causing or viral gene. This

75.

complementary sequence is termed antisense DNA. The

There is much debate over the therapeutic potential

of ribozymal versus antisense gene therapy. Which


of the following would represent advantages of the

antisense and sense strands can hybridize to each other,

preventing translation of the disease-causing or viral


protein product. DNA, rather than RNA, is used to make

ribozymal approach?

antisense strands, because it is inherently stabler.

I.

Additionally, to protect it from enzymatic degradation,


the DNA's phosphate backbone is replaced with a
synthetic compound called methylphosphonate.

II.
III.

The second approach relies on the ability of certain


RNA molecules to act enzymatically and cleave mRNAs

A.
B.
C.
D.

at specific sequences. These RNA enzymes are referred to


as ribozymes. Ribozymes, like tRNA molecules and
ribosomes, have a tertiary structure resulting from base-

Base-paired RNA is less easily degraded than


synthetic single-stranded RNA.
Ribozymes act catalytically.
Individual ribozymes are very broad in their
substrate specificity.
I only
II only
I and II only
I and III only

pairing of part of the RNA strand to itself. Artificial


ribozymes can be designed to cleave HIV viral RNA at
specific sites, preventing it from being translated and
therebyslowing the reproduction of the virus.

76.

Which of the following would NOT affect the Km of


a ribozyme in vitrol

A.
B.

A change in the concentration of substrate


A large change in the pH of the medium

C.

A decrease in the salt concentration of the


medium

D.
73. Which of the following could NOT be treated
effectively by either of the gene therapy techniques

77.

An increase in the temperature of the medium

Of the two gene therapy techniques described in the

passage, which would be the MOST successful in


combating a rapidly mutating strain of the HIV

discussed in the passage?

virus?

A.

Tumors caused by overexpression of an

A.

oncogene

B.

C.

Sickle-cell anemia caused by a homozygous


mutation in the hemoglobin gene
Liver disease caused by the hepatitis B virus

D.

Herpes caused by the herpes simplex retrovirus

The antisense approach, because mutation in


the viral DNA would not affect antisense-

sense binding.

B.

The antisense approach, because DNA-RNA


hybrids would preferentially form with HIV

C.

The ribozyme approach, because ribozymes

D.

The ribozyme approach, because ribozymes


can target conserved regions of HIV mRNA.

mutants.

cleave all mRNA.

Copyright by The Berkeley Review

465

The Berkeley Review

Specializing in NCAT Preparation

Biology

Gene Therapy Strategies

Passage XII

78. Which of the following could be a problem with


antisense gene therapy that would occur in human

patients, but not in human tissue culture?


A.

Lysosomal digestion of synthetic antisense

B.

Immune rejection of synthetic antisense DNA

C.

Failure of DNA-RNA hybridization due to


cellular pH levels

DNA molecules
molecules

D.

Degradation of antisense DNA by restriction


enzymes

79.

What might be a normal function of naturallyoccurring ribozymes?


A.
B.
C.
D.

Splicing of unprocessed mRNA


Proofreading of single-stranded DNA
Catalysis of protein reactions
Cleavage of viral DNA

Copyright by The Berkeley Review

466

The Berkeley Review


Specializing in NCAT Preparation

Biology

Tryptophan Operon

Passage XIII (Questions 80-87)

80.

Passage XIII

The fact that the mRNA in the tryptophan operon is


polycistronic indicates one:

The tryptophan operon, including the promoter (p),

operator (o), attenuator (a) control sites, genes for the


leader sequence (L), and the DNA encoding the enzymes

A.
B.

(A-E) needed to convert chorismate into tryptophan (trp),

C.

mRNA molecule codes for more than one

D.

protein.
mRNA molecule codes for only one protein.

is shown in Figure 1. These five enzymes are synthesized


by the translation of polycistronic trp mRNA.

trpE trpD trpC trpB trpA

Structural genes

81.

protein is coded for by multiple genes.


protein is coded for by a single gene.

As it interacts with the trp operon, tryptophan is


BEST described as:

Figure 1

A.

One way to regulate the Trp operon is through a


specific repressor protein. The repressor protein is
encoded by the trpR gene, and it binds to the operator
region of the trp operon only when tryptophan itself is
bound to the repressor molecule. The repressortryptophan complex binds specifically to a region on the
operator that overlaps with the promoter region. The
result is that no mRNA is produced.
It was discovered that a

82.

162-nucleotide leader

sequence preceded the initiation codon of trpE, the first


enzyme encoded in the operon. In nonmutants with high
tryptophan levels, a transcript of only 130 nucleotides is
found. In nonmutants with scarce trp levels, the mRNA

transcript product is 7000 nucleotides long. Investigators


concluded that the transcription of the trp operon is
regulated by a controlled termination site called an

an inducible repressor.

B.

an inducer.

C.

a corepressor.

D.

an attenuator.

In an experiment with the Trp operon, deletions


were made between the operator and the gene for the
first enzyme (trpE), resulting in a significant
increase in the production of trp mRNA. This
experiment provides evidence for:
A.

signal transduction.

B.

attenuation.

C.

tryptophan-repressor complex repression.

D.

tryptophan induction.

attenuator. The site is located between the operator and

the gene for the first enzyme. The terminator site has a
GC-rich sequence followed by an AT-rich sequence.
Each region exhibits a twofold axis of symmetry.

83.

The attenuator site works with the generator site in

regulating the transcription of trp genes. When tryptophan


is plentiful, initiation of transcription is blocked by
binding of the trp-repressor complex to the operator.
When trp levels fall, some polymerase molecules
dissociate from the template at the attenuation site, while
others proceed to synthesize the entire trp message.

The binding of the tryptophan-repressor protein to


the operator prevents interaction between:
A.
B.
C.
D.

The presence of tryptophan residues at positions 10


and 11 of the leader polypeptide is significant in the
attenuator site's ability to detect the level of tryptophan in
the cell. When tryptophan is abundant, this complete
polypeptide is synthesized, which will terminate
transcription. When tryptophan is scarce, the ribosome

DNA
RNA
RNA
RNA

polymerase and DNA.


polymerase and DNA.
polymerase II and DNA.
polymerase HI and DNA.

84. According to the passage, it is important for


attenuation that:

A.
B.
C.
D.

stalls at tandem UGG codons because of a lack of

tryptophanyl tRNA. The stalled ribosome alters the


structure of the mRNA so that the polymerase

replication be coupled to transcription.


replication be coupled to translation.
transcription be coupled to replication.
transcription be coupled to translation.

transcribingit proceeds beyond the attenuator site.

Copyright by The Berkeley Review

467

The Berkeley Review

Specializing in NCAT Preparation

Biology

Tryptophan Operon

Passage xm

85. The melting of the trp attenuator site on DNA could


be described as:

A.

continuous, as the nucleotide population is not

B.

continuous, as the nucleotide population is


unimportant.
discontinuous, with the A-T rich region
melting after the G-C rich region.
discontinuous, with the A-T rich region
melting before the G-C rich region.

varied.

C.
D.

86. Several other operons for the biosynthesis of amino


acids in E. coli are known to have an attenuator site.

In each case, the leader peptide should contain:

87.

A.
B.

tandem pairs of the codon UGG.


a relatively small number of the codon UGG.

C.

an abundance of amino acid residues of the

D.

kind synthesized by the operon.


a relatively small number of residues of the
kind synthesized by the operon.

Increasing the number of residues in the leader


peptide of the kind synthesized by the operon
MOST likely:
A.

decreases the sensitivity of the detection


system.

B.

increases the sensitivity of the detection


system.

C.

eliminates the need for attenuation.

D.

serves only to alter the primary structure of the


protein.

Copyright by The Berkeley Review

468

The Berkeley Review


Specializing in NCAT Preparation

Biology

Translation

Passage XIV (Questions 88-94)

Passage XIV

90. The BEST experimental method to separate


polyribosomes from single ribosomes is through:

The expression of genetic information often results in


the production of protein molecules from the translation
of messenger RNA (mRNA). To initiate protein synthesis,
initiating factors (IF) bind to the small ribosomal unit,
along with a special initiator transfer RNA molecule.
This complex recognizes the mRNA start codon. The
recognition of the start codon causes the release of the
initiating factors. This is followed by the binding of the
large ribosomal subunit and results in a functional

91.

A.

addition of ribonucleases.

B.

centrifugation.

C.
D.

acidic wash.
addition of radiolabeled amino acids.

In a prokaryotic cell, the special initiator aminoacyl


transfer RNA molecule that binds to the start codon

transports which amino acid?

ribosome.

Multiple initiating events, involving multiple


ribosomes, can occur on a single mRNA transcript. The
resulting polyribosome/polysome structure gives multiple,
simultaneous translations of a single mRNA transcript.
An important eukaryotic IF is eIF-2, which is
instrumental in binding the initiator transfer RNA onto the
small ribosomal subunit during the initiating phase. In
reticulocytes, eIF-2 is believed to be involved in
controlling the overall rate of protein synthesis, with
phosphorylation of one of its subunits reducing eIF-2
activity.

92.

A.

Glycine

B.

Glutamine

C.
D.

Asparagine
Formylmethionine

A polycistronic mRNA molecule is MOST likely to


be found in a cell that is:

A.
B.

C.
Translation is not an error-proof process. Roughly 1 in

every 104 amino acids that are incorporated into proteins

D.

is inserted incorrectly. The fidelity of this process is


contingent on several processes, one being the correct
pairing of bases in the codons in mRNA and anticodons in
the transfer RNA. Cells have evolved a proofreading
mechanism for this pairing process involving the

93.

The delay in elongation, caused by the elongation


factor, acts as a proofreading mechanism, because:

A.

elongation factor, a protein that binds to an incoming


aminoacyl-transfer RNA and to a molecule of GTP. This
factor has the ability to separate in time codon/anticodon
base-pairing and peptide elongation. This short delay

eukaryotic and contains many start codons,


translated separately into multiple proteins.
eukaryotic and gives rise to one polypeptide,
cleaved into many functional proteins.
prokaryotic and contains many start codons,
translated separately into multiple proteins.
prokaryotic and gives rise to one polypeptide,
cleaved into multiple functional proteins.

the delay allows the time needed to insert the


correct amino acid.

B.

the delay allows for hydrolysis of GTP, which


attaches the correct amino acid to the correct
transfer RNA.

between the two events allows for a bound-transfer RNA

C.

molecule to be discharged from the ribosome.

incorrectly bound transfer RNA molecules


form weaker codon/anticodon interactions and

are more likely to dissociate from the

88. In a hypothetical experiment, eukaryotic cells are

ribosome.

starved. Based on information in the passage, one


would expect increased activity from:
A.

a mutase.

B.

a kinase.

C.
D.

a dehydrogenase.
an epimerase.

D.

ribosome.

94. Which of the following statements is FALSE, based


on information in the passage?
A.

89.

Given that the average size of a protein molecule is


400 amino acids, one can conclude that there should

B.

molecules?

C.

50
100

Copyright by The Berkeley Review

C.

The anticodon found on the initiator transfer

D.

The elongation factor is reversibly bound to


the aminoacyl transfer RNA molecule.

RNA is CAU.

10
25

D.

The initiator transfer RNA binds to the start


codon in the P site of the ribosome.
Initiation factors are not needed for secreted

proteins, because these proteins are translated


and transported in the endoplasmic reticulum.

be a translation error in one out of how many protein

A.
B.

the delay allows for hydrolysis of GTP, which


drives the peptide-forming reaction at the

469

The Berkeley Review


Specializing in NCAT Preparation

Biology

Passage XV

Cancerous Gene

96. The tag sequence used in the above experiment

Passage XV (Questions 95-100)

should:

In theory, one could compare the base sequence of all


chromosomes in a normal cell to those in a cancerous cell,

in order isolate a mutation suspected of causing cancer.


However, such a method is entirely impractical. To isolate
the mutation more efficiently, one could exploit the

A.
B.
C.

normally be found in the mouse cell line.


be an unknown, random region of DNA.
not normally be found in E. coli.

D.

not normally be found in human DNA.

process of expressing genetic information. This is


accomplished by using a particular line of mouse cells,
which stop proliferation when the population of cells on
97.

the medium is one cell-layer thick. This is a normal

The Alu sequence is MOST likely used as a tag

process, as these cells are sensitive to their population


density. However, the cells can be induced, through the

because it is:

addition of DNA, to continue multiplying and to enter a


state of uncontrolled growth.

A.
B.

C.

rare within the human genome.


easily mutated.
likely to be near a cancer-causing, mutated

D.

easily inserted into the mouse cell line.

The added DNA responsible for inducing the

gene.

cancerous state is extracted from human tumors and is

tagged before it is added to the mouse cell line. Individual


cells transformed by human DNA into cancerous cells
form a pile of cells known as a focus. Once a focus has
cloned in E. coli. The colonies of E. coli can then be

98.

Mutated gene A has been shown to induce


uncontrolled growth. In vivo studies of the gene

screened for the presence of the tag. The screen involves


the addition of radiolabeled DNA complementary to the

protein product from gene A. From this information,

tag sequence.

it can be concluded that:

One particular method used to tag the human tumor DNA


takes advantage of the Alu sequence found throughout the
entire human genome. Human DNA is cleaved using Alu

A.

appeared, DNA from these focus cells is extracted and

have demonstrated normal levels of mRNA and

restriction sites and is added to the mouse cell line. After

B.

the appearance of a focus and cloning in E. coli, a


radiolabeled probe complementary to the restriction site is
used to isolate a DNA sequence containing not only the
tag, but the mutated gene responsible for inducing a

the mutation in gene A has resulted in a


depressed RNA polymerase binding
frequency.
protein product A is necessary for entering
mitosis.

C.
D.

cancerous state.

the mutation in gene A has resulted in a


significant structural change in protein A.
the cell line used possesses a retarded cellular
ribosome level.

99.

95.

Hybridization between complementary DNA


sequences takes advantage of:
A.

covalent bonds.

Comparing the entire genome of a wild-type cell to a

B.
C.

hydrogen bonds.
hydrophobic base stacking interactions.

cancerous cell in order to isolate a mutation is

D.

ionic bonds.

impractical, because:
A.

B.
C.

D.

it is theoretically impossible to sequence an


entire genome.
it is more important to characterize the faulty
proteins and not the DNA mutations.
naturally occurring mutations would mask
cancer-causing mutations.
the human genome varies among individuals.

Copyright by The Berkeley Review

100. To screen for the presence of the DNA tag, which of


the following experimental methods should be used?

470

A.
B.
C.

Southern transfer
Northern transfer
Western transfer

D.

Gel electrophoresis

The Berkeley Review


Specializing in NCAT Preparation

Biology

Expression of Genetic Information

Section X Answers

Southern Blotting

Passage 1(1 - 5)
1.

A is correct. Even if we don't know the answer, we can eliminate the three wrong answers in this question.
Proteases degrade proteins, not DNA. Choice B is incorrect. Nucleotide synthases are involved in the synthesis of
nucleotides. Choice C is incorrect. DNA gyrases are involved in nicking and reattaching DNA so it does not tangle.
Choice D is incorrect. Restriction enzymes cleave DNA at specific, often palindromic, sequences. Their cleavage
points are predictable, and they therefore are often used to cleave DNA for further study. The correct choice is A.

2.

D is correct. DNA does not form a quad structure. Choice B is incorrect. Phosphates bear many negative charges,
and they do not become positively charged. Choice A is incorrect. Denaturing means that somehow the DNA
structure is changed. The base-pairing is not altered, but the hydrogen bonds holding the base pairs together break
and the two DNA strands separate. Choice C is incorrect. The correct choice is D.

3.

B is correct. In many types of gels, the electrophoretic mobility of a fragment is inversely proportional to the
number of base pairs in it (i.e., its size). The buffers used often produce a similarity in charge between molecules, so
choice A is incorrect. The radioactive molecule used in the Southern blot is employed after the gel, in Step 4. Choice
C is thus incorrect. The linking number is the number of times one strand of DNA wraps around another in a righthanded direction, so choice D is also incorrect. The correct choice is B.

4.

C is correct. Choices A and B are the same answer, just reversed. Both are incorrect. Choice D is a repeat of the

original strand and is incorrect. A probe is the complementary base sequence to the fragment of interest. The
correct choice is C.

5.

A is correct. This is really an easy, read-the-figure type of question. Although you are not told what the fragments
mean, you can still test each statement against the figure. One of the fragments corresponds to a region that indicates
the inheritance of the HC gene. Neither parent carries Fragment V, so choice B is incorrect. Neither child carries
Fragment III, so choice C is incorrect. The children have different fragmentation patterns, indicating different

patterns of inheritance. Choice D is incorrect. Finally, both children have Fragment II, so they would be affected if
this is the HC fragment. The correct choice is A.

Passage II (6 - 11)
6.

Restriction Endonucleases

D is correct. The three restriction sequences for the three restriction endonucleases are given in Table 1. Let's place
each of those restriction sites in a section of DNA. Consider the Pst I site first. After the enzyme cuts at the

appropriate site, we are left with two sticky ends. Both of those sticky ends show a piece of single-stranded DNA,
which bears a 3' end. This is sometimes referred to as a 3'-overhang. Based on this observation, we can eliminate
choice A.

i
i

icmcAci

13'

DNA
I

igacgtci

5'l

1CTGCA-3'

3'l

IG-5'

5'-GI

IT

3-ACGTC1

15'

.>

"15'

T
Sticky end

Pst I site

Consider the Sal I site next. After the endonuclease makes its cut we are again left with two sticky ends. However,
this time the sticky ends can be found at the 5' of each DNA strand. Choice B is a correct answer.
I
GTCGAC
DNA

CAGCTG

5'-TCGAC

;/

3'-G

T
Sticky end

Sal I site

However, before we rush into picking choice B, let's take a look at choice C. If we use the same type of reasoning
for the Taq I restriction endonuclease, we also Find 5' sticky ends. Therefore, both Sal I and Taq I produce 5' sticky
ends. The correct choice is D.

7.

D is correct. If we assume that the DNA sequence is random, then any given base (A, T, G, or C) has a 1/4
probability of occurring at any position in the polynucleotide chain. Table 1 tells us that Alu I contains 4 bases. We

Copyright by The Berkeley Review

471

The Berkeley Review


Specializing in MCAT Preparation

Biology

Expression of Genetic Information

Section X Answers

would expect to find an Alu I restriction site every 256 bases along the length of the DNA. This is calculated as
follows: (I/4)n = (1/4)4 = 1/256, where n is the length of the recognition site (i.e., the number of base pairs). The
length of pBR322 is4.3 kb or 4,300 base pairs. Remember, 1kb is 1,000 base pairs. Dividing 4,300 by 256 gives the
expected number of sites thatAlu I would cut in pBR322. The correct choice is D.
D is correct. We are asked to determine the number of unique restriction endonucleases that can cut a particular

segment of DNA. All we need to do is compare those restriction sites in Table 1 with the DNA sequence in the
question.
Sma I

Xma I

Sail

5'-CGGATCCCGGGTCGACG-3'
3'-GCCTAGGGCCCAGCTGC-5'

Taq I

Bam HI

Be aware of restriction sites that can be cut by two different enzymes, and be aware of smaller restriction sites
within larger restriction sites. In the segment of DNA shown above, the heavy lines indicate the location of the
various restriction sites. The correct choice is D.

B is correct. First, let's consider the sticky ends created by the restriction endonuclease Bam HI. This is shown
below:

DNA 5'I
V3'I

i
IGGATCCI
ICCTAGGI

J3' | ^ 5'I
15'
~^ Tl

IG-3'
5'-GATCC [=] 3'
IGGTAG-V
V-G\
IV
Sticky end

Bam HI site

We want to use a restriction endonuclease that can cut a segment of DNA and give us sticky ends complementary to
those created by Bam HI. Let's immediately examine the answer. The sticky ends created by BglU are shown below.
i
DNA

AGATCTI

5'

'

TCTAGA1

1 3'

:>

15'

T
Bgl II site

Sticky end

Next, we can combine one of the sticky ends from Bam HI with one of the sticky ends from Bgl II as shown below.
An interesting thing happens when we do this: After hydrogen bonding has taken place and the phosphodiester
backbone has been reestablished, we lose the restriction sites for both Bam HI and Bgl II.
Bgl II sticky end
i

kj-v

V-GATdl

ICCTAG-5'

3--AI:'

GGATCT

1 V

DNA
IS'

CCTAGA

Bam HI and Bgl II

Bam HI sticky end

sites are lost

The other three choices (HindUl, Pstl, and Sac I) do not give DNA with sticky ends that are complementary to
those given by Bam HI. The correct choice is B.
10.

A is correct We are digesting two linear DNA molecules. Let's call those DNA molecules DNA #1 (for Lane 1) and
DNA #2 (for Lane 2). Note that if we cut DNA #1 with Eco RI (see below), we get six fragments, each a different
size. This must mean that we have five Eco RI restriction sites in DNA #1.

Copyright by The Berkeley Review

472

The Berkeley Review

Specializing in NCAT Preparation

Biology

Expression of Genetic Information

Section X Answers

Eco RI

EcoRI

EcoRI

Eco RI

Eco RI

EcoRI

EcoRI

EcoRI

New
EcoRI

EcoRI

DNA#1 [

DNA
EcoRI

If we cut DNA #2 with Eco RI, we get seven fragments, each a different size (see above). This means that there are

six Eco RI restriction sites in DNA #2. Note thatFragment X from DNA #2 is a little larger than Fragment D from
DNA #1. Also note that Fragment Y is a little smaller than Fragment D, but a little larger than Fragment E from
DNA #1. Fragments C and E in DNA #2 are the same as Fragments C and E in DNA #1. We can conclude from this
that more DNA and an additional restriction site were added to Fragment D of DNA #1.

If we were to delete DNA from Fragment D of DNA #1 (without adding a new restriction site), we would still get
six fragments. This is not what we see in DNA #2. Eliminate choice B. The same would be true if we were to add
DNA only to Fragment D of DNA #1 (without adding a new restriction site). Eliminate choice C.

If we delete DNA from Fragment D of DNA #1, and as a result a new Eco RI site forms, then we get seven
fragments. This is what we are looking for. However, once we delete DNA from Fragment D, the new fragment that
forms (call it XT') is smaller than the original Fragment D. After electrophoresis, this new Fragment XT' is below
the level of Fragment D on the gel. Furthermore, if we now cut at the new Eco RI restriction site within Fragment
XT', the resulting two fragments still end up below the level of Fragment D after electrophoresis. This is not what
we observe in Lane 2 of the gel for the fragmentation of DNA #2. The correct choice is A.

11.

B is correct The first thing to do when creating a restriction map is to determine the size of the DNA. If we add the
fragment sizes in the first lane (created by a single digest with Bglll), we get 7 kb + 2 kb = 9 kb. If we follow this

same procedure for the remaining fragmentation lanes, we get 9 kb for each of them. This tells us our fragmented
bacterial DNA was originally 9 kb in length. Look at all of the restriction maps in the answer choices. Do all the
fragment sizes add up to 9 kb? Yes, they do.

The next step is to look at the fragmentation sizes in each of the six digest lanes of the gel to see whether they match
up with the answer choices. A single digest with just Bglll gives fragment sizes of 7 kb and 2 kb. We observe this
fragmentation pattern in choices B and C. A single digest with just Eco RI gives fragment sizes of 5 kb and 4 kb. We
observe this fragmentation pattern in choices A and B. At this point in our analysis, choice B looks like a good
answer. But let's make sure.

A single digest with just Pst I gives fragment sizes of 6 kb, 2 kb, and 1 kb. We observe this fragmentation pattern in
choices A, B, and D. A double digest using both Eco RI and Bglll gives fragment sizes of 4 kb, 3 kb, and 2 kb. We
observe this fragmentation pattern in choices B and D. A double digest using both Pst I and Bgl II gives fragment
sizes of 6 kb, 2 kb, and 1 kb. We observe this fragmentation pattern in choices A and B. Finally, a double digest
using both Eco RI and Pstl gives fragment sizes of 3 kb (there are two of them, since the band is twice as thick), 2
kb, and 1 kb. We observe this fragmentation pattern in choices A, B, and C. After considering all the choices, we
find that each digest lane gives the fragmentation pattern shown in choice B. The correct choice is B.

Ribozymes and HIV

Passage III (12 - 17)


12.

D is correct This is a question designed to make sure we understand the roles of the three RNA molecules in the
cell. A ribosome is composed of amino acids and rRNA, and is essentially a surface for assembling proteins.
Statement I is correct. mRNA is the transcript or copy of DNA. Statement II is correct. tRNA is the amino acid
carrier for protein translation. Statement III is correct. The correct choice is D.

13.

D is correct The focus of this question is the mutation of HIV. To combat this research problem most effectively,
one should try multiple attacks simultaneously by linking several ribozymes together. This would cause the most
damage in the first strike and also would attack any mutations that survived and tried to reproduce. Following one
enzyme with another two weeks later could still permit mutations to occur, so choice B is incorrect. Increasing the
concentration of the original ribozyme does not combat mutation, because the survivors would still be able to
recolonize the patient. Choice A is incorrect. A specific ribozyme for each patient would not affect further mutation,
so choice C is incorrect. The correct choice is D.

Copyright by The Berkeley Review

473

The Berkeley Review

Specializing in NCAT Preparation

Biology
14.

Expression of Genetic Information

Section X Answers

B is correct Choice A is incorrect, because highly conserved regions do not have much variability from strain to
strain. HIV can be attacked by ribozymes at any number of sites, not just these conserved regions, meaning that
choice C is also incorrect. And eliminate choice D: These regions cannot attract ribozymes. The action leading to

catalysis is a chance meeting of two molecules that fit together. The correct choice is B.
15.

B is correct Prior to mutation, there are fewer copies of a virus if its mRNA is decreased. Each mRNA does not
become more efficient at translation. Choice A is incorrect. The number of transcripts is not increased, so viral

particles cannot increase. Choice C is incorrect. Choice D refers to mutant (competing) HIV mRNAs, and the
question specifically asks about changes prior to mutation, sochoice Dis incorrect. The correct choice is B.
16.

A is correct. There are only four different nucleic acids used to build every RNA molecule. An enzyme has access

to twenty different amino acids. This increase in the number of potential subunits gives much more variety in
structure and function to protein-based catalysts. Statement I is correct. Enzymes did not outcompete ribozymes by

chopping them up. They merely worked better and were therefore selected by evolutionary pressure. Statement II is
incorrect. Enzymes do notchange the equilibria of reactions, so statement III is incorrect. The correct choice is A.
17.

B is correct Catalysts can participate in many reactions, because they remain unchanged themselves while effecting
change in other molecules. Statement I is correct. Catalysts do increase the rate of reactions, but not the equilibrium,
meaning that statement II is correct. And since we know that the body does synthesize catalysts, statement III is
incorrect. The correct choice is B.

Passage IV (18-23)
18.

The Genetic Code

C is correct. A single-base code can code for four different amino acids. One can arrive at this answer by knowing
that there are four different bases used in DNA and that if the code had only one base, then only four different amino

acids could be coded. Accordingly, a three-base system can code for a total of sixty-four amino acids (4x4x4 =
64). The question asks for the difference between the two, making 64 - 4 = 60 the correct answer. The correct
choice is C.

19.

C is correct To answer this question, one has to know what is meant by an overlapping genetic code. As in the
question, let us assume the base sequence is ATCGGTATA. If this genetic code is overlapping, the first three amino
acids come from ATC, TCG, and CGG respectively. If the code is nonoverlapping, the first three amino acids come
from ATC, GGT, and ATA respectively. Since the question asks about the products of an overlapping code (the
code is, of course, non-overlapping), as many as three amino acids may be affected. The correct choice is C.

20.

C is correct. Without the extra base inserted, the base sequence can generate four amino acids. Remember, the code

is nonoverlapping. Let us look at the amino acids produced with the extra base inserted. The first two amino acids
are not affected by the insertion. However, the third amino acid is. Without the insertion, the triplet is ACA. With
the insertion, the code is ATC. The same logic applies to the fourth amino acid. Thus two amino acids are affected.
The correct choice is C.

21.

B is correct The passage states that part of the protocol of the experiment is to wash the protein-containing

precipitate and measure the level of carbon 14. One can assume we are measuring the radiolabel in the protein. If
this is the case, we must use radiolabeled amino acids as part of the experiment, because this is the only way we can
incorporate radiolabel into the protein. Do not lose track of the big picture. We are controlling the mRNA added and

asking what amino acids are called for. This means we need to keep track of amino acids, so radiolabeling them is
essential. The correct choice is B.

22.

C is correct The passage states that a component of the experiment is the addition of DNAse, which results in a
cessation of protein synthesis. This implies that protein synthesis was occurring and then ceased, and it should be
indicated on a graph by a line with a positive slope followed by a line with zero slope. This alone should eliminate
all choices except C. Looking at the graph in C, the zero-slope line is followed by another rise in protein synthesis.
This occurs because we added our own synthesized mRNA, which created the new protein. Remember, that is the
point of the experiment-the ability to synthesize protein in a cell-free system. The correct choice is C.

23.

A is correct Let us look at the data in Table 1. With phenylalanine, we see a 100% relative amount incorporated.
Remember, the label is on the uracil, and therefore we can conclude that phenylalanine is coded by a triplet
consisting entirely of uracil. The question asks about glycine. Glycine has the lowest count, according to the data.
The fact that it has some count invalidates the idea that the triplet is all guanine. Furthermore, there cannot be an

equal amount of the two bases in a triplet design. Since the count is low, the most logical conclusion is that there is
more guanine than uracil in the code for glycine. The correct choice is A.
Copyright by The Berkeley Review

474

The Berkeley Review

Specializing in MCAT Preparation

Biology

Expression of Genetic Information

Passage V (24 - 29)


24.

Section X Answers

Polymerase Chain Reaction (PCR)

B is correct. Although DNA polymerase I from E. coli could be used in the PCR reaction, the scheme would be

inefficient, because each round of denaturation of the double-stranded DNA helix to form the individual single
strands would result in destruction of the polymerase. Thermus aquaticus is a thermophile, and because of its normal
living conditions, it contains a temperature-resistant polymerase. This polymerase withstands temperatures of 95 C,
which is necessary to denature the DNA. The use of this polymerase allows for increased specificity between the
hybridized primers and the template. This is so because a temperature of 45 C is necessary for the hybridization. At

this temperature, the polymerase is inactive. Therefore, we raise the temperature to 75 C, at which the polymerase
becomes active. By this point, those primers that are not hybridized correctly will dissociate from the incorrect sites.

Even though some of the correctly hybridized primers fall off, a much greater fraction of primers on incorrect sites
fall off. The reaction becomes more specific. Again, the polymerase from E. coli cannot take the heat used in the
denaturation process. The correct choice is B.

25.

A is correct. The melting of DNA is readily monitored by measuring its absorbance of light at 260 nm. The
unstacking of base pairs that occurs during the melting process results in an increased absorbance of light at this
wavelength. This effect is known as hyperchromism. The spontaneous reassociation of separated complementary
strands of DNA is known as annealing. Therefore, this eliminates choices B and D. To discriminate between choices

A and C, one must draw on their knowledge of biochemistry. As stated above, the increased absorption due to basepair unstacking is known as hyperchromism. The correct choice is A.

26.

A is correct. First, we must realize that a suitable clone would contain the inserted fragment. This is why we are
using PCR. We are screening clones to see if a piece of DNA was inserted into the plasmid. The larger piece of
DNA thus be represents the suitable clone, because the foreign piece of DNA was properly inserted. Next, we must
draw on our knowledge of gel filtration chromatography. The apparatus used in this setup includes a column
consisting of porous beads made of an insoluble but highly hydrated polymer. Small molecules can enter these
beads, but larger ones cannot. In the beads, the movement of the small molecules is impeded. The result of this is
that large molecules flow more rapidly through this column and emerge first (because a smaller volume is accessible
to them). Therefore, the product of the suitable clone, a large molecule, emerges first because of its size, not its
charge. The correct choice is A.

27.

A is correct We are looking for the DNA molecule that is expected to have an abundance of A-T base pairs. The
melting temperature of a DNA molecule depends heavily on its base composition. DNA molecules rich in G-C base
pairs have a higher melting temperature (defined as half of helical structure lost) relative to those DNA molecules
with a high A-T base pair content. The reason for this lies in the number of hydrogen bonds between the bases, with
three between G-C and only two between A-T. In addition, adjacent G-C base pairs interact more strongly with one
another than do adjacent A-T base pairs. The DNA melting curves show absorbance as a function of temperature.
Remember that the absorbance rise is indicative of a double-stranded molecule becoming a single-stranded
molecule. We are looking for the molecule that is going to have the highest A-T base-pair content. In other words, it
is going to have the lowest melting point temperature. This corresponds to the curve on the far left, as its absorbance
rise occurs at the lowest temperature. The correct choice is A.

28.

C is correct According to Figure 1, one cycle of PCR uses one molecule of DNA to make two molecules of DNA.
Another round of PCR uses two molecules of DNA to make four molecules of DNA. In other words, the number of

molecules of DNA made during n cycles is 2". Therefore, after seven cycles of the PCR reaction, we should see 128
molecules of DNA. However, the question asks how many strands of DNA exist after seven cycles. Each DNA
molecule is made up of two strands. So 128 x 2 = 256. The correct choice is C.

29.

D is correct. The primers used in the PCR scheme are complementary to the template DNA, which contains the
region of DNA to be amplified. They are not complementary to each other. This eliminates choice A. Furthermore,
these primers probably are not going to contain the same nucleotides. Their sequences depend on the sequence of the
DNA region with which they are hybridizing. This eliminates choice B. As for choice C, the primers are DNA
oligonucleotides. Therefore, they do not contain uracil. Only RNA contains uracil in place of thymine. Choice C can
be eliminated. The only logical choice is D. The primers are complementary to the regions of DNA that flank both
sides of the piece of DNA to be amplified. One primer is complementary to one side, while the other DNA primer
hybridizes with the other. Only then do we get the amplification of DNA that we are interested in. Therefore, the
primers are designed to conform to the regions of DNA that surround the DNA to be amplified. The correct choice
isD.

Copyright by The Berkeley Review

475

The Berkeley Review


Specializing in MCAT Preparation

Biology

Expression of Genetic Information

Lactose Operon

Passage VI (30 - 36)


30.

Section X Answers

C is correct If lactose cannot be utilized by the epithelial cells, then stays in the intestinal lumen. Lactose acts as a

solute in the lumen and begins to draw water out of the lumenal epithelial cells (the reverse of choice C). Too much
water in the lumen can lead to diarrhea. Also, bacterial cells have the ability to take up lactose and metabolize it

anaerobically (fermentation). One ofthe products offermentation is CO2, which isa gas. The correct choice isC.
31.

C is correct The second paragraph of the passage says, "Metabolism of lactose requires galactoside permease, a

cotransporter of protons and lactose...." If protons and lactose are cotransported, it means that they are allowed to
pass through the cell's membrane together. The transport of lactose into the cell is coupled to the energy stored in the
transmembrane potential that resides across the plasma membrane. Recall from metabolism that protons are pumped
out of the cell as the electrons move down the electron transport chain. Not only is a large membrane potential

established, but a high concentration of protons now resides in the medium outside the cell. These gradients of
stored energy can be used to import sugars like lactose into the cytosol. When molecules are transported into a cell
together (or out ofa cell together), the process is referred to as a symport. If molecules move in opposite directions
as they are transported through the cell's membrane, the process is referred to as an antiport. If one molecule is
transported through the cell's membrane, the process is referred to as a uniport. Active transport requires the
hydrolysis of ATP. There was no mention of ATP hydrolysis in the passage. The correct choice is C.
32.

D is correct. In the third paragraph of the passage, we learn that RNA polymerase binds to the promoter (P). The

promoter, as shown in Figure 1 of the passage, is a section of DNA. In the fourth paragraph, we learn that RNA
polymerase can transcribe the structural genes (i.e., can make mRNA). The structural genes also involve a section of
DNA. Since we know that RNA polymerase synthesizes mRNA, we can eliminate choices A and B. Is the RNA

polymerase directed to synthesize mRNA by RNA or DNA? The polymerase is binding DNA. Therefore, it is the
DNA that is directing the RNA polymerase as it synthesizes the mRNA. The correct choice is D.
33.

C is correct. In the fourth paragraph of the passage, we learn that "if lactose is absent from the medium, the lac
repressor binds to the operator, and RNA polymerase is unable to transcribe the structural genes." This tells us that
in order to make a transcript, we must remove the repressor from the operator. How do we remove the repressor?
We need an inducer (see the fourth paragraph again). Lactose can enter the cell and be converted into allolactose,
which is the inducer. Lactose itself is not the inducer. It is allolactose (an isomer of lactose) that is the true inducer.

Once the inducer binds to the repressor, the inducer-repressor complex is no longer active, and it no longer binds to

the operator. With this information alone, we can eliminate choices A and B. RNA polymerase can bind to the
promoter. The inducer-repressor complex (choice D) does not bind to the promoter. If it could, RNA polymerase
would not have a place to occupy in order to begin transcription. Once RNA polymerase binds to the promoter and
the repressor has been removed from the operator (by complexing with the inducer), the DNA is free of
obstructions, and transcription can begin. The correct choice is C.

34.

B is correct We learned what happens to lactose in the first four paragraphs of the passage. What happens to
glucose is contained in the fifth paragraph. When glucose is in the cell, the cAMP concentrations start to decrease.
Right away, we can eliminate choices A and C. We next learn that transcription of the lac operon is reduced some
50-fold. In other words, the synthesis of p-galactosidase is greatly reduced. This is what choices B and D state. We
need something else. If the levels of [3-galactosidase decrease, it must mean that there is a repressor on the operator.
The repressor must be active in order to be occupying the operator. This allows us to eliminate choice D. Let's
continue (even though we now know the answer). If the cAMP levels in the cell are low, there is not enough cAMP
to bind the CRP. The cAMP-CRP complex does not form. Therefore, the CRP cannot be activated. It will remain
inactive until the levels of cAMP once again begin to increase. The correct choice is B.

35.

D is correct The key to this question involves the I" mutation of the i"0+Z+Y+A+ genome. In the last paragraph of
the passage, we find that these types of mutations are called constitutive. They lead to an increase in the synthesis of
lac operon proteins in the absence of inducer. If there is no inducer around, we would expect the repressor to bind to
the operator. However, the repressor is defective due to a mutation. It cannot bind to the operator. Thus, in the
absence of inducer, we wouldexpect to find a lot of the lac Z, lac Y, and lac A proteins being synthesized.
lac Z

lac A

lac Y

high

high

high

We would alsoexpect to see high levels of these gene-product proteins being synthesized in the presence of inducer
as well. The inducer does not bind to the operator, so in this case it does not matter whether it is present. The
correct choice is D.

Copyright by The Berkeley Review

476

The Berkeley Review

Specializing in MCAT Preparation

Biology
36.

Expression of Genetic Information

Section X Answers

Dis correct. Based on the genotype, which enzymes are produced all the time? Let's break the partial diploid into
two haploid segments. Consider the top genotype first (I+OcZ+Y"A"). The I+ gene produces a normal repressor,
which in turn binds to the operator, O. In the absence of an inducer, the repressor binds to the operator, and the
structural genes are not transcribed. In the presence ofan inducer, the repressor does not bind the operator, and the
structural genes are transcribed. However, there is a mutation in the operator, Oc, that allows the structural genes to
be transcribed all the time. The lac Z+ gene is normal and produces a good protein. The lac Y* and lac A" genes are
mutants. The proteins produced from these genes are defective. Therefore, the top genotype permits only the lac Z
gene to be constitutively produced.

Consider the bottom genotype next (I"0+Z"Y+A+). The I~ gene does not produce a normal repressor. The mutant
repressor is not able to bind to the normal operator. In the presence of an inducer, this is not a problem. The inducer
does not bind to the operator anyway. Since the repressor isa mutant, and since nothing is binding the operator, the
structural genes can be transcribed. However, the lacZ" gene bears a mutation, and the protein is defective. The lac

Y+ and lac A+ genes are normal and give good proteins. In the absence of an inducer, the operator region is still
wide open. We get a defective lac Z" protein and good lac Y+ and lac A+ proteins.

Based on this, we might conclude that all three gene products are made constitutively. However, we are dealing with
a partial diploid genotype and not individual haploid genotypes. Let's take a look at the combination of the two
genotypes in the partial diploid:

I+OcZ+Y"A"
I"0+Z"Y+A+

The last paragraph of the passage states that "independent mutations can also occur in the trans-acting (capable of
activity on another chromosome) regulatory gene and in the cis-acting (capable of activity only on the same
chromosome) operator gene." This tells us that the product of the lac I gene is diffusible. In other words, once the
repressor is synthesized by the normal lac I+ gene, it can diffuse though the cytosol and bind to the operator, which

is 0+. In the absence of aninducer, the repressor remains bound to that operator, and transcription cannot proceed. If
an inducer becomes available, it binds to the repressor and removes it from the operator. This is referred to as
induction, and this is what we see occurring in the case of the lac Y+ and lac A+ genes. The only gene that is
synthesized constitutively is the lac Z+ gene. The correct choice is D.

Passage VII (37 - 43)


37.

Enhancers and Gene Expression

B is correct. Some regions of DNA that lack a nucleosome (histone protein complex) can be detected by treating
cell nuclei with trace amounts of DNase I. At low concentrations, the nuclease digests long stretches of nucleosomefree DNA, but not the short stretches of linker DNA between nucleosomes. Many of the DNase-hypersensitive sites
in cell chromatin are located in the regulatory regions of a gene, and more of these sites are present in cells where
the gene is active than in other cells. Most such sites are believed to represent regions from which a nucleosome has
been displaced by sequence-specific DNA-binding proteins involved in eukaryotic gene regulation. Knowing this
information, it is most likely that gene-regulatory proteins found only in erythrocytes bind to the enhancer and cause
displacement of the nucleosome.

Consider the other answers. All cells have the same genome, so the argument that only the RBC has the enhancer
does not work. Therefore, we can eliminate choice A. Choice C is simply a false statement. Just because the cell will
eventually shed its nucleus does not mean that the biology of an intact nucleus can be ignored. Finally, it is not true
that gene regulatory proteins in other cells bind only to promoters and not to enhancers. Most likely, there are generegulatory proteins that bind to both regulatory elements. The point here is that only in red blood cells do we find the
particular gene-regulatory protein that binds to the enhancer and causes displacement of the nucleosome. The
correct choice is B.

38.

C is correct If we are looking for proteins that can bind to DNA, the amino acids that make up those proteins
should carry a positive charge to create an electrostatic attraction. We are therefore looking for basic amino acids,
because they carry positive charges at physiological pH. The two basic amino acids in our choice of answers are

lysine and arginine. Therefore, we should see a high proportion of these amino acids in histone proteins. The
correct choice is C.

Copyright by The Berkeley Review

477

The Berkeley Review

Specializing in MCAT Preparation

Biology
39.

Expression of Genetic Information

Section X Answers

Bis correct. This problem requires a proper interpretation of Figure 1. We are looking at a sequence of 108
nucleotide pairs. The figure tells us that we make 27 mutant P-globin enhancers. Therefore, a block of four
nucleotide pairs is changed in each mutant. The correct choice is B.

40.

Dis correct. Looking at the figures, one should be able to deduce the following: We are inserting amutant p-globin

enhancer in a test plasmid. The oligonucleotide and the cloning site are joined by the enzyme DNA ligase. The

enzyme produced after placing the plasmid inside a leaky erythrocyte is the bacterial enzyme chloramphenicol
acetyl transferase, which is easily assayed. One should realize that we are not measuring the rates of RNA synthesis
directly. We are measuring transcription in an indirect way. Therefore, we can eliminate choices Aand B. Also, we
are not producing the p-globin protein; we are using the mutated p-globin enhancer and producing another protein
(CAT), which is assayed. We can thus eliminate choice C. We are indirectly testing the mutant enhancers for their
effect on RNA transcription by measuring the amount of protein that the recombinant gene produces (as CAT
enzymeactivity). The correct choice is D.

41.

C iscorrect. We are told that a newly defined regulatory element from a vertebrate gene is analyzed, and that many

of the proteins that bind to it (gene-regulatory proteins) turn out to be previously described. This can suggest only

that there is a relatively small number ofgene-regulatory proteins, and that they may control transcription in higher
eukaryotes. Some ofthese gene-regulatory proteins are AP2, ATF, SP1, and NFL None ofthe other conclusions can
be derived from the claim made in the question. All of the other choices are general conclusions believed to be true.
Nonetheless, the information in the question would support only the inference drawn in choice C. The correct
choice is C.
42.

B is correct. The major difference between protein and steroid hormones is that protein hormones bind to receptors
on the surface of the cell and bring about cellular change via secondary messengers. Therefore, we can eliminate
choices C and D. Steroid hormones, which are quite lipid-soluble, have the ability to traverse the lipid bilayer. When

they do, they bind to a receptor molecule located in the cell. The receptor molecule has a region for binding the
steroid itself and a region for binding a DNA molecule. When the steroid binds to the receptor to make a steroid

receptor complex, the complex becomes active and binds to a region of the DNA known as a regulatory element.

The steroid itselfcannot bind DNA, because the DNA-binding region is on the receptor. The correct choice is B.
43.

C is correct. One must look at Figure 4 and interpret the data carefully. The y-axis is percent enhancer activity. Let's

look at the protein NFl-like. If we mutate the region where NFl-like proteins bind, we see 100 percent enhancer
activity. This means that NFl-like proteins do not play a part in this enhancer's activity. In other words, we took
away NFl-like activity and nothing changed. We can conclude that this protein does not make a large contribution
to the enhancer's activity. Therefore, we are looking for mutations that resulted in a low percentage of enhancer

activity. This would indicate that these proteins make a large contribution to the enhancer's activity. From Figure 4,
we see that API-like, AP2-like, and Eryfl proteins do make a large contribution. We know this because when we

mutate the region where these proteins normally bind, the enhancer activity goes way down. The correct choice is
C.

Passage VIII (44 - 50)


44.

Rereplication Block and the Cell Cycle

C is correct. We fused the two cells together, and we found that the GI phase cell nucleus began to replicate DNA.

Therefore, we are talking about a transition from GI to S, and not from S to GI. The cell cycle goes from GI to S.
Based on this information, we can eliminate two of the choices. Next, we ask whether this activator is going to be
diffusible or not. If the molecule is not diffusible, it would most likely be bound to the DNA of the S phase cell. In

that case, it could not act upon the DNA of the GI nucleus. The activator must be diffusible, because it can set the
G2 nucleus into an S-phase activity, namely DNA replication. The correct choice is C.
45.

D is correct. Let us first decide whether the replication is synchronous or not. The passage tells us that during the S

phase, some parts of a chromosome will not yet have been replicated, while others are finished. This describes
something asynchronous. Furthermore, we are talking about a eukaryotic cell that is known to contain multiple
origins ofreplication because ofthe volume ofgenetic material found in its chromosomes. The correct choice is D.
46.

B is correct. One can assume that the replication origin for a frog looks different than one for a bacterium. However,

the bacterial DNA is still subject to a rereplication block. We know nothing of how this block is achieved, so
choices C and D can be eliminated. Nevertheless, since the rereplication block affects the DNA of the bacterium,
onecan assume that the mechanism does not require a specific origin of replication. The correct choice is B.

Copyright by The Berkeley Review

478

The Berkeley Review

Specializing in MCAT Preparation

Biology
47.

Expression of Genetic Information

Section X Answers

C is correct First, we have to decide whether the G2 nucleus is stimulated or prevented from performing DNA
replication. From the passage, we are told that this fusion does not result in the rereplication of the G2 nucleus.
Therefore, we can eliminate answers claiming that the G2 cell is stimulated to begin DNA replication. The next
question becomes whether the molecule responsible for the prevention is bound or diffusible. If the molecule were

diffusible, it would seem logical that the S phase cell would cease DNA replication. The passage tells us differently,
however, so we can infer that the molecule is bound to the DNA. The correct choice is C.

48.

D is correct.Look at Figure 1. In the G2 phase, the proteins are not on the genetic material. In the GI phase, they
are. The only phase in between those two is mitosis. This makes sense. We want to be able to initiate DNA synthesis
again only after mitosis is complete. The two new daughter cells will go on to divide. The rereplication block must
be removed at or near the time of mitosis. The correct choice is D.

49.

D is correct. This question is very straightforward. We are told that one chromosome has been replicated through
ten cycles without separation, so how many strands of chromatin are now side by side? One chromosome replicating
makes two strands of chromatin. Two strands of chromatin replicating makes four strands, and so forth. Two to the
tenth power is 1024. The correct choice is D.

50.

D is correct. The protein bound to the DNA in the model must get there somehow. Recall that in eukaryotic cells,
transcription and translation are separated by a nuclear membrane. The mRNA transcript is translated in the cytosol.
In order for the protein to reenter the nucleus, it must cross back over the nuclear membrane. This is not

accomplished by simple diffusion. Recall that the nuclear membrane contains nuclear pores that allow proteins to
enter the nucleus. The pore recognizes the protein as belonging in the nucleus by means of the protein's nuclear
import signal, and the protein is then transported into the nucleus. The correct choice is D.

Passage IX (51 - 58)


51.

northern and Southern Blotting

C is correct. Albumin is a plasma protein found in an average concentration of 4 gm/dl. Albumin provides the
critical osmotic pressure that regulates the passage of water and diffusible solutes through the capillaries. When the
concentration of albumin is severely reduced, excess extracellular fluid may accumulate in the extravascular tissues.
This condition is known as edema. Albumin can also act as a carrier protein for things like bilirubin and fatty acids.
Based on this information, one can eliminate the other possible answers. Albumin is not likely to be found in the
cytosol of white or red blood cells. Furthermore, there is no evidence that the protein albumin would not be found in
the cerebral spinal fluid. The correct choice is C.

52.

D is correct. The cells that produce albumin are the hepatic parenchymal cells. Thus, the cells that produce albumin
are found inside the liver. This is a very straightforward question that tests previously acquired knowledge not
found in the passage. The correct choice is D.

53.

B is correct. This answer comes from an understanding of the goal of our experiment. We want to amplify the
nucleic acids of these liver cells so we can do further experimentation. Therefore, it is important for us to protect
those molecules we want to use later. We can protect the nucleic acids by inactivating the nucleases. Consider the
other answers. Would we want to inactivate the proteases? It probably does not matter. We are going to do phenol
extractions to remove protein. Furthermore, we are not interested in the protein, just the nucleic acids. Therefore, we
can eliminate choice A. Again, we are not interested in inactivating lipases, as long as the nucleic acids are not
threatened. Therefore, we can eliminate choice C. Finally, human cells do not have cellulases (enzymes able to
degrade the molecule cellulose), so choice D cannot be right. Therefore, the protection of the nucleic acids is central
to our experiment, and thus inactivation of the nucleases is fundamental. The correct choice is B.

54.

B is correct. The RNA from the normal mouse cells is used as an experimental control. For example, when we run a
gel of the two RNAs (one from defective cells and one from wild-type cells) we have a means of comparison, or a
control. Let's consider the other possibilities. Choice A is silly. We do not need the RNA to test the equipment. Both
choices C and D hinge on the word "directly." We cannot use this normal RNA to measure the size of the mutant
RNA directly. We will be able to say whether one is bigger or smaller from the results of the gel, but the actual size
determination comes from running standards (pieces of RNA whose size is known already) on the gel as well. The
same holds true for the amount. The normal amount cannot be used to measure the amount of mutant RNA directly.
(Although, if we so desired, we could measure the amount of mutant RNA directly.) The correct choice is B.

55.

C is correct. From the passage, it can be determined that the wash is done before the autoradiography. From this
alone, one can eliminate choices B and D, because they claim the wash is done after autoradiography. This of course

Copyright by The Berkeley Review

479

The Berkeley Review


Specializing in MCAT Preparation

Biology

Expression of Genetic Information

Section X Answers

would make no sense, because it is the radiolabel that makes the autoradiography process possible. If we washed

away all the radiolabel after the autoradiography, then we would see no autoradiography results. If we washed the

background away after the autoradiography, our results would not be as accurate as they could be. This leaves us
with choices Aand C. Do we wash all the radiolabel away? Ofcourse not. This would give us no results. We wash
the background away only to get better results. Those radiolabeled pieces of DNA that do not really belong (i.e.,

they are not truly hybridizing to the RNA) but they still manage to bind to something anyway should be washed

away. The correct choice is C.

56.

D is correct. From the picture, one can see that the RNA from the defective cell ran farther down on the gel. This
indicates that the protein is indeed smaller. Therefore, based on this information alone, one can eliminate choices B
and C, because they claim the opposite case. In choice A, we are told the defective protein is smaller, but the defect
is explained as arising from a mutation in the RNA. This does not hold. Mutations in RNA come from sloppy
transcription and should not be permanent. It is not right to say that mutant DNA gave rise to mutant RNA. The
faulty RNA is just the transcript of the faulty DNA. In other words, the mutation is in the DNA, the information
molecule. The correct choice is D.

57.

B is correct. The map should show that the restriction nuclease B cuts 2kb to the right of the left end of the
molecule. In addition, the restriction nuclease A cuts 3kb to the left of the right end of the molecule. This is the only

combination that fits with the given data. If we switch the sites (meaning A is where B is), the data would not fit,
because cutting with A gives a piece of DNA that is 2kb long and one that is 8kb long. The correct choice is B.

58.

B is correct. In order for both probes to react, the child must be a heterozygous. The heterozygous child will be in

possession ofa normal and a defective copy ofthe gene. So understanding this, we are really only looking for the
probability of the child to be heterozygous for the trait. In running the cross, one would find that there is a 50%

chance that this is true. The correct choice is B.

Restriction Enzymes

Passage X (59 - 65)


59.

D is correct. Restriction fragments, in general, are 44 = 256 base pairs long. Divide 10,000 by 256 to get

approximately 39. Choice C is off by a factor of 10 and is incorrect. Choices Aand B are other math mistakes, and
they are incorrect. The correct choice is D.

60.

A iscorrect. Approximately 99% of human DNA isidentical among all human individuals. This is highly conserved
DNA, but not completely conserved. Choices Band C are thus incorrect. Only 1% of DNA differs dramatically from
person to person, sochoice D is incorrect, too. Thecorrectchoice is A.

61.

B is correct The probable function of restriction enzymes is defense. The bacterial cell can inactivate foreign,
invading DNA (perhaps from viruses) before the invader has a chance to grow and lyse the bacterial cell or to

incorporate itself into the bacterial genome. (Neither of these possibilities is particularly desirable for the host.)

Choices A, C, and D are incorrect. The correct choice is B.

62.

D is correct. Child C has two copies of normal HbA, so this child does not have sickle-cell anemia. It is a

homozygote for HbA, thus choice A is true. Both parents carry one copy of the gene for HbA and one copy ofthe
gene for HbS. This means they are heterozygotes, so choice B is also true. Child D has 2 copies ofthe sickle cell
anemia gene, and therefore has sickle-cell anemia: Choice C is correct. Confirming that choice D is the false (and
therefore, the best) answer requires applying our knowledge of standard genetics to the heterozygous parents. AS x
AS yields 25% AA, 50% AS, and 25% SS. This gives a 25% chance ofthese parents having a child with sickle-cell
anemia. Choice D is false. The correct choice is D.

63.

C is correct. The children have to carry two copies each of the gene, one on each of their homologous
chromosomes. Statement I is thus incorrect. Unlike the parents, the children are homozygotes, having two identical

copies of the gene. C is homozygous for HbA, and D is homozygous for HbS. Therefore, statement II is correct.
There isonly one spot in the lane on the gel, since the two identical copies migrated to the same spot. On a real gel,
the spots for the homozygous child should be twice as dense as those single ones for the homozygous parents.
Statement III is correct. The correct choice is C.

64.

A is correct. Amniocentesis is a common procedure performed on some pregnant women. A very long needle is

inserted through the abdomen and the uterus to collect samples ofamniotic fluid. Cells from the fetus slough off into
the fluid, and their DNA can be studied after cleavage with restriction enzymes. Fetal cells are not routinely moving

Copyright by The Berkeley Review

480

The Berkeley Review

Specializing in MCAT Preparation

Biology

Expression of Genetic Information

Section X Answers

around in the mother's blood supply, and it would be difficult to separate the two for analysis if this were the case,
so choice B is incorrect. An abortion would not help an in utero diagnosis. ChoiceC is incorrect. A Cesarean section
is used for some deliveries, not for tissue collection. Choice D is incorrect. The correct choice is A.
65.

D is correct. Since DNA is long and fragile, it is usually chopped into small fragments for study and sequencing
with restriction enzymes, so statement I is correct. Although parents and offspring have different DNA, there are

plenty of similarities in the variable regions that could be used to establish paternity along with other techniques.
That means statement II is also correct. So is statement III: Parents who have a familial history of genetic disease
can be screened for major, identified genetic diseases to provide a better understanding of what problems their
children may face. The correct choice is D.

Passage XI (66 - 72)


66.

Arabinose Operon

D is correct. The passage tells us that the production of the C protein is under the control of the araOi operator.
Furthermore, the passage states that if the level of C protein is high, the protein binds to the araOi operatorsite and
inhibits transcription of the ara C gene. In other words, protein C is regulating its own production. Therefore, the
synthesis is autoregulated. The correct choice is D.

67.

C is correct. According to the passage, transcription of the structural genes takes place when the hairpin loop is not
formed. The passage also explains that the loop is not formed because arabinose bound to the C protein alters its
conformation and the C protein binds to both the aral and araOi site. This, along with changes in the cAMP-CAP
complex, brings about transcription of structural genes. It is clear from this information that the C protein is
necessary for transcription. Therefore, a organism lack the araC gene would exhibit inefficient transcription of
structural genes ara B, A, and D. The correct choice is C.

68.

B is correct. One can see from the drawings that accompany the question that the enzyme catalyzed a reaction in
which the structure but not the atomic composition of the substrate has been altered by moving a group from one
position to another within the same molecule. This is describing the work of an isomerase. The correct choice is B.

69.

B is correct. We know from the passage that a hairpin loop of DNA can be formed that prevents the transcription of
the structural genes. The hairpin loop intimately involved the ara O2 site, which is certainly not touching the
structural genes on either side of it. Therefore, DNA-regulatory sites need not be directly in contact with the genes
that they regulate. The correct choice is B.

70.

D is correct. This answer must come from one's previous knowledge of operons. The key is to realize that bacteria
prefer to use glucose as their fuel source, but they can use a whole host of other energy molecules when glucose is
not around. Therefore, the bacteria need some signal to denote when glucose is abundant or scarce. That signal is
cAMP. Glucose lowers the concentration of cAMP in E. coli. Therefore, we can assume that high levels of cellular
cAMP are associated with low levels of glucose. The correct choice is D.

71.

A is correct. The passage tells us that the arabinose operon is found in bacterial systems. Bacteria have only one
molecule involved in transcription, RNA polymerase. Eukaryotic organisms, on the other hand, have three types of
RNA polymerases: RNA pol I (rRNA), RNA pol II (mRNA and hnRNA), and RNA pol III (tRNA and rRNA).
Therefore, the polymerase is RNA polymerase. The correct choice is A.

72.

C is correct. If the genes for the uptake and usage of arabinose were all part of the same operon, the entire operon
would have to be transcribed before the uptake could occur. However, if the genes were separate, transcription of the
uptake and structural genes could occur simultaneously. In this way, as soon as the arabinose entered the organism,
there would be enzymes already present to begin its metabolism. This is certainly a more efficient means of
metabolism. The correct choice is C.

Passage XII (73 - 79)


73.

Gene Therapy Strategies

B is correct. The question asks which problem could not be treated using either of the two gene therapy strategies
described in the passage. The first paragraph of the passage provides us with an important clue: Gene therapy is used
to prevent the expression of both disease-causing human genes and viral genes. The aim of both the gene therapy
strategies is somehow to inactivate a dangerous gene. These approaches would be successful in preventing the
overexpression of a tumor-causing oncogene (eliminating choice A). They would also be effective at preventing the
translation of viral RNAs into DNA (thereby eliminating choices C and D). We are left with answer choice B. In

Copyright by The Berkeley Review

481

The Berkeley Review


Specializing in MCAT Preparation

Biology

Expression of Genetic Information

Section x Answers

sickle-cell anemia, both copies ofthe vital gene coding for a hemoglobin subunit are mutated. The result isa defect

in erythrocyte function. The types ofgene therapy mentioned in the passage could not help this condition, because

74.

inactivation of the mutant hemoglobin gene would only make matters worse. Instead ofdefective hemoglobin, there
would be no hemoglobin! This would not bode well for asickle-cell patient. The correct choice is B.
C iscorrect The hardest part about this question is understanding what itis really asking. Ifa plasmid isintroduced
into a cell line, how could it inactivate expression of a gene that is present in the cell's chromosomes? From the

passage, we learn about the antisense approach to inactivating genes. This question, in essence, is asking how one
might design a plasmid that would encode an antisense RNA that could bind to the sense RNA produced by
transcription ofthe chromosomal gene. Answer choices Aand Bboth mention plasmids that contain mutant gene
sequences. This would be detrimental if we were trying to knock out the expression ofa wild-type chromosomal
gene, because mutant RNA does not form hybrids with the wild-type transcript. Answer choice Dcan be eliminated,
because a gene sequence separated from its promoter is not transcribed. Recall, a promoter sequence lies just
upstream ofthe transcribed portion ofthe gene. Itattracts RNA polymerase and helps initiate transcription. Ifthe
gene sequence is separated from the promoter sequence, the gene might not be transcribed. Even if the wild-type
gene sequence on the plasmid were transcribed, this would not knock out the chromosomal gene's RNA transcript.
We are left with answer choice C. Remember that RNA polymerase can transcribe only in the 5' -> 3' direction. If

we invert the gene sequence on the plasmid relative to its promoter sequence, the RNA polymerase can no longer
read the correct strand of DNA, because this strand is now facing in the 3' ->5' direction. Instead, it reads the

75.

opposite strand, resulting in the production ofantisense mRNA. This antisense transcript can then hybridize with the
sense transcript made by the chromosomal gene, thereby preventing its expression. The correct choice isC.
Biscorrect The question asks which statement would not represent an advantage that ribozyme therapy would hold
over antisense therapy. We are asked to compare the two techniques. Statement I may be a true statement in general,
but it does not represent an advantage of ribozyme therapy, because the antisense approach doesn't use synthetic
RNA~it uses synthetic DNA. Ribozymes are base-paired RNA, but the statement doesn't say that they are any
stabler than the synthetic single-stranded DNA used by the antisense approach. Statement III is also incorrect,
because if individual ribozymes were very broad in their substrate specificity, they would cleave a lot of mRNAs in
the cell, not just the disease-causing ones. This could be harmful and certainly doesn't represent an advantage of
ribozyme therapy. Statement IIiscorrect. Ribozymes act catalytically. We know this because the passage states that
ribozymes act "enzymatically." Enzymes are catalysts; they can be recycled after speeding up a reaction. In what
regard is this an advantage that ribozymes hold over antisense DNA? Antisense DNA is not a catalyst. Once it
hybridizes to its target, it is stuck there for good. It might take many antisense DNA molecules to inactivate a viral
gene, for example, while it might take only a few catalytic ribozymes to accomplish the same task. This is an
advantage for ribozymal therapy. The correct choiceis B.

76.

A is correct The Km of an enzyme is a measure of thatenzyme's affinity for its substrate. It is equal to the amount
of substrate necessary for the catalyzed reaction to proceed at halfof its maximal rate, or 1/2 Vmax. The question
asks for the condition that would not affect the Km . Answer choice A mentions altering the concentration of
substrate. Recall from the Michaelis-Menten curve that changing the substrate concentration has no affect on the Km

of an enzyme. Therefore answer choice A is correct. What factors would affect the affinity of an enzyme for its
substrate? One possibility is the partial or total denaturation of the enzyme. An enzyme can be denatured when its
tertiary (folded) structure is disrupted. In this question, the enzyme we are dealing with is a ribozyme. The tertiary
structure of a ribozyme is dictated fundamentally by the way it base-pairs with itself, much in the same way as a

protein's tertiary structure is dictated by interactions between amino acid residues and the water environment. What
factors could affect base-pairing in the RNA ribozyme? A drastic change in pH could alter hydrogen-bonding
between base-pairs. A decrease in the salt concentration of the medium would mean that the phosphate backbone of
the base-paired RNA would be less stabilized by ionic interactions, leading to denaturation. An increase in
temperature could melt the RNA hybrid, thereby denaturing it. These possibilities prompt us toeliminate choices B,
C, and D. The correct choice is A.

77.

D is correct From the passage, we learn that ribozymes cleave mRNAs at specific sequences. If a ribozyme could
cleave an HIV mRNA at a specific, conserved sequence, such a ribozyme would be capable of chopping up most
mutant forms of the viral RNA as well. This is because a conserved sequence by its definition doesn't change much

from mutant to mutant. It may encode a very essential part of a protein product that can't bealtered. Therefore, even
if the HIV virus mutates, theribozyme could still recognize theconserved sequence and cleave the mRNA, thereby

preventing production of mature viral particles. Antisense DNA, on the other hand, must be complementary to the
mRNA to which it hybridizes. If HIV mutates rapidly, a predesigned antisense strand would not be able to hybridize

Copyright by The Berkeley Review

482

The Berkeley Review

Specializing in MCAT Preparation

Biology

Expression of Genetic Information

Section X Answers

with new mutant mRNA. This eliminates choices A and B. Choice C is incorrect, because if ribozymes cleaved all
mRNAs, they wouldn't be much use as a therapy at all. The correct choice is D.

78.

B is correct. This is the only answer that mentions a problem that could occur only in human patients and not in a
tissue culture of human cells. Human immune response to the synthetic DNA (containing methylphosphonate) could
be a fatal side effect to some kinds of antisense gene therapy. Answer choice A is incorrect, because it describes
something that could occur both in the human patient and in the tissue culture. Answer choices C and D are incorrect
for the same reason. The correct choice is B.

79.

A is correct. Thisquestion can be answered through the process of elimination. We must bear in mind what exactly
ribozymes are capable of, and with this knowledge we can rule out functions that ribozymes could never have in
normal cells. Answer choice B is such an example. From the passage we learn that ribozymes cleave RNA at
specific sites. Nowhere are we told that it can proofread DNA. This is the function of DNA polymerase. Answer
choice C is likewise incorrect: Ribozymes do not catalyze protein reactions. Answer choice D is wrong, because
ribozymes can't cleave DNA at all; they can only cleave RNA. This leaves us with answer choice A. In order to

splice mRNA and remove introns, the mRNA strand must be cleaved at a specificsite. Ribozymes are ideal for this
function, and in fact, this is how they were initially discovered. The correct choice is A.

Passage XIII (80 - 87)


80.

Tryptophan Operon

C is correct. Answering this question correctly just requires one to be familiar with the concept of a mRNA that is
polycistronic. The answer cannot be gathered from information in the passage. An mRNA molecule coding for more
than one protein is known as a polycistronic transcript. Let's consider the other possibilities: Since one protein is not
encoded for by multiple genes, choice A can easily be eliminated. Choice B wants us to believe that one protein is
coded for by a single gene. This statement is true, but it is does not pertain to polycistronic transcripts. The
definition is the relationship between mRNA and proteins, not DNA and proteins. Therefore, choice B can be
eliminated. Finally, choice D tells us that one mRNA molecule codes for only one protein. While this is not the
definition of polycistronic, this method of translation occurs in all eukaryotic organisms. The polycistronic transcript
is not seen in eukaryotic organisms. The correct choice is C.

81.

C is correct From the passage, we learn that the repressor protein alone does not cause the repression of the
tryptophan operon. The presence of tryptophan in a complex with the repressor protein brings about the true
repression. Therefore, tryptophan is best described as a corepressor. Let's consider the other possibilities: The term
inducible repressor is nonsense. It certainly has no application to the tryptophan operon and should be easily
recognized as an incorrect answer. The only example that comes to mind that resembles an inducible repressor is the
situation where glucose in a medium causes a reduction in the synthesis of the enzymes needed to utilize lactose.
This occurs in the lactose operon, and the process is termed "catabolite repression." Nevertheless, choice A does not
best describe tryptophan, and therefore choice A can be eliminated. Eliminating choice B should be easy, because it
is very clear that the presence of tryptophan causes a reduction in the expression of the operon. Therefore,
tryptophan is not acting as an inducer. In considering choice D, one needs only to consider what an attenuator is. It
is a sequence of DNA that offers a means of regulation through differential transcription. This is not the role of
tryptophan, and therefore choice D can be eliminated. The correct choice is C.

82.

B is correct. This answer can be arrived at by looking at the diagram. It is clear from the question that the
attenuatorsite along with the leader sequence has been deleted from the operon. The passage tells us that these DNA
sites are involved in attenuation. Attenuation (along with the repressor-tryptophan complex) regulates tryptophan
synthesis in such a way that the operon is expressed only when tryptophan is needed. If one sees a dramatic increase
in the production of tryptophan mRNA, one can assume that this experiment provides evidence for the role of
attenuation in regulating the synthesis of tryptophan mRNA. Let's consider the other possibilities: The experiment
tells us nothing about signal transduction (the process of extracellular signals resulting in intracellular changes).
Therefore, we can eliminate choice A. Furthermore, this experiment provides no evidence for the tryptophanrepressor complex repression. The operator/promoter is still intact, as nothing is stated about the repressor protein.
Therefore, choice C can be eliminated. Finally, tryptophan induction is not a term defined in the passage, so the
experiment probably does not have anything to do with it. In fact, the term has no set definition, and this answer
choice can be easily eliminated. The correct choice is B.

83.

B is correct The tryptophan-repressor protein binds to the operator and prevents transcription. Therefore, it
prevents the interaction not between DNA polymerase and DNA (choice A), but between RNA polymerase and

Copyright by The Berkeley Review

483

The Berkeley Review

Specializing in MCAT Preparation

BiolOfly

Expression of Genetic Information

section XAnswers

DNA. So the question becomes: Which RNA polymerase? The tryptophan operon is found in E. coli, a bacterium.
Bacteria have only one type of RNA polymerase, known simply as RNA polymerase. Choice B is the best answer.
Eukaryotes, on the other hand, have more than one type of RNA polymerase. In fact, they have three types. The
correct choice is B.

84.

D is correct. During attenuation, it is important for the leader peptide to be translated. That is how the system
detects the level of tryptophan in the cell. If the codon UGG is there, but there is a lack of tryptophanyl tRNA, this
causes a pause and a conformational change in the RNA polymerase. That change results in the RNA polymerase
continuing to transcribe the operon. If there isno pause, the RNA polymerase falls offthe DNA. Therefore, one can
see that it is important that transcription and translation be closely coupled. Based on this information, the other
answers can be easily eliminated. One can see that replication of DNA has little to do with attenuation, but
attenuation regulates the cellular levels of tryptophan. The correct choice is D.

85.

D is correct. The attenuator site, according to the passage, contains a G-C rich region and an A-T rich region.

Therefore, melting the piece of DNA (turning double-stranded DNA into single-stranded DNA) does not occur

abruptly at one temperature, because there is a varied nucleotide composition. The melting can be described as
discontinuous. This eliminates choices A and B. The next decision comes from knowing that it takes less energy to

melt an A-T rich region when compared to a G-C rich region. This, of course, stems from knowing that there are

two hydrogen bonds between adenine and thymine, and three hydrogen bonds between guanine and cytosine. The
correct choice is D.

86.

C is correct Let us look at the example in the passage. The leader peptide of the tryptophan operon contains

tryptophan residues, because that is how the system is designed to measure the level of the amino acid. Therefore,
the leader peptide of any operon should contain a relatively large number of residues of the kind synthesized by the
operon. The operon should not contain tandem pairs of the codon UGG, unless it is the tryptophan operon.
Therefore, choice A is incorrect. If the operon has a relatively small number of the UGG codons, it is most likely by

chance and not design. In other words, a leader peptide does not have tocontain a small number of the UGG codons.
Therefore, choice B can be eliminated. Again, to detect the level of a particular amino acid in the cell better, the

leader peptide of an operon should contain a relatively large number of the residues of the kind synthesized by the
operon. The correct choice is C.
87.

B is correct. Remember, the residues in the leader peptide are the detection system of the operon. Increasing the

number of those residues (the kind synthesized by the operon) would detect the amino acid level in the cell better.

This automatically eliminates choice A. There is no evidence for either choices C or D. Attenuation is clearly not
eliminated by increasing the number of residues in the leader peptide. In addition, the change in residues results in
more than just a change in structure, butalso a change in function. The correct choice is B.

Passage XIV (88 - 94)


88.

Translation

B is correct The passage informs us that in reticulocytes (eukaryotic cell), eIF-2 is involved in controlling the
overall rate of protein synthesis. In addition, we are told that phosphorylation of this protein reduces its activity and
thus most likely reduces the overall rate of protein synthesis. Under starving conditions, one can conclude that
protein synthesis in the cell decreases. Therefore, phosphorylation ofeIF-2 would occur. This requires the work ofa
kinase. One assumption should be made here: Yes, the passage does specifically state "reticulocytes," while the
question asks about changes in the more general term "eukaryotic cell." However, we must use what information the
passage gives us, as this type of questioning can appear on the MCAT. Therefore, we would expect increased
activity from a kinase. The correct choice is B.

89.

B is correct. We are told in the passage that roughly 1 out of every 10,000 amino acids incorporated into proteins is
inserted incorrectly. Then the question says that the average size of a protein is 400 amino acids. Therefore,
10,000/400 = 25. One out of every 25 protein molecules should contain an error. The correct choice is B.

90.

B is correct. According to the passage, a polyribosome is a single mRNA transcript that has multiple ribosomes
attached. Comparing a single ribosome to a polyribosome, we can clearly see a difference in size, with the

polyribosome being larger. We know that centrifugation takes advantage of this situation, based on the fact that
large molecules move faster than smaller ones in a gravitational field. Therefore, centrifugation would allow us to

readily separate these two kinds of ribosomes in solution readily. The correct choice is B.

Copyright by The Berkeley Review

484

The Berkeley Review

Specializing in MCAT Preparation

Biology
91.

Expression of Genetic Information

Section X Answers

D is correct. This question must be answered from a previous knowledge base. It is asking what amino acid is called

for by the start codon in a prokaryotic cell. The answer is a modified methionine. In particular, a formyl group is
attached to make a formylmethionine. In a eukaryotic cell, the start codon calls for methionine. Since the question
asks for the prokaryotic cell, formylmethionine is correct. The correct choice is D.

92.

C is correct. This question requires previous knowledge. A polycistronic transcript contains multiple start codons.
Each start codon results in the translation of a particular protein, each with a unique structure and function. This is in
contrast to a monocistronic transcript, which contains only one recognized start codon, which gives rise to only one
peptide chain. Polycistronic transcripts are often found in prokaryotic cells, while monocistronic transcripts are often
found in eukaryotic cells. The correct choice is C.

93.

C is correct. A proofreading mechanism implies preventing and/or removing an incorrect unit so that a correct unit
can be inserted. Therefore, our answer should address this issue. The only choice that satisfies this criterion is

choice C. Choice A is not the best answer, as it implies that time alone assures that the correct aminoacyl transfer
RNA eventually will be inserted. There is no explanation about any proofreading function. Indeed, there are
hydrogen-bond interactions between the bases in the codon and the anticodon. If there is an incorrect bonding, the
delay makes it very difficult for the incorrect pairing to hold, as such a situation is inherently unstable. The delay
allows for the dissociation of this unstable complex, and the correct aminoacyl transfer RNA can then be
incorporated. The correct choice is C.

94.

B is correct Choice B claims that no initiating factors are needed for secreted proteins. Recall that under the signal
peptide hypothesis, a protein that is to be secreted begins its translation in the cytosol and then is attached to the
endoplasmic reticulum to continue and finish its translation. The key point here is that translation begins in the
cytosol, just like a protein that is not to be secreted. This requires formation of a competent ribosome, which
according to the passage, involves initiating factors. Therefore, we cannot make the claim that initiating factors are
not needed, because they clearly are. The correct choice is B.

Passage XV (95 - 100)


95.

Cancerous Gene

C is correct. The passage states that in theory, one could compare the base sequences of a normal cell and a
cancerous cell. In theory, the differences one would encounter could be attributed to cancer. However, this is not the
case. Even if it were practical to sequence both genomes, we could not ignore the fact that mutations naturally occur.

Therefore, there would be more than just one difference, and one probably could not determine which mutation was
the cause of cancer. Therefore, naturally occurring mutations would mask the cancer-causing gene. The correct
choice is C.

96.

C is correct. In the passage, it is clearly stated that the screening process involves using a sequence of DNA
complementary to the tagging sequence. We want this probe to hybridize only with the tagging region (which is
hopefully attached to our mutated cancer gene), and not with other pieces of DNA. If we used a tag sequence that
was normally found in E. coli, the probe would hybridize to regions of DNA other than the one we are interested in.
This would show up as many false positives, and our isolation technique would be unsuccessful. The correct choice
is C.

97.

C is correct The tagging sequence is hopefully near the cancer-causing mutated gene we are interested in isolating.
From the passage, we are told that the Alu site is found throughout the entire human genome. This is a very
attractive trait as a tag, because we are betting that because the site is so widespread throughout the chromosomes, it
has a solid chance of being near our mutated gene of interest. Therefore, when we clone and screen, we will
hopefully include this mutated gene. The correct choice is C.

98.

C is correct. This question is straightforward. We know from the question that gene A is known to induce cancer.
Also, from the information in the question, we can conclude that there is nothing wrong with the transcription of the
gene or the efficiency of translating the messenger RNA. Therefore, we should conclude that even though the
protein is synthesized, it is useless. Recall that protein function is tied into its structure. Therefore, a nonfunctional
protein probably suffers from a significant structural change that resulted from the gene mutation. The correct
choice is C.

Copyright by The Berkeley Review

485

The Berkeley Review

Specializing in MCAT Preparation

BiolOflV
99.

Expression of Genetic Information

Section x Answers

B iscorrect Two strands ofDNA hybridize because they have complementary bases. Recall, that bases bind toeach

other because of hydrogen bonds. Adenine and thymine share two hydrogen bonds, while guanine and cytosine
share three hydrogen bonds. Therefore, the hybridization occurs because base pairs do form hydrogen bonds with
each other and create a stable structure. Without the hydrogen-bond interaction, the two strands would not anneal.
The correct choice is B.

100. A is correct This question must be answered from a previous knowledge base. A Southern transfer is as follows:
Using electrophoresis, DNA fragments are distinguished on the basis oftheir size. The fragments are transferred to
some sort of membrane and are immobilized. Under hybridizing conditions, labeled oligonucleotides are added and

complementary bases form pairs. The label can then be found with autoradiography. Northern transfers involve
RNA, while Western transfers involve proteins. Although the molecules are different for these experimental
methods, theconcepts are very much the same. The correct choiceis A.

Copyright by The Berkeley Review

486

The Berkeley Review

Specializing in NCAT Preparation

Molecular Cell

Biology
Diagnostic Passages
and Questions
Set I

"The

Berkeley
Specializing in MCAT Preparation

DIRECTIONS: Adescriptive passage precedes manyofthe questions found in the Biological


Sciences section of the exam. The questions are arranged into groups. Examine a passage

before selecting the one best answer choice to each question in the group. There will be
some questions which are independent ofa descriptive passage and independent of each
other. The one best answer choice must be selected for these questions as well. If you are
not sure of an answer, eliminate the alternativechoices that you know to be incorrect and
then choose an answer from the remaining alternatives. Indicateyour choiceby darkening
the corresponding bubbleon youranswer sheet. Aperiodic tableof the elements isprovided
for you at the end of this book,and you mayconsult it whenever you wishduring the exam.

r-t-

Q)

ft

81-4 &-

on

com

8%
3*

OJ
^c

4-

SO

to

oc

Iti

00

OJ
--J

os

Ui

OS

w
oj

u>

u>
til

en

t/l

>

U)

1ft

to

to

Ui
O

OJ
c:

4^
SO

to
^o

SO

4^
00

to
00

00

4i<I

to
-J

4^
OS

to
OS

On

4^

4^

4^

to
4-

U\

to

Ui

OJ

4-

to

Ui

tO

4^

to
to

*0

h-

4^

to

CO

rt>

00

as

&

cro

R" Q

00

0Q

r-t-

ro

en

pr.
n

crq

fD

3.

-3

fD

tu

re

en

~:

r1

Molecular Cell

Biology
Diagnostic Passages
and Questions
Set I

7 Passages with 39 Questions


13 Independent Questions
52 Questions Total

70 Minutes

fgEBKELEY
Specializing in MCAT Preparation

Passage I (Questions 1 - 5)

2.

Biological membranes have the remarkable property of


being able to selectively control what enters and leaves the
cell. In the last several decades, the mechanisms of this

control have been extensively elucidated. Transmembrane


proteins collectively called transporters mediate the passage
of many biological molecules and ions across the plasma
membrane. These transporters fall into several classes:

The Multiple Drug Resistance (MDR) protein is a


primary active transporter which pumps hydrophobic
drugs out of the cell. Which of the following
statements is NOT correct regarding the MDR
uniporter?

A.

ATP is bound and hydrolyzed by the cytoplasmic


portion of the MDR protein.

B.

The MDR protein may allow cells to survive even


when given certain toxic drugs.

I. Passive Channels

C.

These proteins form channels in the plasma membrane


which allow the passive diffusion of specific molecules or

D.

The MDR protein most likely pumps drugs along


their concentration gradient.
The MDR protein does not utilize the gradient of
another molecule to pump the drugs out of the

ions.

cell.

II. Primary Active Transporters

These proteins directly hydrolyze ATP to drive the


transport of molecules or ions across the membrane. They
can fall into one of three classes. Uniporters transport one
thing at a time in a single direction. Symporters transport
two different molecules at the same time and in the same

3.

direction across the membrane. Antiporters transport two


different molecules at the same time but in opposite

A certain passive channel allows small cations to cross


the membrane. The interior of this channel is most

likely lined with:

directions across the membrane.

A.
B.
C.
D.

III. Secondary Active Transporters

These proteins do not hydrolyze ATP directly. Instead,


they transport specific molecules or ions by utilizing the
energy stored in the concentration gradient of a second
molecule or ion. For example, certain lysosomes use a

positively charged amino acids.


negatively charged amino acids.
non-polar amino acids.
hydrophobic amino acids.

primary active transporter to pump H+ from the cytoplasm


into the vesicle, establishing a concentration gradient. They
can then use a secondary active transporter (an antiporter) to
allow H+ to leave the lysosome by flowing down its
concentration gradient, simultaneously driving the inward

4.

The sodium/potassium ATPase can best be described


as a(n):

flow of a different ion.

A.
B.
C.
D.

1.

5.

Which of the following would most likely NOT use


any of the transport mechanisms described in the
passage?

A.

Lipids

B.

Glucose

C.

Ca2+

D.

Amino acids

Copyright by The Berkeley Review

secondary active transporter.


symporter.
uniporter.
antiporter.

A non-hydrolyzable ATP analog is administered to a


cell. Which of the following transporter types would
LEAST be affected?

492

A.
B.
C.

Primary active transporters.


Secondary active transporters.
Antiporters.

D.

Passive channels.

GO ON TO THE NEXT PAGE

Passage II (Questions 6-10)

7.

The parasitic protozoan Trypanosoma brucei is


responsible for causing "sleeping sickness," an often fatal
disease which is a major public health problem in Africa.
The disease is unusual in that it is characterized by repeating
cycles consisting of parasitemia (active disease) followed by

According to information given in the passage, the


various transcriptionally silent VSG genes most likely
differ from each other in:

A.
B.
C.
D.

temporary recovery.

their amino acid sequences.


their operator sequences.
their promoter sequences.
the sequence of their protein-encoding regions.

Trypanosoma is somewhat unique in that it is capable of


periodically changing its surface coat antigenic
determinants, i.e., the parts of the parasite that the immune
system recognizes and produces antibodies against. The
main such determinant in Trypanosoma is encoded by a
series of over 100 different VSG (Variant Surface

Glycoprotein) genes. Only one of these genes is expressed at


any given time. The expressed VSG gene is located in a
particular part of the Trypanosoma genome (the expression

8.

locus). If a VSG gene is present in the expression locus, it


will be transcribed and translated (i.e., the surface antigenic
determinant encoded by that particular gene will be

Which of the following represents a valid hypothesis


for why the VSG genes are only transcribed when they
are in the expression locus?

produced). If it is anywhere else, it will remain


transcriptionally silent.

A.
B.

The locus is present in the heterochromatic region


of the Trypanosoma genome.
The locus is adjacent to a strong promoter

As a result of this system, Trypanosoma can change its

C.

The locus is at the tip of the Trypanosoma

D.

The locus is adjacent to an origin of replication

sequence.

surface antigens by simply excising the VSG gene in the


expression locus and replacing it with a different VSG gene.
The parasite therefore eludes the immune system by
"cloaking" itself in different surface antigens when it comes
under prolonged attack.

chromosome.
sequence.

9.

The variant surface glycoproteins have carbohydrate


groups attached to them:

A.
B.
C.
D.

6.

post-translationally in the golgi apparatus.


pre-translationally in the golgi apparatus.
post-translationally in lysosomes.
pre-translationally in lysosomes.

Which of the following statements is most consistent


with the information given in the passage?
A.

B.

Trypanosoma constantly changes its active VSG


genes, preventing the immune system from ever
being able to mount an antibody-mediated attack.
The immune system successfully destroys most
of the Trypanosoma organisms until a change in
VSG gene expression occurs, halting the

10.

immune attack.

D.

The immune system is unable to recognize and


make antibodies to any of the variant surface
glycoproteins.
The immune system can produce antibodies to
cytoplasmic Trypanosoma proteins other than the
variant surface glycoproteins.

Copyright by The Berkeley Review

VSG genes are placed in the expression locus via a


specialized process known as site-specific
recombination. At what stage of meiosis does more
typical chromosomal recombination (i.e., crossing
over) occur?

A.
B.
C.
D.

493

Telophase of meiosis I.
Prophase of meiosis I.
Anaphase of meiosis I.
Metaphase of meiosis II.

GO ON TO THE NEXT PAGE

13.

Questions 11 through 14 are NOT based on a


descriptive passage.

After consistent aerobic exercise, muscle cells increase


the number of their:

A.

B.

11. The high rate of mutation in the HIV retrovirus has


hindered the development of an effective AIDS
vaccine. HIV mutates more rapidly than other non-

C.

retroviruses because:

D.

hemoglobin molecules in order to meet increased


oxygen demand by the muscles.
hemoglobin molecules in order to meet increased
creatine phosphate demand by the muscles.
mitochondria in order to meet increased oxygen
demand by the muscles.
mitochondria in order to meet increased ATP

demand by the muscles.

A.

there are many mutagens present in HIV's host


cells.

B.

replication of an RNA genome is particularly


error-prone.

C.

D.

non-retroviruses are not enveloped and hence can


not carry mutagens out of the host cell.
DNA viruses do not have evolutionary pressures
to mutate.

14.

A purified solution of isolated ribosomes is stained


with two dyes. The first one appears red and stains
negatively-charged structures, while the second
appears white and stains proteins. What color does the
solution of ribosomes stain?
A.

Red

B.

White

C.
D.

Pink
Clear

12. Cholesterol, a precursor to steroid hormones, is


synthesized:
A.

B.
C.
D.

via translation of cholesterol mRNA by


ribosomes in the rough endoplasmic reticulum.
via transcription of cholesterol mRNA by
cytoplasmic ribosomes.
via a metabolic pathway containing several
protein enzymes.
by transcription and translation of a highlyconserved gene.

Copyright by The Berkeley Review

494

GO ON TO THE NEXT PAGE

Passage III (Questions 15-19)

15. In order to BEST identify which phase of the cell cycle


an individual cell is undergoing, researchers would
most likely analyze:

In typical eukaryotic organisms, there are several phases


of the cell division cycle. A period of cell growth (GI) is
followed by a period of DNA replication (S), after which
comes a second growth period (G2) and finally mitosis and
cytokinesis (M).

It is of vital interest to the cell that each phase be


completed accurately before the next phase ensues. This
completion is ensured by the cell's regulatory machinery.
Proteins such as cyclins serve to establish checkpoints
between sequential phases of the cell cycle. At these
checkpoints, the cell evaluates whether it is ready to move
on to the next phase. If not, the checkpoint proteins prevent
the cell cycle from continuing until the cell has repaired any
potentially lethal DNA damage or has grown enough to
safely undergo mitosis.

A.

the DNA content of the cell.

B.
C.

the protein content of the cell.


the lipid content of the cell.

D.

the number of mitochondria in the cell,

checkpoints.

16. Which of the following can NOT be concluded from


the graph depicting irradiated cells shown in Figure 1?
A.
B.
C.

An experiment was performed to illustrate the effect of


cell cycle checkpoints. A plate of 100 cells, each in various
asynchronous stages of the cell cycle, was cultured and an
assay was performed to determine how many cells were in
each of the individual phases of the cell cycle. This culture
was then irradiated with X-rays to disrupt DNA replication
and damage the chromosomes. These cells were then
allowed to grow for 8 hours after the irradiation. The results
of this experiment are shown in Figure 1.

D.

Irradiation has triggered the cell cycle to halt at


the checkpoint between the G2 and M phases.
Irradiation has triggered the cell cycle to halt at
the checkpoint between the G1 and S phases.
Irradiation has triggered the cell cycle to halt at
the checkpoint between the S and G2 phases.
Damaged DNA can halt the cell cycle at certain
checkpoints.

17. Which of the following is NOT an evolutionary


advantage of having checkpoints between phases of the
cell cycle?

A.
No irradiation

B.
C.
D.

g
3

The time a single-celled organism takes to


reproduce is lengthened.
Harmful DNA damage can be repaired before the
chromosomes are replicated.
Cells are prevented from undergoing mitosis if
they are too small.
Cells in multicellular organisms don't divide in an
uncontrolled manner.

GI

G2

18.
8 Hours after irradiation

Ataxia telangiectasia is a genetic disease in which cell


cycle checkpoints are absent. Which of the following
might be possible symptoms of this disease?
A.
B.
C.
D.

0)

X)

Lower-than-average cancer rate.


Massive cell death following X-ray exam.
Abnormally slow rate of cell division.
Larger-than-normal cell size.

GI

G2

19.

Figure I

In light of the information given in the passage, which


of the following represents the BEST explanation for
the lack of any cells in S phase in the irradiated culture
shown in Figure 1?

A.
B.
C.

Cells undergoing DNA replication


preferentially killed by X-rays.
Entry into mitosis is blocked.
Entry into G1 is blocked.

are

D.

Cells in S have moved into G2, and cells in GI

are blocked from entering S.


Copyright by The Berkeley Review

495

GO ON TO THE NEXT PAGE

Passage IV

21. A point mutation in a 5' splice site of a given gene

(Questions 20 - 25)

might result in:


In 1977 it was discovered that eukaryotic genes

contained segments of DNA that were not translated into


protein. Instead, these segments (called introns) were spliced
out of the gene's mRNA after transcription, leaving only
coding sequences (called exons) behind. It was later
discovered that a complex of proteins collectively referred to
as the spliceosome was responsible for splicing newly-made
mRNA in the nucleus. Unspliced mRNA is confined to the
nucleus until it is processed, after which it is exported to the
cytoplasm for translation.

A.
B.
C.
D.

22.

The spliceosome recognizes and binds to splice sites.


These sites are short, specific consensus sequences present
on each end of an intron: they signal the spliceosome to
excise the intron. Figure 1 shows a simplified splicing
scheme.

Exon

5' splice

3' splice

site

site

^>

(^

Intron

a polypeptide product with a single amino acid


substitution.

a polypeptide product of a different length.


inactivation of the gene's transcription.
hyperactivation of the gene's transcription.

The excised intron sequence depicted in Figure 1


differs from the same intron sequence present in the
genomic DNA in that the excised sequence has:
A.

uracil instead of adenine.

B.

adenine instead of uracil.

C.
D.

a 2' hydroxyl group on the ribose backbone.


no 2' hydroxyl group on the ribose backbone.

Exon

5'

]3'

23.

Spliceosome binding

Which of the following statements are consistent with


the information given in the passage?

and lariat formation

A.

Spliced mRNA is usually longer in length than


the DNA from which it is transcribed.

B.
C.
D.

5'

mRNA that does not get spliced quickly enough


is often entirely translated into protein.
mRNA can't be translated until it is fully spliced.
mRNA is spliced in the cytoplasm.

Cleavage of intron and


ligation of exons
Exon

Exon

24.

5'C

Which of the following might represent an advantage


of having a system of RNA splicing rather than simply
having continuously translated gene sequences without
introns?

Intron

A.

The total size of the genome is larger when


introns are included.

B.
Figure 1: mRNA splicing.

C.
I).

20.

According to Figure 1. the spliceosome performs


functions which are analogous to those of:

25.

RNA polymerase has to transcribe less DNA


when introns are present.
Alternate splicing pathways can provide
evolutionary flexibility.
Without introns, more ribosomes are necessary.

All of the

following would have spliceosomes

EXCEPT:

A.
B.
C.
D.

a ligase and an exonuclease.


a ligase and an endonuclease.
an RNA polymerase and a protease.
a DNA polymerase and a protease.

Copyright by The Berkeley Review

A.

496

fungi.

B.

mammals.

C.

yeast.

D.

bacteria.

GO ON TO THE NEXT PAGE

Passage V (Questions 26 - 30)

27.

Replication of linear chromosomes proceeds via a


replication fork at which DNA is synthesized off of one
parental strand continuously and off of the other parental
strand discontinuously. In both cases, DNA synthesis occurs
in the 5' to 3' direction. Discontinuous replication is
facilitated by primase, which deposits RNA primers that
allow DNA polymerase to create Okazaki fragments, which
the enzyme ligase ultimately seals together.

Somatic cells (i.e., body cells uninvolved in


reproduction) usually possess an inactivated form of
telomerase. These cells most likely:
A.
B.

are incapable of mitotic division.


are limited in the number of times they can

C.

completely lack telomeric sequences at the ends

divide.
of their chromosomes.

D.

undergo meiosis rather than mitosis.

This replication system presents a problem. The extreme


end of a linear chromosome can not be replicated because
there are not enough nucleotides on the parental strand to

serve as a template for producing RNA primers. As a result,


the chromosomes shorten during each DNA replication,
eventually resulting in lethal loss of essential genes.
This "end-replication" problem is solved by the
telomere, a series of repeating sequences at the end of the
chromosome. These repetitive sequences are periodically

28.

In cells with a defective or inactive telomerase enzyme,

shortening of the chromosomes would most likely


occur in which phase of the cell cycle?

extended by the enzyme telomerase, which carries as part of


its structure an RNA strand which recognizes telomeric

sequences and serves as a template to extend them.


Chromosome length is thereby roughly maintained, though

A.
B.
C.
D.

the number of repeating sequences in the telomere may vary.

29.

Gi phase
S phase
G2 phase
M phase

In cancerous cells, the concentration of active

telomerase is generally increased relative to normal


cells. This may contribute to which of the following
aspects of cancer cells?
A.
B.
C.
D.

26. Which of the following classes of organisms would


lack the telomerase enzyme?
A.

Tendency to mutate rapidly.


Ability to undergo unlimited mitotic divisions.
Tendency to lose chromosomal material.
Ability to metastasize throughout the body.

Bacteria

B.

Protists

C.

Fungi

D.

Insects

30.

Copyright by The Berkeley Review

497

The telomerase enzyme is made of both protein and


ribonucleic acid, a trait shared by:
A.

RNA polymerase.

B.
C.

histones.
ribosomes.

D.

tRNA.

GO ON TO THE NEXT PAGE

34. In E. coli, one would expect to find ATP synthase in:

Questions 31 through 34 are NOT based on a


descriptive passage.

31.

A strain of mice suffers from a condition known as

A.
B.

the outer mitochodrial membrane.


the inner mitochondrial membrane.

C.

the plasma membrane.

D.

the cell wall.

severe combined immunodeficiency (SCID). These


mice lack both T and B cells. Which of the following
statements are NOT valid regarding the SCID mice?
A.

C.

The mice would have no self-produced


circulating antibodies.
The mice would accept grafted tissue from
foreign species.
The mice would die more rapidly if infected by

D.

The mice would lack humeral immunity.

B.

the HIV virus.

32.

Which of the following does NOT represent a


difference between RNA and DNA?

A.
B.
C.
D.

DNA
RNA
RNA
DNA

lacks a 2' hydroxyl group.


is less prone to errors.
probably evolved first.
uses thymine.

34. The rate of gene transcription is the highest during


which phase of the cell cycle?
A.
B.
C.
D.

Copyright by The Berkeley Review

498

Interphase
Prophase
Anaphase
Telophase

GO ON TO THE NEXT PAGE

Passage VI (Questions 35 - 40)

37. According to the results of the experiment described in


the passage, which of the following statements are

The bacterium Corynebacterium diphtheriae secretes

NOT valid?

diphtheria toxin, an extremely potent peptide exotoxin which


is largely responsible for the disease symptoms of

A.

Fragment B is the portion of the diphtheria toxin


that binds to and inactivates the target elongation

B.

Fragment A is a necessary component of normal,


active diphtheria toxin.
Fragment B is a necessary component of normal,
active diphtheria toxin.
Fragment A does not directly bind to the
diphtheria cell-surface receptor.

diphtheria.

factor.

Diphtheria toxin (DT) enters cells by binding to


specific receptors and then undergoing endocytosis. The DT
then functions by specifically binding to and inactivating one
of the elongation factors responsible for allowing the transfer
of an amino acid from tRNA to a growing polypeptide chain.
The effect is very potent and only a few molecules of DT are
required to kill an entire cell.

C.
D.

38.

In order to further analyze the structure and function of


diphtheria toxin, a solution of purified DT is treated with a
reducing agent, resulting in the formation of two smaller
polypeptides, Fragment A and Fragment B. The intact,
untreated diphtheria toxin as well as each of the two
fragments are then individually labeled with a fluorescent
marker compound. Each of the labeled proteins are
separately added to cultures of human cells and allowed to

It has been shown that the gene which encodes


diphtheria toxin is actually located in a phage (viral)
genome which has been inserted into the
Corynebacterium chromosome during prior viral
infection. Which of the following statements are valid
with regards to this finding?
A.

The phage genes must contain introns and exons


in order to be properly processed by the

B.

Only bacteria that were infected with the phage


can cause diphtheria.
The phage itself can produce and secrete the
diphtheria toxin.
Inserting its DNA without killing the bacterium is
not evolutionarily advantageous for the phage.

incubate for a short period of time. The cells are then

examined using fluorescent microscopy and results are

bacterium.

shown in Table 1.

C.
Labeled Species

Location of Fluorescent Signal

D.
Fragment A

Extracellular Medium

Fragment B

Intracellular Medium

Intact DT

Intracellular Medium

39.

(Fragments A + B)

Based on the results shown in Table 1, which of the

following hypotheses is MOST valid?

Note: Cells incubated with the intact diphtheria toxin eventually died as a
result of inhibited protein synthesis. Incubation with either Fragment A or
Fragment B had no lethal effect on the cells, however.

A.

Table 1

Fragment A is the portion of the diphtheria toxin


that binds to and inactivates the target elongation
factor.

B.

35.

According to information given in the passage, an


intact DT protein most likely consists of Fragment A
and Fragment B linked together by a:

C.

A.

peptide bond.

D.

B.

disulfide bond.

C.
D.

hydrogen bond.
hydrophobic interaction.

In order for diphtheria toxin to be lethal, it must


be cleaved into its two fragments before it binds
to its target cell.
Fragment B is not needed for a functional, lethal
diphtheria toxin.
Fragment A has an alpha-helical secondary
structure.

40.

36. Diphtheria toxin is toxic to hamsters but has no effect


on rats. Which of the following best explains this

Researchers are currently trying to specifically target


diphtheria toxin against tumor cells. Which of the
following must NOT be true in order for this approach
to be successful in treating cancer?

observation?

A.
A.
B.

Rats and hamsters have different sets of tRNA

C.

Rats lack cell receptors which bind the DT


peptide.
Hamsters undergo more mRNA translation than

cells.

B.
C.

molecules.

D.

The diphtheria toxin must be made to recognize


and bind cell-surface receptors unique to cancer

Rats differ from hamsters in their protein


synthesis machinery.

D.

Cancer cells must be capable of endocytosis.


Cancer cells must not have a drastically altered
protein synthesis machinery.
The diphtheria toxin must be altered such that it
can freely cross lipid membranes.

rats.

Copyright by The Berkeley Review

499

GO ON TO THE NEXT PAGE

Passage VII (Questions 41 - 47)

41.

If the P generation cross shown in Figure 1 were


altered such that sinistral males were crossed to dextral

Certain traits are inherited and expressed in patterns


which are different from those predicted by simple

females, the Fj generation would phenotypically:

Mendelian genetics. One such trait is the direction of shell


coiling in the snail Limnaea peregra. This species of snail

A.

remain entirely sinistral.

has a shell which can coil either to the right (dextral) or to


the left (sinistral). Shell coiling direction seems to be
controlled by a single gene, of which two alleles exists: a

B.
C.

consist of 50% sinistral and 50% dextral snails.


consist of 75% dextral and 25% sinistral snails.

D.

appear entirely dextral.

dominant dextral (D) and a recessive sinistral (d).

The unusual inheritance pattern for coiling direction,


known as the maternal effect, is shown in Figure I. Since
the direction of shell coiling is determined in the early
embryonic stages, it appears that the mother's genotype
determines the distribution of factors in the ovum, leading to
a maternal pattern of inheritance.

The maternal effect should be distinguished from


extranuclear inheritance, a process whereby genes located in
organelles are inherited almost entirely from the mother (as
the ovum contains most of the cytoplasm and organelles that
will make up the zygote).
sinistral

42.

dextral

If inheritance of shell coiling were sex-linked rather


than a result of maternal effect, we would expect the F\
generation shown in Figure 1 to consist of:

P generation

A.

males which are all sinistral and females which

C.

are all dextral.


females which are all sinistral and males which
are all dextral.

D.

F| generation

entirely dextral snails.

B.

25% dextral males, 25% sinistral males, and 50%


dextral females.

D/D

D/d

D/d

d/d

Figure 1

43.

The recessive allele which is responsible for the


sinistral phenotype might:
A.

be the result of a frameshift mutation.

B.

encode a more active protein than the wild-type


allele.

C.

be present in mitochondrial DNA rather than in

D.

encode a protein which binds to and inhibits the


wild-type protein.

nuclear DNA.

Copyright by The Berkeley Review

500

GO ON TO THE NEXT PAGE

47. Which of the following statements are true with regards


44. In the F2 generation shown in Figure 1, why are the did
snails entirely dextral?

to the maternal effect pattern of inheritance?

A.
A.

The sinistral trait


heterozygous snails.

is

only expressed

B.
C.

The mother of these snails was Did.


The father of these snails was Did.

D.

The penetrance of the sinistral trait is extremely

C.

The genotype of an individual is determined by

D.

the phenotype of that individual.


The genotype of an individual is determined by
the phenotype of the mother.

the genotype of the mother.

low.

45.

B.

The phenotype of an individual is determined by


the genotype of that individual.
The phenotype of an individual is determined by

in

If shell coiling direction did not display a maternal


effect but rather were expressed as a simple recessive,
autosomal trait, what would the ratio of dextral to

sinistral snails be in the F2 generation shown in Figure


1?
A.

46.

1:1

B.

1:3

C.
D.

3:1
4:0

A defect in the mitochondrial genome causes an


inheritable disease. If a diseased female snail in the P

generation shown in Figure 1 were crossed to an


unaffected male, how would the F\ generation be
affected?
A.

0% of the males and 100% of the females would

B.

be diseased.
50% of both males and females

would be

diseased.

C.
D.

The entire F\ generation would be diseased.


There is not enough information to answer this
question.

Copyright by The Berkeley Review

501

GO ON TO THE NEXT PAGE

50. Cross-linking of cysteine residues in polypeptides


occurs mainly:

Questions 48 through 52 are NOT based on a


descriptive passage.

A.
48.

Fluorescent In Situ Hybridization (FISH) is a technique


in which a labeled DNA probe is added to a set of
metaphase chromosomes. The location of hybridization
of the probe can reveal the chromosomal location of a
gene of interest. Which of the following must NOT be
true for this technique to be successful?
A.

B.

inside the cell due to the reducing environment of


the cytoplasm.
outside of the cell due to the oxidizing
environment of the extracellular medium.

C.
D.

inside the cell due to the polar environment of the


cytoplasm.
outside of the cell due to the reducing
environment of the extracellular medium.

The metaphase chromosomes and the DNA probe


should be double-stranded.

B.

The DNA probe should be labeled with a


fluorescent molecule which can be visualized

microscopically.

C.
D.

The DNA probe should contain part of the


sequence of the gene of interest.
The DNA probe should contain a sequence that is
unique to the gene of interest.

51.

Complementary DNA (cDNA) is experimentallyproduced DNA which can encode a gene of interest.
cDNA has no introns and therefore does not need to be

spliced. Which of the following represents a valid


technique for producing cDNA for a given gene of
interest?
49.

A mutation in a highly conserved region of a gene


would have a great likelihood of:

A.
B.
C.

A.

adversely affecting an organism.


giving an evolutionary advantage to an organism.
not affecting the function of the product of the

B.
C.

gene.

D.

beneficially affecting the function of the product


of the gene.

Copyright by The Berkeley Review

D.

502

Use DNA polymerase to make a copy of the


DNA encoding the gene.
Use reverse transcriptase to make a copy of the
spliced mRNA encoding the gene.
Use DNA polymerase to make a copy of the
spliced mRNA encoding the gene.
Use RNA polymerase to make a copy of the
spliced mRNA encoding the gene.

GO ON TO THE NEXT PAGE

52.

One strand of a double-stranded DNA molecule is

labeled with P32 and allowed to undergo two rounds of


replication in vitro. What is the percentage of
radioactive DNA in the final sample?
A.

12.5%

B.

25%

C.
D.

50 %
100%

Copyright by The Berkeley Review

503

STOP, YOU ARE FINISHED

Biology

Molecular Cell Biology Diagnostic Set 1

Passage I (Questions 1-5)

Answers

Membrane Transporters

A is the best answer. Lipids are easily soluble in the lipid membrane. They can usually pass through without the aid of any
transporter. The types of transporters described in the passage are reserved for molecules that are insoluble in the lipid
membrane. Glucose is one such molecule. As a carbohydrate, it is very polar and can't directly pass through the membrane
without the help of a protein. Calcium, being a charged ion. can't pass through without help either. Amino acids are polar,
charged molecules at physiological pH and hence need transporters as well. The best answer is A.
C is the best answer. Recall that active transporters such as the MDR protein pump ions against their concentration
gradient. This is why energy is required by these transporters. Passive transporters allow ions and molecules to flow along
their concentration gradient and hence they don't require energy input. The other answer choices are correct with regards to
MDR and can be eliminated. Choice A is true because MDR is a primary active transporter and hence must hydrolyze ATP
directly. It would have to do so on the cytoplasmic side because that is where ATP mainly exists (inside the cell, not
outside). Choice B is valid because MDR pumps hydrophobic drugs out of the cell, perhaps allowing them to survive more
effectively in the presence of such toxic compounds. Choice D is correct because MDR is not a secondary active transporter.
These transporters do utilize the gradient of other molecules. MDR is a primary active transporter. The best answer is C.
B is the best answer. The channel mentioned in the question allows cations to pass through. Cations are positively charged
ions which cannot directly pass through the hydrophobic lipid bilayer because of their charge. Therefore, a channel which
allows them to pass through must have an interior lined with amino acids of negative charge. The cations can then be in an
"attractive" environment, stabilized by the negative charge as they pass through the channel. If the channel were lined with
positively charged amino acids, the cations could never enter the channel because they would be repelled by the like charge.
Hydrophobic and non-polar amino acids would not be suitable because the charge of the cations would not be stabilized.
The best answer is B.

D is the best answer. The Na+/K+ ATPase is an antiporter, transporting two potassium ions inwards and three sodium ions
outwards simultaneously; according to the passage, this fits the definition of an antiporter. It is not a symporter because
sodium and potassium are not transported in the same direction. It is not a uniporter because it transports more than one
different ion at a time. It is not a secondary active transporter because, as its name implies, it directly hydrolyzes ATP. Be
sure to carefully read the descriptions of the various types of transporters given in the passage. The best answer is D.

D is the best answer. A non-hydrolyzable ATP analog would bind to ATP-binding sites on certain transporters, effectively
preventing normal ATP from binding and eliminating the function of the affected transporters. Passive channels would be
least affected because they don't rely of ATP hydrolysis to carry out their functions (see description given in the passage).
Primary active transporters directly hydrolyze ATP and hence would be affected. Secondary active transporters don't
directly hydrolyze ATP, but they do rely on primary active transporters to establish a gradient which they can use to drive
transport; they would therefore be affected. Antiporters would probably be affected because most of them are either primary
of secondary active transporters. The best answer is D.

Passage II (Questions 6 -10)

Sleeping Sickness

B is the best answer. The passage tells us that the sleeping sickness disease is characterized by repeating cycles of active
disease and recovery. Which of the answer choices would best fit this observed characteristic of the disease? Choice B. The

cyclic periods of disease are caused by the parasite running rampant while the immune system begins to produce antibodies
to the variant surface glycoprotein which is first expressed. Since the production of antibodies takes time, the disease state
persists until the antibody levels are high enough to significantly damage the parasite population. Once such damage occurs,
the period of temporary recovery ensues. At this point, the parasite changes its surface VSG, halting the immune attack
because the previous antibodies can no longer bind the new VSG; this causes another period of active disease and the cycle
continues. We can eliminate the other answer choices because they don't fit the cyclic characteristic of the disease. Choice A
is incorrect. If the virus changed it surface antigens so fast that the immune system could never mount an antibody-mediated
attack, the cycles of temporary recovery in sleeping sickness would not be observed. We can use this same logic to
eliminate choice C; the immune system can produce antibodies to the VSGs, it just takes a period of time to get significant
quantities of antibodies produced. Choice D can be eliminated because cytoplasmic proteins are not easily recognized by the
immune system because they are not directly accessible. Additionally, we can infer that antibodies are not significantly
made to proteins other than VSGs because this would result in the complete destruction of the parasite (since Trypanosoma
can only vary its VSGs). This would not fit with the cyclic nature of the disease. The best answer is B.
D is the best answer. Each VSG gene encodes a different surface glycoprotein; this is how the parasite produces different
surface antigens in an attempt to evade immune attack. The passage tells us that surface antigens are varied by expressing
different VSG genes. This means that the VSG genes must vary in the sequence of their protein-encoding regions (i.e., each
VSG encodes a different protein product). The other answer choices can be eliminated alter consideration. Choice A is

r<B
Copyright by The Berkeley Review-

504

The Berkeley Review


Specializing in MCAT Preparation

Biology

Molecular Cell Biology Diagnostic Set I

Answers

incorrect because genes do not have amino acid sequences, they have nucleotide sequences. Pay close attention to wording!
Choices B and C can both be ruled out because the question asks about the transcriptionally silent VSG genes. Both
promoter and operator sequences are part of the regulatory apparatus of a gene. They affect the degree to which a gene is
transcribed. Ifthese VSG genes are all not being transcribed, they probably don't differ much in their promoter or operator
sequences, and there is no evidence given in the passage to imply that they do. The best answer is D.
8.

Bis the best answer. Recall that promoters are sequences of DNA to which RNA polymerase is attracted; they "promote"
the transcription of genes they control. If a normally untranscribed (silent) VSG gene is excised from its original location
and placed into a region adjacent to a strong promoter sequence, that VSG gene might now be transcribed. ThLs is one valid
hypothesis to explain why VSG genes are only transcribed in the expression locus. The other answer choices do not
represent valid hypotheses. Choice A can be eliminated because heterochromatic regions of chromosomes contain

heterochromatin, which is generally not transcribed in eukaryotes. A VSG gene placed in this nonsense region of the DNA
would probably not be transcribed, and hence the expression locus is probably not in a heterochromatic region. Choice C is
also incorrect. The tips of linear chromosomes usually contain telomeric sequences which are not transcribed. Choice D can
be eliminated because an origin of replication sequence is a site where DNA starts to be replicated (i.e., a site where the
DNA replication machinery first binds and goes to work). It has little affect on gene transcription. The best answer is B.
A is the best answer. Any modifications to proteins that occur after translation (i.e.. conversion from mRNA to

9.

polypeptide) are referred to as post-translational modifications. The addition of carbohydrate groups to proteins (making
them glycoproteins) is a post-translational modification. The polypeptide must be produced before a carbohydrate can be
attached to it (usually by a specific enzyme). The attachment of carbohydrate groups usually occurs in the Golgi apparatus.
If we didn't know this, we could still eliminate choices C and Dbecause we know that lysosomes are organelles that degrade
cellular and extracellular products. Carbohydrate addition couldn't occur in lysosomes, and it can't occur prior to translation
of mRNA. The best answer is A.
10.

B is the best answer. Answering this question requires a firm understanding of meiosis and the process oi genetic
recombination, or "crossing-over." Meiosis is subdivided into two distinct phases, I and II. In meiosis I, two pairs of
homologous chromosomes (each from a different parent) are separated into two daughter cells. In meiosis II, those daughter
cells each divide, separating the sister chromatids of each homologue they were given. Crossing-over occurs between
homologous chromosomes (i.e., between a maternal and a paternal chromosome). By definition, the only time both maternal
and paternal homologues are present in the same cell is during meiosis I. Hence we can eliminate choice D. Meiosis I is

further divided into phases not unlike those of regular mitotic division. Crossing-over can only occur when homologous
chromosomes meet and form connections with each other. This occurs during prophase, when the chromosomes are
condensed and all jumbled together (and thus are able to form chiasmata. or connections with each other). Choice C can be

eliminated because in anaphase, the two homologous chromosomes are being pulled apart by the spindle apparatus and
hence they can't get together to undergo recombination. Likewise, choice A can be ruled out because during telophase,
homologous chromosomes have been pulled to opposite poles and can't come in contact with one another. The best answer
is B.

(Questions 11-14)
11.

Not Based on a Descriptive Passage

B is the best answer. Recall that retroviruses have RNA for genetic material, rather than DNA. In the case of HIV. the

RNA genome is replicated by the reverse transcriptase enzyme, which has no exonuclease activity and is particularly prone
to making mistakes (approximately 1 per replication). This leads to a high rate of mutation when HIV replicates, resulting in
new viral antigens which can evade the immune system, making vaccines useless. This problem can be answered by
eliminating incorrect choices as well. Choice A can be ruled out because there are not many mutagens present in HIV's host
cells (T-cells). If there were, the host cells themselves would constantly mutate, a feature which is not observed. Choice C

can be eliminated because even though retroviruses such as HIV do have membrane envelopes, they are also encapsulated
by a protein coat which would protect them from mutagens. This answerchoice is designed to trick us by capitalizing on our
knowledge of the differences between retroviruses and non-retroviruses. Don't be fooled. Choice D is wrong because all
viruses have evolutionary pressures to mutate; this is how they have been able to survive for millions of years. The host is
constantly trying to devise ways to destroy viruses, and those that mutate may produce variants which are more "fit." Hence
both DNA and RNA viruses are under pressure to evolve. The best answer is B.

12.

C is the best answer. This question tests our understanding of the Central Dogma of Molecular Biology (i.e.. DNA makes
RNA makes protein). Cholesterol is a non-polar, cyclic molecule which is not protein in nature. Hence DNA can't directly
encode cholesterol; DNA can only encode RNA which can get translated into protein. Cholesterol can only be synthesized
via a metabolic pathway containing protein enzymes which catalyze each step. Choices A. B. and D can be eliminated
because neither DNA nor RNA can directly encode cholesterol since it is not a protein. Remember, however, that the series
of enzymes in the cholesterol-synthesis pathway are each encoded by genes. The best answer is C.

Copyright by The Berkeley Review0

505

The Berkeley Review

Specializing in MCAT Preparation

Biology
13.

Molecular Cell Biology Diagnostic Set I

Answers

D is the best answer. Aerobic exercise requires endurance; large amounts of ATP are required by muscle cells in order to

produce such sustained activity. Therefore, in order to adapt to consistent aerobic exercise, the number of mitochondria in
muscle cells increases, allowing more ATP production. We can eliminate the other answer choices. Choices A and B are
incorrect because hemoglobin is not present inside muscle cells; do not confuse it with myoglobin (which is present in

muscles). Hemoglobin is present in circulating erythrocytes. Choice C can be eliminated because although oxygen demand
in aerobically exercising tissue is increased, mitochondria do not help to meet this demand. In fact, an increased number of
mitochondria is part of the reason that these muscle cells need more oxygen (i.e., for oxidative phosphorylation and ATP
14.

production). The best answer is D.


C is the best answer. Ribosomes are composed of both protein and RNA. Hence, the red dye would stain the RNA (a

negatively-charged structure due to its phosphate backbone) while the white dye would stain the protein. The overall
solution would look pink (red + white = pink). The solution would not look red nor white alone because the two dyes would
overlap; both are staining parts of the ribosome. Additionally, the solution would not look clear; this would only happen if
no dye bound at all. The best answer is C.
^__
Cell Cycle Checkpoints
Passage III (Questions 15 -19)
15.

A is the best answer. The question asks how one would BEST determine which stage of the cell cycle a particular cell is
undergoing at a given time. In order to answer this, we must think about what differentiates each of the phases of the cell
cycle from one another. During mitosis, or M phase, the DNA content of the cell is split between two daughter cells. Each
of these daughter cells therefore enters GI phase with half as much DNA as a cell in M phase. During S phase, DNA is
replicated. This leads to G2 phase, where the DNA content of the cell is twice that of a GI cell. Cells in S phase have an
amount of DNA somewhere between a G1 and a G2 cell. The upshot of all this is that DNA content can therefore be used to

discriminate effectively between the various stages of the cell cycle. The other answer choices represent considerably less
effective means for doing so. Choice B can be eliminated because looking at protein content would not differentiate cell
phases as effectively because there is considerable variability in rates of protein synthesis from cell to cell. For the same
reasons, choices C and D can be eliminated. These factors would not clearly distinguish between the various stages, though

they could give one a rough estimate of whether GI or G2 has been completed. The be.st answer is A.
16.

C is the best answer. In order to best understand this question, we must carefully analyze the graph in Figure I which

depicts the number of cells in each of the various phases of the cell cycle 8 hours after irradiation with X-rays. We must
then compare these findings with the non-irradiated group. Irradiation causes DNA damage, and according to the passage
such damage should cause the cell cycle to halt at certain regulatory checkpoints. According to the graph depicting normal,
non-irradiated cells, there are some cells in GI, S, and G2 at any given time. Once irradiated and allowed to grow for 8
hours, however, there are no longer any cells in S-phase. The number of cells in G2 goes up dramatically, while the number
of cells in GI drops. What is going on? DNA damage is halting the cells at the checkpoint between GI and S phase and at
the checkpoint between G2 and M phase. Cells in GI are prevented from entering S phase, while cells already in S phase at
the time of the irradiation complete DNA replication as be.st the can and proceed to G2 phase. This explains the lack of cells
in S phase and allows us to eliminate answer choice B. Cells that are in G2 phase also have X-ray damaged DNA and hence
are prevented (by the G2/M checkpoint) from entering mitosis. As a result, the number of cells "stuck" in G2 increases,
explaining the higher peak on the graph. This allows us to eliminate answer choice A. Choice D can be eliminated because it
is a valid conclusion. The passage tells us that X-rays induce DNA damage, and the graphs show us that the cell cycle has
been halted at certain checkpoints. This leaves us with answer choice C as the statement that can NOT be concluded from
the information given. As noted above, cells in S phase at the time of irradiation are able to leave and enter G2. This means
that they can successfully cross the S/G2 checkpoint. Irradiation therefore does not block entry of cells into G2 phase. The
best answer is C.

17.

A is the best answer. An evolutionary advantage is anything that better enables an organism to survive and reproduce. Cell
cycle checkpoints usually confer an evolutionary advantage on a cell for several reasons, some of which are mentioned in
the answer choices. For example, the checkpoint between GI and S phase prevents the replication of chromosomes with
severely damaged DNA by halting the cell cycle prior to DNA synthesis. If such a checkpoint were absent, the cell would
replicate the damaged DNA and pass it on, possibly with lethal results. Hence, the checkpoint helps the cell to more
effectively reproduce; eliminate choice B as we are looking for the answer choice that does not represent an evolutionary

advantage. Choice C can also be eliminated because the checkpoint between G2 and M phase ensures that the cell has
grown to a desirable level before mitosis. Otherwise, cells that are extremely small might attempt to divide, possibly with
lethal consequences or with daughter cells starting out with a selective disadvantage. In multicellular organisms,
checkpoints ensure that cells don't divide out of control by regulating when mitosis occurs. When these checkpoints break
down or are overridden, cancer can occur. Hence the checkpoints promote survival of the organism; eliminate choice D.
This leaves us with choice A as the correct answer. If cell cycle checkpoints increased the time a single-celled organism

took to reproduce, no direct evolutionary advantage might be observed. This is because the goal of reproduction and
propagation of the species is delayed. The best answer is A.
Copyright by The Berkeley Review,

506

The Berkeley Review


Specializing in MCAT Preparation

Biology
18.

Molecular Cell Biology Diagnostic Set I

Answers

Bis the best answer. If cell cycle checkpoints were removed entirely, cells would proceed through the cycle even if DNA

were significantly damaged. According to the passage, checkpoints normally halt the cell cycle If potentially lethal DNA

damage is incurred. This would give the cell a chance to repair the damage before replicating its chromosomes or dividing

X-rays are pretty strong mutagens, i.e.. they can damage DNA. Normal cells can repair the damage before proceeding, but
cells in ataxia patients don't have a chance. They try to replicate the damaged DNA and either die or produce daughter"cells

with fatally damaged chromosomes. These patients' cells die in large numbers when they undergo even a normal X-ray

examination. We can eliminate the other answer choices. Choice Acan be ruled out because DNA damage is not repaired in
these patients and is instead passed on to daughter cells after mitosis. Damaged DNA can cause cancer, and hence we would

expect the cancer rate to be increased in these individuals. Choice C is incorrect because without cell cycle checkpoints,
cells rapidly proceed through the division cycle unhindered and hence divide faster. Choice D is likewise incorrect because

cells without checkpoints may not reach sufficient size during the growth phases before they divide (see the passage). The
be.st answer is B.
19.

I) is the be.st answer. This is the best explanation using the information about cell cycle checkpoints given in the passage.
After the cell line was irradiated, cells that were in S phase were allowed to proceed into G2 phase, which helps to explain
the large G2 peak. New cells are prohibited from entering S phase due to the Gl/S checkpoint which is triggered by
excessive DNA damage (caused by the X-rays). Hence there are no cells in S phase 8 hours after irradiation. Let's consider
the other answer choices. Choice A can be eliminated because according to the graphs, cells have not been killed. The total
number of cells, judging by the peaks in the two graphs, does not change significantly. Therefore, the cells in S-phase most
likely moved into G2 phase; they were not killed. Choices B and C can both be ruled out because neither instance would
have directly caused the observed deficit of cells in S phase. The best answer is D.

Passage IV (Questions 20 - 25)


20.

RNA Splicing

B is the best answer. The question basically asks what functions the spliceosome performs and which enzymes perform
similar biochemical tasks. According to Figure I, the spliceosome assembly cleaves the mRNA at the intron/exon boundary.
Since it is cleaving within the nucleic acid sequence, this action is comparable to an endonuclease. E.vr;nucleases cleave base
pairs at the ends of a nucleic acid sequence (eliminate choice A). Restriction enzymes are examples of endonucleases. After
the intron is removed, the spliceosome catalyzes the sealing together of the separated exons. Such sealing is similar to the

actions of the enzyme ligase. The other answer choices do not represent enzymes which have analogous functions to the
spliceosome. RNA polymerase serves to transcribe DNA to RNA, while DNA polymerase is functional in replicating DNA.
A protease is an enzyme which cleaves polypeptides. None of these functions are analogous to those of the spliceosome.
The be.st answer is B.

21.

B is the best answer. A point mutation is a change in a single base pair. If such a mutation were to occur in the 5' splice site
of a given gene, the consensus sequence of the site would be altered, possible destroying its function. In other words, it
would no longer serve as a splice site. During spliceosome attachment, normal splicing would not occur and part of the
intron might be actually translated into part of the final protein product rather than being excised before translation. This
would result in a protein product of altered length. The other answer choices represent situations that would not occur given
a simple point mutation in a 5' splice site. A protein product with a single amino acid substitution (e.g., glycine for arginine)
would not result because the mutation is occurring in an intron. a segment of DNA which does not normally encode part of
the final protein product. An amino acid substitution could only occur if the point mutation affected an exon. Figure 1
shows that the 5' splice site is in the intron. adjacent to the exon. Eliminate choice A. Choices C and D can also be

eliminated because gene transcription would be affected if the mutation were somewhere in the gene's operon; a mutation in
the coding part of the gene is unlikely to affect the actual transcription of the gene. The best answer is B.
22.

C is the best answer. The question asks us to compare the excised intron shown in Figure I with the same intron in the
genomic DNA prior to transcription. The excised intron is made of RNA, whereas the intron in the genomic DNA (back on

the chromosomes prior to transcription) is made of c/cavyribonucleic acid (DNA). The RNA has a 2' hydroxy group on its
ribose backbone while the DNA doesn't; this is a difference between the two and hence choice C is the correct answer while

choice D can be eliminated. Choices A and B are meant to trick us; we know that uracil is present in RNA but not DNA, but
it replaces thymine, not an adenine. Be careful! The be.st answer is C.

23.

C is the best answer. The passage states that unprocessed (i.e.. unspliced) mRNA is confined to the nucleus. Recall that
active ribosomes are not present in the nucleus, only in the cytoplasm. This situation is logical because it prevents the
translation of unspliced mRNA. Only fully spliced mRNA is exported to the cytoplasm for translation. This leads us to
choose answer choice C while eliminating answer choice B. The other answer choices are not consistent with information
given in the passage. Choice A can be eliminated because spliced mRNA would be shorter than the DNA from which it is

transcribed because the DNA contains introns. while the mRNA does not. Choice D is incorrect in light of the information
in the passage, which states that mRNA is spliced in the nucleus. The best answer is C.

Copyright by The Berkeley Review

507

The Berkeley Review


Specializing in MCAT Preparation

Biology
24.

Molecular Cell Biology Diagnostic Set I

Answers

C is the best answer. Eukaryotes are at an advantage by having an RNA splicing system, rather than simply having

uninterrupted, coding gene sequences. Alternate splicing, or splicing a gene in a different way, can lead to new protein

products and variability, resulting in increased evolutionary adaptability and flexibility. If this notion did not seem

immediately apparent, the question could still be answered through a process ofelimination. Choice Acan be ruled out
because a larger genome in itself is not necessarily an advantage unless the extra DNA contains useful information. Introns

do not contain such information, and hence the size they add to the genome does notconferany advantage (in fact, it may
make it more difficult to replicate and maintain). Eliminate choice A. Choice B is incorrect; RNA polymerase has to
transcribe more DNA when introns are present. It transcribes the entire gene, introns and all. Only later are the introns
excised. Choice D makes little sense; introns have no effect on how many ribosomes a cell needs. Ribosomes simply

25.

translate spliced mRNA into protein. The best answer is C.


D is the best answer. In the first sentence of the passage, it is stated that eukaryotic genes have introns and therefore need

splicing. From this, we must infer that prokaryotes do not. Spliceosomes are not present in prokaryotes such as bacteria
because prokaryotic genes do not posses introns and do not need to be spliced. This fact can be further deduced when we
consider that bacteriado not have nuclei; they would have no way to separate processed mRNA from unprocessed mRNA.

Fungi (e.g., mushrooms), mammals, and yeast are all eukaryotes. The best answer is D.

Passage V (Questions 26 - 30)


26.

Telomerase

A is the best answer. Bacteria have circular chromosomes and hence do not suffer from the "end-replication" problem

(since they don't have "ends"). This problem is only present in linear chromosomes. Protists (e.g., amoeba, etc.), fungi (e.g.,
mushrooms) and insects are all eukaryotes and hence have linear chromosomes which must have telomeres and telomerase

enzyme in order to properly replicate without shortening. The best answer is A.


27.

B is the best answer. When telomerase is inactivated in somatic cells, every mitotic division of these cells leads to shorter

and shorter chromosomes. Recall from the passage that telomeric sequences are presentat the ends of the chromosomes and
these sequences consist of multiple, repeating units. When telomerase is inactive and doesn't extend these sequences, each
cell division leads to the gradual loss of a segment of the telomere. After multiple divisions, the telomere is completely lost
and the next DNA to be lost consists of essential genes. Hence, somatic cells are limited in the number of times they can

divide before they lose vital genes. This is one reason why human somatic cells in cell cultures can only divide a finite
number of times. Eliminate the other answer choices after consideration. Choice A is incorrect because somatic cells are

capable of division, just not an infinite number of divisions. Choice C is also incorrect because somatic cells only lose their
telomeric sequences after multiple divisions. Choice D can be ruled out because somatic cells are not germ line cells; they
can't undergo meiosis. The be.st answer is B.
28.

B is the best answer. This question requires us to recall the different phases of the cell cycle. GI phase is the period after
mitosis during which the cell experiences growth and protein synthesis. S phase (synthesis) comes next; DNA is replicated
during this phase. It is during this phase that the chromosomes would shorten if there were no active telomerase, because
such shortening would result from incomplete replication of the lagging strand of DNA. G2 is the next phase; it involves the
further growth of the cell and further protein synthesis. M phase is mitosis; the chromosomes segregate and the cell divides
during this phase. It is only during S phase that DNA is being replicated; do not confuse cell replication with DNA
replication. The best answer is B.

29.

B is the best answer. Recall from an earlier question that somatic cells lack active telomerase, limiting the number of times
they can divide. Cancerous cells, on the other hand, have high concentrations of active telomerase, allowing them to divide
indefinitely without any shortening of their chromosomes. This contributes to cancer cells' immortality, meaning they can
divide in culture forever. We can eliminate the other answer choices. Choice A is incorrect because an active telomerase

enzyme does not contribute to an increased mutational rate. Choice C is wrong because active telomerase would prevent
loss of chromosomal material. Choice D is invalid because the prevention of chromosomal shortening does not directly
allow a cancer cell to metastasize. In order to spread to other parts of the body, other cellular processes must be altered in
cancer cells (i.e., the ability to migrate and chew through obstacles). The best answer is B.
30.

C is the best answer. This question simply requires that we understand the molecular nature of the different answer
choices. Ribosomes are made of both protein and RNA, a combination which aids the function of protein translation from
mRNA. The other answer choices are incorrect. Choice A, RNA polymerase, is a protein enzyme that transcribes RNA

using DNA as a template. RNA is not part of the enzyme's structure, however. Histones (choice B) are proteins which serve
to form a scaffolding for DNA in chromosomes. In other words. DNA strands wind around histones ("beads on a string") in

order to pack into chromosomes more efficiently. Histones are made of protein only. Choice D, tRNA (transfer RNA) is a
molecule made purely of ribonucleic acid. Although tRNA may bind to individual amino acids, such amino acids are not
"proteins" and hence tRNA is not a protein/RNA hybrid like telomerase. The best answer is C.

Copyright by The Berkeley Review,

508

The Berkeley Review

Specializing in MCAT Preparation

Biology

Molecular Cell Biology Diagnostic Set I

(Questions 31-34)
31.

Answers

Not Based on a Descriptive Passage

C is the best answer. Recall that the HIV virus infects certain T-cells. causing their destruction and the ultimate death of

the organism due to immunodeficiency. In the case ofSCID mice, the immune system is already deficient (there are no T
cells lor the HIV virus to infect). Hence, these mice would not die more rapidly if infected by the HIV virus. If we didn't
remember that HIV infected T-cells, we could still answer this question through a process of elimination. Recall that we are
looking for the invalid statement. Choice Ais valid because without B-cells, SCID mice can't produce antibodies of their

own. Choice Bis valid because grafted tissue from a foreign species would not be rejected by SCID mice because they have

a non-functional immune system. Choice Dis also valid because without B-cells, humeral (antibody-mediated) immunity
would not exist. The best answer is C.
32.

Bis the best answer.. This statement is incorrect because RNA is much more prone to errors than DNA. RNA polymerase
has no exonuclease, or "proofreading", activity. DNA polymerase does have exonuclease activity and hence can correct
mistakes before they permanently incorporated into the DNA. This makes sense evolutionarily because an RNA molecule

with a mistake in it gets translated into a single aberrant protein molecule, while DNA with a mistake in it get transcribed
and translated into a permanently aberrant protein for every generation to come. It is therefore very important that DNA be
less error-prone than RNA. We can eliminate the other answer choices because they do represent real differences between

RNA and DNA. Eliminate choice Abecause deoxyribonucleic acid does indeed, as its name suggests, lack a 2' hydroxyl
group while RNA possesses one. Choice C can be eliminated because it is now believed that primitive RNA molecules
evolved before almost anything else and were able to catalyze other reactions. DNA then evolved later as a more stable way
of storing genetic information. Choice D can be ruled out because DNA does indeed use thymine while RNA uses uracil.
The best answer is B.
33.

C is the best answer. Recall that ATP synthase is the transmembrane channel through which protons travel down their
electrochemical gradient, producing ATP from ADP as they do so. In eukaryotes, ATP synthase is embedded in the inner
mitochondrial membrane, harnessing protons which travel from the intermembrane space (after having been pumped there
by the cytochrome complex proteins) into the matrix. In prokaryotes like E. coli, however, organelles such as mitochondria
are not present (if they were, they would be as big as the entire bacterium). But bacteria do indeed accomplish aerobic
respiration. Instead of using the inner mitochondrial membrane, they use their plasma membrane. Protons are pumped out of
the cytoplasm and return along their gradient through the synthase. Make sure to eliminate choice D because the bacterial

cell wall is not the same as the plasma membrane. The cell wall consists of a carbohydrate-rich peptidoglycan layer and
sometimes an external lipid membrane. ATP synthase is located exclusively at the plasma, or inner, membrane. The best
answer is C.
34.

A is the best answer. Interphase is the part of the cycle during which the cell is not dividing and the chromosomes are
decondensed into chromatin, allowing RNA polymerase greater access to the cell's genes. It is during interphase that most
transcription and protein synthesis occur. Prophase, anaphase, and telophase are components of mitosis, or cell division.
Starting at prophase, the cell's DNA condenses into visible chromosomes, restricting access to RNA polymerase. Very little
gene transcription occurs during mitosis. The best answer is A.

Passage VI (Questions 35 - 40)


35.

Diphtheria Toxin

B is the best answer. In the passage, it is stated that a reducing agent is used to cleave intact diphtheria toxin into the two

fragments (A and B). What type of bond would a reducing agent be most likely to cleave? A disulfide bond (between cys
residues) because the reduction of the S-S bond cleaves it and forms two terminal SH groups on each of the cysteine side
chains. If the two fragments were linked together by a disulfide bond, a reducing agent would break them apart. The other
answer choices can be eliminated after consideration. Peptide bonds are broken by hydrolysis (addition of H2O across the
bond). If the reducing agent could cleave peptide bonds, we wouldn't be left with two fragments, we would be left with
hundreds of free amino acids. Eliminate choice A. Choices C and D both represent weak interactions that can contribute to

the folding and aggregation of subunits in a protein. These can't be the primary forces holding the two fragments together
because a reducing agent would not cleave them. The be.st answer is B.
36.

C is the best answer. We must keep in mind that rats and hamsters are relatively similar creatures. In other words, they are
evolutionarily rather close together. We would not expect them to differ in highly conserved aspects of their molecular
machinery. Neither their tRNAs nor their overall cellular protein synthesis machinery should differ much; eliminate choices
A and B. Choice D can be ruled out because even if hamsters did perform more mRNA translation than rats, this would not

explain their susceptibility to the diphtheria toxin. If protein synthesis is halted altogether, regardless of how much

translation normally occurs, the organism dies. This leaves us with choice C as the answer. A cell receptor is just one
membrane protein: it can easily vary over relatively short evolutionary distances (i.e.. between hamsters and rats). If rats

lack the receptor, the diphtheria toxin can't bind to the cell and therefore can be endocytosed and cause harm. The best
answer is C.

Copyright by The Berkeley Review

509

The Berkeley Review

Specializing in MCAT Preparation

Biology
37.

Molecular Cell Biology Diagnostic Set I

Answers

A is the best answer. From Table 1 we learn that Fragment B, when incubated with a culture of cells, is found inside the

cells after the incubation period. This implies that it is able to bind to a cellular receptor and become internalized (via

endocytosis, more specifically receptor mediated endocytosis). From the note at the bottom ofthe table, however, we learn
that incubation with Fragment B alone does not kill the cells. This is evidence that although Fragment Bcan enter the cells,
it can't inactivate the target elongation factor described in the passage. If it could, the cells would die as the result of
incubation with Fragment Balone. Since we are looking for the invalid answer, the other answer choices can be eliminated

because they are valid. Choice Bis a correct statement because without Fragment A, Fragment B is incapable ofkilling

cells. Only intact diphtheria toxin is lethal to the cultured cells; therefore, both Fragment A and Fragment Bare necessary
components. Hence choice C is also a true statement and can be eliminated. Choice Dis a valid statement. Fragment Adoes
not directly bind to the diphtheria cell-surface receptor. Ifit did, we would have seen its fluorescent signal either inside the
cell or bound to the exterior of the plasma membrane. Instead, it is localized in the extracellular medium, unassociated with
cells. The best answer is A.

38.

B is the best answer. The question states that the gene which encodes DT was introduced into the Corynebacterium

genome by a bacteriophage. This means that only bacteria which have been infected by the phage carry the gene, and
therefore only these bacteria can cause diphtheria. Choice A is incorrect because if the phage genes contained introns and
exons, the bacteria would not be able to process them after transcription. In other words, prokaryotes such as bacteria are

incapable of mRNA splicing; only eukaryotes can do this. Choice C can be ruled out because the phage, as a typical virus,
can't do much of anything itself. It lacks protein synthesis machinery and therefore would not be able to produce norsecrete

DT by itself without the help of a host cell. Remember that viruses are obligate parasites. Choice D is incorrect. Viral
infection that does not result in lysis, or bursting, of the victim cell is termed lysogenic infection. The virus instead inserts its

own genome within the DNA of the host cell. As a result, every time the host cell divides, the viral DNA gets replicated.
This promotes the survival of the viral species and therefore is an evolutionary advantage. Incidentally, certain
environmental signals can trigger the viral DNA to excise itselfand begin reproducing active phages. The best answer is B.
39.

A is the best answer. From Table 1, we learn that only intact diphtheria toxin is capable of killing the cultured cells
(eliminate choice B). Fragment B enters the cells, but does not kill them, while Fragment A can't enter the cells. When the

two fragments are combined in the intact DT protein, the combination is capable of both entering the cell and killing it. This
implies that Fragment A is the "active" portion of the toxin which actually binds to and inactivates the target elongation
factor, preventing protein translation from continuing and killing the cell. Fragment B is necessary for recognizing and
binding to a specific cell-surface receptor, triggering the endocytosis of the entire DT molecule (including the attached
Fragment A); eliminate choice C. Choice D can be ruled out because it is a hypothesis with no supporting evidence shown
in Table I. In other words, we are shown no evidence to lead us to believe that Fragment A has an alpha-helical secondary
structure. The best answer is A.

40.

D is the best answer. We are looking for the answer choice which is not consistent with a successful cancer treatment. In
order for diphtheria toxin to be an effective treatment for cancer, it must selectively kill tumor cells while not harming other
body cells. If diphtheria toxin were altered (maybe with the attachment of a hydrophobic moiety) such that it could cross
lipid membranes freely, it would be able to enter any cell in the body and the results would be fatal for the cancer patient.
The other answer choices all represent valid requirements for effective tumor treatment. Choice A is valid because it

provides a means through which cancer cells can selectively bind to tumor cells, rather than normal body cells. Choice B is
valid because cancer cells must be capable of endocytosis in order to internalize the diphtheria toxin (recall the mechanism
described in the passage). Choice C is valid because diphtheria toxin inactivates a specific component of the protein
synthesis machinery. If cancer cells have a drastically different machinery, they might be unaffected by the toxin and the
therapy would fail. The best answer is D.

Passage VII (Questions 41 - 47)


41.

Maternal Effect

D is the best answer. Inheritance of the sinistral coiling trait follows the maternal effect pattern, meaning that the genotype
of the mother must be did in order for the progeny to express sinistral coiling. The question states that we are crossing
sinistral males to dextralfemales. This is the reverse of what is given in Figure 1; hence, all of the Fj generation will appear
dextral. This is because the mother is now DID, and therefore the ovum she produces will be DID, making the offspring all

dextral. Remember that the maternal effect pattern of inheritance involves the mother's genotype determining the offspring's

phenotype. The otheranswer choices do not express this pattern of inheritance. The best answer is D.
42.

B is the be.st answer. In sex-linked inheritance, the gene of interest is present on the X-chromosome. Hence, if inheritance

of shell-coiling direction were sex-linked, according to Figure I all of the males of the Fj generation would be sinistral
while all of the females of the F| generation would be dextral. This is because the males, who inherit their X-chromosome
from their mother, would all have the genotype dlY, while the females (who get one X from the father and one from the

Copyright by The Berkeley Review

510

The Berkeley Review


Specializing in MCAT Preparation

Biology

Molecular Cell Biology Diagnostic Set I

Answers

mother) would all be DID. Since d is recessive, it would only be expressed in the males, who would be sinistral. The other

answer choices do not show outcomes which would be representative of a sex-linked pattern of inheritance. The best
answer is B.

43.

Ais the best answer. This question tests our molecular understanding of recessive mutations. The sinistral phenotype can

only be expressed when only the recessive dallele is present. Let's approach this problem through a process ofelimination.
II the d allele encoded a more active protein than the wild-type allele, it would be expected to behave in a dominant manner;
even ifthe wild-type allele were present (as in heterozygotes), the d allele would be expressed because it encodes a more
active protein. Since this is not what is observed, eliminate choice B. If the d allele were encoded by mitochondrial DNA

rather than nuclear DNA, we would not observe a maternal effect pattern o\' inheritance. Since mitochondria are mainly
inherited from the mother (through the cytoplasm of the ovum), we would expect all of the F2 generation shown in Figure 1

to be sinistral (because their mothers would have been affected). Since this is not observed, eliminate choice C. If the d

allele encoded a protein which bound to and inhibited the wild-type protein, we would expect it to behave in a dominant,

rather than recessive, fashion; this is because it would actively inactivate the wild-type protein, making even heterozyotes

express the sinistral phenotype. Since this is not observed, eliminate choice D. This leaves us with choice A as the correct
answer. Aframeshift mutation usually completely destroys a protein by altering the reading frame ofthe RNA polymerase.
Such an inactive protein would not be a problem if a wild-type copy were also present (as in heterozygotes). When the
animal is homozygous (did), no working protein is present and the animal expresses the sinistral phenotype. This is a valid
theory for the molecular nature of the recessive sinistral allele. The best answer is A.
44.

B is the best answer. The maternal effect dictates that the mother's genotype will affect the progeny's phenotype. This is
because the mother's genotype will determine the makeup of the ovum that she produces. This ovum will then become
fertilized and eventually develop into the progeny snail. According to the passage, the distribution of factors (i.e., RNA,
protein, etc.) in the ovum may determine the coiling direction of the adult. Hence, if the mother were Did, her ovum would

be Did, and all the progeny (regardless of their individual genotypes) would express a dextral coiling. This is why the F?
generation is entirely dextral, even though 1/4 of them are did. We can eliminate the other answer choices. Choice A is
incorrect because sinistral is not expressed in heterozygotes, only in homozygous did snails. Choice C can be eliminated

because the father's genotype does not affect the progeny's phenotype when the maternal effect is present. Choice D is
wrong because in homozygotes, the penetrance of the sinistral trait is practically 100%. Penetrance is the degree to which a
trait is phenotypically expressed; in this case, we can't use penetrance as a descriptive term because we are not dealing with
typical Mendelian expression patterns. The best answer is B.
45.

C is the best answer. If shell coiling direction were expressed in a typically Mendelian fashion, did animals would be

sinistral and Did animals would be dextral (since the <7 allele is recessive). According to Figure 1, 1/4 of the F2 generation
would be did. therefore the ratio of dextral to sinistral snails would be 3 dextral to 1 sinistral. In typical Mendelian
inheritance, the genotype of the individual determines the phenotype of the individual. The best answer is C.
46.

C is the best answer. According to the passage, extranuclear inheritance involves the inheritance of genes located in
organelles such as mitochondria (which have their own independently-replicating genome). Since the cytoplasm of the
embryo is mainly donated by the ovum, mitochondria are mostly inherited from the mother. Hence if the mother in the P
generation shown in Figure 1 had a disease caused by a defect in the mitochondrial genome, all oi' her children would inherit

the disease because they would all receive their mitochondria from her. The other answer choices do not represent such an
extranuclear pattern of inheritance. The be.st answer is C.
47.

B is the best answer. As mentioned in the answers to the above questions, the phenotype of the individual (in this case,
dextral or sinistral) is determined by the genotype of the mother when the maternal effect is present. If the mother is did, the

children will be sinistral. This is because the mother's genotype determines the distribution of coiling direction-determining
factors in the ovum. If the children are did but the mother is Did. the children are dextral. A phenotype is defined as the
physical expression of a gene (i.e., a physical trait). A genotype is defined as the actual genes that an organism possesses
(i.e., Did or did). The best answer is B.

(Questions 48 - 52)

48.

Not Based on a Descriptive Passage

A is the best answer. In order for the DNA probe to form a hybrid with the metaphase chromosomes, both the probe and
the chromosomes must be single-stranded. This way they are capable of base-pairing with one another. If both were double
stranded, they could not form hybrids with each other. The otherchoices can be eliminated because theyshould hold true if
the FISH technique is to be successful in finding the chromosomal location of a gene of interest. As the name implies, the
DNA should indeed be labeled with a fluorescent molecule so it can me visualized under a microscope; eliminate choice B.
Choice C is also important for the technique to work; in order to hybridize to the location of the gene of interest, the DNA

probe should contain some of its sequence so it can properly base-pair. Choice D is likewise an important prerequisite. If the
Copyright ) by The Berkeley Reviewr

51 1

The Berkeley Review

Specializing in MCAT Preparation

Biology

Molecular Cell Biology Diagnostic Set I

Answers

probe didn't contain a sequence that was unique to the gene of interest, it might form hybrids with other regions of the
chromosomes, making it impossible to accurately localize the gene ofinterest. The best answer is A.
49.

A is the best answer. The term "highly conserved" refers to nucleotide sequences which have not changed significantly

over long stretches of evolutionary time. For example, the actin gene in mice and humans has highly conserved sequences.

Sequences are usually highly conserved because any alteration in them by mutation would prove fatal to the organism,
preventing the mutated sequence from being passed on to future generations. Hence, conserved sequences stay stable for a
Ion" time and across many different species. A mutation in a conserved sequence has a high likelihood of adversely

affecting an organism for the reasons stated above. We can also answer the question by using a process of elimination.
Choice B can be ruled out for the reasons given above; an alteration in a conserved sequence would probably make an

organism less fit to survive and reproduce. Choice Ccan be eliminated because con.served sequences usually encode part of
aprotein which can't be changed much if it is to retain its function. Such an alteration might actually have a large chance of
affecting the function of the product of the gene. Likewise, choice Dcan be eliminated. Asequence is usually conserved
because mutations in it don't improve the function ofthe gene product. Hence, over millions of years ofevolution, the gene

sequence doesn't change because such achange might only be detrimental, not beneficial. The best answer is A.
50.

B is the best answer. Recall that disulfide bridges (S-S bonds) form between cysteine residues when the terminal sulfhydryl

(SH) groups are oxidized to form S-S dimers. We can deduce that this is oxidation because hydrogens are lost (they are

gained during reduction). With this knowledge, we can eliminate incorrect answer choices. Choices A and D refer to
reducing environments and hence would not be conducive to the oxidation of sulfhydryl groups necessary cross-link
cysteine residues. Choice C can be eliminated because although the inside of the cell is polar, this has little effect on the

cross-linking of cysteine. We are looking for an oxidizing environment, and therefore choice B is the correct answer. The
extracellular medium is generally capable of oxidizing SH groups to form cys-cys dimers, while the cytoplasm tends to be
more of a reducing environment. Hence, secreted proteins generally have disulfide bridges while intracellular proteins do
not. The best answer is B.

51.

B is the best answer. The question states that cDNA is DNA which encodes a gene of interest and does not need to be

spliced. In other words, cDNA does not contain intervening sequences (introns); cDNA only contains the expressed
sequences (exons) of the gene. How can we make cDNA? We must use as a template something which already has the

introns spliced out of it: spliced mRNA. Eliminate choice Asince DNA would contain introns. How do we make DNA from

an RNA template? We must use the reverse transcriptase enzyme found in certain retroviruses. Reverse transcriptase makes
DNA using an RNA transcript. The final product is a DNA copy of the intron-free mRNA. Eliminate choices C and D
because DNA polymerase and RNA polymerase can only use DNA as a template. The best answer is B.
52.

A is the best answer. The DNA replications and radioactive labeling are shown below.
32p

DNA

replication
25%

r
12.5%

Initially, 50% of the DNA is radioactively labeled. After one round o\' replication, 25% is labeled. After two rounds, as the
question asks, 12.5% of the final DNA sample is labeled with radioactive phosphorus. This experiment illustrates that DNA
replication is indeed semiconservative. The best answer is A.

Copyright by The Berkeley Review

512

The Berkeley Review

Specializing in MCAT Preparation

Biology

Molecular Cell Biology Diagnostic Set I

Answers

Passage Topics
Molecular Cell Biology
Diagnostic Set I

Passage I
Passage II
Passage III
Passage IV
Passage V
Passage VI
Passage VII

Copyright byTheBerkeley Review

Membrane Transporters
Sleeping Sickness
Cell Cycle Checkpoints
RNA Splicing
Telomerase

Diphtheria Toxin
Maternal Effect

Score

Molecular Cell Biology

>13

42-52

11-12

37-41

10

34-36

32-33

30-31

27-29

24-26

21 -23

<4

0-21

513

The Berkeley Review

Specializing in NCAT Preparation

!./.; :

:-;il-.;"

Biology

Glossary

MCAT Words and Phrases

Glossary
Most of the items listed in this glossary are important words or phrases that have appeared in one of the
following publications of the Association of American Medical Colleges (AAMC):
1.

MCAT Practice Tests I-XI

2. MCAT Practice Items for the Biological Sciences


3.
4.

MCAT Student Manual


The Official Guide to the MCAT Exam

These words or phrasesare specific to the Biological Sciences portionof the MCAT. We have chosen them based

on student response and on how important we feel they are inbuilding a good conceptual understanding of the
information basic to the biological sciences.

Aerobe, aerobic
Afferent arteriole
Afferent fiber

ABO blood system


Absorption

African sleeping sickness

Acentric chromosome

Agar
Agglutination
Agnathia
Agonist
Alanine (Ala)

Acetaldehyde
Acetic acid

Acetylcholine (ACh)
Acetylcholinesterase
Acetyl-CoA
Acid, acidic

Albinism

Acidophil, acidophilic
Acquired immune deficiency syndrome (AIDS)

Albumen
Albumin

Acrosome

Alcohol

Actin

Alcohol dehydrogenase
Aldehyde

Actinomycin D
Action potential
Activated fatty acid
Activation energy
Active transport
Adaptive radiation

Aldosterone
Alkaline
Allantoin
Allele

Allergen
Allergy, allergic reaction
Allopurinol

Adenine
Adenosine

Allosteric

Adenosine diphosphate (ADP)


Adenosine triphosphate (ATP)
Adenylate cyclase
Adipose cells
Adipose tissue

Amide

Adrenal cortex

Amine (primary, heterocyclic)

Alveolar pressure
Alveolus, alveoli (pi.)
Ames test

Adrenal medulla

Amino acid

Adrenal gland

Ammonia

Adrenaline

Amniotic membrane

Adrenergic receptors (alpha, beta)


Adrenocortical hyperplasias

Amoeba, amoebae or amoebas (pi.)


Amphipathic
Amphoteric
Ampicillin

Adrenocorticotropic hormone (ACTH)


Adsorption
Copyright by The Berkeley Review

515

The Berkeley Review

Specializing in MCAT Preparation

Biology

Glossary

NCAT Words and Phrases

Autosomal dominant

Anabolism, anabolic
Anabolize

Autosomal recessive

Anaerobe, anaerobic

Auxotroph

Anaphase
Androgen

Axon

Axial

Azide (N3)
Azidothymidine (AZT)

Anemia

Aneuploidy
Angiotensin I
Angiotensin II

Angiotensin-converting enzyme (ACE)

p-galactosidase
B lymphocyte
Bacillus thuringiensis

Angiotensinogen
Aniline

Anolis

Back-mutate

Anoxia

Bacteriophage
Bacterium, bacteria (pi.)

Antagonist
Anterior pituitary

Barbiturates

Anthrax

Basal metabolic rate (BMR)

Antibiotic

Base

Antibody

Base pair
Base pair substitution

Antidiuretic hormone (ADH)

Antigen, antigenic
Antigenic shift

Basement membrane
Basic

Antihistamine

Basophile, basophilic

Antioxidants

Antiplatelet antibody

Bicarbonate ion (HC03")

Antisense

Bicuspid valve

Antitoxin

Bile

Aorta

Bile salts

Apex
Apocarboxylase
Aqueous base
Arginine (Arg)

Bilirubin

Bilirubin diglucuronide
Biliverdin

Biliverdin reductase

Binary fission
Biocytin

Arthritis

Artery, arterial
Arthropod
Asclepius syriaca (milkweed)
Aseptic
Asparagine (Asn)
Aspartic acid (Asp)

Biotin

Biotinidase

1,3-Bisphosphoglycerate
Bladder
Blastomere

Blood clotting
Blood pH
Blood platelet
Blood pressure

Asthma

ATP synthase
ATP synthetase
ATPase

Artery

Blood-testis barrier

Arterioles
Atrioventricular node

Body mass index (BMI)


Bombay blood type
Bone deposition

Atrium (left, right), atria (pi.)


Atrophy

Bone marrow

Autonomic nervous system

Bone remodeling
Bone resorption
Bowman's capsule

Au tophagosomes

Brain

Autosomal

Brain stem

Atropine
Autoimmune disorder

Copyright by The Berkeley Review

516

The Berkeley Review


Specializing in NCAT Preparation

Biology

Glossary

Bronchodilation

NCAT Words and Phrases

Centromere

Bronchi (left, right)

Centrosome

Bronchiole

Cerebellum

Brood

Cerebrum

Brooding behavior
Bubonic plague

Cervix, cervical
Channel blocker

Buffer

Chemoattractant
Chemotaxis

Bundle of His

Chemotherapy
Chitin
Calcitonin

Cholesterol

Calcium

Cholic acid

Calcium carbonate

Choline

Calcium casemate

Chordate

Calmodulin

Chorioallantois

Calvin cycle
Camphoric acid

Chromatid
Chromatin

Chroma tography

Cancer

Chromosome

Capillary
Capillary bed
Capsule
Carbaminohemoglobin
Carbohydrate
Carbonic anhydrase
Carboxyl
Carboxylase
Carboxylate, carboxylation
Carboxylic acid
Carboxypeptidase
Carcinogen, carcinogenic

Chylomicron
Chyme
Chymotrypsin
Chymotrypsin mechanism
Cilium, cilia (pi.)
Circular DNA

Circular muscle

Cisterna, cisternae (pi.)


Citric acid cycle
Class
Clone

Clonal deletion

Cardiac muscle

Clostridium tetani

Cardiac output
Cardiac stroke volume

Clotting factor

Cardiopulmonary circulation

Codominant

Carrier

Codon

Cartilage
Cartilaginous Haversian canal

Coelom

Casein

Colchicine

Catabolism, catabolic

Collecting duct

Coevolution

Catabolize

Colon

Catalyze
Catechol-O-methyltransferase (COMT)
CD4 lymphocyte
CD8 lymphocyte
Cell cycle

Color blindness

Committed stem cell


Common bile duct

Competitive inhibitor
Congenital

Congenital adrenal hyperplasia


Conjugate, conjugation
Conjugation pilus

Cell membrane
Cell wall

Cellular respiration
Cellulose

Connective tissue

Central nervous system (CNS)


Centrifugation
Centrifuge

Constitutive
Contractile
Conus arteriosis

Convergent evolution

Centriole

Copyright by The Berkeley Review

517

The Berkeley Review


Specializing in NCAT Preparation

Biology

Glossary

NCAT Words and Phrases

Deoxyribose
2-Deoxythymidine
Dephosphorylation
Depolarization
Depolymerization
Deprotonation

Coprophagy
Cornea

Corpuscle
Cortex, cortical
Cortisol
Cortisone

Crayfish

Dermis

Creatine

Desquamation

Creatine phosphate

Deterministic

Crohn's disease

Deuterium

Cross-bridge

Diabetes (type 1, type 2)

Crossover

Diabetes mellitus

Cryoprotection

1,2-Diacylglycerol
Dialysate
Dialysis
Diaphragm

Cutaneous

Cyclic AMP (cAMP)


Cyclin
Cycloheximide
Cysteine (Cys)
Cystic fibrosis (CF)
Cytochalasin
Cytochrome b
Cytochrome c
Cytochrome oxidase
Cytochrome P-450
Cytokine
Cytoplasm
Cytosine
Cytoskeleton
Cytosol
Cytotoxic T cells
Cytotoxic T lymphocytes (CTLs)

Diarrhea

Diastolic blood pressure


Differentiation

DiGeorge syndrome
Digestion
2,3-Diglobulomuctase
Diglyceride acyltransferase (DGAT)
Dihydroxyacetone
Dilate

Diol

Diploid
Distal tubule

Diurnal rhythms
DNA(A,B,&Z)

DNA-dependent DNA polymerase


DNA-dependent RNA polymerase
DNA fingerprinting
DNA polymerase

D
Dalton

DNA recombination

Damage-accumulation theory of aging


Danaus plexippus (monarch butterfly)

DNA repair
Dominant

Dark reactions (Calvin cycle)


Daughter cell
Daughter strand

Dopamine
Dorsal root ganglion
Down syndrome (trisomy 21)
Drosophila melanogaster (fruit fly)

DDT

Dead space

Ductus arteriosus

Deaminase

Duodenal pH

Decarboxylation
Dehydration
Dehydrogenase

Duodenum

Deletion

Early gene

Deletion mutation
Delta endotoxin

Ebola virus

Denatured

Ectoderm

Dendrite

Ectopic pregnancy
Ebony color

Deoxygenation
Deoxyribonucleic acid (DNA)
Deoxyribonucleoside
Copyright by The Berkeley Review

Efferent fiber
Efferent vessel
518

The Berkeley Review


Specializing in NCAT Preparation

Biology

Glossary

Electrical potential
Electron transport
Electron-withdrawing group
Electrophile, electrophilic
Enzyme-linked immunoabsorbent assay (ELISA)

NCAT Words and Phrases

External respiration
Extracellular matrix

Extracellular medium
Extrachromosomal

Elution

Embryo
Embryogenesis
Embryonic
Emphysema

F factor plasmid
Facilitated diffusion

Facultative anaerobe
FAD

Emulsifier

FADH2

Encode

Fallopian tube
Familial hypercholesterolemia
Family
Fatty acid

Endocrine system
Endocytosis
Endocytotic vesicle
Endoderm

Feedback-inhibited

Endogenous
Endoplasmic reticulum

Feminization

Fermentation

Endosome

Fetus

Endothelial cells

Fibrillin

Endothelium, endothelia (pi.)

Fibrin

Endotoxin

Fibrinogen

Enol

Fibroblast

Enzyme
Eosinophile
Epidermis
Epididymis
Epigenetic
Epimerize
Epinephrine
Epithelial
Epithelial, simple squamous
Equatorial
Erythrocyte

Fibronectin
Fibrosis

Fight-or-flight response
First filial (Fi) generation
Fledgling
Fluorescence
Fluorescent molecules

Follicle (primary, secondary, tertiary)


Follicle-stimulating hormone (FSH)
Formaldehyde
Frameshift mutation

Escherichia coli

Free radicals

Essential amino acid

Fructose

Ester

Fruit flies

Esterification

Fungus, fungi (pi.)

Estradiol

Estrogen
Ethanol

G protein

Ether

Eukaryote

Galactose

Excision

Gallbladder
Gamete

Excitatory

Gametogenesis
Ganglion, ganglia (pi.)
Gap junctions
Gastric enzyme

Excrete

Exocrine system
Exocytosis
Exogenous
Exon

Gastric mucosal cells

Exonuclease

Gastrointestinal tract

Exoskeleton

Gastrulation

Expiration

Gaucher's disease

Copyright by The Berkeley Review

519

The Berkeley Review


Specializing in NCAT Preparation

Biology

Glossary

NCAT Words and Phrases

Gel chromatography
Gel electrophoresis

Guanine

Gene

Guanosine diphosphate (GDP)


Guanosine triphosphate (GTP)

Guanosine

Genes, linked
Genes, unlinked

Genetic code

Genetic engineering
Genetic theory of aging
Genitals or genitalia

Habituation
Haldane effect
Hantavirus

Genome

Haploid

Genotype
Genus, genera (pi.)

Hardy-Weinberg equilibrium (p2 + 2pq + q2)

Germ cell

Heart rate

Heart contraction

Germ cell layer


Gibbs free energy

Heat-killed bacteria
Heatstroke

Globin

Helicobacter pylori

Globular heads, myosin

a-Helix

Glomerular filtrate

Helper T cell (helper T lymphocyte)


Hematopoiesis, hemopoiesis
Hematopoietic stem cell (HSC)

Glomerular filtration

Glomerulus, glomeruli (pi.)


Glucocerebrosidase (acid beta-)

Heme

Glucocerebrosides

Heme catabolism

Glucocorticoid

Heme oxygenase

Gluconeogenesis

Hemocoel

Glucose

Hemodialysis
Hemoglobin (Hb)
Hemolysis
Hemophilia
Hemophilia factor VIII
Hemorrhage
Hemorrhagic disease
Hepatectomy
Hepatic
Hepatitis
Hepatocytes
Hepatopancreas

Glucose-6-phosphate
Glutamic acid (Glu)
Glutamine (Gin)

Glyceride
Glycerol
Glycerol-3-phosphate
Glycine (Gly)
Glycogen
Glycolipid
Glycolysis
Glycoprotein
Glycoside
Glycosidic linkages (alpha, beta)
Golgi apparatus
Golgi body
Gonadotropin-releasing hormone (GnRH)
Gonadotropins

Herbivore
Heritable

Hermaphroditism
Heterodimer

Heterotropic
Heterotrophic
Heterozygous

Gonads
Gonorrhea

n-Hexanoic acid

Gout

Hibernation

Gp 120 proteins

High-density lipoprotein (HDL)


Hippuric acid

Gradient
Gramicidin

Histamine

Gram-negative
Gram-positive
Granulocytic leukocyte
Griscelli syndrome

Histidine (His)
Histidine-deficient
Histone

Holocarboxylase synthetase

Growth hormone (GH)


Copyright by The Berkeley Review

Homeostasis
520

The Berkeley Review


Specializing in NCAT Preparation

Biology

Glossary

Homogenized
Homologous chromosomes
Homozygous

NCAT Words and Phrases

n vitro
n vivo

nactivated X chromosome

Hormone

nbreeding

Host cell

nclusions

Human chorionic gonadotropin (hCG)


Human immunodeficiency virus (HIV)
Huntington's chorea
Hybrid
Hybridize
Hydration
Hydroboration
Hydrolysis
Hydrophilic
Hydrophilic amino acid
Hydrophobic
Hydrophobic amino acid
Hydrostatic pressure
Hydroxyl
Hydroxylation
Hydroxyprogesterone
Hyperglycemia
Hyperosmotic
Hyperplasia
Hyperpolarization
Hypertension
Hypocalcemia
Hypodermis
Hypoglycemia
Hypotension
Hypothalamus, hypothalamic
Hypothalamic neurons
Hypothalmic-pituitary axis
Hypothermia
Hypovolemic shock
Hypoxia

ncomplete penetrance
nflammatory bowel disease
nflammatory response
nhibin

nhibitory
nnervation

noculate

nspiration
nsulin

nsulin resistance

ntegration
ntegrins
ntercostal muscles, external
nterferon
ntermolecular

nternal respiration
nterneuron

nterphase
nterstitial fluid

nterstitial space
ntestinal goblet cell
ntestinal wall

ntracellular medium
ntramolecular

ntrapleural
ntrapleural pressure
ntrauterine

ntravenous infusion
ntron

nvariant

on-exchange chromatography
onic

onic imbalance

rreversible inhibitor

Imidazole ring

schemia

Immune reaction

soelectric point

Immune response
Immunity
Immunity, cell-mediated
Immunity, humoral

soenzymes

sogamy

soleucine (lie)
someric

Immunization

somerization

Immunodeficiency
Immunoglobulins (IgA, IgD, IgE, IgG, IgM)
Immunoglobulin IgY (birds, reptiles)
Implantation
Imprinting
Innate immune system

sotonic

sotope

Intercalated discs

Jacob-Monod model
Jaundice

In utero

Jejunum

Copyright by The Berkeley Review

521

The Berkeley Review


Specializing in NCAT Preparation

Biology

Glossary

Juxtaglomerular apparatus
Juxtaglomerular cells

NCAT Words and Phrases

Liver

Locus, loci (pi.)


Logarithmic microbial growth
Loop of Henle
Low-density lipoprotein (LDL)

K
Kanamycin

Lumbar

Keratin

Luteinizing hormone (LH)


Lymph nodes
Lymphatic (system and vesicles)
Lymphocyte
Lysine (Lys)
Lysis
Lysogen, lysogenic
Lysosomal enzymes
Lysosome
Lysosome, secretory
Lytic granules

Keratinized

Keratinocytes
oc-ketoglutaric acid
Keto

Ketone

Ki-67 protein
Kidney

Killer T cell (killer T lymphocyte)


Kindred analysis
Kinetochore

Kinetochore microtubule

Kingdom
Krebs cycle
Kymograph

M
Macroautophagy
Macroautophagy allele (beclin 1)
Macroautophagy protein (beclin 1)
Macronucleus

Lactalbumin

Macrophage
Major histocompatibility complex (MHC)
Malpighian tubules

Lactase
Lactation

Lacteals

Mammal

Lactic acid

Marfan syndrome

Lactoglobulin

Mast cells

Lactose

Medulla oblongata
Megakaryocytes

Late gene
LD50 test

Meiosis, meiotic

Leptin

Melanin

Lethal mutation

Melanocyte
Melanophilin protein

Leucine (Leu)

Leukocyte

Melanosomes

Leukotrienes

Membrane-bound enzyme
Membrane-bound receptor
Menopause

Lewis acid
Lewis base

Leydig cells
Ligament
Ligand
Light reactions
Lipase
Lipid
Lipid bilayer
Lipid-soluble
Lipophilic
Lipophobic
Lipopolysaccharide
Lipoprotein
Liposome
Liposome-DNA complex
Copyright by The Berkeley Review

Menstruation
Mesoderm

Messenger RNA (mRNA)


Metabolism, metabolic
Metabolic acidosis
Metabolize

Metaphase
Methanol

Methionine (Met)

Methylation
Microbe

Microcirculation

Microfilament
Micronucleus
522

The Berkeley Review


Specializing in NCAT Preparation

Biology

Glossary

NCAT Words and Phrases

Microtubule

Neurosecretory

Missense mutation

Neurotransmitter

Mitochondrial DNA (mtDNA)

Neurulation

Mitochondrial genome

Neutrophile

Mitochondrial membrane, inner


Mitochondrial membrane, outer

Niche

Nitrogenous base
Nitrogenous waste

Mitochondrial myopathies (MDS and MERRF)


Mitochondrion, mitochondria (pi.)

Nitrous oxide

Noncompetitive blocker
Noncompetitive inhibitor

Mitosis, mitotic

Mixed competitive inhibitor


Mixed noncompetitive inhibitor
Molecular hybridization

Nonessential amino acid


Nonheritable

Monoamine oxidase (MAO)

Nonkinetochore microtubule

Monogastric

Nonsense mutation

Monosaccharide

Norepinephrine
Nuclear envelope

Morphine
Morphology
Mosaic hypothesis
Motility

Nuclear membrane
Nucleic acid

Nucleocapsid

Motor neuron

Nucleolus

Mucosa

Nucleophile, nucleophilic
Nucleoplasm

Mucus (n.), mucous (adj.)


Multiple genes
Multiple sclerosis
Mutagenic

Nucleoside
Nucleotide
Nurse cells

Mutation

Nutrient

Mycobacterium tuberiilisis
Myelin
Myocyte
Myoglobin
Myosin
Myosin light chain
Myosin light chain kinase (MLCK)
Myosin Va protein
Myxovirus influenzae

Nutrition

O
Obligate aerobe
Obligate anaerobe
Obligate parasite
Ohm's law
Oleic acid

Oligonucleotide
Omasum

Oncogene
Oocyte (primary, secondary)
Oogonia
Open circulatory system
Opiate
Optical antipole
Optic cup

Na+/K+ -ATPase

Na+/K+ pump
NAD+
NADH

Natural killer cell


Natural selection

Order

Necrosis

Organelle
Osmolality

Negative feedback
Nematode

Osmoregulatory

Nephron

Osmosis, osmotic

Neuroeffector

Osmotic pressure

Neurofibromatosis type I (NFI)

Osteoblast

Neuromodulator

Osteoclast

Neuron

Osteocyte
Osteoporosis

Neuropeptide Y
Copyright by The Berkeley Review

523

The Berkeley Review


Specializing in NCAT Preparation

Biology

Glossary

Phosphatidylcholine
Phosphoglyceride
Phospholipid bilayer
Phosphoprotein
5-phosphoribosylpyrophosphate(5-PRPP)
Phosphorothioate
Phosphorylation
Photophosphorylation
Photosynthesis
Phylum, phyla (pi.)
Physostigmine
Pied flycatcher (Ficedula hypoleuca)
Pigment

Ovary
Oviduct
Ovulation

Ovum, ova (pi.)


Oxidation

Oxidative metabolism

Oxidative phosphorylation
Oxidizing condition
Oxygenation
Oxytocin

P50

Pilomotor

Pancreas, pancreatic
Parasympathetic nervous system
Parathyroid hormone (PTH)

Pilus, pili (pi.)


Pineal gland
Pinealectomy
Piperidine

Parent cell
Parent strand

Pithed

Parental (P) generation

Pituitary gland
pKa

Parkinson's disease

Parthenogenesis
Partial pressure

Placebo effect
Placenta

Passive diffusion

Planarian flatworm

Pathogen, pathogenic
Pedigree

Plasma

Plasma clearance

Penicillin

Plasma membrane

Pepsin
Peptidase
Peptide
Peptide bond
Peptidoglycan
Pericytes

Plasma osmolarity
Plasmid
Plateau

Platelet

Pleural cavity
Pluripotent stem cell

Perineum

Pneumonia

Peripheral nervous system (PNS)

Point mutation

Peristalsis

Polar

Peritoneal cavity

Polarized

Peritoneum

Pollen

Peritubular capillaries
pH (acidic, basic, neutral)
pH optimum
Phage progeny
Phagocytosis

Pollination

Polygenic
Polymerize, polymerization
Polynucleotide
Polypeptide
Polysaccharide
Polyunsaturated

Phalloidin

Pharynx
Phenol

Positive feedback

Phenotype
Phenylalanine (Phe)
Phenylalanine hydroxylase
Phenylketonuria
Phosphate anhydride
Phosphate ester
Phosphatase

Postmenopausal
Postpubescent
Postsynaptic membrane
Potomac horse fever

Premature stop codon


Preprotein
Presynaptic membrane

Phosphatidic acid
Copyright by The Berkeley Review

NCAT Words and Phrases

524

The Berkeley Review


Specializing in NCAT Preparation

Biology

Glossary

Primary structure

NCAT Words and Phrases

Renal

Probe

Renal filtrate

Progesterone
Prokaryote

Renin-angiotensin system
Replication
Repolarization
Reproductive tract

Proline (Pro)

Prophage
Prophase
Prostaglandin
Prostate gland

Resonance

Respiratory acidosis
Respiratory tract
Reticulum, reticula (pi.)

Protease

Proteolytic

Retina

Protonation

Retrovirus

Proto-oncogene
Prototroph
Protozoon, protozoa (pi.)

Reverse transcriptase
Rhesus (Rh) factor
Rheumatic fever

Rh-negative
Rh-positive
Rib cage

Provirus

Proximal tubule

Pseudohermaphroditism
Puberty
Pulmonary artery
Pulmonary function
Pupillary light reflex

Ribonucleic acid (RNA)


Ribonucleoside
Ribose

Ribosomal RNA (rRNA)

Ribosomes, eukaryotic (40S, 60S, 80S)


Ribosomes, prokaryotic (30S, 50S, 70S)
Ribozyme

Purine

Pyloric valve
Pyridine
Pyrimidine
Pyrrolidine

Ribulose
Rickets

Pyruvate

Rickettsia

Ringer's solution

Pyruvic acid

RNAase

RNA-dependent DNA polymerase


RNA-dependent RNA polymerase

Quaternary structure

Roan

Quinine
Quinone

Rough endoplasmic reticulum (RER)


Rumen

Rab27a protein

S phase
Saccharomyces cerevisiae

Racemize

Radial muscle

Sacral

Radioimmunoassay (RIA)
Rana sylvatica (frog)
Rate-determining step
Reabsorption
Reading frame
Receptor

Salivary gland
Salmonella typhiimirium
Saponification
Sarcomere

Sarcoplasmic reticulum
Saturated hydrocarbon

Recessive

Second filial (F2) generation


Secondary messengers
Secondary structure

Recombination

Red blood cell (RBC)


Reduction

Secrete

Regulative hypothesis
Releasing factor
Reperfusion

Secretor gene
Secretory protein
Self-tolerance

Copyright by The Berkeley Review

525

The Berkeley Review


Specializing in NCAT Preparation

Biology

Glossary

Semen

Spore

Semiconservative replication

Start codon

NCAT Words and Phrases

Seminal vesicle

Stearic acid

Seminiferous tubule

Stem cell

Semipermeable membrane

Stereochemistry

Sense

Stereoselective

Sensory neuron
Sepsis
Septic shock
Septicemia

Stochastic

Stomach acidity
Stop codon
Staphylococcus aureus
Streptococcus pyogenes
Streptomycin

Serine (Ser)
Serotonin

Serotonin biosynthetic pathway


Serotonin receptors (5-HT2A, 5-HT2B)

Striated muscle

Sertoli cells

Strict anaerobe

Strict aerobe

Serum, sera (pi.)

Subchromosomal

Sex chromosomes

Subspecies

Sex pilus

Succinic acid

Sex-linked dominant

Sucrose

Sex-linked recessive

Superantigen
Support cell
Suppressor T cells
Sweat gland
Symbiotic
Sympathetic nervous system
Synapse
Synapsin
Synapsis
Synaptic cleft
Synaptic transmission

Sex-linked trait
Sickle-cell anemia

Signal hypothesis
Signal peptidase
Signal peptide
Signal recognition particle (SRP)
Simple diffusion
Sinoatrial node
Sinus venosus
Sinusoids
Skeletal muscle

Synovial joints (pivot, plane, hinge, ball-and-socket)

Small intestine

Synthesis, synthetic
Synthesize
Synthetase
Systemic artery
Systemic circulation
Systemic vein
Systolic blood pressure

Smallpox
Smooth endoplasmic reticulum (SER)
Smooth muscle

Sodium-potassium pump (Na+/K+ pump)


Solubility
Somatic

Somatic nervous system


Species
Spectrophotometry
Spectroscopy
Sperm
Spermatid
Spermatocyte, primary and secondary
Spermatogenesis
Spermatogonium, spermatogonia (pi.)
Spermatozoon, spermatozoa (pi.)
Spinal cord
Spiral valve
Spleen
Splicing
Spontaneous mutation
Copyright by The Berkeley Review

T helper (TH) cell


T lymphocyte
T tubules

Tactile pressure
Tautomerism

Taxon, taxa (pi.)


Taxonomic unit

Telophase
Tendon

Tertiary structure
Testis, testes (pi.)
Testosterone

526

The Berkeley Review

Specializing in NCAT Preparation

Biology

Glossary

NCAT Words and Phrases

Tetanospasmin

Tubulin

Tetanus

Tumor suppressor gene


Tumorigenesis
Turner syndrome
Tyrosine (Tyr)

Tetrad

Tetrahymena
Tetraploid
Thermogenesis
Thermoregulation

Thick filament

Ubiquinone

Thin filament

Ulcer

Thiol

Ulcerative colitis

Thoracic cavity

Ulex

Thoracic wall

Ultraviolet radiation

Thorax, thoracic

Uracil

Threonine (Thr)

Urea

Thrombin

Urease

Thrombocytes
Thymine
Thymus
Thyroid gland
Thyroid-stimulating hormone (TSH)
Thyroxine

Urethra
Uric acid

Uridine nucleotide

Uridine triphosphate (UTP)


Urine

Uterus, uterine

Tidal volume

Tight junctions
Totipotent stem cell
Vaccine

Toxin, toxic

Vaccinia virus

Toxic shock syndrome (TSS)

Vagina
Vagus nerve

Trachea
Trans

Valine (Val)

Transcription
Transcriptional regulation

Valinomycin
Variable surface glycoproteins

Transduction

Vascular resistance

Transfer RNA (tRNA)

Vascular smooth muscle

Transformation

Vasoconstriction

Transgenic

Vas deferens

Translation

Vasodilation

Translocation

Vasopression
Vegetative state

Transmembrane

Treadmilling
Tricarboxylic acid (TCA) cycle
Triglyceride
Triglycerides, intramuscular (IMTGs)
Tropomyosin
Troponin
True-breeding variety
Trypanosome
Trypanosomiasis
Tryptophan (Trp)
Tryptophan hydroxylase (TPHI, TPH2 genes)
Trypsin
Tubal pregnancy

Vein, venous
Vena cava

Ventilation rate

Ventral nerve cord

Ventricle (left and right)


Venules

Vertebra, vertebrae (pi.)


Vertebrate

Very low-density lipoprotein (VLDL)


Vestigial wings
Villus, villi (pi.)
Viral coat

Viral envelope
Viral genome

Tuberculosis
Tubular load

Virion

Tubular transport maximum (Tm)


Copyright by The Berkeley Review

527

The Berkeley Review


Specializing in NCAT Preparation

Biology

Glossary

NCAT Words and Phrases

Virulence
Virus

Visceral

Viscosity
Visual centers
Vitamin C
Vitamin D
Vitamin E

Vole

Waring blendor
Water-soluble

Watson-Crick base pairs


Western blotting
White blood cell (WBC)

Wild type
Wobble hypothesis

Xanthine

Xanthine oxidase
X chromosome
X-chromosome inactivation

X-ray crystallography
X-ray diffraction
Xylose
Xylulose

Y chromosome
Yeast

Yeast artificial chromosome (YAC)

Yersina pestis
Yops proteins

Z line (or Z disk)

Zeamays(corn)
Zileuton
Zwitterion

Zygote
Zymogen

Copyright by The Berkeley Review

528

The Berkeley Review


Specializing in NCAT Preparation

Last, First (Please PrintClearly)

Practice Sets:

Name:

Spring

Biology Practice Sets 6-10


(Make five copies, one for each section of the book.)

1.0
2- 0
3- 0
4. 0
5. 0

6-10

O 20ri

Summer <0

26.
27.
28.
29.
30.

0
0
0

51.
52.
53.
54.
55.

0
0
0
0
0

76.
77.
78.
79.
80.

0
0
0
0
0

6.
7.
89.
10.

0
0
0
0

31.
32.
33.
34.
35.

0
0
0
0
0

56.
57.
58.
59.
60.

0
0
0
0

81.
82.
83.
84.
85.

0
0
0
0
0

11.
12.
13.
14.
15.

0
0
0
0
0

36.
37.
38.
39.
40.

0
0
0
0
0

61.
62.
63.
64.
65.

0
0
0
0
0

86.
87.
88.
89.
80.

0
0
0
0
0

16.
17.
18.
19.
20.

0
0
0
0
0

41.
42.
43.
44.
45.

0
0
0
0
0

66.
67.
68.
69.
70.

0
0
0
0
0

91.
92.
93.
94.
95.

0
0
0
0
0

21.
22.
23.
24.
25.

0
0
0
0
0

46.
47.
48.
49.
50.

0
0
0
0
0

71.
72.
73.
74.
75.

0
0
0
0
0

96.
97.
98.
99.
100.

0
0
0
0
0

Copyright by The Berkeley Review

Biology

Raw

Section

Score

529

Year

Estimated
Scaled Score

The Berkeley Review


Specializing in NCAT Preparation

Notes

Notes

Notes

Notes

Notes

Notes

Notes

PERKELEY

PERIODIC TABLE OF THE ELEMENTS

He

1.0

4.0

10

Li

Be

Ne

6.9

9.0

10.8

12.0

14.0

16.0

19.0

20.2

11

12

13

14

15

16

17

18

Na

Mg

Al

Si

CI

Ar

23.0

24.3

27.0

28.1

31.0

32.1

35.5

39.9

19

20

21

22

23

24

25

26

I 27

28

29

30

31

32

33

34

35

36

Ca

Sc

Ti

Cr

Mn

Fe

Co

Ni

Cu

Zn

Ga

Ge

As

Se

Br

Kr

39.1

40.1

45.0

47.9

50.9

52.0

54.9

55.8

58.9

58.7

63.5

65.4

69.7

72.6

74.9

79.0

79.9

83.8

37

38

39

40

41

42

43

44

45

46

47

48

49

50

51

52

53

54

Rb

Sr

Zr

Nb

Mo

Tc

Ru

Rh

Pd

Ag

Cd

In

Sn

Sb

Te

Xe

85.5

87.6

88.9

91.2

92.9

95.9

(98)

101.1

102.9

106.4

107.9

112.4

114.8

118.7

121.8

127.6

126.9

131.3

55

56

72

73

74

75

76

77

78

79

80

81

82

83

84

85

86

Cs

Ba

Hf

Ta

Re

Os

Ir

Pt

Au

Hg

Ti

Pb

Bi

Po

At

Rn

132.9

137.3

178.5

180.9

183.9

186.2

190.2

192.2

195.1

(209)

(210)

(222)

87

88

89 R

105

106

107

108

109

110

116

117

118

Fr

Ra

Ac

104
Rf

Db

Sg

Bh

Hs

Mt

5?t
LaT
138.9

(223) (226) (227)

(261) (262)

(266) (264)

(277) (268)

197.0 200.6
HI

112

204.4
113

207.2 209.0
114

115

Uuq Uup Uuh Uus Uno


(271) (272) (277) (287) (289) (291) (292) (292) (293)
Ds

Rg

Cn

Uut

58

59

60

61

62

63

64

65

66

67

68

69

70

71

Ce

Pr

Nd

Pm

Sm

Eu

Gd

Tb

Dy

Ho

Er

Tm

Yb

Lu

140.1

140.9

144.2

158.9

162.5

164.9

167.3

168.9

173.0

175.0

90

91

92

93

94

95

96

97

98

99

100

101

102

103

Th

Pa

Np

Pu

Am

Cm

Bk

Cf

Es

Fm

Md

No

232.0

(145) 150.4 152.0 157.3

(231) 238.0 (237) (244)

Specializing in MCAT Preparation

(243) (247)

(247) (251)

(252) (257)

(258) (259)

Lr

(260)

Potrebbero piacerti anche